You are on page 1of 579

3rd Edition



Malathi Murugesan
MD (Microbiology), DTM&H (RCP London), PGDID
Consultant Microbiologist
Vellore, Tamil Nadu

CBS Publishers & Distributors Pvt Ltd


• New Delhi • Bengaluru • Chennai • Kochi • Kolkata • Mumbai
• Hyderabad • Nagpur • Patna • Pune • Vijayawada
DISCLAIMER
This book contains questions based on important
topics frequently asked in previous years National
Level PG Entrance Examinations and State Level
Examinations in India. Often repeated topics
and sub-topics have been included for students’
benefit. We do not claim that these questions are
exact or similar to questions asked in any recent
examinations in India. If any such similarity is
found, it is purely coincidental and by chance.

ISBN: 978-93-89941-98-2

Copyright © Author & Publishers

Third Edition: 2020


Second Edition: 2019-20

All rights reserved. No part of this book may be reproduced or transmitted in any form or by any means, electronic or
mechanical, including photocopying, recording, or any information storage and retrieval system without permission,
in writing, from the author and the publishers.
Published by Satish Kumar Jain and produced by Varun Jain for
CBS Publishers & Distributors Pvt Ltd
4819/XI Prahlad Street, 24 Ansari Road, Daryaganj, New Delhi 110 002, India.
Ph: +91-11-23289259, 23266861, 23266867 Website: www.cbspd.com
Fax: 011-23243014
e-mail: delhi@cbspd.com; cbspubs@airtelmail.in.
Corporate Office: 204 FIE, Industrial Area, Patparganj, Delhi 110 092
Ph: +91-11-4934 4934 Fax: 4934 4935
e-mail: feedback@cbspd.com; bhupesharora@cbspd.com

Branches
• Bengaluru: Seema House 2975, 17th Cross, K.R. Road, Banasankari 2nd Stage,
Bengaluru 560 070, Karnataka
Ph: +91-80-26771678/79 Fax: +91-80-26771680 e-mail: bangalore@cbspd.com
• Chennai: 7, Subbaraya Street, Shenoy Nagar, Chennai 600 030, Tamil Nadu
Ph: +91-44-26680620, 26681266 Fax: +91-44-42032115 e-mail: chennai@cbspd.com
• Kochi: 68/1534, 35, 36-Power House Road, Opp. KSEB, Cochin-682018, Kochi, Kerala
Ph: +91-484-4059061-65 Fax: +91-484-4059065 e-mail: kochi@cbspd.com
• Kolkata: 6/B, Ground Floor, Rameswar Shaw Road, Kolkata-700 014, West Bengal
Ph: +91-33-22891126, 22891127, 22891128 e-mail: kolkata@cbspd.com
• Mumbai: 83-C, Dr E Moses Road, Worli, Mumbai-400018, Maharashtra

Ph: +91-22-24902340/41 Fax: +91-22-24902342 e-mail: mumbai@cbspd.com

Representatives
• Hyderabad Patna +91-9334159340
+91-9885175004 •


• Pune +91-9623451994 • Vijayawada +91-9000660880


Printed at:
Dedication
This book is dedicated to my parents
Mrs M Thamayanthi DCE
Mr S Murugesan MA
who molded and guided me in all my tough times...

Preface
To my dear PG aspirants,
Microbiology and infectious diseases need regular updates because of their emerging and re-emerging trends. With the
recent novel corona virus panicking whole world, the question which has been haunting every heart is: “Are we standing on
the threshold of a third world war”—Man versus Microbes?
Pandemic and Epidemic outbreaks make a huge impact on health, economy, travel and also our day to day life gets
affected on a large scale. There is a difference in understanding microbiology during your UG days and your PG preparation.
This book MICRONS—Microbiology Simplified will make you feel the difference. To prepare for a PG entrance examination,
you need to be smarter than being intelligent.
The most important chapters are Sterilization and Disinfection, Antimicrobial resistance, Mycobacterium, Spirochaetes,
Hepatitis, HIV, Influenza, recent outbreaks and parasitology images.
When you are writing your exam papers, if you feel the questions are tough, you are not alone who think like this. There
are a number of others who are competing with you. Common questions never decide the seats. The rare and exceptions
make the difference between you and your close competitor. While reading, you better highlight the most common facts,
exceptions and recent updates. This book makes it easier for you because the important facts are highlighted well to make
a quicker revision.
I am happy to share that Microns – Microbiology Simplified – 3rd edition is released with added features and has been
updated with recent questions and notes on novel corona virus.
As I always say, students are most important for us and their opinions and ideas about a book are highly valuable for
every teacher/author. My book is not an exception. I would heartily welcome your feedback and work accordingly to
improve my book. Please share your feedback on my mail id or facebook page.
Thank you dear students !

With Love
Malathi Murugesan MBBS MD DTM&H PGDID
drmalathi13@gmail.com
Join author`s Facebook page for further updates and active discussion with author
MICRONS - Microbiology Simplified by Malathi M
From the Publisher’s Desk
Dear Readers,
I extend my warm welcome and convey my heartfelt thanks for appreciating the CBS Exam Books
for another successful year. It has been an amazing journey so far and I am highly grateful for your
support and cooperation to help us achieve various milestones in this whole span of time. The
mission with which we started in the year 2015 was to bring nothing but the best of everything
to our target audience and today I can proudly say that we have maintained that standard and are
committed to continue the same in future as well.
Every single title under the banner of CBS Exam Books has been developed and nurtured
like an infant. The authors and our entire team work day and night to bring the best in everything
for you. Be it content, presentation, social media contests and offers, we strive to meet your
expectations with every passing year. Your trust has motivated us to maintain and upgrade
ourselves during this period. I am extremely thankful to all our authors who are the real pillars of
the complete series of CBS Exam Books. The contributions of our esteemed authors have laid the
foundations of CBS Exam Books.
At this juncture, I can recall these lines by Drake,
“Sometimes it’s the journey that teaches you a lot about your destination”.
We have grown and changed with the passage of time to upgrade our ways of providing our readers with maximum benefits and help
them manage their time and efforts in effective manner. Previous year was the year of great achievements. Let me show you a glimpse of our
successful journey:
• Most of the titles of CBS Exam Books received wide acceptance and recognition by the readers of proving their usefulness and supremacy.

To mention a few, SARP Anatomy, CRISP, Surgery Sixer, Complete Review of Pathology, Conceptual Review of Pharmacology, SOCH,
Forensic Medicine, Complete Review of Medicine, Conceptual Review of PSM, MICRONS, My PGMEE Notes, AIIMS MedEasy, and
PRIMEs. With your constant support and our consistent efforts, I am sure that we will together witness an exponential acceptance of all
CBS Exam Books in coming future as well.
• The presence of CBS Exam Books has broadened through our various social media platforms. We have received great appreciation for our

regular Facebook activities such as online test series, giveaways, scientific content for knowledge enhancement, authors’ live sessions, and
various contests, like Bid 2 Win, Fastest Finger First, Book Fair and Facebook Community Awards. Join us on all these platforms to avail
and enjoy our exciting offers and benefits.

A book is incomplete if it does not have the right readers. We value you and your feedback. Please share your feedback and suggestions directly
with me at bhupesharora@cbspd.com. We promise to deliver in our books, what you desire to see.
I would like to sum up with these eternal lines of Robert Frost:
Woods are lovely dark and deep,
But I have promises to keep.
And miles to go before I sleep,
And miles to go before I sleep!
Wishing you success in all your endeavors!

Bhupesh Arora
Vice President – Publishing & Marketing
(PGMEE and Nursing Division)
Email: bhupesharora@cbspd.com
Mobile: (+91) 9555590180
Acknowledgements
My gratitude to the contributors who helped me in preparing this manuscript:
™ Dr M Jane Esther, MD (micro), FID, Consultant and Head of microbiology, Doctors' Diagnostic Centre, Trichy, Tamil Nadu
™
™ Dr K Deepika, MD (Microbiology), Consultant Microbiologist, NG Hospital and Research Centre, Coimbatore for her contribution on
™
chapter Gastrointestinal infections
™ Dr M Muniraj, MD DVL, Final year postgraduate, Madras Medical College, Chennai for his contribution on STD
™
™ Dr P Divya, DGO, Consultant Gynaecologist, WCF Hospital, Kolathur and GIFT clinic, Chennai for her contribution on Infections
™
related to Obstetrics and Gynecology
™ Dr J Divya John Stephy MD (Pharmacology), Assistant Professor of Pharmacology, Government Vellore Medical College and Hospital
™
for her contribution on Antimicrobial Chemotherapy—A Short Review
™ Dr FM Zafar, DLO, ENT specialist, Asst. Surgeon, Cheyyur GH for his contribution on Infections of Ear, Nose and Throat
™
™ Dr Kenny Robert J MS (GS), Consultant General and Laparoscopic Surgeon, Assistant Professor of General Surgery, Government
™
Royapettah Hospital, Kilpauk Medical College, Chennai for his contribution on surgical site and related infections
™ Dr Karthik Jayachandran, MS Orthopedics, Assistant Professor, Trichy SRM Medical College Hospital and Research Center, Irungalur,
™
Trichy for his contribution on Infections of Bones and Joints
™ Dr MJ Felix Emerson, D. ortho, DNB Orthopaedics (II year), Ganga Orthopaedic Hospital and Research Centre, Coimbatore, Tamil
™
Nadu
™ Dr R Gokul, MD (Anesthesiology), 2nd year postgraduate, Government Vellore Medical College, Vellore, Tamil Nadu
™
™ Dr Neha Singh, II year MD (micro), Rajendra Institute of Medical Sciences, Ranchi, Jharkhand
™
™ Dr V Srinidhi MBBS, Chengalpattu Government Medical College, Chengalpattu, Tamil Nadu
™
™ S Suganya, CRRI, Chengalpattu Government Medical College, Chengalpattu, Tamil Nadu
™
™ S Abirami, final year MBBS, Chengalpattu Government Medical College, Chengalpattu, Tamil Nadu
™
™ T Karthick, final year MBBS, Chengalpattu Government Medical College, Chengalpattu, Tamil Nadu
™
™ M Meena, final year MBBS, Chengalpattu Government Medical College, Chengalpattu, Tamil Nadu
™
™ N Dhanushya, final year MBBS, Chengalpattu Government Medical College, Chengalpattu, Tamil Nadu
™
™ GR Manoj Kumar, final year MBBS, Chengalpattu Government Medical College, Chengalpattu, Tamil Nadu
™
™ D Rishaba Sri, Pre-final year MBBS, Tagore Medical College and Hospital, Chennai, Tamil Nadu
™
My sincere acknowledgement to my mentors, friends and relatives who motivated and stood on my side in all my tough times:
™ I would like to express thanks and happiness to my pillar of support, my husband Dr R Jagadish, MD (Biochemistry), Assistant Professor,
™
Christian Medical College, Vellore, Tamil Nadu
™ Dr A Vijayalakshmi, MD (Microbiology), Professor & HOD, Chengalpattu Government Medical College, Chengalpattu, Tamil Nadu
™
™ Dr B Palani Kumar, MD (General medicine), Associate Professor, Government Thoothukudi Medical College, Tuticorin, Tamil Nadu
™
™ Dr S Jamuna Rani, MD (Pathology), Associate Professor, Tagore Medical College and Hospital, Chennai, Tamil Nadu
™
™ Dr K Shakthesh, MS(ENT), Hopkins ENT Clinic, Tambaram, Chennai, Tamil Nadu
™
™ Dr K Bharani Raj Kumar, MS (General Surgery), Professor, Tagore Medical College and Hospital, Chennai, Tamil Nadu
™
™ Dr AVM Balaji, MD (Microbiology), Sr Assistant Professor of Microbiology, Stanley Medical College, Chennai, Tamil Nadu
™
™ Dr R Rajamahendran, MS, MRCS (Edinburgh), MCh (Surgcial Gastroenterology, FMAS), Consultant Gastrointestinal Surgeon,
™
Director/Founder: KONCPT PG Medical Coaching Center, Tamil Nadu, India, Director: RRM Gastrosurgical and Research Center,
Villupuram, Tamil Nadu
™ Dr T Antan Uresh Kumar, MS, MCh Urology, FMAS, Founder KONCPT, laparoscopic transplant surgeon, Madras Kidney Foundation,
™
Chennai, Tamil Nadu
™ My sister: M. Abirami
™
™ My father-in-law Mr KM Ramu, mother-in-law Mrs J Krishnaveni and to all my maternal and paternal relatives.
™
™ Last but not least, I have to thank my twin children Master MJ Aathreyan and Master MJ Aaruthran, who strengthened my motherhood
™
and gifted me the memories of maternity with this wonderful piece of book.
I would also like to thank Mr Satish Kumar Jain (Chairman) and Mr Varun Jain (Managing Director), M/s CBS Publishers and Distributors
Pvt Ltd for providing me the platform in bringing out the book. I have no words to describe the role, efforts, inputs and initiatives undertaken
by Mr Bhupesh Arora Vice President – Publishing & Marketing, PGMEE and Nursing Division for helping and motivating me.
I must admit that I have been highly demanding on the precision of expression of the content from the staff members of CBS publishers.
I thank Dr Mrinalini Bakshi (Editorial Head & Content Strategist) for her editorial support and Ms Nitasha Arora (Production Head & Content
Strategist), Dr Anju Dhir (Project Manager & Senior Scientific Coordinator), Mr Shivendu Bhushan Pandey (Senior Editor), Mr Ashutosh
Pathak (Senior Proof Reader) and all the production team members, Mr Chaman Lal, Mr Prakash Gaur, Mr Phool Kumar, Mr Bunty Kashyap,
Mr Chander Mani, Ms Tahira Parveen, Ms Babita Verma, Ms Manorama Gupta, Mr Raju Sharma, Mr Manoj Chaudhary, Mr Vikram Chaudhary,
Mr Manoj Malakar and Mr Rahul Negi for devoting laborious hours in designing and typesetting of the book.
Contents
Preface ------------------------------------------------------------------------------------------------------------------------------------------- iii

Acknowledgements----------------------------------------------------------------------------------------------------------------------------------------vii
Recent Outbreak 2020--------------------------------------------------------------------------------------------------------------------------------------xi
Latest Exam Questions 2020–2019--------------------------------------------------------------------------------------------------------------------- xiii
Sample Video Questions----------------------------------------------------------------------------------------------------------------------------- xxxvii
Image-Based Concept Zone--------------------------------------------------------------------------------------------------------------------xli

UNIT 1 GENERAL MICROBIOLOGY



Chapter 1. Introduction, History and Microscopes---------------------------------------------------------------------------------- 3–10
Chapter 2. Morphology and Physiology of Bacteria--------------------------------------------------------------------------------11–25
Chapter 3. Sterilization and Disinfection--------------------------------------------------------------------------------------------26–38
Chapter 4. Culture Media and Culture Methods------------------------------------------------------------------------------------39–48
Chapter 5. Bacterial Genetics, Resistance and Susceptibility Testing-------------------------------------------------------------49–59

UNIT 2 BACTERIOLOGY

Chapter 6. Staphylococcus-----------------------------------------------------------------------------------------------------------63–72
Chapter 7. Streptococci---------------------------------------------------------------------------------------------------------------73–84
Chapter 8. Pneumococcus------------------------------------------------------------------------------------------------------------85–89
Chapter 9. Neisseria-------------------------------------------------------------------------------------------------------------------90–95
Chapter 10. Corynebacterium------------------------------------------------------------------------------------------------------- 96–102
Chapter 11. Bacillus----------------------------------------------------------------------------------------------------------------- 103–108
Chapter 12. Clostridium------------------------------------------------------------------------------------------------------------- 109–118
Chapter 13. Enterobacteriaceae---------------------------------------------------------------------------------------------------- 119–134
Chapter 14. Vibrio------------------------------------------------------------------------------------------------------------------- 135–141
Chapter 15. Pseudomonas, Acinetobacter and Burkholderia-------------------------------------------------------------------- 142–146
Chapter 16. Haemophilus, Francisella and Pasteurella--------------------------------------------------------------------------- 147–151
Chapter 17. Brucella and Bordetella----------------------------------------------------------------------------------------------- 152–155
Chapter 18. Mycobacterium-------------------------------------------------------------------------------------------------------- 156–166
Chapter 19. Spirochaetes------------------------------------------------------------------------------------------------------------ 167–175
Chapter 20. Rickettsia and Chlamydia--------------------------------------------------------------------------------------------- 176–186
Chapter 21. Helicobacter and Campylobacter------------------------------------------------------------------------------------ 187–189
Chapter 22. Mycoplasma and Legionella------------------------------------------------------------------------------------------ 190–193
Chapter 23. Miscellaneous Bacteria------------------------------------------------------------------------------------------------ 194–202

UNIT 3 VIROLOGY

Chapter 24. Introduction and General Properties of Viruses-------------------------------------------------------------------- 205–213
Chapter 25. Bacteriophages--------------------------------------------------------------------------------------------------------- 214–217
Chapter 26. Poxviruses-------------------------------------------------------------------------------------------------------------- 218–221
Chapter 27. Herpesviruses---------------------------------------------------------------------------------------------------------- 222–230
Chapter 28. Adenovirus------------------------------------------------------------------------------------------------------------- 231–232
Chapter 29. Picornaviruses--------------------------------------------------------------------------------------------------------- 233–238
Chapter 30. Orthomyxoviruses----------------------------------------------------------------------------------------------------- 239–243
Chapter 31. Paramyxovirus--------------------------------------------------------------------------------------------------------- 244–249
Chapter 32. Arthropod- and Rodent-Borne Viral Infections--------------------------------------------------------------------- 250–256
Chapter 33. Rhabdovirus------------------------------------------------------------------------------------------------------------ 257–261
Chapter 34. Hepatitis Virus--------------------------------------------------------------------------------------------------------- 262–271
Chapter 35. Human Immunodeficiency Virus------------------------------------------------------------------------------------ 272–279
Chapter 36. Miscellaneous Viruses------------------------------------------------------------------------------------------------- 280–286
UNIT 4 PARASITOLOGY
 Contents

Chapter 37. Introduction to Parasitology------------------------------------------------------------------------------------------ 289–295
Chapter 38. Flagellates–I------------------------------------------------------------------------------------------------------------ 296–299
Chapter 39. Hemoflagellates-------------------------------------------------------------------------------------------------------- 300–304

Chapter 40. Leishmania------------------------------------------------------------------------------------------------------------- 305–308
Chapter 41. Apicomplexa----------------------------------------------------------------------------------------------------------- 309–317
Chapter 42. Toxoplasma, Ciliate Protozoa----------------------------------------------------------------------------------------- 318–321
Chapter 43. Coccidian Intestinal Parasites---------------------------------------------------------------------------------------- 322–325
Chapter 44. Helminthology Cestodes---------------------------------------------------------------------------------------------- 326–332
Chapter 45. Trematodes------------------------------------------------------------------------------------------------------------- 333–338
Chapter 46. Nematodes------------------------------------------------------------------------------------------------------------- 339–347
Chapter 47. Filarial Nematode------------------------------------------------------------------------------------------------------ 348–353

UNIT 5 MYCOLOGY

Chapter 48. Characteristics and Laboratory Diagnosis of Fungi----------------------------------------------------------------- 357–363
Chapter 49. Superficial Mycoses---------------------------------------------------------------------------------------------------- 364–370
Chapter 50. Endemic/Systemic Mycoses------------------------------------------------------------------------------------------ 371–375
Chapter 51. Opportunistic Mycoses------------------------------------------------------------------------------------------------ 376–383
Chapter 52. Miscellaneous Fungi--------------------------------------------------------------------------------------------------- 384–386

UNIT 6 IMMUNOLOGY

Chapter 53. Immunity -------------------------------------------------------------------------------------------------------------- 389–392
Chapter 54. Structure and Functions of Immune System ------------------------------------------------------------------------ 393–400
Chapter 55. Antigens ---------------------------------------------------------------------------------------------------------------- 401–403
Chapter 56. Antibodies ------------------------------------------------------------------------------------------------------------- 404–410
Chapter 57. Complement System-------------------------------------------------------------------------------------------------- 411–415
Chapter 58. Antigen-Antibody Reactions------------------------------------------------------------------------------------------ 416–421
Chapter 59. Immune Response ---------------------------------------------------------------------------------------------------- 422–426
Chapter 60. Hypersensitivity ------------------------------------------------------------------------------------------------------- 427–431
Chapter 61. Immunodeficiency Diseases ----------------------------------------------------------------------------------------- 432–435
Chapter 62. Autoimmunity --------------------------------------------------------------------------------------------------------- 436–437
Chapter 63. Transplantation and Tumor Immunology--------------------------------------------------------------------------- 438–440
Chapter 64. Immunohematology -------------------------------------------------------------------------------------------------- 441–444

UNIT 7 APPLIED MICROBIOLOGY



Chapter 65. Applied Microbiology------------------------------------------------------------------------------------------------- 447–450

UNIT 8 INFECTIOUS DISEASES



Chapter 66. Introduction------------------------------------------------------------------------------------------------------------ 453–456
Chapter 67. Fever of Unknown Origin--------------------------------------------------------------------------------------------- 457–458
Chapter 68. Infections of Ear, Nose and Throat----------------------------------------------------------------------------------- 459–466
Chapter 69. Infections of Eye------------------------------------------------------------------------------------------------------- 467–470
Chapter 70. Infection in Lower Respiratory Tract – Pneumonia----------------------------------------------------------------- 471–472
Chapter 71. Gastrointestinal Infections-------------------------------------------------------------------------------------------- 473–478
Chapter 72. Cardiovascular Infection – Endocarditis----------------------------------------------------------------------------- 479–481
Chapter 73. CNS Infections--------------------------------------------------------------------------------------------------------- 482–484
Chapter 74. Skin and Soft Tissue Infections--------------------------------------------------------------------------------------- 485–486
Chapter 75. Infections of Bones and Joints---------------------------------------------------------------------------------------- 487–489
Chapter 76. Sexually Transmitted Infections-------------------------------------------------------------------------------------- 490–491
Chapter 77. Urinary Tract Infections----------------------------------------------------------------------------------------------- 492–493
Chapter 78. Infections Related to Obstetrics and Gynecology------------------------------------------------------------------- 494–496
Chapter 79. Surgical Site and Related Infections---------------------------------------------------------------------------------- 497–500
Chapter 80. Infections in Special Hosts-------------------------------------------------------------------------------------------- 501–503
Chapter 81. Antimicrobial Chemotherapy – A Short Review -------------------------------------------------------------------- 504–507
x
Chapter 82. Multiple Choice Questions-------------------------------------------------------------------------------------------- 508–510
Self-Assessment ------------------------------------------------------------------------------------------------------------------------- 511–527

 Latest Questions Papers 2020–2019
Recent Outbreak 2020
Ref: CDC

CORONA VIRUS
Coronaviruses are named for the crown-like spikes on their surface. There are four main sub-groupings of coronaviruses, known as
alpha, beta, gamma, and delta.


HUMAN TYPES
1. 229E (alpha coronavirus)
2. NL63 (alpha coronavirus)
3. OC43 (beta coronavirus)
4. HKU1 (beta coronavirus)
5. MERS-CoV (the beta coronavirus that causes Middle East Respiratory Syndrome, or MERS)
6. SARS-CoV (the beta coronavirus that causes severe acute respiratory syndrome, or SARS)
7. 2019 Novel Coronavirus (2019-nCoV)

Electron microscopic image of corona virus


Courtesy: National Institute of Allergy and Infectious Diseases (NIAID)

NOVEL CORONA VIRUS


yy 2019 Novel Coronavirus (2019-nCoV) is a virus identified as the cause of an outbreak of respiratory illness first detected in Wuhan,
China.
yy Incubation period: 2 – 14 days
yy Route of transmission: Droplets (respiratory secretions)
yy Precautions: N95 Mask, gloves, fluid repellent gowns, goggles
yy Clinical features:
 Fever, cough, shortness of breath


 In few people it develops pneumonia




 No specific age predilection seen




yy Diagnosis: Sequencing (other methods yet to develop); Testing done only in NIV, Pune for India.
yy No antiviral drugs/vaccines currently not available.
yy Antiviral combinations like lopinavir and ritonavir give promising improvement. But efficacy of the drug for novel corona virus is
not proven by clinical trials. xi
LATEST EXAM QUESTIONS
2020–2019
RECENT PATTERN QUESTIONS 2020 7. The following fungal infection occurs most commonly

1. Which of the following is the vector for Zika virus? due to thorn prick:
 Latest Exam Questions 2020–2019

a. Blastomycosis b. Sporotrichosis

a. Aedes aegypti b. Culex tritaeniorhynchus

c. Coccidioidomycosis d. Paracoccidioidomycosis

c. Phlebotomus papatasii d. Anopheles

8. An 8-year-old female is diagnosed with meningococcal


2. A 30-year-old man reports to the OPD with a history

meningitis. What advice should be given to children of


of clean-cut wound before four hours. He had already


received a dose of TT before 12 years. What is the ideal household and school contacts with the primary case to
vaccination regimen for him? avoid outbreak?
a. Single dose of TT a. Single dose of meningococcal Men4B vaccine

b. Erythromycin prophylaxis

b. Full dose of TT

c. Chemoprophylaxis and two doses of vaccine


c. Simultaneous administration of TT and TIG

d. Single dose of PCV vaccine


d. No need of vaccine

9. An HIV patient with CD4 count less than 200/Cu.mm


3. What is the immunological method depicted in the

came to OPD with whitish lesions in the mouth as given


following rapid test assay for Hepatitis B?


in the image. Mucosal biopsy on HPE shows yeast and
pseudohyphae. What is the diagnosis?
a. Oral hairy leukoplakia b. Oral candidiasis

c. Lichen planus
d. Kaposi sarcoma

10. Which among the following is a category A bioterrorism


agent?
a. Coxiella
b. Brucella

c. Nipah virus
d. Bacillus anthracis

11. Which of the following organelle in a bacterium helps in


adhesion?
a. Cytoplasmic membrane

b. Mesosomes

c. Fimbriae

d. Lipopolysaccharide

12. Identify the following immunoglobulin


a. ELISA
b. Immunochromatography

c. CLIA
d. Immunofluorescene

4. Identify the species from the following peripheral smear


of a man who had fever with chills.

a. IgM b. IgA

c. IgG d. IgD

13. Stool examination of a child is showing the following


soil transmitted helminthic egg. What is the diagnosis?

Courtesy: CDC/Dr. Mae Melvin


a. Plasmodium vivax
b. Plasmodium falciparum

c. Plasmodium ovale
d. Plasmodium knowlesi

5. The active disinfectant component in bleaching powder


is:
a. Hypochlorous acid
b. Hypochlorite

c. Chlorine
d. Hydrogen

6. What is the etiological agent for donovanosis?


a. Klebsiella granulomatis
Courtesy: CDC/B.G. Partin

b. Hemophilus ducreyi

c. Leishmania donovani

a. Ascaris lumbricoides
b. Hook worm

xiv d. Treponema pallidum


c. Trichiuris trichiura
d. Taenia solium

14. The mechanism of resistance to penicillin by production 18. HIV patient presented with diarrhea. On stool
of beta lactamases results in: examination, following acid fast organisms was seen.

 Latest Exam Questions 2020–2019


a. Drug efflux What is the drug of choice in this patient?
b. Alteration in penicillin binding proteins
c. Breaks the drug chemical bonding
d. Decreases MIC
15. An eight-year-old child presented to casualty with acute
febrile illness and petechial rash. What is the most likely
diagnosis?


a. TMP-SMX b. Nitazoxanide
c. Primaquine d. Niclosamide
19. Diagnostic method of choice for leptospirosis:
a. Cold agglutination test b. MSAT
c. MAT d. Latex agglutination test
20. Investigation of choice for neurosyphilis:
a. Meningococci
a. VDRL b. FTA-ABS
b. Pneumococci
c. RPR d. TPI
c. Hemophilus influenza
d. Malaria 21. A person working in an abattoir presented with malig-
nant pustule on hand; What is the causative agent?
a. Clostridium botulinum b. Clostridium perfringens
RECENT PATTERN QUESTIONS 2019 c. Bacillus anthracis d. Streptococcus pyogenes
16. True about Congenital Rubella syndrome is: 22. Ideal dose of Diphtheria antitoxin given for treatment is:
a. It will become a chronic infection a. 10,000 to 1,00,000 units b. 20,000 to 1,00,000 units
b. Virus can be isolated only upto 6months after birth c. 10,000 to 2,00,000 units d. 20,000 to 2,00,000 units
c. Triad of CRS are cataract, cardiac defects, cerebral 23. Infection that causes acute febrile illness with jaundice
palsy and conjunctivitis is:
d. Infection is most serious after five months of pregnancy a. Malaria b. Leptospirosis
c. Pertussis d. Typhoid
17. A 9 years old child presented to OPD with complaints of
24. A neonate was found to have cataract, deafness and
high grade fever, vomiting, one episode of seizure. CSF
cardiac defects. Which group of viruses does the mother
examination was done and Gram staining of the culture
was infected with:
showed the following finding. What is the probable
a. Togaviridae b. Flaviviridae
causative agent?
c. Bunyaviridae d. Arenaviridae
25. Which vaccine is contraindicated in pregnancy?
a. Hepatitis A b. Hepatitis B
c. Rabies d. Chicken pox
26. Which vaccine strain is changed every yearly?
a. Influenza b. Rabies
c. Hepatitis d. Ebola
27. A 5 years old child presented to the OPD with complaints
of rectal prolapse; On examination stunting and growth
retardation was documented; What is the parasitological
cause for this clinical feature?
a. Trichuris trichiura b. Trichinella spiralis
c. Giardia lamblia d. Enterobius vermicularis
a. Hemophilus influenzae 28. Flask shaped ulcers seen in a dysentry patient is
b. Neisseria meningitidis diagnostic of:
c. Streptococcus pneumoniae a. Shigellosis b. Amoebiasis
d. Escherichia coli c. Giardiasis d. Typhoid

xv
29. A 35 years old man presented with dry cough and rusty 34. Contact isolation is done for:
colored sputum; He has history of eating in chinese a. Mumps
 Latest Exam Questions 2020–2019

restaurant very often with consumption of crabs often; b. MRSA


What is the probable causative agent in this condition? c. Diphtheria
a. Diphyllobothrium latum d. Typhoid
b. Pneumocystis jirovecii 35. Coxsackie virus A causes:
c. Paragonimus westermani a. RMSF
d. Strongyloides stercoralis b. HFMD
30. Culture media for Legionella: c. Yellow fever
a. BCYE agar d. Pleurodynia
b. Baird Parker medium 36. Infection of following organism has clinical features
c. Macconkey agar resembling erythroblastosis foetalis?
d. PLET medium a. Cytomegalovirus
b. Ebstein Barr virus
31. Special stain for Cryptococcus:
c. Toxoplasmosis
a. Ziehl-Neelsen stain

d. Herpes virus
b. Mucicarmine stain
37. All are true about congenital toxoplasmosis except:
c. Malachite green
a. Chorioretinitis
d. Albert stain
b. Jaundice
32. Disk diffusion method is also known as:
c. Macrocephaly
a. Kirby Bauer method d. Cerebral calcification
b. E test-method 38. An AIDS patient presented to OPD with dyspnoea and
c. MIC method respiratory illness; Which of the following is suitable to
d. Stokes method diagnose the opportunistic infection commonly seen in
33. A child is suffering from recurrent chronic infections AIDS patient?
with encapsulated bacteria; Which subclass of IgG does a. Sputum microscopy
the child has deficiency? b. Broncho alveolar lavage (BAL)
a. IgG1 b. IgG2 c. Chest Xray
c. IgG3 d. IgG4 d. CT scan

39. Identify the following life cycle:

a. Plague b. Japanese encephalitis c. Influenza d. Nipah virus


xvi
AIIMS NOVEMBER 2019
40. Which of the following toxin acts by this mechanism as shown in the figure:

 Latest Exam Questions 2020–2019





a. Tetanus b. Botulism c. Neuroborreliosis d. Neurosyphilis

41. The CLED medium is preferred over other media for the 44. A Giemsa stain of a thin peripheral blood smear is
culture of the organisms in case of UTI because:- prepared. Which of the following cannot be diagnosed?
a. It inhibits proteus swarming a. Bartonella henselae b. Coxiella burnettii
b. It differentiates Lf from NLF c. Toxoplasma gondii d. Ehrlichia chaffeensis
c. It helps growth of candida and Staphylococcus 45. Mechanism of action of toxin produced by the bacterium
d. It identifies pseudomonas shown in the figure:
42. Which of the following organism cause multiple alveolar-
like mass in liver?
a. Echinococcus multilocularis
b. E. granulosus
c. Amoebic liver abscess
d. Cysticercus cellulose
43. A 25-Year-old man presents with urethral discharge
for the last three days. A gram stained smear of the
discharge is shown in the figure. All of the following are
true about the likely etiology except:-

a. Increase in cAMP
b. Increase in cGMP
c. ADP-ribosylation of ribosyl transferase
d. ADP-ribosylation of Gs protein
46. All can be seen intracellularly in hepatocytes except:-
a. Plasmodium b. Toxoplasma gondii
c. Leishmania d. Babesia
a. Virulence factor Pili 47. Beta-1, 3-D glucan assay test is used to diagnose all
b. Intracellular except:-
c. Both catalase and oxidase positive a. Aspergillosis b. Mucormycosis xvii
d. Show twitching motility c. Candidiasis d. Pneumocystis
48. True about γδ T cell; except:

AIIMS MAY 2019
a. It is type of helper T cells

51. A 40-year-old male from Himachal Pradesh has


 Latest Exam Questions 2020–2019

b. It is an antigen presenting cell


presented with verrucous lesions in the skin with


c. It resides in gastrointestinal tract epithelium

following microscopic findings from the tissue biopsy.
d. It shows direct cytotoxicity.
What is the diagnosis?
49. Assertion: Anaphylactoid reaction can be caused by 1st

exposure.
Reason: Involves mast cell degranulation but not IgE.

a. Both Assertion and Reason are independently true/


correct statements and the Reason is the correct


explanation for the Assertion
b. Both Assertion and Reason are independently true/

correct statements, but the Reason is not the correct


explanation for the Assertion
c. Assertion is independently a true/correct statement,

but the Reason is independently a false/incorrect



statement
d. Assertion is independently a false/incorrect statement,
a. Chromoblastomycosis b. Sporotrichosis

but the Reason is independently true/correct statement c. Rhinosporidiosis


d. Mycetoma

e. Both Assertion and Reason are independently false/



52. An HIV patient with fever, chronic diarrhea presented

incorrect statements to OPD. His stool was examined which showed the
following oocysts. Identify the components used for
50. Patient has recovered from a hepatitis B infection.
staining the stool?

Identify the serological marker shown by the arrow in


the image below:

a. Diluted Carbol Fuchsin, 20% H2SO4 and Methylene


blue
b. Strong Carbol Fuchsin, 0.5% H2SO4 and Loeffler’s

methylene blue
c. Strong Carbol Fuchsin, 1% H2SO4, iodine and methylene

blue
a. Anti HbC d. Diluted Carbol Fuchsin, 5% H2SO4 and methylene blue

b. Anti HbS 53. Arrange the order of Gram staining from below:

c. HBsAg I. Mordant
II. Acetone alcohol

d. Anti HBe III. Crystal violet


IV. Diluted carbol fuschin

a. I, II, III, IV
b. III, I, II, IV

c. IV, II, III, I


d. III, II, I, IV

54. Match the following with regards to Biomedical waste


management:

Column-A Column-B
1. Yellow
a. Glassware globes

2. Red
b. Scalpel blade

3. Blue
c. Chemical waste

4. White transparent
d. syringe wrapper

a. 1-C, 2-D, 3-A, 4-B b. 1-D, 2-A, 3-B, 4-C


xviii

c. 1-A, 2-B, 3-C, 4-D


d. 1-B, 2-C, 3-D, 4-A

55. A 48-year-old man, hospitalized for 17 days and on IV 61. A 22-year-old college going adult has H/o sexual
antibiotics followed by oral course of drugs for Acute exposure with commercial sex worker, now presenting

 Latest Exam Questions 2020–2019


febrile illness with neutropenia. He had a history of with painful genital lesions and lymphadenopathy in
dysuria while admitted and now complaints of loose right inguinal region. Identify the diagnosis?
stools and abdominal pain. Which antibiotic usage has
the highest risk for developing pseudomembranous
colitis?
a. Carbapenems
b. Aminopenicillin
c. Macrolides
d. Fluoroquinolones
56. Which among the following is a subcutaneous fungal
agent?
a. Cryptococcus neoformans
b. Histoplasma capsulatum
c. Sporothrix schenckii


d. Talaromyces marneffi
57. Choose the wrong statement about Plasmodium species:
a. Drug resistance falciparum not seen in India a. Syphilis b. Chancroid
b. Presence of Duffy blood group antigen is protective for c. LGV d. Gonorrhea
vivax 62. Which among the following occupation is a risk factor
c. Cerebral malaria with shock by P.falciparum is called for this presenting illness?
algid malaria
d. Relapse is seen in P.ovale
58. Mw vaccine extracted from which bacteria:-
a. M. indicus pranii
b. M. Welchii
c. M. leprae
d. M. bovis
59. A hospital has reported an outbreak of MRSA infection.
On investigation, it was found that staff nurses and
doctors had nasal carriage of MRSA. Which of the
following drug helps in removal of colonization?
a. Oral vancomycin
b. Inj cephalosporin
a. A lifeguard in swimming pool
c. Topical bacitracin
b. A poultry worker
d. Inhaled colistin c. Farmer
60. The following vector is not involved in transmission of d. A kennel worker
this disease: 63. Transmission assessment survey is carried out for:
a. P. vivax b. P. falciparum
c. Filariasis d. Leishmania
64. Which immunoglobulin is elevated in a case of chronic
allergy?
a. IgA b. IgM
c. IgE d. IgG
65. A patient who got exposed to Hepatitis B infection;
which of the following markers will always be present in
the patient even he becomes chronic or recurrent?
a. HbsAg b. Anti HBs Ab
c. HbcAB d. HbeAg

JIPMER MAY 2019


a. Kyasanur forest disease 66. Auto infection is not seen in:
b. Kala azar a. Hymenolepis nana b. Hymenolepis diminuta
c. Chandipura encephalitis c. Strongyloidosis stercoralis
d. Carrion`s disease d. Taenia solium
xix
67. The smallest helminth among the following is: 79. Which among the following is a bile esculin positive
a. Hymenolepis nana b. Hymenolepis diminuta organism and also shows growth in 6.5% Sodium
 Latest Exam Questions 2020–2019

c. Diphyllobothrium latum d. Balantidium coli Chloride?


68. Tungiasis is a. Streptococcus agalactiae
a. Ectoparasitic skin infection b. Streptococcus pneumoniae
b. Ectoparasitic intestinal infection c. Enterococcus faecalis
c. Endoparasitic skin infection d. Streptococcus viridans
d. Endoparasitic intestinal infection 80. All of the following Mycobacteria are pigmented except:
69. The gold standard investigation for Zika virus is: a. M.kansasii b. M.simiae
a. PCR c. M.avium d. M.scrofulaceum
b. Plaque reduction neutralization test
c. IgM antibodies
PGI MAY 2019
d. RT – PCR
70. Gene responsible for semi synthetic penicillin resistance 81. True about Chikungunya virus:
in Methicillin Resistant Staphylococcus aureus (MRSA): a. It is a DNA virus
a. Mec A b. Mec B b. Aedes aegypti is the vector responsible for transmission
c. Mec C d. Mec D c. Commonly presents as fever and arthralgia

71. Panton Valentine leukocidin is produced by d. In acute phase of illness, virus cannot be isolated from
a. Staphylococcus epidermidis blood
b. Pseudomonas e. Persistent virus presence in muscle and joint space
c. Vibrio cholerae may be cause of chronic arthralgia
d. Staphylococcus aureus 82. True about Dengue fever:
72. Which of the following is the amplifying host for a. Also called as Saddle back fever
Japanese encephalitis? b. Symptoms appear within 48 hours of infection
a. Pig b. Heron c. Serum aminotransferase level may be increased
c. Horse d. Egrets d. Rat is the reservoir of the virus
73. Transovarian transmission is seen among: e. Maculopapular rashes over trunk and limbs
a. Ricketssia akari b. Ricketssia ricketssii 83. Which of the following organism can be cultured on
c. Ricketssia prowazekki d. Ricketssia typhi Lowenstein-Jensen (LJ) medium:
74. A retroviral positive patient presents with frequent a. Mycobacterium tuberculosis
diarrheal episodes; Stool examination revealed acid b. Mycobacterium leprae
fast oocyst of size 8-10 uns. What could be the probable c. Mycobacterium bovis
diagnosis? d. Mycobacterium fortuitum
a. Toxoplasma gondii e. Nocardia
b. Cryptosporidium parvum 84. True statement(s) about latent TB infection is/are:
c. Cyclospora cayetanensis a. Significant chest x-ray findings
d. Sarcosystis hominis b. Rifampicin and INH are given for treatment
75. Encrusted cystitis is caused by: c. Standard TB regimen is used for treatment
a. Corynebacterium xerosis d. Interferon Gamma Release Assay (IGRA) test is used for
b. C. urealyticum diagnosis
c. C.renale e. Mantoux skin test is negative
d. C.pseudotuberculosis 85. All are true about Diphtheria in children except:
76. Elek’s gel precipitation test is used as a confirmatory a. Incubation period is 1-6 days
test for: b. Nasal form is most fatal form
a. Corynebacterium jeikeium c. Pharyngotonsillar diphtheria usually present as sore
b. C.diphtheriae throat
c. Clostridium tetani d. Membrane can cover pharynx, tonsils, soft and hard
d. Cl.perfringens palate
77. Which among the following is a cocco bacilli: e. Tetracycline is given for treatment
a. Escherichia coli 86. True about serology in an individual with acute hepatitis
b. Listeria monocytogenes B which is highly infectious:
c. Haemophilus influenzae a. HBsAg b. Anti HBs
d. Acinetobacter baumannii c. IgM anti HBc d. HBeAg
78. All are true about immune re constitution inflammatory e. IgG anti HBc
syndrome (IRIS) except: 87. Type II hypersensitivity reaction(s) is/are:
a. It occurs only when CD4 cell count is less than 50 a. Rheumatoid arthritis
b. Develops after initiation of anti retro viral therapy b. Arthus reaction
c. Associated with delayed type of hypersensitivity c. Serum sickness
d. Does not require specific antimicrobial therapy for d. Autoimmune haemolytic anemia
recover e. Good pasture syndrome
xx
88. Gold standard investigation for brucellosis: 92. True about Mucor:
a. Radioimmunoassay a. Unicellular organism having sporangium

 Latest Exam Questions 2020–2019


b. ELISA b. Aseptate hyphae
c. Blood culture c. (1–3)-β-D-glucan assay is helpful in diagnosis of
d. Standard agglutination test invasive mucormycosis
e. RT-PCR d. Colony grow very rapidly and changes color
89. Which of the following statement(s) is/are true about e. Filamentous fungus found in decaying fruits and vege-
Melioidosis: tables
a. Caused by parasite found in soil and water 93. Reverse algorithm for the diagnosis of syphilis is/are:
b. Main source of infection is contaminated water or a. VDRL then RPR
inhalation b. RPR then VDRL
c. Presents with pneumonia and skin abscess c. TPPA followed by VDRL
d. May takes years before becoming symptomatic d. Enzyme immunoassay (EIA) followed by RPR
e. Eradication treatment is required with cotrimoxazole e. RPR followed by Enzyme immunoassay (EIA)
or doxycycline for at least 12 weeks 94. Among the following pathogenic protozoa, which is/are
90. Which of the following can be used to culture Ureaplasma the member of mastigophora;
urealyticum: a. Toxoplasma gondii b. Trypanosoma cruzi


a. PPLO broth b. MacConkey agar c. Balantidium Coli d. P. vivax
c. Blood agar d. Alkaline peptone water e. Giardia lamblia
e. SP4 medium 95. Which of the following dyads of HIV virus structural
91. Mycoplasma genitalis causes: proteins/components is/are correct:
a. Cervicitis b. Non-gonococcal urethritis a. gp 120: Core Ag b. gp 41: Envelop Ag
c. Bacterial vaginosis d. Urinary stones c. p17: Shell Ag d. p15: Core Ag
e. Pneumonia e. p24: polymerase Ag

xxi
ANSWERS AND EXPLANATIONS
 Latest Exam Questions 2020–2019

RECENT PATTERN QUESTIONS 2020 4. Ans.  (b) Plasmodium falciparum

1. Ans.  (a) Aedes aegypti Ref: T.B. of medical parasitology – S.C. Parija – 4th edition –
Page 114;
Ref: Ananthanarayan and Paniker’s Textbook of Micro­
biology – 10th ed – Page 530 Features P.vivax P.falciparum P.malariae P.ovale
Vector and arboviral illness Diagnostic Trophozo- Ring forms Trophozoites Trophozoites
forms ites Gametocytes Schizonts Schizonts
Mosquito borne Chikungunya—Aedes
seen in Schizonts (banana Gametocytes Gametocytes
Dengue—Aedes
peripheral Gameto- shaped)
Yellow fever—Aedes
smear cytes
Zika virus—Aedes
JE—Culex Infected Enlarged Normal Normal Enlarged
RBC
Tick borne KFD

Crimean Congo hemorrhagic fever Dots Schuffner’s Maurer’s Ziemann’s James’ dots
Colorado tick fever dots dots dots
Rodent borne Hanta virus Others Relapse Recrudes- - Relapse
Lassa fever cence
Hemorrhagic fever with renal syndrome
5. Ans.  (a) Hypochlorous acid
2. Ans.  (a) single dose of TT
Ref: Park T.B. of Social and preventive medicine
Ref: Ananthanarayan and Paniker’s Textbook of Micro­
• Bleaching powder is Sodium hypochlorite in powdered
biology – 10th ed – Page 266

form
Categories Wound less than 6 Other • The active form of all chlorinated compounds are

hours, old, clean, wounds hypochlorous acid


non penetrating,
with negligible tissue 6. Ans.  (a) Klebsiella granulomatis
damage • Earlier called as Calymmatobacterium granulomatis

A. Complete course of Nothing needed Nothing • Gram negative rod; It has characteristic safety pin

toxoid or booster in appearance


previous 5 years • It causes a venereal disease named as donovanosis or

B. Complete course of Toxoid 1 dose Toxoid 1 granuloma inguinale


toxoid or booster in dose • Lab diagnosis: Demonstration of donovan bodies in

previous 5 -10 years Giemsa stain from lesions in the genital area
C. Complete course of Toxoid 1 dose Toxoid 1 • DOC: Tetracycline

toxoid or booster dose dose and


7. Ans.  (b) Sporotrichosis
in > 10 years of age human
tetanus IG Ref: Ananthanarayan and Paniker’s Textbook of Micro­
D. Has not had a Toxoid complete Toxoid biology – 10th ed – Page 606
complete course of course complete Sporotrichosis:
toxoid or unknown course + • Causative agent: Sporothrix scheckii (Thermally

status of vaccination human dimorphic)


tetanus Ig • Fungal infection affecting cutaneous, subcutaneous and

lymphatic tissue
3. Ans.  (b) Immunochromatography
• Most commonly affects gardeners, forest workers →

Ref: Ananthanarayan and Paniker T.B of microbiology – After a minor trauma (Rose thorn prick) → nodules
10th edition – Page 111 formed → ulceration → necrosis → Fixed cutaneous
Immunochromatographic test: sporotrichosis → spread to lymphatics → lymphocuta-
• Rapid test method

neous sporotrichosis → systemic spread to the bones,
• Done in a small cassette which is already impregnated

joints and meninges
with antibody/antigen with colloidal gold dye conjugate • DD: Nocardiosis, Tularemia, Non-TB mycobacterial

• When antigen – antibody reaction occurs – it is visible as


• infection and leishmaniasis
colored product • Lab diagnosis: KOH/HPE – shows asteroid body

xxii • Used in diagnosis of HIV, HBV, HCV and many other



• Culture: Yeast phase in 37deg C and mycelia phase in

infections 25deg C
• Serology: Slide latex agglutination test – Antigen is
• 11. Ans.  (c) Fimbriae
peptido – L- rhamno – D –mannan - ≥1:4 titre is positive
Ref: Ananthanarayan and Paniker’s Textbook of Micro­

 Latest Exam Questions 2020–2019


(mainly helpful in pulmonary sporotrichosis)
biology – 10th ed – Page 12
• Treatment: KI – Potassium iodide, Itraconazole,

fluconazole, cryotherapy • Fimbriae – Organ of adhesion (also called as Pili)


• Itraconazole is the DOC for lympho cutaneous



• Flagella – Organ of locomotion

sporotrichosis (IDSA guidelines)


• Severe pulmonary or CNS spreads needs AmB lipid therapy

12. Ans.  (b) Ig A
Ref: Ananthanarayan and Paniker T.B of microbiology –
8. Ans.  (c) Chemoprophylaxis and two doses of vaccine
10th edition – Page 100
Ref: Ananthanarayan and Paniker’s Textbook of Micro­ IgA:
biology – 10th ed – Page 237 • Immunoglobulin seen in saliva, tears, colostrum,

CDC recommends post exposure prophylaxis for contacts respiratory and GI secretions
and vaccination for children and adolescents: • Two forms are seen:

Vaccination:
• Vaccination available for Groups A,C,Y and W-135

Serum IgA Monomer


• Group B – vaccine available only in European union –

Seen in serum
named as 4CmenB (not in India) Secretory IgA Dimer
United by J chain
Age group Type of vaccine It has a secretory component – that protects
6 weeks to 18 months Hib-Men-TT (combined) four doses IgA from denaturation by bacterial proteases

9 – 23 months Tetravalent conjugate vaccine–two doses 13. Ans.  (c) Trichiuris trichiura
(conjugation with diphtheria toxoid –
Menactra vaccine Ref: T.B. of medical parasitology – S.C.Parija – 4th edition –
Page 265
2-55 years Tetravalent conjugate vaccine–Menveo
Trichuris Trichiura:
• Chemoprophylaxis and Treatment for carrier eradica-

• Definitive host: Humans (the only host)

tion – Rifampicin or Ciprofloxacin • Infection is acquired by ingestion of soil with


embryonated eggs (has rhabditiform larvae)


9. Ans.  (b) Oral Candidiasis • It usually affects children and remains asymptomatic;

Ref: Ananthanarayan and Paniker’s Textbook of Micro­ Heavy infection causes rectal prolapse in children;
biology – 10 th ed – Page 578 appendicitis;
• Lab diagnosis:
• Image has DD of all of the above options; But microscopy

ƒ Demonstration of eggs in feces; Characteristics of egg


has confirmed as yeast and pseudohyphae which


ƒ

are:
denotes it is oral candidiasis.
○ Barrel shaped
• Oral hairy leukoplakia is caused by Epstein-Barr virus

○ Bile stained egg


(EBV) infection of the oral mucosa. It most often occurs


○ Mucous plugs at both ends – which are not bile


in association with  HIV infection. It has been less

frequently described in immunosuppressed patients, stained


especially following organ transplantation, and is rare in
14. Ans.  (c) Breaks the drug chemical bonding
immune competent individuals.
Ref: Ananthanarayan and Paniker’s Textbook of
10. Ans.  (d) Bacillus anthracis Microbiology – 10th ed – Page 202
Categories of Most common Bioterrorism Agents that can be • Penicillinases are beta lactamases which are plasmid

Used as an Warfare mediated.They can be transmitted by transduction or


Categories Features Organisms conjugation. These enzymes from the bacteria breaks
the drug and inactivates it.
A High risk and gives a threat to Anthrax • Another mechanism is alteration in the penicillin
national security Botulism

binding protein PBP2a and changes in bacterial surface


Plague receptors reduces binding affinity of beta lactam
Small pox antibiotics to cells.
Tularemia
Ebola 15. Ans.  (a) Meningococci
Marburg
Ref: Ananthanarayan and Paniker’s Textbook of Micro­
B Easy to disseminate but low Brucellosis
biology – 10th ed – Page 235
mortality Glanders
Q fever • Most common form of infection is asymptomatic

Ricin toxin carriage of organism in nasopharynx. xxiii


• A non blanching petechial rash is seen in >80% of the
C Mass dissemination is possible Nipah virus

cases.
• Meningococcal meningitis presents as fever, vomiting,

Acid fast oocysts Treatment of choice


irritability and head ache. The classical signs like neck
Cryptosporidium In AIDS patients: Nitazoxanide
 Latest Exam Questions 2020–2019

rigidity and photobhobia are usually not seen in infants


parvum Others: Paromomycin and Spiramycin
and children.
Isospora belli TMP-SMX
• Along with meningitis, features of septicemia are also
Cyclospora cayetanensis Self limiting; severe cases – TMP-SMX

seen in 20-40%.
• Septicemia can go for shock and death.

19. Ans.  (c) MAT


• Chronic meningococcemia can present as rash, fever,

joint pain, arthritis and splenomegaly. Ref: Harrisons T.B of internal medicine - 19th edition – page
• Post meningococcal sequelae is seen in some patients

1144
which manifest as arthritis, serositis and pericarditis. • A definitive diagnosis of leptospirosis is based on:

ƒ Isolation of the organism from the patient or


ƒ

ƒ A positive result in the polymerase chain reaction


Recent Pattern Questions 2019 ƒ

(PCR) or
ƒ Seroconversion or a rise in antibody titer. (MAT)
16. Ans.  (a) It will become a chronic infection
ƒ

• In cases with strong clinical evidence of infection, a


Ref: Jawetz TB of medical microbiology – 27th edition – page single antibody titer of 1:200–1:800 (depending on

597 whether the case occurs in a low- or high-endemic area)
in the microscopic agglutination test (MAT) is required.
• Rubella infection gets transmitted by vertical

transmission and birth defects are more common when 20. Ans.  (a) VDRL
acquired during first trimester of pregnancy.
• Birth defects are uncommon when infection is acquired

Ref: Harrisons T.B of internal medicine - 19th edition –
after 20 weeks of gestation. page 1136
• Congenital Rubella syndrome leads to cardiac defects,

• Sample of diagnosis for neurosyphilis is CSF

cataract and deafness; (Classical triad). Other mani- • Only tests that can be done for neurosyphilis are:

festations are hepatosplenomegaly, thrombocytopenic ƒ VDRL


ƒ

purpura, myocarditis and bone lesions. ƒ FTA-ABS


ƒ

• Intrauterine infection of Rubella is associated with



• The diagnosis of asymptomatic neurosyphilis is made

chronic persistence of disease. in patients who lack neurologic symptoms and signs but
• Viral detection can be done in all fluids and it is excreted

who have CSF abnormalities including mononuclear
upto 12-18 months of age. pleocytosis, increased protein concentrations, or CSF
reactivity in the VDRL test.
17. Ans.  (c) Streptococcus pneumoniae • When VDRL is negative, FTA-ABS is done to confirm the

test.
Ref: Ananthanarayan and Paniker T.B of microbiology – • Ideally both tests here are used in diagnosis; But when

10th ed – page 225 you have to choose single best option – best is VDRL
• Gram staining clearly shows Gram positive cocci
• according to CDC site and Mandells ID book.
(violet/purple colored) in pairs – classical image of
Streptococcus pneumoniae or pneumococci 21. Ans.  (c) Bacillus anthracis
• Clinical features also helpful to confirm the diagnosis

Ref: Ananthanarayan and Paniker’s Textbook of Micro­
• Infections caused by Pneumococcus:

biology – 10th ed – Page 250
ƒ Pneumonia
ƒ

• Clinical clue here are: abattoir who has contact with


ƒ Meningitis
ƒ

animal products;
ƒ Otitis media
ƒ

• Classical malignant pustule is seen in anthrax. It occurs


ƒ Bacteremia
ƒ

usually in face and neck


ƒ Septic arthritis
ƒ

• A papule is seen at the site of entry followed by vesicle


• It is the most important cause of infections in splenec-



and necrotic ulcer.
tomy patients, chronic alcoholics, sickle cell anaemia • The lesion characteristic of cutaneous anthrax is central

patients. black eschar; This lesion is called as malignant pustule

18. Ans.  (b) Nitazoxanide


Ref: T.B of medical parasitology – S.C.Parija – 4th edition –
page 151
• HIV patient with diarrhea having acid fast organisms

shows that organism may be:


ƒ Cryptosporidium parvum
ƒ

ƒ Isospora belli
ƒ

ƒ Cyclospora cayetanensis
ƒ

xxiv • Image shows 4-5um sized oocysts – suggestive of Cryp-



Central black eschar in cutaneous anthrax
tosporidium (Courtesy: CDC/F. Marc LaForce, MD)
22. Ans.  (b) 20,000 to 1,00,000 units • Because of these variations, vaccines should be modified

according to the current prevalent strain.


Ref: Ananthanarayan and Paniker’s Textbook of

 Latest Exam Questions 2020–2019


Microbiology – 10th ed – Page 245 27. Ans.  (a) Trichuris trichiura
• Specific treatment of diphtheria consists of antitoxic and

Ref: T.B. of medical parasitology – S.C.Parija – 4th edition –


antibiotic therapy. page 265
• Antitoxin should be given immediately when diphtheria
• Clinical features of rectal prolapse is a direct clue for

is suspected, as the fatality rate increases with delay in •

starting antitoxic treatment. diagnosis: Trichuriasis


• Antibiotic treatment only supplements and does not

• This infection is acquired by ingestion of soil with

replace antitoxic therapy. embryonated eggs (has rhabditiform larvae)


• Diphtheria antitoxin should be given in respiratory • It usually affects children and remains asymptomatic

• Heavy infection causes rectal prolapse in children;


diphtheria – 20,000 to 1,00,000 units •

appendicitis;
23. Ans.  (b) leptospirosis • Lab diagnosis: Demonstration of barrel shaped eggs

with mucous plugs in feces


Ref: Harrisons T.B of internal medicine - 19th edition – page
1143


Differential diagnosis for acute febrile illness are:
Infections Classical symptoms
Dengue Fever + Arthralgia + Rash
Malaria Intermittent fever + Splenomegaly + chills
Chikungunya Fever + Arthralgia
Leptospirosis Fever + Jaundice + Conjunctivitis
Scrub typhus Fever + Eschar
Enteric fever Fever + Splenomegaly

24. Ans.  (a) Togaviridae


Ref: Jawetz TB of medical microbiology – 27th edition – page Figure 3: Egg of Trichuris trichiura (Courtesy: CDC/B.G. Partin)
597
• Congenital Rubella syndrome leads to cardiac defects,

28. Ans.  (b) Amoebiasis
cataract and deafness; (Classical triad). Other mani- Ref: T.B. of Medical Parasitology–S.C.Parija–4th edition–
festations are hepatosplenomegaly, thrombocytopenic Page 33
purpura, myocarditis and bone lesions. • Clinical features of amoebiasis are:

• Rubella belongs to Togaviridae


a. Intestinal amoebiasis – characteristic flask shaped


ulcers
25. Ans.  (d) Chicken pox b. Amoebic liver abscess – Anchovy sauce pus
Ref: Harrisons T.B of internal medicine - 19th edition – page c. Lung abscess
1143 d. Brain abscess
• All live attenuated vaccines are contraindicated in

29. Ans.  (c) Paragonimus westermani
pregnancy. Ref: TB of Medical Parasitology–S.C.Parija–4th edition–
• Examples for live attenuated vaccines are:

Page 235
a. OPV A patient with history of crab eating and respiratory
b. Yellow fever vaccine symptoms gives clinical clue for Paragonimus westermani
c. Varicella zoster vaccine (chicken pox) infection
d. MMR • First intermediate host–Snails

e. Influenza (attenuated vaccine) • Second intermediate host–Fresh water crab or crayfish


• Infective form–Metacercariae
26. Ans.  (a) Influenza

• It causes paragonimiasis in pulmonary system; it causes


Ref: Harrisons T.B of internal medicine - 19th edition – page a granulomatous reaction that leads to blood mixed
1209 sputum–consists of golden brown eggs; a fibrous tissue
• Influenza virus has two important antigens: Haemag- that may go for cavitation in some cases;
• Treatment: Praziquantel

glutinin (H) and Neuraminidase (N) •

• These antigens undergo periodic antigenic variations



30. Ans.  (a) BCYE agar
Major antigens variations are seen only with influenza A Ref: Harrisons T.B of medicine – 19th ed – page 1018
viruses and may be associated with pandemics – called
• Legionella isolation is done from respiratory secretions
as antigenic shifts.

– culture media is BCYE agar


• Minor variations causing outbreaks are called as xxv
• Buffered Charcoal Yeast Extract Agar (BCYE)

antigenic drifts.

• Baird Parker agar – Staphylococci

38. Ans.  (b) Broncho alveolar lavage (BAL)


• MacConkey agar – Urine sample – to differentiate LF and

 Latest Exam Questions 2020–2019

NLF Ref: Greenwood – medical microbiology – 18th ed – page 570


• PLET medium – Bacillus cereus

• Pneumocystis jirovecii pneumonia is more common in

31. Ans.  (b) Mucicarmine stain AIDS patients.


Ref: Paniker T.B of microbiology – 10th ed – page 617 • Fever, unproductive cough, progressive shortness of

breath are presenting complaints.


• Stains for Cryptococcus: India Ink stain (negative stain)

• Diagnosis is confirmed by:


done for capsule demonstration and Mucicarmine stain ƒ Fungal cysts (Toluidine blue) from BAL
ƒ

in HPE ƒ Fungal DNA dectection by PCR


• Mucicarmine stain is also used for Rhinosporidium.
ƒ

32. Ans.  (a) Kirby-Bauer method 39. Ans.  (b) Japanese encephalitis

Ref: Paniker T.B of microbiology – 10th ed – page Ref: Greenwood – medical microbiology – 18th ed – page 530

• Kirby Bauer method is the conventional method where



• From the image, it is understood that:

antibiotic disks are kept in equal distance in a lawn ƒ Vector – mosquito (Culex)
ƒ

ƒ Host and reservoir – Pigs (amplifiers) and Herons


culture of bacterium in Mueller Hinton agar.

ƒ

ƒ Accidental host – Humans


• E test is Epsilometer test used for MIC detection ƒ

• The agent is Japanese encephalitis


33. Ans. (b)  IgG2


Ref: Review of medical microbiology and immunology –
AIIMS NOVEMBER 2019
13th ed – page 1127
• IgG has four subclasses: IgG1, IgG2, IgG3 and IgG4
40. Ans.  (b) Botulism

• IgG2 antibody is directed against polysaccharide


antigens and hence it is most important defence against Ref: Ananthanarayan and Paniker’s TB of Microbiology –
encapsulated bacteria. 10th Ed – Page 269
34. Ans.  (c) Diphtheria • Clostridium botulinum is a Gram positive exotoxin

Ref: Greenwood – medical microbiology – 18th ed – page 202 producing organism.


• Strict isolation is must for patient diagnosed with • Eight types of botulinum toxins – A,B,C1, C2, D,E,F,G

• All are neurotoxins except C2 (which is an enterotoxin)


diphtheria. •

• Even when clinically suspected, patient must be isolated. • Action of toxin: Powerful exotoxin will be released by

the organism after death → causes proteolysis of SNARE


• A staff who has known immunisation history should


proteins → blocks the production or release of acetyl

nurse the patient.


choline at the neuromuscular junction and synapses.
35. Ans.  (b) HFMD
Ref: Greenwood – medical microbiology – 18th ed – page 487 41. Ans.  (c) It helps growth of Candida and Staphylococcus
Coxsackie virus group Clinical features Major difference between MacConkey agar and CLED agar
in urine culture is:
A (1-24) • Aseptic meningitis

CLED has additional advantage of:


• Febrile illness

• Candida growth
• Herpangina

• Staphylococcus and other Gram-positive cocci growth


• Hand, foot and mouth disease (HFMD)

B (1-6) • Neonatal disease



42. Ans.  (a) Echinococcus multilocularis
• Bornholm disease

Ref: T.B. of parasitology – Chatterjee – 12th Edition – page 201


• Myocarditis
• Echinococcus multilocularis causes multilocular cyst in

• Hepatitis

• Aseptic meningitis

liver – it spreads to other sites like brain and called as
malignant hydatid disease
36. Ans.  (a) Cytomegalovirus • Definitive host – Foxes

• Intermediate host – Rodents


Ref: Greenwood – medical microbiology – 18th ed – page 439

• Accidental host – Humans


• Clinical features of congenital CMV infection is similar

to those of Erythroblastosis fetalis 43. Ans.  (d) Show twitching motility


• Symptoms are:

ƒ IUGRƒ
Ref: Ananthanarayan and Paniker’s TB of Microbiology –
ƒ Hepatosplenomegaly
ƒ
10th Ed – Page 211
ƒ Jaundice
ƒ • Image showing Gram staining of urethral discharge –

ƒ Thrombocytopenia
ƒ Gram negative diplococci – Neisseria gonorrhea
ƒ Microcephaly
ƒ • Intracellular, non-motile, non-sporing organism

• It is catalase and oxidase positive; Ferments glucose not


37. Ans.  (c) Macrocephaly

xxvi maltose
Ref: Q.21
• Most important virulence factor is pili – outer membrane
• 50. Ans.  (a) Anti HbC
protein
Ref: Ananthanarayan and Paniker’s TB of Microbiology –

 Latest Exam Questions 2020–2019


44. Ans.  (b) Coxiella burnetti 10th Ed – Page 548

Ref: Ananthanarayan and Paniker’s TB of Microbiology – HBsAg HBeAg Anti HBc Anti HBs Anti HBe Interpretation
10th Ed – Page 413 + + IgM – – Acute HBV
• Coxiella burnetti caused Q Fever. It belongs to

infection; highly
Rickettsiaceae family which are obligate intracellular infectious
organisms. + + IgG – – Late/chronic
• They cannot be seen in routine staining;

HBV infection
or carrier state;
45. Ans.  (c) ADP ribosylation highly infectious
Ref: Ananthanarayan and Paniker’s TB of Microbiology – + – IgG – +/– Late/Chronic
10th Ed – Page 241 HBV infection or
• Diphtheria toxin causes ADP ribosyl transferase action

carrier state; low
– which inactivates of ribosomal elongation factor eEF2 infectivity


resulting from ADP ribosylation during protein synthesis – +/– IgM – +/– Seen rarely
which leads to cell death in early acute
• Mechanism of other toxins are given in chapter 14 Vibrio

HBV infection;
cholerae infectious
46. Ans.  (d) Babesia – – IgG +/– +/– Remote HBV
infection;
Ref: T.B. of medical parasitology – S.C. Parija – 4th edition infectivity nil or
– page 138 very low
• Haemo flagellates (Leishmania), Plasmodium and

– – – + – Immunity
Toxoplasma has its life cycle in hepatic stages of human. following HBV
• Babesia is seen inside RBCs. Rupture of these RBCs

vaccine
cause generates cell debris which gets accumulated in
kidney causing renal failure and anemia. It has no cycle
in liver. AIIMS MAY 2019

47. Ans.  (b) Mucormycosis 51. Ans.  (a) Chromoblastomycosis


Ref: T.B of Mycology – Jagdish Chander – 3 Edition – Page
rd
Ref: Ananthanarayan and Paniker T.B of microbiology –
353 10th ed – page 605
• Invasive fungal infections especially due to invasive
• • Chromoblastomycosis is a fungal infection caused by

candidiasis and aspergillosis can be detected by 1,3 beta pigmented fungi (also called as dematiaceous fungi)
D glucan assay. It is also called as G test. • It most commonly affects the agricultural workers and

• The test is helpful in detection of pneumocystis.


• woodcutters, and usually follows a trauma.
• It produces verrucous, warty, cutaneous nodules. Hence

48. Ans.  (c) It resides in gastrointestinal epithelium also called as verrucous dermatitis.
Ref: Ananthanarayan and Paniker’s TB of Microbiology – • Agents:

ƒ Fonsecaea spp.,
10th Ed – Page 133 ƒ

ƒ Exophiala spp.,
• T cell has two types of receptors – αβ or γδ
ƒ

ƒ Cladophialophora
• Most common receptors are αβ T cell type; T cells are
ƒ

• Lab diagnosis: KOH mount or tissue biopsy and staining


seen in the white pulp of the spleen and periarterial of the lesions shows – irregular, dark brown bodies with
lymphoid collection; It is not seen in the gastrointestinal septae (yeast like bodies) – called as Sclerotic bodies
tract. (seen in image as brown colored)
• Treatment: Amphotericin B, Itraconazole
49. Ans.  (b) Both Assertion and Reason are independently

true/correct statements, but the Reason is not the 52. Ans.  (b) Strong Carbol Fuchsin, 0.5% H2SO4 and
correct explanation for the Assertion Loeffler’s methylene blue
Ref: Ananthanarayan and Paniker’s TB of Microbiology – Ref: Ananthanarayan and Paniker T.B of microbiology –
10th Ed – Page 167 10th ed – page 12
Anaphylactoid reaction: • HIV patient with diarrhea, image showing blue
• Some drugs and chemicals can simulate anaphylaxis

background with pink oocysts clearly suggests acid fast


• The reaction is due to direct activation of complements staining showing coccidian parasites (Cryptosporidium xxvii

which releases anaphylatoxins; It is not IgE mediated. or Cyclospora)


Ziehl-Neelsen staining or acid fast staining: • The following table shows the risk factors for CDI

• This method is mainly to visualize the acid-fast


• • Treatment for CDI: Vancomycin and or metronidazole

 Latest Exam Questions 2020–2019

organisms. Organism is seen in pink color with a blue


background. Table 1:  Risk factor for clostridium difficle infection
Procedure: (Ref: Mandells TB of infectious diseases 8th edition, page 2749)
• Strong carbol fuchsin with heat – primary stain

• 20% sulphuric acid – decolorization (Standard method);



Any antibiotic versus no antibiotic:
RNTCP suggests 25% sulphuric acid Number of antibiotics (risk increases with number)
• Methylene blue – Counter stain

Days of antibiotics (increased risk with increased days)
Type of antibiotic:
List of acid-fast organisms and % of H2SO4: Highest risk: Clindamycin, fluoroquinolones, cephalosporins
of second generation and higher
Acid Fast Organism Concentration of Sulphuric Acid Moderate risk: Penicillins, macrolides, penicillin β-lactamase
Mycobacterium 20% H2SO4 (WHO); inhibitors, carbapenems, vancomycin, metronidazole
tuberculosis 25% H2SO4 (RNTCP) Lower risk: Aminoglycosides, tetracyclines, trimethoprim,
Mycobacterium leprae 5% sulfonamides, rifampin
Proton-pump inhibitors and histamine type 2 blockers
Nocardia 1%

Patient age (increased risk with age of the patient)
Spores 0.5% Prior hospitalization
Oocyst of Coccidian 0.5% Severity of underlying illness
parasites Abdominal surgery
Nasogastric tube
53. Ans.  (b) III, I, II, IV Duration of hospitalization
Long-term care residency
Ref: Ananthanarayan and Paniker’s Textbook of
Microbiology – 10th ed – Page 13 56. Ans.  (c) Sporothrix schenckii
Gram staining procedure:
Ref: Ananthanarayan and Paniker’s Textbook of Micro­
• Primary staining with Gentian violet, Crystal violet and
• biology – 10th ed – Page 593
Methyl violet
• Mordant—iodine

Classification of fungi based on pathogenesis
• Decolorization—acetone, acid alcohol, alcohol

Superficial mycoses Dermatophytosis, Candidiasis,
• Counterstain—diluted carbol fuschin (1:10/1:20)

Tinea, Piedra, Pityriasis
versicolor
54. Ans.  (a) 1-C, 2-D, 3-A, 4-B
Cutaneous mycoses Candidiasis
Ref: BMW 2016 guidelines, Ministry of health and family Subcutaneous mycoses Mycotic mycetoma
welfare Chromoblastomycosis
Rhinosporidiosis
Sporotrichosis
Deep or systemic or visceral Blastomycosis
mycoses Cryptococcosis
Paracoccidioidomycosis
Coccidioidomycosis
Histoplasmosis
Candidiasis
Opportunistic mycoses Mucor, Aspergillus, Candida,
Penicillium
Mycotoxicoses Aspergillus, Claviceps purpura

57. Ans.  (b) Presence of Duffy blood group antigen is


55. Ans.  (d) Fluoroquinolones protective for vivax

Ref: Mandells’ infectious diseases – 8th edition – page 2749 Ref: Harrison’s T.B. on internal medicine – 19 th ed – page 1371
• History of antibiotic exposure for longer period of time

Host defenses against falciparum malaria are:
(even more than 7 days) is a risk factor for developing • The geographic distributions of sickle cell disease

Clostridium difficle infection. This CDI is called as • Hemoglobins C and E


pseudomembranous colitis. • Hereditary ovalocytosis


• It is one of the most common health care associated



• Thalassemias

infections in the world. • Glucose- 6-phosphate dehydrogenase (G6PD) deficiency


xxviii •
Host defenses against vivax malaria are: Table 2:  Arthropod-borne diseases
• Absence of Duffy blood group antigen; Individuals with
Arthropod Borne Diseases

 Latest Exam Questions 2020–2019


the Duffy-negative phenotype are resistant to P. vivax
invasion, and the molecular mechanism that gives rise Sandfly •• Kala azar
to the phenotype Fy(a - b -) in black individuals has •• Sand fly fever – three days fever
been associated with a point mutation - 33TC expressed •• Oriental sore
in homozygosity in the FYB allele; It is most commonly •• Oroya fever or Carrion’s disease
seen in Africa •• Chandipura virus
• Duffy antigen – gene – DARC (Duffy antigen/chemokine

Rat flea • Bubonic plague

receptor) • Endemic typhus


• Intermediate host for H.nana


58. Ans.  (a) M.indicus pranii


Louse • Epidemic typhus

Ref: Khullar G et al., (2017), Generalized granulomatous • Trench fever


dermatitis following Mycobacterium w (Mw) immuno­therapy • Epidemic relapsing fever


in lepromatous leprosy, Dermatol Ther. 2017 Mar;30(2)


Trombiculid mites • Scrub typhus

• Mycobacterium w (Mw) vaccine is a heat-killed


Itch mites • Scabies


suspension derived from a non-pathogenic, cultivable,


atypical mycobacterium named Mycobacterium indicus Hard tick (Ixodid) •• Babesiosis


pranii. •• KFD
• Mw immunotherapy has been reported to be efficacious

•• Tick typhus
as an adjunct to multidrug therapy multibacillary •• Q fever
regimen in leprosy patients with high bacillary index. •• Tularemia
• Cutaneous reactions are predominant adverse effects

Soft tick • Endemic relapsing fever


associated with the administration of vaccines.


Chandipura virus (genus Vesiculovirus, family Rhabdoviri-
59. Ans.  (c) Topical bacitracin
dae) causes outbreaks and leads to encephalitis. Chandip-
Ref: Harrison TB of internal medicine – 19th ed – page 959 ura virus encephalitis may be transmitted by sandfly bites
and has been identified in Aedes aegypti mosquitoes. 
• The anterior nares are the most common site of

staphylococcal colonization in humans. 61. Ans.  (b) Chancroid


• It is because of attachment of S. aureus to keratinized

epithelial cells of the anterior nares.


Disease Ulcer Lymphnodes
• Factors that contribute to colonization include the
Primary syphilis Chancre painless Painless-bilateral

ƒ Influence of other resident nasal flora and their


ƒ

bacterial density, Chancroid Painful Painful-usually unilateral


ƒ Host factors
ƒ

Donovanosis Painless Pseudobubo (lesion


ƒ Nasal mucosal damage (use of inhalational drugs)
ƒ
(beefy red) appearing in groin
may present like
Colonization areas are: lymphadenopathy prior to
• Anterior nares

rupture)
• Damaged skin
LGV Transient Painful usually

• Groin (umbilicus)

• Oropharynx

Herpes Painful Painful

• Primary prevention of S. aureus infections in the hospital



Chancroid Bubo LGV Bubo
setting involves hand washing and careful attention to
90% unilateral 2/3 unilateral
appropriate isolation procedures.
Matted Not matted
To eradicate the colonization:
• Decolonization is with topical agents (e.g., mupirocin)

Unilocular Multilocular
to eliminate nasal colonization Rupture with single opening Rupture with multiple opening
• Chlorhexidine baths to eliminate cutaneous colonization

Accompanied by genital ulcer Not accompanied by genital


with S. aureus ulcer
60. Ans.  (c) Kyasanur forest disease Groove sign negative Groove sign positive

Ref: Harrison TB of internal medicine – 19th ed – page 1306-


1308;

xxix
Primary syphilis – presents as chancre (Hunterian Chancre) • It is of two types:

ƒ Cutaneous larva migrans


Chancre Chancroid
ƒ
 Latest Exam Questions 2020–2019

ƒ Visceral larva migrans


ƒ

Single Multiple
Painless Painful Table 3: Cutaneous larva migrans and visceral larva

Clean base Necrotic base migrans


Bilateral inguinal nodes Unilateral inguinal nodes
Cutaneous Larva Migrans Visceral Larva Migrans
No exudates Yellow exudates
• • Ancylostoma braziliense (M/c) •• Toxocara canis
• Ancylostoma caninum (second M/c) • Toxocara cati
62. Ans. (a) A lifeguard in swimming pool
• •

• Necator americanus • Gnathostoma


• •

Cutaneous larva migrans (CLM) is a clinical syndrome • • Ancylostoma duodenale •• Anisakis


consisting of an erythematous migrating linear or • • Gnathostoma spinigerum
serpiginous cutaneous track; • • Strongloides
It is also called as creeping eruptions • • Loa loa
Individuals at greatest risk include travelers, children, • • Fasciola
swimmers in lakes, and laborers whose activities bring • Paragonimus

their skin in contact with contaminated soil


Larva migrans: • The question is quite a tricky one. A lifeguard in

• It is caused by nematodes that usually cause infection



swimming pool has risk comparatively, as poultry has
to animals no link with these organisms. Most of the organisms are
• When they accidentally enter humans – it cannot

dog and canines’ sources.
complete the cycle and hence gets arrested in the body
causing larva migrans

63. Ans. (c) Filariasis


  • Once MDA has stopped, TAS is used as a surveillance

tool to determine that infection


Ref: https://www.who.int/lymphatic_filariasis/global_
• A transmission assessment survey (TAS) is designed to
progress/transmission_assessment_survey/en/

evaluate whether transmission of lymphatic filariasis


• Filariasis elimination needs mass drug administration.

have reached lower level or not, based on that MDA can
• WHO recommends the transmission assessment survey be stopped.
xxx

(TAS) to determine when infections have been reduced


below these target thresholds and MDA can stop.
Transmission assessment Survey (TAS)
A TAS is the basis for a decision to move from MDA to post-MDA surveillance.

 Latest Exam Questions 2020–2019


Technical Aspect Guidance
Geographical area Evaluation unit (EU)
When survey should be conducted • When all the eligibility criteria are met

• At least 6 months after the last round of MDA


Target population Children aged 6–7 years


Diagnostic tests W. bancrofti areas: ICT
Brugia spp. areas: Brugia RapidTM
Survey design Cluster sampling of systematic sampling in schools or the community, or a census

A TAS is a simplified version of the ‘stopping-MDA survey’ protocol.

64. Ans. (c) IgE



Ref: Owen Kuby – Immunology – 8th ed – page 487
• Allergy (Type I hypersensitivity) is always IgE mediated, both in acute and chronic.

• E.g. Hay fever, Asthma


Figure 1: Pathogenesis and mechanism of type I hypersensitivity (Ref: Kuby)

Different examples for allergies (Ref: Kuby)


Disorder Symptoms Common trigger Notes about mechanism
IgE mediated (acute)
Hives (urticaria) Wheal and flare swellings Multiple foods
triggered by ingestion of skin
contact
Contd... xxxi
Disorder Symptoms Common trigger Notes about mechanism
Oral allergy Itchiness, swelling of mouth Fruits, vegetables Due to sensitization by inhaled
 Latest Exam Questions 2020–2019

pollens, producing IgE that


cross-reacts with food proteins
Asthma, rhinitis Respiratory distress Inhalation of aerosolized food Mast-cell mediated
proteins
Anaphylaxis Rapid, multiorgan inflammation Peanuts, tree nuts, fish, shell-
that can result in cardiovascular fish, milk, etc.
failure
Exercise-induced anaphylaxis As above, but occurs when one Wheat, shellfish, celery (may be
exercises after eating trigger due to changes in gut absorp-
foods tion associated with exercise)
IgE and cell mediated (chronic)
Atopic dermatitis Rash (often in children) Egg, milk, wheat, soy, etc. May be skin T cell mediated
Gastrointestinal inflammation Pain, weight loss, edema, and/ Multiple foods Eosinophil mediated

or obstruction
Cell mediated (chronic)
Intestinal inflammation brought Most often seen in infants: Cow’s milk (directly of via breast TNF-α mediated
about by dietary protein (e.g. diarrhea, poor growth, and/or milk), say, grains
enterocolitis, proctitis) bloody stools
Adapted from S.H. Sicherer and H. A. Sampson, 2009, Food allergy, Annual Review of Medicine 60:261-277

65. Ans.  (c) HbcAB


Ref: Ananthanarayan and Paniker’s Textbook of Micro­biology – 10th ed – page 550
•• HbcAg – cannot be demonstrable in blood
•• The earliest antibody marker to be identified is Anti HbcAg – IgM
•• After six months IgG starts appearing
•• Question dint mention IgM or IgG (hence that’s the answer)

Serological markers for diagnosing HBV infection


HBsAg HBeAg Anti HBc Anti HBs Anti HBe Interpretation
+ + IgM – – Acute HBV infection; highly infectious
+ + IgG – – Late/chronic HBV infection or carrier state; highly infectious
+ – IgG – +/– Late/Chronic HBV infection or carrier state; low infectivity
– +/– IgM – +/– Seen rarely in early acute HBV infection; infectious
– – IgG +/– +/– Remote HBV infection; infectivity nil or very low
– – – + – Immunity following HBV vaccine

JIPMER MAY 2019 67. Ans.  (a) Hymenolepis nana


Ref: Medical parasitology – Arora – 3rd ed – page 209
66. Ans.  (b) Hymenolepis diminuta
• Hymenolepis nana is the smallest intestinal cestode that

Ref: Medical parasitology – Arora – 3rd ed – page 261 infects human – named as Dwarf tapeworm
• Auto infection is caused by certain parasites, when the

• H.diminuta – size of egg is larger than nana

eggs or larvae gets entered into the body from one cavity
to another. 68. Ans.  (a) Ectoparasitic skin infection
• Parasites causing auto infection are: (Mneumonic: CS –

Ref: https://emedicine.medscape.com/article/231037-
TECH) treatment
ƒ Cryptosporidium parvum
• Tungiasis is a neglected tropical disease caused by the
ƒ

ƒ Strongyloides stercoralis

permanent penetration of the female sand flea (also


ƒ

ƒ Taenia solium
called jigger flea)
ƒ

ƒ Enterobius vermicularis
• After penetration, most commonly on the feet, the flea
ƒ

xxxii ƒ Capillaria philippinensis


undergoes an impressing hypertrophy


ƒ

ƒ Hymenolepis nana
ƒ
Host and Dead end → Humans
Reservoir host → Herons

 Latest Exam Questions 2020–2019


Amplifier host → Pigs
Vector → Culex tritaeniorhynchus and
vishnui

73. Ans.  (d) Ricketssia typhi


Ref: Ananthanarayan and Paniker’s TB of microbiology –
10th Ed – page 417
Transovarian transmission of R.typhi occurs in ticks –
which acts as both vector and reservoir – caused Endemic
typhus
Figure 2: Showing the skin lesions of tungiasis

Any skin lesion is a traveller or from endemic areas, the 74. Ans.  (c) Cyclospora cayetanensis


following differentials are diagnosed:
• Cutaneous larva migrans

Ref: T.B of medical parasitology – S.C. Parija – 4th Ed – page
• Pyoderma

155
• Arthropod-reactive dermatitis

Coccidean Parasites Identification Features


• Myiasis
Cyclospora Oocysts are of spherical or oval;

• Tungiasis
cayetanensis Size is around 8 – 10um; It has two

• Urticaria
sporocyts; Each sporocyts has

• Fever and rash


2 sporozoites

• Cutaneous leishmaniasis
Acid fast oocyst; Partially acid fast

69. Ans.  (d) RT PCR Cryptosporidium Oocysts are of spherical or oval;


parvum Size is around 4-5um; It has 4
Ref: https:/www.cdc.gov/zika/
sporozoites
• Diagnosis of zika virus – samples: Whole blood, urine,

Acid fast oocyst


CSF, amniotic fluid, semen and saliva
• Gold standard method – RT PCR Cystisospora belli Oocyst looks ellipsoidal (boat
shaped); 20-33*10-19um; Each

• < 7 days of illness only RT PCR can detect; >7days of


sporocyts has 2 sporozoites; Acid

illness – serological tests can help.


fast
70. Ans.  (a) Mec A
75. Ans.  (b) Corynebacterium urealyticum
Ref: Ananthanarayan and Paniker’s TB of microbiology –
10th Ed – page 202 Ref: https://www.ncbi.nlm.nih.gov/pmc/articles/
PMC6442166/
Resistance in Staphylococcus aureus:
1.  Production of beta lactamases • Encrusted cystitis is a condition caused by urea splitting

2.  Alteration of penicillin binding protein (PBP2a) bacteria.


3.  Chromosomally mediated resistance – SCC mec – Mec

• Among the given options, C.urealyticum is urease

A gene producer and causes rarely encrysted cystitis.

71. Ans.  (d) Staphylococcus aureus 76. Ans.  (b) Corynebacterium diphtheriae

Ref: Ananthanarayan and Paniker’s TB of microbiology – Ref: Ananthanarayan and Paniker’s TB of microbiology –
10th Ed – page 204 10th Ed – page 242
Lab diagnosis of C.diphtheriae
Enzymes Toxins 1. Staining techniques to demonstrate the metachromatic
• • Coagulase •• Cytolytic toxins – hemolysis granules – Albert staining, Neisser’s staining, Ponder’s
• • Lipid hydrolases •• Panton Valentine leukocidin staining
• • Hyaluronidase •• Enterotoxin 2. Growth medium – Loefflers serum slope (selective
• • Nuclease •• TSST medium)
•• Epidermiolytic toxin 3. Toxin demonstration can be done only by:
i. Animal inoculation
72. Ans.  (a) Pig
ii. Elek’s gel precipitation test – confirmatory method
Ref: Ananthanarayan and Paniker’s TB of microbiology – iii. PCR
10th Ed – page 528
xxxiii
77. Ans.  (d) Acinetobacter baumannii
 Latest Exam Questions 2020–2019

Ref: Ananthanarayan and Paniker’s TB of microbiology – 10th Ed – page 321

Gram-positive cocci Gram-negative cocci Gram-positive bacilli Gram-negative bacilli Gram-negative cocco bacilli
Staphylococci Neisseria Corynebacterium E.coli Acinetobacter
Streptococci Clostridium Proteus Haemophilus
Enterococci Klebsiella Gardnerella
Pseudomonas Chlamydia
Citrobacter
Enterobacter
Vibrio
Salmonella
Shigella

78. Ans. (a) It occurs only when CD4 cell count is less PGI MAY 2019

than 50
Ref: https://www.uptodate.com/contents/immune- 81. Ans.  (b) Aedes aegypti is the vector responsible for
reconstitution-inflammatory-syndrome/ transmission; (c) Commonly presents as fever and
arthralgia; (d) In acute phase of illness, virus cannot be
• IRIS - a collection of inflammatory disorders associated

isolated from blood


with paradoxical worsening of pre-existing infectious
processes following the initiation of antiretroviral Ref: Ananthanarayan and Paniker’s T.B of microbiology –
therapy (ART) in HIV-infected individuals  10th Ed – page 524
• Increases in T lymphocytes after initiation of ART are

• Chikungunya virus is an RNA virus belongs to the family


also accompanied by increased in vitro lymphocyte Togaviridae.


proliferation responses, increased markers of immune • Vector is Aedes aegypti; No animal reservoir

activation, and pathogen-specific delayed hyper­ • An abrupt onset of fever with joint pain is the feature.

sensitivity Reactive arthritis may persist for a year.


• IRIS usually develops 1-2 days after initiation of anti retro

• RT PCR can be used; Viral isolation can be done but


viral therapy especially in a patient with Tuberculosis. difficult.


• Patient who developed IRIS with opportunistic infections

• Ideal tests: Serological ELISA techniques.


can continue ART; if no opportunistic infection, ART


can be discontinued and re started again. 82. Ans.  (a) Also called as Saddle back fever; (b) Symptoms
appear within 48 hours of infection; (e) Maculopapular
79. Ans.  (c) Enterococcus faecalis rashes over trunk and limbs
Ref: Ananthanarayan and Paniker’s TB of microbiology – Ref: Ananthanarayan and Paniker’s T.B of microbiology –
10th Ed – page 219 10th Ed – page 529
• Enterococcus species are Gram positive cocci that are

• Dengue virus – belongs to flaviviridae family. It is RNA


arranged at an angle to each other. virus with four serotypes – DEN 1-4
• Special characteristics:

• Vector is Aedes aegypti; Day biting mosquitoes


ƒ Temperature for growth – 10 to 45 deg C


ƒ

• Incubation period – 3 to 14 days; Causes fever, arthralgia


ƒ Bile esculin hydrolysis – positive


ƒ

in the first phase – Break bone fever.


ƒ Grows in 6.5% Sodium chloride
ƒ

• Maculopapular rash appears in 3-5 days, followed by


ƒ Intrinsically resistant to Penicillin, Cephalosporins


ƒ

critical phase of plasma leakage – shock syndrome


and low-level gentamicin (especially in secondary infection)
80. Ans.  (c) Mycobacterium avium 83. Ans.  (b) All except leprae
Ref: Ananthanarayan and Paniker’s Textbook of Micro­ Ref: Ananthanarayan and Paniker’s T.B of microbiology –
biology – 10th ed – Page 367 10th Ed – page 352
Runyon Classification Species Lowenstein Jensen medium – composed of coagulated
hen’s eggs, mineral salt solution, asparagines and
Photochromogens M. kansasii, M. marinum
malachite green;
Scotochromogens M. scrofulaceum, M. gordonae Malachite green inhibits the growth of other bacteria and
Non-photochromogens M. avium, M. intracellulare, makes the media selective for Mycobacterium; It allows
M. ulcerans, M. xenopi growth of M.tuberculosis, atypical Mycobacteria and
Nocardia.
Rapid growers M. fortuitum, M. chelonae,
xxxiv M. smegmatis, M. phlei
84. Ans.  (a) Significant chest X-ray findings and (c) Standard HBsAg HBeAg Anti HBc Anti HBs Anti HBe Interpretation
TB regimen is used for treatment
– – IgG +/– +/– Remote HBV

 Latest Exam Questions 2020–2019


Ref: Ananthanarayan and Paniker’s T.B of microbiology – infection;
10th Ed – page359 infectivity nil or
WHO LTBI guidelines very low
Latent TB infection: – – – + – Immunity
• A state of persistent immune response to stimulation by

following HBV
Mycobacterium tuberculosis antigens with no evidence vaccine
of clinically manifest active TB.
• There is no gold standard test for direct identification of
• 87. Ans.  (d) Autoimmune haemolytic anemia; (e) Good
Mycobacterium tuberculosis infection in humans. pasture syndrome
• The vast majority of infected people have no signs or
Ref: Ananthanarayan and Paniker’s TB of Microbiology –

symptoms of TB but are at risk for active TB disease.


10th Ed – Page 168
• Mantoux test will be positive. IGRA cannot differentiate

active and latent infection. Type Typical Time Clinical Manifestation or


of Onset Disease


85. Ans.  (a) Incubation period is 1-6 days; (c) Pharyn-
gotonsillar diphtheria usually present as sore throa; I (Immediate, Minutes •• Systemic anaphylaxis
(d) Membrane can cover pharynx, tonsils, soft and hard anaphylactic) •• Urticaria
palate •• Asthma
•• Hay fever
Ref: Ananthanarayan and Paniker’s T.B of microbiology – •• Allergic rhinitis
10th Ed – page 239 •• Allergic conjunctivitis,
Corynebacterium diphtheriae – Gram positive rods that •• Angioedema
have characteristic metachromatic granules; II (Cytotoxic) Hours to days ••Hemolytic anemia,
Incubation period – 2 to 5 days with a range of 1-10 days ••Neutropenia,
Toxin production is the major pathogenetic virulence factor ••Thrombocytopenia,
which causes necrosis and pseudomembrane formation in ••ABO transfusion reactions,
the tonsillar area ••Rh incompatibility
Pharyngeal diphtheria is the commonest type. When (erythroblastosis fetalis,
involving the regional lymph nodes, can cause respiratory hemolytic disease of the
obstruction. newborn)
Treatment: Erythromycin and Antitoxin • Rheumatic fever

• Goodpasture’s syndrome
86. Ans.  (a) HBsAg; (c) IgM anti HBc; (d) HBeAg

• Grave’s disease

Ref: Ananthanarayan and Paniker’s TB of Microbiology – III (Immune 2 to 3 weeks • Systemic lupus

10th Ed – Page 548 Complex) erythematosus


• Rheumatoid arthritis
HBsAg HBeAg Anti HBc Anti HBs Anti HBe Interpretation

• Poststreptococcal

+ + IgM – – Acute HBV glomerulonephritis


infection; highly • IgA nephropathy

infectious • Serum sickness


• Hypersensitivity
+ + IgG – – Late/chronic

pneumonitis (e.g., farmer’s


HBV infection
lung)
or carrier state;
highly infectious IV (Delayed) 2 to 3 days • Contact dermatitis,

• Tuberculin skin test


+ – IgG – +/– Late/Chronic reaction,


HBV infection • Stevens Johnson Syndrome,

or carrier state; • Toxic epidermal necrolysis,


low infectivity • Erythema multiforme


– +/– IgM – +/– Seen rarely


88. Ans.  (c) Blood culture
in early acute
HBV infection; Ref: Ananthanarayan and Paniker’s TB of Microbiology –
infectious 10th Ed – Page 349
Contd... Brucellosis:
• Blood culture is the most definitive method of diagnosis

– Castaneda method; Automated methods with xxxv


prolonged incubation (Average 7 days)
• Serological methods – Standard agglutination test and

92. Ans.  (b) Aseptate hyphae


ELISA; SAT method is most sensitive method.
Ref: Ananthanarayan and Paniker’s TB of Microbiology –
 Latest Exam Questions 2020–2019

89. Ans.  (c) Presents with pneumonia and skin abscess; 10th Ed – Page 614
(d) May takes years before becoming symptomatic; • Septate hyphae and Filamentous – Aspergillus

(e) Eradication treatment is required with cotrimoxazole • Aseptate hyphae – Mucor, Rhizopus

or doxycycline for at least 12 weeks • Beta D glucan assay is helpful only for invasive

Ref: Ananthanarayan and Paniker’s TB of Microbiology – candidiasis and aspergillosis


10th Ed – Page 323 • Most common environmental fungi that grows in fruits

and vegetables
• Melioidosis – caused by Gram negative bacilli bacteria –
• Rapid grower; Produces thick cottony fluffy colonies

Burkholderia pseudomallei

• It is most commonly seen in vegetations, rice paddy


93. Ans.  (d) Enzyme immunoassay (EIA) followed by RPR


fields (Thailand and South East Asian countries)
• The traditional testing algorithm for syphilis begins
• Infection occurs by inoculation, inhalation, ingestion

with a screening nontreponemal test such as rapid


causing pneumonia, septicemia and acute pulmonary


plasma reagin (RPR), with positive results followed
infection, also cutaneous lesions.
by a confirmatory treponemal test such as fluorescent

• Incubation period: Average 1-21 days. Melioidosis may
treponemal antibody or  T. pallidum  particle

also remain latent for months or years before symptoms


agglutination assay (TP-PA)
develop.
• Because of cost issues and rapid testing kits, a reverse
• Treatment: Ceftazidime followed by carbapenem or

testing algorithm which first screens with a treponemal


TMP-SMX for a minimum of 8 weeks


test (e.g., syphilis IgG); reactive samples are then tested
90. Ans.  (a) PPLO broth by RPR which is used to assess disease activity 

Ref: Ananthanarayan and Paniker’s TB of Microbiology – 94. Ans.  (e) Giardia lamblia
10th Ed – Page 393 The protozoa that are infectious to humans can be classified
• Mycoplasma does not have cell wall and cannot be
• into four groups based on their mode of movement:
culture in routine cultivation methods. • Sarcodina – the ameba, e.g., Entamoeba
•  

• Culture needs PPLO broth and growth is very slow.


• • Mastigophora – the flagellates, e.g., Giardia, Leishmania
•    

• PPLO – Pleuro pneumonia like organisms (Mycoplasma


• • Ciliophora – the ciliates, e.g., Balantidium
•  

family) • Sporozoa – organisms whose adult stage is not motile


e.g., Plasmodium, Cryptosporidium


   

91. 11. Ans.  (a) Cervicitis; (b) Non-gonococcal urethritis;


(c) Bacterial vaginosis 95. Ans.  (b) gp 41: Envelop Ag
Ref: Ananthanarayan and Paniker’s TB of Microbiology – Ref: Ananthanarayan and Paniker’s TB of Microbiology –
10th Ed – Page 393 10th Ed – Page 575
Gene product Description
Organism Site of Disease that are caused
gp160 Precursor of envelope glycoproteins
infection
gp120 Outer envelope glycoproteins of virion
M.pneumoniae Respiratory •• Pharyngitis
infection •• Tracheobronchitis p66 Reverse transcriptase and RNase H from
•• Pneumonia polymerase gene product
•• Arthritis p55 Precursor of core proteins, polyprotein
•• Pericarditis from gag gene
• Hemolytic anaemia
p51 Reverse transcriptase, RT

•• GBS
•• Skin lesions gp41 Transmembrane envelope glycoproteins,
•• Encephalitis TM
M.genitalium Genito urinary • Non gonococcal

p32 Integrase, IN
infection urethritis (NGU) p24 Nucleocapsid core protein of virion
• PID
p17 Matrix core protein of virion

M.hominis Both • Pyelonephritis


respiratory and • Postpartum fever


genitourinary • Systemic infections


infection
U.urealyticum Both •• NGU
•• Pyelonephritis
xxxvi •• Spontaneous abortion
•• Premature birth
  Sample Video Questions
Sample Video Questions
1. In a war conflict zone, 150 patients got acute watery
3. Identify the following organism visualised by wet mount

diarrhea. On stool examination, following motility of vaginal discharge?


pattern is seen. Which culture media is ideal for
identifying the organism?

a.
Candida albicans
b. Cryptococcus neoformans
a. DCA agar b. TCBS agar

c. Trichomonas vaginalis
c. MacConkey agar d. Blood agar

d.
Gardnerella vaginalis
2. A 28 year old newly married female presented with

4. Vector for this African eye worm is:


complaints of intense itching per vagina and greenish

discharge. On wet mount of vaginal discharge, following


organism is seen. What morphological stage is this?

a.
Simulium

b.
Chrysops
a. Cyst b. Trophozoites c. Anopheles
xxxvii

c. Tachyzoites
d. Oocysts

d.
Xenopsylla
5. A patient who had undergone renal transplantation 7. An asymptomatic man on routine stool examination for
three months before has presented with bloody diarrhea. health check up showed the following finding. What is
  Sample Video Questions

Stool sample shows the following organisms on culture the organism seen?
measuring 280 um. Which of the following is true among
the following:

a. It causes Loeffler's pneumonia a. Entamoeba coli


b. The video shows the filariform larvae of the parasite b. Balantidium coli
c. It is acquired by contamination of food/water c. Balamuthia mandrillaris
d. They are monoecious since they undergo partheno- d. Dientamoeba fragilis
genesis

6. A 43 years old man came with acute abdomen to 8. Following staining method is used to demonstrate:
casualty. Immediate laparatomy and bowel resection
showed following findings. What is the causative agent?

a. Ancylostoma duodenale a. Cell wall


b. Necator americanus b. Spores
c. Ascaris lumbricoides c. Capsule
d. Enterobius vermicularis d. Flagella

xxxviii
9. What type of culture streaking method is shown here:
10. Identify the organism:

  Sample Video Questions


a. Stab culture
b. Streak culture
a. Strongyloides
b. Hookworm

c. Lawn culture
d. Liquid culture
c. Borrelia
d. Wucheria

For video, scan this QR Code

ANSWERS FOR VIDEO QUESTIONS

1. Ans. (b) TCBS agar


  3. Ans. (a) Candida albicans

• Video shows typical darting motility of Vibrio cholerae


• •• Vaginal discharge shows yeast cells with pseudohyphae
• Vibrio cholerae has polar flagella and exhibits fast
• •• True yeasts : Cryptococcus
motility •• Yeast with pseudohyphae: Candida albicans
• Clinical features shown as acute watery diarrhea causing
• •• Hyphae are seen in other fungi like dermatophytes,
cholera outbreak in a camp Aspergillus, Rhizopus, Mucor
• Vibrio cholerae is a Gram negative bacilli which grows

in selective medium called Thiosulfate-citrate-bile 4. Ans. (b) Chrysops


salt – sucrose agar (TCBS). Colonies of V.cholerae are • Loiasis, called African eye worm by most people, is

yellowish in colour because of sucrose fermentation. caused by the parasitic worm Loa loa.
• It is transmitted to humans by the repeated bites of

2. Ans. (b) Trophozoites


deerflies of the genus Chrysops.


• A sexually active female with greenish discharge
• • These flies are seen in West and Central Africa.

indicates Trichomoniasis • Infection with the parasite can also cause repeated

• Video showing twitching motility of Trichomonas


• episodes of itchy swellings of the body known as Calabar
vaginalis swellings.
• Trichomonas vaginalis has only trophozoite stage; it do
• • In some patients, visible movement of worm is seen

not have cystic stage. inside the eye; but it wont cause much effects to the eye.

xxxix
5. Ans.  (b) The video shows the filariform larvae of the 7. Ans.  (b) Balantidium coli
parasite • Balantidium coli, a large ciliated protozoan parasite.
  Sample Video Questions

• The clinical clue to identify the organisms are: 1) renal


• • Cysts are the parasite stage responsible for transmission

transplantation 2) diarrhea of balantidiasis.


• The microbiological clue is stool shows larvae (only
• • The host most often acquires the cyst through ingestion

in two organisms – stool shows larvae, i.e., They are 1) of contaminated food or water.
Strongyloides stercoralis 2) Hook worm • Most cases are asymptomatic.

• Infections are more common in HIV, immunosuppressed


• • Clinical manifestations, when present, include

and post-renal transplantation patients persistent diarrhea, occasionally dysentery, abdominal


• Diagnosis: By demonstration of rhabditiform larvae
• pain, and weight loss. Symptoms can be severe in
in freshly passed stool; Filariform larvae can be debilitated persons.
demonstrated by the culture of the stool by Harada Mori
filter paper method. 8. Ans.  (c) Capsule
• Treatment is Ivermectin and Thiabendazole

• India Ink staining is shown in the video

• It is mainly used to stain capsule for Cryptococcus


6. Ans.  (c) Ascaris lumbricoides neoformans.


• Infection with  Ascaris  starts with the ingestion of
• • CSF sample are directly stained with India ink stain.

embryonated eggs which hatches into larva to pierce


through the duodenal wall to reach portal circulation, 9. Ans.  (c) Lawn culture
and via liver they migrate to lungs where they change • Lawn culture is plating the culture media plate all

into larva. the sides and it is mainly helpful for antimicrobial


• Larvae of Ascaris reach alveoli and with the secretions

susceptibility testing.
move up to upper respiratory tract, and with the swal- • Streak culture is done with bacteriological loop and

lowed sputum completes the journey in small bowel. used to isolate organism.
• In small bowel, they develop in an adult and usually live

asymptomatically, but in cases with high worm load they 10. Ans.  (c) Borrelia
can also produce intestinal obstruction or perforation • Typical spiral structure which are motile in dark ground

peritonitis;  microscopy describes that the organism is a spirochaete.


• In tropical countries with acute abdomen, intestinal

• Among the options, Borrelia is the spirochaete.

obstruction with Ascaris lumbricoides must be kept in


mind.

xl
Image-Based Concept Zone
Parasitology
1. Cyst of Entamoeba histolytica 2. Cysts of Giardia lamblia

• E. histolytica is an intestinal protozoan • Ellipsoidal in shape




• Mature cyst has four typical nuclei (Quadrinucleate) • Mature cyst has four nuclei


• One nucleus is clearly visible here • Identification is by correlation with clinical features


DD: Cyst of Entamoeba coli-it has 8 nuclei like malabsorption, steatorrhea and by seeing cysts in
feces.

3. Trypanosoma brucei 4. Amastigotes of Leishmania donovani

• Peripheral blood film shows Trypanosoma brucei • Image shows smear from bone marrow


gambiense • Many amastigotes are seen invading the REC and

• It occurs in three forms: Long slender with flagellum, macrophages nearby

intermediate form and short stumpy form without • Amastigotes seen near macrophages suggest

flagellum. L. donovani
• Metacycylic trypomastigote is the infective form for

man
• Short stumpy form is the infective form for tsetse fly

Image-Based Concept Zone  Parasitology

5. Gametocyte of Plasmodium falciparum 6. Schizont stage of P. vivax




• Forms seen in peripheral smear are rings and • P. vivax causes benign tertian malaria




crescents (gametocytes)–Banana shaped Prominent feature is presence of Schuffner’s dots


• Infected RBC–normal in size and possesses 6–12 • Infected RBC is bigger than normal




maurer’s dots Trophozoite stage divides to form a Schizont stage


• Duration of erythrocytic schizogony–36 to 48 hours • Image shows Schizont which has merozoites inside it



• No exo erythrocytic cycle

• Most pathogenic; highest level of parasitemia

• Causes black water fever and cerebral malaria

7. Cyst of Balantidium coli 8. Babesia microti

• Large round-shaped cyst • Babesia infections are transmitted by ticks




• It has macro nuclei and micro nuclei • The peripheral smear image shown here shows rings


• Macro nuclei is seen in this image at opposite to each other

• Micro nuclei is not seen here • When four rings are seen like this–it gives typical


• Tiny cilia is seen near the edge of the cysts (ciliated Maltese cross pattern of Babesiosis

protozoa) • But you cannot see in all the smears

• With clinical correlation–diagnosis has to be made

xlii
Image-Based Concept Zone  Parasitology
9. Oocysts of Cryptosporidium parvum 10. Oocysts of Cyclospora cayetanensis



• Oocysts are of spherical or oval; • Oocysts are of spherical or oval



• Size is around 4–5 mm; • Size is around 8–10 mm


• It has 4 sporozoites (can be seen only under • •
It has two sporocyts

microscope–cannot be seen in this image) • Each sporocyts has 2 sporozoites

• Acid fast oocyst • Acid fast oocyst; Partially acid fast


• Identification is with help of clinical features and size

is smaller than Cyclospora

11. Oocysts of Cystisospora belli 12. Egg of Taenia

• Oocyst looks ellipsoidal (boat shaped); • Bile stained



• Bigger in size

• Outer–thin layer

• Each sporocyts has 2 sporozoites;

• Inner–oncosphere

• Acid fast

• Embryo–Hexacanth embryo


• Egg of T. saginata and T. solium cannot be differentiated

•  Infective eggs:

ƒ Taenia saginata–infective to cattle only;
ƒ
ƒ T. solium–infective to pigs and human beings
xliii
ƒ
• Egg of Taenia saginata is acid fast

Image-Based Concept Zone  Parasitology

13. Egg of Hymenolepis nana 14. Egg of Schistosoma




A B C

(A) Schistosoma haematobium


ƒ Terminal spine

ƒ
(B) Schistosoma mansoni
ƒ Lateral spine

ƒ
(C) Schistosoma japonicum
ƒ Lateral small knob

ƒ
• Smallest cestode infecting humans

• Egg is Spherical which has two lining membranes

• It has three pairs of hooklets

• Polar filaments seen

• Non bile stained egg

15. Egg of Fasciola 16. Fertilized egg of Ascaris lumbricoides

• Bile stained egg • Round to oval in shape




• Oval shaped egg • Bile stained with unsegmented ovum


• Operculated with a large unsegmented ovum • Has a thick albuminous coat


• It is difficult to differentiate egg of Fasciola hepatica • It has a clear crescentic area between the ovum and


and Fasciolopsis buski egg shell

xliv
Image-Based Concept Zone  Parasitology
17. Egg of hookworm 18. Egg of Enterobius vermicularis



• Egg is oval shaped • Also called as thread worm, pin worm, seat worm



• Nonbile stained egg (Nematode)

• It has a thin transparent hyaline shell • Eggs are colorless, not bile stained, plano convex



• Ovum is segmented with four blastomeres (Flattened on one side)

• It is not possible to differentiate egg of A. duodenale • Surrounded by a thin, smooth, transparent shell


and N. americanus and usually contain well developed larvae inside it
(a coiled tad pole like larva)
• Identification is by characteristic eggs and clinical

history that affects mostly children causing auto
infection and nocturnal itching

19. Larvae of Strongyloides stercoralis 20. Egg of Trichuris trichiura

• The Smallest Nematode



• Opportunistic pathogen in immunocompromised

hosts • Called as whipworm

• Most commonly seen form in stols–Rhabditiform • Lives in large intestine: Caecum and vermiform


larvae appendix
• Infective form is Filariform larvae • Eggs have characteristic mucous plugs at both poles;


• No intermediate host Eggs are bile stained

• Causes larva currens • Embryonated eggs are the infective form


• Pulmonary lesions, intestinal symptoms, hyperinfec- • Causes mucoid diarrhea, malnutrition


tion and disseminated infections in immunocompro- • In children it causes rectal prolapse

mised • DD for egg: Capillaria phillipensis
xlv

• Baermann technique–for culture

Mycology
Image-Based Concept Zone  Mycology

21. True Yeast–Cryptococcus sp., 22. Yeast-like fungi-Candida spp




• Image showing wet mount with India Ink stain shows • Image showing Gram staining of the specimen which


capsulated yeast cells has Gram-positive budding yeast cells with pseudo
• Only fungi that is capsulated is Cryptococcus neofor- hyphae or hyphae

mans • Only fungi that can be seen by Gram staining is

Candida–since pseudo hyphae is seen–the species is
Candida albicans

23. Septate Molds-Aspergillus spp 24. Aseptate molds-Mucor spp

• Image showing Septate hyphae with acute angle • Microscopy of the clinical specimen shows: Broad,


branching at 45°C aseptate hyphae
• Typical dichotomous acute angled branching is seen • Sporangiophores arising randomly along the aerial


in Aspergillus spp., mycelium; No rhizoids
xlvi • Same image–if rhizoids are seen–then it is Rhizopus

spp.
Image-Based Concept Zone  Mycology
25. Dermatophytes-Trichophyton rubrum 26. Dermatophytes-Microsporum canis



• Image showing septate thin hyphae with abundant • Image showing thin septate hyphae, Abundant spindle


tear drop shaped micro conidia (Bird on Fence shaped macroconidia with no microconidia
pattern) • This feature suggests of Microsporum canis

• This features suggests Trichophyton rubrum

27. Coccidioides spp 27. Histoplasma capsulatum

• Image showing: Body tissue or Culture isolate at 37°C • Image showing Spores with tubercles (finger-like


showing Spherules which are thick double layered projections)
wall filled inside with endospores • This projections are characteristic of Histoplasma

• Spherules are characteristic of Coccidioides spp

xlvii
Image-Based Concept Zone  Mycology

28. Rhinosporidium seeberi 29. Chromoblastomycosis





• Image showing Spherules that contain numerous • Image showing irregular, dark brown with a septate


endospores are seen yeast like bodies–Sclerotic bodies
• These endosporulation is seen in Rhinosporidiosis • Sclerotic bodies are unique for Chromoblastomycosis


xlviii
GENERAL
1

MICROBIOLOGY
Unit Outline

1 Chapter 1 Introduction, History and Microscopes


Chapter 2 Morphology and Physiology of Bacteria
Chapter 3 Sterilization and Disinfection
Chapter 4 Culture Media and Culture
Methods
Chapter 5 Bacterial Genetics, Resistance and
Susceptibility Testing
Introduction, History 1
and Microscopes
INTRODUCTION y This core is surrounded by a protein coat

y
y Viruses can reproduce only with the help of cellular environ-
General Microbiology

y
ment of other organisms (intracellular).
Medical microbiology is the study of microbial organisms that
infect humans. Based on structures, replication and pathogenesis Prokaryotes Bacteria and Archaea
microbes are divided into Bacteria, Virus, Fungi and Parasites. Eukaryotes Fungi, Protozoa and Algae

Types of Microorganisms SCIENTISTS AND THEIR DISCOVERIES


Bacteria
Louis Pasteur
y Bacteria are simple and unicellular organisms
y He is known as the father of microbiology.
y
y They come under prokaryotes
y
y
y All the bacteria are covered by cell walls which are composed y After Leeuwenhoek discovered the invisible world of
y
y
of proteoglycans microbes, the dispute about spontaneous generation theory
y They reproduce by a process called binary fission by which started.
y Pasteur conducted an experiment and disproved the theory
y
— bacteria divide into two equal cells
y
y Most of the bacteria derive food by photosynthesis (on its of spontaneous generation.
y He then proposed Germ theory of disease by stating that
y
own) and some obtain nutrition by inorganic substances
y
y Many bacteria have the capacity to mobilize with the help of invisible microbes in the air causes infection.
y
flagella. y He introduced many techniques in sterilization namely:
y
 Steam sterilizer

 Hot air oven
Archaea

 Autoclaving
y Prokaryotic cells

 Pasteurization of milk
y

y Do not have peptidoglycan y His contributions in vaccine designing are:
y
y
y They are classified into three main groups namely: Methano-  Anthrax
y

gens, Extreme halophiles and Extreme thermophiles.  Cholera in fowls

 Rabies
Fungi

y He introduced liquid media (nutrient broth) for the growth
y
y Belong to Eukaryotes of microbes.
y
y Cell walls are made up of chitin
y
y Most of the typical fungi grow as mold.
y
Protozoa
y They are unicellular, eukaryotic microbes
y
y They can move with the help of pseudopodia, flagellum or
y
cilia
y They can reproduce sexually or asexually.
y
Algae
y Algae are photosynthetic eukaryotes which have characteris-
y
tic presence of cellulose.

Viruses
y They are smaller than other microbes and acellular organisms Figure 1: Father of microbiology – Louis Pasteur
y
y Virus particle has a core that is made up either of DNA or RNA
y
Robert Koch Exception to Koch’s Postulates
Unit 1     General Microbiology

y He is known as father of medical microbiology


y
y Mycobacterium leprae – cannot be cultured in vitro
y

y His contributions in identification of microbial organisms are


y
y Treponema pallidum – cannot be cultured in vitro
y

as follows: y Neisseria gonorrhea – no animal model for experimental


y

 Solid media (Agar) for the growth of organisms



inoculation
 Methods to isolate organisms from pure culture

y Partially satisfied Koch’s postulates by Escherichia coli – it
y

 Hanging drop method to test the motility of an organism



showed bacterial pathogenicity in an in vitro model (Tissue
 Staining techniques

cultures) rather than in an animal model.
y He discovered the following organisms:
y
y Rickettsia spp and Chlamydia spp – It can be grown only in
y

 Anthrax bacilli

cell culture media and very difficult to grow in agar plates.
 TB bacilli
Remember


 Cholera bacilli


Koch's Phenomenon
• It is seen in tuberculosis.

• Koch observed that guinea pig that has already got infected

with TB bacillus will produce a hypersensitivity reaction


when it is injected with TB bacilli or protein – this is called
Koch’s phenomenon.

Organism Not satisfying the Organism Partially


Koch’s postulates satisfying the Koch’s
postulates
• Mycobacterium leprae
• • Escherichia coli

• Treponema pallidum

• Neisseria gonorrhoeae

Figure 2:  Father of medical microbiology – Robert Koch

Remember
• • Vaccination of Anthrax bacilli
– Louis Pasteur
• • Organism per se Anthrax bacilli – Robert Koch
• • Liquid media – Louis Pasteur
• • Solid media – Robert Koch

Koch Postulates
y Bacterium should be constantly associated with the lesions
y

of the disease
y It should be possible to isolate the bacterium in pure culture
y

from the lesions


y Inoculation of the pure culture into suitable lab animals
y

Figure 3: Father of aseptic surgery – Joseph Lister


should produce lesions of the disease
y It should be possible to reisolate the bacterium in pure culture
Remember
y

from the lesions produced in the lab animals


y Specific antibodies to the bacterium should be demonstrable
y
• • Father of Microbiology – Louis Pasteur
in the serum of the patients with the disease. • • Father of Medical Microbiology – Robert Koch
• • Father of Aseptic Surgery – Joseph Lister
Molecular Koch Postulates • • Father of Chemotherapy – Paul Ehrlich
y Additional postulates added to original Koch’s postulate by
y

Stanley Falkow. Table 1: Scientists and their contributions


y The phenotype or property under investigation should be


Scientists Contributions
y

significantly associated with pathogenic strains of a species


and not with non pathogenic strains. Louis Pasteur • Vaccine for rabies

y Specific inactivation of the gene or genes associated with


y
(Father of Microbiology) • Vaccine for anthrax

the suspected virulence trait should lead to a measurable • Technique of sterilization


decrease in pathogenicity or virulence • Disproved theory of spontaneous


y Reversion or replacement of the mutated gene with the wild-


y
generation
type gene should lead to restoration of pathogenicity or • Proposed germ theory of disease
4 virulence.

• Coined the term vaccine


contd...
Table 3:  Golden Age of Microbiology – Discoveries

Chapter 1     Introduction, History and Microscopes


Scientists Contributions
Robert Koch • • Mycobacterium tuberculosis Years Scientists Discoveries
(Father of Medical • • Vibrio cholera 1857 Pasteur Fermentation
Microbiology) • Staining techniques
1861 Pasteur Disproved spontaneous generation

• • Pure culture on solid media


• • Koch’s postulates 1864 Pasteur Pasteurization
• • Koch’s phenomenon 1867 Lister Aseptic surgery
Joseph Lister Aseptic precautions by using phenol 1879 Neisser Neisseria gonorrhea
(Father of aseptic for disinfection
1881 Koch Pure cultures
surgery)
1881 Finley Yellow fever
Paul Ehrlich • Father of chemotherapy

(Father of chemotherapy) • Standardization of toxin and



1882 Koch Mycobacterium tuberculosis, Agar
antitoxins 1883 Koch Vibrio cholerae
1884 Metchnikoff Phagocytosis
Anton Van Leeuwenhoek Microscope 1884 Hans Gram Gram staining
1884 Escherich Escherichia coli
Edward Jenner Small pox vaccine
1887 Petri Petri dish
Kary B Mullis PCR 1889 Kitasato Clostridium tetani
Barbara McClintock Transposons 1890 Von Bering Diphtheria antitoxin
Ernst Ruska Electron microscope 1890 Ehrlich Theory of immunity
Ignaz Semmelweis Handwashing 1898 Shiga Shigella dysenteriae

Karl Landsteiner Human blood groups 1908 Ehrlich Syphilis


1910 Chagas Trypanosoma cruzi
Niels Jerne Natural theory of antibody synthesis

Frank Burnet Clonal selection theory


MICROSCOPES
y Microbial organisms are tiny to see by naked eye; hence to
y

Alexander Fleming Penicillin – antibiotic (1929) visualize the microbes – we need microscope
y Leeuwenhoek was the person who first used microscope to
y

visualize bacteria
Nobel Prize Winners of 2015 y Limit of resolution (ability to distinguish two different
y

objects). Resolution is the ability of the lenses to distinguish


y William C Campbell and Satoshi Õmura for discovering novel
fine details and structure.
y

therapy against infections caused by roundworm parasites


 For unaided eye – 200 microns
y Tu Youyou with William C Campbell and Satoshi Õmura for


 Light microscope – 300 nm


y

discovering a novel therapy against malaria




 Electron microscope – 0.1 nm (or) 0.5 nm (according to




Table 2:  Name of a few organisms that have been kept Prescott)
after their discoverers y Refractive index is a measure of the light-bending ability of a
y

medium
Name after Discoverer Organism y Microscopes are classified based on the presence of number,
y

types of lenses, and light source as follows.


Klebs-Löffler bacillus Corynebacterium diphtheriae
Preisz-Nocard bacillus Corynebacterium Light Microscope
pseudotuberculosis
y Any type of microscope that uses visible light to observe
y

Koch-weeks bacillus Haemophilus aegyptius specimens is called as light microscope.


y A modern compound microscope has many lenses and light
y

Johne’s bacillus Mycobacterium paratuberculosis to have illumination.


y Finely grounded lenses are arranged in an order that when
y

Gaffky Eberth bacillus Salmonella Typhi light rays from an illuminator passes through the condenser,
Whitmore bacillus Burkholderia pseudomallei it gets passed through the specimen.
y From the specimen the light rays pass through the objective
Mycobacterium intracellulare
y

Battey bacillus lenses which is the lens that is closest to the specimen.
Eaton’s agent Mycoplasma pneumoniae y Then the image of the specimen gets magnified higher times
y

that is being visualised in ocular lens.


Pfeiffer’s bacillus Haemophilus influenzae 5
Magnification of Lens y Dyes used are: FITC, Auramine rhodamine, Acridine orange
Unit 1     General Microbiology

y The same principle can also be used for diagnostic techniques


Total magnification of lens is calculated by multiplying ocular
y

like:
lens magnification and objective lens magnification,
 Fluorescent antibody technique
i.e. Total magnification = Ocular lens magnifications × Objective


 Immunofluorescence
lens magnification


y Antibodies are tagged with dyes and this will help to bind with
y

Table 4:  Total magnification of a lens the antigen, which can be detected by the color produced.

Objective Lens Ocular Total Table 5:  Types of Microscopes


Lens Magnification
Microscopes Characteristics
Low power objective lens - 10X 10X 100 X
Light • With the help of fixation and staining –

High power objective lens - 45X 10X 450 X microscope bacteria is visualized under compound
Oil immersion lens - 100X 10X 1000 X lenses
• Universally used microscopy to study

bacteria
Types of Microscopes • To visualize:

ƒ Gram staining
Bright Field Microscopes
ƒ

ƒ Acid fast staining


ƒ

Under the routine conditions, the field of vision in a compound ƒ Motility by hanging drop preparation
ƒ

ƒ Slide agglutination test


light microscope is brightly illuminated. By focusing the light, ƒ

ƒ Slide culture for fungi


the condenser produces a brightfield illumination — termed as
ƒ

ƒ Capsule staining
Bright Field Microscope.
ƒ

Phase contrast • Principle used is structures within the cells


microscope differ in thickness or refractive index;


Dark-field Microscope • This phase difference are converted into

y A dark field microscope is used for examining live micro-


y
differences in intensity of light, which
organisms that are: produces light and dark contrasts in the
 Not visible in ordinary light microscope

images
 Cannot be stained by ordinary methods • Mainly used for:

ƒ Studying microbial motility




 Or its morphology gets distorted by staining methods


ƒ

ƒ Shape of living cells




y The only difference between bright field and dark field ƒ

ƒ To detect endospores
y

microscope is the condenser.


ƒ

ƒ To see inclusion bodies


y In this condenser, the light that is reflected off, i.e. it gets
ƒ

ƒ To study eukaryotic cells


ƒ

turned away from the specimen that enters the objective lens.
y This technique helps to examine the unstained organisms in Fluorescent • By using fluorescent dyes tagged to the

microscope target bacteria – diagnosis is done;


y

liquid
y Mainly helpful in identification of Treponemes. • Immunofluorescence is a method where

dye is conjugated with antibody – this


y

Phase Contrast Microscope reacts with antigen and produces color


y The principle of this type is based on waving nature of light • Dyes mainly used are:

ƒ Acridine orange
y

rays. ƒ

ƒ Fluorescein isothiocyanate (FITC)


y Because of waving pattern we have ups and downs in light ƒ

ƒ DAPI
y

rays that gives a phase difference. ƒ

ƒ Rhodamine
y When two sets of light rays are brought together this forms
y
ƒ

an image of the specimen on the ocular lens based on phase Dark-field • Very slender organisms like spirochetes

differences between the two lights. microscope that cannot be seen in light microscope
y This will give a contrast where the internal structures of a
y

needs dark field;


cell can be seen more sharply defined. • The difference between light-and dark-field

microscope is a presence of condenser


Fluorescence Microscope (dark-field stop underneath the condenser)
y The ability of a substance to absorb shorter wavelength of
y
• This dark-field stop will produce a cone

light and to emit longer wavelength is called as fluorescence. of light so that only the light entering the
y Some organism has in built nature for fluoresce and called as
y
objective comes from the specimen alone
autofluorescence. making the surrounding dark
y The dyes that are used for visualising organisms is called as
y
• Helps to see the motility of spirochaetes

Fluorochromes.
contd...
6
Chapter 1     Introduction, History and Microscopes
Microscopes Characteristics
Inverted • Helps in cell culture

microscope
Electron • Instead of light, a beam of electrons is

microscope used;
• Organism is freezed (Freeze etching) –

cellular ultrastructure is studied;


• Mainly for direct visualization of virus from

specimens; Eg.: Rota virus from stools


• Techniques used in electron microscopy

are:
ƒ Shadow casting
ƒ

ƒ Negative staining
ƒ

ƒ Freeze etching
ƒ

Interference • Helps in quantitative measurement of


microscope chemical constituents of cells (lipids,


proteins, nucleic acids)
• Differential interference microscope gives a

three-dimensional image
Polarization • To study the intracellular structures using

microscope differences in birefringence


Confocal • Gives three-dimensional image that is

microscope studied through a computer connected Figure 4: Inverted microscope (Courtesy: CDC/ Dr. Paul Fernhoff)
with microscopy
• Mainly helps in study of biofilms that

is attached with surfaces like joint


replacements

Remember
Light Microscope Phase Contrast Microscope
To visualize: Mainly used for:
• Gram staining
• • Studying microbial motility

• Acid fast staining


• • Shape of living cells

• Motility by hanging drop preparation


• • To detect endospores

• Slide agglutination test


• • To see inclusion bodies

• Slide culture for fungi


• • To study eukaryotic cells

• Capsule staining

Dark Field Microscope Electron Microscope


Helps to see the motility of spirochetes Mainly for direct visualization of virus from specimens; Eg: Rota
virus from stools; Instead of light, a beam of electrons is used;
Polarization Microscope Fluorescent Microscope
To study the intracellular structures using differences in Dyes mainly used are:
birefringence • Acridine orange

• Fluorescein isothiocyanate (FITC)


• DAPI

• Rhodamine

7
MULTIPLE CHOICE QUESTIONS
Unit 1     General Microbiology

History 12. Side chain theory for antibody production is proposed


by (Recent Pattern 2017)
1. Germ theory of disease was proposed by:
a. Robert Koch b. Paul Ehrlich
(Recent Pattern Nov15)
c. Elie Metchnikoff d. Louis Pasteur


a. Louis Pasteur b. James Lind


13. Medium of travel in electron microscope
c. Aristotle d. Pattenkoffer
2. Technique of sterilization was introduced by:  (Recent Pattern 2017)
 (Recent Pattern July 15) a. Air b. Water
a. Robert Koch b. Edward Jenner c. High vacuum d. Oil
c. Louis Pasteur d. Lister 14. Which method of diagnosis is shown here? 
3. Mycobacterium tuberculosis was discovered by:  (AIIMS May 2018)
 (Recent Pattern Dec 13)
a. Louis Pasteur b. Robert Koch
c. Lister d. Jenner
4. Microscope was invented by.
 (Recent Pattern Aug 13)
a. Ronald Ross
b. Robert Koch
c. Antonie van Leeuwenhoek
d. Louis Pasteur
5. Who discovered electron microscope?
a. Robert Koch b. Paul Ehrlich
c. Elie Metchnikoff d. Ernst Ruska

Microscopes a. Dark ground microscopy


6. Arrangement of lens from eye to source of light, in light b. Phase contrast microscopy
microscope: (Recent Pattern Dec 13) c. Flourescent microscopy
a. Ocular lens: objective lens: condenser lens d. Electron microscopy
b. Subjective lens: ocular lens: condenser lens 15. Identify the condenser in the following microscope
c. Condenser lens: objective lens: ocular lens image:  (AIIMS May 2018)
d. Subjective lens: condenser lens: ocular lens
7. Light microscopy resolution: (PGI May 12)
a. 200 nm b. 20 nm
c. 0.2 nm d. 300 nm
8. Dark ground microscopy is used to see:
a. Refractile organism  (Recent Pattern Dec 14)
b. Flagella
c. Capsule
d. Fimbriae
9. Dye used for direct immunofluorescence: 
 (Recent Pattern Dec15)
a. India ink b. Nigrosine
c. Rhodamine d. Basic fuschin
10. Shadow casting is used in: (Recent Pattern Dec 16)
a. Light microscopy
b. Electron microscopy
c. Optical microscopy
d. Fluorescence microscopy
11. Bifringence polarization microscopy is used for: a. Above the stage
 (Recent Pattern Dec 15) b. Below the stage
a. Flagella b. Intracellular structures c. Near the eyepiece
c. Capsule d. Spores d. Above the objective lenses

8
ANSWERS AND EXPLANATIONS

Chapter 1     Introduction, History and Microscopes


1. Ans.  (a) Louis Pasteur 8. Ans.  (b) Flagella
Ref: Ananthanarayan and Paniker’s Textbook of Microbiol- Ref: Ananthanarayan and Paniker’s Textbook of Microbiol-
ogy,10th ed, Page 3 ogy – 10th ed – Page 11
• Louis Pasteur – Disproved the theory of spontaneous

• Dark-field microscope is used to see very slender


generation and postulated germ theory of disease. organisms like spirochaetes and to see the flagella
• While fimbriae can be demonstrated by hemagglutina-
2. Ans.  (c) Louis Pasteur

tion
Ref: Ananthanarayan and Paniker’s Textbook of Microbiol-
ogy – 10th ed – Page 3 9. Ans.  (c) Rhodamine
• Pasteur is the one who introduced techniques of

Ref: Ananthanarayan and Paniker’s Textbook of Microbiol-


sterilisation like steam steriliser, hot air oven and ogy – 10th ed – Page 11
autoclave
• Fluorescent dyes used in immunofluorescence are

3. Ans.  (b) Robert Koch ƒ Auramine


ƒ

ƒ Rhodamine
Ref: Ananthanarayan and Paniker’s Textbook of Microbiol-
ƒ

ogy – 10th ed – Page 4 ƒ Lissamine


ƒ

ƒ FITC (M/c used) – Fluorescein isothiocyanate


• Robert Koch discovered Mycobacterium tuberculosis
ƒ

and Vibrio cholerae 10. Ans.  (b) Electron microscopy


4. Ans.  (c) Antonie van Leeuwenhoek Ref: Ananthanarayan and Paniker’s Textbook of Microbiol-
Ref: Ananthanarayan and Paniker’s Textbook of Microbiol- ogy – 10th ed – Page 12
ogy – 10th ed – Page 3 • To visualize the organism under electron microscope –

• Antonie Van Leeuwenhock – first time observed and



two types of techniques are followed
reported bacteria with his simple hand made microscope ƒ Shadow casting
ƒ

ƒ Negative staining with phosphotungstic acid


ƒ

5. Ans.  (d) Ernst Ruska


11. Ans.  (b) Intracellular structures
Ref: Ananthanarayan and Paniker’s Textbook of Microbiol-
ogy – 10th ed – Page 4 Ref: Ananthanarayan and Paniker’s Textbook of Microbiol-
• Ernst Ruska in 1934 developed the electron microscope

ogy – 10th ed – Page 12
– to visualise viruses • Polarization microscope: To study the intracellular

structures using differences in birefringence


6. Ans.  (a) Ocular lens: objective lens: condenser lens
Ref: Ananthanarayan and Paniker’s Textbook of Microbiol- 12. Ans.  (b) Paul Ehrlich
ogy – 10th ed – Page 10
Ref: Essentials of Medical Microbiology – Apurba Sastry –
• Arrangement of lenses in bright field or light microscope

Page 3
is ocular lens in eye piece which is of 5X or 10X
• Objective lens which has low power (10X) , high power

• Father of microbiology → Louis Pasteur

(45X) and oil immersion (100X) lenses • Germ theory of disease, Nutrient broth, pasteurization

• Condenser lens is located in the diaphram which helps



of milk, autoclaving, hot air oven and steam sterilizer
in focussing of lights from light rays Louis Pasteur
• Ocular lens → Objective lenses → Condenser lens
• • Father of antiseptic surgery → Joseph Lister

• Introduction of solid media, pure culture techniques,


7. Ans.  (d) 300 nm anthrax bacilli, tubercle bacilli, cholera bacilli, Koch's
Ref: Ananthanarayan and Paniker’s Textbook of Microbiol- phenomenon → Robert Koch
ogy – 10th ed – Page 11 • Father of chemotherapy → Paul Ehrlich

• Resolving power of a microscope is the ability to



• Acid fast nature of tubercle bacillus, toxin-antitoxin

differentiate two different objects as different interaction called as Ehrlich phenomenon, side chain
• It is limited by the wavelength of the light

theory for antibody production
• Limit of resolution of a light microscope is 300 nm
• • Phagocytosis → Elie Metchnikoff

9
13. Ans.  (c) High vacuum 15. Ans.  (b) Below the stage
Unit 1     General Microbiology

Ref: Essentials of Medical Microbiology – Apurba Sastry – Ref: Ananthanarayan and Paniker’s Textbook of Microbiol-
Page 12 ogy – 10th ed – Page 10
• In light microscope the medium of travel is air, here it is

high vacuum

Features Light Electron


microscope microscope
Magnification 1000 – 1500 100,000
Best resolution 0.2 um 0.5 nm
Radiation source Visible light Electron beam
Medium travel Air High vacuum
Specimen mount Glass slide Metal grid
Type of lens Glass Electromagnet

14. Ans.  (a) Dark ground microscopy


Ref: Ananthanarayan and Paniker’s Textbook of
Microbiology – 10th ed – Page 211 

• The above image has a dark background with refractile

• A modern compound microscope has many lenses and

organism – clearly states that it is an image of dark light to have illumination


ground microscopy • Finely grounded lenses are arranged in an order that

• Wet films are prepared from the exudates and it is



when light rays from an illuminator pass through the
examined under dark ground illumination. condenser, it gets passed through the specimen.
• T. pallidum is identified by its slender spiral structure

• From the specimen the light rays passes through the

and its slow motility objective lenses which is the lens that is closest to the
specimen.
• Then the image of the specimen gets magnified higher

times that is being visualised in ocular lens.


• Ocular lens → Objective lenses → Condenser lens

10
Morphology and 2
Physiology of Bacteria

A B

Figures 1A and B: Eukaryotic and Prokaryotic cells

Table 1:  Difference between Prokaryotic and Eukaryotic cells


Characteristics Prokaryote Eukaryote
Chromosome Circular, one Linear, many
All cellular organelles Absent Present
Chemical nature Muramic acid Sterols
Nuclear membrane and nucleolus Absent Present

y Bacteria are prokaryotes y Demonstration of cell wall is by:


y
y
y They do not have all the cell organelles to carry out life process  Plasmolysis
y

y Divide by binary fission  Microdissection
y

y Contain both DNA and RNA  Reaction with specific antibodies
y

y Even Chlamydia and Rickettsiae come under prokaryotes  Mechanical rupture of cell
y

y Mitochondria is absent, hence the respiratory enzymes  Different staining methods
y

are not located in mitochondria in bacteria; they are in  Electron microscopy

mesosomes (Chondroids)
Table 2:  Characteristic of cell wall in Gram-positive and
y Principal sites of respiratory enzymes in bacteria— Meso-
Gram-negative Bacteria
y
somes
Cell Wall Gram-Positive Gram-Negative
Cell Wall Thickness Thicker Thinner
y Cell wall— gives shape and rigidity of the cell Amino acids Few Many
y
y Cell wall is made up of mucopeptide (peptidoglycan)—
Lipopolysaccharides Absent Present
y
formed by N-acetyl glucosamine and N-acetyl muramic acid
(LPS)
molecules — linked with peptide pentaglycine bridges
Teichoic acid Present Absent
y LPS – endotoxin nature
Unit 1     General Microbiology
y
y Protoplasts: When a Gram-positive bacterium is kept in hypertonic solution, it becomes protoplast (Remember as P-P)
y
y Spheroplasts: When a Gram-negative bacterium is lysed by lysozyme it becomes spheroplasts
y
y Sterols are present in the cell membrane in Mycoplasma
y
Figure 2: Gram-positive and Gram-negative cell wall
Cell Wall Deficient Forms
y Β1 → 4 linkage of the peptidoglycan backbone is hydrolyzed
y
by the enzyme lysozyme – causes cell wall deficiency
y L forms are formed in certain species of bacteria spontaneously
y
or because of cell wall attacking drugs (Penicillin group)
y L forms are produced more readily with penicillin than with
y
lysozyme, suggesting the need for residual peptidoglycan.
y L forms revert back to normal cell when we remove the
y
stimulus
y Mycoplasma are cell wall deficient bacteria that have ste-
y
rols

The difference between L forms and mycoplasmas is that


when the murein is allowed to reform:
• L forms revert to their original bacteria shape

• But mycoplasmas never do

Gram Staining
Hans Christian Gram (1884) devised a method of staining for
the following purposes:
y To differentiate Gram-positive and Gram-negative organisms
y
y To identify the organism
y
y To see the size, shape and arrangement of the microbe.
y
Procedure
y Primary staining: Pararosaniline dyes, Crystal violet, Methyl
y
violet, Gentian violet
y Mordant: Iodine
y
y Decolorisation: Alcohol, Acid-alcohol, Aniline, Acetone
12 Figure 3: Procedure of Gram staining
y
y Counter staining: Carbol fuchsin, safranine and neutral red
y
Chapter 2     Morphology and Physiology of Bacteria
Remember
Gram Staining
• Gram positive looks violet

• Gram negative looks pink

• Only fungus that is visualized by Gram staining is Candida

(Yeast cells)
• Gram staining is the most important method of diagnosis in

Neisseria gonorrhea (Intracellular diplococci)
• Gram staining helps to differentiate pathogenic and

commensal bacteria (by looking at the number of pus cells)

Table 3: Characteristics of bacteria Figure 5: Gram-negative organism—pink color



Gram-positive cocci in grape-like clusters Staphylococci (Courtesy: CDC/ Dr. W.A. Clark)
Gram-positive cocci in chains Streptococci
Gram-positive cocci in capsules Pneumococci
Remember
Organisms which show bipolar staining or safety pin appear-
Gram-negative diplococci, adjacent sides Neisseria ance are:
flattened • Calymmatobacterium granulomatis


Gram-positive, club-shaped pleomorphic Corynebacteria • Burkholderia mallei


rods • Burkholderia pseudomallei

• Vibrio parahemolyticus
Gram-negative rods with pointed ends Fusobacteria

• Yersinia pestis
Gram-negative curved rods Comma-shaped

• Haemophilus ducreyi
vibrios

Gram-negative straight rods with rounded Enterobacteriaceae Cell Wall of Mycobacteria
ends
y Cell wall is different from that of other bacteria, hence it
Spiral rods Spirilla
y
cannot be seen in Gram staining
Gram-negative curved rods Helicobacter y It has high concentration of mycolic acid which are lipids
y
y Acid-fast organism: because of their resistance to decoloriza-
y
Remember tion with acid-alcohol

Bacteria that cannot be seen in Gram staining: Ziehl-Neelsen Staining or Acid Fast Staining
(Mneumonic: Tiny Rascals Lack Colour in Microscope) y This method is mainly used to visualize the acid fast organisms.
• Treponema pallidum
y
Organism is seen in pink color with a blue background.

• Rickettsiae
Procedure

• Legionella pneumophila

• Chlamydiae y Strong carbol fuchsin with heat – primary stain

• Mycobacteria
y
y 20% sulphuric acid – decolorization (Standard method)

• Mycoplasma pneumonia
y
y RNTCP suggests 25% sulphuric acid

y
y Methylene blue – Counter stain
y
Figure 6: Sputum sample with acid-fast bacilli seen inside an
Figure 4: Gram-positive organism—violet color epithelial cell (Pink colored) – Mycobacterium tuberculosis 13
(Courtesy: CDC/ Dr. Richard Facklam) (Courtesy: CDC/ Ronald W Smithwick)
Table 4:  How will you differentiate acid-fast bacilli? Haemophilus influenzae
Unit 1     General Microbiology


Bacillus anthracis

Acid fast organisms Percentage of H2SO4


 Clostridium perfringens


Mycobacterium tuberculosis 20% (Standard method), 25%  Cryptococcus (fungi)


in RNTCP  Yersinia pestis


Mycobacterium leprae 5% (less acid fast) y Demonstration of capsule is by:

y
 Negative staining
Nocardia 0.5%


 Quellung reaction in Pneumococcus
Oocysts of coccidian parasites 0.25 to 0.5%


 Serological methods
Bacterial spores 1%


y Capsule cannot be seen in Gram staining because it has less
Eggs of Taenia saginata

y
affinity for basic dyes
Rhodococcus
Legionella micdadei
• When the layer is well organized and not easily washed off, it


is called a capsule.
Capsule • A slime layer is a zone of diffuse, unorganized material that is


y It is a protective covering seen in few bacteria removed easily.
y
y Made up of polysaccharide except Bacillus anthracis – • A glycocalyx is a network of polysaccharides extending from


y
made up of polypeptide the surface of bacteria and other cells.
y Capsulated organisms:
y
(Mneumonic: Some Nasty Killers Have Big Capsule Protec-
Flagella

tion)
 Streptococcus pneumoniae y Fimbriae: Organ of adhesion
y

 Neisseria meningitidis y Flagella: Organ of locomotion
y

 Klebsiella spp y Types of flagella: (They are given in figure below)

y
Figure 7: Different types of flagella

14
Table 5: Types of flagella

Chapter 2     Morphology and Physiology of Bacteria



Remember
Numbers of flagella present Examples Organisms that are motile at 25°C but non-motile at 37°C:
Atrichous bacteria – no flagella Lactobacillus • Listeria monocytogenes


• Yersinia pseudotuberculosis
Monotrichous – single flagella Vibrio cholera, Pseudomonas,


• Yersinia enterocolitica
at one side Campylobacter


Lophotrichous – Tufts of Spirilla, Bartonella
flagella at one side bacilliformis Pili and Fimbriae
y Many Gram-negative bacteria have short, fine, hair-like
Amphitrichous – Flagella at Alcaligenes faecalis

y
appendages that are thinner than flagella and not involved in
both ends
motility; called as fimbriae
Peritrichous – Flagella all over E. coli, Salmonella, Proteus y Sex pili are genetically determined by sex factors or

y
the cell conjugative plasmids and are required for bacterial mating
Amphi lophotrichous – Tufts of Spirillum serpens y Pili are basically organ of attachment

y
flagella at both sides
Remember
Demonstration of Flagella • Organ of locomotion — Flagella


• Organ of attachment — Pili
y Flagella cannot be seen under light microscope


y
y Only motility of the bacilli is seen in microscope by hanging
Spores
y
drop preparation
y Demonstration of flagella is done by Leifson method and Ryu Few bacteria produce spores which are nothing but highly
y
method resistant forms of bacteria which gets formed under nutritionally
y Seen under dark ground illumination deficient conditions
y
y Special staining like silver staining
y
y Electron microscopy Remember
y
y U tube method, Cragie tube method in semisolid medium
• Aerobic spore-bearing organism: Bacillus
y

• Anaerobic spore-bearing organism: Clostridium
Three Parts of Flagella

There are three parts: Demonstration of Spores
y Filament, cell surface to the tip. Filament is made of protein
y Modification of Ziehl-Neelsen technique with 0.25% to 0.5%
y
called Flagellin.
y
H2SO4
y A basal body is embedded in the cell
y In Gram staining – spores look as unstained areas
y
y A short, curved segment, the hook, links the filament to its
y
y Schaeffer and Fulton’s method of spore staining – using
y
basal body and acts as a flexible coupling.
y
malachite green

Table 6: Types of motility



Name of motility Examples
Actively motile Pseudomonas
Lashing motility Borrelia
Darting motility Vibrio cholerae
Tumbling motility Listeria monocytogenes
Falling leaf like motility Giardia lamblia
Football motility Balantidium coli
Serpentine motility Salmonella
Gliding motility Mycoplasma Figure 8: Spores in Gram staining (look at the unstained area in
Swarming Proteus, Clostridium the bacilli – they are the spores)
(Courtesy: CDC/ Dr. William A. Clark)
Corkscrew type Treponema
Flexion, and extension Leptospira

15
Table 8:  Morphological forms of bacteria
Unit 1     General Microbiology

High Yield
Bacterial forms Features
Albert Staining
• It is the staining method used for metachromatic granules of Cocci Occur in clusters – E.g. Staph.aureus
Occur in chains – E.g. S. pyogenes

Corynebacterium diphtheriae.
• Albert stain has: Occur in pairs (diplococci) – E.g. Neisseria
Occur in tetrads – E.g. Micrococci

ƒ Malachite green
ƒ
ƒ Toluidine blue
Straight rods Uniform thickness, rounded ends, pointed
ƒ
ƒ Iodine and potassium iodide
ends, club form
ƒ
• Body of bacilli stains green with the metachromatic granules
E.g.: Enterobacteriaceae

looking bluish black at the ends
Curved rods Comma shaped, spiral, screw shaped
E.g. Vibrio, Spirochaetes
Inclusion Bodies
Mycoplasmas Bacteria without a rigid cell wall; coccoid
y Inclusion bodies are granules of organic or inorganic
cells, long threads
y
material that often are clearly visible in a light microscope.
y These bodies usually are used for storage of carbon Chlamydiae Two forms: spherical /oval elementary
y
compounds, inorganic substances and energy – hence they bodies;spherical/oval reticular bodies
are also called as storage granules. Rickettsiae Short coccoid rods
y Organic inclusion bodies usually contain either glycogen or
y
poly- β-hydroxybutyrate.
y Examples are: PHYSIOLOGY OF BACTERIA
y
 Poly-β-hydroxybutyrate granules y Generation time for bacteria – time between two cell divisions
y

 Glycogen and sulfur granules or the time required from a bacterium to give rise to two

 Carboxysomes daughter cells (20-20-20)

 Gas vacuoles.  E. coli – 20 minutes


 TB – 20 hours
Intracytoplasmic Inclusions

 Lepra bacilli – 20 days

Volutin or metachromatic granules are common in: y Bacterial growth curve
y
(Mneumonic: Cellphone-GSM)  When organisms are cultured from another broth to new

y Corynebacterium diphtheria liquid medium, the period that is taking to enter into new
y
y Gardenella vaginalis culture medium and replicates can be drawn phase by
y
y Mycobacteria phase termed as bacterial growth curve.
y
y Spirillum volutans
y
Table 7:  Staining methods and their uses
Staining methods Uses
Gram staining Universal staining method for
most bacteria and yeast cells
Ziehl-Neelsen staining Mycobacterium species
Modified ZN staining Spores, Coccidian oocysts,
Nocardia
Albert staining Corynebacterium diphtheria
Neisser’s staining granules
Ponders staining
Schaeffer and Fulton’s method Spores
Negative staining Capsule demonstration –
Indian ink staining Cryptococcus
Figure 9: Bacterial growth curve
Fontana staining Leptospirosis, Treponemes
y Lag phase: When microorganisms are introduced into fresh
Silver impregnation staining Treponemes
y
culture medium, there will be no immediate increase in cell
Warthin-Starry silver staining Helicobacter pylori number occurs, and therefore this period is called the lag
Giemsa stain Inclusion bodies of Chlamydia phase
trachomatis y Log phase: During the exponential or log phase,
y
microorganisms are growing and dividing at the maximal
Periodic acid schiff stain, HPE for fungi
16 Gomori methenamine stain
rate possible given their genetic potential, the nature of the
medium, and the conditions under which they are growing.
Toluidine blue staining Pneumocystis jirovecii Their rate of growth is constant during the exponential phase;
y Stationary phase: Eventually population growth ceases and y Obligate aerobes: Grow only in the presence of oxygen

Chapter 2     Morphology and Physiology of Bacteria


y
y
the growth curve becomes horizontal. This is to limitation E.g. Pseudomonas, Mycobacterium tuberculosis, Bacillus,


of nutrition and organisms go to starvation. At this phase, Brucella and Nocardia
sporulation starts. y Facultative anaerobes: Aerobes that can grow anaerobically

y
y Phase of decline: Nutrient deprivation and the buildup of also
y
toxic wastes lead to the decline in the number of viable cells is E.g. E. coli and S. aureus
characteristic of the death phase.


y Facultative aerobes: Anaerobes that can also grow aerobically

y
Table 9: Bacterial growth phase E.g. Lactobacillus



y Obligate anaerobes: Grow only in the absence of oxygen
Growth phase Events

y
E.g. Clostridium


Lag phase Cell size is maximum y Aero tolerant anaerobes: Tolerate oxygen for some time, but

y
Log phase Cell size is smaller – Staining is uniform do not use it, e.g. Cl. histolyticum
Stationary phase Sporulation occurs – Staining is not uniform;
Metabolic products like exotoxins are Table 11: Classification of bacteria based on oxygen


produced requirement
Phase of decline Involution forms occurs; Death of cells (due
Obligate aerobes Pseudomonas, Mycobacterium
to autolytic enzymes)
tuberculosis, Bacillus, Brucella and
Nocardia
Nutritional Requirement for Microbes Obligate anaerobes Clostridium
y Microbial cell composition shows that over 95% of cell dry
y
weight is made up of carbon, oxygen, hydrogen, nitrogen, Facultative aerobes Lactobacillus
sulfur, phosphorus, potassium, calcium, magnesium and
Facultative anaerobes E. coli, S. aureus
iron. These are called macroelements or macronutrients.
y Certain micronutrients like manganese, zinc, cobalt, Aero tolerant anaerobes Clostridium histolyticum
y
molybdenum, nickel and copper are also needed by most
cells. Microaerophilic Campylobacter Helicobacter sp.,

Capnophilic (CO2) Brucella


Table 10: Classification of microbes based on modes of

sources of energy
Source Classification Definition Based on Temperature Requirement
Carbon Autotrophs Only autotrophs can use CO2 as y Growth below 20°C: Psychrophiles; E.g. Saprophytes
y
sources their sole or principal source of y Growth between 25°C to 40°C: Mesophiles E.g. Most of the
y
carbon. pathogenic bacteria
y Growth above 55°C–80°C: Thermophiles E.g. Bacillus
Heterotrophs Organisms that use reduced,
y
stearothermophilus
preformed organic molecules as
carbon sources are heterotrophs Table 12: Classification of bacteria based on tempera-

Energy Phototrophs Phototrophs use light as their ture requirement
sources energy source
Chemotrophs Chemotrophs obtain energy
from the oxidation of chemical
compounds (either organic or
inorganic)
Electron Lithotrophs Lithotrophs (“rock-eaters”) use
sources reduced inorganic substances as
their electron source
Organotrophs Organotrophs extract electrons
from organic compounds

CLASSIFICATION
Based on Oxygen Requirement
y Microaerophilic: Grows in the presence of low oxygen
17
y
tension (5–10%)
E.g. Campylobacter and Helicobacter sp.,

BACTERIOCINS y Many medical conditions including indwelling medical
Unit 1     General Microbiology

y
devices, dental plaque, peritonitis, urogenital infections
y Group of highly specific antibiotic like substances which are and upper respiratory tract infections are associated with
y
produced by certain strains of bacteria active against other formation of biofilms.
strains which belong to same or different species y Bacteria commonly involved include Enterococcus faeca-

y
lis, Staphylococcus aureus, Staphylococcus epidermidis,
Table 13: Bacteriocins produced by certain strains of Streptococcus viridans, Escherichia coli, Klebsiella pneu-

bacteria moniae, Proteus mirabilis and Pseudomonas aeruginosa.

Colicin Escherichia coli Remember


Pyocin Pseudomonas aeruginosa Biofilm Producing Bacteria
• CONS – Staphylococcus epidermidis (m/c)


Megacin Bacillus megaterium • Enterococcus faecalis


• Streptococcus viridans
Diphthericins Coryne.diptheriae


• E. coli


• K. pneumonia


• Proteus mirabilis
BIOFILM PRODUCTION


• Pseudomonas aeruginosa


y Biofilms are microbially derived sessile communities which
y
are characterized by the cells that are attached to a substratum y The nature of the biofilm makes a barrier to eradication and

y
in an irreversible form. confers high level of resistance to antimicrobial agents.
y They produce a matrix of extracellular polymeric substances y So the microorganisms growing in a biofilm exhibits
y
y
(EPS) and are embedded in the matrix exhibiting an altered intrinsically more resistant to antimicrobial agents.
phenotype with respect to growth rate and gene transcription. y Prevention of CoNS infection has largely concentrated on
y
y Biofilm mechanism is basically some chemotactic particles prevention of indwelling catheter associated infection due
y
or pheromones which make the bacteria communicate with to biofilms.
each other within the biofilm, a phenomenon called quorum y Catheters should be inserted with meticulous attention to
y
sensing. aseptic practices. Staff should adhere to appropriate aseptic
y Factors influencing biofilm formation: protocols in caring out the indwelling catheterization.
y
 Availability of key nutrients y Prophylactic antibiotics will help in slightly slow progression

y
 Surface chemotaxis of biofilms in biomaterials.

 Bacterial motility

 Surface adhesins

18
MULTIPLE CHOICE QUESTIONS

Chapter 2     Morphology and Physiology of Bacteria


1. Eukaryotes are different in causing infection because 12. Not a component of gram stain: (Recent Pattern Dec 12)
(Recent Pattern Dec 12) a. Methylene blue

a. Divide by binary fission b. Ethanol
b. Highly structured cell with organized cell organelles c. Iodine
c. Don’t have all organelles d. Gentian violet
d. Evolutionally ancient 13. Gentian violet colouration of gram positive bacteria is
2. Which is eukaryote: (PGI Nov 10, May 15) due to: (Recent Pattern Nov 15)
a. Mycoplasma a. Peptidoglycan b. Capsule
b. Bacteria c. Cell membrane d. None of the above
c. Fungus 14. Which of the following are AFB positive with 5%
d. Chlamydia sulphuric acid: (PGI May 15)
3. True about bacteria: (PGI May 15) a. M. avium
a. Mitochondria always absent b. M. leprae
b. Sterols always present in cell wall c. M. tuberculosis
c. Divide by binary fission d. Nocardia
d. Can be seen only under electron microscope e. Rhodopus
4. Which of the following organism contain both DNA and 15. Modified Ziehl-Neelsen staining is used for:
RNA  (AIIMS May 14) a. M. leprae  (Recent Pattern Dec 16)
a. Bacteria b. Plasmid b. M. bovis
c. Prion d. Viroid c. Nocardia
5. Porin present in: (Recent Pattern July 15) d. All of the above
a. Cell wall of gram positive bacteria 16. Decolourization in ZN staining is done by:
b. Cell membrane of gram positive bacteria a. 1% sulphuric acid  (Recent Pattern Jun 14)
c. Cell wall of gram negative bacteria b. 5% sulphuric acid
d. Outer membrane of gram negative bacteria c. 10% sulphuric acid
6. Wet India ink preparation is used to demonstrate: d. 20% sulphuric acid
(Recent Pattern 10) 17. The difference between gram +ve and gram –ve

a. Flagella b. Capsule organism is that gram +ve organism contains:
c. Spore d. Fimbriae (Recent Pattern July 15)

7. Silver impregnation technique is used in the identifica- a. Teichoic acid
tion of: (Recent Pattern Aug 13) b. Muramic acid
a. Spirochetes b. Leptospira c. N-acetyl neuraminic acid
c. Borrelia d. All of the above d. Aromatic amino acids
8. All organism shows bipolar staining, except: 18. Periplasmic space is seen in: (Recent Pattern Dec 15)
(Recent Pattern Dec 13) a. Gram positive bacteria

a. Calymmatobacterium granulomatis b. Gram negative bacteria
b. Y. pestis c. Acid fast bacteria
c. Pseudomonas mallei d. All
d. H. influenzae 19. Mesosomes in bacteria are functional unit for
9. Bacteria with safety pin appearance: (Recent Pattern Dec 14)

(Recent Pattern Dec 16) a. Lipid storage
b.

a. H. influenzae Protein synthesis
b. Vibrio parahemolyticus c. Respiratory enzymes
c. Vibrio vulnificus d. None
d. Salmonella paratyphi 20. Which of the following is not capsulated:
10. Safety pin appearance of bacteria is seen in: (Recent Pattern Dec 13)

(Recent Pattern Aug 13) a. Pneumococcus b. Cryptococcus
c.

a. Vibrio cholerae b. Chlamydia Meningococcus d. Proteus
c. H influenza d. Y.pestis 21. Metachromatic granules are found in:
11. Correct order of gram staining is: (Recent Pattern Dec 14)

(Recent Pattern Dec 12) a. Diphtheria b. Mycoplasma
c. Gardenella vaginalis d.

a. Gentian violet—Iodine – carbol fuchsin Staphylococcus
b. Iodine – Gentian violet – carbol fuchsin 22. Metachromatic granules are stained by:
c. Carbol fuchsin – Iodine – Gentian violet (Recent Pattern Dec 14)

d. Carbol fuchsin – Gentian violet – Iodine a. Ponder’s stain b. Negative stain
c. Gram’s stain d. Leishman stain 19
23. Albert’s stain is used for: (Recent Pattern Dec 13) 37. Generation time for Mycobacterium tuberculosis:
Unit 1     General Microbiology

a. Staphylococcus a. 20 minutes b. 14 -16 hours


b. Corynebacterium diphtheriae c. 12 – 13 days d. 20 days
c. C. perfringens 38. Capsulated organisms are all except:
d. C. tetani a. Bacillus anthracis
24. Peritrichous flagella are seen in(Recent Pattern Dec 15) b. Cryptococcus neoformans
a. Vibrio cholera b. Proteus c. Klebsiella pneumonia
c. Campylobacter d. Legionella d. Proteus mirabilis
25. Non-motile organism: (Recent Pattern Nov 13) 39. Identify the following staining method:
a. Escherichia coli b. Vibrio cholera
c. Proteus mirabilis d. Shigella dysenteriae
26. Cell wall deficient organism are:(Recent Pattern Dec 13)
a. Chlamydia b. Mycoplasma
c. Streptococcus d. Anaerobes
27. Maximum cell size in bacterial cell growth cycle:
(Recent Pattern 15)

a. Lag phase
b. Log phase
c. End of plateau phase
d. Early stage of decline
28. True regarding lag phase is: (Recent Pattern Aug 13)
a. Time taken to adapt in the new environment a. Spore staining b. Albert staining
b. Growth occurs exponentially c. Acid fast staining d. Sudan black lipid staining
c. The plateau in lag phase is due to cell death 40. Whitmore bacillus is
d. It is the 2nd phase in bacterial growth curve a. Hemophilus influenza
29. Thermophile bacteria grow at: (Recent Pattern Dec 15) b. Burkholderia pseudomallei
a. 20°C b. 20-40°C c. Corynebacterium pseudotuberculosis
c. 40-60°C d. 60-80°C d. Burkholderia mallei
30. Which of the following is microaerophilic 41. Growth below 20°C – bacteria are grouped as

 (Recent Pattern July 15) a. Mesophiles b. Thermophiles
a. E. coli c. Psychrophiles d. Capnophiles
b. Bacteriodes 42. Gram positive cocci arranged in tetrads
c. Clostridium a. Staphylococcus sp., b. Micrococcus
d. H. pylori c. Sarcina d. Enterococcus
31. Continuous cell culture of bacteria: 43. Which organism shows following type of flagellum (elec-
(Recent Pattern Nov 15) tron microscopic image)

a. U tube
b. Craige tube
c. Chemostat device
d. Agar dilution method
32. Concentration of sulfuric acid in acid fast staining for
Nocardia
a. 20% b. 5%
c. 1% d. 0.5%
33. Which is not present in Gram negative bacteria?
a. Peptidoglycan b. Teichoic acid
c. LPS d. Porin channels
34. Which of the following is microaerophilic:
a. Campylobacter b. Vibrio a. Vibrio cholera b. Spirillum minus
c. Bacteroides d. Pseudomonas c. Escherichia coli d. Campylobacter sp.,
35. Cells are uniformly stained and active in which phase? 44. All are obligate aerobes except:
a. Lag phase b. Log phase a. Brucella abortus b. Bacillus anthracis
c. Stationary phase d. Decline phase c. Lactobacillus sp., d. Pseudomonas sp.,
36. Tumbling motility is seen in: 45. Cell wall of gram negative bacteria contains: 
a. Listeria monocytogenes (PGI May 2018)

b. Mycoplasma a. Teichuronic acids b. Peptidoglycan
c. Clostridium tetani c. Lipopolysaccharide d. Teichoic acid
20 d. Campylobacter jejuni e. Lipoprotein
ANSWERS AND EXPLANATIONS

Chapter 2     Morphology and Physiology of Bacteria


1. Ans.  (b) Highly structured cell with organized cell • Mainly used for Spirochetes


organelles • Organism which comes under spirochetes are Trepone-


ma, Borrelia, Leptospira
Ref: Ananthanarayan and Paniker’s Textbook of Microbiol-
ogy – 10th ed – Page 8,9 8. Ans.  (d) H. influenzae
• When compared to prokaryotes, Eukaryotes have a
Ref: Ananthanarayan and Paniker’s Textbook of Microbiol-

complexed cell structure with all the organelles that are
necessary for life ogy – 10th ed – Page 317, 337
• Hence the presentation of infection is different Organisms which show bipolar staining or safety pin

appearance are:
2. Ans.  (c) Fungus
• Calymmatobacter granulomatis


Ref: Ananthanarayan and Paniker’s Textbook of Microbiol- • Burkholderia mallei


ogy – 10th ed – Page 8 • Burkholderia pseudomallei


• Vibrio parahemolyticus
Prokaryotes Bacteria, Chlamydiae, Mycoplasma, Blue


• Yersinia pestis
green algae, Archaea


• Haemophilus ducreyi


Eukaryotes Fungus, Algae other than blue green algae,
Protozoa 9. Ans.  (b) Vibrio parahemolyticus
Ref: Ananthanarayan and Paniker’s Textbook of Microbiol-
3. Ans.  (a) Mitochondria always absent; (c) Divide by
ogy – 10th ed – Page 317, 337
binary fission
• Already explained Q.8
Ref: Ananthanarayan and Paniker’s Textbook of Microbiol-

ogy – 10th ed – Page 8,9 10. Ans.  (d) Y. pestis
• Bacteria belongs to prokaryotes Ref: Ananthanarayan and Paniker’s Textbook of Microbiol-

• Do not have any cellular organelles
ogy – 10th ed – Page 317, 337

• Sterols are absent in cell wall exception is Mycoplasma

• Bacteria divides by binary fission • Already explained Q.8


4. Ans.  (a) Bacteria 11. Ans.  (a) Gentian violet—Iodine – carbol fuchsin

Ref: Ananthanarayan and Paniker’s Textbook of Microbiol- Ref: Ananthanarayan and Paniker’s Textbook of Microbiol-
ogy – 10th ed – Page 8 ogy – 10th ed – Page 13
  Bacteria has both DNA and RNA Gram staining procedure:
Primary staining with Gentian violet, Crystal violet
5. Ans.  (d) Outer membrane of gram negative bacteria
and Methyl violet
Ref: Jawetz Medical microbiology – 27th ed – Page 26 Mordant - iodine
• Outer membranes are seen only in the Gram negative Decolourisation – Acetone, Acid alcohol, alcohol

cell wall
Counterstain – Diluted Carbol Fuschin (1:10/1:20)
• Through this outer layer – many LPS molecules are seen

called as porins
12. Ans.  (a) Methylene blue
• These are actually channels which helps in the passage

of nutrients, proteins and antibiotics Ref: Ananthanarayan and Paniker’s Textbook of Microbiol-
ogy – 10th ed – Page 12
6. Ans.  (b) Capsule
• Methylene blue is a counterstain that is used in acid fast

Ref: Ananthanarayan and Paniker’s Textbook of Microbiol- staining
ogy – 10th ed – Page 18 • Methyl violet is the primary stain that is used in Gram

Demonstration of capsule is by: staining
• Negative staining in wet films with India Ink
13. Ans.  (a) Peptidoglycan

• Quellung reaction

Ref: Ananthanarayan and Paniker’s Textbook of Microbiol-
7. Ans.  (d) All of the above ogy – 10th ed – Page 12
Ref: Ananthanarayan and Paniker’s Textbook of Microbiol- • Peptidoglycan layer is thicker in Gram positive bacteria

ogy – 10th ed – Page 377 • Hence the primary stain is getting trapped inside this PG

• Silver impregnantion technique is used to see delicate layer which cannot get washed off with decolourisation 21

organisms
• While in Gram negative, the PG layer is thin and • Spores
Unit 1     General Microbiology



trapping of the dye is less and it also gets washed off with • Coccidian parasites


decolourisation
16. Ans.  (d) 20% sulphuric acid
14. Ans.  (b) M. leprae
Ref: Ananthanarayan and Paniker’s Textbook of Microbiol-
Ref: Ananthanarayan and Paniker’s Textbook of Microbiol- ogy – 10th ed – Page 13
ogy – 10th ed – Page 13 • Already explained Q.14


• Standard AFB techniques used 20% H2SO4

• Modifications of AFB are done for few acid fast organisms 17. Ans.  (a) Teichoic acid

• 5% H2SO4 is used for M.leprae as it is less acid fast Ref: Ananthanarayan and Paniker’s Textbook of Microbiol-

ogy – 10th ed – Page 16
15. Ans.  (d) All of the above
Ref: Ananthanarayan and Paniker’s Textbook of Microbiol- Properties in cell wall Gram positive Gram
ogy – 10th ed – Page 13 negative
Modified Ziehl Neelsen staining is used for: Thickness Thicker Thinner
• Mycobacterium leprae Amino acids Few Many

• Atypical Mycobacteria Lipopolysaccharides (LPS) Absent Present

• Nocardia
Teichoic acid Present Absent

18. Ans.  (b) Gram negative bacteria
Ref: Jawetz Medical microbiology – 27th ed – Page 26
• A small space that is located between cytoplasmic membrane and cell wall is called as periplasmic space

• It is seen in Gram negative bacterial cell wall

22 Figure: Gram negative cell wall (Adapted from Jawetz medical microbiology)
Chapter 2     Morphology and Physiology of Bacteria
19. Ans.  (c) Respiratory enzymes 25. Ans.  (d) Shigella dysentriae
Ref: Ananthanarayan and Paniker’s Textbook of Microbiol- Ref: Ananthanarayan and Paniker’s Textbook of Microbiol-
ogy – 10th ed – Page 17 ogy – 10th ed – Page 20
• Mitochondria is absent – hence the respiratory enzymes • Shigella sp are non motile GNB


are not located in mitochondria in bacteria; they are in Ref: Q.24
mesosomes (Chondroids)
• Principal sites of respiratory enzymes in bacteria – 26. Ans.  (b) Mycoplasma

Mesosomes Ref: Ananthanarayan and Paniker’s Textbook of Microbiol-
ogy – 10th ed – Page 21
20. Ans.  (d) Proteus
• Mycoplasma has no cell wall; It has sterols in cell


Ref: Ananthanarayan and Paniker’s Textbook of Microbiol- membranes
ogy – 10th ed
27. Ans.  (a) Lag phase
Capsulated organisms:
• Pneumococcus Ref: Ananthanarayan and Paniker’s Textbook of Microbiol-

• Bacillus anthracis ogy – 10th ed – Page 22

• Meningococcus • Initial period of adaptation time into a new culture

• Klebsiella sp


medium is called as Lag phase

• H.influenza • In this phase – there is no increase in the number of cells

• Yersinia


• There is only increase in the size of the cells

• Cl.perfingens



• Cryptococcus (Fungi) 28. Ans.  (a) Time taken to adapt in the new environment

21. Ans.  (a) Diphtheria; (c) Gardenella vaginalis Ref: Ananthanarayan and Paniker’s Textbook of Microbiol-
ogy – 10th ed – Page 22
Ref: Ananthanarayan and Paniker’s Textbook of Microbiol-
• Already explained Q.1
ogy – 10th ed – Page 18

Volutin or metachromatic granules are common in: 29. Ans.  (d) 60-80°C
• Corynebacterium diphtheria Ref: Ananthanarayan and Paniker’s Textbook of Microbiol-

• Mycobacteria ogy – 10th ed – Page 25

• Gardnerella vaginalis
• Thermophilic bacteria grow at higher temperatures

• Spirillum volutans

• Growth above 55°C – 80°C – Thermophiles


22. Ans.  (a) Ponder’s stain   Eg.: Bacillus stearothermophilus

Ref: Ananthanarayan and Paniker’s Textbook of Microbiol- 30. Ans.  (d) H. pylori
ogy – 10th ed – Page 14, 18,241
Ref: Ananthanarayan and Paniker’s Textbook of Microbiol-
Metachromatic granules are seen by: ogy – 10th ed – Page 25
• Albert stain
• Microaerophilic – grow in the presence of low oxygen

• Neisser’s stain

tension (5-10%)

• Ponder stain
  Eg.: Campylobacter, Helicobacter sp.,

23. Ans.  (b) Corynebacterium diphtheriae
31. Ans.  (c) Chemostat device
Ref: Ananthanarayan and Paniker’s Textbook of Microbiol-
Ref: Ananthanarayan and Paniker’s Textbook of Microbiol-
ogy – 10th ed – Page 14
ogy – 10th ed – Page 22
• Albert stain has two steps which stains the granules of
• Continuous culture of bacteria is needed for industrial

C. diphtheriae

or research purposes
• Bluish purple coloured - metachromatic granules
• Usually done as liquid cultures

• Also called as polar bodies, Volutin granules, Babes

• Mechnism used by Chemostat or Turbidostat

Ernst granules

• This constantly gives new medium and removes old

24. Ans.  (b) Proteus medium and maintains the nutrition

Ref: Ananthanarayan and Paniker’s Textbook of Microbiol- 32. Ans.  ( c) 1%


ogy – 10th ed – Page 19,287
Ref: Essentials of Medical Microbiology – Apurba Sastry –
• Peritrichous flagella – Flagella all around the bacilli Page 16

E.g. E. coli, Proteus, Listeria, Salmonella except galli-
• Mycobacterium tuberculosis → 20-25% 23
narum-pullorum

• Mycobacterium leprae → 5% • Swarming motility → Proteus, Cl. tetani
Unit 1     General Microbiology



• Nocardia →1% • Corkscrew, lashing, flexion extension → Spirochete


• Acid fast parasites → 0.25% to 0.5%

37. Ans.  (b) 14 -16 hours
33. Ans.  (b) Teichoic acid
Ref: Essentials of Medical Microbiology – Apurba Sastry –
Ref: Essentials of Medical Microbiology – Apurba Sastry – Page 26
Page 18 • Generation time: time required for a bacterium to give


Characters Gram positive Gram negative rise to two daughter cells under optimum condition
cell wall cell wall • E. coli and other pathogenic bacteria–20 minutes


• Mycobacterium tuberculosis: 14 to 16 hours
Peptidoglycan layer Thicker Thinner


• Myobacterium leprae: 12 to 13 days


Pentaglycine bridge Present Absent
38. Ans.  (d) Proteus mirabilis
Lipid content Nil or scanty Present
Lipopolysaccharide Absent Present Ref: Essentials of Medical Microbiology – Apurba Sastry –
Page 21
Teichoic acid Present Absent
Capsulated bacteria:
Aromatic Absent Present • Pneumococcus
aminoacids


• Meningococcus


• Haemophilus influenza
34. Ans.  (a) Campylobacter


• Klebsiella pneumoniae


Ref: Essentials of Medical Microbiology – Apurba Sastry – • Pseudomonas aeruginosa


Page 28 • Bacillus anthracis

• Cryptococcus (fungus)
• Microaerophilic: grow in the presence of low oxygen •
Composition of most of the capsulated organisms is poly-

tension (5-10%), E.g. Campylobacter, Helicobacter sp.,
saccharide
• Obligate aerobes: grow only in the presence of oxygen
Exception: Bacillus anthracis – Polypeptide

E.g. Pseudomonas, M.tb, Bacillus, Brucella and Nocardia
Strep. pyogenes – Hyaluronic acid
• Facultative anaerobes: aerobes that can grow

anaerobically also, E.g. E. coli, Staph aureus 39. Ans.  (b) Albert staining
• Facultative aerobes: anerobes that can also grow

aerobically, E.g. Lactobacillus Ref: Essentials of Medical Microbiology – Apurba Sastry –
• Obligate anaerobes: grow only in the absence of Page 16

oxygen, Eg: Clostridium • Gram stain: Gram positive (violet colour), Gram nega-
• Aerotolerant anerobes – tolerate oxygen for some time,

tive (Pink)

but do not use it, E.g. Cl. histolyticum • Acid fast stain: Blue background with pink curved bacilli

(Acid fast bacilli)
35. Ans.  (b) Log phase • Albert stain: Differentiaties metachromatic granules

Ref: Essentials of Medical Microbiology – Apurba Sastry – from those with and without (to diff Corynebacterium
Page 27 sp.,) Positive in C. diphtheria; Green bacilli with brown
granules
Characters Lag phase Log Stationary Decline • Lipids are stained by Sudan black stain

Important Accumulation Uniformly Gram Produce • Spores are stained by Malachite green stain

features in of enzymes stained variable involution
each phase and meta- Small size Produces forms 40. Ans.  (b) Burkholderia pseudomallei
bolites Meta- granules, Ref: Essentials of Medical Microbiology – Apurba Sastry –
Maximum bolically spores, Page 5
size active exotoxin,
antibiotics, Kleb Loeffler bacillus C. diphtheriae
bacteriocin Preisz nocard bacillus C. pseudotuberculosis
36. Ans.  (a) Listeria monocytogenes Koch week bacillus Haemo. aegypticus
Johne`s bacillus Myco. Paratuberculosis
Ref: Essentials of Medical Microbiology – Apurba Sastry –
Page 23 Gaffky eberth bacillus S. Typhi
Types of motility: Whitmore bacillus Burk. pseudomallei
• Tumbling motility → Listeria Battey bacillus Myco. intracellulare

• Gliding motility → Mycoplasma
Eatons agent Myco. pneumoniae

24 • Stately motility → Clostridium

• Darting motility → Vibrio cholera, Campylobacter Pfeiffer`s bacillus Haemophilus influenzae

41. Ans.  (c) Psychrophiles • Peritrichous (over the entire cell surface): Salmonella

Chapter 2     Morphology and Physiology of Bacteria



Typhi, Escherichia coli
Ref: Essentials of Medical Microbiology – Apurba Sastry –
• Amphitrichous (Single flagellum at both the ends):
Page 29


Alcaligenes faecalis, Spirillum
• Growth below 20°C – Psychrophiles; Eg.Saprophytes

• Growth between 25°C to 40°C – Mesophiles E.g.; Most of 44. Ans.  (c) Lactobacillus sp.,

the pathogenic bacteria
Ref: Essentials of Medical Microbiology – Apurba Sastry –
• Growth above 55°C– 80°C– Thermophiles E.g.: Bacillus
Page 28

stearothermophilus
• Microaerophilic: grow in the presence of low oxygen


42. Ans.  (b) Micrococcus tension (5-10%), E.g. Campylobacter , Helicobacter sp.,
• Obligate aerobes: grow only in the presence of oxygen,
Ref: Essentials of Medical Microbiology – Apurba Sastry –


E.g. Pseudomonas, M. tuberculosis, Bacillus, Brucella
Page 17
and Nocardia
Gram positive cocci arranged in • Facultative anaerobes: aerobes that can grow
• Clusters: Staphylococcus


anaerobically also, E.g. E. coli, S. aureus

• Chain: Streptococcus • Facultative aerobes – anerobes that can also grow

• Pairs: Lanceolate shaped – Pneumococcus


aerobically, E.g. Lactobacillus

• Tetrads: Micrococcus • Obligate anaerobes: grow only in the absence of

• Octate: Sarcina


oxygen, E.g. Clostridium

• Pair with spectacle eye shaped: Enterococcus • Aerotolerant anerobes – tolerate oxygen for some time,


but do not use it, E.g. Cl. histolyticum
43. Ans.  (b) Spirillum minus
Ref: Essentials of Medical Microbiology – Apurba Sastry – 45. Ans.  (b,c,e) b. Peptidoglycan; c. Lipopolysaccharide;
Page 23 e. Lipoprotein

• Monotrichous (Single polar flagellum): Vibrio cholera, Ref: Jawetz medical microbiology – 27th ed – Page 26

Pseudomonas and Campylobacter • Both Gram positive and Gram negative has
• Lophotrichous (Multiple polar flagella): Spirillum

peptidoglycans

• Amino acids and lipids (lipoproteins) are plenty in

Gram negative
• LPS are present only in Gram negative

25
3 Sterilization and
Disinfection
STERILIZATION
y Methods that are used to kill the microorganisms are called as sterilization methods.
y
y There are various methods which cover different spectrum of microbes and different mechanism of action
y
Table 1:  Agents of sterilization
Physical agents Chemical agents
Dry heat Flaming Alcohols Ethyl alcohol
Incineration Isopropyl alcohol
Hot air oven
Moist heat Pasteurization Aldehydes Formaldehyde
Inspissation Glutaraldehyde
Boiling Orthophthalaldehyde
Arnold steamer Peracetic acid
Tyndallization Hypochlorous acid
Autoclaving
Filtration Candle filters Dyes Aniline dyes
Sintered glass filters Acridine dyes
Membrane filters
Radiation Nonionizing radiation Halogens Iodine
Ionizing radiation Chlorine
Phenols Carbolic acid
Lysol
Cresol
Surface active agents Cetrimide or cetavlon
Benzalkonium chloride
Gases Ethylene oxide
Formaldehyde gas
Betapropiolactone
Hydrogen peroxide fogging
Miscellaneous Metallic salts
Plasma sterilization

Table 2:  Temperature at which organisms are killed


Organisms Temperature at which it is killed
All nonsporing bacteria 60°C for 30 minutes
S. Aureus and S. faecalis 60°C for 60 minutes
Yeasts, molds and vegetative bacteria 80°C for 5-10 minutes
Spores of Clostridium botulinum 120°C for 4 minutes or 100°C for 30 minutes
All viruses except polio and HBV 60°C
Polio and HBV 60°C for 30 minutes and 10 hours respectively
Mechanism of Action of Sterilizing Agents Table 3: Holding period used in hot air oven

Chapter 3     Sterilization and Disinfection


y Alteration of membrane permeability


y

Temperature (°C) Holding Time (Minutes)


y Damage to proteins
y

y Damage to nucleic acids


y
160 120
170 60
Remember
Organisms in decreasing order of resistance 180 30
Prions
↓ Moist Heat
Endospores of Bacteria
Moist heat kills microbes by coagulation of proteins, i.e.
↓ denaturation. Most commonly employed method in moist heat
Mycobacteria
is boiling.

Protozoal Cysts
Table 4: Pasteurization of milk

Vegetative Protozoa Holder method 63°C for 30 minutes


↓ Coxiella burnetti is heat resistant and
GNB survives holder method

Fungi Flash method 72°C for 15-20 seconds followed by
↓ quickly cooling to 13°C
Nonenveloped Viruses
↓ Ultra high 74 to 140°C in 5 seconds and cooled
GPC/GPB temperature method back to 74°C

Enveloped Viruses Inspissation
y A method that is used for media that contains egg or serum
Physical Agents
y

y Because if such media are autoclaved it leads to coagulation


y

and the property of solidification is lost


Dry Heat y Mainly used for Lowenstein Jensen (LJ) media and Loeffler’s
y

Mechanism of action of dry heat is: Protein denaturation, serum slope


damage by oxidizing molecules and destroying the cellular y Temperature 80-85°C /half an hour for three successive days
y

constituents by elevated level of electrolytes getting accumulated


causing toxicity Tyndallization
y Flaming is the method that is used for sterilizing inoculating
y

loop or wire, tips of forceps y Also called as intermittent sterilization


y

y Incineration is used in disposal of biomedical wastes


y
y A method that is used for media that contains sugars or
y

y Hot air oven:


y
gelatin
 Mechanism: conduction of hot air

y Temperature 100°C /20 minutes for three successive days.
y

 Holding period: 160°C/ 2 hours (M/c)




 Used to sterilize glassware, forceps, scissors, scalpels, glass




Remember
syringes, swabs, liquid paraffin, dusting powder, fats and
grease
 Precaution while using hot air oven are:


Š Overloading should not be done


Š

Š The items should be properly arranged with space to


Š

have free flow of air circulation


Š Before keeping inside hot air oven – the items should
Š

be completely dry
Š Flasks and test tubes (glass wares) should be wrapped
Š

well to avoid breakage


 Disadvantages of hot air oven:


Š It is a bad conductor of heat and hence low penetration


Š

Š A fan is needed inside to allow even distribution of air


Š

Š Rubber material cannot be sterilized


Š

Š Culture media cannot be sterilized


Š

27
Autoclaving
Unit 1     General Microbiology

y Mechanism is: Steam under pressure


Remember
Sterilization Controls
y

y When water inside the autoclave boils, its vapor pressure


y

is equal to that of surrounding atmospheric pressure – the To check the efficiency of sterilization – certain controls are
pressure inside a closed vessel gets increased. This steam has used
the highest penetration power. Controls used in the process of sterilization
y Holding period that is usually used are 121°C for 15 minutes
y

Physical control Check the temperature by thermocouples


– 15 lb/inch2 pressure
Chemical Browne’s tube is used – Green color seen –
y The most effective method to kill spores
control it means effective sterilization
y

y Used for sterilization of culture media, clothes, aprons, gloves,


y

surgical trays and instruments except sharps Biological Spores are used in form of tapes or packets
control and checked for color change or growth
Table 5: Holding period used in autoclaving to know the sterilization has occurred
properly

Temperature (°C) Holding Time (Minutes)


121 15 Filtration
126 10 y Method used to remove bacteria from liquids like serum,
y

sugar solutions, urea solutions and antibiotics.


134 3 y Types of filters:
y

 Candle filters: used for purification of water for drinking




purposes; Two types – (1) Unglazed ceramic filters (Cham-


berland and Doulton), (2) Diatomaceous earth filters
(Berkefeld and Mandler)
Š Asbestos filters
Š

Š Sintered glass filters


Š

Š Membrane filters – Best method; Average pore diame-


Š

ter which is most common used is 0.22 nm


y HEPA – High efficiency particulate air filters – removes 99.97%
y

of particles that have a size of 0.3 um or more.

Radiation
y Ionizing radiation: X-rays, Gamma rays and cosmic rays;
y

These have high penetrating power; also called as cold


sterilization; mainly used for sterilization of plastics,
syringes, swabs, catheters, animal feeds, card board, oils,
greases, fabric and metal foils
y Nonionising radiation
y

 UV rays: Mainly used for sterilization of biosafety




cabinets, entry ways in ICU and OT


 Infrared rays: Used for prepacked items like syringes and


catheters

Table 6: Comparison between ionizing and Non-ionizing


Radiation
Ionizing Radiation Nonionizing Radiation
Figure 1: Laboratory autoclave (Courtesy: Dr.Jane Esther, Doctor’s There is no heat – hence it is It has heat liberation
Diagnostic centre, Trichy) called as cold sterilization
E.g. X-rays, Gamma rays and E.g. UV rays and IR rays
cosmic rays
Helpful for sterilization of Helpful for disinfection of
packed items ICU and closed cabinets like
biosafety cabinets

28
Chemical Agents

Chapter 3     Sterilization and Disinfection


Remember
Alcohols
y Ethyl alcohol or ethanol: used as skin antiseptics
y

y Isopropyl alcohol: for disinfection of clinical thermometers


y

Remember

Phenols
y Distillation of coal tar yields phenol
y

y It damages the cell membranes and causes protein denatur-


y

ation
y These are not readily inactivated by presence of organic
y

matter
y For surface disinfection – lysol and cresol are used
Aldehydes
y

y Chlorhexidine or hibitane is used as skin antiseptic for


y

y Formaldehyde: for fumigation of wards, isolation rooms and


y wound dressing
laboratories; Gas is irritant and toxic on inhalation y It is not sporicidal.
y

y Glutaraldehyde: Effective against TB bacilli, fungi and


y

viruses; used to sterilize endoscopes, cystoscopes, rubber Gases


anesthetic tubes, plastic endotracheal tubes and polythene
y Ethylene oxide
tubing; Concentration used is 2% - Cidex;
y

 Colorless, Highly explosive gas: effective against all mi-


y Orthophthalaldehyde: Used to clean endoscopes


crobes including spores


y Peracetic acid: Effective against MRSA, VRE and Clostridium
y

 Mainly used for sterilizing




difficile Š Heart lung machines


Š

y Hypochlorous acid: Generated from sodium hypochlorite


y

Š Respirators
Š

and Hydrogen peroxide; active against biofilms Š Sutures


Š

Š Dental equipment
Š

Table 7: Aldehydes used in the process of sterilization



Š Glass
Š

Š Metal surfaces
Formaldehyde Glutaraldehyde
Š

 It has two cycles: Cold cycle at 37±5°C; Warm cycle at




• Bactericidal, Virucidal and


• • Not sporicidal

54±5°C
Sporicidal y Beta propiolactone: Carcinogenic; not used now; previously
y

• Used to preserve anatomical • Used to disinfect


• •
used for fumigation and for inactivation process in vaccines
specimens endoscopes, cystoscopes y Hydrogen peroxide fogging: Nowadays formaldehyde
y

• Used to sterilize OT in

fumigation has been replaced by H2O2 fogging; it is not toxic
gaseous form
• More toxic
• • Less toxic

Surface Active Agents
y These agents produce a reduction in surface tension
y

y Cationic compounds are also called as Quaternary ammoni-


Halogens
y

um compounds
y Iodine: Compounds of iodine with nonionic wetting or
y
y Action is on bacteria; no action on spores, TB bacilli and most
y

surface active agents is known as iodophores – these have viruses


more action than solutions of iodine; Povidone iodine (10%) y Agents are:
y

is an effective skin disinfectant  Acetyl trimethyl ammonium bromide – Cetavlon or




y Chlorine: mainly used for disinfection of town water supplies,


y cetrimide 29
swimming pools, food and dairy industries.  Benzalkonium chloride

Table 8: Classification of surface active agents DISINFECTION
Unit 1     General Microbiology

Anionic Sulfide, Fluoride, Bromide, Iodide Table 11: Levels of disinfection


Cationic Sodium, Iron, lead Low level disinfectant Quaternary ammonium compounds
Non ionic Polyoxyethylene (Tween/triton) (Benzalkonium chloride)
Intermediate level Isopropyl alcohol, Phenol
Amphoteric Tego compounds
disinfectant
High level disinfectant Glutaraldehyde, Formaldehyde,
Plasma Sterilization Hydrogen peroxide and Chlorine
y Plasma is the fourth state of matter
Disinfection of Sputum
y

y Using a radio frequency energy – an electromagnetic field is


y

created y y Treating with 5% phenol or cresol


y Hydrogen peroxide vapors are introduced into this which
y

y y Boiling
generates free radicals — plasma y y Incineration or burning
y Used to sterilize arthroscopes and urethroscopes
y

y y Autoclaving (most efficient method)

Table 9: Method of sterilization and their biological


Disinfection of Blood Spills

control
WHO guidelines on blood spillage management state the
Methods Sterilized from following steps:
y Wear gloves
Hot air oven Clostridium tetani nontoxigenic strain
y

y Use disposable paper towels to absorb as much of the body


y

Autoclave Geobacillus stearothermophilus fluids as possible.


y Wipe the area with water and detergent until it is visibly clean.
Filtration Serratia marcescens
y

y Saturate the area again with sodium hypochlorite 0.5% (10


y

Ionizing radiation Bacillus pumilus 000 ppm available chlorine). This is a 1:10 dilution of 5.25%
sodium hypochlorite bleach, which should be prepared daily.
Ethylene oxide Bacillus globigii
y Rinse off the tongs, brush, and pan, under running water and
y

Plasma sterilization Bacillus subtilis subsp.,niger place to dry.


Geobacillus stearothermophilus y Remove gloves and discard them.
y

y Wash hands carefully with soap and water, and dry thoroughly
y

Table 10: Spaulding’s classification



with single-use towels.
y Record the incident in the incident book if a specimen was
y

Spaulding’s Examples lost, or persons were exposed to blood and body fluids.
classification
Critical device Surgical instruments, cardiac and urinary
Remember
catheters, implants, eye and dental Prions are killed by:
instruments • Autoclaving at 134°C for 1 to 1.5 hours

• Treatment with 1 N Sodium hydroxide (NaOH) for 1 hour


Semicritical device Respiratory therapy equipment, • Treatment with 0.5% Sodium hypocholorite (NaOCl) for 2

anesthesia equipment, endoscopes, hours


laryngoscopes and probes
Noncritical device BP cuff, ECG electrodes, bed pans, Testing of Disinfectants
crutches, stethoscope, thermometer y Rideal-Walker test (Phenol coefficient)
y

Noncritical Surfaces of medical equipment,  The disinfectant that need to be tested is checked for


environmental Examination table, computers sterilization of a known suspension of S. typhi in a given


surfaces period of time divided by the dilution of phenol, which
sterilizes the suspension in the same time.
 Phenol coefficient value >1 is considered as satisfactory
Sterilization of Operation Theaters


 This test does not give proper result when there is a pres-


y Formaldehyde fumigation is done by adding formalin to


y
ence of organic matter
potassium permangante for the desired area. The room y Chick-Martin test: This test can be done even in the presence
y

should be sealed with electrical supply cut for 48 hours. To of organic matter
neutralize the toxic gas after 48 hours – ammonia is used. But y Kelsey-Sykes or in use test: This test helps the capacity of
y

due to toxicity nowadays it is not used a disinfectant to retain its activity when repeatedly using in
30
y Hydrogen peroxide fogging.
y sterilization.
SUMMARY OF STERILISATION

Chapter 3     Sterilization and Disinfection


Method Used Material Sterilised
Isopropyl alcohol Clinical thermometers
Hot air oven Paraffin, Glass syringe, Flask, Grease, Fat
Formaldehyde Operation theatre, lab fumigation
Orthophthaldehyde and glutaraldehyde Cystoscope, Bronchoscope
Ethylene oxide Heart lung machine
Filtration Vaccine, Sera, Antibiotic, Sugar
Cresol Sharp instrument
Pasteurisation Milk
Ionising radiation Plastic syringe, Catgut, Catheter
Autoclaving Culture media, all sutures except catgut

31
MULTIPLE CHOICE QUESTIONS
Unit 1     General Microbiology

1. All are methods of sterilization, except: 11. Autoclaving is done in: (Recent Pattern Dec 14)
 (Recent Pattern July16) a. Dry air at 121°C and 15 lb pressure
a. Ionizing radiation b. Steam at 100°C for 30 minutes
b. Ethylene oxide c. Steam at 121°C for 15 minutes
c. Formaldehyde d. Dry air at 160°C for 30 min
d. Chlorhexidine 12. Temperature required for holding period of 60 minutes
2. The best skin disinfectant is: (Recent Pattern Dec 14) in hot air oven: (Recent Pattern Dec 15)
a. Alcohol a. 160°C b. 170°C
b. Savlon c. 120°C d. 130°C
c. Betadine 13. Operation theaters are sterilized by: (DPG 10)
d. Phenol a. Carbolic acid spraying
3. Most commonly used antiseptic: b. Washing with soap and water
 (Recent Pattern Dec 15) c. Formaldehyde
a. Povidone iodine b. Crystal violet d. ETO gas
c. Hibitane d. H2O2 solution 14. Glutaraldehyde is used to sterilize:
4. Which of the following can be reliably used for hand  (Recent Pattern July16)
washing? (PGI Nov 15) a. Endoscopes
a. Chlorhexidine b. Corrugated rubber anesthetic tube
b. Isopropyl alcohol c. Plastic endotracheal tubes
c. Lysol d. All of the above
d. Cresol 15. Endoscope tube is sterilized by:(Recent Pattern Dec 13)
e. Glutaraldehyde a. Glutaraldehyde
5. True about hand hygiene: (Recent Pattern July 16) b. Formalin
a. Betadine can cause irritation
c. Autoclaving
b. Alcohol based preparation are used
d. Boiling
c. Hot water is best
16. Percentage of glutaraldehyde used:
d. Glutaraldehyde is used
(Recent Pattern Dec 13)
6. Sterilization of culture media containing serum is by:


a. 1% b. 2%
(Recent Pattern Dec 12)
c. 3% d. 4%


a. Autoclaving
17. Ionizing radiation commonly used for disinfection:
b. Micropore filter
(Recent Pattern Dec 15)
c. Gamma radiation


a. UV rays b. Infrared
d. Centrifugation
7. All are methods of sterilization by dry heat, except: c. X-rays d. Gamma rays
(Recent Pattern Dec 14) 18. Gamma irradiation used for all of the following, except:
(Recent Pattern July 16)


a. Flaming 

b. Incineration a. Syringes
c. Hot air oven b. Catgut suture
d. Autoclaving c. Grafts
8. Lethal effect of dry heat is due to: d. Endoscope
 (Recent Pattern Dec 15) 19. Irradiation can be used to sterilize A/E: 
a. Denaturation of proteins a. Bone graft  (AIIMS May 10)
b. Oxidative damage b. Suture
c. Toxicity due to metabolites c. Artificial tissue graft
d. All of the above d. Bronchoscope
9. Tyndallization is a type of: (Recent Pattern Dec 12) 20. Inspissation is used for: (Recent Pattern July 16)
a. Intermittent sterilization a. Sputum
b. Pasteurisation b. Protein containing culture medium
c. Boiling c. Serum containing culture medium
d. Autoclaving d. Plasma sterilization
10. Glass vessels and syringes are best sterilised by  21. All are sporicidal agents, except:
 (Recent Pattern Dec 13)  (Recent Pattern Dec 13)
a. Hot air oven a. Formaldehyde
b. Autoclaving b. Glutaraldehyde
32 c. Irradiation c. Ethylene oxide
d. Ethylene dioxide d. Isopropyl alcohol
22. Spores of bacteria are destroyed by: 35. Temperature used for Pasteurisation is:

Chapter 3     Sterilization and Disinfection


 (Recent Pattern Dec 13)  (Recent Pattern 2018)
a. Alcohol b. Lysol a. 72°C for 20 min b. 63°C for 30 min
c. Halogen d. Ionizing radiation c. 100°C for 10min d. 94°C for 20 min
23. According to Spaulding classification system of steril- 36. Blood spill in operation theatre is cleaned with:
ization, following is true except:(Recent Pattern Nov 10)  (Recent Pattern 2017)
a. “Non-critical devices” come into contact with intact a. Chlorine compound
skin b. Phenolic compound
b. Semi-critical equipment need low level sterilization c. Quartnery ammounium compounds
c. “semi-critical devices” come into contact with non- d. Alcoholic compounds
sterile mucous membrane or nonintact skin 37. Bacterial indicator for dry heat sterilization is?
d. Cardiac catheter is critical equipment  (PGI Pattern 2017)
24. High level disinfectant are used for: a. Bacillus subtilis
 (Recent Pattern July 16) b. Bacillus pumilis
a. Stethoscopes b. Electronic thermometers c. Bacillus stearothermophilus
c. Bronchoscope d. Surgical instruments d. Coxiella burnetti
25. Which of the following is an intermediate level disinfec- e. Bacillus anthracis
tant: (Recent Pattern Dec 13) 38. All of the following are killed in holders method of
a. 2% glutaraldehyde b. Ethylene oxide pasteurization except:
c. Hypochlorite d. None a. Mycobacteria b. Brucellae
26. Best virucidal disinfectant is: (Recent Pattern Nov 15) c. Salmonellae d. Coxiella burnetti
a. Phenol 39. Tyndallization is steaming for 3 consecutive days at:
b. Hypochlorite a. 100°C for 10 minutes
c. BPL b. 100°C for 20 minutes
d. Formaldehyde c. 80°C for 15 minutes
27. Best indicator for sterilization by autoclaving: d. 85°C for 20 minutes
 (Recent Pattern Dec 15) 40. Biological indicator for autoclaving:
a. Bacillus subtilis b. Geobacillus a. Geobacillus stearothermophilus
c. Bacillus pumilus d. Clostridium b. Clostridium perfringens
28. Plasma sterilization accuracy is assessed by using c. Bacillus subtilis
 (Recent Pattern Nov 10) d. Clostridium tetani
a. Bacillus subtilis 41. HEPA filter removes particle of size:
b. Bacillus stearothermophilus a. 0.5 um b. 0.3 um
c. Staphylococcus aureus c. 0.8 um d. 1 um
d. Clostridium tetani 42. Sterilisation of endoscopes and cystoscopes is by
29. Browne’s tube is used for: (Recent Pattern Dec 12) a. Glutaraldehyde b. Ortho-phthaladehyde
a. Steam sterilization b. Radiation c. Formaldehyde d. ULPA filters
c. Chemical sterilization d. Filtration 43. Prions are best killed by:
30. Heat labile liquids are sterilized by: a. Autoclaving at 121°C
 (Recent Pattern Dec 15) b. 5% Formalin
a. Hot air oven b. Autoclaving c. Sodium hydroxide for 1hr
c. Membrane filter d. Moist heat d. Sodium hypochloride for 10min
31. Sputum can be disinfected by all, except: 44. Low level of disinfectant is:
 (AIIMS May 12, Nov 10) a. Benzalkonium chloride b. Isopropyl alcohol
a. Autoclaving b. Boiling c. Glutaraldehyde d. Hydrogen peroxide
c. Cresol d. Chlorhexidine 45. Surgical blade is sterilized by:
32. Phenol coefficient indicates: (Recent Pattern July15) a. Autoclave b. Gamma radiation
a. Efficacy of a disinfectant c. Hot air oven d. Steaming
b. Dilution of a disinfectant 46. Which of the following is used for inactivation of vac-
c. Quantity of a disinfectant cines?
d. Purity of a disinfectant a. Ethylene oxide b. Isopropyl alcohol
33. HEPA filter is used to disinfect: (Recent Pattern Dec15) c. Rectified sprit d. Beta propiolactone
a. Water b. Air 47. According to Spaulding’s classification, ECG electrodes
c. Culture d. Blood comes under:
34. Quaternary ammonium compound disinfectants are: a. Critical device
 (Recent Pattern July 15) b. Semi critical device
a. Anionic b. Cationic c. Non critical device
c. Neutral d. Gases d. Non critical environmental surface 33
48. Most commonly used pore size of membrane filters is: 52. Ionizing radiation includes all except:
Unit 1     General Microbiology

a. 0.22 um b. 0.24 um a. X-rays b. Cosmic rays


c. 0.26 um d. 0.20 um c. Y rays d. UV rays
49. Method of disinfection of contact lenses: 53. Plasma sterilisation is done by: 
a. Tincture iodine b. Isopropyl alcohol  (AIIMS May 2018)
c. Hydrogen peroxide d. Chlorine a. Hydrogen peroxide b. Ethylene oxide
50. Sterilization of serum is done by: c. Gamma rays d. UV rays
a. Filtration b. IR radiation 54. An intern while doing phlebotomy spilled blood in the
c. Boiling d. Pasteurisation floor accidentally. What is the next ideal step in the
51. Quaternary ammonium compounds kill all except: disinfection of blood?  (AIIMS Nov 2018)
a. Escherichia coli a. Pour 1% hypochlorite solution
b. Pseudomonas sp b. Cover with a cloth/material
c. Staphylococcus aureus c. Mop the floor
d. Mycobacterium tuberculosis d. Call infectious control unit

ANSWERS AND EXPLANATIONS

1. Ans.  (d) Chlorhexidine • Culture media containing serum, amino acids or other

thermolabile constituents gets decomposed during


Ref: Ananthanarayan and Paniker’s Textbook of Microbiol-
autoclaving
ogy – 10th ed – Page 34
• Hence filtration is the ideal method

• Chlorhexidine or hibitane is a skin antiseptic and it’s a


disinfectant 7. Ans.  (d) Autoclaving


• All other methods come under sterilization
Ref: Ananthanarayan and Paniker’s Textbook of Microbiol-

2. Ans.  (c) Betadine ogy – 10th ed – Page 29


• Methods of dry heat sterilization are flaming, incinera-
Ref: Greenwood – Medical microbiology – 16th ed – Page 81

tion and hot air oven


• The best skin disinfectant is Betadiene – 10% Povidone

• Autoclaving used moist heat with steam under pressure

iodine (M/c used)


• Because it is less irritant and causes less staining.
• 8. Ans.  (d) All of the above

3. Ans.  (a) Povidone iodine Ref: Ananthanarayan and Paniker’s Textbook of Microbiol-
ogy – 10th ed – Page 29
Ref: Greenwood – Medical microbiology – 16th ed – Page 81
• Mechanism of action of dry heat is:

• Already explained in Q.2


ƒ Protein denaturation
ƒ

ƒ Damage by oxidizing molecules


4. Ans.  (a) Chlorhexidine; b) Isopropyl alcohol
ƒ

ƒ Destroying the cellular constituents


ƒ

Ref: Greenwood – Medical microbiology – 16th ed – Page 80 ƒ Elevated level of electrolytes causing toxicity
ƒ

• Best disinfectants for hand washing are:


9. Ans.  (a) Intermittent sterilization


ƒ Povidone iodine
ƒ

ƒ Chlorhexidine
ƒ
Ref: Ananthanarayan and Paniker’s Textbook of Microbiol-
ƒ Isopropyl alcohol
ƒ
ogy – 10th ed – Page 31
• Tyndallization is also called as intermittent sterilization
5. Ans.  (b) Alcohol based preparation are used

• Media containing sugars or gelatin are sterilized by


Ref: Greenwood – Medical microbiology – 16th ed – Page 81 heating at 100°C/20 minutes on 3 successive days
• The most effective preparations are aqueous and alcohol

10. Ans.  (a) Hot air oven


based solutions
• Povidone iodine preparation are most commonly used

Ref: Ananthanarayan and Paniker’s Textbook of Microbiol-
ogy – 10th ed – Page 30
6. Ans.  (b) Micropore filter • Hot air oven is used for sterilization of Glass ware,

Ref: Ananthanarayan and Paniker’s Textbook of Microbiol- forceps, scissors, scalpels, glass syringes, swabs, liquid
ogy – 10th ed – Page 32 paraffin, dusting powder, fats and grease
34
11. Ans.  (c) Steam at 121°C for 15 minutes 18. Ans.  (d) Endoscope

Chapter 3     Sterilization and Disinfection


Ref: Ananthanarayan and Paniker’s Textbook of Microbiol- Ref: Ananthanarayan and Paniker’s Textbook of Microbiol-
ogy – 10th ed – Page 31 ogy – 10th ed – Page 32
• Steam under pressure
• • Already explained in Q.14 and Q.17

• Holding period that is usually used are 121°C for 15


minutes 19. Ans.  (d) Bronchoscope


Ref: Ananthanarayan and Paniker’s Textbook of Microbiol-
12. Ans.  (b) 170 °C
ogy – 10th ed – Page 32
Ref: Ananthanarayan and Paniker’s Textbook of Microbiol- • Already explained Q.14 and Q.17

ogy – 10th ed – Page 31


20. Ans.  (b) Protein containing culture medium
Temperature (°C) Holding Time (Minutes)
160 120 Ref: Ananthanarayan and Paniker’s Textbook of Microbiol-
ogy – 10th ed – Page 30
170 60
• Inspissation: A method that is used for media that

180 30 contains egg (Protein)


• Because if such media are autoclaved it leads to

13. Ans.  (c) Formaldehyde coagulation and the property of solidification is lost.
Ref: Ananthanarayan and Paniker’s Textbook of Microbiol- • Mainly used for Lowenstein Jensen (LJ) media and

ogy – 10th ed – Page 34 Loeffler’s serum slope


• Temperature 80-85°C /half an hour for three successive
• Formaldehyde fumigation is the ideal choice for steril-

days

ization of OT
• The question is controversial as we used Loeffler’s
• But the drawback is because it is irritant and toxic

serum slope also in inspissation which contains serum;


14. Ans.  (d) All of the above But the most commonly used media is LJ in inspissation
hence I have chosed option B
Ref: Ananthanarayan and Paniker’s Textbook of Microbiol-
ogy – 10th ed – Page 33 21. Ans.  (d) Isopropyl alcohol
Glutaraldehyde is used for disinfection of: Ref: Jawetz medical microbiology – 25th ed – page 63-67
• Cystoscopes
• List of sporicidal agents:

• Endoscopes

ƒ Glutaraldehyde

• Bronchoscopes
ƒ

ƒ Formaldehyde

• Corrugated rubber anesthetic tubes


ƒ

ƒ Beta propiolactone

• Face masks
ƒ

ƒ Ethylene oxide

• Plastic endotracheal tubes


ƒ

ƒ Halogens

• Polythene tubing
ƒ

ƒ Peracetic acid

15. Ans.  (a) Glutaraldehyde ƒ Hydrogen peroxide


ƒ

Ref: Ananthanarayan and Paniker’s Textbook of Microbiol- 22. Ans.  (c) Halogen
ogy – 10th ed – Page 33 • Already explained Q.21

• Already explained Q.14


23. Ans.  (b) Semi critical equipment need low level


16. Ans.  (b) 2% sterilization
Ref: Ananthanarayan and Paniker’s Textbook of Microbiol- Ref: Ananthanarayan and Paniker’s Textbook of Microbiol-
ogy – 10th ed – Page 33 ogy – 10th ed – Page 35
• 2% Cidex – Glutaraldehyde is used.
• • According to Spaulding classification – semi critical

items are the one that comes in contact with the mucous
17. Ans.  (d) Gamma rays membranes or nonintact skin
Ref: Ananthanarayan and Paniker’s Textbook of Microbiol- • These need high level of disinfection

ogy – 10th ed – Page 32 • Glutaraldehyde is the most commonly used disinfectant


• X rays, Gamma rays and cosmic rays comes under



24. Ans.  (c) Bronchoscope
ionizing radiation
• Gamma rays are most commonly used for sterilizing of

Ref: Ananthanarayan and Paniker’s Textbook of Microbiol-
plastics, syringes, swabs, catheters, animal feeds, card ogy – 10th ed – Page 35
board, oils, greases, fabric and metal foils, bone and • Bronchoscopes comes under semi critical items and
35

tissue grafts. need high level of disinfection


• Stethoscopes and thermometers need low level of
Unit 1     General Microbiology


32. Ans.  (a) Efficacy of a disinfectant
disinfection
Ref: Ananthanarayan and Paniker’s Textbook of Microbiol-
25. Ans.  (c) Hypochlorite ogy – 10th ed – Page 35

Ref: Ananthanarayan and Paniker’s Textbook of Microbiol- • Phenol coefficient is used term in Rideal Walker test

ogy – 10th ed – Page 37 where phenol is taken as standard for checking the
efficiency of a disinfectant
• Among level of disinfectants – glutaraldehyde and

ethylene oxide comes under high level 33. Ans.  (b) Air
• Iodophores and hypochlorite are intermediate level of

disinfectants Ref: Handbook of modern hospital safety – page150


• HEPA – High efficiency particulate air filter

26. Ans.  (d) Formaldehyde • Used in operation theatres and laboratories


Ref: Ananthanarayan and Paniker’s Textbook of Microbiol- • It removes >95% particles with a diameter of more than

ogy – 10th ed – Page 34 0.3um

• Among all the options – formaldehyde is the highest level



34. Ans.  (b) Cationic
of disinfectant which covers all organisms including
spores Ref: Ananthanarayan and Paniker’s Textbook of Microbiol-
ogy – 9th ed – 34
27. Ans.  (b) Geobacillus • Surface active agents produce a reduction in surface

Ref: Ananthanarayan and Paniker’s Textbook of Microbiol- tension


ogy – 10th ed – Page 32 • Among them cationic compounds are also called as

quaternary ammonium compounds


• Geobacillus stearothermophillus is the biological indi-

cator for sterilization by autoclaving 35. Ans.  (b) 63°C for 30 min

28. Ans.  (b) Bacillus stereothermophilus Ref: Ananthanarayan and Paniker’s Textbook of Microbiol-
ogy – 10th ed – Page 30
Ref: Gas plasma sterilization in microbiology – page 71
• Pasteurisation is a method for sterilization of milk
• Bacillus stearothermophilus is the biological indicator

• Two methods are commonly used for pasteurisation


for plasma sterilization (Ref: the above table in text) ƒ Holder method → 63 °C for 30 minutes
ƒ

ƒ Flash method → 72°C for 20 seconds


29. Ans.  (c) Chemical sterilization
ƒ

Ref: Ananthanarayan and Paniker’s Textbook of Microbiol- 36. Ans.  (a) Chlorine compound
ogy – 10th ed – Page 30 Ref: https://www.cdc.gov/infectioncontrol/pdf/guidelines/
• Chemical control of sterilization - Browne’s tube is used

disinfection-guidelines.pdf
• When green color is seen – it means effective sterilization

• Chlorine is mainly used for disinfection of water –


swimming pools, tanks


30. Ans.  (c) Membrane filter
• Its compound sodium hypochlorite is the best

Ref: Ananthanarayan and Paniker’s Textbook of Microbiol- disinfectant for surface blood spills
ogy – 10th ed – Page 32 • Concentration used is 0.5 to 5% depend upon the

• Autoclaving and hot air oven is not suitable for hot labile contamination
• Immediately seeing a blood spill, cover the area with

items as they use higher temperatures •

• Membrane filters are ideal to sterilize serum, heat labile



tissue paper and pour 1% sodium hypochlorite over
liquids, oils, antimicrobials, solutions and liquid culture it for 15-30 minutes and then discard with universal
media precautions

31. Ans.  (d) Chlorhexidine 37. Ans.  (a) Bacillus subtilis

Ref: NCA review for the laboratory clinical sciences – page Ref: Ananthanarayan and paniker’s textbook of microbiol-
112 ogy – 9th edition p30
• Chlorhexidine is a skin disinfectant which is not suitable

• EXPL: Biological indicator in HOT AIR OVEN-The

for sputum disinfection spores of non toxigenic strain of Clostridium tetani or


• Disinfection of sputum:

Bacillus subtilis are used as microbiological test of dry
ƒ Treating with 5% phenol or cresol heat efficiency.
• Paper strips impregnated with 106 spores are placed in
ƒ

ƒ Boiling
ƒ

ƒ Incineration or burning envelopes and inserted into suitable packs.


• After sterilization, the strips are removed and inoculated
ƒ

36 ƒ Autoclaving (most efficient method)


ƒ

in thioglycollate or cooked meat media and incubated


for sterility test under strick anaerobic conditions for 43. Ans.  (c) Sodium hydroxide for 1hr

Chapter 3     Sterilization and Disinfection


five days at 37oC.
Ref: Essentials of Medical Microbiology – Apurba Sastry –
38. Ans.  (d) Coxiella burnetti Page 34
• Most resistant of all is prions; which is killed only by
Ref: Essentials of Medical Microbiology – Apurba Sastry –

ƒ 0.5% hypochlorite for 2 hr


Page 32 ƒ

ƒ 1N sodium hydroxide for 1 hour


• Method for sterilization of milk is pasteurization
ƒ

ƒ Autoclaving 134°C for 1-1.5 hours


• Methods: Holder method – 63°C for 30 minutes


ƒ

• Flash method – 72°C for 20 seconds followed by rapid


44. Ans.  (a) Benzalkonium chloride


cooling to 13° or lower)
• All non sporing pathogen are killed except Coxiella

Ref: Essentials of Medical Microbiology – Apurba Sastry –
burnetti which may survive Page 36
• Low level disinfectant – Quaternary ammonium
39. Ans.  (b) 100°C for 20 minutes

compounds (Benzalkonium chloride)


Ref: Essentials of Medical Microbiology – Apurba Sastry – • Intermediate level disinfectant – Isopropyl alcohol,

Page 33 Phenol
• Tyndallization also called as intermittent sterilization

• High level disinfectant – Glutaraldehyde, Formaldehyde,

– steaming at 100°C for 20 min for three consecutive Hydrogen peroxide, Chlorine
days
45. Ans.  (b) Gamma radiation
40. Ans.  (a) Geobacillus stearothermophilus Ref: Essentials of Medical Microbiology – Apurba Sastry –
Ref: Essentials of Medical Microbiology – Apurba Sastry – Page 41
Page 40 • Best method for sterilizing critical medical and surgical

instruments which are not damaged by heat, steam,


Hot air oven Clostridium tetani non toxigenic strain
pressure or moisture is STEAM
Autoclave Geobacillus stearothermophilus • Best method for metal instruments and surgical blade –

Filtration Brevundimonas diminuta, Serratia Gamma radiation (especially packed items)


Ionising radiation Bacillus pumilus
46. Ans.  (d) Beta propiolactone
Ethylene oxide Bacillus globigi
Ref: Essentials of Medical Microbiology – Apurba Sastry –
Plasma Geobacillus stearothermophilus,
Page 40
sterilization Bacillus subtilis subsp.,niger
• BPL (beta propiolactone – 0.2%) – active against all

41. Ans.  (b) 0.3 um microorganisms including spores


• Disadvantage: Low penetrating power, Carcinogenic
Ref: Essentials of Medical Microbiology – Apurba Sastry –

• Not used for fumigation


Page 35

• Used for inactivation of vaccines


• HEPA – High efficiency particulate air filters – removes


99.97% of particles that have a size of 0.3 um or more 47. Ans.  (c) Non critical device
• ULPA filters – ultra low particulate/penetration air –

remove from the air at least 99.999% of dust, pollen, Ref: Essentials of Medical Microbiology – Apurba Sastry –
mold, bacteria and airborne particles with a size of Page 42
0.12um or larger
Spaulding’s Examples
42. Ans.  (a) Glutaraldehyde classification
Critical device Surgical instruments, cardiac and urinary
Ref: Essentials of Medical Microbiology – Apurba Sastry –
catheters, Implants, eye and dental
Page 37
instruments
• 2% cidex – glutaraldehyde – helps in disinfection of

endoscopes and cystoscopies Semi critical Respiratory therapy equipments,


• Ortho-phthalaldehyde: used for sterilization of

device anaethesia equipments, endoscopes,
endoscopes and cystoscopies; laryngoscopes, probes
• Advantages of Ortho-phthalaldehyde over cidex:

Non critical BP cuff, ECG electrodes, Bed pans,
ƒ Does not require activation
ƒ
device Crutches, Stethescope, Thermometer
ƒ Low vapour property
Non critical Surfaces of medical equipments,
ƒ

ƒ Better odour
environmental Examination table, computers
ƒ

ƒ More stable during storage


ƒ

surfaces 37
ƒ Increased mycobactericidal activity
ƒ
Unit 1     General Microbiology

48. Ans.  (a) 0.22um 52. Ans.  (d) UV rays


Ref: Essentials of Medical Microbiology – Apurba Sastry – Ref: Essentials of Medical Microbiology – Apurba Sastry –
Page 35 Page 36
• Membrane filters: most widely used for bacterial
• • Ionizing radiations – include X rays, Gamma rays, Cobalt

filtration 60 source
• Size – 0.22um; removes most of the bacteria; allows
• • Excellent agent for sterilization; also called as cold

viruses to pass through sterilization


• Filters of size 0.45um – retain coliform bacteria in water
• • Used for sterilization of

microbiology; ƒ Disposable plastic supplies, disposable rubber or


ƒ

• 0.8um filters – used to remove airborne microorganisms


• plastic syringes, infusion sets and catheters
in clean rooms and for the production of bacteria free ƒ Catgut sutures, bone and tissue grafts, adhesive
ƒ

gases. dressings
ƒ Irradiation of food
ƒ

49. Ans.  (c) Hydrogen peroxide • Biological control: Bacillus pumilus


Ref: Essentials of Medical Microbiology – Apurba Sastry –


53. Ans.  (a) Hydrogen peroxide
Page 38
• Hydrogen peroxide – high level disinfectant; chemical

Ref: Ananthanarayan and Paniker’s Textbook of Micro­
sterilant biology – 10th ed – Page 35
• Used to disinfect ventilator, soft contact lenses,
• Plasma sterilisation:
tonometer biprisms • Plasma is the fourth state of matter

• Used for plasma sterilization


• • Using a radio frequency energy – an electromagnetic

• Environmentally safe; not carcinogenic nor mutagenic


• field is created
• Hydrogen peroxide vapours are introduced into this –

50. Ans.  (a) Filtration that generates free radicals – plasms


Ref: Essentials of Medical Microbiology – Apurba Sastry – • Used to sterilise arthroscopes, urethroscopes

Page 35
54. Ans.  (b) Cover with a cloth/material
• Filters – Depth and membrane filters

• Filtration used for



Ref: WHO Guidelines on Drawing Blood: Best Practices in
ƒ Sterilization of sera, sugar and antibiotic solutions
ƒ
Phlebotomy.
ƒ Separation of toxins and phages from bacteria
ƒ WHO guidelines on blood spillage management state the
ƒ To obtain bacteria free filtrates of clinical samples for
ƒ following steps:
virus isolation • Wear gloves

ƒ Purification of water
ƒ • Use disposable paper towels to absorb as much of the

body fluids as possible.


51. Ans.  (d) Mycobacterium tuberculosis • Wipe the area with water and detergent until it is visibly

Ref: Essentials of Medical Microbiology – Apurba Sastry – clean.


Page 39 • Saturate the area again with sodium hypochlorite 0.5%

(10 000 ppm available chlorine). This is a 1:10 dilution


• Quaternary ammonium compounds – cationic disinfec-

of 5.25% sodium hypochlorite bleach, which should be


tants prepared daily.
• Kill Gram positive bacteria more than Gram negative

• Rinse off the tongs, brush, and pan, under running water

bacteria and place to dry.


• Does not kill spores or M. tuberculosis

• Remove gloves and discard them.


• Wash hands carefully with soap and water, and dry


thoroughly with single-use towels.


• Record the incident in the incident book if a specimen

was lost, or persons were exposed to blood and body


fluids.

38
Culture Media and 4
Culture Methods
CULTURE MEDIA y Concentration of agar:

y
 Solid agar preparation: Japanese agar – 2%, New Zealand


y Microorganisms need proper growth constituents for its agar 1.2%
y
multiplication.  Semisolid agar – 0.5%


y Culture media is prepared for meeting their requirements and  Solid agar to inhibit swarming by Proteus – 6%
y

also helpful in a way using their properties for identification.
y Media is divided into solid and liquid media:
Peptone Water
y
 Liquid media is mainly used for bulk cultures of organisms

as needed for vaccine production or industrial purposes. y It is the basal liquid medium

y
 Solid media is agar based, where agar acts as solidifying y It is prepared by peptone + NaCl + distilled water

y

agent. Gelatin was once used as solidifying agent. But it y It is the basal media for preparation of all media
y
gets melted at very low temperature (24°C) and cannot y It is helpful in culture of organisms and to check for motility
y
withstand autoclaving. testing
y It is used in indole test
y
Table 1:  Examples of various culture media 
Basal Solid Medium
Culture media Example
type y Nutrient broth–meat extract + peptone + NaCl
y
y Nutrient agar–Nutrient broth + agar
Liquid BHI broth, peptone water, nutrient broth
y
y Promotes the growth of most of the non-fastidious bacteria
y
Solid Nutrient agar, blood agar, chocolate agar y It is the media for most of the biochemical tests, to study
y
Simple Nonnutrient agar, nutrient agar the morphology of colony and to isolate the pure culture of
organisms.
Complex TCBS agar
Synthetic or Hank’s balanced salt solution
defined
Enriched Blood agar, BHI agar, Chocolate agar, Todd
Hewitt broth
Enrichment Tetrathionate broth, Selenite F medium
Selective Salmonella Shigella agar, Thayer-Martin
medium, LJ medium
Indicator MacConkey agar
Differential Mannitol salt agar, MacConkey agar, CLED
Transport Stuart’s transport medium
Anaerobic media Thioglycollate medium, RCM
Figure 1: Nutrient agar plate showing colonies (Courtesy: Dr
Deepika K, NG Hospital and Research Centre, Coimbatore)
Agar-Agar
y It is a long chain polysaccharide
Blood Agar
y
y It is prepared from seaweeds
y
y Agar as such has no nutritive value; it just acts as solidifying y It is an example for enriched medium
y
y
agent y It is prepared by adding sterile blood to nutrient agar – then
y
y It does not interfere with the growth of any bacteria melted and cooled to 50°C for solidification
y
y The melting temperature of agar is 95°C and it usually sets at y Concentration of blood that is added is usually around 5–10%
y
y
42°C y Usually sheep blood is added; other bloods are of human and
y
horse.
Differential Media
Unit 1     General Microbiology

Remember y MacConkey agar and CLED agar comes under this category
• Blood agar helps to differentiate hemolysis and separate
y

y It differentiates between two groups of bacteria by using


certain organisms:
y

ƒ Partial or alpha hemolysis a Greenish discoloration


ƒ
an indicator, which changes the color of the colonies of a
surrounding the colonies—E.g. Pneumococci, Viridans particular group of bacteria but not the other group
streptococci y Both agar media differentiate lactose and non-lactose
y

ƒ Complete or beta hemolysis–Zone of complete clearing of


ƒ fermenters
blood around the colonies due to complete lysis of RBC’s y Both media inhibit the swarming of proteus
y

– E.g. Beta-hemolytic Streptococci, Staphylococcus aureus y Pseudomonas colonies are seen as greenish color in both
ƒ No or gamma hemolysis: no color change E.g. Enterococcus
y

y The main difference between MacConkey agar and CLED agar:


ƒ Target hemolysis – a narrow zone of complete hemolysis
y

 CLED has additional advantage of supporting growth


ƒ

followed by wide zone of incomplete hemolysis Eg: Clos-




tridium perfringens of GPC like Staphylococcus, Streptococcus and Candida


species.

Remember
• MacConkey agar is prepared by following ingredients

(PLANTS):
ƒ Peptone
ƒ

ƒ Lactose
ƒ

ƒ Agar ƒ

ƒ Neutral red (indicator)


ƒ

ƒ Sodium taurocholate
ƒ

ƒ Waterƒ

Figure 2: Blood agar (Courtesy: Dr Deepika K, NG Hospital and


Research Centre, Coimbatore, Tamil Nadu)

Table 2: Examples of bacteria for their respective hemolysis


Hemolysis Examples
Alpha hemolysis Streptococcus pneumoniae, Viridans streptococci
Beta hemolysis Streptococcus pyogenes, Staphylococcus aureus
Gamma Enterococcus (Remember: Enterococcus
hemolysis can produce alpha, beta and gamma (no)
hemolysis)
Target hemolysis Clostridium perfringens

Chocolate Agar
y It is a heated blood agar, prepared by adding 5–10% of sheep
Figure 4: MacConkey agar with LF (pink) colonies (Courtesy: CDC)
y

blood to the molten nutrient agar at 70°C, so that the RBCs


will be lysed and the content of the RBCs will be released–
changes the color from red to Brown
y Supports growth of fastidious bacteria. E.g. Haemophilus
y

influenzae

40 Figure 3: Chocolate agar (Courtesy: Dr. Deepika K, NG Hospital and Figure 5: MacConkey agar with NLF (Colorless) colonies
Research Centre, Coimbatore, Tamil Nadu) (Courtesy: CDC)
Löwenstein-Jensen Media Transport Media

Chapter 4     Culture Media and Culture Methods


y It is used for primary isolation of Mycobacterium species
y
y Some organisms are so fragile and needs to be cautious while
y

y Also promotes the growth of Nocardia species


y
transporting the specimens to isolate those organisms
y The ingredients are mineral salts, asparagine, glycerol,
y
E.g. Neisseria gonorrhoeae, Chlamydia trachomatis
malachite green and egg y Media that are used in such conditions are called as transport
y

y Since egg is used as the solidifying agent—autoclaving


y

media
cannot be done
y Inspissation is the method of sterilization for LJ media
Table 3:  Examples for transport media
y

y Mycobacterium tuberculosis produce—rough, tough, buff


y

colored dry irregular colonies Streptococcus Pike’s medium


y Mycobacterium bovis produces smooth, white, emulsifiable
Neisseria • Amie’s medium
y

colonies.

• Stuart’s medium

Vibrio cholerae • Venkatraman Ramakrishnan medium


• Autoclaved sea water


• Cary Blair medium


Shigella and Buffered glycerol saline


Salmonella Cary-Blair medium
Chlamydia Dacron or Rayon swab in sucrose phosphate
trachomatis glutamate buffer or M4 media 
Viral specimens • Eagle’s minimum essential medium

• Viral transport medium


Blood Culture Liquid Media


Figure 6: LJ media (Courtesy: Dr Vanathi S, KAP Viswanathan y Isolation of organisms from blood needs special media and
y

Government Medical College, Trichy, Tamil Nadu) inoculation


y Once the vein is selected, the skin area need to be cleaned
y

Anaerobic Media with 70% isopropyl alcohol


y Venipuncture is done–blood withdrawn and inoculated in the
y Robertson cooked meat medium - Growth of anaerobes
y

liquid broth
y

changes the medium into:


y For adults  10–20 mL of blood
 Red color—Saccharolytic
y

y For children  1–5 mL of blood




 Black color—Proteolytic
y

y Blood is mixed with the medium in the ratio of 1:10




y Thioglycollate broth with hemin and vitamin K


y

y Blood culture media available in automated systems are:


y

y Anaerobic broth
y

 BHI broth
y

y Smith Noguchi medium




 Tryptic soy broth


y

 Columbia broth
Media for Special Use


 Brucella broth


y Sabouraud’s dextrose agar – For isolation of fungi


y
 Thioglycollate broth


y Cation adjusted Mueller-Hinton agar (MHA) – used for


y
y Media should be incubated for up to 7 days to rule out
y

antimicrobial susceptibility testing brucellosis


y CHROM agar - for speciation of Candida based on the color
y

produced by the species Table 4:  Types of culture media and its purpose
Type of culture media Purpose
Chemically defined Growth of chemo and photo
media autotrophs
Complex media Growth of chemoheterotrophs
Reducing media Growth of obligate anaerobes
Selective media Suppression of unwanted microbes
and enhancing the growth of
pathogen
Differential media Differentiation of colonies from
other organisms
Enrichment media Suppression of unwanted microbes
Figure 7: CHROMogenic medium – Each species produces and enhancing the growth of
different colors (Courtesy: Image from Author’s own thesis work) pathogen to detectable levels within 41
short period of time (liquid media)
Anaerobic Culture Methods
Unit 1     General Microbiology

y McIntosh Fildes Jar: It is a metallic jar with a lid which has


y

an inlet and outlet. This tube will suck the oxygen inside.
Aluminum pellets coated with palladium serve as catalyst.
The oxygen inside will be replaced with nitrogen and H2 and
CO2.
y GasPak: It is a commercially available envelope with pre
y

packed mixture that liberates H2 and CO2 replacing oxygen


when kept inside the anaerobic jar.
y Pre-reduced anaerobic system
y

y Anaerobic chamber
y

y Robertson cooked meat medium


y

y Thioglycollate broth with hemin and vitamin K


y

Effectiveness of anaerobiosis can be checked by the following


methods:
y Chemical indicator: Reduced methylene blue – remains
y

colorless in anaerobic conditions, but turns blue on exposure


Figure 8: Blood culture bottles (left – uninoculated BHI broth; to oxygen.
right – broth inoculated with blood) (Courtesy: Dr Vanathi S, KAP y Biological indicator: Pseudomonas incubated along with
y

Viswanathan Government Medical College, Trichy, Tamil Nadu) other plates. Absence of Pseudomonas which is an obligate
aerobe – indicates perfect anaerobiosis.
CULTURE METHODS
Automated Culture Systems
y Helps in detection of bacterial growth. Based on production
Table 5: Types of culture methods
y

of CO2 as the end product of metabolism of bacteria. Sensors


Streak culture Surface plating method routinely used for will detect this. E.g.: BACTEC and BacT Alert
isolation of organisms y Identification systems that uses the biochemical
y

characteristics or metabolism property of bacteria as the


Stroke culture For pure growth of organism used for slide mechanism to detect. E.g.: VITEK, Phoenix
agglutination or other diagnostic test y MALDI-TOF: Matrix assisted laser desorption/ionization-
y

Lawn or carpet For antimicrobial susceptibility testing time of flight mass spectrometry – This identifies the bacteria
culture based on a unique protein that is present in the cell.
Stab culture For biochemical reactions
Remember
Pour plate culture For quantitative estimation of bacteria. Tests used for identification of bacteria:
E.g. urine culture
• IMViC tests: Indole test, Methyl red test, Voges Proskauer

Liquid culture For blood culture and sterility testing test, Citrate test
• Kovac’s reagent used in indole test

• TSI: Triple sugar iron agar contains glucose, lactose and


sucrose. Its indicator is phenol red


• Catalase test: For differentiation of Staphylococcus and

Streptococcus
• Urease test: Medium used is Christensen's urease medium

Figure 9: Colonies grown by streak culture in culture plate


(Courtesy: Dr M Gomathi, Department of Microbiology, Figure 10: Battery of biochemical reactions done for a culture
Government Chengalpattu Medical College, Tamil Nadu) isolate (Courtesy: Dr M Gomathi, Department of Microbiology,
42
Government Chengalpattu Medical College, Tamil Nadu)
Chapter 4     Culture Media and Culture Methods
Kovac’s reagent Kovac’s method
Reagent that is used in Indole test: After adding the reagent in A type of Oxidase test; especially done for Neisseria colonies. A
peptone water culture—if red colored ring is produced—then it strip of filter paper soaked with oxidase reagent is placed and
means indole positive organims. colonies are smeared over it. Within ten seconds, if the smeared
area turns violet – it means oxidase positive

Catalase test Coagulase test


To differentiate between Staphylococci and Streptococci (in GPC) To differentiate between Staphylococcus aureus and CONS
• Also helpful to identify GNB Two types:
• All species in Enterobacteriaceae are catalase positive except (i) Slide coagulase–Clumping factor or bound coagulase
Shigella dysenteriae type 1 (ii) Tube coagulase–Free coagulase
Hydrogen peroxide is added in a slide (Slide catalase test) or tube Few colonies are taken and mixed with rabbit or human plasma;
(Tube catalase test) – colonies are taken in a wooden stick and Coagulase enzyme reacts with coagulase reacting factor in the
placed over H2O2 – If the organism produces catalase – it splits the plasma – binds to prothrombin – converts fibrinogen into fibrin – a
H2O2 and releases hydrogen and nascent oxygen; which in turn visible clot is formed
produces air bubbles

Figure 11: Tube catalase test (note the bubbles seen)


(Courtesy: Dr M Gomathi, Government Chengalpattu Medical College, Tamil Nadu)

43
MULTIPLE CHOICE QUESTIONS
Unit 1     General Microbiology

1. Simple basal media is:  (Recent Pattern July 15) 15. In a patient with UTI, CLED (Cystine, lactose electrolyte
a. Simple nutrient agar deficient) media is preferred over MacConkeys media
b. Alkaline peptone water because: (AIIMS May 16)
c. Glucose broth a. It is a differential medium
d. Blood agar b. It inhibits swarming of Proteus
2. Which of the following is a differential medium:  c. Promotes growth of pseudomonas
 (Recent Pattern July 15) d. Promotes growth of staphylococcus aureus and Candida
a. MacConkey agar 16. Which anticoagulant is used when blood is sent for
b. Nutrient agar blood culture:  (Recent Pattern Dec 13)
c. Deoxycholate citrate agar a. Sodium citrate b. EDTA
d. Selenite F broth c. Oxalate d. SPS
3. Which is enrichment media: (Recent Pattern Dec12) 17. In blood culture the ratio of blood to reagent is 
a. Selenite F broth b. Chocolate media  (Recent Pattern Dec 14)
c. Meat extract media d. Egg media a. 1:5 b. 1:20
4. Chocolate agar is an example of:  c. 1:10 d. 1:100
 (Recent Pattern Aug 13) 18. Specific reason to disallow the sample for culture
a. Enriched medium b. Enrichment medium  (Recent Pattern Dec 13)
c. Selective medium d. Transport medium a. Sample brought within 2 hours of collection
5. Blood agar is an example of:  (Recent Pattern Dec 12) b. Sample brought in sterile plastic container
a. Enriched media b. Indicator media c. Sample brought in formalin
c. Enrichment media d. Selective media d. Sample obtained after cleaning the collection site
6. Loffler’s medium is:  (MH 11) 19. Which of the following transport media is used for
a. Indicator medium b. Selective medium Streptococcus?
c. Enrichment medium d. Enriched medium a. Amies medium b. Stuart`s medium
7. Selenite F broth is an enrichment media for:  c. Pike`s medium d. Cary Blair medium
 (Recent Pattern Dec 16) 20. Which concentration of agar is used to inhibit swarming?
a. Salmonella b. Shigella a. 1% b. 2%
c. E. coli d. Campylobacter c. 0.5% d. 6%
8. DCA media used in differentiation of which infection: 21. Which of the following is an example of Differential
 (Recent Pattern Nov 15) media?
a. Salmonella b. Staphylococcus aureus a. CLED agar b. DCA agar
c. H. influenza d. Bordetella c. XLD agar d. LJ medium
9. Example of selective medium is:(Recent Pattern July 16) 22. Identify the following culture method?
a. LJ medium b. Blood agar
c. Selenite F broth d. Chocolate agar
10. In nutrient agar conc. of agar is: (Recent Pattern Dec 12)
a. 1% b. 2%
c. 3% d. 4%
11. Robert Koch assistant advised him to use agar instead
of gelatin for solidifying culture media for cultivation of
bacteria as: (Recent Pattern Dec 14)
a. Agar has more nutrition
b. Gelatin melts at 37 °C
c. Gelatin is not easily available
d. Agar is cheaper
12. Medium for growth of anaerobic bacteria: 
 (Recent Pattern Dec 12)
a. SN medium b. LJ medium
c. Blood agar d. None of the above a. Stroke culture b. Stab culture
13. Lactose fermentation is seen in: (Recent Pattern Dec 13) c. Lawn culture d. Streak culture
a. Blood agar b. Chocolate agar 23. Indicator of anaerobiosis in GasPak system?
c. MacConkey agar d. LJ medium a. Staphylococcus aureus
14. Triple iron sugar medium contains all, except: b. Pseudomonas aeruginosa
 (Recent Pattern Dec 15) c. Bacteroides fragilis
a. Lactose b. Sucrose d. Clostridium perfingens
44 c. Glucose d. Maltose
24. Identify the following biochemical test? 27. Advantage of CLED agar over MacConkey agar is:

Chapter 4     Culture Media and Culture Methods


a. Supports the growth of Gram positive bacteria
b. Prevent the swarming of Proteus
c. Differentiates LF and NLF
d. Helpful in diagnosis of UTI
28. Indole positive organism is:
a. Escherichia coli
b. Klebsiella pneumoniae
c. Pseudomonas aeruginosa
d. Shigella sonnei
29. Which of the following type of hemolysis is seen in
Pneumococci?
a. alpha hemolysis b. beta hemolysis
a. Oxidase test b. Coagulase test c. No hemolysis d. Alpha prime hemolysis
c. Catalase test d. Urease test 30. Loeffler's serum slope is used for the isolation of:
25. Identify the following culture media? a. Clostridium tetani
b. Haemophilus influenza
c. Streptococcus pneumoniae
d. Corynebacterium diphtheria
31. Diffusible pigment is produced by:
a. Pseudomonas aeruginosa
b. Staphylococcus aureus
c. Enterococcus sp.,
d. Staphylococcus albus
32. Which of the following organism(s) can not be cultured
in artificial media:  (PGI May 2018)
a. Mycobacterium leprae
b. Klebsiella rhinoscleromatis
c. Rhinosporidium seeberi
d. Pneumocystis jirovecii
e. Aeromonas hydrophila
a. Blood agar b. Chocolate agar 33. Which organism shows pitting in blood agar? 
c. MacConkey agar d. BHI agar  (PGI Nov 2018)
26. Bacteriological loop is made up of: a. Eikenella corrodens b. Streptococcus pyogenes
a. Nichrome wire b. Copper wire c. Pasteurella multocida d. Haemophilus influenzae
c. Ferric wire d. Bronze wire e. Pseudomonas stutzeri

ANSWERS AND EXPLANATIONS


1. Ans.  (a) Simple nutrient agar 3. Ans.  (a) Selenite F broth
Ref: Ananthanarayan and Paniker’s Textbook of Microbiol- Ref: Ananthanarayan and Paniker’s Textbook of Microbiol-
ogy – 10th ed – Page 40 ogy – 10th ed – Page 40
• Simple or basal media – E.g. Nutrient broth/agar
• • Enrichment media are used to suppress commensal

• Consists of peptone, meat extract, sodium chloride,


• bacteria—it allows the growth of only pathogenic
water and agar bacteria
• Percentage of agar used is 2%
• • Examples are tetrathionate broth – for Salmonella and

Selenite F broth—for Shigella


2. Ans.  (a) MacConkey agar
4. Ans.  (a) Enriched medium
Ref: Ananthanarayan and Paniker’s Textbook of Microbiol-
ogy – 10th ed – Page 42 Ref: Ananthanarayan and Paniker’s Textbook of Microbiol-
• Differential media helps to differentiate the organism

ogy – 10th ed – Page 40
based on a property • Some organisms need the exact nutritional requirements

• Examples are MacConkey medium and CLED



for their growth
• MacConkey medium helps to differentiate lactose

• Hence basal media are enriched with either blood,

45
fermenting and non-lactose fermenting colonies serum or egg—called as enriched medium
• When blood is added to nutrient agar and heated—it
Unit 1     General Microbiology

10. Ans.  (b) 2%


gives chocolate agar
• Chocolate agar is helpful for the growth of H. influenzae,

Ref: Ananthanarayan and Paniker’s Textbook of Microbiol-
N. meningitidis, N. gonorrhoeae and Pneumococcus ogy – 10th ed – Page 40
• Already explained in Q.1
5. Ans.  (a) Enriched media

Ref: Ananthanarayan and Paniker’s Textbook of Microbiol- 11. Ans.  (b) Gelatine melts at 37°C
ogy – 10th ed – Page 40 Ref: Ananthanarayan and Paniker’s Textbook of Microbiol-
• Already explained in Q.4
• ogy – 10th ed – Page 40
• Gelatin was initially used as solidyfiing agent – but the
6. Ans.  (b) Selective medium

limitation is it gets melted at 42°C


Ref: Ananthanarayan and Paniker’s Textbook of Microbiol- • Agar has no nutritive value

ogy – 10th ed – Page 243 • Agar melts at 98°C and usually sets at 42°C depending

• Loeffler's serum slope is a selective medium for



on the agar concentration. This has an additional
Corynebacterium diphtheriae advantage because we are using higher temperatures in
• Selective medium helps for the growth of specific

autoclaving
organism
12. Ans.  (a) SN medium
7. Ans.  (b) Shigella Ref: Ananthanarayan and Paniker’s Textbook of Microbiol-
Ref: Ananthanarayan and Paniker’s Textbook of Microbiol- ogy – 10th ed – Page 46
ogy – 10th ed – Page 40 • Anaerobic broth is prepared by adding fresh animal

• Already explained in Q.3



tissues in broth – tissues used are rabbit kidney, spleen,
testes or heart (Smith- Noguchi medium)
8. Ans.  (a) Salmonella
13. Ans.  (c) MacConkey agar
Ref: Ananthanarayan and Paniker’s Textbook of Microbiol-
ogy – 10th ed – Page 303 Ref: Ananthanarayan and Paniker’s Textbook of Microbiol-
ogy – 10th ed – Page 42
• DCA – Deoxycholate citrate agar medium
• Already explained in Q.2

• Fecal samples should be plated directly on the selective



medium called DCA to isolate Salmonella and Shigella


14. Ans.  (d) Maltose
9. Ans.  (a) LJ medium Ref: Ananthanarayan and Paniker’s Textbook of Microbiol-
Ref: Ananthanarayan and Paniker’s Textbook of Microbiol- ogy – 10th ed – Page 51
ogy – 10th ed – Page 42 • TSI: Triple sugar iron agar. It contains glucose, lactose

• Lowenstein-Jensen medium – used for primary isolation and sucrose. Indicator is phenol red
• TSI is an example for composite media i.e. able to bring

of Mycobacterium species •

• Also promotes the growth of Nocardia sp



out many properties
• The ingredients are mineral salts, asparagine, glycerol,
15. Ans.  (d) Promotes growth of Staphylococcus aureus

malachite green and egg


and Candida
• Since egg is used as the solidifying agent – we cannot do
• The main difference between MacConkey agar and

autoclaving •

• Inspissation is the method of sterilization for LJ media



CLED: CLED has additional advantage of supporting
growth of GPC like Staphylococcus, Streptococcus and
Candida species.

16. Ans.  (d) SPS


Ref: Bailey and Scott T.B of diagnostic microbiology – 12th
ed – Page 712
• SPS: Sodium polyanethol sulfonate is the anticoagulant

used in blood culture medium


• It has additional property of anticomplementary activity,

inhibits lysozyme activity, interferes with phagocytosis


and inactivates aminoglycosides
Figure 12: LJ media (Courtesy: Dr Vanathi S, KAP Viswanathan
Government Medical College, Trichy, Tamil Nadu)
46
17. Ans.  (c) 1:10 ƒ Bacteriophage typing

Chapter 4     Culture Media and Culture Methods


ƒ

ƒ For bacterial antigens and vaccines


Ref: Bailey and Scott T.B of Diagnostic Microbiology – 12th
ƒ

ed – Page 712 23. Ans.  (b) Pseudomonas aeruginosa


• Blood culture medium – 50–100 mL
Ref: Essentials of Medical Microbiology–Apurba Sastry–

• Blood added correspondingly is 5–10 mL


Page 50

•• Ratio is 1:10
•• Why we have to dilute? • Effectiveness of anaerobiasis can be checked by the

ƒ To neutralize the anti-complementary or anti-


ƒ
following methods:
microbial activity ƒ Chemical indicator: Reduced methylene blue–
ƒ

ƒ To match the body surface area – culture


ƒ
remains colourless in anaerobic conditions, but turns
blue on exposure to oxygen
18. Ans.  (c) Sample brought in formalin ƒ Biological indicator: Pseudomonas incubated along
ƒ

with other plates. Absence of Pseudomonas which is


Ref: Bailey and Scott T.B of Diagnostic Microbiology – 12th
an obligate aerobe–indicates perfect anaerobiasis
ed – Page 712
• Formalin kills all the microorganisms including spores
• 24. Ans.  (c) Catalase test
• Hence specimens that are coming in formalin bottles
Ref: Essentials of Medical Microbiology–Apurba Sastry–

are not accepted for microbiology laboratory.


Page 55
19. Ans.  (c) Pike`s medium • Catalase test is used to differentiate between Staphylo-

coccus (Catalase positive) and Streptococcus (Catalase


Ref: Essentials of Medical Microbiology–Apurba Sastry–
negative)
Page 45
• Also positive for members of families Enterobacteria-

• Streptococcus–Pike`s medium

ceae, Vibrio
• Neisseria–Amie`s medium and Stuart`s medium

• False positive: Since blood has catalase colonies from

• Vibrio cholera–Venkatraman Ramakrishnan medium,



blood agar result in false positive reactions. Use of iron
autoclaved sea water, Cary Blair medium wire or loop for picking up colonies may also produce
• Shigella, Salmonella–Buffered glycerol saline, Cary Blair

false positive results


medium • Nutrient agar is the ideal medium to perform the test

20. Ans.  (d) 6% 25. Ans.  (b) Chocolate agar


Ref: Essentials of Medical Microbiology–Apurba Sastry– Ref: Essentials of Medical Microbiology–Apurba Sastry–
Page 41 Page 45
Concentration of agar: • It is heated blood agar, prepared by adding 5–10% of

• Solid agar preparation: Japanese agar–2%, New Zealand



sheep blood to the molten nutrient agar at 70°C, so that
agar 1.2% the RBCs will be lysed and the content of the RBCs will
• Semisolid agar–0.5% be released–changes the colour from red to BROWN
• Supports growth of fastidious bacteria E.g.: Haemophilus

• Solid agar to inhibit swarming by Proteus–6% (Agar



Melting temperature – 45°C) influenza

21. Ans.  (a) CLED agar 26. Ans.  (a) Nichrome wire

Ref: Essentials of Medical Microbiology–Apurba Sastry– Ref: Essentials of Medical Microbiology–Apurba Sastry–
Page 32 Page 47
• Differential media: MacConkey agar, CLED agar

• Inoculation of specimen onto the culture media is

• It differentiates between two groups of bacteria by using



carried out with the help of loops and straight wires–
an indicator, which changes the colour of the colonies made up PLATINUM or NICHROME wires
of a particular group of bacteria but not the other group. • Platinum–costly–so not widely used

22. Ans.  (c) Lawn culture 27. Ans.  (a) Supports the growth of Gram positive bacteria

Ref: Essentials of Medical Microbiology–Apurba Sastry– Ref: Essentials of Medical Microbiology–Apurba Sastry–
Page 47 Page 46
• Lawn or Carpet culture: Uniformly thick surface growth • CLED agar–Cyteine Lactose Electrolyte Deficient media

• Differential medium similar to MacConkey agar


of the bacterium on the solid medium •

• Useful in:

• Advantages over Mac–less inhibitory and supports the

ƒ Antimicrobial susceptibility testing by disk diffusion


ƒ
growth of Gram positive bacteria and Candida (Except
beta hemolytic Streptococcus)
method
• All other options fit both for CLED and Mac

47
Unit 1     General Microbiology

28. Ans.  (a) Escherichia coli 31. Ans.  (a) Pseudomonas aeruginosa
Ref: Essentials of Medical Microbiology–Apurba Sastry– Ref: Essentials of Medical Microbiology–Apurba Sastry–
Page 56 Page 54
• Reagent used: Kovac's reagent
• • Pigment production is characteristic for certain bacteria

• Indole positive: Red coloured ring–Organisms positive:


• and helps in identification of bacteria
E. coli, Proteus vulgaris, Vibrio cholera, Klebsiella • Diffusible pigments: diffuse throughout the media plate

oxytoca ƒ E.g. Pseudomonas aeruginosa (Blue green colour)


ƒ

• Indole negative: Yellow coloured ring–Organisms: Kleb-


• • Non diffusible pigments: only colonies are coloured

siella pneumoniae, Proteus mirabilis, Pseudomonas, ƒ E.g. Staphylococcus aureus, Serratia marsescens
ƒ

Shigella, Salmonella etc.,


32. Ans.  (a,c,d) a. Mycobacterium leprae; c. Rhinosporidi-
29. Ans.  (a) Alpha hemolysis um seeberi; d. Pneumocystis jirovecii
Ref: Essentials of Medical Microbiology–Apurba Sastry– Ref: Ananthanarayan and Paniker`s T.B of microbiology –
Page 54 10th ed – page 372
• Partial or alpha hemolysis–Greenish discoloration
• Organisms that are non cultivable in culture media are:
surrounding the colonies-E.g. Pneumococci, Viridans • Mycobacterium leprae

Streptococci • Treponema pallidum


• Complete or beta hemolysis–Zone of complete clearing


• • Rickettsiae

of blood around the colonies due to complete lysis of • Chlamydiae


RBC`s–E.g.: Beta hemolytic Streptococci, Staph aureus • Rhinosporidium seeberi


• No or gamma hemolysis–no colour change


• • Pneumocystic jirovecii

E.g. Enterococcus
33. Ans.  (a and e) a. Eikenella corrodens; e. Pseudomonas
30. Ans.  (d) Corynebacterium diphtheria stutzeri
Ref: Essentials of Medical Microbiology–Apurba Sastry– Ref: https://jcm.asm.org/content/jcm/3/3/381.full.pdf
Page 45 • Colony morphologies help us to identify the organism

• Loeffler’s serum slope is an example for Enriched media


• but biochemical tests are must to confirm the organism
• It contains serum
• • Eg: Hemolysis seen in Streptococcus group

• Used for isolation of C. diphtheriae (growth occurs in


• • Pigment production seen only in certain bacteria like

4–6 hours) Pseudomonas aeruginosa, Serratia marsescens


• Pitting of colonies and corrosion are seen in

ƒ Eikenella corrodens
ƒ

ƒ Pseudomonas stutzeri
ƒ

48
Bacterial Genetics, Resistance 5
and Susceptibility Testing
BACTERIAL GENETICS y A plasmid called the fertility or F factor plays a major role in

y
conjugation in E. coli.
Gene transfer occurs in bacteria horizontally and vertically:
Vertical transmission is from parents to offsprings; Horizontal
transmission is done by three main methods namely: TRANSPOSONS
1. Transformation
y The chromosomes of prokaryotic and eukaryotic cells have

2. Transduction

y
some pieces of DNA that move around the genome. Such

3. Conjugation
movement is called transposition.

y DNA segments that carry the genes required for this process
Transformation
y
and consequently move about chromosomes are called as
y Transformation is a process of random uptake of free or mobile genetic elements or transposable elements or
y
naked DNA fragments from medium to bacterial cell. transposons
y Griffith experiment in Pneumococci—demonstrates this y It was first discovered in the 1940s by Barbara McClintock.
y
y Natural transformation has been studied in—Streptococcus,
y
y
Bacillus, Haemophilus, Neisseria, Acinetobacter and Pseudo-
monas. BLOTTING
y Blotting technique refers to a method of transferring nucleic
Transduction
y
acid or proteins from gel onto a carrier followed by their
y Transduction is defined as transmission of a portion of DNA detection by using specific nucleic acid probes
y
from one bacterium to another by bacteriophage. y Southern blotting is done for identification of DNA fragments
y
y Generalized transduction: Transfer any part of the donor by DNA-DNA hybridization technique.
y
bacterial genome to recipient bacteria. y Northern blotting is done for identification of RNA where the
y
y Restricted transduction: Only a particular part is transferred RNA mixture is separated by gel electrophoresis technique.
y
y Studied exclusively in lambda phage of E. coli y Western blotting is a technique used for the identification of
y
y
proteins; it is also called as immunoblotting. Protein antigen
Conjugation mixture is separated by a method SDS-PAGE. Western blot
y Conjugation is the transfer of genetic material from one bac- method is used in diagnosis of HIV.
y Eastern blot – modification of western blot used to analyze
y
terium to another by forming a mating through conjugation
y
tube proteins for post-translational modifications.
y Through this method, transfer of plasmids coding for multiple
y
drug resistance occurs (R factor) Table 1: Blotting techniques

y R factor is a plasmid with two components—RTF + r
y
determinants

PLASMID
y Plasmids are small double-stranded DNA molecules (dsDNA).
y
y They are usually circular, that can exist independently of host
y
chromosomes and are present in many bacteria.
y They have their own replication origins and so they replicate
y
on their own.
y An episome is a plasmid that can exist either with or without
y
being integrated into the host’s chromosome.
y Conjugative plasmid is a type that have genes for pili and
y
can transfer copies of themselves to other bacteria during
conjugation.
Unit 1     General Microbiology

Remember Types of PCR Characteristics


Applications of Recombinant DNA Technology qPCR Quantitative or Real time PCR: Measure the
• Production of vaccines quantity of PCR product while the process is
running (hence named as real time)

• Production of antigens used in diagnostic kits like ELISA


Detection is based on: Nonspecific fluorescent

• Production of proteins used in therapy like human growth


dyes and specific set of DNA probes

hormone, insulin etc.


• Transgenic animals
Nested PCR Two set of primers are used to increase the

• Gene therapy
specificity; more specific

Multiplex PCR More than one pathogen can be identified


POLYMERASE CHAIN REACTION using multiple primers

Definition
y PCR is a technique used in molecular biology where many
y

copies of DNA or a gene is amplified from a single nucleic acid.


y It was first discovered by Karry B Mullis in 1983
y

y DNA replication occurs in a cycle where the product of first


y

cycle become template for successive and it multiplies in


exponential phase.
Three main steps of PCR are:
y Denaturation – the two strands of DNA are melted at high
y

temperature to give separate strands.


y Annealing – temperature is lowered for the primers to anneal
y

to the strands
y Extension: Once the primer is attached to the target – replica-
y

tion starts with the help of added nucleotides


Table 2: Main steps of polymerase chain reaction

Steps of PCR Temperature Figure 1: Agarose gel electrophoresis of a PCR product (Bands
are seen – positive)(Courtesy: Dr M Gomathi, Government
Denaturation 94 - 96°C Chengalpattu Medical College, Tamil Nadu)

Annealing 60°C
NONAMPLIFICATION TESTS
Extension 72°C y Loop mediated isothermal assay (LAMP): In PCR, different
y

temperatures are used. Here at constant temperature the


y During the denaturation step, the enzyme that is added
y reaction is done. Hence there is no need of thermocycler.
should not get denatured. Hence, thermostable DNA poly- Primers used are called as loop primers.
merase is added – taq polymerase (obtained from Thermus y DNA microarray: DNA spots are coated on a microarray plate
y

Aquaticus) and this hybridize with the cDNA in the test sample—most
specific and can easily detects large number of nucleic acid
Table 3: Polymerase chain reaction: (Amplification test)

sequences in the target simultaneously.


Types of PCR Characteristics Sequencing based assays:
y Genetic mapping
Conventional PCR done with the help of thermocycler
y

y Next generation sequencing


PCR and PCR product is visualized by agarose gel
y

electrophoresis
ANTIMICROBIAL DRUG RESISTANCE
Reverse RNA is converted to dsDNA with the help Antimicrobial drugs have four main mechanism of action:
transcriptase of reverse transcriptase and then PCR is 1. Inhibition of cell wall synthesis

RT-PCR performed; Helpful for RNA viruses 2. Inhibition of cell membrane function

3. Inhibition of protein synthesis


Contd... 4. Inhibition of nucleic acid synthesis


50
Classification of Antimicrobial Drugs

Chapter 5     Bacteria Genetics, Resistance and Susceptibility Testing


Figure 2: Antimicrobial drugs

Beta Lactamases
y These are the enzymes produced by the bacteria to open the beta lactam ring in the penicillin group of antimicrobials
y

y They are called as penicillin destroying enzymes


y

y Beta lactamases are mainly produced by:


y

 Staphylococcus aureus


 CONS


 Enterococci sp.,


 Neisseria gonorrhoeae


 Haemophilus influenzae


 Moraxella catarrhalis


y Beta lactamases production is the most common mechanism of drug resistance


y

y Continuous mutations in due course have led to extended profile of resistance—Extended spectrum beta lactamases, AmpC beta
y

lactamases and Metallo beta lactamases.


y Mechanism of beta lactamase production:
y

 Inactivation of the penicillin through b-lactamase- or penicillinase-mediated hydrolysis of the b-lactam ring of the antibiotic.


 Alteration of the target- intrinsic resistance involving a lowering of the affinity or the amount of the PBPs


 Tolerance to the bactericidal effect of b-lactam antibiotics




y Detection methods:
y

 Penicillin disc diffusion test




 Penicillin broth microdilution test




 Penicillin zone edge test




 PCR for blaZ gene




51
Figure 3: Illustration demonstrating b-lactamase mechanism
Extended Spectrum Beta Lactamases: (ESBL) Metallo Beta Lactamase (MBL)
Unit 1     General Microbiology

y ESBLs are b-lactamases capable of conferring bacterial resis-


y y These enzymes confer resistance to all carbapenams (Imi-
y

tance to the penicillin’s, first-, second-, and third- generation penams, Meropenams, Ertapenams), all betalactams, amino-
cephalosporins and aztreonam (but not the cephamycins or glycosides and quinolones.
carbapenems) by hydrolysis of these antibiotics, and which
are inhibited by b-lactamase inhibitors such as clavulanic
acid.
Carbapenamase
y Carbapenamases are b-lactamases with versatile hydrolytic
y

Remember capacities.
y They have the ability to hydrolyze penicillins, cephalosporins,
ESBL producing organisms:
y

monobactams, and carbapenams.


• Escherichia coli
y Bacteria producing these b-lactamases may cause serious

• Klebsiella sp.
y

infections in which the carbapenamases activity renders


• Enterobacter sp.
many b-lactams ineffective.

• Proteus sp.
y KPC: Klebsiella pneumoniae carbapenamase is widely

• Salmonella sp.
y

spreading and nowadays recent isolates are even resistant to


y Detection methods:
y
colistin
 Screening test: Disk diffusion test


 Confirmatory test: Double disk diffusion test and Broth




microdilution test.

Figure 4: Antibiotic resistance of ESBL and MBL

Bush Jacoby classification of Beta lactamases: (Important groups)


Bush Jacoby Group Molecular Class Resistance To Characteristics
1 C Cephalosporins Greater hydrolysis of cephalosporins than
benzylpenicillin; hydrolyzes cephamycins

2a A Penicillins Greater hydrolysis of benzylpenicillin than cephalosporins

2be A Extended-spectrum Increased hydrolysis of oxyimino-β-lactams (cefotaxime,


cephalosporins, monobactams ceftazidime, ceftriaxone, cefepime, aztreonam)

2d D Cloxacillin Increased hydrolysis of cloxacillin or oxacillin

3a C Carbapenams Broad-spectrum hydrolysis including carbapenems but


not monobactams

52
Mechanism of Drug Resistance

Chapter 5     Bacteria Genetics, Resistance and Susceptibility Testing


Figure 5: Drug resistance gene transfer

Based on the mechanism—whether it is due to mutation or y MIC - Dilution tests – Agar dilution and broth dilution
y

transferred from commensals or neighbor bacteria—certain y Automated methods


y

characteristics are seen


Description
Table 4: Differences between mutational and

y Kirby Bauer method:


transferable drug resistance
y

 It is the conventional method- antibiotic disks are kept at




equal distance in a lawn culture of bacterium in MHA –


Mutational drug resistance Transferable drug resistance zone of inhibition is measured.
One drug resistance at a time Multiple drug resistance  Only five drugs need to be kept in 100 mm plate


 Equal spacing should be given


Low degree resistance High degree resistance


 Zone of inhibition is to be measured after full incubation




Can be overcome by high drug High doses ineffective period


doses  The interpretative guidelines is by – clinical laboratory


Drug combinations can prevent Combinations cannot prevent standards institute (CLSI)

Resistance does not spread Spreads to same or different


species
Mutants may be defective Not defective
Virulence may be low Virulence not decreased

Remember
MRSA active cephalosporins:
• Ceftaroline

• Ceftabiprole

ANTIMICROBIAL SUSCEPTIBILITY TESTING


y Mueller Hinton agar is the best medium to do antimicrobial
y

susceptibility testing of most of the bacteria Figure 6: Kirby Bauer method – disk diffusion method
y It almost supports growth of all non-fastidious bacteria.
y
(Courtesy: Dr M Gomathi, Chengalpattu Government Medical
College, Tamil Nadu)
Methods
y Kirby Bauer disk diffusion method
y
y Stokes method: MHA is divided into three parts. Test
y

y Stokes method organism inoculated on the central one third and control
y

53
y E test
y strain on upper and lower thirds of the plate
y Epsilometer test (E test): y MIC tests:
Unit 1     General Microbiology

y y

 A predefined antibiotic dilution level made at certain


  Detect the minimum inhibitory concentration of antimi-


concentration is impregnated in a strip (E strip) crobial at which the organisms are killed
 Commercially available
  Divided into agar dilution method and broth dilution


 Quantitative method of detecting MIC by using principles


 method
of both dilution and diffusion of antibiotic into the
medium

Figure 8: MIC test by broth dilution method (Serial dilutions


of drug are taken) (Courtesy: Dr M Gomathi, Chengalpattu
Government Medical College, Tamil Nadu)

Remember
Automated systems that can detect susceptibility:
• VITEK

• MGIT

Figure 7: E-test strip method (Courtesy: Dr M Gomathi, • Molecular method – PCR helps to detect the resistance

Chengalpattu Government Medical College, Tamil Nadu) genes.

54
MULTIPLE CHOICE QUESTIONS

Chapter 5     Bacteria Genetics, Resistance and Susceptibility Testing


Bacterial Genetics and Resistance 12. Unique feature of LAMP assay: (PGI Pattern 2017 )
a. Thermocycler is used
1. Mechanism of direct transfer of free DNA is:
b. Forward and Reverse primer are same
a. Transformation b. Conjugation
c. Primers are not used
c. Transduction d. Transposition
d. Constant temperature
2. Phage mediated transfer of DNA from one bacterium to
e. Drug resistance can be detected
another is called as:
13. Which of the following is a quantitative type of PCR?
a. Transformation b. Conjugation
(PGI Pattern 2017)
c. Transduction d. Transposition
a. Conventional PCR b. Multiplex PCR
3. Western blotting technique is used to detect:
c. Nested PCR d. Real time PCR
a. RNA b. DNA
e. qPCR
c. Proteins d. Nucleotides
14. Few of the following is/are amplication based molecular
4. Recombinant DNA technology is useful in all of the
methods except: (PGI Pattern 2017 )
following except:
a. Polymerase chain reaction
a. Vaccine production b. ELISA – antigen
b. Reverse transcriptase PCR
c. Transgenic animals d. Southern blot
c. GenXpert
5. Bacterial toxins that are coded by lysogenic phage
d. DNA microarray
except:
e. LAMP assay
a. Diphtheria toxin b. Cholera toxin
c. TSST d. Botulinum toxin C
6. F factor carries some chromosomal DNA, which is called Antimicrobial Testing
as: 15. Antimicrobial susceptibility testing is done in:
a. F factor b. Hfr a. Nutrient agar b. Mueller-Hinton agar
c. RTF d. F’ factor c. Blood agar d. Chocolate agar
7. Transmission of resistance by means of R factor is by: 16. E test is a method based on:
a. Transformation b. Conjugation a. Diffusion
c. Transduction d. Transposition b. Dilution method
8. Identify the correct statement about mutational drug c. Both diffusion and dilution
resistance? d. Automated
a. Multiple drug resistance at the same time 17. MRSA is mediated by:
b. High degree resistance a. Plasmid b. Chromosome
c. Resistance can be overcome by combination of drugs c. Transposons d. None
d. Virulence not decreased 18. Identify the following antimicrobial susceptibility
9. ESBL includes all except: method?
a. Resistance to penicillins
b. Resistance to 1, 2 and 3 generation cephalosporins
c. Resistance to monobactam
d. Resistance to carbapenems
10. Why do bacteria contain restriction enzymes?
(PGI Pattern 2017)
a. To cleave RNA for incorporation into ribosome 


b. To extend the length of bacterial chromosomes 


c. To prevent foreign DNA from incorporating into a


bacte rial genome

 


d. To process the exons from prokaryotic mRNA 


e.  To proteolytically cleave nuclear promoters


11. Which is/are statements are correct regarding trans-


posons? (PGI Pattern 2017)
a. They encode enzymes that degrade the ends of
bacterial chromosomes
b. They are short sequences that often mediate
antimicrobial resistance
c. Small transposons are called as insertion sequences a. Kirby Bauer method b. Modified Kirby Bauer
d. They are short sequences of RNA that integrates with method
nucleic acid of host c. Stokes method d. E test 55
e. They cut nucleic acids at palindromic sequences
19. Which of the following is an anti-pseudomonal penicil- 20. Most common cause of acquired resistance is due to:
Unit 1     General Microbiology

lin? a. Overuse and misuse of antimicrobials


a. Carbenicillin b. Evolution of resistant strains
b. Cloxacillin c. Inherent characteristics
c. Oxacillin d. Mutation
d. Methicillin

ANSWERS AND EXPLANATIONS


1. Ans.  (a) Transformation 5. Ans.  (c) TSST
Ref: Essentials of Medical Microbiology – Apurba Sastry – Ref: Essentials of Medical Microbiology – Apurba Sastry –
Page 70 Page 73
• Transformation is a process of random uptake of free or
• Phage mediated toxins are:
naked DNA fragments from medium to bacterial cell. • Diphtheria toxin

• Griffith experiment – Pneumococci – demonstrates this


• • Cholera toxin

• Natural transformation has been studied in – Streptococ-


• • Verocytotoxin of E. coli

cus, Bacillus, Haemophilus, Neisseria, Acinetobacter and • Streptococcus pyrogenic exotoxin (SPE) – A and C

Pseudomonas • Botulinum toxin C and D


2. Ans.  (c) Transduction 6. Ans.  (d) F’ factor


Ref: Essentials of Medical Microbiology – Apurba Sastry – Ref: Essentials of Medical Microbiology – Apurba Sastry –
Page 71 Page 73
• Transduction is defined as transmission of a portion of
• • When the F’ factor reverts from the integrated to free

DNA from one bacterium to another by bacteriophage. state – it sometimes carry with it some chromosomal
• Generalized transduction: Transfer any part of the
• DNA from adjacent site of its attachment.
donor bacterial genome to recipient bacteria • Such an F factor which carries the chromosomal DNA is

• Restricted transduction: Only a particular part is


• named as F’ factor
transferred
• Studied exclusively in lambda phage of E. coli

7. Ans.  (b) Conjugation
Ref: Essentials of Medical Microbiology – Apurba Sastry –
3. Ans.  (c) Proteins
Page 73
Ref: Essentials of Medical Microbiology – Apurba Sastry – • Conjugation: Transfer of genetic material from one

Page 77 bacterium to another by forming a mating through


• Blotting technique refers to a method of transferring
• conjugation tube
nucleic acid or proteins from gel onto a carrier followed • Through this method, transfer of plasmids coding for

by their detection by using specific nucleic acid probes multiple drug resistance occurs
• Blotting techniques:
• • R factor is a plasmid with two components: RTF + r

ƒ Southern blot – DNA


ƒ determinants
ƒ Northern blot – RNA
ƒ

ƒ Western blot – Proteins


ƒ
8. Ans.  (c) Resistance can be overcome by combination of
ƒ Eastern blot – modification of western blot used to
ƒ
drugs
analyze proteins for post translational modifications Ref: Essentials of Medical Microbiology – Apurba Sastry –
Page 82
4. Ans.  (d) Southern blot
Ref: Essentials of Medical Microbiology – Apurba Sastry – Mutational drug Transferable drug
Page 76 resistance resistance
Applications of Genetic engineering/Recombinant DNA One drug resistance at a Multiple drug resistance
technology: time
• Production of vaccines
Low degree resistance High degree resistance

• Production of antigens used in diagnostic kits like ELISA


• Production of proteins used in therapy like human



Can be overcome by high High doses ineffective
growth hormone, insulin etc. drug doses
• Transgenic animals

Drug combinations can Combinations cannot
56 • Gene therapy

prevent prevent
contd...
• In PCR, different temperatures are used; Here at

Chapter 5     Bacteria Genetics, Resistance and Susceptibility Testing


Mutational drug Transferable drug •

constant temperature the reaction is done; Hence there


resistance resistance
is no need of thermocycler;
Resistance does not spread Spreads to same or different • Primers used are called as loop primers.

species • There are separate forward and reverse primers


Mutants may be defective Not defective • Mainly used for diagnosis of Mycobacterium

tuberculosis
Virulence may be low Virulence not decreased
13. Ans.  (d and e) Real time PCR; qPCR
9. Ans.  (d) Resistance to Carbapenems
Ref: Ananthanarayan and Paniker T.B of microbiology –
Ref: Essentials of Medical Microbiology – Apurba Sastry – 10th edition – Page 66
Page 83
• Quantitative or Real time PCR – Measure the quantity of
• Extended spectrum beta lactamases (ESBL)

PCR product while the process is running (hence named


• Resistant to all penicillins and 1st, 2nd, 3rd generation

as real time)
cephalosporins and monobactams • Detection is based on: Non specific fluorescent dyes and
• Remain sensitive to Carbapenems and cephamycins

specific set of DNA probes


10. Ans  (c) To prevent foreign DNA from incorporating into 14. Ans.  (d and e) DNA microarray; LAMP assay
a bacterial genome
Ref: Ananthanarayan and Paniker T.B of microbiology –
Ref: Jawetz – medical microbiology – 27th edition – Page 119 10th edition – Page 66
• Restriction enzymes (restriction endonucleases)

• Amplification tests:

provide bacteria helps to distinguish between their own ƒ PCR


ƒ

DNA and DNA from other biologic sources. ƒ RT PCR


• These enzymes hydrolyze DNA at restriction sites
ƒ

ƒ Real time PCR or qPCR


ƒ

determined by specific DNA sequences ranging from 4 ƒ Nested PCR


ƒ

to 13 bases. ƒ Multiplex PCR


• Restriction enzymes that recognize more than 10 bases
ƒ

• Non-amplification tests:

are useful for construction of a physical map and for ƒ Loop mediated isothermal assay (LAMP) – In PCR,
ƒ

molecular typing by Pulse field gel electrophoresis. different temperatures are used; Here at constant
temperature the reaction is done; Hence there is no
11. Ans.  (b and c) They are short sequences that often
need of thermocycler; Primers used are called as loop
mediate antimicrobial resistance; Small transposons
primers.
are called as insertion sequences
ƒ DNA microarray: DNA spots are coated on a
ƒ

Ref: Ananthanarayan and Paniker T.B of microbiology – microarray plate and this hybridise with the cDNA in
10th edition – Page 63 the test sample – most specific and can easily detects
• Transposons are short sequences of DNA that have

large number of nucleic acid sequences in the target
the ability to move around in a cut and paste manner simultaneously
between chromosomal and extra chromosomal DNA
molecules 15. Ans.  (b) Mueller-Hinton agar
• They are also called as jumping genes

Ref: Essentials of Medical Microbiology – Apurba Sastry –


• The two ends of transposons has inverted repeat

Page 82
sequences of nucleotides
• Mueller-Hinton agar is the best medium to do antimi-
• Small transposons (1-2kb) is called as insertion

crobial susceptibility testing of most of the bacteria


sequenes or IS
• It almost supports growth of all non-fastidious bacteria
• Transposons are not self replicating

• They depend on chromosomal or plasmid DNA for


16. Ans.  (c) Both diffusion and dilution


replication
• Resistance determining segments of the R factor evolved

Ref: Essentials of Medical Microbiology – Apurba Sastry –
from transposons–hence involved in antimicrobial Page 86
resistance • Epsilometer test (E test)

• Quantitative method of detecting MIC by using


12. Ans.  (d) Constant temperature

principles of both dilution and diffusion of antibiotic


Ref: Ananthanarayan and Paniker T.B of microbiology – into the medium
10th edition – Page 66
• Loop mediated isothermal assay (LAMP) – non amplifi-

cation method
57
17. Ans.  (b) Chromosome 19. Ans.  (a) Carbenicillin
Unit 1     General Microbiology

Ref: Essentials of Medical Microbiology – Apurba Sastry – Ref: Essentials of Medical Microbiology – Apurba Sastry –
Page 82 Page 79
• Methicillin Resistant Staphylococcus aureus (MRSA)
• • Anti-pseudomonal penicillin – Carbenicillin, Ticarcillin

• Target site of penicillin – PBP gets altered to PBP-2a


• and Piperacillin
• Altered protein do not bind to beta lactam antibiotics –

prevent them from inhibiting the cell wall synthesis 20. Ans.  (a) Overuse and misuse of antimicrobials
Ref: Essentials of Medical Microbiology – Apurba Sastry –
18. Ans.  (c) Stokes method
Page 78
Ref: Essentials of Medical Microbiology – Apurba Sastry – • Resistance – Intrinsic and Acquired

Page 85 • Acquired is mainly due to over use and misuse of


• Stokes method – MHA is divided into three parts


• antimicrobial agents
• Test organism inoculated on the central one-third and
• • Intrinsic resistance. E.g.: Gram-negative bacteria are

control strain on upper and lower thirds of the plate resistant to vancomycin

58
HIGH YIELDING FACTS TO BE REMEMBERED IN GENERAL MICROBIOLOGY

Chapter 5     Bacteria Genetics, Resistance and Susceptibility Testing


Father of microbiology Louis Pasteur
Father of medical microbiology Robert Koch
Side chain theory of antibody production Paul Ehrlich
Father of antiseptic surgery Joseph Lister
Contributions of Louis Pasteur Principles of fermentation, sterilization techniques, pasteurization,
Vaccines against anthrax and rabies, germ theory of disease, liquid
media
Who introduced solid media? Robert Koch
Koch’s phenomenon Guinea pigs already infected with TB bacillus develops a
hypersensitivity reaction when injected with tubercle bacilli or its
protein
Exception to Koch’s postulates Mycobacterium leprae, Treponema pallidum, Neisseria
gonorrhoeae
Who invented electron microscope? Ernst Ruska
Who described and coined the term phagocytosis? Élie Metchnikoff
Who discovered PCR? Kary B Mullis
Types of PCR Conventional PCR, Reverse transcriptase PCR (for RNA), real time
PCR (Quantitative)
Automated culture systems in diagnostic bacteriology BACTEC, BacT/Alert, ESP culture system
Bacterial identification systems MALDI TOF, VITEK 2, MicroScan Walkaway System
Automated system for cultivation of Mycobacterium tuberculosis Mycobacterial Growth Indicator Tube (MGIT)
GenXpert is used for Diagnosis of TB bacilli and for identification of Rifampicin
Resistance – based on real time PCR method
Three steps of PCR Denaturation – 95°C, Primer annealing – 55°C, Extension – 72°C
Solidifying agent in LJ media is Egg
How to sterilize LJ media? Inspissation
How to sterlize serum containing media Inspissation or Tyndallization
Newer agent for OT sterilization that has replaced formaldehyde is EcoShield
called as:
Examples for indicator and differential media MacConkey agar and CLED agar
Which culture method helps in quantitative estimation? Pour plate culture
Vaccine preparation is done by which culture method Liquid cultures – antigen is extracted
What are the antimicrobial susceptibility-testing methods? Disk diffusion, dilution, epsilometer (E test), Automated method,
molecular methods
Media that is used for AST Mueller-Hinton agar
The organism broth prepared should match which of the McFarland 0.5
McFarland standard unit for antimicrobial susceptibility testing
Which of the methods can calculate minimum inhibitory Agar dilution method, Broth dilution method, Epsilometer test
concentration?
Who described transposons? Barbara McClintock

59
e s
N o t
2

BACTERIOLOGY
Unit Outline

1 Chapter 6: Staphylococcus
Chapter 7: Streptococci
Chapter 8: Pneumococcus
Chapter 9: Neisseria
Chapter 10: Corynebacterium
Chapter 11: Bacillus
Chapter 12: Clostridium
Chapter 13: Enterobacteriaceae
Chapter 14: Vibrio
Chapter 15: Pseudomonas, Acinetobacter and
Burkholderia
Chapter 16: Haemophilus, Francisella and
Pasteurella
Chapter 17: Brucella and Bordetella
Chapter 18: Mycobacterium
Chapter 19: Spirochaetes
Chapter 20: Rickettsia and Chlamydia
Chapter 21: Helicobacter and Campylobacter
Chapter 22: Mycoplasma and Legionella
Chapter 23: Miscellaneous Bacteria
6
Staphylococcus
The most important organism coming under this genus are:
Toxins • Cytolytic toxins – α,β,γ,δ hemolysins
y Staphylococcus aureus


• Panton valentine leukocidin
y
y CONS (Coagulase negative staphylococci)


• Enterotoxin
y
 Staphylococcus epidermidis


• TSST (Toxic shock syndrome toxin)

 Staphylococcus saprophyticus


• Epidermolytic toxin

 Staphylococcus haemolyticus



 Staphylococcus lugdunensis
Protein A

STAPHYLOCOCCUS AUREUS y It binds to Fc terminal of IgG antibodies (except IgG3); it is a

y
B cell mitogen
• Gram-positive cocci y Coating of IgG antiserum with protein A – leads to agglutina-
y
tion when mixed with proper antigen – this reaction is called

• Arrangement: Clusters
as coagglutination test

• Key test for identification: Catalase test and Coagulase test

(Both positive)
Coagulase
y Two types of coagulase:
y
 Bound coagulase (also called as clumping factor)

 Free coagulase

y Coagulase test is helpful for speciation of Staphylococci
y
Coagulase test positive Staphylococcus aureus
Coagulase test negative CONS

y Two types of coagulase test are done:


y
 Slide coagulase test – detects bound coagulase

 Tube coagulase test – detects free coagulase

Figure 1: Gram-positive cocci in clusters
(Courtesy: CDC/ Dr Richard Facklam)
Pathogenesis
y Staphylococcus aureus causes wide variety of diseases that
y
includes diseases caused due to direct infection and due to
toxin secretion
y Main pathogenicity is because of certain virulence factors:
y
Table 1:  Virulence factors of Staph. aureus  
Cell wall factors • Peptidoglycan (made up of

polysaccharides)
• Teichoic acid

• Capsule (seen only in fresh strains)
Figure 2: Tube coagulase test

Surface proteins • Protein A
(Courtesy: Dr Prabha P, Kilpauk Medical College, Chennai,

• Clumping factor Tamil Nadu)

Enzymes • Coagulase
Hyaluronidase

• Lipid hydrolases

• Hyaluronidase This enzyme helps in the break down of surrounding tissues and

• Nuclease leads to spreading of infection
contd...

Toxins Table 2:  Clinical Spectrum of diseases caused by
Unit 2     Bacteriology

y Cytolytic toxins: It has four types of hemolysins and one


y
staphylococci
leucocidin:
Type Diseases
 Alpha hemolysin: this protein gets inactivated at 70°C and


reactivated at 100°C Skin and soft tissues •• Carbuncle


 Beta hemolysin: It is responsible for hot cold phenomenon

•• Furuncle
(hemolysis can be seen at 37°C) •• Impetigo
 Gamma hemolysin

•• Folliculitis
 Delta hemolysin

•• Mastitis
 Leucocidin: Also called as Panton Valentine toxin (PVL)

•• Cellulitis
– also called as synergohymenotropic toxins Muscle and bones • Septic arthritis (Most common cause is

y Enterotoxin:
y
Staphylococcus aureus in both children
 Food poisoning caused by Staphylococcus aureus is

and adults)
because of a preformed toxin named as Enterotoxin • Osteomyelitis (especially vertebral

 As this is preformed – the incubation period is very less



osteomyelitis)
– 1 to 6 hours after consuming the food – the symptoms • Pyomyositis

starts
Respiratory infections • VAP
 Food products responsible for these symptoms are – milk,

• Community acquired pneumonia




meat and milk products


(usually after influenza viral infection –


 Types of enterotoxins are: A, B, C1-3, D, E and H
post influenza pneumonia)


 Most common toxin responsible for food poisoning – Type




A Cardiac infections • Infective endocarditis in native valve


y Toxic shock syndrome toxin:


y
(left sided); prosthetic valve; and IV
 Certain Staphylococcus aureus strains which have a phage

drug users (right sided)
coding of Group I releases TSST toxin Blood borne • Septic shock

 This toxin has an association with tampon usage and other


Toxin mediated • Toxic shock syndrome


wound infections

• Food poisoning
 TSST causes systemic illness

• Staphylococcal scalded skin syndrome




 It leads to fever, hypotension, vomiting and diarrhea


 TSST – also named as Enterotoxin type F (or pyrogenic




exotoxin C) Laboratory Diagnosis


y Gram staining from the specimen shows – pus cells with
y

Gram-positive cocci in clusters


High Yield y In culture:
y

 In nutrient agar – strains produces golden yellow pigment




• TSST and Staphylococcal enterotoxins are super antigens



and oil paint appearance (Fig. 4)
 Pigmentation can be enhanced by adding milk in media or


Ludlam’s medium
y Exfoliative Toxin:
y
 In blood agar: some strains produces beta hemolysis (Fig. 5)


 Also called as epidermolytic toxin or Staphylococcal



 Selective medium – mannitol salt agar


scalded skin syndrome toxin (SSSS) y Unique battery of tests for S. aureus are:
y

 Two milder forms are seen:



 Coagulase test


Š Bullous impetigo
Š
 Heat stable thermo nuclease test


Š Pemphigus neonatorum
Š
 Phosphatase test


 Two severe forms are seen:



 DNAse test


Š Newborn illness – called as Ritter’s disease


Š
 Mannitol salt agar media – yellow colored colonies – it is


Š Older patients – TEN (Toxic epidermal necrolysis)


Š
the selective media for S. aureus
 Potassium tellurite agar – black colonies


 Gelatin liquefaction – positive




y Bacteriophage typing:
y

 Typing by phage done by pattern method




64
Chapter 6     Staphylococcus
Figure 5: Staphylococcus aureus showing hemolysis
(Courtesy: Dr Prabha P, Kilpauk Medical College Chennai,
Tamil Nadu)
Antimicrobial Susceptibility of
Staphylococcus Aureus

Figure 3: Species identification of Gram-positive cocci

Figure 4: Golden yellow pigmented Staphylococcus aureus


colonies (Courtesy: Dr J Pandian, Coimbatore Medical College,
Tamil Nadu)

Figure 6: Antimicrobial resistance in Staphylococcus aureus 65


Table 3:  Drug resistant strains of S. aureus Treatment of Staphylococcus Aureus
Unit 2     Bacteriology

Infections
Resistance Points
MRSA – Methicillin • MRSA can be detected by cefoxitin disk

Table 4:  Treatment of S. aureus
resistant S.aureus diffusion method
Type and susceptibility Treatment
• Cefoxitin acts as a surrogate marker for
of infection

Methicillin resistance
Penicillin sensitive S.aureus Penicillin
BORSA - Borderline • Some strains when screened by cefoxitin

resistant S.aureus disk diffusion method gets missed – but Penicillin resistant S.aureus Methicillin, Nafcillin, Oxacillin
they are resistant MRSA Vancomycin
• These organisms can be diagnosed by

Alternate drugs:
oxacillin screen agar Teicoplanin, daptomycin, linezolid,
VRSA – Vancomycin • Vancomycin resistance:

quinupristin/dalfopristin,
resistant S.aureus • VRSA – Vancomycin resistant

ceftobiprole
Staphylococcus aureus – has both van A VRSA Teicoplanin, daptomycin, linezolid,
and mec A genes quinupristin/dalfopristin
• VISA – Vancomycin intermediate
Skin infections (MRSA) Clindamycin, cotrimoxazole,

Staphylococcus aureus – Do not


doxycycline and linezolid
carry resistant genes; But due to the
presence of thick wall; antibiotics cannot Nasal carriers Mupirocin ointment
penetrate and thus leads to resistance
• VRSA can be detected by MIC method

COAGULASE NEGATIVE STAPHYLOCOCCI


• No vancomycin disk diffusion method
y CONS are the most common causes for prosthetic device

hVISA – • hVISA – heteroresistant VISA – This is


infections.
heteroresistance used to denote for heterogeneously y They are mostly present in normal human skin flora.
y

VISA resistant S.aureus – which likely progress


to VISA later
• Can be detected by: E test and

Staphylococcus Epidermidis
population analysis profile y Most commonly isolated CONS from clinical specimens
y

Inducible • When clindamycin is kept for checking



y As S. epidermidis is a skin commensal – it is important to
y

clindamycin susceptibility – it may be sensitive obtain the clinical history and to check for repeat isolation to
resistance • When an erythromycin disc is placed

term it as a pathogen.
near clindamycin disc – it shows a y Its virulence is mediated by biofilm production.
y

special D shaped flattening towards y This biofilm production helps the organism adhere to im-
y

erythromycin disc (D test) plants like prosthetic devices and shunts.


• In this case - clindamycin should be

y It causes infective endocarditis in prosthetic patients.
y

reported as resistant

Staphylococcus Saprophyticus
y Mostly causes UTI in sexually active females
y

y Unique identification feature is Novobiocin resistant


y

y DD: Staphylococcus xylosus which is an animal pathogen


y

that causes rarely human infections; it is also Novobiocin


resistant.

Figure 7: Cefoxitin disc test for MRSA detection


(Courtesy: Dr J Pandian, Coimbatore Medical College, Tamil Nadu)

66
MULTIPLE CHOICE QUESTIONS

Chapter 6     Staphylococcus
1. True about Staphylococcus aureus- 10. Staphylococcal scalded skin syndrome is caused by-
 (Recent pattern July 2016)  (Recent pattern Dec 2012)
a. Microaerophilic a. Hemolysin
b. Produce lemon yellow colonies b. Coagulase
c. Grows with 10% NaCl c. Enterotoxin
d. All are true d. Epidermolytic toxin
2. Mannitol salt agar is used for isolation of- 11. Staphylococcal pathogenicity is indicated by-
 (Recent pattern Nov 2015)  (Recent pattern Nov 2015)
a. Gonococcus b. Pneumococcus a. Coagulase positivity
c. Staphylococcus d. Pseudomonas b. Hemolysis
3. Most common site of Staphylococcus carrier- c. Lipoteichoic acid
 (Recent pattern Dec 2016) d. Endotoxin
a. Skin 12. Protein A of Staphylococcus binds to-
b. Nose  (Recent pattern Dec 2016)
c. Oropharynx a. IgA
d. Perineum b. IgG
4. Most common staphylococcal phage strain causing c. IgD
infection- (Recent pattern Dec 2013) d. IgE
a. 80/81 13. Synergohymenotropic toxins of Staphylococcus con-
b. 79/80 sists- (PGI Nov 2015)
c. 3A/3C a. α toxin
d. 69/70 b. b toxin
5. Oil paint appearance on nutrient agar is seen in- c. d toxin
 (Recent pattern Dec 2016) d. Panton-Valentine toxin
a. Streptococcus pyogenes
e. g toxin
b. Staphylococcus aureus
14. Incubation period of staphylococcal food poisoning
c. Bordetella pertussis
are- (PGI Nov 2015)
d. H. influenzae a. 4-6 hours b. 6-12 hours
6. Catalase positive novobiocin resistant bacteria c. 12-18 hours d. 18-24 hours
 (Recent pattern Dec 2016) 15. Enterotoxin responsible for most of the cases of food
a. Staphylococcus aureus poisoning by Staphylococcus-
b. Staphylococcus epidermidis  (Recent pattern June 2014)
c. Staphylococcus saprophyticus a. Type E b. Type C
d. None of the above c. Type B d. Type A
7. All of the following statements about Staphylococcus 16. Staphylococcus aureus causes-
aureus are true except- (All India 2010)  (Recent pattern Dec 2012)
a. Most common source of infection is cross infection a. Erythrasma b. Chancroid
from infected people c. Acne vulgaris d. Bullous impetigo
b. About 30% of general population is healthy nasal 17. The most common cause of scalded skin syndrome is-
carriers.  (Recent pattern Dec 2013)
c. Epidermolysin and TSS toxin are superantigens. a. Staphylococci b. Pneumococci
d. Methicillin resistance is chromosomally mediated. c. Enterococci d. Meningococci
8. To differentiate between Staphylococcus epidermidis 18. Staphylococcus aureus does not cause which of the
and Staphylococcus saprophyticus, which is used- following skin infection- (Recent pattern July 2016)
 (Recent pattern June 2014) a. Ecthyma gangrenosum
a. Coagulase test b. Bullous impetigo
b. Catalase test c. Botryomycosis
c. Novobiocin sensitivity d. Cellulitis
d. None of the above 19. Which organism causes toxic shock syndrome-
9. True about protein A of Staph aureus-  (Recent pattern Dec 2013)
 (Recent pattern June 2014) a. Pneumococcus
a. Causes opsonization b. E. coli
b. Binds to Fc part of IgG c. Staphylococcus aureus
c. Stimulate phagocytosis d. Enterococcus
d. T-cell mitogen 67
20. Which of the following organism is implicated in the 28. A patient developed cellulitis. Isolated organism is
Unit 2     Bacteriology

causation of botryomycosis- (Recent pattern Dec 2014) showing positive result in coagulase test. The causative
a. Staphylococcus aureus organism is- (AIIMS Nov 2016)
b. Staphylococcus albus a. Staph aureus

c. Pseudomonas aeruginosa b. Staph epidermidis


d. Streptococcus pneumoniae c. Strep pyogenes


21. All of the following are true about Methicillin resistance d. Pneumococcus

in MRSA, except- (All India 2011) 29. Catalase positive coagulase negative beta hemolytic
a. Resistance is produced as a result of altered PBP’s bacteria- (Recent pattern Nov 2014)
b. Resistance may be produced by hyperproduction of a. Strep pyogenes
beta lactamase b. Staph aureus
c. Resistance is primarily mediated/transmitted via c. Coagulase negative staph
plasmids d. Enterococci
d. Resistance may be missed at incubation temperature of 30. Staphylococcus differs from Streptococcus by-
37°C during susceptibility testing  (Recent pattern Dec 2014)
22. Blood culture is positive in which infection of Staph a. Coagulase test
lococcus aureus- (Recent pattern Dec 2012) b. Catalase test
a. TSS c. Phosphatase
b. SSSS d. Gram-negative
c. Infective endocarditis 31. Catalase positive beta hemolytic Staphylococcus-
d. Impetigo  (Recent pattern Dec 2012)
23. Staphylococcus oxacillin resistance is best detected by- a. S.aureus
b. S.epidermidis
 (Recent pattern June 2014) c. S.saprophyticus d.
None
a. Cefixime MIC 32. Staphylococcus aureus differs from Staphylococcus
b. Cefoxitin disc diffusion epidermidis by- (Recent pattern Nov 2014)
c. Oxacillin disc diffusion a. Is coagulase positive
d. Oxacillin agar b. Forms white colonies
24. Drug of choice for MRSA- (Recent pattern Dec 2013) c. A common cause of UTI
a. Penicillin G d. Causes endocarditis in drug addicts
b. Ceftriaxone 33. Most common cause of native valve endocarditis-
c. Vancomycin  (Recent pattern Dec 2013)
d. Cefazolin a. Staphylococcus aureus
25. All are true regarding resistance of penicillin in Staphy- b. Coagulase negative Staphylococcus
lococcus aureus, except- c. Streptococcus
 (Recent pattern Dec 2015) d. Enterococcus
a. Penicillinase production is transmitted by transduction 34. Test to differentiate staphylococci from micrococci-
b. Methicillin resistance is due to change in PBP  (Recent pattern Dec 2016)
c. Hospital strains mostly produce type D penicillinase a. Catalase test b. Coagulase test
d. Penicillinase production is plasmid mediated c. Novobiocin sensitivity d. Oxidation fermentation
26. A patient has prosthetic valve replacement and he de- 35. Acute bacterial prostatitis is caused by:
velops endocarditis 8 months later. Organism responsi-  (Recent pattern 2018)
ble is- (AIIMS Nov 2010) a. Enterococcus
a. Staph aureus b. Streptococcus viridans
b. Strep viridans c. Staphylococcus aureus
c. Staph epidermidis d. Proteus
d. HACEK 36. Regarding resistance in Staphylococcus aureus, is/are
27. Which of the following statements is most correct correct?  (PGI pattern May 2012)
regarding resistance to methicillin in MRSA- a. Plasmid mediated resistance is responsible for beta
 (Recent pattern 2011) lactamase production
a. Resistance is produced as a result of alteration in b. SCC mecA type IV is associated with hospital acquired
Penicillin Binding Protein (PBP) infections
b. Resistance is produced by production of beta lactamase c. mecA and mecC genes are responsible for resistance to
c. Resistance is mediated by plasmids nafcillin and methicillin
d. Expression of resistance is enhanced by incubating at d. Methicillin can be given to Vancomycin resistance
37°C during susceptibility testing strains
e. Resistance to other drugs occur in MRSA strains

68
ANSWERS AND EXPLANATIONS

Chapter 6     Staphylococcus
1. Ans.  (c) Grows with 10% NaCl 7. Ans.  (c) Epidermolysis and TSS toxin are superantigens
Ref: Ananthanarayan and Paniker’s Textbook of Microbiol- Ref: Ananthanarayan and Paniker’s Textbook of Microbiol-
ogy – 10th ed – Page 206 ogy – 10th ed – Page 204
Salt milk agar and salt broth (containing 8-10% NaCl) are • Staphylococcal enterotoxins and Toxic shock syndrome

used as selective media for isolation of Staphylococcus toxin (TSST-1) are superantigens.
aureus. • Epidermolysin is not a superantigen.

2. Ans.  (c) Staphylococcus 8. Ans.  (c) Novobiocin sensitivity


Ref: Ananthanarayan and Paniker’s Textbook of Microbiol- Ref: Ananthanarayan and Paniker’s Textbook of Microbiol-
ogy – 10th ed – Page 206 ogy – 10th ed – Page 208
https://microbiologyinfo.com/mannitol-salt-agar-for-the- • Both are coagulase test negative (CONS). S. saprophyticus

isolation-of-staphylococcus-aureus/ is novobiocin resistant which its specific identification


Mannitol Salt Agar (MSA) is used as a selective and feature is.
differential medium for the isolation and identification
of Staphylococcus aureus from clinical and non-clinical 9. Ans.  (b) Binds to Fc part of IgG
specimens. Ref: Ananthanarayan and Paniker’s Textbook of Microbiol-
• It contains mannitol and NaCl (7.5%)

ogy – 10th ed – Page 203


• S. aureus produces yellow colonies due to mannitol
• Protein A of Staphylococcus aureus is anti-phagocytic,

fermentation. •

anti-complementary and a B-cell mitogen.


3. Ans.  (b) Nose • It binds to Fc portion of any IgG molecule, leaving Fab

portion free to combine with its specific antigen. This


Ref: Ananthanarayan and Paniker’s Textbook of Microbiol-
property is made use of in co-agglutination test.
ogy – 10th ed – Page 205
About 10–30% of healthy persons carry Staphylococci in 10. Ans.  (d) Epidermolytic toxin
their nose and about 10% in perineum and hair. Vaginal
Ref: Ananthanarayan and Paniker’s Textbook of Microbiol-
carriage is about 5–10%.
ogy – 10th ed – Page 204.
4. Ans.  (a) 80/81 • Exfoliative (epidermolytic) toxin, also known as ET or

exfoliatin is responsible for Staphylococcal scalded skin


Ref: Essentials of Medical Microbiology – Apurba Sankar
Sastry 10th ed – Page 218 syndrome (SSSS).
• The severe form of SSSS is known as Ritter’s disease
• Phage type 80/81 is most commonly associated with

in newborn and toxic epidermal necrolysis in older


outbreaks in hospitals. It is known as the epidemic strain


patients.
of S. aureus.
11. Ans.  (a) Coagulase positivity
5. Ans.  (b) Staphylococcus aureus
Ref: Ananthanarayan and Paniker’s Textbook of Microbiol-
Ref: Ananthanarayan and Paniker’s Textbook of Microbiol-
ogy – 10th ed – Page 203
ogy – 10th ed – Page 206
• Staphylococcus aureus grows confluently on nutrient

• Coagulase is an enzyme which brings about clotting of

agar slope presenting characteristically with an oil-paint human and rabbit plasma. It acts with coagulase reacting
appearance. factor present in plasma, binding to prothrombin and
converting fibrinogen to fibrin.
6. Ans.  (c) Staphylococcus saprophyticus • It is the test for Staphylococcal pathogenicity.

Ref: Ananthanarayan and Paniker’s Textbook of Microbiol- 12. Ans.  (b) IgG
ogy – 10th ed – Page 208
Ref: Ananthanarayan and Paniker’s Textbook of Microbiol-
• Staphyloccus saprophyticus is a coagulase negative
ogy – 10th ed – Page 203

Staphylococci which can be identified by its resistance


to novobiocin. • Protein A of Staphylococcus binds to Fc portion of any

• They are present in the normal human skin and



IgG molecule, leaving Fab portion free to combine with
periurethral area and can cause urinary tract infection its specific antigen.
in young, sexually active females. • This property is made use of in co-agglutination test.

69
13. Ans.  (d) PVL and (e) g toxin
Unit 2     Bacteriology

20. Ans.  (a) Staphylococcus aureus


Ref: Ananthanarayan and Paniker’s Textbook of Microbiol- Ref: Ananthanarayan and Paniker’s Textbook of Microbiol-
ogy – 10th ed – Page 204 ogy – 10th ed – Page 401 y and 604
• Panton-Valentine Leukocidin (PVL) is a two component
• • Botryomycosis is a mycetoma like lesion caused by

toxin composed of S and F components. Such bi- Staphylococcus aureus and other pyogenic bacteria.
component membrane active toxins have been grouped
as synergohymenotropic toxins. 21. Ans.  (c) Resistance is primarily mediated/transmitted
• PVL, gamma hemolysin and Luk-M toxins of Staph

via plasmids
aureus belongs to this group. Ref: Ananthanarayan and Paniker’s Textbook of Microbiol-
14. Ans.  (a) 4-6 hours ogy – 10th ed – Page 202
• The mechanism of Methicillin resistance in Staphyococci
Ref: Ananthanarayan and Paniker’s Textbook of

(MRSA) is regulated by a set of chromosomal genes


Microbiology – 10th ed – Page 204
called Staphylococcal cassette chromosomal mec genes
• The incubation period of Staphylococcal food poisoning

(SCC mec).
is 2-6 hours.
• Enterotoxin is responsible for the manifestations which
• 22. Ans.  (c) Infective endocarditis
include nausea, vomiting and diarrhea.
Ref: Ananthanarayan and Paniker’s Textbook of Microbiol-
• The toxin is a preformed toxin and therefore the
ogy – 10th ed – Page 593

incubation period is short.


• TSS and SSSS are toxin mediated diseases and impetigo

15. Ans.  (d) Type A is localized infection of skin and subcutaneous tissue. So
Ref: Ananthanarayan and Paniker’s Textbook of Microbiol- the infecting organism is not cultivable from blood.
ogy – 10th ed – Page 204 • Infective endocarditis due to Staphylococcus aureus

is an endovascular infection and so is blood culture


Type A is responsible for most of the cases. positive.
• There are 8 antigenic types of enterotoxin named A, B,

C1-3, D, E and H. 23. Ans.  (b) Cefoxitin disk diffusion


16. Ans.  (d) Bullous impetigo Ref: Performance standards for antimicrobial susceptibility
testing – CLSI M100S27 (2017) –Page 132
Ref: Ananthanarayan and Paniker’s Textbook of Microbiol-
ogy – 10th ed – Page 204 • Cefoxitin resistance testing indicates mecA mediated

oxacillin resistance.
• Milder forms of Staphylococcal scalded skin syndrome
• Cefoxitin is a surrogate marker for MRSA; hence disk

are pemphigus vulgaris and bullous impetigo.


diffusion with cefoxitin helps in the identification of


• Epidermolytic toxin is responsible for this condition.
MRSA strains.

17. Ans.  (a) Staphylococci


24. Ans.  (c) Vancomycin
Ref: Ananthanarayan and Paniker’s Textbook of
Microbiology – 10th ed – Page 204 Ref: Ananthanarayan and Paniker’s Textbook of Microbiol-
ogy – 10th ed – Page 208
• The most common cause of scalded skin syndrome is
• Vancomycin is the drug of choice for MRSA.

Staphylococcus causing Staphylococcal scalded skin •

syndrome (SSSS). • In VRSA, linezolid can be used.


• Refer answer no.16 for further details.


25. Ans.  (c) Hospital strains mostly produce type D

18. Ans.  (a) Ecthyma gangrenosum penicillinase

Ref: Ananthanarayan and Paniker’s Textbook of Microbiol- Ref: Ananthanarayan and Paniker’s Textbook of Microbiol-
ogy – 10th ed – Page 204 and 321 ogy – 10th ed – Page 202
• Ecthyma gangrenosum is a skin lesion caused by

• Penicillinases are b-lactamases which are plasmid

Pseudomonas aeruginosa. mediated. They can be transmitted by transduction or


• The other three conditions given as options B, C and D

conjugation.
are caused by Staphylococci. • Alteration in the penicillin binding protein PBP2a and

changes in bacterial surface receptors reduces binding


19. Ans.  (c) Staphylococcus aureus affinity of b-lactam antibiotics to cells.
Ref: Ananthanarayan and Paniker’s Textbook of Microbiol- • This is the mechanism imparting resistance to all

ogy – 10th ed – Page 204 and 205 b-lactam antibiotics and has been named MRSA.
• Toxic shock syndrome (TSS) is a common toxin

70 mediated Staphylococcal disease.


• TSS is mediated by Toxic shock syndrome toxin (TSST).

Chapter 6     Staphylococcus
26. Ans.  (c) Staph epidermidis 30. Ans.  (b) Catalase test
Ref: Ananthanarayan and Paniker’s Textbook of Microbiol- Ref: Ananthanarayan and Paniker’s Textbook of Microbiol-
ogy – 10th ed – Page 208 ogy – 10th ed – Page 207
• Staphylococci are catalase positive unlike Streptococci
• S. epidermidis is the most common cause of early •

which are negative.


prosthetic device associated infections like endocarditis.


• Catalase test is used to differentiate Staphylococci from

Classification of Most common agent Streptococci.


endocarditis associated
31. Ans.  (a) S aureus
Native valve Staphylococcus aureus
Ref: Ananthanarayan and Paniker’s Textbook of Microbiol-
endocarditis
ogy – 10th ed – Page 202 (Table 21.1) and 206
Prosthetic valve Early prosthetic valve endocarditis
• Staphylococcus aureus produces beta hemolysis on
endocarditis (within 12 months of valve

blood agar.
replacement): S. epidermidis
• The hemolysis is marked when incubated under 20-25%
Late prosthetic valve

carbon dioxide.
endocarditis (After 12 months
of valve replacement): Viridans 32. Ans.  (a) Is coagulase positive
streptococci
Ref: Ananthanarayan and Paniker’s Textbook of Microbiol-
Endocarditis in IV Staphylococcus aureus
ogy – 10th ed –Page 202 (Table 21.1) and 208.
drug abusers
 able 4:  Differences between S.aureus and
T
Subacute endocarditis Viridans streptococci S.epidermidis
Test S.aureus S.epidermidis
27. Ans.  (a) Resistance is produced as a result of alteration
in Penicillin Binding Protein (PBP) Coagulase + –

Ref: Ananthanarayan and Paniker’s Textbook of Microbiol- Mannitol fermentation + –


ogy – 10th ed – Page 202 Heat-stable nuclease + –
• Resistance is produced as a result of alteration in
Phenolphthalein + –

Penicillin Binding Protein (PBP).


• Methicillin resistance is chromosomally acquired.

phosphatase
• Incubation at 30°C allows detection of methicillin

Beta hemolysis on + –
resistance. blood agar
28. Ans.  (a) Staphylococcus aureus Golden yellow pigment + –
Ref: Ananthanarayan and Paniker’s Textbook of Microbiol- Sensitivity to + –
ogy – 10th ed – Page 204 lysostaphin
• Staphylococcus aureus is the most common cause of

• S. epidermidis is a common cause of stitch abscess. It

localized pyogenic infections. can cause cystitis, central line associated blood stream
• It is coagulase test positive.

infection and infection of mechanical devices. It causes
endocarditis in drug addicts.
29. Ans.  (c) Coagulase negative staph
Ref: Ananthanarayan and Paniker’s Textbook of Microbiol- 33. Ans.  (a) Staphylococcus aureus
ogy – 10th ed – Page 207 Ref: Essentials of Medical Microbiology – Apurba Sankar
• Staphylococci are catalase positive unlike Streptococci

Sastry 10th ed – Page 215
which are negative. Staphylococcus aureus is the most common cause of native
• Catalase test is used to differentiate Staphylococci from

valve endocarditis followed by Streptococci and CONS.
Streptococci.
• Among Staphylococci, Stapahylococcus aureus is
• 34. Ans.  (d) Oxidation fermentation
coagulase positive whereas the other Staphylococcal
Ref: Ananthanarayan and Paniker’s Textbook of Microbiol-
species that are coagulase negative are grouped together
ogy – 10th ed – Page 209
as Coagulase negative Staphylococci (CONS). CONS are
catalase positive and coagulase negative. Micrococci are Gram-positive cocci which are catalase
positive. They can be mistaken for S. aureus. They are
oxidative in Hugh-Leifon’s oxidation-fermentation test.
71
Staphylococci and utilize glucose oxidatively and ferment • Resistance to drugs occur in Staph.aureus by various
Unit 2     Bacteriology

glucose as well. Micrococci cannot ferment glucose but mechanisms


can only utilization glucose oxidatively. • Beta lactamase production is the most common and its

plasmid mediated, transferred by conjugation. It means


35. Ans.  (c) Staphylococcus aureus resistance to penicillin, ampicillin, Piperacillin
Ref: Bailey T.B of surgery – Chapter 77 – Page 1357 • Resistance to nafcillin and methicillin – MRSA – due to

SCC mec A gene and newly identified mec C gene;


• The usual most common organism responsible for acute

• SCC – Staphylococcal cassette chromosome – Type IV


prostatitis is Escherichia coli – responsible for community acquired infections and


• Other organisms responsible are: (in the order)

Type I, II, III, VI, VIII are responsible for hospital ac-
ƒ Staphylococcus aureus
ƒ

quired infections
ƒ Staphylococcus albus
ƒ

• VISA – Vancomycin intermediate sensitive staph aureus


ƒ Streptococcus fecalis
ƒ

• VRSA – Vancomycin resistant staph aureus


ƒ Neisseria gonnorrhea
ƒ

• Drug of choice for MRSA – severe infections – Vancomy-


ƒ Chlamydia trachomatis
ƒ

cin
• Drug of choice for VRSA – Linezolid, Teicoplanin
36. Ans.  (a) Plasmid mediated resistance is responsible

for beta lactamase production, (c) mecA and mecC


genes are responsible for resistance to nafcillin and
methicillin
Ref: Jawetz Microbiology – page 204 – 27th edition

72
7
Streptococci
y Many species comes under family of Streptococci
y
y The species are classified based on certain properties as given
y
below.
Table 1:  Classification of Streptococci
Based on hemolysis in blood • Alpha hemolytic - Viridans

agar group
• Beta hemolytic

• Gamma (no hemolysis) –

Enterococcus group
Beta hemolytic is classified A to V (Except I and J)
based on the carbohydrate Group A – Eg: Streptococcus
antigen (Lancefield grouping) pyogenes Figure 1: Classification of streptococcus based on hemolysis  
Group A Streptococci – 80 types
classified based on M protein
(Griffith typing)

Table 2:  Different species of streptococci

Species Lancefield Type of Laboratory


group hemolysis tests
S. pyogenes A Beta Bacitracin
sensitive; PYR
test positive;
Ribose not
fermented
S. agalactiae B Beta CAMP test
positive;
hippurate
hydrolysis
Viridans – Not typed Alpha Optochin
S. mitis resistant
S. mutans
S. salivarius
S. sanguinis
Enterococcus sp D No hemolysis- PYR positive;
gamma Growth in
6.5% NaCl Figure 2: Strains of β-hemolytic streptococci
Non- D No hemolysis No growth in
enterococcus sp -gamma 6.5% NaCl
STREPTOCOCCUS PYOGENES (GAS) Diseases caused by Streptococcus Pyogenes
Unit 2     Bacteriology

• Gram-positive cocci

Necrotizing Fasciitis
• Arrangement: In pairs and chains

y Also called as hemolytic streptococcal gangrene
y

• Key test for identification: Catalase test - negative, Beta type



y Caused by M types 1 and 3 forming pyrogenic exotoxin A
y

of hemolysis, Bacitracin test – sensitive. y Because of extensive necroses: it is called as flesh eating
y

bacteria
y Vancomycin is the DOC
Virulence Factors
y

y Capsule: Helps in the inhibition of phagocytosis


y
Acute Rheumatic Fever
y Carbohydrate antigen: This antigen is responsible for cross
y y It is a complication or post sequel of acute pharyngitis
y

reactivity leading to autoimmunity y Occurs after 2-3 weeks of infection


y

y Protein antigens: (MTR)


y y This is mainly an immunological mediated injury because of
y

 M proteins – Griffith typing; helpful in virulence


 cross reactivity
 T proteins – not associated with virulence
 y M protein present in S. pyogenes cross reacts with the antigens
y

 R proteins – not associated with virulence


 in the heart and joint tissues – leading to injury
y Pili – made up of M proteins and lipoteichoic acid
y y Most important strains that cause RF are M types 5,18 and
y

24
Toxin Secreted by Streptococcus Pyogenes Acute Glomerulonephritis
y Erythrogenic, Dick or scarlational toxin:
y
y It is a sequela of impetigo or sometimes pharyngitis
y

 Intradermal injection into susceptible individuals



y Post-impetigo – nephritogenic strains commonly involved
y

 Produce an erythematous reaction (Dick test)



are M types 49, 52, 53, 57, 58, 59, 60, 61
 Effect of toxin is seen by induction of fever

y Postpharyngitis – nephritogenic strains are M types
y

 Three types are there– A, B, C



3,4,12,21,25
 A and C – Phage coded; B –Chromosomal


Table 4:  Antigenic cross reactivity


y Hemolysins – Streptolysin O and S.
y

 Streptolysin O: Antibody to this appears post infections;


 S.pyogenes Human
ASO titer helps to diagnose; oxygen labile Hyaluronic acid in capsule Human synovial fluid
 Streptolysin S: Responsible for hemolysis seen around
Cell wall proteins Myocardium


streptococcal colonies on the surface of blood agar plates;


oxygen stable Group A carbohydrates Cardiac valves
y Streptokinase – Fibrinolysin – It is responsible for breaking
y

Cytoplasmic membrane antigens Vascular intima


down of fibrin barrier and spread of infections
y Deoxyribonucleases (Streptodornase, DNAse) – depolymer-
y
Peptidoglycans Skin antigens
ization of DNA; helps to liquefy the thick pus; four types A,
B, C, D; type B is the most antigenic; Demonstration of anti Lab Diagnosis of Streptococcus Pyogenes
DNAse B antibody – retrospective diagnosis of S.pyogenes
infection – especially in skin infections
(GAS) Infection
y Hyaluronidase helps in the spread of infection along the
y
y Gram staining – shows Gram-positive cocci in pairs and chains
y

intercellular spaces

Table 3:  Diseases caused by Streptococcus pyogenes


Suppurative diseases Nonsuppurative diseases
• Pharyngitis – most common
• • Acute rheumatic fever

cause is S. pyogenes in • Poststreptococcal


children glomerulonephritis
• Scarlet fever

• Skin and soft tissue


infections:
ƒ Impetigo
ƒ

ƒ Pyoderma
ƒ

Figure 3: Gram-positive cocci in chains (Courtesy: CDC)


ƒ Cellulitis
y Culture needs to be done in blood agar for identification of
ƒ

ƒ Necrotizing fasciitis
y

type of hemolysis – Streptococcus pyogenes belongs to beta


ƒ

• Toxic shock syndrome


type of hemolysis;

• Puerperal sepsis
y Key test is Bacitracin disk (0.04U) test – all strains of

• Abscesses
y

74 S.pyogenes are sensitive to bacitracin.



y Emm typing: Nonserological typing method for Group A y It causes infections in the neonates to those who has born to

Chapter 7     Streptococci
y y

Streptococci – based on M protein – Types M (1-81) – done by women who carry GBS in their vagina
molecular based typing method y Two different types of infections are seen:
y

y Antibody detection:
y  Early onset disease (presented <12 hours of birth)


 ASO titer (antibodies against Streptolysin O) – rise of titer


  Late onset disease (after 7 days and up to 3 months of


occurs after 3 to 4 weeks of exposure to infection; Helpful birth) – meningitis is the most common manifestation
in diagnosis of rheumatic fever y Type III capsular polysaccharide strains are the most
y

common cause for neonatal infections


High Yield y Unique characteristic of GBS is production of a phospholipase
y

that is responsible for CAMP test


Why ASO titer is not helpful in postskin infections that cause y CAMP Test:
y

glomerulonephritis?  Group B Streptococci, when streaked at right angles to




• Lipids that are present in the skin inactivates the Streptolysin


a streak of Staphylococcus aureus on a blood agar plate


O antigen; hence antibodies are not produced actively – after overnight incubation – an arrow head type of
• In such situation – ideal choice of diagnosis is anti DNAse B

hemolysis can be seen in the intersecting area


 This was originally described by Christie, Atkins and


Rheumatic fever ASO titer (>200 todd units) – detected by Munch-Petersen (CAMP) test
latex agglutination test
Poststreptococcal Anti DNAse B antibody (>300 units); and
glomerulonephritis anti hyaluronidase

Figure 5: CAMP test (Courtesy: CDC/ Dr Richard Facklam)


y Drug of choice: Penicillin; Empirical therapy with ampicillin
y

and gentamicin;
y Prophylaxis need to be given for women carrying GBS:
y

 Intrapartum antibiotic prophylaxis – 37 weeks of gesta-




tion – drugs given are ampicillin or penicillin


 For those who are allergic to penicillin groups – alternative


Figure 4: Sensitivity testing of Streptococcus pyogenes drugs are clindamycin or vancomycin


(Courtesy: Dr J Pandian, Coimbatore Medical College, Tamil Nadu)
GROUP D STREPTOCOCCI – ENTEROCOCCI
y Normally seen as an inhabitant of GIT
Table 5:  Treatment of S. pyogenes Infections y

y Most important pathogens are:


y

Type of infection Treatment drug  E. faecalis (M/c)




 E. faecium
Pharyngitis Benzathine penicillin


 E. gallinarum


Impetigo Benzathine penicillin  E. casseliflavus




Cellulitis Penicillin G y Gram-positive cocci arranged at acute angles to each other


y

y In blood agar – it can exhibit any type of hemolysis – alpha,


y

Necrotizing fasciitis Surgical debridement + beta or gamma


Penicillin G /Clindamycin y Characteristics are – Bile-Esculin test positive
y

Streptococcal TSS Penicillin G + Clindamycin + IV y These can grow at wide range of temperatures from 10°C to
y

immunoglobulin 45°C
y Commonly causes UTI, bacteremia, nosocomial infections
y

and bacterial endocarditis


GROUP B STREPTOCOCCI y This group is intrinsically resistant to many group of antimi-
y

crobials namely:
y Streptococcus agalactiae comes under GBS.
 Penicillin
y

y Main virulence factor is capsule – based on the capsular




 Cephalosporins
y

polysaccharide – many types are classified: Types Ia, Ib and




 Gentamicin (low levels)


II – IX


y Acquired resistance has also evolved due to Van A, B, C, D, E


y 10 to 40% of the females harbour S. agalactiae in their
y

genes to vancomycin – VRE– vancomycin resistant enterococci 75


y

genital tract (GBS colonization)


Unit 2     Bacteriology

High Yield High Yield


Intrinsic Resistance to Enterococci: Acquired Resistance to Enterococci:
• β-lactams (particularly cephalosporins and penicillinase
• • High concentration of β-lactams (alteration of PBPs of

resistant penicillins) production of β-lactamase)


• Low concentrations of aminoglycosides
• • Glycopeptides like Vancomycin and teicoplanin

• Clindamycin
• • High concentration of aminoglycosides

• Fluoroquinolones
• • Tetracycline

• Cotrimaxazole
• • Erythromycin

• Fluoroquinolones

• Rifampin

• Chloramphenicol

• Fusidic acid

Table 6:  Vancomycin resistant enterococci


Characteristics Van A Van B Van C Van D Van E
Vancomycin MIC μg/mL 64 to >1000 4 to 1024 2 to 32 128 16
Teicoplanin 16–512 <0.5 <0.5 4 0.5
MIC μg/mL
Most frequent species E. faecalis E. faecalis E. casseliflavus E. faecium E. faecalis
E. faecium E. faecium E. gallinarum
Genetic determinant Acquired Acquired Intrinsic Acquired Acquired
Transferable Yes Yes No No No

Treatment of Enterococcal Infections MISCELLANEOUS STREPTOCOCCI


y Ideal management needs mandatory antimicrobial suscepti-
y

bility testing Group C • Mostly it causes infections in animals


y Combination group of antimicrobials should be started


y

Streptococci • S. equisimilis is the most common human


infection causing agent


Table 7:  Treatment of Enterococcal infections • Causes pharyngitis and tonsillitis; also

causes deep infections


Endovascular Ampicillin + Aminoglycoside • S. equisimilis is the source of

infections (or) Streptokinase – which is helpful for


Ampicillin + Ceftriaxone thrombolysis in Myocardial infarction

Non-endovascular Ampicillin alone or with aminoglycosides Nonenterococcal •• Most common organism is S. bovis
bacteremia Group D •• Normal commensal in fecal flora
Streptococci • Also causes UTI, SABE
Meningitis Ampicillin + aminoglycoside/ceftriaxone

•• DOC – Penicillin
UTI Fosfomycin Group F • This group of Streptococci poorly grows in

VRE High dose daptomycin +/- aminoglycoside Streptococci blood agar; it needs special incubation
(or) • Proper incubation with CO2 is needed –

Linezolid which gives pinpoint colonies in the agar


(or) media
Quinupristin/Dalfopristin • Hence it is called as ‘minute streptococci’

76
VIRIDANS STREPTOCOCCI y A patient who is at risk with any cardiac valvular diseases

Chapter 7     Streptococci
y

needs to get antimicrobial prophylaxis before any dental


• Gram-positive cocci

procedure to prevent infective endocarditis
• Catalase negative alpha hemolytic

y Prophylaxis given with amoxycillin or penicillin.
y

• Produces greenish color discoloration in blood agar


NUTRITIONALLY VARIANT STREPTOCOCCI


y Most common species that comes under viridans group are:
y

 S. mitis

y y Also called nutritionally deficient Streptococci
 S. mutans

y y Need pyridoxal or cysteine for their growth on blood agar
 S. salivarius

y y May be seen in normal flora
 S. sanguis

y y Causes culture negative Infective endocarditis or brain ab-
scess.
Table 8:  Diseases caused by Viridans group of streptococci
Group of Viridans Diseases caused
Anginosus • Bacteraemia

• Endocarditis

• Abscess

Mitis Endocarditis
Mutans • Endocarditis

• Dental caries

Salivarius Endocarditis (rare)

77
MULTIPLE CHOICE QUESTIONS
Unit 2     Bacteriology

1. C-carbohydrate in Streptococcus haemolyticus is 12. Which of the following factor is mainly responsible for
important for- (Recent Pattern Dec 2014) virulence in Streptococcus- (Recent Pattern Nov 2014)
a. Lancefield classification a. Carbohydrate b. Streptokinase
b. Phagocytic inhibition c. Streptodornase d. M protein
c. Toxin production 13. Crystal violet blood agar is used for which bacteria-
d. Hemolysis  (Recent Pattern Dec 2016)
2. All are medically important streptococci except- a. Corynebacterium diphtheriae
 (Recent Pattern Nov 2013) b. Staphylococcus aureus
a. Strep pyogenes b. Strep agalactiae c. β-hemolytic streptococcus
c. Strep equisimilus d. Strep salivarius d. Meningococcus
3. CAMP test is positive for- (Recent Pattern July 2015) 14. Which toxin of Streptococcus causes hemolysis-
a. Group A streptococcus b. Group B streptococcus  (PGI Nov 2014)
c. Group C streptococcus d. Group D streptococcus a. Streptolysin O b. Streptolysin S
4. Griffith typing is done for- (Recent Pattern July 2016) c. Streptodornase d. Hyaluronidase
a. Staphylococcus b. Streptococcus 15. Streptococcal cell wall polysaccharide cross reacts with-
c. Meningococcus d. Gonococcus  (Recent Pattern Dec 2016)
5. Group B beta hemolytic streptococci is- a. Myocardial muscle b. Cardiac valve
 (Recent Pattern Dec 2012) c. Endocardium d. Synovial fluid
a. Strep pneumonia b. Strep pyogenes 16. Streptococcal toxic shock syndrome is due to the
c. Strep agalactiae d. Enterococcus following virulence factor- (Recent Pattern Dec 2014)
6. Causative agent of acute rheumatic fever- a. M protein b. Pyrogenic exotoxin
 (Recent Pattern July 2016) c. Streptolysin O
a. Group-A b-hemolytic streptococcus d. Carbohydrate cell wall
b. Group-B b-hemolytic streptococcus 17. Streptolysin O functionally and structurally related to-
c. Group-C b-hemolytic streptococcus  (PGI Nov 2014)
d. Group-D b-hemolytic streptococcus a. Tetanolysin
7. A child presents with sepsis. Bacteria isolated showed b. Pneumolysin
beta hemolysis on blood agar, resistance to bacitracin, c. Streptolysin S
and a positive CAMP test. The most probable organism d. Clostridium perfringens toxin
causing infection is- (All India 2010) e. Listeriolysin
a. Streptococcus pyogenes 18. All are true about Streptococcus, except-
b. Streptococcus agalactiae  (AIIMS May 2010, 2011)
c. Enterococcus a. Streptodornase cleaves DNA
d. Streptococcus pneumonia b. Streptolysin O is active in reduced state
8. Bacitracin sensitivity is used to differentiate c. Streptokinase is produced from serotype A, C, K
 (Recent Pattern 2015) d. Pyrogenic toxin A is plasmid mediated
a. Group A Streptococcus from staphylococcus 19. Virulence factor of Group A beta hemolytic Streptococci-
b. Group A Streptococcus from other beta hemolytic  (Recent Pattern Dec 2014)
streptococci a. Protein M b. Protein T
c. Group A Streptococcus from gamma hemolytic strepto- c. Protein R d. Lipoteichoic acid
cocci 20. Which of the following Streptococcal antigen cross
d. Group A Streptococcus from alpha hemolytic strepto- reacts with synovial fluid? (Recent Pattern 2008)
cocci a. Carbohydrate [Group A]
9. Thick pus of Streptococci is converted thin by enzyme- b. Cell wall protein
 (Recent Pattern July 2016) c. Capsular hyaluronic acid
a. DNAse b. Streptokinase d. Peptidoglycan
c. RNAse d. C5a peptidase 21. Scarlet fever is caused due to-
10. Which streptodornase is most antigenic in human  (Recent Pattern August 2013)
beings (Recent Pattern Dec 2016) a. Streptococci b. Staphylococci
a. A b. B c. Klebsiella d. Proteus
c. C d. D 22. Most common age group affected by Streptococcus
11. Streptococcus pyogenes shows pathogenicity by all pyogenes- (Recent Pattern Dec 2016)
except- (Recent Pattern July 2015) a. <5 years b. 5-15 years
a. M protein b. Pyrotoxin c. 20-25 years d. 30-40 years
c. Pili d. Streptolysin O
78
23. The most common organism causing cellulitis- 31. Which serum is used for testing streptococcal pyrogenic

Chapter 7     Streptococci
 (Recent Pattern Nov 2015) toxin- (Recent Pattern July 2015)
a. Streptococcus pyogenes a. Convalescent human serum
b. Streptococcus faecalis b. Serum from patient of acute scarlet fever
c. Streptococcus viridans c. Horse serum
d. Microaerophilic streptococci d. None of the above
24. In a school child had abscess on lower leg. Swab taken 32. Streptococcal skin disease is diagnosed by antibody
revealed Gram-positive b-hemolytic streptococci, against- (Recent Pattern June 2014)
and these were bacitracin sensitive. School physician a. DNAse A b. DNAse B
observed that similar organism was isolated from c. DNAse C d. DNAse D
throats of many other children. Which of the following is 33. Anti DNAase B is used for diagnosis of-
true statement with regards to this patient-  (Recent Pattern Dec 2013)
 (AIIMS Nov 2015) a. Staphylococcus b. Streptococcus
a. Difference in surface protein can differentiate the c. Corynebacterium d. Neisseria
pathogenic bacteria from the pharyngeal culture 34. ASLO titer is done for which bacteria-
bacteria (Recent Pattern Dec 2015)
b.


Component C carbohydrate can differentiate the a. Streptococcus b. Staphylococcus


pathogenic bacteria from the throat culture bacteria c. Gonococcus d. Meningococcus
c. MEG 3 positive are throat culture streptococci 35. Culture medium used for Streptococcus pneumoniae-
d. Depending on the M protein the cutaneous pathogenic  (Recent Pattern July 2016)
bacteria can be differentiated from the pharyngeal a. Human blood agar b. Sheep blood agar
culture bacteria c. MacConkey’s agar d. Deoxycholate agar
25. A child presents with infective skin lesion of the leg. 36. A 11-year-old child presented with sore throat since 3
Culture was done which showed Gram-positive cocci
days, which medium is used to culture the throat swab-
in chains which were hemolytic colonies. The test to
(Recent Pattern July 2016)
confirm the organism is- (AIIMS Nov 2011)


a. Blood agar b. LJ medium


a. Bile solubility b. Optochin sensitivity
c. Stewart medium d. Chocolate agar
c. Bacitracin sensitivity d. Catalase positive
37. A patient admitted to an ICU is on central venous line for
26. False regarding Streptococcus pyogenes
the last one week. He is on ceftazidime and amikacin.
(Recent Pattern Dec 2012)
After 7 days of antibiotics he develops a spike of fever


a. Causes necrotizing fasciitis


and his blood culture is positive for Gram-positive
b. Beta hemolytic
cocci in chains, which are catalase negative. Following
c. M protein is virulence factor
d. Resistant to bacitracin this, vancomycin was started but the culture remained
27. Streptococcus is called as flesh eating bacteria due to positive for the same organism even after 2 weeks of
its- (Recent Pattern June 2014) therapy. The most likely organism causing infection is-
a. Streptolysin A b. Streptolysin O  (AIIMS Nov 2011)
c. Pyrogenic exotoxin d. Hyaluronidase a. Staphylococcus aureus
28. True about Streptococcus pyogenes are all except- b. Viridans streptococci
 (Recent Pattern July 2016) c. Enterococcus faecalis
a. Causes only localized infection d. Coagulase negative staphylococcus
b. Rheumatic fever is nonsuppurative complication 38. Bile esculin test is for- (Recent Pattern Dec 2016)
c. Erythrogenic toxin causes scarlet fever a. Streptococcus pyogenes b. Streptococcus equinus
d. Glomerulonephritis is due to antigenic cross-sensitivity c. Enterococcus d. Streptococcus mutans
29. A boy with skin ulcer on leg, culture reveals beta 39. A beta hemolytic bacteria is resistant to vancomycin.
hemolysis. Culture from school children with sore It shows growth in 6.5% NaCl, is non bile sensitive. It is
throat some days back also revealed beta hemolysis. likely to be- (Recent Pattern Nov 2014)
What is the similarity between both- (AIIMS Nov 2010) a. Streptococcus agalactiae
a. Mec A gene is related to it b. Streptococcus pneumoniae
b. M protein is same c. Enterococcus
c. Carbohydrate antigen is same d. Streptococcus bovis
d. Strains causing both are same 40. Which group of Streptococcus grows at > 60°C
30. Regarding ASO titer all are seen except-  (Recent Pattern Dec 2012)
 (AIIMS Nov 2009) a. A b. B
a. ASO can be increased in school children c. C d. D
b. Maybe negative in poststreptococcal glomerulonephri- 41. Dental caries is caused by- (Recent Pattern Dec 2016)
tis a. Streptococcus pyogenes
c. ASO titer included in major criteria in Jones criteria b. Streptococcus mutans
d. May not be elevated in 20% cases of carditis c. Enterococcus
d. H. influenzae 79
42. False about Gram-positive cocci is-
Unit 2     Bacteriology

45. The following test is helpful in identification of: 


 (Recent Pattern 2008) (AIIMS May 2018)
a.


Staphylococcus saprophyticus causes UTI in females


b. Most enterococci are sensitive to penicillin
c. Nonpathogenic strains are coagulase negative
d. Neonatal meningitis causing streptococci hydrolyses
hippurate
43. Heating at 60°C for 30 minutes would isolate-
 (Recent Pattern Dec 2014)
a. Staphylococci b. Enterococci
c. Micrococci d. Streptococci
44. Which of the following is/are feature of streptococcus
agalactiae rather than staphylococcus aureus?
 (PGI May 2018)
a. Catalase positive b. Bacitracin resistant
c. Coagulase negative d. α hemolysis
e. β hemolysis

Staph aureus b.
a. Strep pyogenes
Clostridium botulinum d. Bacillus
c.

ANSWERS AND EXPLANATIONS

1. Ans.  (a) Lancefield classification 4. Ans.  (b) Streptococcus


Ref: Ananthanarayan and Paniker’s Textbook of Microbiol- Ref: Ananthanarayan and Paniker’s Textbook of Microbiol-
ogy – 10th ed – Page 211 ogy – 10th ed – Page 211
• Hemolytic Streptococci were classified by Lancefield

• Griffith typing is used for further classification of

serologically into groups based on the nature of a Streptococcus pyogenes, belonging to Lancefield group A.
carbohydrate (C) antigen on their cell wall. • Based on M proteins on cell surface, they are subdivided

• These are known as Lancefield groups. A-H and K-V are


into M types. Till now, about 80 types of S. pyogenes have


twenty of them that have been identified and named so been identified.
far.
5. Ans.  (c) Strep agalactiae
2. Ans.  (d) Strep salivarius Ref: Ananthanarayan and Paniker’s Textbook of Microbiol-
Ref: Ananthanarayan and Paniker’s Textbook of Microbiol- ogy – 10th ed – Page 211
ogy – 10th ed – Page 211 and 220. • S. agalactiae is group B beta hemolytic Streptococci.

• S. salivarius belongs to viridians group Streptococci.



• S. pyogenes, which is also beta hemolytic belongs to

They are normal residents of the mouth. group A.


• S. pyogenes, S. agalactiae and S. equisimilis are medically

important Streptococci. 6. Ans.  (a) Group-A b-hemolytic streptococcus


Ref: Ananthanarayan and Paniker’s Textbook of Microbiol-
3. Ans.  (b) Group B streptococcus
ogy – 10th ed – Page 215
Ref: Ananthanarayan and Paniker’s Textbook of Microbiol- S. pyogenes is group A beta hemolyic Streptococcus which
ogy – 10th ed – Page 219 causes acute rheumatic fever and poststreptococcal
• CAMP test differentiates Group B Streptococci (GBS)

glomerulonephritis as nonsuppurative diseases.
(CAMP test positive) from other beta hemolytic
Streptococci (Negative). 7. Ans.  (b) Streptococcus agalactiae
• An accentuated zone of hemolysis when S. agalactiae
Ref: Ananthanarayan and Paniker’s Textbook of Microbiol-

is inovulated perpendicular to a streak of S. aureus


(Staphylococcus plazens streak producing beta lysin) ogy – 10th ed – Page 211
grown on blood agar. • Beta hemolysis is produced both by S. pyogenes and

• Listeria monocytogenes is also CAMP test positive.


• S.agalactiae. S.pyogenes is sensitive to bacitracin

80
whereas S.agalactiae is resistant. S.agalactiae is CAMP • Pili help in attachment of Streptococci to epithelial cells.

Chapter 7     Streptococci

test positive whereas S.pyogenes is CAMP test negative. • Streptolysin O appears to be an important virulent

• The differences between S.pyogenes and S.agalactiae



factor. It is cardiotoxic and leucotoxic.
have been shown in the table below. • Streptococcal pyrogenic exotoxin is injected

intradermally to identify children susceptible to scarlet


S.pyogenes S.agalactiae fever. Only some strains of S. pyogenes produce this
toxin. It is of historical interest.
Group A B
12. Ans.  (d) M protein
Hemolysis Beta Beta Ref: Ananthanarayan and Paniker’s Textbook of Microbiol-
ogy – 10th ed – Page 213
Habitat Throat, skin Female genital tract, rectum
M protein is the most important protein used for typing
Bacitracin Sensitive Resistant as well as for virulence. It acts as a virulence factor by
inhibiting phagocytosis.
PYR test Positive Negative
13. Ans.  (c) b-hemolytic streptococcus
CAMP test Negative Positive Ref: Ananthanarayan and Paniker’s Textbook of Microbiol-
ogy – 10th ed – Page 217
Hippurate Negative Positive • Crystal violet blood agar is used for selective isolation of

hydrolysis S. pyogenes.
• S. pyogenes is more resistant to crystal violet than many

other bacteria including S.aureus.


8. Ans.  (b) Group A streptococcus from other beta
hemolytic streptococci 14. Ans.  (b) Streptolysin S
Ref: Ananthanarayan and Paniker’s Textbook of Microbiol- Ref: Ananthanarayan and Paniker’s Textbook of Microbiol-
ogy – 10th ed – Page 217 ogy – 10th ed – Page 213
• Bacitracin sensitivity is used to differentiate S.pyogenes

• Streptolysin S is an oxygen stable hemolysin responsible

from other beta hemolytic Streptococci. for the hemolysis seen around streptococcal colonies on
• A filter paper disc containing 0.04 U bacitracin is applied

the surface of blood agar plates.
on the surface of inoculated blood agar. After incubation, • Streptolysis O is also hemolytic, but is oxygen labile. So

a wide zone of hemolysis is seen with S. pyogenes, but hemolysis is seen only in the subsurface and not on the
not with other Streptococci. surface cultures.

9. Ans.  (a) DNAse 15. Ans.  (b) Cardiac valve


Ref: Ananthanarayan and Paniker’s Textbook of Microbiol- Ref: Ananthanarayan and Paniker’s Textbook of Microbiol-
ogy – 10th ed – Page 214 ogy – 10th ed – Page 213
• Streptodornase, also called deoxyribonuclease (DNase)

• Streptococcal polysaccharide (group A carbohydrate)

causes depolymerization of DNA. They help to liquefy cross reacts with cardiac valves.
thick pus and are responsible for the thin serous Other antigenic cross reactions are as follows:
character of streptococcal exudates.
• This property has been applied therapeutically in

Cell wall proteins Myocardium
liquefying localized collections of thick exudates as in Capsular hyaluronic acid Human synovial fluid
empyema.
Cytoplasmic membrane antigens Vascular intima
10. Ans.  (b) B Peptidoglycans Skin antigens
Ref: Ananthanarayan and Paniker’s Textbook of Microbiol- • These antigenic cross reactions are responsible for

ogy – 10th ed – Page 214 immune mediated tissue damage in Streptococcal


• Four antigenically distinct DNases, A, B, C and D have

diseases.
been recognized of which type B is most antigenic in
humans. 16. Ans.  (b) Pyrogenic exotoxin
• Demonstration of anti-DNase B antibody is useful in
Ref: Ananthanarayan and Paniker’s Textbook of Microbiol-

retrospective diagnosis of S. pyogenes infection.


ogy – 10th ed – Page 214
11. Ans.  (b) Pyrotoxin • Streptococcal pyrogenic exotoxins are superantigens

Ref: Ananthanarayan and Paniker’s Textbook of Microbiol- (like Staphylococcal enterotoxins and TSS toxin), which
ogy – 10th ed – Page 213 are T cell mitogens that induce a massive release of
inflammatory cytokines, causing tissue damage, fever
• M protein is an important virulence factor of S. pyogenes 81
and shock.

which acts by inhibiting phagocytosis.


17. Ans.  (d) Clostridium perfringens toxin • Streptococcal infections are more frequent in children
Unit 2     Bacteriology

at 5-8 years age than in children below the age of 2 years


Ref: Ananthanarayan and Paniker’s Textbook of Microbiol-
or in adults.
ogy – 10th ed – Page 213 and 259
• They are common in winter in the temperate countries.

• Theta toxin (θ) of C. perfringenes is an oxygen labile


hemolysin antigenically related to Streptolysin O. It is 23. Ans.  (a) Streptococcus pyogenes


also lethal and cytolytic.
Ref: Ananthanarayan and Paniker’s Textbook of Microbiol-
• Streptolysin O resembles the oxygen labile hemolysins
ogy – 10th ed – Page 215

of C. perfringenes, C. tetani and S. pneumoniae.


• S. pyogenes has the predilection to produce lymphangitis

18. Ans.  (d) Pyrogenic toxin A is plasmid mediated and cellulitis.


• They produce pyogenic infections with the tendency to
Ref: Ananthanarayan and Paniker’s Textbook of Microbiol-

spread along lymphatics and through blood stream.


ogy – 10th ed – Page 214
• Three types of Streptococcal pyrogenic exotoxins have
• 24. Ans.  (d) Depending on the M protein the cutaneous
been identified - A, B and C. pathogenic bacteria can be differentiated from the
• Types A and C are coded for by bacteriophage genes and
• pharyngeal culture bacteria
type B gene is chromosomal. None of them is plasmid
Ref: Ananthanarayan and Paniker’s Textbook of Microbiol-
mediated.
ogy – 10th ed – Page 213
19. Ans.  (a) Protein M • M protein the most important protein used for typing of

S. pyogenes.
Ref: Ananthanarayan and Paniker’s Textbook of Microbiol-
• M protein can be extracted by Lancefield’s acid
ogy – 10th ed – Page 213

extraction method and typing can be done with type


• M protein is the most important virulence factor which

specific sera.
acts by inhibiting phagocytosis.
• The T and R proteins have no relation to virulence.
• 25. Ans.  (c) Bacitracin sensitivity

20. Ans.  (c) Capsular hyaluronic acid Ref: Ananthanarayan and Paniker’s Textbook of Microbiol-
ogy – 10th ed – Page 217
Ref: Ananthanarayan and Paniker’s Textbook of Microbiol-
• The likely organism in this scenario is S. pyogenes.
ogy – 10th ed – Page 213

• Bacitracin sensitivity differentiates S. pyogenes from


• Capsular hyaluronic acid cross reacts with human



other beta hemolytic Streptococci.
synovial fluid. • S. pyogenes is bacitracin sensitive whereas the others are

Other antigenic cross reactions are as follows. resistant.


Cell wall proteins Myocardium
26. Ans.  (d) Resistant to bacitracin
Streptococcal polysaccharide Cardiac valves Ref: Ananthanarayan and Paniker’s Textbook of Microbiol-
(group A carbohydrate) ogy – 10th ed – Page 217
Cytoplasmic membrane antigens Vascular intima • S. pyogenes is bacitracin sensitive.

• Refer Answer no.25.


Peptidoglycans Skin antigens

27. Ans.  (c) Pyrogenic exotoxin


• These antigenic cross reactions are responsible for

Ref: Ananthanarayan and Paniker’s Textbook of Microbiol-
immune mediated tissue damage in Streptococcal ogy – 10th ed – Page 215
diseases. • Nacrotizing fasciitis is particularly associated with M

types 1 and 3 of S.pyogenes that produces pyrogenic


21. Ans.  (a) Streptococci exotoxin.
• These strains have earned a notorious name, “Flesh

Ref: Ananthanarayan and Paniker’s Textbook of Microbiol- eating bacteria.”


ogy – 10th ed – Page 213
28. Ans.  (a) Causes only localized infection
• Scarlet fever is caused by Streptococcus pyogenes which
Ref: Ananthanarayan and Paniker’s Textbook of Microbiol-

produces erythrogenic or scarlatinal or dick toxin.


ogy – 10th ed – Page 212
• It is characterized by acute pharyngitis with extensive
• S. pyogenes causes pyogenic infection with a tendency

erythematous rash. •

to spread locally, along lymphatics and through blood


22. Ans.  (b) 5-15 years stream.

Ref: Ananthanarayan and Paniker’s Textbook of Microbiol- 29. Ans.  (c) Carbohydrate antigen is same
ogy – 10th ed – Page 216 Ref: Ananthanarayan and Paniker’s Textbook of Microbiol-
82 ogy – 10th ed – Page 215
• S. pyogenes causes both these conditions. The skin lesion • Human blood agar is not recommended because it can

Chapter 7     Streptococci
• •

is caused by serotypes belonging to higher numbered contain infectious organisms and antibiotics.
M types and the throat infection by lower numbered M
types. 36. Ans.  (a) Blood agar
• Both the strains are S. pyogenes which belongs to

Lancefield group A, i.e. they share a common C- Ref: Ananthanarayan and Paniker’s Textbook of Microbiol-
carbohydrate antigen. ogy – 10th ed – Page 214 and 217.
30. Ans.  (c) ASO titer included in major criteria in Jones • Sore throat is the most common Streptococcal disease.

criteria • For culture of Streptococcus pyogenes, the sample is


inoculated on 5% sheep blood agar and incubated at


Ref: Davidson’s Principles and Practice of Medicine- 22nd
ed- Page 37°C and looked for growth of beta hemolytic colonies.
• The major Jones criteria include carditis, polyarthritis,

37. Ans.  (c) Enterococcus faecalis


chorea, erythema marginatum and sub cutaneous
nodules. Ref: Ananthanarayan and Paniker’s Textbook of Microbiol-
• Minor criteria are Arthralgia, fever, raised ESR or CRP

ogy – 10th ed – Page 220
and prolonged PR interval on ECG.
• Enterococci are intrinsically resistant to cephalosporins
• Evidences for previous Streptococcal infection are •

and offer lower level resistance to some aminoglycosides.


positive throat culture/ raising ASO titer.


So, the patient has not responded to initial treatment
31. Ans.  (a) Convalescent human serum with ceftazidime had amikacin.
Ref: Ananthanarayan and Paniker’s Textbook of Microbiol- • Recently, vancomycin resistant strains are also

ogy – 10th ed – Page 213 increasing. So the isolate responsible for infection in this
• Convalescent human serum was used in testing

patient is Vancomycin resistant Enterococci (VRE).
Streptococcal pyrogenic exotoxin. Blanching of the
38. Ans.  (c) Enterococcus
skin on injection of convalescent serum was used as
a diagnostic test for scarlet fever (Schultz Charlton Ref: Ananthanarayan and Paniker’s Textbook of Microbiol-
reaction). ogy – 10th ed – Page 219 and 220
• This test is only of historical interest now.

• Bile esculin test is done for identification of Enterococci

32. Ans.  (b) DNAse B which shows blackening of bile esculin agar.
• They can grow in the presence of 40% bile and can
Ref: Ananthanarayan and Paniker’s Textbook of Microbiol-

ferment esculin to esculetin which is responsible for the


ogy – 10th ed – Page 218 black color.
• Anti-DNase B test is very useful for the retrospective

diagnosis of streptococcal pyoderma, for which ASO is 39. Ans.  (c) Enterococcus
of much less value.
Ref: Ananthanarayan and Paniker’s Textbook of Microbiol-
• Titers greater than 300 are considered significant.
ogy – 10th ed – Page 211, 219 and 220

33. Ans.  (b) Streptococcus • Enterococci most of the time produce gamma hemolysis

(nonhemolytic) on sheep blood agar, however they can


Ref: Ananthanarayan and Paniker’s Textbook of Microbiol- produce alpha and beta hemolysis as well.
ogy – 10th ed – Page 218 • They can grow in 6.5% NaCl, tolerate 60°C for 30 minutes,

• Refer Answer no. 32



cause blackening of bile esculin media.
• Enterococci are intrinsically resistant to cephalosporins

34. Ans.  (a) Streptococcus and offer lower level resistance to some aminoglycosides.
Ref: Ananthanarayan and Paniker’s Textbook of Microbiol- Recently, vancomycin resistant strains are increasing
ogy – 10th ed – Page 218 (VRE) which is due to changes in Van genes.
• Antistreptolysin O titration is the standard test for

retrospective diagnosis of streptococcal infection. 40. Ans.  (d) D


• ASO titer higher than 200 is indicative of prior

Ref: Ananthanarayan and Paniker’s Textbook of Microbiol-


streptococcal infection. High levels are usually found in ogy – 10th ed – Page 220
acute rheumatic fever but in glomerulonephritis titers • Enterococci are group D Streptococci that are relatively

are low. heat resistant and can survive for 30 minutes at 60°C.
35. Ans.  (b) Sheep blood agar
41. Ans.  (b) Streptococcus mutans
Ref: Ananthanarayan and Paniker’s Textbook of Microbiol-
ogy – 10th ed – Page 40 and 211 Ref: Ananthanarayan and Paniker’s Textbook of Microbiol-
ogy – 10th ed – Page 221
• Sheep blood agar is preferred for Streptococci because

• Streptococcus mutans is an alpha hemolytic


hemolysis is clearly differentiable. streptococcus which is part of normal flora of oral cavity. 83
• It can cause dental caries and endocarditis in 44. Ans.   (c) Coagulase negative
Unit 2     Bacteriology

individuals with risk factors (dental extraction in people


with damaged heart valves). This bacterium has a Ref: Ananthanaryan and Paniker TB of microbiology - 10th
polysaccharide coat (glycocalyx) that allows it to stick to ed - page 211
teeth. It also produces acid from sugar in saliva which Staph aureus also exhibits beta hemolysis; hence thats not
promotes erosion of tooth enamel. a reliable method for distinguishing;
Staph aureus is catalase positive and Strep is catalase
42. Ans.  (b) Most enterococci are sensitive to penicillin negative; (Question is about features of S. agalactiae)
Coagulase test is positive in Staph aureus and negative
Ref: Ananthanarayan and Paniker’s Textbook of Microbiol- in others (hence it is helpful to distinguish); Even though
ogy – 10th ed – Page 220 coagulase test is basically helpful to distinguish CONS
• Most Enterococci are resistant to penicillin. So antibiotic

from staph aureus - here based on the options - option C


susceptibility testing is necessary. is correct
• Enterococci are intrinsically resistant to cephalosporins

and show low level resistance to aminoglycosides. 45. Ans.  (b) Streptococcus pyogenes
Ref: Ananthanarayan and Paniker’s Textbook of Microbiol-
43. Ans.  (b) Enterococci ogy – 10th ed – Page 217
Ref: Ananthanarayan and Paniker’s Textbook of Microbiol- • Figure shows blood agar culture with Bacitracin disk

ogy – 10th ed – Page 220 and Penicillin disk which gives clue that organism is
• Refer Answer no 39.

Streptococcus
• Key test is for Streptococcus pyogenes is Bacitracin disk

(0. 04U) test – all strains of S. pyogenes are sensitive to


bacitracin.

84
8
Pneumococcus
PNEUMOCOCCUS y Pneumococcal pneumonia: (Community acquired pneu-

y
monia)
 Most common cause of lobar pneumonia
• Gram-positive diplococci


 Types 1–8 are the most common causes of pneumonia in

• Lanceolate shaped arrangement


adults

• Has a capsular polysaccharide
 Types 6, 14, 19, 23 are common causes in children


 Empyema is the most common focal complication of
y Pneumococci belong to alpha-hemolytic streptococci


pneumonia
y
y It can be differentiated from alpha hemolytic viridans group
y Meningitis
y
by:

y
y Bacteremia
 Bile solubility test

y
y Endocarditis

 Optochin sensitivity test
y
y Septic arthritis

y
y Pneumococci is the prime important cause for sepsis in
Capsule
y
splenectomy patients.
y Capsule of Pneumococci is the single most important
Laboratory Diagnosis
y
virulence factor
y Capsular polysaccharide is immunologically distinct and it y Gram staining of the sputum or other specimens – shows
y
y
has 91 types plenty of pus cells with Gram-positive diplococci with
y This C-Polysaccharide can be detected by serological tests lanceolate shaped
y
from samples like CSF and urine and helps in the laboratory y Care must be taken in reporting of sputum as pharngeal
y
diagnosis of pneumococcal infections contamination with commensal pneumococci will lead to
y Methods of detection of capsule are: misdiagnosis. Hence always look for pus cells with clinical
y
 Gram stain – a halo around the diplococci correlation;

 Capsular staining y Growth of specimen under proper incubatory conditions

y
 Quellung reaction: When capsular strains are mixed (in CO2 at 37°C) in a blood agar plate – shows alpha colored

with that specific antipolysaccharide serum – it leads to hemolytic colonies; pin point in appearance;
swelling and agglutination reaction which can be visible
by microscopic examination; the serum that is used
for such test should be polyvalent antiserum named as
omniserum;
 Latex agglutination tests (CRP)

High Yield
• Vaccination is given based on the capsular types (most

common and virulent types are taken for consideration for
vaccine manufacturing)

Clinical Manifestations
Factors that are responsible for pneumococcal infections
• Abnormality of respiratory tract Figure 1: Gram-positive pneumococci (Courtesy: CDC)

• Chronic diseases like sickle cell anemia and nephrosis
y Bile solubility test: Colonies of Pneumococci made as a broth

• Alcohol or drug intoxication
y
or direct culture broth are mixed with sodium deoxycholate

• Splenectomy
(2%) and suspended. After sometime when compared with

y Most common pneumococcal syndrome is otitis media a control tube – lysis of pneumococci occurs; this is because
y
(affects younger children) autolytic enzyme (autolysin) in pneumococci activates the
surface active agents
y Optochin sensitivity test: Pneumococci are Optochin Prevention
Unit 2     Bacteriology

sensitive (colonies won’t grow surrounding optochin disk) in


y Two types of vaccines are available:
contrast viridans streptococci are Optochin resistant.
y

(i) Polysaccharide vaccine containing 23 types (PPSV23)


(ii) Pneumococcal conjugate vaccine containing capsular


 

polysaccharide which is conjugated with diphtheria


(PCV13 – Prevnar13)
y PCV 13 is recommended vaccine now for children in four
y

dose series at 2, 4, 6 and 12–15 months of age


y PPSV23 vaccine should not be given for children under 3
y

years of age.

High Yield
Figure 2: Optochin sensitivity test
Vaccination is indicated in adults for the following risk factors:
(PPSV23)
Treatment • Splenectomy

• Nonfunctional spleen
Table 1:  Drug of choice for infections caused by

• Chronic diseases like nephrosis and sickle cell diseases


Pneumococci

• Immunocompromised patients like AIDS


• Diabetes mellitus
Type of infection Drug of choice •

• Cochlear implants

Pneumonia (Penicillin Penicillin group • CSF rhinorrhea or trauma leading to CSF leak

susceptible strains) • Age above 65 years.


Pneumonia (Penicillin resistant Macrolides and cephalosporins


strains)
Meningitis (>1 month of age Vancomycin + Cefotaxime/
group) Ceftriaxone
Acute otitis media Amoxicillin

86
MULTIPLE CHOICE QUESTIONS

Chapter 8     Pneumococcus
1. Capsule of pneumococcus is- (Recent Pattern July 2016) 9. A chronic alcoholic is presenting with clinical features
a. Polypeptide b. Polysaccharide of meningitis. Most likely organism which will grow on
c. Lipopolysaccharide d. Virulence factor CSF culture- (Recent Pattern Dec 2016)
2. Quellung reaction is seen in-(Recent Pattern Dec 2013) a. Streptococcus pneumoniae
a. Group B Streptococcus b. Staphylococcus b. N. meningitidis
c. Pneumococcus d. Enterococcus c. Listeria monocytogenes
3. Draughtman concentric rings on culture are produced d. E. coli
by- (Recent Pattern Dec 2013) 10. After splenectomy, most common infection-
a. Yersinia pestis b. H. ducreyi  (Recent Pattern Dec 2013)
c. B. pertussis d. Pneumococci a. Pneumococcal b. E. coli
4. Streptococcus pneumoniae produces which type of c. Klebsiella d. Streptococcus
hemolysis- (Recent Pattern July 2015) 11. Postsplenectomy patient is in need of _______________
a. Alpha b. Beta vaccination- (Recent Pattern Dec 2012)
c. Gamma d. Any of the above a. Pneumococcal b. Rotavirus
5. The following are features of Pneumococcus except- c. BCG d. MMR
 (Recent Pattern Dec 2014) 12. Pneumococcal vaccine is prepared from-
a. Bile insolubility b. Optochin sensitivity  (Recent Pattern Dec 2013)
c. Gram-positive d. Encapsulated a. Cell surface antigen
6. Bile solubility is used for- (Recent Pattern Dec 2013) b. Capsular polysaccharide
a. Differentiation of Staphylococcus from streptococcus c. From exotoxin
b. Differentiation of group B streptococci from other d. From M protein
streptococci 13. Which of the following statements about Pneumococcus
c. Differentiation of pneumococci from streptococci is false- (Recent Pattern 2011)
d. Differentiation of streptococci from Neisseria a. Capsule aids in virulence
7. A patient presents with signs of pneumonia. The b. Most common cause of otitis media
bacterium obtained from sputum was a Gram-positive c. Causes mild form of meningitis
cocci which showed alpha hemolysis on sheep agar. d. Respiratory tract of carriers is the most important
Which of the following test will help to confirm the source of infection
diagnosis? (All India 2011) 14. Streptococcal pneumonia present at- 

a. Bile solubility  (Recent Pattern Dec 2016)


b. Coagulase test a. <5 years b. 5–15 years
c. Bacitracin test c. 20–25 years d. 30–40 years
d. cAMP test 15. Most common infection caused by Streptococcus pneu-
8. A person presents with pneumonia. His sputum was sent moniae- (Recent Pattern July 2015)
for culture. The bacterium obtained was Gram-positive a. Otitis media b. Sore throat
cocci in chains and alpha-hemolytic colonies on sheep c. Meningitis d. Pneumonia
agar. Which of the following will help in confirming the 16. Most common causative organism for lobar pneumonia
diagnosis- (AIIMS May 2012) is- (Recent Pattern July 2016)
a. Novobiocin a. Staphylococcus aureus
b. Optochin b. Streptococcus pyogenes
c. Bacitracin c. Streptococcus pneumonia
d. Oxacillin d. Haemophilus influenzae

87
ANSWERS AND EXPLANATIONS
Unit 2     Bacteriology

1. Ans.  (b) Polysaccharide 6. Ans.  (c) Differentiation of pneumococci from


streptococci
Ref: Ananthanarayan and Paniker’s Textbook of Microbiol-
ogy – 10th ed – Page 225 Ref: Ananthanarayan and Paniker’s Textbook of Microbiol-
• Capsule of Pneumococci is a polysaccharide. It diffuses

ogy – 10th ed – Page 225
into the culture medium or infective exudates and • Bile solubility test is used for differentiation of Pneumo-

tissues. So it is also called the specific soluble substance. cocci from viridians Streptococci.
• Pneumococci is bile soluble whereas other streptococci

2. Ans.  (c) Pneumococcus are not bile soluble.


Ref: Ananthanarayan and Paniker’s Textbook of Microbiol-
7. Ans.  (a) Bile solubility
ogy – 10th ed – Page 225
• Streptococcus pneumoniae/Pneumococci show capsu-

Ref: Ananthanarayan and Paniker’s Textbook of Microbiol-
lar swelling or quellung reaction. ogy – 10th ed – Page 224
• A suspension of S. pneumoniae is mixed with a drop of
• • Gram-positive cocci causing pneumonia which causes

the type-specific antiserum and a loopful of methylene alpha-hemolysis on blood agar is S. pneumoniae. It
blue solution. In the presence of homologous antiserum can be confirmed by 3 biochemical tests - Optochin
the capsule becomes apparently swollen, sharply sensitivity, Bile solubility and Inulin fermentation.
delineated and refractile.
8. Ans.  (b) Optochin
3. Ans.  (d) Pneumococci
Ref: Ananthanarayan and Paniker’s Textbook of Microbiol-
Ref: Ananthanarayan and Paniker’s Textbook of Microbiol- ogy – 10th ed – Page 225
ogy – 10th ed – Page 224 • Gram-positive cocci causing pneumonia which causes

• Streptococcus pneumoniae produces colonies with


• alpha-hemolysis on blood agar is S. pneumoniae. It
draughtsman or carrom coin appearance on blood agar. can be confirmed by 3 biochemical tests - Optochin
• On blood agar, after incubation for 18 hours, the
• sensitivity, Bile solubility and Inulin fermentation.
colonies are small, dome shaped and glistening with
an area of greenish discoloration around them. On 9. Ans.  (a) Streptococcus pneumoniae
further incubation the colonies become flat with raised Ref: Ananthanarayan and Paniker’s Textbook of Microbiol-
edges and central umbonation, so that concentric ogy – 10th ed – Page 227
circles are seen on the surface when viewed from above.
(Draughtsman or carom coin appearance) • Infection due to S. pneumoniae usually results only

in pharyngeal carriage. Disease results only when


4. Ans.  (a) Alpha host resistance is lowered by contributory factors like
respiratory viral infection, pulmonary congestion,
Ref: Ananthanarayan and Paniker’s Textbook of Microbiol- stress, malnutrition, immunodeficiency or alcoholism.
ogy – 10th ed – Page 225 • The patient in the given scenario is a chronic alcoholic.

• S. pneumoniae produces alpha-hemolysis when grown


• So he is highly prone for invasive pneumococcal disease
on blood agar. – meningitis and septicemia.

5. Ans.  (a) Bile insolubility 10. Ans.  (a) Pneumococcal


Ref: Ananthanarayan and Paniker’s Textbook of Microbiol- Ref: Ananthanarayan and Paniker’s Textbook of Microbiol-
ogy – 10th ed – Page 224 ogy – 10th ed – Page 227
• S. pneumoniae are bile soluble. If a few drops of 10%
• • Splenectomy is an important predisposing condition

sodium deoxycholate solution is added to 1 mL of for Pneumococcal infection and it is the most common
overnight suspension of Pneumococci, culture clears infection in splenectomised patients.
due to lysis of cocci. • Polyvalent polysaccharide vaccine (capsular vaccine)

• The key biochemical features for identification of


• for Pneumococci is recommended for use in splenecto-
Pneumococci in laboratory are: mised patients.
ƒ Optochin sensitivity
ƒ

ƒ Bile solubility
ƒ

ƒ Inulin fermentation
ƒ

88
11. Ans.  (a) Pneumococcal 14. Ans.  (a) <5 years

Chapter 8     Pneumococcus
Ref: Ananthanarayan and Paniker’s Textbook of Microbiol- Ref: Ananthanarayan and Paniker’s Textbook of Microbiol-
ogy – 10th ed – Page 228 ogy – 10th ed – Page 227
• Pneumococcal polyvalent polysaccharide vaccine (cap-
• • S. pneumoniae more commonly affects the extremes

sular vaccine) is recommended for use in splenecto- of age. Very old and very young population groups are
mised patients. more susceptible. Among the children, less than 2 years
is most susceptible.
12. Ans.  (b) Capsular polysaccharide
15. Ans.  (a) Otitis media
Ref: Ananthanarayan and Paniker’s Textbook of Microbiol-
ogy – 10th ed – Page 228 Ref: Ananthanarayan and Paniker’s Textbook of Microbiol-
• Pneumococcal vaccine is prepared from capsular

ogy – 10th ed – Page 227
polysaccharide. It is a polyvalent polysaccharide • The most common pneumococcal infections are otitis

vaccine representing the capsular antigens of the 23 media and sinusitis.


most prevalent serotypes. • Prior respiratory infection or allergy causing congestion

and blockade predispose to these conditions.


13. Ans.  (c) Causes mild form of meningitis
16. Ans.  (c) Streptococcus pneumonia
Ref: Ananthanarayan and Paniker’s Textbook of Microbiol-
ogy – 10th ed – Page 227 Ref: Ananthanarayan and Paniker’s Textbook of Microbiol-
• Meningitis is the most serious of pneumococcal

ogy – 10th ed – Page 226
infections. • S. pneumoniae is one of the most common bacteria

• It is usually secondary to other pmeumococcal infections


• causing pneumonia, both lobar and broncho pneumo-
like pneumonia, otitis media, sinusitis or conjunctivitis nia.
but in some cases, other foci of infection may not be
demonstrable.

89
9
Neisseria
y This group of bacteria are Gram-negative Diplococci
Infections Presentation
y
y Two most important species that are important for infections
Gonococci in • Infection affects the endocervix
y
are:


 Neisseria gonorrhoeae women • It leads to cervicitis, salpingitis, PID and


sterility

 Neisseria meningitidis
• Leads to purulent vaginal discharge and


intermenstrual bleeding
NEISSERIA GONORRHOEAE (MEANS FLOW
OF SEEDS)
Disseminated • Called as DGI – Disseminated gonococcal


• Intracellular Gram-negative diplococci infections infection
• Most common cause of septic arthritis in

• Nonmotile, nonspore forming organism


sexually active adults is DGI

• Catalase and oxidase positive
• It presents as arthritis, tenosynovitis and

• Ferments glucose not maltose


skin lesions

y Neisseria gonorrhoeae is a Gram-negative diplococci, non- • In such condition - diagnosis is difficult
y
motile, nonsporing cocci, kidney shaped with concave sur-


because organism cannot be cultured
face facing each other.
y It is catalase positive, oxidase positive.
y
y It ferments glucose but not maltose, sucrose or lactose. Gonococci in • Gonococcal infected mother giving
y

neonates birth to child via vaginal route leads to
Virulence Factors and Pathogenesis infection in baby
• This presents as purulent conjunctivitis
y Pili outer membrane protein (OMP) is the most important

named as Ophthalmia neonatorum
y
virulence factor
• Prevented by application of 0.5%
y Pileated strains adhere more to cells derived from human

erythromycin ointment or 1%
y
mucosal surfaces and are more virulent
tetracycline ointment
y Proteins: Opa, Por, Rmp
y
y Lipooligosaccharide
y
y IgA1 protease Gonococci in • Anorectal gonorrhea – leading to proctitis

y
y In addition to the antigenic structures, host factors are also homosexuals
y
important in the pathogenesis of infection.
y Activation of phosphatidyl-choline – specific phospholipase
y
C and sphingomyelinase by the organism resulting in the
release of ceramide and diacylglycerol, paves a way for the
Laboratory Diagnosis
entry of N. gonorrhoeae into the epithelial cells. y Gram staining shows plenty of polymorphonuclear leukocytes
y
(PMN) with intracellular Gram-negative diplococci

Clinical Manifestations Specimen Sensitivity Specificity


Gonococci causes localized and also systemic infections
Urethral exudates from men 90% 99%
Table 1:  Infections caused by gonococci Endocervical exudates from 50% 95%
Infections Presentation women
Gonococci in men • Most common presentation is urethritis
y Gram staining is confirmatory for males

• Epididymitis also occurs
y
y Gram staining is not confirmatory for females – additional

• Presents as dysuria and purulent
y
nucleic acid amplification test or culture should be done

discharge
contd...
Resistance

Chapter 9     Neisseria
y Two types of beta lactamases resistance:
y

 Plasmid mediated beta lactamase resistance – named as




PPNG – Penicillinase producing Neisseria gonorrhea


 Chromosomal mediated resistance (TEM type)


y Quinolone resistance: QRNG (Quinolone resistance Neisseria


y

gonorrhea)
y Tetracycline resistance: Both plasmid and chromosomal
y

mediated types are seen

NEISSERIA MENINGITIDIS

Figure 1:  Intracellular Gram-negative Diplococci • • Intracellular Gram negative Diplococci


(Courtesy: CDC/Bill Schwartz) • • Nonmotile, non-spore forming organism
y Endocervical swab should be collected with Dacron or rayon • • Catalase and oxidase positive
• Ferments glucose and maltose
y

or charcoal impregnated swabs. •

y As gonococci is highly susceptible to drying, temperature or


y

toxic substances like cotton swabs, it should be inoculated Antigenic Structure


immediately in a selective medium like Modified Thayer
Martin medium. y Serogrouping is done based on capsular polysaccharides
y

y Culture of Gonococci needs special media like modified y Total number of serogroups are 13
y

y The most important one that causes disease in humans are A,


y

Thayer Martin medium – chocolate agar containing y

antimicrobials like Vancomycin, Colistin and Nystatin B, C, X, Y and W-135


y Culture is the “Gold Standard” for the definitive diagnosis
Group A Epidemics
y

of gonorrhea. The sensitivity of culture is 85−95% in case of


acute infection and the specificity is 100%, whereas in chronic Group B Both epidemics and outbreaks
infections it is 50% sensitivity. Group C Localized outbreaks
y When colonies are seen within 48 hours of incubation under
y

proper CO2 conditions – oxidase test should be performed –


Gonococci colonies are oxidase positive Pathogenesis
y Oxidase test is done by Kovac’s method where the colonies
y
y Humans are the only host for pathogenic infections
y

are taken in filter paper moistened with oxidase reagent – y Portal of entry – Nasopharynx
y

appearance of purple color within 30 sec is considered as y From there – organism enters into the blood stream and
y

positive causes systemic infections


y Nucleic acid amplification tests (NAAT) are done in high
y
y Meningococcemia – presents as fever with hemorrhagic
y

prevalence populations; NAAT cannot be used as prognostic rash and in some patients it may lead to DIC and circulatory
test as it may give positive results up to 3 weeks after treatment collapse; it is termed as Waterhouse-Friderichsen syndrome
also. y Meningococcal disease is favored by deficiency of terminal
y

complement components (C5-C9)


Treatment y The most common complication of meningococcemia is
y

meningitis
Table 2:  Treatment of clinical conditions caused by
y Case fatality rate in untreated meningitis is around 80%
gonococci
y

y 5–30% of the normal population carries meningococci in


y

Condition Treatment their nasopharynx – they are the most important sources of
Uncomplicated genital or Ceftriaxone 250 mg IM single infection
rectal infections dose y Meningococci belt – sub-saharan Africa.
y

(or)
400 mg oral cefixime single dose Clinical Features
Uncomplicated urethritis, Ceftriaxone 250 mg IM single y Most common form of infection is asymptomatic carriage of
y

cervicitis, proctitis dose organism in nasopharynx.


(alternative regimen) + azithromycin 1 g orally once or y A nonblanching petechial rash is seen in >80% of the cases.
y

doxycycline bd 100 mg for 7 days y t presents as fever, vomiting, irritability and head ache. The
y

Disseminated infections Ceftriaxone classical signs like neck rigidity and photophobia are usually
not seen in infants and children.
Ophthalmia neonatorum Ceftriaxone single dose
y Along with meningitis, features of septicemia are also seen in
y

Pregnant women with Azithromycin 20–40%.


gonorrhea y Septicemia can go for shock and death.
y
91
y Chronic meningococcemia can present as rash, fever, joint y CSF culture done in chocolate agar plates incubated under
Unit 2     Bacteriology

y y

pain, arthritis and splenomegaly. proper CO2 conditions and looked for colonies
y Postmeningococcal sequelae is seen in some patients which
y

manifest as arthritis, serositis and pericarditis. High Yield


Differential diagnosis for acute purulent meningitis are:
• Neisseria meningitidis

• Streptococcus pneumoniae

• Haemophilus influenzae

y Nasopharyngeal swab is the specimen of choice for detection


y

of carriers – transport medium used is Stuart’s medium


y Postmortem cause of death is meningococcal meningitis can
y

be confirmed from brain specimen when collected within 12


hours of death.

Treatment
y Penicillin G is the DOC for meningococcal disease
Figure 2:  Figure showing petechial rashes of
y

y Empirical treatment for meningitis is Ceftriaxone


Meningococcal infection
y

y y Chemoprophylaxis – Rifampicin or Ciprofloxacin


y Treatment for carrier eradication – Rifampicin or Cipro-
High Yield
y

floxacin
Differential diagnosis for infectious etiology for petechial rashes:
• Enteroviruses

Vaccination
• Measles

• EBV•
y Vaccination available for Groups A, C, Y and W-135
y

• CMV•
y Group B – vaccine available only in European union – named
y

• Parvovirus

as 4CmenB (not in India)
• Influenza virus

• Staphylococci

Table 3:  Type of vaccine for age-groups different
• Streptococci
Age group Type of vaccine

• Pneumococci

• Meningococci
• 6 weeks to 18 months Hib-Men-TT (combined) four doses
9–23 months Tetravalent conjugate vaccine – two
doses (conjugation with diptheria
Laboratory Diagnosis toxoid – Menactra vaccine
y Specimens: Nasopharyngeal swab, CSF, blood, scrapings
2–55 years Tetravalent conjugate vaccine – Menveo
y

from petechiae
y Gram staining of CSF shows intracellular Gram-negative
y

Diplococci with abundant polymorphonuclear leukocytes NONGONOCOCCAL URETHRITIS


Causative organisms:
y Chlamydia trachomatis
y

y Ureaplasma urealyticum
y

y Mycoplasma hominis
y

y Herpes
y

y Cytomegalovirus
y

y Gardnerella vaginalis
y

y Acinetobacter lwoffi
y

y Candida albicans
y

y Trichomonas vaginalis
y

Figure 3: Gram staining image of N. meningitidis in specimen


(Courtesy: CDC/ Dr Brodsky)
92
MULTIPLE CHOICE QUESTIONS

Chapter 9     Neisseria
1. Genus Neisseria is- (Recent Pattern July 2015) 12. Meningitis with rash is seen in-
a. Gram-positive Diplococci  (Recent Pattern Dec 2012)
b. Gram-negative Diplococci a. Neisseria meningitidis
c. Gram-negative coccobacilli b. H.influenzae
d. Gram-positive bacilli c. Streptococcus agalactiae
2. Gonococcus is- (Recent Pattern Dec 2012) d. Pneumococcus
a. Extracellular Gram-positive 13. A young lady complains of sore throat for 3 days along
b. Intracytoplasmic Gram-positive with fever and headache. On examination, she was
c. Intracytoplasmic Gram-negative severely dehydrated, her BP was found to be 90/50 mm
d. Intranuclear Gram-negative Hg and on the distal aspect of the cuff, small red spots
3. Meningococci differ from gonococci in that they- were noted. What could be the most probable etiological
 (Recent Pattern Nov 2014) agent responsible for causing these symptoms-
a. Are intracellular  (AIIMS May 2013)
b. Possess a capsule a. Brucella abortus b. Brucella suis
c. Cause fermentation of glucose c. Neisseria meningitidis d. Staphylococcus aureus
d. Are oxidase positive 14. Waterhouse-Friderichsen syndrome is seen in-
4. Differentiating feature of Neisseria gonococcus from  (Recent Pattern Dec 2016)
Neisseria meningitidis is- (Recent Pattern Aug 2013) a. Pneumococci b. N.meningitidis
a. Lactose fermentation b. Maltose fermentation c. Pseudomonas d. Yersinia
c. Mannitol fermentation d. Sucrose fermentation 15. Vaccine is available against which type of meningococ-
5. Most common genetic play in Neisseria infection is- cus- (Recent Pattern Dec 2013)
 (Recent Pattern Dec 2013) a. Type A b. Type B
a. Male gender c. Type A and C d. Type B and D
b. HLA B27 16. The vaccine against N.meningitidis contains-
c. Complement deficiency  (Recent Pattern Dec 2014)
d. IgA deficiency a. Whole bacteria
6. Where does gonococci initially infect- b. Capsular polysaccharide
 (Recent Pattern Dec 2014) c. Somatic ‘O’ antigen
a. Vagina b. Cervix d. Lipopolysaccharide-protein complex
c. Uterus d. Fallopian tubes 17. Capsular polysaccharide derived vaccine is available for
7. A patient with conjunctival infection, which led to all meningococci except- (Recent Pattern Dec 2016)
corneal perforation was positive for Gram-negative a. Group A b. Group B
coccoid appearance on Gram stain. Further investigation c. Group C d. Group Y
showed small translucent colorless organisms which 18. Selective medium for meningococcal infection is-
are oxidase positive. What could be the most probable  (Recent Pattern Nov 2015)
causative organism- (AIIMS May 2013) a. DCA b. Blood agar
a. Moraxella catarrhalis c. LJ medium d. Thayer Martin medium
b. Neisseria gonorrhoeae 19. Best site for collection of specimen for N. meningitidis-
c. Pseudomonas aeruginosa  (Recent Pattern Dec 2013)
d. Acinetobacter lwoffi a. Oral swab
8. Virulence of gonococci is due to- b. Nasal swab
 (Recent Pattern Dec 2012) c. Nasopharyngeal swab
a. Pili b. Endotoxin d. Skin lesions
c. Exotoxin d. None 20. Most severe form of conjunctivitis in neonates is caused
9. Transport medium for gonococcus- by:  (Recent Pattern 2018)
 (Recent Pattern June 2014) a. Chlamydia trachomatis
a. Pike’s medium b. Stuart medium b. Staphylococcus aureus
c. VR medium d. Thayer Martin medium c. Neisseria gonorrhea
10. Selective medium for gonococci- d. Streptococcus agalactiae
 (Recent Pattern Dec 2014) 21. Non-gonococcal urethritis is/are caused by: 
a. Thayer-Martin medium b. LJ medium  (PGI Nov 2017)
c. DCA medium d. MacConkey’s medium a. Mycoplasma hominis
11. CDC recommended treatment for uncomplicated gon- b. Mycoplasma genitalium
orrhea- (Recent Pattern July 2015) c. Chlamydia trachomatis
a. Ceftriaxone b. Tetracycline d. Treponema pallidum
c. Azithromycin d. Cotrimoxazole e. None of above 93
ANSWERS AND EXPLANATIONS
Unit 2     Bacteriology

1. Ans.  (b) Gram-negative Diplococci 7. Ans.  (b) Neisseria gonorrhoeae


Ref: Ananthanarayan and Paniker’s Textbook of Microbiol- Ref: Ananthanarayan and Paniker’s Textbook of Microbiol-
ogy – 10th ed – Page 230 ogy – 10th ed – Page 235
• The genus Neisseria consists of gram negative cocci
• • N. gonorrhoeae can cause conjunctivitis and corneal

arranged in pairs (Diplococci). perforation.


• It contains two important pathogens, N. meningitidis
• • Colonies of Gonococci are small, round, translucent,

and N. gonorrhoeae convex or slightly umbonate with a finely granular


structure and lobate margins.
2. Ans.  (c) Intracytoplasmic Gram-negative • They are gram-negative Diplococcic that are oxidase

Ref: Ananthanarayan and Paniker’s Textbook of Microbiol- positive. They are predominantly found within
ogy – 10th ed – Page 211 polymorphs.

• N. gonorrhoeae is predominantly located inside the



8. Ans.  (a) Pili
polymorphs (intracellular).
• Intracytoplasmic localization of N. gonorrhoeae was

Ref: Ananthanarayan and Paniker’s Textbook of Microbiol-
confirmed by electron microscopy. ogy – 10th ed – Page 234
• Pili are hair-like structures that act as virulence actors in

3. Ans.  (b) Possess a capsule gonococci by promoting attachment to host cells and by
Ref: Ananthanarayan and Paniker’s Textbook of Microbiol- inhibiting phagocytosis.
ogy – 10th ed – Page 230 • Pili undergo antigenic and phase variation.

• The main difference between meningococci and


9. Ans.  (b) Stuart medium


gonococci is the presence of a polysaccharide capsule in
N. meningitidis which is absent in N. gonorrhoeae. Ref: Ananthanarayan and Paniker’s Textbook of Microbiol-
ogy – 10th ed – Page 237
4. Ans.  (b) Maltose fermentation • Stuart’s medium is the transport medium used for

Ref: Ananthanarayan and Paniker’s Textbook of Microbiol- specimens suspected to contain gonococci.
ogy – 10th ed – Page 234 • Specimens should be collected in charcoal impregnated

swabs and sent to the laboratory in Stuart’s transport


• Maltose fermentation differentiates meningococci from

medium.
gonococci.
• Meningococci ferments maltose but gonococci do not

10. Ans.  (a) Thayer-Martin medium
ferment it.
Ref: Ananthanarayan and Paniker’s Textbook of Microbiol-
5. Ans.  (c) Complement deficiency ogy – 10th ed – Page 234

Ref: Ananthanarayan and Paniker’s Textbook of Microbiol- • The selective medium used for isolation of gonococci is

ogy – 10th ed – Page 232 Thayer-Martin medium.


• Thayer-Martin medium is nothing but chocolate agar
• Meningococcal disease is favored by deficiency of

added with vancomycin, colistin and nystatin.


terminal complement components. (C5-C9)
11. Ans.  (a) Ceftriaxone
6. Ans.  (b) Cervix
Ref: Ananthanarayan and Paniker’s Textbook of Microbiol-
Ref: Ananthanarayan and Paniker’s Textbook of Microbiol- ogy – 10th ed – Page 237
ogy – 10th ed – Page 235
• The CDC recommended treatment for uncomplicated
• In women the initial gonococcal infection involves

gonorrhea (in 1993) is ceftriaxone 125-mg single IM


urethra and cervix. The vaginal mucosa is not usually dose or ciprofloxacin 500 mg single oral dose.
affected in adults because the stratified squamous • Since 2006, due to increase in ciprofloxacin resistance,

epithelium is resistant to infection by cocci and also the dosage is 125 mg IM single dose, with azithromycin
because of acidic pH of vaginal secretions. single dose or with doxycycline 100 mg twice a day for
7 days.

94
12. Ans.  (a) Neisseria meningitidis

Chapter 9     Neisseria
18. Ans.  (d) Thayer martin medium
Ref: Ananthanarayan and Paniker’s Textbook of Microbiol-
ogy – 10th ed – Page 232 Ref: Ananthanarayan and Paniker’s Textbook of Microbiol-
ogy – 10th ed – Page 231
• In meningococcemia, the patient presents with acute
• Selective medium for meningococcal infection is modi-

fever, malaise, prostration, a typical petechial rash. •

• Meningitis with rash is a feature of meningococcal



fied Thayer-Martin medium (with vancomycin, colistin
meningitis. and nystatin).

13. Ans.  (c) Neisseria meningitidis 19. Ans.  (c) Nasopharyngeal swab

Ref: Ananthanarayan and Paniker’s Textbook of Microbiol- Ref: Ananthanarayan and Paniker’s Textbook of Microbiol-
ogy – 10th ed – Page 232 ogy – 10th ed – Page 232

• Meningococcemia presents as fever, chills, malaise,



• Nasopharyngeal swab is the best site for collection of

petechial rash. About 10% develop pneumonia. samples for identification of meningococcal carriers
• A few develop fulminant meningococcemia (Waterhouse

because human nasopharynx is the only reservoir of
– Friderichsen syndrome) which is an overwhelming meningococcus.
and fatal condition characterized by shock, DIC and • Healthy carriers can serve to infect their contacts.

multisystem failure. • An increase in carrier rate heralds the onset of an


• The case scenario given here is fulminant meningococ-



epidemic.
cemia.
20. Ans.  (c) Neisseria gonorrhea
14. Ans.  (b) N. meningitidis Ref: T.B of Opthalmology – Khurana – 4th edition – Page 55,
Ref: Ananthanarayan and Paniker’s Textbook of Microbiol- 71
ogy – 10th ed – Page 232 • The causes of neonatal conjunctivitis are Neisseria

• Refer answer no.13 gonorrhea, Chlamydia trachomatis


• Neisseria gonorrhea causes Ophthalmia neonatorum

15. Ans.  (c) Type A and C which is most severe of all; nowdays due to prophylaxis
the prevalance is decreased
Ref: Ananthanarayan and Paniker’s Textbook of Microbiol- • Complications are untreated cases, especially of gono-

ogy – 10th ed – Page 233 coccal ophthalmia neonatorum, may develop corneal
• Monovalent and polyvalent vaccines containing the
• ulceration, which may perforate rapidly resulting in cor-
capsular polysaccharides of groups A, C, W-135 and Y neal opacification or staphyloma formation.
are available.
• There is no group B vaccine available at present.

21. Ans:  (a) Mycoplasma hominis; (b) Mycoplasma geni-
talium; (c) Chlamydia trachomatis
16. Ans.  (b) Capsular polysaccharide
Ref: Ananthanarayan and Paniker’s Textbook of Microbiol-
Ref: Ananthanarayan and Paniker’s Textbook of Microbiol- ogy – 10th ed – Page 237
ogy – 10th ed – Page 233 Causative organisms for NGU are:
• Meningococcal vaccines contain capsular polysaccha-
• • Chlamydia trachomatis

rides. The vaccines induce good immunity after a single • Ureaplasma urealyticum

dose in older children and adults. • Mycoplasma hominis


• Mycoplasma genitalium

17. Ans.  (b) Group B • Herpes


Ref: Ananthanarayan and Paniker’s Textbook of Microbiol- • Cytomegalovirus


ogy – 10th ed – Page 233 • Gardnerella vaginalis


• Acinetobacter lwoffi
• Monovalent and polyvalent vaccines containing the

• Candida albicans

capsular polysaccharides of groups A, C, W-135 and Y • Trichomonas vaginalis


are available.
• There is no group B vaccine available at present.

• Immunity is type specific.


95
10
Corynebacterium
CORYNEBACTERIUM SPECIES Pathogenesis
y The primary modes of spread consist of close contact with
• Gram-positive rods

y
infectious material from respiratory secretions (direct or via

• Nonmotile airborne droplet) or from skin lesions

• No capsule y Humans are the only reservoir of infections

• Confirmatory tests: Albert stain – shows metachromatic

y
y Infections are mainly transmitted from asymptomatic

granules; Growth in Loeffler’s serum slope

y
carriers.
y Diphtheria bacilli produce powerful exotoxin that is

y
Table 1: Different species of Corynebacterium responsible for pathogenesis

Corynebacterium Species
High Yield
Main pathogenic C. diphtheriae
• Characteristics of diphtheria toxin:
Lipophilic C. jeikeium

ƒ Lethal dose: 0.1 ug/kg
C. minutissimum
ƒ
ƒ Structure: It has two subunits; A – Active domain which is
ƒ
Nonlipophilic C. ulcerans responsible for action and B – binding domain which trig-
C. pseudotuberculosis ger the host cell
C. minutissimum ƒ Host cell receptor – CD9 and HBEGR like precursor which is
ƒ
Diphtheroids C. xerosis is located on cell membrane of the host cell.
C. pseudodiphtheriticum ƒ Mechanism: Entry of bacteria is by receptor mediated
ƒ
endocytosis
Associated genera Propionibacterium acnes ƒ Action: It inhibits the protein synthesis by inactivating EF2
P. granulosum
ƒ
(elongation factor 2)
P. avidum
C. parvum y Standard strain that is used for toxin production is Park
y
Williams 8 strain
y Factors affecting toxin production:
CORYNEBACTERIUM DIPHTHERIAE
y
 Concentration of iron has an influence in toxin production

y Gram-positive rods that have characteristic club shaped  Optimum level of iron for toxin production is 0.1 mg/L

y
appearance because of swelling at end (due to presence of  Inhibition of toxin occurs when iron level is 0.5 mg/L

metachromatic granules)  Toxin production depends on presence of corynephages

y Due to palisade arrangement of bacilli – it gives Chinese called tox + bacteriophages
y
character pattern
y Granules are storage factor also called as volutin granules or High Yield
y
Babes-Ernst granules
Toxin production is based on following:
There are at least four biotypes of  Corynebacterium diph- • Presence of bacteriophages


theriae: gravis, intermedius, mitis and belfanti • Concentration of iron

• Biotype

• DT repressor gene – the gene that is responsible for iron
Remember

regulator
Metachromatic granules are seen in: (Mneumonic: Cellphone-
GSM)
• Corynebacterium diptheria Clinical Features

• Gardnerella vaginalis y Toxin produced by diphtheria bacilli gets absorbed into the

y
• Spirillum volutans mucous membranes and causes necrosis

• Mycobacterium (few species) y Necrosed membranes leads to pseudomembrane formation

y
y Removal of pseudomembrane leads to bleeding
y
y Regional lymph nodes are enlarged that gives an appearance y Microscopic detection of C. diphtheriae:

Chapter 10     Corynebacterium


y y

of Bull neck  Immunofluorescence: Diptheria bacilli can be detected




y Pharyngeal diphtheria is the most commonest type


y
from direct smears by staining with specific antibody;
y Toxin absorption leads to systemic spread – affects all system
y
more specific method;
that may present as: y To know whether a person is susceptible to diphtheria or not:
y

 Myocarditis


certain tests are done:


 Degeneration of liver  In vivo test: Schick test (neutralization based assay) – not


 Tubular necrosis of kidney




in use now
 Demyelination
 In vitro test: Measurement of antitoxin levels—antitoxin


 Nerve palsy


levels > 0.1 IU/mL of blood is confirmed as protective


 Soft palate paralysis


immunity
 Hemorrhage
y Typing methods used for Corynebacterium:


y Complications are:
y

 Biotyping – based on biochemical reactions


y

 Asphyxia


 Ribotyping – Most useful method presently for typing




 Circulatory failure


the strains of C. diphtheriae




 Postdiphtheritic paralysis


 Molecular methods: Pulse field gel electrophoresis,


 Mechanical obstruction 

y Cutaneous diphtheria can occur in alcoholics, homeless


y
Random amplification of polymorphic DNA or amplified
strangers and in temperate climates fragment length polymorphism
y Nontoxigenic strains can cause infections even in
y
 Bacteriophage typing


immunized individuals (as immunization is against only


toxin producing strains) – but those infections are usually Table 2:  Different types of diphtheria bacilli
mild
Gravis ••Colonies are called as Daisy head colony
• Variable hemolysis
Laboratory Diagnosis

••Glycogen and starch fermentation positive


y To demonstrate polar bodies:
y
••High case fatality rate
 Albert staining

••Paralytic complications and hemorrhage are
 Neisser’s stain

more
 Ponder’s stain

• Most associated with epidemics

Intermedius •• Colonies are called as Frogs’ egg colony


•• Nonhemolytic
•• Glycogen and starch fermentation negative
•• High case fatality rate
•• Hemorrhage complications are more
•• More associated with epidemics
Mitis •• Colonies are called as Poached egg colony
•• Usually hemolytic
•• Glycogen and starch fermentation negative
•• Less lethal
•• Obstructive lesions are more
•• More associated with endemics

Treatment
Never wait for laboratory results when you suspect diphtheria
clinically. If the clinical features shows diphtheria – send
Figure 1: Albert stain for C. diphtheriae (Courtesy: CDC) sample and start treatment immediately
y Diphtheria antitoxin should be given in respiratory diphtheria
y

y Selective medium: – 20,000 to 1,00,000 units


y Procaine Penicillin is the DOC followed by oral penicillin V
y

 Loeffler’s serum slope


y

– 14 days


 Potassium tellurite blood agar


y Alternative drug – Erythromycin – 14 days course


 Modified Tinsdale’s medium


y

y Close contacts and treating physicians should receive 7–10




y Hiss serum sugars – biochemical test helpful in diagnosis y

days of oral erythromycin or one dose of IM benzathine


y

y Toxin demonstration is by: (culture should be tested for toxin)


penicillin (vaccinated contacts)
y

 In vivo test – subcutaneous test and intracutaneous test


y When the contacts are not vaccinated or vaccination history


 In vitro test – Elek’s gel precipitation test, Tissue culture


y

is unknown – it is must to provide Tdap vaccine (for adults)




test, PCR and ELISA 97


 Immunochromatographic strip assay

Vaccination The United States Centers for Disease Control and Prevention
Unit 2     Bacteriology

(CDC) recommends that adults receive a diphtheria-toxoid–


y In India, according to the schedule at 6, 10, 14 weeks – DPT
containing vaccine (Tdap or Td as indicated) every 10 years after
y

vaccine is provided followed by booster doses


completing a primary childhood vaccination series (not followed
y Tdap vaccine should be given in adults for those who do not
in India)
y

have previous history of vaccination

OTHER CORYNEBACTERIA

Table 3:  Features of other Corynebacteria


Corynebacteria Features
C. minutissimum • Causative agent for Erythrasma

• Affects axilla and groin


• It is a lipophilic corynebacterium

C. jeikeium •• Causes nosocomial infections


•• Causes infective endocarditis
•• High mortality
•• Usually multi drug resistant
•• Often responds only to vancomycin

Arcanobacterium hemolyticum • Causes pharyngitis


• Causes wound infections


Diphtheroids • C. pseudodiphtheriticum

• C. xerosis

• Diphtheroids are nondiphtherial corynebacteria which are Gram-positive bacilli arranged


in palisade appearance – cause infections in immunosuppresed states as they are normal


commensals.
• Clinically resembling Diphtheria is caused by C. ulcerans and C. pseudotuberculosis

• Differentiation is by urease test which is positive in above organisms and negative in


C. diphtheriae
Corynebacterium parvum • Immunomodulator

98
MULTIPLE CHOICE QUESTIONS

Chapter 10     Corynebacterium


1. C. diphtheriae is also called as- (Recent Pattern 2012) 10. Lysogenic conversion is seen in- 

a. Koch’s bacillus  (Recent Pattern Nov 2014)


b. Roux bacillus a. Diphtheria b. Salmonella
c. Klebs-Löffler bacillus c. Staphylococcus d. E. coli
d. Yersin bacillus 11. Not true about Corynebacterium hofmannii-
2. Daisy head colonies are seen with-  (Recent Pattern 2016)
 (Recent Pattern Dec 2013) a. A diphtheroid
a. Staphylococcus aureus b. Nonpathogenic saprophyte
b. Corynebacterium diphtheriae c. Toxigenic
c. Staph pyogenes d. Also known as C. pseudodiphthericum
d. Anthrax 12. Most dangerous type of diphtheria-
3. Poached egg colony on tellurite agar is produced by  (Recent Pattern July 2016)
which strain of Corynebacterium diphtheriae- a. Faucial b. Laryngeal
 (Recent Pattern Dec 2015) c. Nasal d. Cutaneous
a. Gravis b. Mitis 13. ‘Bull-neck’ deformity is seen in severe cases of which of
c. Intermedius d. Minimus the following? (AIIMS May 2014)
4. One of the following about Corynebacterium diphtheri- a. Diphtheria
ae is wrong- (Recent Pattern Dec 2014) b. Tubercular lymphadenitis
a. Gram-negative c. Mumps
b. Ernst Babes granules seen d. Goiter
c. Chinese letter pattern 14. Most common complication of diphtheria is-
d. Prevented by immunization  (Recent Pattern 2016)
5. A 10-year-old child presented with severe sore throat a. Myocarditis b. Pneumonia
and a gray pseudomembrane over pharynx and tonsils. c. Meningitis d. Endocarditis
The organism causing this infection is most likely- 15. Phage mediated change in C. diphtheriae is which of the
 (AIIMS May 2014) following- (AIIMS Nov 2015)
a. A Gram-positive bacillus a. Transformation b. Transduction
b. A Gram-negative bacillus c. Conjugation d. None of the above
c. A catalase positive Gram-positive coccus that grows in 16. True about Corynebacterium diphtheriae-
clusters  (Recent Pattern 2012)
d. A single-stranded positive sense RNA virus a. All types produce toxin
6. Diphtheria toxin acts by- (Recent Pattern Dec 2014) b. Toxin production is dependent upon critical concen-
a. Inhibiting acetyl choline release tration of iron
b. Inhibiting glucose transport c. Heat stable toxin
c. Increasing levels of cyclic AMP d. Inhibit cAMP
d. Inhibiting protein synthesis 17. What genetic peculiarity is there in pathogenic/
7. Receptor for diphtheria toxin lies at- toxigenic strain of Corynebacterium diphtheriae but not
 (Recent Pattern July 2015) in nonpathogenic (nontoxigenic) strain-
a. Cell membrane b. Mucous membrane  (Recent Pattern July 2016)
c. Nucleus d. None a. Presence of R-factor
8. Toxin production by Corynebacterium diphtheriae is b. Presence of (tox-) gene
due to- (Recent Pattern Dec 2013) c. Integrated temperate gene
a. Chromosomal mutation d. Presence of MDR gene
b. Presence of plasmid 18. Elek’s gel precipitation test is seen in-
c. Lysogenic transversion  (Recent Pattern Aug 2013)
d. Transformation a. Clostridium b. Corynebacterium
9. Which of the following statements about C. diphtheriae c. Bacteroides d. Campylobacter
is false- (All India 2011) 19. A child comes with fever, cold, cough, membrane over
a. Toxin production is mediated by native chromosome tonsils, nasal swab is taken. Culture should be done on
b. Organism may be identified by tests for toxigenicity which medium for earliest diagnosis-
c. Toxin acts by inhibiting protein synthesis  (Recent Pattern 2012)
d. Toxin may affect the heart and nerves a. Loeffler’s serum slope b. LJ media
c. McConkey’s agar d. Citrate media

99
20. Confirmatory tests for C. diphtheriae- 25. Erythrasma is caused by- (Recent Pattern 2012)
Unit 2     Bacteriology

 (Recent Pattern Nov 2014) a. Corynebacterium minutissimum


a. Serological tests b. Diphtheriae
b. Isolation in selective media c. C. ulcerans
c. Tests for toxin d. C. vaginale
d. Gram staining 26. A child presents to the hospital with fever, sore throat,
21. Ehrlich phenomenon is seen in- malaise, and cervical lymphadenopathy. On clinical
 (Recent Pattern Nov 2014) examination, grey to white exudates is seen in the
a. Mycobacterium tuberculosis pharynx. A stain was used and following organism seen.
b. Proteus The dye used for staining contains: 
c. Staphylococcus  (AIIMS Nov 2018)
d. Corynebacterium
22. Most important element in treatment of diphtheria-
 (Recent Pattern Dec 2013)
a. Antitoxin b. Tetracycline
c. Erythromycin d. Penicillin
23. Schick test is for- (Recent Pattern 2012)
a. Susceptibility to diphtheria
b. Susceptibility to tetanus
c. Susceptibility to scarlet fever
d. Susceptibility to RF
24. Toxin is/are produced by all except-
 (Recent Pattern Dec 2014)
a. Corynebacterium diphtheriae
a. Carbolfuschin, acetone, methylene blue
b. Corynebacterium hofmannii
b. Carbolfuschin, acetone, iodine
c. Corynebacterium ulcerans
c. Malachite green, toluidine blue, Iodine
d. Corynebacterium pseudotuberculosis
d. Iodine, Brilliant green, acetone

ANSWERS AND EXPLANATIONS


1. Ans.  (c) Klebs-Löffler bacillus Gravis Daisy head colony
Ref: Ananthanarayan and Paniker’s Textbook of Microbiol- Intermedius Frog’s egg colony
ogy – 10th ed – Page 239 Mitis Poached egg colony
• C. diphtheriae is known as Klebs-Löffler bacillus.

• It was first observed and described by Klebs and first



4. Ans.  (a) Gram-negative
cultivated by Leffler and hence the name.
Ref: Ananthanarayan and Paniker’s Textbook of Microbiol-
ogy – 10th ed – Page 239
2. Ans.  (b) Corynebacterium diphtheriae
• Corynebacterium diphtheriae are Gram-positive bacilli.

Ref: Ananthanarayan and Paniker’s Textbook of Microbiol- • They are nonacid fast, nonmotile with irregularly stained

ogy – 10th ed – Page 243 segments due to the presence of granules.


• On tellurite blood agar, Gravis type of C. diphtheriae

produces flat colony with raised dark center and 5. Ans.  (a) A Gram-positive bacillus
crenated edge with radial striation giving daisy head Ref: Ananthanarayan and Paniker’s Textbook of Microbiol-
appearance - Daisy head colony. ogy – 10th ed – Page 239
• The organism causing leathery pseudomembrane is
3. Ans.  (b) Mitis

Corynebacterium diphtheriae.
Ref: Ananthanarayan and Paniker’s Textbook of Microbiol- • It is a Gram-positive bacillus.

ogy – 10th ed – Page 243


6. Ans.  (d) Inhibiting protein synthesis
• Poached egg colony is produced by mitis type of C.

diphtheriae. Ref: Ananthanarayan and Paniker’s Textbook of Microbiol-


• Mitis type colonies appear flat with central elevation

ogy – 10th ed – Page 240
(Poached egg appearance) in 2–3 days of incubation in • The diphtheria toxin acts by inhibiting protein synthesis.

tellurite blood agar. • Fragment B of the toxin helps in binding and fragment
100

• Colony morphology of the three types of C. diphtheriae


• A inhibits polypeptide chain elongation by inactivating
on tellurite blood agar the elongation factor, EF-2.
• It has special affinity for certain tissues such as 14. Ans.  (a) Myocarditis

Chapter 10     Corynebacterium


myocardium, adrenals and nerve endings.


Ref: Ananthanarayan and Paniker’s Textbook of
7. Ans.  (a) Cell membrane Microbiology – 10th ed – Page 241; Essentials of Medical
Microbiology – Apurba Sankar sastry Page 245.
Ref: https://www.ncbi.nlm.nih.gov/pubmed/7836353
• Myocarditis and polyneuropathy are the late toxic

• The toxin binds to heparin-binding epidermal growth



manifestations of diphtheria occurring after weeks of
factor precursor (HB-EGF) present on cell membrane of infection.
the host cell. The complex undergoes endocytosis by the
host cell. 15. Ans.  (d) None of the above

8. Ans.  (c) Lysogenic transversion Ref: Ananthanarayan and Paniker’s Textbook of Microbiol-
ogy – 10th ed – Page 60 and 240
Ref: Ananthanarayan and Paniker’s Textbook of Microbiol-
• Lysogenic conversion contributes to toxigenicity of C.
ogy – 10th ed – Page 240

diphtheriae. (corynephage mediated)


• The toxigenicity of diphtheria bacillus depends on the

• In transduction, the phage only acts as a vehicle carrying

intracellular presence of corynephages (tox+), which act bacterial genes from one cell to other but in lysogenic
as the genetic determinant in toxin production. conversion, the phage DNA itself is the new genetic
• The infecting phage (beta or others) renders the

element.
strain toxigenic – this is known as lysogenic or phage
conversion. 16. Ans.  (b) Toxin production is dependent upon critical
concentration of iron
9. Ans.  (a) Toxin production is mediated by native
chromosome Ref: Ananthanarayan and Paniker’s Textbook of Microbiol-
ogy – 10th ed – Page 240
Ref: Ananthanarayan and Paniker’s Textbook of Microbiol-
• Toxin production by diphtheria bacillus depends on the
ogy – 10th ed – Page 240

intracellular presence of corynephages.


Refer answer no.8 • However, toxin production is also influenced by the

concentration of iron in the medium. The optimum


10. Ans.  (a) Diphtheria level of iron for toxin production is 0.1 mg/L, while a
concentration of 0.5 mg/L inhibits the formation of toxin
Ref: Ananthanarayan and Paniker’s Textbook of Microbiol-
ogy – 10th ed – Page 240 17. Ans.  (b) Presence of (tox-) gene
• Lysogenic conversion is responsible for the toxicity of C.

Ref: Ananthanarayan and Paniker’s Textbook of Microbiol-


diphtheriae. ogy – 10th ed – Page 240
• Strains which are rendered toxigenic by infection with
• Presence of tox gene contributes to the toxigenicity of

corynephages can produce toxin and cause disease. •

the pathogenic C. diphtheriae strain.


11. Ans.  (c) Toxigenic • Those strains that lack the tox gene are nonpathogenic.

Ref: Ananthanarayan and Paniker’s Textbook of Microbiol- 18. Ans.  (b) Corynebacterium
ogy – 10th ed – Page 246
Ref: Ananthanarayan and Paniker’s Textbook of Microbiol-
• C. hofmanii is a diphtheroid. It is a normal commensal

ogy – 10th ed – Page 243


found in throat. It does not produce toxin.
• It is also called C. pseudodiphtheriticum.

• Elek’s gel precipitation test is done for demonstrating

the toxicity of C. diphtheriae.


12. Ans.  (b) Laryngeal • It is an in vitro test to demonstrate toxigenicity.

Ref: Ananthanarayan and Paniker’s Textbook of Microbiol- 19. Ans.  (a) Loefflers serum slope
ogy – 10th ed – Page 241
Ref: Ananthanarayan and Paniker’s Textbook of Microbiol-
• Asphyxia due to mechanical obstruction of the

ogy – 10th ed – Page 242


respiratory passage by pseudomembrane can occur
if pseudomembrane extends to larynx. It is the most • The features given in the scenario are suggestive of

dangerous type requiring emergency tracheostomy. diphtheria. The bacteria causing diphtheria is C.
diphtheriae.
13. Ans.  (a) Diphtheria • Growth of C. diphtheria is rapid on Loeffler’s serum

slope. Colonies can be visualized in 6-8 hours. So the


Ref: Ananthanarayan and Paniker’s Textbook of Microbiol- media to be inoculated is Loeffler’s serum slope.
ogy – 10th ed – Page 241
• In malignant or hypertoxic diphtheria, there is severe
101

toxemia with adenitis (enlargement of lymphnodes in


the neck) giving a bull neck appearance.
20. Ans.  (c) Tests for toxin 24. Ans.  (b) Corynebacterium hofmannii
Unit 2     Bacteriology

Ref: Ananthanarayan and Paniker’s Textbook of Microbiol- Ref: Ananthanarayan and Paniker’s Textbook of Microbiol-
ogy – 10th ed – Page 242 ogy – 10th ed – Page 246
• Any isolate of C. diphtheriae should be tested for
• • C. hofmanii is a diphtheroid. It is a normal commensal

toxigenicity for the bacteriological diagnosis to be found in throat. It does not produce toxin.
complete. • All the other three species given as options are toxin

• It may be done by the in vivo (subcutaneous and


• producers.
intra cutaneous tests) or in vitro methods (Elek’s gel
precipitation test.) 25. Ans.  (a) Corynebacterium minutissimum
• Tissue culture test, PCR for tox gene detection and ELISA

Ref: Ananthanarayan and Paniker’s Textbook of Microbiol-


for toxin detection are other diagnostic tests. ogy – 10th ed – Page 246
21. Ans.  (d) Corynebacterium • Erythrasma is a localized infection of the stratum

corneum usually affecting the axilla and groin.


Ref: https://medical -dictionary.thefreedictionary.com/Eh- • It is caused by Corynebacterium minutissimum.

rlich + phenomenon • This is a lipophilic corynebacteria which can be readily


• Ehrlich phenomenon is the difference between the amount


• grown in media containing 20% fetal calf serum.
of diphtheria toxin that will exactly neutralize one unit of
antitoxin. 26. Ans. (c) Malachite green, toluidine blue, Iodine
Ref: Monica cheesbrough Laboratory practice in tropical
22. Ans.  (a) Antitoxin
countries - Vol II – page 44
Ref: Ananthanarayan and Paniker’s Textbook of Microbiol- • Corynebacterium diphtheriae causes diphtheria; clin-

ogy – 10th ed – Page 245 ically it presents as fever, malaise, cervical lymphade-
• Specific treatment of diphtheria consists of antitoxic and
• nopathy (bull neck appearance) and pseudomembrane
antibiotic therapy. formation in the throat.
• Antitoxin should be given immediately when diphtheria
• • Most common type is pharyngeal (faucial) diphtheria.

is suspected, as the fatality rate increases with delay in • C. diphtheria is diagnosed clinically by staining of throat

starting antitoxic treatment. swab with special stains like


• Antibiotic treatment only supplements and does not
• ƒ Albert stain
ƒ

replace antitoxic therapy. ƒ Ponders’ stain


ƒ

ƒ Neisser’ stain
ƒ

23. Ans.  (a) Susceptibility to diphtheria • The above image is Albert staining method which

Ref: Ananthanarayan and Paniker’s Textbook of Microbiol- has green bacilli with brownish black metachromatic
ogy – 10th ed – Page 245 granules
• Albert stain has two different staining compositions:
• Schick test is done for determining the susceptibility of

First dye is Toluidine blue, Malachite green and glacial


an individual to diphtheria. acetic acid; the Second dye is Iodine
• It is an in vivo skin based on neutralization test to

determine the antitoxin level in circulation.

102
11
Bacillus
BACILLUS SPECIES y Mechanism of action of toxin:

y
 Upon entry into the host cells, the individual toxin


fractions are split
• Gram-positive rod
 EF and LF are internalised into the cell

• Aerobic, spore producing


 These factors act upon endosomes and lead to increased

• Occurs in chains


cAMP – oedema

y Based upon the portal of entry and clinical presentation – it

y
Bacillus Anthracis is of four types:
 Cutaneous anthrax (95%)


 Inhalational anthrax (5%)
Remember

 Gastrointestinal anthrax (rare)


Special Features (First of Many)  Injection anthrax (IV drug users)

• First pathogenic bacterium to be looked under microscope
Cutaneous Anthrax: (Hide Porter’s Disease)

• First communicable disease that has been identified as

transmission by infected blood y Occurs usually in face and neck
• First bacillus to be isolated in pure culture
y
y A papule is seen at the site of entry followed by vesicle and

• First bacterium identified that possess spores
y
necrotic ulcer.

• First bacterium used for the preparation of an attenuated y The lesion characteristic of cutaneous anthrax is central

vaccine
y
black eschar

Characteristics
y Gram-positive rods
y
y Non-motile bacilli (Exception in Bacillus family)
y
y Spores are formed only in culture of bacilli and in the soil;
y
sporulation never occurs in the animal body
y Spores are highly resistant; it resist heat at 140°C for 1–3 hours
y
and boiling for 10 minutes.

Pathogenesis
y Humans acquire infection incidentally by contact with
y
infected animals or animal products.
y Portal of entry is spores entering through injured skin or
y
through mucous membranes; very rarely biting of insects like Figure 1: Central black eschar in cutaneous anthrax
Stomoxys calcitrans can transmit mechanically. (Courtesy: CDC/ F. Marc LaForce, MD)
y Main virulence factors responsible for disease are: y This lesion is called as malignant pustule
y
y
 Capsular polypeptide (the only bacterium where capsule y It generally resolves spontaneously.

y
is made of polypeptide; rest of the capsules are made up of
polysaccharides) – polypeptide nature is poly D glutamic Pulmonary Anthrax: (Wool-sorter’s Disease)
acid
y Incubation period can go for as long as 6 weeks
 Anthrax toxin: (made up of three fractions)
y
y Marker hemorrhagic necrosis and edema of mediastinum

Š Protective antigen factor (PA)
y
can be seen
Š
Š Edema factor (EF) y Hemorrhagic meningitis and bowel ulceration may also be
Š
y
Š Lethal factor (LF) seen
Š
y Toxin and capsule production is mediated by plasmid y Case fatality rate is higher
y
y
Intestinal Anthrax y Selective medium used for B. anthracis is PLET Medium –
Unit 2     Bacteriology

Polymyxin, Lysozyme, EDTA and thallous acetate


y This condition is relatively rare as the mode of infection is
y Confirmatory test for anthrax:
y

eating carcasses of animals dying of anthrax


y

 Lysis by specific anthrax gamma bacteriophage


y Not seen nowadays


 Detection of capsule by fluorescent antibody


y

 Identification of toxin genes by PCR


Anthrax in India


y Anthrax is an enzootic disease in India


y
Table 1:  CDC updated postexposure and treatment
y Infections are acquired from cattle’s – most affected regions
y
guidelines for Anthrax
Andhra Pradesh and Tamil Nadu border
Uncomplicated Fluoroquinolones or doxycycline for 7-14
cutaneous anthrax days
Laboratory Diagnosis
y Anthrax is a highly contagious bioterrorism warfare agent –
y
Systemic anthrax Antitoxin with antimicrobial therapy
hence specimens need to handled only in Biosafety level II/ Inhalational anthrax, Human anthrax immunoglobulin
III cabinets systemic cases which (Anthrasil), Monoclonal antibodies to
y Specimens are taken based on type of infection like fluid, pus,
y
are severe protective antigen – Obiltoxaximab or
blood, CSF, stool and animal carcasses Raxibacumab
y Gram staining shows:
y

Anthrax meningitis Three antimicrobial drugs – Clindamycin


 Gram-positive bacilli in chains with central spores – giving

with other groups for 60 days
the appearance of Bamboo stick
Postexposure Ciprofloxacin or doxycycline for 60 days
prophylaxis for along with vaccine - AVA BioThrax
inhalational anthrax

Vaccination
y y Pasteur’s anthrax vaccine
y y Sterne vaccine
y y Mazzuchi vaccine
y y Anthrax vaccine adsorbed (AVA) vaccine for human use (AVA
BioThrax) – pre-exposure prophylaxis – five doses need to be
given for high risk people (this vaccine is available only US
Department of defense)

Figure 2: Bamboo stick appearance of Bacillus anthracis Bacillus Cereus


(Courtesy: CDC)
y B. cereus is motile, noncapsulated
y

y Demonstration of capsule is done by M’Fadyean’s reaction –


y
y It causes food poisoning in two forms:
y

stain used is polychrome methylene blue  Emetic type – associated with fried rice, milk and pasta


y Culture of specimens in blood agar and gelatin stab culture


y
 Diarrheal type – associated with meat and sauces


shows: y It causes preformed toxins – food intoxication


y

 Colonies of B. anthracis when looked under microscope




– the edge of the colonies has a comma shaped out Table 2:  Types of food intoxication caused by Bacillus
growths like curly hair – which is called as Medusa head cereus 
appearance
 In gelatin stab culture – it has a inverted fir tree appearance
 Emetic type Diarrheal type
Due to heat stable enterotoxin Due to heat labile enterotoxin
High Yield Preformed toxin in the food Either preformed or it may
Special features in diagnosis of B. anthracis form in the intestine
• Bamboo stick appearance

Incubation period = 1–5 hours IP = 8–16 hours


• M’Fadyean’s reaction
Usually occurs due to Due to any food

• Medusa head appearance


consumption of fried rice

• Inverted fir free appearance


• String of pearls reaction


• Nausea and vomiting; No Diarrhea and abdominal pain
diarrhea
y In solid medium which has penicillin in it – B. anthracis grows
Caused by serotypes 1, 3, 5 Caused by serotypes 2, 6, 8, 9,
y

well and looks like String of Pearls (this helps to differentiate


it from a closed species named as B. cereus) 10, 12
104
y Diagnosis is by culture of feces or vomitus in MYPA agar – Treatment

Chapter 11     Bacillus


y

mannitol-egg yolk-phenol red- polymyxin agar


y Food borne infections are mild and self-limiting
y Presence of colony count of ≥105 is diagnostic of intoxication
y

y Nonfood borne infections: Treatment is vancomycin or


y

y B. cereus can also cause eye infections – keratitis and


y

clindamcycin
y

endophthalmitis
y Wound infections: Ciprofloxacin
y It also causes nosocomial infections, catheter infections,
y

pneumonia, osteomyelitis
y It is resistant to most of the antimicrobials including penicillin
y
ANTHRACOID BACILLI
and cephalosporins A large number of bacillus species are nonpathogenic and
collectively called as anthracoid bacilli or pseudoanthrax bacilli
because of resemblance to anthrax bacilli. These are seen in
cultures as contaminants.

Table 3:  Differentiating features of Anthrax and Anthracoid bacilli

Anthrax bacilli Anthracoid bacilli


Nonmotile Generally motile
Capsulated Noncapsulated
Grow in long chains Grow in short chains
Medusa head colony Not present
No growth in penicillin agar Grow usually
Hemolysis absent or weak Usually well marked
Inverted fir tree growth Rapid liquefaction
No turbidity in broth Turbidity seen
Salicin fermentation negative Usually positive
No growth at 45°C Grow usually
Growth is inhibited by chloral hydrate Not inhibited
Susceptible by gamma phage Not susceptible
Pathogenic to lab animals Not pathogenic

105
MULTIPLE CHOICE QUESTIONS
Unit 2     Bacteriology

1. Spore forming anaerobic Gram-positive bacilli- 11. True about anthrax toxin are all except-
 (Recent Pattern July 2016)  (Recent Pattern 2016)
a. Bacillus anthracis b. Clostridia a. Has three fractions
c. Corynebacterium d. Peptostreptococcus b. Increase cAMP
2. Bacillus anthracis is- (Recent Pattern July 2016) c. Coded by plasmid
a. Gram-negative b. Nonmotile d. Inhibits protein synthesis
c. Noncapsulated d. Nonsporing 12. True about Bacillus anthracis- (PGI November 2014)
3. Cell wall of Bacillus anthracis is composed of- a. Zoonotic disease
 (Recent Pattern July 2016) b. Man to man transmission possible
a. Peptidoglycan b. Polysaccharide c. Agent for bioterrorism
c. Lipopolysaccharide d. Polypeptide d. Antibiotic has no role
4. Capsule of Bacillus anthracis is formed of- e. McFadyean’s reaction seen
 (Recent Pattern July 2016) 13. A person working in an abattoir presented with pustule
a. Polysaccharide b. Lipopolysaccharide on hand which turned into ulcer, which will best help in
c. Polypeptide d. Long chain fatty acids diagnosis- (AIIMS Nov 2012)
5. Medusa head colonies on nutrient agar is seen in- a. Polychrome methylene blue
 (Recent Pattern July 2015) b. Carbol fuchsin
a. Pneumococcus b. Legionella c. Acid fast stain
c. Brucella d. Anthrax d. Calcofluor-white
6. Inverted fir tree appearance on gelatin stab is character- 14. Anthrax bacilli differs from anthracoid bacillis by being-
istic of- (Recent Pattern Dec 2014)  (Recent Pattern Dec 2014)
a. Mycoplasma b. Bacillus anthracis a. Noncapsulated
c. Clostridium d. Bacteroides b. Strict aerobe
7. The “string of pearl” colonies on nutrient agar is pro- c. Nonmotile
duced by- (Recent Pattern Dec 2013) d. Hemolytic colonies on blood agar
a. Klebsiella b. Proteus 15. Characteristic of Bacillus cereus causing food poisoning
c. Bacillus d. Salmonella is- (AIIMS Nov 2010)
8. McFadyean reaction seen with which organism- a. Presence of fever
 (Recent Pattern July 2015) b. Presence of pain in abdomen
a. Clostridium perfringens c. Absence of vomiting
b. Clostridium botulinum d. Absence of diarrhea
c. Bacillus cereus 16. A person after 6 hours of consuming rice pudding
d. Bacillus anthracis in restaurant develops vomiting. True statements
9. PLET medium is used in- (Recent Pattern Dec 2015) regarding food poisoning- (PGI Nov 2011, 2013)
a. Plague b. Anthrax a. Caused by Staphylococcus aureus
c. Typhoid d. Cholera b. Preformed toxin
10. ‘Malignant pustule’ is- (Recent Pattern Nov 2014) c. Caused by Vibrio parahaemolyticus
a. Anthrax ulcer d. Heat labile toxin
b. Proliferating rodent ulcer e. Culture of food is more useful than stool
c. Malignant melanoma
d. Marjolin’s ulcer

106
Chapter 11     Bacillus
ANSWERS AND EXPLANATIONS

1. Ans.  (b) Clostridia 7. Ans.  (c) Bacillus


Ref: Ananthanarayan and Paniker’s Textbook of Microbiol- Ref: Ananthanarayan and Paniker’s Textbook of Microbiol-
ogy – 10th ed – Page 248 and 257 ogy – 10th ed – Page 252
• The genus Clostridium consists of spore forming
• • When Bacillus anthracis is grown on solid medium

anaerobic Gram-positive bacilli. containing 0.05 – 0.5 U of penicillin/mL, the cells


• Aerobic, spore forming Gram-positive bacilli are placed
• become large and spherical and occur in chains on the
in the genus Bacillus. surface of the agar, resembling a string of pearls.
• This string of pearls reaction clearly differentiates

2. Ans.  (b) Nonmotile B.anthracis from B.cereus and other aerobic spore
Ref: Ananthanarayan and Paniker’s Textbook of Microbiol- bearers.
ogy – 10th ed – Page 248
8. Ans.  (d) Bacillus anthracis
• Bacillus anthracis is aerobic, spore forming Gram-

positive bacilli. Ref: Ananthanarayan and Paniker’s Textbook of Microbiol-


• It is non motile, unlike most other members of this ge-

ogy – 10th ed – Page 251
nus. • When blood films containing anthrax bacilli are stained

with polychrome methylene blue for few seconds


3. Ans.  (a) Peptidoglycan and examined under the microscope, an amorphous
Ref: Ananthanarayan and Paniker’s Textbook of Microbiol- purplish material is noticed around the bacilli. This
ogy – 10th ed – Page 16 represents the capsular material and is characteristic of
anthrax bacillus.
• B.anthracis is a Gram-positive bacterium. So its cell wall

• This is called M’Fadyean’s reaction.


is made up of peptidoglycan and teichoic acid.


9. Ans.  (b) Anthrax
4. Ans.  (c) Polypeptide
Ref: Ananthanarayan and Paniker’s Textbook of Microbiol-
Ref: Ananthanarayan and Paniker’s Textbook of Microbiol- ogy – 10th ed – Page 252
ogy – 10th ed – Page 249
• PLET medium is a selective medium for B.anthracis, the
• The capsule of Bacillus anthracis is polypeptide in

causative agent of anthrax.


nature composed of a polymer of d-glutamic acid. It is • PLET medium consists of Polymyxin, Lysozyme,

plamid bone. Ethylene diamine tetraacetic acid (EDTA) and thallous


• Capsule aids virulence by inhibiting phagocytosis.

acetate added to heart infusion agar.


5. Ans.  (d) Anthrax 10. Ans.  (a) Anthrax ulcer
Ref: Ananthanarayan and Paniker’s Textbook of Microbiol- Ref: Ananthanarayan and Paniker’s Textbook of Microbiol-
ogy – 10th ed – Page 251 ogy – 10th ed – Page 250
• Bacillus anthracis produces irregularly round colonies

• In cutaneous anthrax, the lesion starts as a papule,


that are 2-3 mm in diameter, raised, dull, opaque, becomes vesicular, the whole area becomes congested,
white with frosted glass appearance. When viewed edematous, several satellite lesions filled with serum
under low power in a microscope, the edge of the or yellow fluid are arranged around a central necrotic
colony is composed of long, interlacing chains of bacilli lesion which is covered by a black eschar. The lesion is
resembling locks of matted hair. This is called the called a malignant pustule.
medusa head appearance.
11. Ans.  (d) Inhibits proteins synthesis
6. Ans.  (b) Bacillus anthracis
Ref: Ananthanarayan and Paniker’s Textbook of Microbiol-
Ref: Ananthanarayan and Paniker’s Textbook of Microbiol- ogy – 10th ed – Page 249
ogy – 10th ed – Page 251
• Anthrax toxin is a complex of three fractions. They are
• Bacillus anthracis shows inverted fir tree appearance in

non toxic individually but the whole complex produces


gelatin stab culture, with slow liquefaction commencing local edema and generalized shock.
from the top giving the characteristic appearance. • These three factors are:

(i)    Protective antigen factor (PA or factor II)


(ii)  Edema factor (OF or factor I)
(iii)      Lethal factor (LF or Factor III)
Mechanism of action is through increased cAMP 107
12. Ans.   (a) Zoonotic disease; (c) Agent for bioterrorism;
Unit 2     Bacteriology

Anthrax bacilli Anthracoid bacilli


(e) McFadyean’s reaction seen
Susceptible to gamma phage Not susceptible
Ref: Ananthanarayan and Paniker’s Textbook of Microbiol-
Pathogenic to laboratory Not pathogenic
ogy – 10th ed – Page 249
animals
• Anthrax is a zoonotic infection. Human anthrax is con-

tracted from animals, directly or indirectly. 15. Ans.  (b) Presence of pain in abdomen
• Human to human transmission does not occur.

Ref: Ananthanarayan and Paniker’s Textbook of Microbiol-


13. Ans.  (a) Polychrome methylene blue ogy – 10th ed – Page 253
• Both the types (Emetic and diarrheal) are characterized
Ref: Ananthanarayan and Paniker’s Textbook of Microbiol-

by abdominal pain.
ogy – 10th ed – Page 251
• Emetic type presents with vomiting and diarrheal type
• Polychrome methylene blue is helpful to stain the

presents with diarrhea.


specimens to detect anthrax bacilli; the clinical picture
gives presumptive diagnosis of anthrax 16. Ans.  (b) Preformed toxin

14. Ans.  (c) Nonmotile Ref: Ananthanarayan and Paniker’s Textbook of Microbiol-
ogy – 10th ed – Page 253
Ref: Ananthanarayan and Paniker’s Textbook of Microbiol-
• Two types of food poisoning have been associated with
ogy – 10th ed – Page 253

B.cereus – Diarrheic type and Emetic type.


• Bacillus anthracis is nonmotile unlike most other

members of the genus which are motile. Diarrheal type Emetic type
Associated with cooked meat Usually cooked fried rice from
Table:  Differences between Bacillus Anthracis and
and vegetables Chinese restaurants
Anthracoid Bacilli
Diarrhea and abdominal pain Acute nausea and vomiting are
Anthrax bacilli Anthracoid bacilli predominant symptoms. present. Diarrhea is rare.
Non motile Motile Vomiting is rare
Capsulated Noncapsulated Symptoms appear 8–16 Symptoms appear 1–5
Grow in long chains Grow in short chains hours after ingestion of hours after ingestion of
contaminated foods. contaminated foods.
Medusa head colony Not present
No growth in penicillin agar Grow usually B.cereus is not found in large Bacillus cereus is present in
numbers in fecal specimens large numbers in food and
Hemolysis absent or weak Usually well – marked
from these patients faeces of these patients
Inverted fir tree growth and Rapid liquefaction
slow gelatin liquefaction Caused mostly by serotypes 2, Caused mostly by serotypes
6, 8, 9, 10 or 12 1, 3 or 5
No turbidity in broth Turbidity usually
Salicin fermentation negative Usually positive Toxin is produced in the Pre-formed toxin (toxin
intestine formed in food)
No growth at 45°C Usually grows
Heat labile (inactivated by Heat stable
Growth inhibited by chloral Not inhibited
heating to 56°C for 5 minute)
hydrate

108
12
Clostridium
CLOSTRIDIUM SPECIES y Based on the type of toxin production – Cl. perfringens is

y
divided into A to E
• Gram-positive rods y Alpha toxin:

y
 Produced by most of the strains especially Type A Cl.per-

• Anaerobic


fringens

• Spore forming bacilli
 It is lecithinase which splits lecithin and causes gas

• All are motile with peritrichous flagella except – Cl. perfringens


gangrene

and Cl. tetani type VI (are nonmotile)
 Toxin needs Ca++ and Mg++ ions which splits lecithin into
• Confirmatory tests are Gram staining, culture, Nagler reaction,


phosphorylcholine and diglyceride

toxin demonstration  Toxin demonstration is done by a method called Nagler’s


reaction
Table 1:  Position of spores in Clostridium species y Enterotoxin:
y
 Enterotoxin produced by Cl. perfringens is very powerful
Position of spores Examples 
 It needs 108 vegetative bacilli to get entered inside the gut

Central Cl. bifermentans to produce enterotoxin.
Subterminal Cl. perfringens Clinical Features
Cl. sordellii
Cl. botulinum y Spores of Clostridia enters the traumatized areas and
y
Cl. sporogenes germinate – produce gas – tissues get distended and interferes
Cl. difficile with blood supply along with necrotizing toxin release
Cl. bifermentans leads to toxic tissue necrosis – gas gangrene – also called as
Clostridial myonecrosis:
Oval and terminal Cl. tertium y Food poisoning caused by Cl. perfringens occurs due to meat
y
Spherical and terminal Cl. tetani consumption. It presents after an incubation period of 8–24
hours – presents as abdominal pain, diarrhea and vomiting;
Table 2:  Species of clostridium based on biochemical usually it is a self-limiting illness.
reaction in Robertson’s cooked meat broth y Gangrenous appendicitis – usually caused by type A strains
y
and rarely by type D strains
Predominantly Cl. sporogenes, Cl. botulinum A, B, E, y Necrotizing enteritis – caused by type C strains; Pig Bel – it is
y
proteolytic Cl. bifermentans, Cl. histolyticum caused due to feasting on pig meat along with sweet potatoes
Predominantly Cl. perfringens, Cl. novyi, Cl. septicum, y Acute emphysematous cholecystitis
y
saccharolytic Cl. difficile y Post cholecystectomy septicemia
y
y Brain abscess, meningitis
Only proteolytic Cl. tetani
y
y Panophthalmitis
y
Only saccharolytic Cl. botulinum C, D, E y Thoracic infections
y
Neither proteolytic Cl. cochlearum y Urogenital infections
y
nor saccharolytic
High Yield
CLOSTRIDIUM PERFRINGENS Gas Gangrene
• Also called as anaerobic myositis, Clostridial myositis or

y Most common species that cause invasive infections Clostridial myonecrosis (Crepitant cellulitis)
y
y It is normally seen as an commensal in human large intestine • Causative organisms:
y

ƒ Cl. perfringens
ƒ
Pathogenesis ƒ Cl. novyi
ƒ
ƒ Cl. septicum
y It is mainly mediated by toxin
ƒ
ƒ Cl. histolyticum
y
 Major toxins – alpha and beta
ƒ
ƒ Cl. sporogenes

 Minor toxins are many namely gamma, delta, theta, kappa,
ƒ

lambda contd...
Unit 2     Bacteriology

ƒ ƒ Cl. fallax
ƒ ƒ Cl. bifermentans
ƒ ƒ Cl. sordellii
ƒ ƒ Cl. aerofoetidum
ƒ ƒ Cl. tertium
ƒ ƒ Anaerobic streptococci
ƒ ƒ Facultative anaerobes like E.coli, Proteus and Staphylococci

Laboratory Diagnosis
y Edge of the muscle is taken as specimen and Gram staining
y

done with the muscle film

Table 3: Features of direct gram staining in gangrene


Direct Gram staining feature in gas Probable Figure 2: Nagler’s reaction (alpha toxin)
gangrene diagnostic agent (Courtesy: CDC/Dr Stuart E Starr)
Presence of large numbers of regularly Cl. perfringens y Reverse CAMP Test
brick shaped, Gram-positive bacilli without
y

 Cl. perfringens is streaked in the center of the plate


spores


 Strep. agalactiae (Group B) is streaked at right angles to it




Boat or leaf shaped pleomorphic bacilli – Cl. septicum  When there is zone of enhanced hemolysis, an arrow


Citron bodies shaped zone occurs – positive test


Large bacilli with oval, subterminal spores Cl. novyi
Treatment
Slender bacilli with round, terminal spores Cl. tetani or
Table 4: Treatment of Cl. perfringens infection
Cl. tetanomorphum

Type of Cl. perfringens Treatment


infection
Clostridial sepsis Penicillin plus clindamycin
(or) alternatively any one of
following:
• Clindamycin

• Metronidazole

• Vancomycin

Gas gangrene Penicillin G plus clindamycin


(or)
Cefoxitin plus clindamycin

Figure 1: Gram-positive bacilli (Clostridium perfringens) CLOSTRIDIUM TETANI


(Courtesy: CDC) y Gram-positive slender bacillus with has spores in spherical
y

y Aerobic and Anaerobic culture:


y
shape with terminal bulging position – that gives characteristic
 Culture in blood agar shows target hemolysis (means

‘drum stick appearance’
there is a narrow zone of complete hemolysis due to theta
toxin and a wide zone of incomplete hemolysis due to
alpha toxin)
 Gram staining from the culture should show spores –


based on the position of spores – diagnosis can be made

Remember
Nagler’s Reaction
• Agar used is Filde’s peptic digest sheep blood agar with 20%

human serum
• One half of the culture plate is added with perfringens antitoxin

• When Cl. perfringens is grown in agar plate – lecithinase


toxin is secreted out – this toxin neutralize the antitoxin


seen in one half of the plate
110 • On the other side without antitoxin – this lecithinase toxin

Figure 3: Characteristic drum stick appearance of clostridium


produces opacity around the colonies (alpha toxin)
tetani (Courtesy: CDC)
y Cl. tetani has ten (10) serological types; among them type VI is

Chapter 12     Clostridium


y

Categories Wound less than 6 Other


nonflagellated – rest all are flagellated
hours, old, clean, wounds
nonpenetrating, with
Pathogenesis negligible tissue damage
y Pathogenesis is mediated by toxin; it produces two type of
y

D – Has not had a Toxoid complete course Toxoid


toxins complete course complete
 Tetanolysin (Hemolysin)


of toxoid or course +
 Tetanospasmin (Neurotoxin)


unknown status of human


y The toxin that is responsible for tetanus is tetanospasmin
y

vaccination tetanus Ig
y Mechanism of action of tetanospasmin is it blocks synaptic
y

inhibition in the spinal cord (presynaptic block of glycine y Antibiotic prophylaxis – Metronidazole and penicllin
y

and GABA) y Passive immunization should be started as early as possible


y

y Spinal inhibition is lost – that leads to uncontrolled impulse


y after exposure – Human antitetanus immunoglobulin (TIG) –
generations and spread in CNS 10000 IU diluted given as slow IV
y Muscle go for rigidity and spasms leading to tonic muscular
y

spasms. Vaccination
y Prophylaxis is given as DPT vaccine in 6,10,14 weeks followed
Clinical Features
y

by additional dose at 14-16 months and booster dose at


y Tetanus – defined as “acute onset of hypertonia or painful
y 10 years and after that every 10 years
muscular contractions and generalized muscle spasms
without other apparent medical cause” (CDC) CLOSTRIDIUM BOTULINUM
y Neonatal tetanus is defined by WHO as “an illness that
y

occurring in a child who has the normal ability to suck and y Gram-positive, producing subterminal, oval and bulging
y

cry in the first 2 days of life but loses this ability between 3 and spores
28 days of life and becoming rigid and has spasms” y Eight types of botulinum strains are classified based upon
y

y Incubation period = 6 to 12 days


y
their toxins
y It presents as lock jaw (Trismus), muscle pain, otitis media,
y
y Types: A, B, C1, C2, D, E, F, G
y

difficulty in swallowing, laryngeal obstruction and air way y All are neurotoxins except C2 (which is an enterotoxin)
y

block. y Unique feature of Cl. botulinum: Powerful exotoxin that is


y

produced by the organism will not be released during the


Laboratory Diagnosis life of organism; it is seen intracellularly; only after death of
bacilli – the toxin is released by autolysis
y Microscopic appearance – characteristic drum stick appear-
y

y The most potent toxin ever in world is botulinum toxin – the


ance
y

lethal dose for humans is 1-2 μg


y Culture in blood agar – showing swarming
y

y Mechanism of action of toxin: proteolysis of SNARE proteins


y

y Toxigenicity testing in animals


y

– blocks the production or release of acetyl choline (Ach) at


the NM junction and synapses
Treatment y Person to person transmission does not occur
y

y Toxin does not cross BBB


Table 5:  Base of tetanus toxoid vaccination in case of y

y Types C and D – the toxin production is phage mediated


injury
y

Categories Wound less than 6 Other Clinical Features


hours, old, clean, wounds
y Food borne botulism:
nonpenetrating, with
y

 Human illness is due to types A, B, E and F


negligible tissue damage


 Source of infection is home preserved foods like ham,




A – Complete Nothing needed Nothing sausage, canned foods, meat and meat products
course of toxoid or  Symptoms occurs after a IP of 12-36 hours


booster in previous  Vomiting and diarrhea are not seen




5 years  Case fatality is 25-70%




B – Complete Toxoid 1 dose Toxoid 1  Death occurs due to respiratory arrest




course of toxoid or dose y Wound botulism


y

booster in previous y Infant botulism: occurs due to contaminated honey –


y

5–10 years presents as constipation, lethargy, weakness, loss of head


control (floppy baby syndrome)
C – Complete Toxoid 1 dose Toxoid 1
y Adult intestinal colonization botulism
course of toxoid or dose and
y

y Iatrogenic botulism – illegal injection of toxins for cosmetic


booster dose in human
y

purposes (e.g.: eyelid treatment, ptosis) 111


> 10 years of age tetanus IG
y Intentional botulism – bioterrorism agent
(Contd...)
y
Laboratory Diagnosis y When patients are on prolonged antimicrobial usage – due to
Unit 2     Bacteriology

alteration in the normal flora – Cl. difficle causes colitis – it is


y Demonstration of bacilli or toxin in food or feces
named as pseudomembranous colitis
y

y Animal inoculation done by macerating the food material


y Antibiotics associated diarrhea is mainly because of usage
y

and inoculating the filtrate intraperitoneally; animals used or


y

of clindamycin, ampicillin or fluoroquinolones


mice or guinea pig
y Universally accepted assay for botulism is mouse bioassay.
Laboratory Diagnosis
y

y By colonic visualization – microabscesses can be seen


Treatment y
y

y Culture in selective media


y Botulism antitoxin heptavalent is given for all adult forms of
y

y y Detection of toxins by ELISA from stool samples


botulism y y Cell cultures – using Hep-2 and human diploid cell cultures
y For infant botulism – a separate antitoxin that has A and B is
y

y y Molecular methods (which detects genes for toxin A or B)


also available
y During outbreak a prophylactic dose of antitoxin should be
y Treatment
given IM for all the exposed persons. y It is treated by vancomycin or metronidazole
y

CLOSTRIDIUM DIFFICILE MISCELLANEOUS - CLOSTRIDIUM


y Long, slender Gram-positive bacilli
y y Cl. septicum: Secretes four distinct toxins; it is associated
y

y Spores are large, oval and sub-terminal


y with gas gangrene; Citron bodies with boat or leaf shaped
y It causes infection mostly in those whose normal flora is lost –
y pleomorphic bacilli is suggestive of Cl.septicum
it is an opportunistic organism y Cl. novyi: Four types are present namely A, B, C and D; only
y

y It secretes two exotoxins namely A and B


y type A is associated with gas gangrene
y Toxin A – potent enterotoxin which attaches to gut receptors
y y Cl. histolyticum is associated with gas gangrene, cellulitis,
y

y Toxin B – Cytotoxin
y abscess formation and endocarditis in IV drug users
y Toxin genes are located in the pathogenicity island in
y y Cl. sorderlli can cause endophthalmitis, can cause infection
y

chromosomes of endometrium after childbirth or abortion and it can lead to


toxic shock syndrome.

112
MULTIPLE CHOICE QUESTIONS

Chapter 12     Clostridium


1. Which of the following Clostridia is non-invasive: 12. Opacity around colonies of Clostridium perfringens is
 (Recent Pattern July 2016) due to: (Recent Pattern Nov 2014)
a. Clostridium novyi a. Theta toxin
b. Clostridium botulinum b. Lecithinase
c. Clostridium perfringens c. Desmolase
d. Clostridium tetani d. Cytokinin
2. Drumstick appearance is seen in: 13. True about gas gangrene are all except:
 (Recent Pattern Dec 2014)  (Recent Pattern Dec 2016)
a. Cl. tetani a. Most common cause is Cl. perfringens
b. Cl. tetanomorphum b. Extensive necrosis of muscles
c. Cl. sphenoids c. Cl. perfringens produce heat-labile spores
d. All d. Metronidazole is the drug of choice
3. Tennis racket shape is seen in:  14. Gas gangrene is caused by all except: (AIIMS May 2009)
 (Recent Pattern Dec 2016) a. Cl. histolyticum
a. Cl. perfringens b. Cl. botulinum b. Cl. novyi
c. Cl. tertium d. Cl. bifermentans c. Cl. septicum
4. Saccharolytic species of Clostridia: d. Cl. sporogenes
 (Recent Pattern Dec 2013) 15. Regarding Clostridium perfringens gas gangrene, false
a. Cl. tetani b. Cl. cochlearum is: (AIIMS Nov 2010)
c. Cl. septicum d. None a. Common cause of gas gangrene
5. Which one of the following species of Clostridium does b. Nagler reaction positive
not break carbohydrate and protein: c. Most common toxin is hyaluronidase
 (Recent Pattern June 2014) d. Food poisoning strain of Cl. perfringens produces heat
a. Sporogenes b. Septicum
resistant spores
c. Cochlearum d. Novyi
16. “Citron bodies” boat or leaf shaped pleomorphic organ-
6. Double zone of hemolysis is seen in:
ism in an exudate is: (Recent Pattern Dec 2013)
(Recent Pattern Dec 2015)
a. Cl. welchii


a. Staphylococcus aureus
b. Cl. sporogenes
b. Streptococcus pyogenes
c. Cl. septicum
c. Clostridium perfringens
d. Cl. tetani
d. Corynebacterium diphtheriae
17. Nagler’s reaction is due to which toxin of Cl. perfringens:
7. Most common organism responsible for gas gangrene
a. Alpha toxin  (Recent Pattern July 2016)
is: (Recent Pattern Dec 2014)
b. Epsilon toxin
a. Clostridium perfringens
b. Clostridium difficile c. Kappa toxin
c. Clostridium tetani d. Delta toxin
d. Clostridium septicum 18. Nagler’s reaction is given by: (Recent Pattern Dec 2016)
8. All Clostridia cause myonecrosis except: a. Clostridium bifermentans
(Recent Pattern July 2015) b. Cl. perfringens
c.


a. C.septicum b. C.difficile Cl. sordelli


c. C.novyi d. C.welchii d. All of the above
9. Gas gangrene is due to: (PGI Nov 2014) 19. Nagler’s reaction is shown by:(Recent Pattern Aug 2013)
a. Alpha toxin b. Theta toxin a. Clostridium tetani
c. Beta toxin d. Delta toxin b. Clostridium welchii
e. Epsilon toxin c. Mycobacterium tuberculosis
10. True about Cl. perfringens are all except: d. Mycobacterium leprae
 (Recent Pattern July 2016) 20. Clostridium tetani is: (Recent Pattern Dec 2016)
a. Invasive as well as toxigenic a. Gram-positive bacilli
b. Alpha toxin is detected by Nagler’s reaction b. Gram-negative bacilli
c. Beta toxin is most important in gas gangrene c. Gram-positive cocci
d. Theta toxin is perfringolysin d. Gram-negative cocci
11. Each of the following statements concerning Clostridium 21. Swarming growth on culture is characteristic of which
perfringens is correct except:(Recent Pattern Dec 2014) Gram-positive organism: (Recent Pattern Dec 2013)
a. It causes gas gangrene a. Clostridium welchii
b. It causes food poisoning b. Clostridium tetani
c. It produces exotoxin c. Bacillus cereus 113
d. It is a Gram-negative rod that does not ferment lactose d. Proteus mirabilis
22. All are true regarding tetanus except: 32. Botulism is most commonly due to:
Unit 2     Bacteriology

 (AIIMS November 2010)  (Recent Pattern Dec 2012)


a. Transmission through contaminated wounds and a. Egg
injuries b. Milk
b. More common in winters and dry weather c. Meat
c. Reservoir in soil and intestines of humans and animals d. Pulses
d. No herd immunity or life-long immunity 33. Which of the following is false about botulism?
23. Most common symptom of tetanus is:  (Recent Pattern July 2015)
 (Recent Pattern July 2016) a. It is a type of food poisoning
a. Tonic-clonic seizures b. Hemiplegia b. Botulinum toxin is a potent neurotoxin
c. Lock jaw d. Opisthotonus c. It is an infection and not an intoxication
24. Mechanism of action of tetanospasmin: d. The causative organism is Clostridium botulinum
 (Recent Pattern Dec 2012) 34. Botulism causes: (Recent Pattern Dec 2012)
a. Inhibition of GABA release a. Descending flaccid paralysis
b. Inhibition of cAMP b. Descending spastic paralysis
c. Inactivation of Ach receptors c. Ascending paralysis
d. Inhibition of cGMP d. Ascending spastic paralysis
25. Tetanus affects primarily: (Recent Pattern Dec 2012) 35. Most common organism responsible for pseudomem-
a. Presynaptic terminal of spinal cord branous colitis: (Recent Pattern Dec 2013)
b. Postsynaptic terminal of spinal cord a. Clostridium difficile
c. Neuromuscular junction b. Clostridium botulinum
d. Muscle fibers c. Clostridium histolyticum
26. Toxigenicity of tetanus is done on: d. Clostridium butyricum
36. Cause of Clostridium difficile associated diarrhea:
(Recent Pattern Aug 2013)
 (Recent Pattern Dec 2012)


a. Rabbit b. Horse
a. Trauma
c. Mouse d. Guinea pig
b. Dairy products
27. All of the following statements regarding Clostridium
c. Fried rice
tetani are true, except: (All India 2011)
d. Antibiotic use
a. Spores are resistant to heat
37. A patient of acute lymphocytic leukemia with fever and
b. Primary immunization consists of three doses
neutropenia develops diarrhea after administration of
c. Incubation period is 6-10 days
amoxicillin therapy, which of the following organism is
d. Person to person transmission does not occur
most likely to be the causative agent:
28. Botulism is a disease of: (Recent Pattern Dec 2013)
(AIIMS Nov 2008)
a. Neural transmission caused by the toxin of the


a. Salmonella typhi
bacterium Clostridium botulinum b. Clostridium difficile
b. Muscular transmission caused by the toxin of the c. Clostridium perfringens
bacterium Clostridium botulinum d. Shigella flexneri
c. Neuromuscular transmission caused by the toxin of the 38. Patient presenting with abdominal pain, diarrhea
bacterium Clostridium botulinum is taking clindamycin for 5 days. Treated with
d. Non neuromuscular transmission caused by the toxin metronidazole, the symptoms subsided. What is the
of the bacterium Clostridium botulinum causative agent- (Recent Pattern Dec 2013)
29. Botulinum toxin acts on: (Recent Pattern Dec 2012) a. Clostridium difficile
a. Sympathetic system

b. Clostridium perfringens
b. Parasympathetic system c. Clostridium welchii

c. Amygdala d. Clostridium marneffi


d. Motor cortex 39. Best method of diagnosis for Clostridium difficile
30. Among the toxin produced by Clostridium botulinum,  (AIIMS Nov 2017)
the non-neurotoxic one is: (Recent Pattern Dec 2014) a. Pure strain isolation from culture
a. A b. B b. Immunofluorescence
c. C1 d. C2 c. Toxin detection by ELISA
31. Mechanism of action of botulinum toxin: d. Toxin gene detection by PCR
 (Recent Pattern Dec 2015) 40. Trismus and Ophisthotonus are caused by  (CET 2018)
a. Increased cAMP a. Clostridium botulinum
b. Increased cGMP b. Clostridium tetani
c. Inhibition of acetylcholine release c. Clostridium perfringens
d. Inhibition of noradrenaline release d. Clostridium sporogenes
114
ANSWERS AND EXPLANATIONS

Chapter 12     Clostridium


1. Ans.  (b) Closridium botulinum 3. Ans.  (c) Cl. tertium
Ref: Ananthanarayan and Paniker’s Textbook of Microbiol- Ref: Ananthanarayan and Paniker’s Textbook of Microbiol-
ogy – 10th ed – Page 258 ogy – 10th ed – Page 257
• C. botulinum bacilli are virtually noninvasive and
• • Oval and terminal spore resembling tennis racket is

noninfectious. Botulism is caused by ingestion of characteristic of C.tertium.


preformed toxin in food. Spore position Appearance Example
• C. tetani has little inverse property. C. perfringenes is
Central or Spindle shape C. bifermentans

also invasive besides toxin production.


equatorial
2. Ans.  (d) All Subterminal Club-shaped C. perfringenes
Ref: Ananthanarayan and Paniker’s Textbook of Microbiol- Oval and terminal Tennis-racket C. tertium
ogy – 10th ed – Page 266 appearance
• C. tetani, C. tetanomorphum and C. sphenoides have

Spherical and Drumstick C. tetani
drum stick appearance (spherical, terminal and bulging terminal appearance
spores) and are indistinguishable.

4. Ans.  (c) Cl. septicum


Ref: Ananthanarayan and Paniker’s Textbook of Microbiology – 10th ed – Page 258
• In Robertson’s cooked meat medium, saccharolytic Clostridia turn the meat pink while proteolytic turns meat black producing

foul and pervasive odors.


• It is used to classify Clostridia.

Both proteolytic and saccharolytic Slightly proteolytic Saccharolytic but not Neither proteolytic
Proteolytic Saccharolytic but not saccharolytic proteolytic nor saccharolytic
predominating predominating
C. bifermentans C. perfringenes C. tetani C. fallax C. cochlearium
C. botulinum A,B,F C. septicum C. botulinum C, D, E
C. histolyticum C. chauvoei C. tertium
C. sordelli C. novyi C. tetanomorphum
C. sporogenes C. difficile C. sphenoides

5. Ans.  (c) cochlearum C. seticum next (20–40%) and C. histolyticum less


often.
Ref: Ananthanarayan and Paniker’s Textbook of Microbiol-
• Other Clostridia usually found are C. sporogenes,
ogy – 10th ed – Page 258

C. fallax, C. bifermentans, C. sordellii, C. aerofoetidum


• Refer answer 4.

and C. tertium. These may not be pathogenic by


themselves.
6. Ans.  (c) Clostridium perfringens
Ref: Ananthanarayan and Paniker’s Textbook of Microbiol- 8. Ans.  (b) C. difficile
ogy – 10th ed – Page 263 Ref: Ananthanarayan and Paniker’s Textbook of Microbiol-
• On fresh and heated blood agar, target hemolysis or

ogy – 10th ed – Page 261
double zone hemolysis is produced by C. perfringens. • C. difficile does not cause myonecrosis.

• A narrow zone of complete hemolysis due to theta toxin



• Refer to answer no.7

and a much wider zone of incomplete hemolysis due to


alpha toxin are seen. 9. Ans.  (a) Alpha toxin

7. Ans.  (a) Clostridium perfringens Ref: Ananthanarayan and Paniker’s Textbook of Microbiol-
ogy – 10th ed – Page 259
Ref: Ananthanarayan and Paniker’s Textbook of Microbiol-
• The alpha toxin produced by C. perfringenes is responsi-
ogy – 10th ed – Page 261

ble for the profound toxemia of gas gangrene. It is lethal,


• C. perfringens is the most important of the Clostridia

dermonecrotic and hemolytic.
causing gas gangrene (about 60%), with C. novyi and • It is a phospholipidase (lecithinase C).
115

10. Ans.  (c) Beta toxin is most important in gas gangrene 16. Ans.  (c) Cl. septicum
Unit 2     Bacteriology

Ref: Ananthanarayan and Paniker’s Textbook of Microbiol- Ref: Ananthanarayan and Paniker’s Textbook of Microbiol-
ogy – 10th ed – Page 259 ogy – 10th ed – Page 262
• Alpha toxin is the most important toxin causing gas
• • Citron bodies are boat or leaf – shaped pleomorphic,

gangrene. irregularly stained bacilli in direct microscopic


• Beta toxin is associated with lamb dysentery, enteritis in
• examination of gram stained films – suggestive of
animals and enteritis necroticans in human beings. C. septicum.

11. Ans.  (d) It is a Gram-negative rod that does not ferment 17. Ans.  (a) Alpha toxin
lactose
Ref: Ananthanarayan and Paniker’s Textbook of Microbiol-
Ref: Ananthanarayan and Paniker’s Textbook of Microbiol- ogy – 10th ed – Page 259
ogy – 10th ed – Page 263 and 264 • Nagler’s reaction is seen as opalescence around bacterial

• Clostridia are gram-positive rods.


• colonies in serum or egg yolk media.
• C. perfringenes ferment glucose, maltose, lactose and
• • It is due to the alpha toxin (lecithinase) produced by

sucrose with production of acid and gas. C. perfringenes.

12. Ans.  (b) Lecithinase 18. Ans.  (d) All of the above
Ref: Ananthanarayan and Paniker’s Textbook of Microbiol- Ref: Ananthanarayan and Paniker’s Textbook of Microbiol-
ogy – 10th ed – Page 259 ogy – 10th ed – Page 263
• Alpha toxin is a lecithinase which splits lecithin into
• • Other Clostridia producing alpha toxin (C. novyi,

phosphoryl choline and diglyceride in the presence of C. sordelli, C. bifermentans, C. baratti) also give positive
calcium and magnesium ions. Nagler reaction.
• This reaction is seen as opalescence around bacterial
• • In case of Clostridia other than C. perfringenes, the toxin

colonies in serum or egg yolk media and is specifically is not neutralized by the antitoxin of C. perfringenes
neutralized by the antitoxin. except C. bifermentans which produces a serologically
• This specific lecithinase effect is known as Nagler’s
• related lecithinase.
reaction.
19. Ans.  (b) Clostridium welchii
13. Ans.  (c) Cl.perfringens produce heat-labile spores
Ref: Ananthanarayan and Paniker’s Textbook of Microbiol-
Ref: Ananthanarayan and Paniker’s Textbook of Microbiol- ogy – 10th ed – Page 263
ogy – 10th ed – Page 258 • Nagler reaction is produced by C. perfringenes (other

• Spores of food poisoning strains of type A and certain


• names C. welchii, Bacillus aerogens capsulatus,
strains of type C are heat resistant. Autoclaving at 121°C B. phlegmatis emphysematosae.)
for 15 minutes is needed to destroy them. • Other Clostridia producing alpha toxin (C. novyi,

• Other spores are usually deatroyed by boiling within 5


• C. sordelli, C. bifermentans, C. baratti) also give positive
minutes. Nagler’s reaction.

14. Ans.  (d) Cl. sporogenes 20. Ans.  (a) Gram-positive bacilli
Ref: Ananthanarayan and Paniker’s Textbook of Microbiol- Ref: Ananthanarayan and Paniker’s Textbook of Microbiol-
ogy – 10th ed – Page 261 ogy – 10th ed – Page 264
• C. perfringens is the most important of the Clostridia
• • C. tetani is a gram-positive, slender bacillus with a

causing gas gangrene (about 60%), with C. novyi and straight axis, parallel sides and rounded ends.
C. seticum next (20–40%) and C. histolyticum less often. • The spores are terminal, spherical and bulging giving

• C. sporogenes might be found on wound. But they are


• the bacillus a drumstick appearance.
not pathogenic.
21. Ans.  (b) Clostridium tetani
15. Ans.  (c) Most common toxin is hyaluronidase
Ref: Ananthanarayan and Paniker’s Textbook of Microbiol-
Ref: Ananthanarayan and Paniker’s Textbook of ogy – 10th ed – Page 267
Microbiology – 10th ed – Page 259 • C. tetani produces swarming growth. An extremely fine,

• The most abundantly produced and most important


• translucent film of growth is produced which is visible
toxin of C. perfringens is alpha toxin, which is a only at the advancing edge.
lecithinase

116
22. Ans.  (b) More common in winters and dry weather 28. Ans.  (c) Neuromuscular transmission caused by the

Chapter 12     Clostridium


toxin of the bacterium Clostridium botulinum
Ref: Ananthanarayan and Paniker’s Textbook of Microbiol-
ogy – 10th ed – Page 266 Ref: Ananthanarayan and Paniker’s Textbook of Microbiol-
• Tetanus is common in warm climate, in developing

ogy – 10th ed – Page 269
countries where the soil is made fertile with organic • The toxin of C. botulinum acts by blocking the

manure. production or release of acetyl choline at the synapses


and neuromuscular junctions.
23. Ans.  (c) Lock jaw
29. Ans.  (b) Parasympathetic system
Ref: Ananthanarayan and Paniker’s Textbook of Microbiol-
ogy – 10th ed – Page 266 Ref: Ananthanarayan and Paniker’s Textbook of Microbiol-
• The first symptom of the onset of tetanus is trismus or

ogy – 10th ed – Page 269
lock jaw. • Botulinum toxin acts on postganglionic parasympathetic

nerve endings.
24. Ans.  (a) Inhibition of GABA release • The manifestations of botulism are due to decreased

Ref: Ananthanarayan and Paniker’s Textbook of Microbiol- acetylcholine in cranial nerve and parasympathetic
ogy – 10th ed – Page 265 nerve terminals – diplopia, dysphasia, dysarthria,
descending symmetric flaccid paralysis ending in death
• Tetanospasmin specifically blocks synaptic inhibition in

by respiratory paralysis.
the spinal cord, presumably at inhibitory terminals that
use GABA and glycine as neurotransmitters. The toxin 30. Ans.  (d) C2
acts presynaptically.
• The abolition of spinal inhibition causes uncontrolled

Ref: Ananthanarayan and Paniker’s Textbook of Microbiol-
spread of impulses initiated anywhere in the CNS. This ogy – 10th ed – Page 269
results in muscle rigidity and spasms. • A, B, C1, C2, D, E, F and G are the eight types of

• Its action is similar to that od strychnine, but strychnine


• C. botulinum strains based on immunological
acts post-synaptically. differences in the toxins produced by them.
• All of these except C2 are neurotoxic. C2 toxin alone

25. Ans.  (a) Presynaptic terminal of spinal cord shows enterotoxic activity.
Ref: Ananthanarayan and Paniker’s Textbook of Microbiol-
31. Ans.  (c) Inhibition of acetylcholine release
ogy – 10th ed – Page 265
• Tetanus toxin acts presynaptically in the spinal cord.

Ref: Ananthanarayan and Paniker’s Textbook of Microbiol-
• Its mechanism of action is similar to strychnine. But the

ogy – 10th ed – Page 269
difference is that strychnine acts postsynaptically. • The toxin of C. botulinum acts by blocking the

production or release of acetyl choline at the synapses


26. Ans.  (c) Mouse and neuromuscular junctions.
Ref: Ananthanarayan and Paniker’s Textbook of Microbiol-
32. Ans.  (c) Meat
ogy – 10th ed – Page 267
• In vivo toxigenicity of tetanus toxin is done on mouse.

Ref: Ananthanarayan and Paniker’s Textbook of Microbiol-
A 2-4 day old- cooked meat culture (0.2 mL) is injected ogy – 10th ed – Page 269
into the tail of a mouse. • The source of botulism is usually preserved food – meat

• Symptoms develop in 12-24 hours, beginning with


• and meat products, canned vegetables, fish and other
stiffness of tail. Rigidity proceeds onto leg on inoculated sea foods.
side, opposite leg, trunk, forelimbs and so on. The
animal dies in 2 days. 33. Ans.  (c) It is an infection and not an intoxication
• A second mouse that has received 1000 U of tetanus

Ref: Ananthanarayan and Paniker’s Textbook of Microbiol-


antitoxin acts as the control. ogy – 10th ed – Page 269
27. Ans.  (a) Spores are resistant to heat • The manifestations of botulism are due to botulinum

toxin.
Ref: Ananthanarayan and Paniker’s Textbook of Microbiol- • C. botulinum is virtually noninfective.

ogy – 10th ed – Page 265


• The resistance of tetanus spores to heat appears to be

34. Ans.  (a) Descending flaccid paralysis
subject to strain differences. Most are killed by boiling Ref: Ananthanarayan and Paniker’s Textbook of Microbiol-
for 10-15 minutes but some resist boiling for up to 3 ogy – 10th ed – Page 269
hours.
• Autoclaving at 121°C for 20 minutes ensures destruction • The characteristic pattern of botulism is symmetric

117

of spores. descending flaccid paralysis.


• The manifestations of botulism are due to decreased • Broad spectrum antibiotics like clindamycin, ampicillin
Unit 2     Bacteriology

• •

acetylcholine in cranial nerve and parasympathetic or fluoroquinolones predispose to C. difficile infection.


nerve terminals – diplopia, dysphasia, dysarthria,
descending symmetric flaccid paralysis ending in death 38. Ans.  (a) Clostridium difficile
by respiratory paralysis. Ref: Ananthanarayan and Paniker’s Textbook of Microbiol-
ogy – 10th ed – Page 270 and 271
35. Ans.  (a) Clostridium difficile
• C. difficile is the most common cause of health care

Ref: Ananthanarayan and Paniker’s Textbook of Microbiol- associated diarrhea. It occurs following the use of broad
ogy – 10th ed – Page 270 spectrum antibiotics like clindamycin, ampicillin or
• C. difficile is the most common cause of pseudomem-

fluoroquinolones.
branous colitis. • C. difficile disease is treated by discontinuing the

• It causes health care associated diarrhea in many



antibiotics causing the disease and administering
developed countries following the use of broad vancomycin or metronidazole.
spectrum antibiotic like clindamycin, ampicillin or
39. Ans.  (c) Toxin detection by ELISA
fluoroquinolones.
Ref: Ananthanarayan and Paniker T.B of Microbiology –
36. Ans.  (d) Antibiotic use 10th ed – page 270
Ref: Ananthanarayan and Paniker’s Textbook of Microbiol- • Main stay of diagnosis: Detection of toxin A and/or B in

ogy – 10th ed – Page 270 stool by ELISA


• Other methods are: Demonstrating toxin by Hep-2 cell
• C. difficile causes health care associated diarrhea in

cultures, Toxin gene identification, culture on selective


many developed countries following the use of broad


media
spectrum antibiotic like clindamycin, ampicillin or
fluoroquinolones. 40. Ans.  (b) Clostridium tetani

37. Ans.  (b) Clostridium difficile Ref: Ananthanarayan and Paniker’s Textbook of Microbiol-
ogy – 10th ed – Page 265
Ref: Ananthanarayan and Paniker’s Textbook of Microbiol-
• Toxin of tetani causes presynaptic block of glycine and
ogy – 10th ed – Page 270

GABA leads to loss of spinal inhibition


• C. difficile causes antibiotic associated diarrhea. It is

• Clinically patient present as Trismus – lock jaw, muscle


an opportunistic organism that causes disease when pain, laryngeal obstruction and in severe form it leads to
normal host flora is lost. ophisthotonus position

118
13
Enterobacteriaceae
y A group of Gram-negative bacilli that are:
y
 Nonsporing

 Nonacid fast

 Ferment sugars

 Grows readily in ordinary media

 Many are motile by peritrichous flagella except few

 All are oxidase negative

y Classification of Enterobacteriaceae:
y
 Bergey’s classification

 Based on lactose fermentation

 Edward – Ewing classification

y Based on Edward and Ewing, 8 tribes (I – VIII) are present in
y
Enterobacteriaceae under which genus are described.

Table 1:  Enterobacteriaceae members according to


fermentation of lactose Figure 1: Gram-negative bacilli (E. coli)
(Courtesy: CDC/ Dr WA Clark)
Based on fermentation Examples
of lactose Virulence Factors
Lactose fermenters Escherichia, Klebsiella, Antigens
Enterobacter, Edwardsiella, Serratia
y Surface antigens:
Late lactose fermenters Citrobacter, Shigella sonnei
y
 Somatic antigen – O antigen

Non lactose fermenters Salmonella, Shigella, Proteus,  Flagellar antigen – H antigen

Providencia, Morganella, Hafnia,  Capsular antigen – K antigen

 Fimbrial antigen – F antigen

Table 2:  Tribes of Enterobacteriaceae y Colonization factor antigens (CFA) – helps in adherence of
y
organism
Tribes Genus y P fimbriae – seen in UPEC – Uropathogenic E. coil – which
y
Tribe I Escherichia, Shigella binds specifically to P blood group substance on RBCs
Tribe II Edwardsiella
‘O’ Antigen
Tribe III Salmonella y O Ag is the external part of the cell wall - LPS
y
Tribe IV Citrobacter y O antigens are resistant to heat and alcohol
y
Tribe V Klebsiella, Enterobacter, Serratia, Hafnia, Pantoea y Antibodies to O antigens are IgM type
y
y Usually O antigens are shared commonly between same
Tribe VI Proteus, Providencia, Morganella
y
tribes or family that leads to cross reactions
Tribe VII Yersinia y It is nonspecific
y
y Based on typing of ‘O’ groups – we can differentiate whether it
Tribe VIII Erwinia
y
is normal bowel flora or pathogenic
y Types 1, 2, 3, 4 etc. (early O groups) are normal flora
TRIBE I: ESCHERICHIA COLI
y
y Types 26, 55, 86, 111 (later O groups) are pathogenic
y
Escherichia coli is a part of normal bowel flora in humans; it also
causes numerous infections: ‘H’ Antigen
• Gram-negative bacilli y These antigens are located in flagella
y

• Motile by peritrichate flagella y It can be denatured/removed by heat or alcohol
y

• Lactose fermenting y Specific to an organism (no cross reactions)
y

• Ferments all sugars with production of acid and gas y Antibodies to H antigens are IgG type

y
• IMViC = ++--

‘K’ Antigen If delayed: Refrigeration should be done – refrigerated
Unit 2     Bacteriology

 

sample is accepted for a maximum of 4-6 hours


y Located in the envelope or microcapsule
 Other way to collect urine is from catheters and in infants
y

y K antigen encloses the O antigen




and babies by suprapubic aspiration


y

y Microscopic examination of urine – when Gram stained


Toxins
y

smear shows one bacilli/ HPF – then it is indicative of UTI


y Hemolysins
y
(But culture is confirmatory)
y CNF 1 – Siderophores
y
y Culture of urine: Media used are:
y

y Enterotoxins – Heat labile toxin and heat stable toxin


y
 MacConkey agar or CLED agar


 Blood agar


Table 3:  Classification of E. coli  CLED agar is preferred over MAC because:


Š It supports the growth of both Gram-positive cocci like


Enteropathogenic Watery diarrhea in infants and children;
Š

Staphylococci and Gram-negative bacilli


E. coli (EPEC) do not produce enterotoxin; not invasive Š It supports growth of Candida species
Š

Enterotoxigenic Causes traveler’s diarrhea; produces Š We can differentiate LF and NLF


Š

E. coli(ETEC) enterotoxins – heat labile toxin or heat y Quantitative count estimation of colonies (by Kass criteria)
y

stable toxin; adheres to intestinal mucosa is must to diagnose UTI:


by means – fimbrial or colonization factor  ≥105 colonies/mL = diagnosed as UTI


antigen; noninvasive  Exception: 102 colonies/mL is enough to diagnose UTI in




Enteroinvasive Resembles that of shigella; certain conditions like


E. coli (EIEC) enteroinvasive; penetrates HeLa or HEP-2 Š When the organism is Staphylococcus
Š

cells in tissue culture; this penetration Š Catheterized samples


Š

is plasmid mediated; VMA – virulence Š Hematogenous infections


Š

marker antigen Š Patients already on antimicrobials


Š

Enterohemorrhagic Named as verotoxigenic E. coli; produces


E. coli (EHEC) shiga like toxin; inhibits protein synthesis;
causes HUS; serotype O157:H7 – does
not ferment sorbitol like others – can be
detected by Sorbitol MacConkey media
Enteroaggregative Causes persistent diarrhea; stacked brick
E. coli (EAEC) adherence pattern on Hep-2 cells
Uropathogenic Most common cause of UTI in a
E. coli (UPEC) community; P fimbriae is the virulence
factor

Clinical Manifestations
y Urinary tract infections (UTI):
y

 Most common cause of UTI overall is E. coli



Figure 2: E. coli – lactose fermenting colonies grown in urine
 Infection usually presents as uncomplicated cystitis
 sample (Courtesy: Dr Prabha P, Kilpauk Medical College,
 It is mainly an ascending type of infection from the gut
 Chennai, Tamil Nadu)
flora that causes lower UTI y Other supportive tests done in urine sample to diagnose UTI:
y

 In pregnant women—5 – 7% are suffering from asymp-


 Catalase test


tomatic bacteriuria because of E. coil  Griess nitrite test




y It causes diarrhea – infantile diarrhea, adult and traveler’s


y

 Triphenyl tetrazolium chloride test




diarrhea  Dip slide culture




y Hemolytic uremic syndrome (HUS)


y

 Leukocyte esterase test




y Septicemia
y

y Pyogenic infections
y

y Meningitis in infants
y
Sample: Feces in Diarrhea
y Culture of stool specimen in MacConkey agar and serogroup-
y

ing
Laboratory Diagnosis
Table 4:  Toxin demonstration tests
Sample – Urine in UTI
y Clean voided, mid-stream urine sample need to be collected
y
In vivo tests In vitro tests
y For culture – transportation should be within ½ hour to a
y
Rabbit ligated ileal loop tests Tissue culture test
120 maximum of 2 hours
Infant rabbit bowel test Chinese hamster ovary cells

(Contd...)
Virulence Markers

Chapter 13     Enterobacteriaceae


In vivo tests In vitro tests
Infant mouse intra gastric ELISA y Invasiveness is the hallmark virulence for shigellosis
y

y Exotoxin secreted by Shigella dysenteriae type 1 – has less


Adult rabbit skin permeability test Passive agglutination test
y

role in pathogenesis; it is similar to EHEC verotoxin;


DNA probes y Endotoxin is LPS in cell wall – that causes irritation to bowel
y

y Cell culture to demonstrate toxins


y
wall.
y PCR to identify the genes
Clinical Features
y

y Infection is mainly transmitted through direct contact (feco


TRIBE I: SHIGELLA SPECIES y

oral route), through fomites, through flies and as a part of gay


• • Gram-negative bacilli bowel syndrome in homosexuals
• • Nonmotile, no capsule y Incubation period is 1-2 days
y

• • Nonlactose fermenting (Exception: Sh. sonnei) y Abdominal pain, fever and watery diarrhea
y

• • Specific media: DCA, XLD, Selenite F broth y Tenesmus seen


y

y Can also cause hemolytic-uremic syndrome


y

y Four important species: Sh. dysentriae, Sh.flexneri, Sh.boydii,


y
y S. dysenteriae type 1: Causes arthritis, toxic neuritis, conjunc-
y

Sh.sonnei tivitis, parotitis and intussusception


y Differentiation of Shigella species done by mannitol fermen-
y
y Self-limiting condition usually
y

tation or ODC decarboxylation


Laboratory Diagnosis
Table 5: Differentiation of Shigella species by Mannitol
y Stool samples collected and transported in buffered glycerol

fermentation and ODC decarboxylation


y

saline or Gram-negative broth.


Group Species Serotypes Mannitol ODC y Inoculate the stool samples in Selenite F broth for 6 hours and
y

fermentation subculture from the broth to culture plates.


y Specific culture media used are:
A S. dysenteriae 10 – –
y

 Deoxycholate citrate agar (DCA)




B S. flexneri 6 + –  Xylose lysine deoxycholate agar (XLD)




C S. boydii 15 + –  Hektoen enteric agar (HE)




 Salmonella shigella agar (SS)


D S. sonnei 1 + +


y All the culture isolates need to be confirmed by polyvalent


y

and monovalent sera.


Remember
Important Points to be Remembered
• All are catalase positive except Shigella dysenteriae type 1

• Shigella dysenteriae type 1 – is called as Shiga bacilli


• Shigella sonnei and Shigella flexneri type 6 are always


indole negative
• S. flexneri type 6 has three biotypes namely – Boyd 88,

Manchester and Newcastle


• S. flexneri is the most common species isolated in India.

Pathogenesis Figure 3: Colorless (nonlactose fermenting) colonies of Shigella


y Usually the infection is limited to GIT; invasiveness in blood
y

(Courtesy: CDC)
is rare.
y Infective dose for shigellosis is 10-100 (as it can survive acid
y
Treatment
environment). y Uncomplicated shigellosis is self-limiting
y

y Organism invades the mucosal epithelial cells called as M


y
y Those with systemic presentations and debilitated persons –
y

cells  and escapes by phagocytic vacuoles  multiplies treatment with antibiotics is must
and spread within cell cytoplasm and pass to adjacent cells y First line drug: Ciprofloxacin
y

(Basolateral spread) continues spreading and leads to y Second line drugs: Pivmecillinam, Ceftriaxone, Azithromycin
y

inflammation and cell death necrosis causes transverse


ulcers. (pseudomembrane) TRIBE II: EDWARDSIELLA TARDA
y Invasive character of Shigella is because of plasmid that
y

codes for protein called Virulence marker antigens (VMA). y Gram-negative bacilli
y

y Motile
y

y The name tarda means weak – because it has a weak fermen-


y

tation of carbohydrates 121


y It is a part of normal gastrointestinal flora y H antigen:
Unit 2     Bacteriology

y y

y Its strict pathogenesis is uncertain


y  Flagellar antigen; heat labile


y But it has been isolated from clinical specimens like blood,


y  More specific and helps in confirmation in serology


urine, wounds.  It has phase variation – phase I and II




 S.Typhi exist in only phase 1 and hence it is monophasic




y Vi antigen:
TRIBE III: SALMONELLA y

 Heat labile


 It is a surface polysaccharide antigen enveloping the O


• Gram-negative bacilli


antigen

• Motile with peritrichate flagella


 Vi antigen is also seen in:

• Exceptions: Salmonella Gallinarum and Pullorum are non




Š Salmonella Paratyphi C

motile
Š

Š Salmonella Dublin
• Special media: DCA, XLD, SS agar, Wilson Blair bismuth sulfate
Š

Š Citrobacter (Bethesda-Ballerup group)


medium
Š

 Poorly immunogenic
• H2S production occurs (black colored colonies)


 After infections, antibodies to Vi antigen is very low and


• Broths: Tetrathionate broth and Selenite F broth




hence not helpful in diagnosis


• IMViC = -+-+
 Antibody also disappears so fast in human body during

y Salmonellae can be classified into two groups:


y convalescence.
 Typhoidal Salmonellae:
Pathogenesis


Š Typhoid bacilli
Š

Š Paratyphoid bacilli – A, B, C
Š y Route of infections: Contaminated food or water
y

 Nontyphoidal Salmonellae: Salmonella gastroenteritis


 y Infectious dose ranges from 200 CFU to 106 CFU
y

y Characteristics of Salmonella bacilli:


y y When the salmonella bacilli enters the small intestine, it
y

 All ferment sugars and form acid and gas


 penetrates the mucosal layer and resides in the Peyer’s
 Exception is S. Typhi – anaerogenic – does not produce
 patches.
gas y After crossing the epithelial layer in SI, the bacilli is
y

 S. Typhi needs tryptophan as the growth factor


 phagocytosed by macrophages and live intracellularly
 Classification of Salmonellae is done based on O and
 y This phagocytosed bacilli travels throughout the body
y

H antigens and phase variation of H antigen named as via macrophages to lymphatic and reaches the reticulo-
Kauffmann White scheme endothelial system

Clinical Features
y Fever caused by S. Typhi  Typhoid fever
y

y Fever caused by S. Paratyphi  Paratyphoid fever


y

y Fever caused by both collectively called as Enteric fever


y

y Incubation period varies from 10–14 days


y

y Symptoms: Prolonged fever, head ache, myalgia, sweating,


y

anorexia, abdominal pain, nausea, diarrhea, coated tongue,


splenomegaly
y Physical findings: Rose spots, relative bradycardia
y

y Complications: GI bleeding, Intestinal perforation, GBS,


y

neuritis, muttering delirium, DIC, pancreatitis, arthritis, py-


elonephritis, osteomyelitis, HUS, endophthalmitis, pancre-
atitis, hepatic abscess
y Carrier state:
y

 Untreated patients – excrete in feces for up to 3 months




 1-4% go for chronic asymptomatic carriage sheds for >1




year

Table 6:  Carriers of Salmonella


Figure 4: Typical black colored salmonella colonies in XLD agar
(Courtesy: CDC) Convalescent carriers Shed bacilli 3 weeks – 3 months

Salmonella Typhi and Paratyphi Temporary carriers Shed bacilli 3 months – 1 year

Antigenic Structure Chronic carriers (Permanent More than 1 year


y O antigen:
y
carriers)
 Also called as Boivin antigen


122  It is not affected by boiling, alcohol or acids




 The antigen is not specific; it can cause cross reactions



Laboratory Diagnosis Table 8:  Interpretation of Widal Test

Chapter 13     Enterobacteriaceae


y Diagnosis of enteric fever: (BASU)
y

Organism O H AH BH
antigen antigen antige antigen
Table 7:  Diagnosis of enteric fever S. Typhi + + – –
Week of diagnosis Diagnostic method S. Paratyphi A + – + –
First week Blood culture S. Paratyphi B + – – +
Second week Widal test – Antibody detection – • O antigen positive means - Agglutination titer should be >

O – 1:200 1:200
H – 1:100 • H antigen positive means - Agglutination titer should be >1:100

Third week Stool culture Why there is variation in titer interpretation:


Fourth week Urine culture • Depends on the dilution method used it varies slightly

• Depends on the baseline titer of the endemic area - that is - In


y Definitive diagnosis is by isolation of S. Typhi or Paratyphi


y
endemic areas -most of the people have pre-existing antibodies
from blood, bone marrow or other sites, or from rose spots, - so when active infection is to interpreted - we need to set a
stool specimens range above this level - hence the titer of O above 1:200 and H
y Sensitivity of culture:
y
above 1:100 is positive; When the titre is less than this, do serial
 Blood culture is 40-80%

testing of serum after two weeks to check for increase in titre.
 Bone marrow culture is 55-90%

Why paratyphoid O antigens not used:
• Typhoid and paratyphoid O antigens - cross react due to

Widal Test sharing of factor 12


y It is an example for tube agglutination test (serological test)
y y Other Test:
y

y Not useful nowadays because of endemic titer difference


y  IgM Typhidot


y Two tubes are used namely Felix tube and Dreyer’s tube
y  PCR based tests


(i) Felix tube – O antigen


 WBC count showing leucopenia with relative lymphocy-


(ii) Dreyer’s tube – H antigen


tosis
y Antibodies present in the human body reacts with
y y Typing of Salmonella is done at National Salmonella Phage
y

commercially prepared O and H antigens Typing centre – located at Lady Hardinge Medical college,
y O antigen that is common to all (as it is not specific)
y New Delhi
y H antigen of S. Typhi, S. Paratyphi A and S. paratyphi B
y y S. Typhi Phage type A and E1 are common in India
y

y Serial dilutions were prepared in the ratio of 1:20, 1:40, 1:80


y

and so on
Diagnosis of Carriers
y When antibodies meet antigens at zone of equivalence
y

according to lattice hypothesis – agglutination occurs y y Vi antigen testing – presence shows carriers state
y H agglutination looks like loose, cotton wooly clumps
y
y y Isolation from feces or from bile, urine
y O agglutination looks like disc like pattern
y
y y Sewer swab technique
y The titer at which the agglutination occurred is the
y
y y Filtration through millipore membrane
interpretation
y Every endemic area has its own antibody levels in the human
y
Treatment
body – it should be identified by studies and only above that y Treatment: Ceftriaxone is the DOC even for MDR; alternate
y

level of antibodies are taken as pathogenic level of exposure – drugs are ciprofloxacin, azithromycin; For carriers – Ampicil-
it is called as baseline titer lin or amoxicillin
y Interpretative value: (usually)
y
y MDR Salmonella – multidrug resistance to the following
y

 O – 1:200

drugs (CAT)
 H – 1:100

 Chloramphenicol


 Ampicillin
High Yield


 Trimethoprim


y FQ’s resistance is also increasing in India


• How to make Widal a best test:
y

ƒ Do Widal test only during second week of fever (as anti-


Empirical treatment for Enteric Azithromycin or Ceftriaxone
ƒ

bodies start appear)


fever
ƒ Do follow up sample testing – if there is rise in titer in
ƒ

second sample then it is infectious Confirmed case of Typhoid Ciprofloxacin


ƒ Ask vaccination history – vaccinated people has antibodies
ƒ
fever
ƒ Compare with other blood investigations
MDR Salmonella Ciprofloxacin or Azithromycin
ƒ

ƒ H antigens are more specific and persist longer than O


ƒ

antigens Carrier of typhoid Ampicillin or Cipro for 6 weeks 123


Typhoid Vaccines y Three species are seen: C.freundii, C.koseri, C.amalonaticus
Unit 2     Bacteriology

y C. freundii strains are called as Bethesda–Ballerup group


y Parenteral TAB vaccine
y

y These are part of normal intestinal flora


y

y Parenteral Vi Polysaccharide vaccine


y

y It can cause UTI, biliary tract infections, surgical site


y

y Typhoral vaccine
y

infections, neonatal meningitis, brain abscess, bactermia in


y

neutropenic patients
Typhoral Vaccine
y It is an oral live attenuated vaccine from S. Typhi strain named
y

TRIBE V: KEHS GROUP: (KLEBSIELLA,


Ty21a
y It lacks the UDP-galactose-4 epimerase enzyme (Gal E
y
ENTEROBACTER, HAFNIA, SERRATIA)
mutant) Klebsiella Species
y When taken orally, it gets multiplied for some period of time
y

and then self-destructs by itself. • • Gram-negative bacilli


y Hence the body recognizes it as active infection (but because
y
• • Nonmotile
of mutation – pathogenesis will not be seen) • • Lactose fermenting
y Immunity develops after a week and may last up to 4-5 years
y
• • Capsulated
y It is given after 6 years of age
y
• • IMViC = --++
y Four doses: One capsule every other day for a week (day 1,
y Klebsiella pneumoniae is seen in respiratory tract and in stool
y

day 3, day 5, and day 7). The last dose should be given at least
y

samples (5% of normal individuals has commensals in feces)


1 week before travel to allow the vaccine time to work. (If
y It causes pneumonia and extensive hemorrhagic necrotizing
traveling to endemic areas)
y

consolidation of lung
y It also causes UTI especially in catheterised individuals,
Parenteral Vi Polysaccharide Vaccine
y

bacteremia, pyogenic liver abscess and hospital acquired


y One dose provides protection. It should be given at least 2
y
infections
weeks before travel to allow the vaccine time to work. y Klebsiella pneumoniae has subspecies namely:
y

y A booster dose is needed every 2 years for people who remain


y
 K.pneumoniae pneumoniae


at risk.  K.pneumoniae ozaenae




 K.pneumoniae rhinoscleromatis


Nontyphoidal Salmonellosis Klebsiella Friedlander’s bacilli – most common


y These are caused by:
y
pneumoniae species – causes pneumonia
 S. typhimurium


Klebsiella ozaenae Frisch bacillus – causes atrophic rhinitis –


 S. enteritidis


causes foul smelling discharge


 S. newport
Klebsiella oxytoca Causes nosocomial infections


 S. heiderlberg


 S. javiana

y K. ozanae causes progressive atrophy of nose and pharynx
y

y NTS is more common in tropical countries


y
which presents as a foul smelling discharge
y It is mainly associated with consumption of animal products
y
y K. rhinoscleromatis causes rhinoscleroma, a destructive
y

especially chicken eggs granuloma of nose and pharynx


y Hence Pasteurization of eggs is must to avoid infection
y
y Another species, K.granulomatis (previously called as Ca-
y

y MDR Salmonella was first identified in S. typhimurium


y
lymmatobacterium granulomatis) – causes chronic ulcer-
– named as ACSSuT – that is it is resistant to Ampicillin, ative disease of genital region (STD) – named as Granuloma
Chloramphenicol, Streptomycin, Sulfonamides, Tetracycline inguinale
y It causes gastroenteritis, bacteremia, CNS infections, UTI,
y
y K. oxytoca is another species which is also isolated from
y

bone and joint infections clinical specimens


y Salmonella osteomyelitis occurs in sickle cell disease
y

patients ENTEROBACTER SPECIES


y Treatment:
y

 Gastroenteritis – Ciprofloxacin

y Three species are seen:
y

 E. cloacae
 Systemic infections – Ceftriaxone


 E. aerogenes


 E. sakazakii (this has been included now in Cronobacter)


TRIBE IV: CITROBACTER SPECIES


y Gram-negative, lactose fermenter


y

y Motile, IMViC = --++


y

• • Gram-negative bacilli y Normal commensal found in feces


y

• • Motile y Causes UTI, pneumonia, nosocomial infections, wound and


y

• • Late lactose fermenter device infections


• Citrate utilizing organism y Most of the strains are intrinsically resistant to ampicillin,
124

y

• • H2S producer – C. freundii first and second generation cephalosporins


SERRATIA MARCESCENS Dienes phenomenon – to detect swarming growth

Chapter 13     Enterobacteriaceae




 Swarming cannot be seen in MacConkey agar; it can be


y Gram-negative, pigment producing organism
y

see in nutrient agar and blood agar


y It produces a diffusible pigment named as Prodigiosin which
y

gives the culture pink or magenta colored


y Serratia when causes respiratory infection – it produces
y

colored sputum due to pigment production that may


be mistaken from hemoptysis – hence it is termed as
pseudohemoptysis
y But only 10% of the isolates produce the red pigment
y

y It causes pneumonia, bacteremia and endocarditis in


y

narcotics addicts (usually the nonpigmented isolated are


more causing hospital acquired infections)

Figure 6: Swarming nature of Proteus sp., (Dienes’ phenomenon)


(Courtesy: CDC/ Dr John J Farmer)
y It causes UTI, bacteremia, pneumonia, focal lesions in
y

debilitated patients and infections in burn patients


y It is a powerful urease splitter; hence in UTI – it causes
y

hydrolysis of urea and liberates ammonia – ammonia is an


alkalyzer which leads to alkalinization of urine
y Alkalinization leads to formation of struvite stones (Staghorn
y

calculi)
y P.mirabilis is the most common isolate in clinical specimens
Figure 5: Pigmented colonies of Serratia in blood agar
y

y P. mirabilis is sensitive to ampicillin and cephalosporins


(Courtesy: CDC/ Dr Negut)
y

usually; in contrast, P.vulgaris is highly resistant to antibiotics

TRIBE VI: PROTEUS, PROVIDENCIA AND Providencia Species


MORGANELLA GROUP y y Species are Providencia rettgeri, P. alcalifaciens, P. stuartii
y These are seen as members of normal intestinal microbiota
Proteus Species y
y

y P. stuartii causes UTI and infections in burns


y Drugs sensitive usually are Amikacin and ciprofloxacin
• Gram-negative bacilli
y

• Motile
TRIBE VII: YERSINIA

• • Nonlactose fermenter
• Pleomorphic, no capsule
• Gram-negative bacilli

• Fishy odor •

• Y. pestis has bipolar staining – safety pin appearance


• Powerful urease producer •

• Pleomorphic

• PPA reaction positive •

• Y. pestis – nonmotile

• Swarming seen •

• Y. pseudotuberculosis and enterocolitica – motile at 22°C


y Proteus species are of Proteus vulgaris, Proteus mirabilis, Pro-


y

teus penneri
y Unique characteristic is swarming growth:
y
Yersinia Pestis
 To inhibit swarming:

y Gram-negative pleomorphic bacilli
y

Š Increased (6%) concentration of agar


Š
y It has characteristic bipolar staining with special stains like:
y

Š Incorporation of chloral hydrate (1:500)


Š
 Wright stain


Š Sodium azide (1:500)


Š
 Giemsa stain


Š Alcohol (5-6%)
Š
 Wayson stain


Š Sulfonamide
Š
y It can be grown in basal media like nutrient agar and blood
y

Š Surface active agents


Š
agar
Š Boric acid (1:1000)
Š
y Nonlactose fermenter in MacConkey agar
y

125
Unit 2     Bacteriology

Types of plague Features


Pneumonic plague • Spreads by droplet infection during

epidemics
• Causes hemorrhagic pneumonia

• Cyanosis occurs

• Highly infectious and highly fatal


Septicemic plague • Complication of bubonic or pneumonic


plague

Figure 7: Bipolar staining of Y. pestis (Courtesy: CDC)

Antigenic Structure
y Plague toxin – two toxins:
y

 Endotoxin – LPS in cell wall




 Murine toxins – toxin for animals




y Heat labile protein named as Fraction I (seen in all of the


y

virulent strains)
y Antigens V and W
y

y Bacteriocin named as Pesticin


y
Figure 8: Gangrene seen in plague infection
y Coagulase producer
y

y Fibrinolysin
y

Laboratory Diagnosis
y Unidentified surface component
y Buboes are collected and made special stains to visualize the
y

bacilli
Pathogenesis y Wayson stain characteristically shows the bipolar appearance
y

y Plague is a zoonotic disease (infects rodents which are the


y
y A colorless growth in macConkey agar with clinical
y

reservoir of Y. pestis) history suggestive of plague should be confirmed with


y Transmitted to humans by rat flea (Vector)
y
biochemical reactions followed by definitive identification
y Vector: Xenopsylla cheopis in North India and X.astia in
y
with immunofluorescent or dialysis by a specific Y. pestis
South India bacteriophage (CDC)
y Flea gets infected when feeding on the rodents – the bacilli
y
y Stalactite growth in ghee broth
y

multiplies in the stomach and blocks the proventriculus – this y Serological tests showing a titer >1:16 is suggestive of plague
y

is called as blocked flea. (R/o vaccination history)


y The period between the ingestion of infected blood and
y

blocking of proventriculus is called as Extrinsic incubation Treatment


period. y 10 days course of Streptomycin is the DOC
y

y When this blocked flea ferociously bites human – Y. pestis


y

y Alternative drugs are doxycycline, levofloxacin, chloram-


y

enters through the bitten wound phenicol


y Organisms then multiply in the macrophages and goes for
y

y Plague prophylaxis → Doxycycline or cotrimaxozole


y

lymphatic spread and causes inflammation y Immunoprophylaxis → killed vaccine given subcutaneously
y

y It spreads systemically and causes necrosis


y

two doses at interval of 1-3 months followed by third dose


around six months later
Clinical Features
y Incubation period is 2-7 days
y

Yersiniosis
Table 9:  Types of plague depending on clinical y It is a collective term used for zoonotic infections that are
y

presentation caused other than by Y. pestis


y Organisms are:
y

Types of plague Features  Y. pseudotuberculosis




Bubonic plague • IP = 2–5 days



 Y. enterocolitica


• Inguinal nodes becomes enlarged (Bubo)



y Yersinia pseudotuberculosis:
y

• DIC occurs once the bacilli enters the



 Not grown well in MacConkey agar


bloodstream  Motile at 22°C and non motile at 37°C




 Causes tuberculosis like lesion in infected animals and


126 • Case fatality is 30–90%



hence the name


(Contd...)
y Yersinia enterocolitica: It can cause:

Chapter 13     Enterobacteriaceae


y 

 Motile at 22°C and non motile at 37°C


 Š Self-limiting gastroenteritis in young children
Š

 Nonlactose fermenting colonies


 Š Mesenteric adenitis and inflammatory terminal ileitis
Š

 It has more than 70 serotypes


 in older children
 Usually isolated from domestic animals; humans get
 Š Systemic disease in HLA B 27 individuals
Š

infected by contamination in food or drinks y It needs cold enrichment media for growth
y

 Person to person transmission does not occur


 y Treatment: Mostly self-limited; aminoglycosides, chloram-
y

phenicol, TMP-SMX can be given

MULTIPLE CHOICE QUESTIONS


1. Which of the following is non – lactose fermenting 10. E. coli causing hemolytic uremic syndrome:
bacteria: (Recent Pattern Nov 2015)  (Recent Pattern Dec 2015)
a. E. coli b. Klebsiella a. Enteropathogenic
c. Citrobacter d. Salmonella b. Enterotoxigenic
2. E. coligives pink color with: c. Enteroinvasive
 (Recent Pattern Dec 2014) d. Enterohemorrhagic
a. Chocolate agar b. L.J Medium 11. Most common strain of E. coli giving rise to traveler’s
c. MacConkey’s medium d. Saline broth diarrhea is: (Recent Pattern Dec 2013)
3. Which of the following is late lactose fermenter: a. Enteroinvasive E. coli (EIEC)
 (Recent Pattern July 2016) b. Enteropathogenic E. coli (EPEC)
a. E. coli b. Klebsiella c. Enterotoxigenic E. coli (ETEC)
c. Salmonella d. Shigella sonnei d. Enteroaggregative E. coli (EAEC)
4. True about Enterobacteriaceae family: 12. In E. coli true is: (Recent Pattern Dec 2012)
 (Recent Pattern Dec 2015) a. ETEC is invasive
a. Oxidase positive b. EPEC acts via cAMP
b. Catalase negative c. Pili present in uropathogenic type
c. Reduces nitrates to nitrites d. ETEC causes HUC
d. Glucose non fermenters 13. Most important serotype of E. coli causing Hemolytic
5. Enterobacteriaceae is classified based on: Uremic Syndrome: (Recent Pattern July 2016)
 (Recent Pattern July 2015) a. O157: H7 of EHEC
a. Mannitol fermentation b. O107: H7 of EIEC
b. Catalase and oxidase reaction c. O157: H7 of ETEC
c. Oxygen requirement d. O109: H7 of EAEC
d. Lactose fermentation 14. Culture media used for diagnosis of EHEC O157:H7 is:
6. Watery Diarrhea in children is caused by:  (Recent Pattern Dec 2015)
 (Recent Pattern Dec 2012) a. O2 culture
a. EHEC b. EPEC b. Sorbitol MacConkey media
c. EIEC d. EAEC c. XLD agar
7. Enterohemorrhagic, enterotoxic and enteroinvasive are d. Deoxycholate media
types of: (AIIMS Nov 2014) 15. MC cause of diarrhea in children of developing countries
a. E. coli is: (Recent Pattern Dec 2012)
b. Klebsiella a. EHEC b. ETEC
c. Shigella c. EAEC d. EIEC
d. Streptococcus pneumoniae 16. Toxin acting on cGMP- (Recent Pattern Dec 2013)
8. Regarding ETEC true is: (AIIMS Nov 2010) a. Heat stable E. coli toxin
a. Invades submucosa b. Heat labile E. coli toxin
b. Most common in children of developing countries c. Cholera toxin
c. Fomite borne and person to person d. Shiga toxin
d. Not a common cause of traveler’s diarrhea 17. Most common cause of UTI in young female is:
9. Sereny test is positive in: (Recent Pattern July 2016)  (Recent Pattern August 2013)
a. Enteroinvasive E. coli (EIEC) a. Staph. saprophyticus
b. Enteropathogenic E. coli (EPEC) b. E. coli
c. Enterotoxigenic E. coli (ETEC) c. Klebsiella
d. Enteroaggregative E. coli (EAEC) d. Proteus 127
18. Most common organism implicated in the etiology of 30. Shigellosis is best diagnosed by:
Unit 2     Bacteriology

urinary tract infection in the community is:  (Recent Pattern Dec 2013)
 (Recent Pattern Dec 2013) a. Stool examination b. Stool culture
a. E. coli c. Sigmoidoscopy d. Enzyme
b. Proteus 31. Culture medium for transport of stools in suspected
c. Pseudomonas case of Shigellosis: (Recent Pattern July 2016)
d. Streptococci a. Deoxycholate medium
19. Phenylalanine deaminase test is positive in: b. Blood agar
 (Recent Pattern Dec 2015) c. Nutrient broth
a. Salmonella d. Buffered glycerol saline
b. Proteus 32. Salmonella and Shigella can be differentiated from
c. Vibrio cholerae other Enterobacteriaceae member by isolation on:
d. Helicobacter
 (Recent Pattern Dec 2016)
20. Proteus antigen cross reacts with:
a. MacConkey agar
(Recent Pattern August 2013)
b. Mannitol salt agar


a. Klebsiella b. Rickettsiae
c. BYCE medium
c. Chlamydiae d. E. coli
d. XLD agar
21. Proteus isolated from a patient of UTI will show which
biochemical reaction: (Recent Pattern Dec 2016) 33. The following are gas producing Salmonella except:
a. Phenyl-pyruvic acid reaction  (Recent Pattern Nov 2014)
b. Bile esculin reaction a. S. typhi
c. Colchicine sensitivity b. S. enteritidis
d. Bacitracin sensitivity c. S. cholera
22. Frisch bacillus affects most commonly: d. S. typhimurium
 (Recent Pattern Dec 2015) 34. Pea-soup stool is characteristically seen in:
a. Mouth b. Nose  (Recent Pattern Dec 2014)
c. Eye d. Ear a. Cholera b. Typhoid
23. ‘Hebra’ nose is caused by: (Recent Pattern Dec 2016) c. Botulism d. Polio
a. Frisch bacillus b. Staph aureus 35. Diagnosis of typhoid in the first week is by:
c. Pseudomonas d. C. diphtheria  (Recent Pattern Dec 2012)
24. Friedlander’s bacillus is: (Recent Pattern Dec 2013) a. Widal test b. Stool culture
a. E. coli c. Urine culture d. Blood culture
b. Pseudomonas aeruginosa 36. Clinical significance of Vi antigen of S.typhi is:
c. Klebsiella pneumoniae  (Recent Pattern Dec 2016)
d. Vibrio parahemolyticus a. Helps in diagnosis
25. Most virulent variety of Shigellosis is caused by: b. Highly immunogenic
 (Recent Pattern Dec 2014) c. Most important antigen of Widal test
a. S. dysenteriae b. S. sonnei d. Antibody against Vi antigen is used for diagnosis of
c. S. flexneri d. S. boydii carrier
26. Which of the following toxins acts by inhibition of 37. True about Widal test: (Recent Pattern Dec 2016)
protein synthesis: (Recent Pattern Dec 2016) a. Anti-O antibody persists longer
a. Cholera toxin
b. O antigen of S.paratyphi is used
b. Shiga toxin
c. H-antigen is most immunogenic
c. Pertussis toxin
d. Felix tube is used for O agglutination
d. LT of enterotoxigenic E. coli
38. Most immunogenic in typhoid:
27. True about Shiga toxin: (Recent Pattern July 2016)
(Recent Pattern Dec 2013)
a. Produced by Shigella sonnei


b. Chromosomal encoded a. O antigen b. H antigen


c. Acts by stimulating adenylyl cyclase c. Vi antigen d. Somatic antigen
d. An endotoxin 39. Absence of Vi antigen in a typhoid patient has:
28. True about Shiga toxin: (Recent Pattern Nov 2015)  (Recent Pattern Dec 2016)
a. An endotoxin a. Good prognosis
b. Inhibit protein synthesis b. Bad prognosis
c. Activate adenylyl cyclase c. No indication with prognosis
d. Increase cGMP d. Widal negative
29. Selective medium for Shigella: 40. Not true about Vi polysaccharide vaccine of typhoid:
 (Recent Pattern Dec 2015)  (Recent Pattern Dec 2016)
a. Chocolate agar b. BYCE agar a. Single dose is given b. Revaccination at 3 years
128 c. Hektoen agar d. EMJH medium c. Given at birth d. Given subcutaneously
41. Temporary carrier of typhoid is infective for  51. A farmer presents to the emergency department with

Chapter 13     Enterobacteriaceae


 (Recent Pattern Nov 2014) painful inguinal lymphadenopathy and history of fever
a. <3 Weeks b. 3 weeks – 3 months and flu like symptoms. Clinical examination reveals an
c. 3 months – 1 year d. >1 year ulcer in the leg. Which of the following strains should
42. Vi antigen is found in:  (PGI Nov 2016) be used to detect suspected bipolar stained organisms:
a. Salmonella paratyphi A  (AIIMS Nov 2012, (Recent Pattern 2011)
b. Salmonella paratyphi C a. Albert’s stain b. Wayson’s stain
c. Salmonella dublin c. Ziehl Neelson stain d. Mc Fayden’s stain
d. Klebsiella pneumoniae 52. Which is not true about yersiniosis: 
e. Citrobacter fregalis  (Recent Pattern Dec 2012)
43. All are true about non typhoid salmonella except: a. Zoonosis b. Caused by Y. pestis
 (AIIMS May 2011) c. By Y. enterocolitica d. By Y. pseudotuberculosis
a. Poultry is source
53. The drug of choice for chemoprophylaxis in contacts of
b. Can cause infective disease in neonates
a patient of pneumonic plague is: 
c. Blood culture is more sensitive than stool culture in
(Recent Pattern Dec 2016)
gasteroenteritis in adults


a. Penicillin
d. Resistance to fluoroquinolone has emerged
44. Salmonella other than S. Typhi and S. Paratyphi can b. Rifampicin
cause: c. Erythromycin
a. Typhoid fever b. Enteric fever d. Tetracycline
c. Gastroenteritis d. All of the above 54. Izumi fever is caused by:  (Recent Pattern Dec 2016)
45. DT 104 strain is belongs to which of the following bacte- a. Pseudomonas auerginosa
ria:  (PGI Nov 2012) b. Burkholderia mallei
a. Salmonella gallinarum b. Salmonella typhi c. Yersinia pseudotuberculosis
c. Salmonella enteritidis d. Salmonella paratyphi A d. Pasteurella multocida
e. Salmonella typhimurium 55. Appendicitis like syndrome is caused by all except:
 (Recent Pattern Nov 2014)
a. Yersinia enterocolitica
Yersinia b. Yersinia pseudotuberculosis
46. Causative agent of plague:  (Recent Pattern Dec 2013)
c. Pasteurella septica
a. Yersinia pestis
b. Yersinia enterocolitica d. Yersinia pestis
c. Yersinia pseudotuberculosis 56. What type of E. coli is shown in the following image?
d. Pasteurella septica  (AIIMS May 2018)
47. Stalactite growth in ghee broth is due to: 
 (Recent Pattern Nov 2014)
a. H. influenza
b. C. diphtheria
c. Y. pestis
d. T. pallidium
48. Pneumonic plague is caused by: 
 (Recent Pattern Dec 2016)
a. Bite of infected flea
b. Direct contact with infected tissue
c. Ingestion of contaminated food
d. Droplet infection
49. Reservoir of plague is:  (Recent Pattern Dec 2015)
a. Domestic rat b. Wild rat
c. Rat flea d. Man
50. False statement about plague is: 
 (Recent Pattern Dec 2013) a. ETEC b. EIEC
a. It is a gram–negative coccobacillus responding to c. EPEC d. EAEC
streptomycin 57. Proteus mirabilis is intrinsically resistant to: 
b. Bubonic plague is the most common form  (PGI Nov 2018)
c. Pneumonic plague develops most rapidly and is most a. Tetracycline b. Colistin
frequently fatal c. Nitrofurantoin d. Ceftriaxone
d. The bubo of plague is characterised by intense cellulitis e. Cotrimoxazole

129
ANSWERS AND EXPLANATIONS
Unit 2     Bacteriology

1. Ans.  (d) Salmonella • 5 types of diarrheogenic E. coli are


ƒ EPEC – causes diarrhea in infants and children


Ref: Ananthanarayan and Paniker’s Textbook of Microbiol-
ƒ

ƒ ETEC – MCC of travellers diarrhea


ogy – 10th ed – Page 279
ƒ

ƒ EIEC – causes illness similar to shigellosis


ƒ

• All the options provided belong to the group


ƒ EHEC – causes hemolytic uremic syndrome (HUS)


ƒ

Enterobacteriaceae which are classified on the basis of ƒ EAEC – causes persistent diarrhea.
ƒ

lactose fermentation.
• The lactose fermenters are :
• 7. Ans.  (a) E. coli
ƒ E. coli
Ref: Ananthanarayan and Paniker’s Textbook of Microbiol-
ƒ

ƒ Citrobacter
ogy – 10th ed – Page 285
ƒ

ƒ Klebsiella
ƒ

ƒ Enterobacter
ƒ
• Already explained in Q.6

• The lactose non-fermenters are:


8. Ans.  (c) Fomite borne and person to person

ƒ Salmonella
ƒ

ƒ Shigella
ƒ

Ref: Ananthanarayan and Paniker’s Textbook of Microbiol-


ƒ Proteus
ƒ

ogy – 10th ed – Page 285

2. Ans.  (c) MacConkey’s medium • Enterotoxigenic E. coli (ETEC) affects all age groups and

is endemic in developing countries. It is the MCC of


Ref: Ananthanarayan and Paniker’s Textbook of Microbiol- Travelers diarrhea. It is spread by contaminated water
ogy – 10th ed – Page or food. The pathogenesis is through production of toxin
• MacConkeys medium is a differential media to

and not by invasion.
differentiate between lactose fermenters and
nonfermenters. E. coli being a lactose fermenter 9. Ans.  (a) Enteroinvasive E. coli (EIEC)
produces pink colored colonies in this media. Ref: Ananthanarayan and Paniker’s Textbook of Microbiol-
ogy – 10th ed – Page 286
3. Ans.  (d) Shigella sonnei
• Enteroinvasive E. coli is diagnosed by Sereny test which

Ref: Ananthanarayan and Paniker’s Textbook of Microbiol- involves inoculating suspension of bacteria into guinea
ogy – 10th ed – Page 279 pigs leading to mucopurulent conjunctivitis.
• Though shigella are lactose non-fermenters, S. sonnei is
10. Ans.  (d) Enterohemorrhagic

an exception as it is a late lactose fermenter.


Ref: Ananthanarayan and Paniker’s Textbook of Microbiol-
4. Ans.  (c) Reduces nitrates to nitrites ogy – 10th ed – Page 285
Ref: Jawetz book on medical microbiology, 26th edition pg • Already explained in Q.6

229
Members of the family Enterobacteriaceae have the follow- 11. Ans.  (c) Entero-toxigenic E. coli (ETEC)
ing characteristics: Ref: Ananthanarayan and Paniker’s Textbook of Microbiol-
• Gram-negative rods

ogy – 10th ed – Page 285
• Grow on peptone or meat extract media without
• Already explained in Q.6 and 8

addition of sodium chloride or other supplements


• Grow well on MacConkey agar


12. Ans.  (c) Pili present in uropathogenic type


• Are facultative anaerobes

• Ferment glucose often with gas production



Ref: Jawetz book on medical microbiology 26th edition pg
• Catalase positive and oxidase negative

233
• Reduces nitrate to nitrite.

• Option A: ETEC is not invasive and acts through its toxin

• Option B: EPEC acts by disruption of brush border of


5. Ans.  (d) Lactose fermentation intestinal mucosa


Ref: Ananthanarayan and Paniker’s Textbook of Microbiol- • Option C: Uropathogenic E. coli elaborate a specific

ogy – 10th ed – Page 279 type of pilus which helps in adhesion, colonization and
subsequent infections.
• The classification is already explained in Q.1
• Option D: HUS is produced by EHEC.

6. Ans.  (b) EPEC


13. Ans.  (a) O157: H7 of EHEC
Ref: Ananthanarayan and Paniker’s Textbook of Microbiol-
Ref: Jawetz book on medical microbiology 26th edition pg
130 ogy – 10th ed – Page 285
234
• HUS is caused by enterohemorrhagic E. coli (EHEC) 20. Ans.  (b) Rickettsiae

Chapter 13     Enterobacteriaceae


most commonly the O157:H7 serotype.


Ref: Ananthanarayan and Paniker’s Textbook of Microbiol-
14. Ans.  (b) Sorbitol MacConkey media ogy – 10th ed – Page 289
• There is sharing of alkali stable carbohydrate antigen of
Ref: Ananthanarayan and Paniker’s Textbook of Microbiol-

Rickettsia and non-motile strains of Proteus mirabilis


ogy – 10th ed – Page 286
(OX-K) and Proteus vulgaris (OX-2, OX-19) which
• O157:H7 serotype of EHEC does not ferment sorbitol and

forms the basis of the Weil-Felix reaction (a heterophile


thus sorbitol MacConkey agar is used for its diagnosis. agglutination reaction).

15. Ans.  (b) ETEC 21. Ans.  (a) Phenyl- pyruvic acid reaction
Ref. Jawetz Book on Medical Microbiology 26 edition pg
th
Ref: Ananthanarayan and Paniker’s Textbook of Microbiol-
234 ogy – 10th ed – Page 289
• Though EPEC is most commonly associated with

diarrhea in infants and children, since it is not provided Test Organism


in the option we need to go for option B as ETEC is Optochin sensitivity Strep. pneumonia
mentioned as a very important cause of diarrhea in Quellung reaction
children in developing countries.
CAMP reaction Strep. agalactiae
16. Ans.  (a) Heat stable E. coli toxin Nagler reaction Clostridium perfringens
Ref: Ananthanarayan and Paniker’s Textbook of Microbiol- McFadyen’s reaction B. anthracis
ogy – 10th ed – Page 282 Kanagawa phenomenon Vibrio parahemolyticus
Heat stable Heat labile E. Verocytotoxin or Bile esculin test Enterococci
E. coli toxin coli toxin cholera shiga like toxin Phenyl pyruvic acid reaction Proteus
toxin
Bacitracin sensitivity Strep.pyogenes
Plasmid mediated Plasmid mediated Phage coded
– E. coli Neil-mooser reaction Rickettsia typhi
Chromosome
medicated – 22. Ans.  (b) Nose
Cholera Ref: Ananthanarayan and Paniker’s Textbook of Microbiol-
ogy – 10th ed – Page 287
Acts through cGMP Acts through cAMP Inhibits ribosome
and decreases • Frisch bacillus is the other name of Klebsiella rhinoscle-

protein synthesis romatis the causative agent of rhinoscleroma which is a


granulomatous condition affecting most commonly the
nose.
17. Ans.  (b) E. coli
23. Ans.  (a) Frisch bacillus
Ref: Ref. Jawetz Book on Medical Microbiology 26th edition
pg 233 Ref: Ananthanarayan and Paniker’s Textbook of Microbiol-
ogy – 10th ed – Page 287
• E. coli is the most common cause of complicated and

uncomplicated UTI and accounts for 90% of first UTIs • The term Hebra nose is related to the appearance of the

in young women. nose in nodular stage of rhinoscleroma. Tapir nose is the


term during the cicatrizing stage.
18. Ans.  (a) E. coli
24. Ans.  (c) Klebsiella pneumoniae
Ref: Ananthanarayan and Paniker’s Textbook of Microbiol-
ogy – 10th ed – Page 283 Ref: Ananthanarayan and Paniker’s Textbook of Microbiol-
ogy – 10th ed – Page 287
• Already explained in Q.17

19. Ans.  (b) Proteus Bacteria Other name


Corynebacterium diphtheria Kleb loefller’s bacillus
Ref: Ananthanarayan and Paniker’s Textbook of Microbiol-
ogy – 10th ed – Page 289 Corynebacterium Preisz-Nocard bacillus
• Proteus differs from others Enterobacteriaceae by

pseudotuberculosis
presence of enzyme phenylalanine deaminase which Haemophilus aegyptius Koch weeks bacillus
converts phenylalanine to phenyl pyruvic acid. This is
Haemophilus influenzae Pfeiffer’s bacillus
called as phenylalanine deaminase test. 131
Contd...
The best way to diagnose shigella is by culture of mucus
Unit 2     Bacteriology

Bacteria Other name


flakes of stool onto selective media.
Klebsiella pneumoniae Friedlander’s bacillus
Klebsiella ozanae Abel’s bacillus 31. Ans.  (d) Buffered glycerol saline

Mycobacterium tuberculosis Koch’s bacillus Ref: Ananthanarayan and Paniker’s Textbook of Microbiol-
ogy – 10th ed – Page 294
Mycobacterium Johne’s bacillus
The transport media used for shigella is Sach’s buffered
paratuberculosis
glycerol saline.
Mycoplasma Eaton agent
32. Ans.  (a) MacConkey agar
Pseudomonas pseudomallei Whitmore’s bacillus
Mycobacterium intracellulare Battey’s bacillus Ref: Ananthanarayan and Paniker’s Textbook of Microbiol-
ogy – 10th ed – Page 294
Klebsiella rhinoscleromatis Frisch bacillus
Salmonella and Shigella are non-fermenters of lactose and
thus by culturing on MacConkey agar can be differentiated
25. Ans.  (a) S. dysenteriae from other Enterobacteriaceae.
Ref: Ananthanarayan and Paniker’s Textbook of Microbiol-
ogy – 10th ed – Page 292 33. Ans.  (a) S. typhi
• The most virulent variety of shigellosis is caused by

Ref: Ananthanarayan and Paniker’s Textbook of Microbiol-
Sh. dysenteriae type 1, while the mildest form is caused ogy – 10th ed – Page 297
by Sh.sonnei. Salmonella ferment glucose with production of acid and
gas with the exception of S.typhi which is anaerogenic.
26. Ans.  (b) Shiga toxin
Ref: Ananthanarayan and Paniker’s Textbook of Microbiol- 34. Ans.  (b) Typhoid
ogy – 10th ed – Page 293 Ref: Ananthanarayan and Paniker’s Textbook of Microbiol-
Toxins that inhibit protein synthesis ogy – 10th ed – Page 301
• Sh. dysenteriae type 1

The term pea-soup stools is used in typhoidal diarrhea


• Diphtheria

whereas the term rice-water stools refers to cholera stools.


• Pseudomonas

• Verotoxin = shiga like toxin of E. coli


• 35. Ans.  (d) Blood culture

27. Ans.  (b) Chromosomal encoded Ref: Ananthanarayan and Paniker’s Textbook of Microbiol-
ogy – 10th ed – Page 303
Ref: Ananthanarayan and Paniker’s Textbook of Microbiol-
The mnemonic BASU helps remember the order of
ogy – 10th ed – Page 292
positivity of various investigations in typhoid
• Shiga toxin is an exotoxin which is chromosomally

• B – blood culture -1st week


encoded and produced by Sh. dysenteriae type 1 and • A – antibodies (Widal test) – 2nd week

acts by inhibiting protein synthesis. • S – Stool culture – 3rd week


• U – urine culture – 4th week.


28. Ans.  (b) Inhibit protein synthesis

Ref: Ananthanarayan and Paniker’s Textbook of Microbiol- 36. Ans.  (d) Antibody against Vi antigen is used for diagno-
ogy – 10th ed – Page 292 sis of carrier
• Already explained in Q.27

Ref: Ananthanarayan and Paniker’s Textbook of Microbiol-
ogy – 10th ed – Page 299
29. Ans.  (c) Hektoen agar • Vi polysaccharide antigen is used for the identification

Ref: Ananthanarayan and Paniker’s Textbook of Microbiol- of carriers of salmonella.


ogy – 10th ed – Page 291
37. Ans.  (c) H-antigen is most immunogenic
The selective media for salmonella and shigella are:
• Xylose-lysine decarboxylase (XLD)

Ref: Ananthanarayan and Paniker’s Textbook of Microbiol-
• Deoxycholate citrate agar (DCA)

ogy – 10th ed – Page 299
• Hektoen enteric agar

• • H (flagellar antigen) – strongest antigen


• Samonella-shigella agar

• • Vi (capsular antigen) – weaker than H antigen


• O (somatic antigen) – weakest antigen
30. Ans.  (b) Stool culture

• • 2 types of tubes are used in Widal reaction – a narrow


Ref: Ananthanarayan and Paniker’s Textbook of Microbiol- tube with conical bottom (Dreyer’s agglutination tube)
132 ogy – 10th ed – Page 294 for H agglutination with the formation of cotton wooly
clumps and a round bottomed tube (Felix tube) for O 46. Ans.  (a) Yersinia pesits

Chapter 13     Enterobacteriaceae


agglutination with the formation of disc-like pattern.
Ref: Ananthanarayan and Paniker’s Textbook of Microbiol-
38. Ans.  (b) H antigen ogy – 10th ed – Page 325
• Yersinia pestis – causative agent of plague.
Ref: Ananthanarayan and Paniker’s Textbook of Microbiol-

• Yersinia pseudotuberculosis – a primary pathogen of


ogy – 10th ed – Page 299

rodents.
• Already explained in Q.37

• Yersinia enterocolitica – causes enteric and systemic


disease in animals and humans.


39. Ans.  (a) Good prognosis • Pasteurella spp. – hemorrhagic septicemia in animals

Ref: Ananthanarayan and Paniker’s Textbook of Microbiol- and occasionally local or systemic infections in human.
ogy – 10th ed – Page 299
47. Ans.  (c) Y . pestis
• Vi antigen is associated with virulence of the organism

and thus its absence is associated with good prognosis. Ref: Ananthanarayan and Paniker’s Textbook of Microbiol-
ogy – 10th ed – Page 326
40. Ans.  (c) Given at birth • When Yersinia pestis is grown in ghee broth (flask of

Ref: Ananthanarayan and Paniker’s Textbook of Microbiol- broth with oil or ghee floated on top), a characteristic
ogy – 10th ed – Page 306 growth which hangs down from the surface into the
broth, resembling stalactite occur.
• Vi polysaccharide vaccine is an injectable vaccine given

as a single dose. Being a polysaccharide vaccine it is not 48. Ans.  (d) Droplet infection
given to infants and young children <2 years.
Ref: Ananthanarayan and Paniker’s Textbook of Microbiol-
41. Ans.  (c) 3 months – 1 year ogy – 10th ed – Page 328
Ref: Ananthanarayan and Paniker’s Textbook of Microbiol- • Pneumonic plague is spread by droplet infection. The

ogy – 10th ed – Page 302 bacilli spread through the lungs producing hemorrhagic
pneumonia. Cyanosis is very prominent. It is highly
• Convalescent carrier – from clinical cure to next 3
infectious as the bloody mucoid sputum that is coughed

months.
out contains bacilli in enormous numbers.
• Temporary carrier – sheds typhoid bacilli for 3 months –

1 year after the onset of illness 49. Ans.  (c) Rat flea
• Chronic carrier – sheds bacilli for more than 1 year.

Ref: Ananthanarayan and Paniker’s Textbook of Microbiol-


42. Ans.  (a) Salmonella paratyphi A; (b) Salmonella ogy – 10th ed – Page 328
paratyphi C • Species of rat fleas acting as vectors in

Ref: Ananthanarayan and Paniker’s Textbook of Microbiol- • North India – Xenopsylla cheopis

ogy – 10th ed – Page 299 • South India – Xenopsylla astia


• Vi antigen is present only in S. typhi and S. paratyphi.


50. Ans.  (d) The bubo of plague is characterised by intense


cellulitis
43. Ans.  (c) Blood culture is more sensitive than stool
culture in gasteroenteritis in adults Ref: Ananthanarayan and Paniker’s Textbook of Microbiol-
ogy – 10th ed – Page 328
Ref: Ananthanarayan and Paniker’s Textbook of Microbiol-
ogy – 10th ed – Page 307 • As the plague bacillus usually enters through flea bites

on the legs, the inguinal nodes are involved and hence


• Blood culture is negative in patients with gastroenteritis.
the ‘bubonic’ (bubon means groin). The glands become

44. Ans.  (c) Gastroenteritis enlarged and suppurate.

Ref: Ananthanarayan and Paniker’s Textbook of Microbiol- 51. Ans.  (b) Wayson’s stain
ogy – 10th ed – Page 307
Ref: Ananthanarayan and Paniker’s Textbook of Microbiol-
Nontyphoidal salmonellosis is caused by ogy – 10th ed – Page 329
• S. typhimurium causing gastroenteritis
• The clinical features are suggestive of bubonic plague.

• S. choleraesuis causes bacteremia.


Hence, smears from the bubo should be stained with


45. Ans.  (e) Salmonella Typhimurium Wayson stain, to show the bipolar stained Yersinia pestis.

Ref: Ananthanarayan and Paniker’s Textbook of Microbiol- 52. Ans.  (b) Caused by Y.pestis
ogy – 10th ed – Page 307
Ref: Ananthanarayan and Paniker’s Textbook of Microbiol-
• DT-104 strain is a multi-drug resistant strain of ogy – 10th ed – Page 330 133

S. typhimurium which can cause blood stream


infections.
• Yersiniosis denotes infections with Yersiniae other • Y. enterocolitica – causes mesenteric adenitis and
Unit 2     Bacteriology

• •

than Y.pestis. they include zoonotic infections by inflammatory terminal ileitis in older children that may
Y. pseudotuberculosis and Y. enterocolitica. mimic appendicitis.
• Pasteurella spp. – causes local suppuration following

53. Ans.  (d) Tetracycline animal bites (wound infection, cellulitis, abscess,
Ref: Ananthanarayan and Paniker’s Textbook of Microbiol- osteomyelitis), meningitis, pneumonia, bronchitis,
ogy – 10th ed – Page 330 sinusitis, appendicitis and appendicial abscess.

• An exposed person, whether vaccinated or not, should



56. Ans.  (d) EAEC
be given chemoprophylaxis with cotrimoxazole or
tetracycline orally for at least 5 days. Ref: Mandell’s T. B of infectious diseases – 8th edition – page
2513
54. Ans.  (c) Yersinia pseudotuberculosis • EAEC strains, first recognized as a cause of diarrhea

Ref: Ananthanarayan and Paniker’s Textbook of Microbiol- in 1987, are defined by their aggregative pattern of
ogy – 10th ed –Page 330 adherence to tissue culture cells
• These bacteria form two-dimensional clusters when
• Y. pseudotuberculosis causes scarlet like fever which is

they adhere to cells in vitro, to glass slides, to plastic, or


called Izumi fever in India. to the intestinal mucosa
55. Ans.  (d) Yersinia pestis 57. Ans.  (a,b,c) a. Tetracycline; b. Colistin; c. Nitrofurantoin
Ref: Ananthanarayan and Paniker’s Textbook of Microbiol- Ref: Manual of clinical microbiology – 11th edition – page
ogy – 10th ed –Page 330,331 730
• Y. pseudotuberculosis – the natural mode of infection

• Proteus mirabilis is intrinsically resistant to Nitrofuran-


is through alimentary tract. It can sometimes cause toin, colistin and tetracycline.
appendicitis like syndrome.

134
14
Vibrio
VIBRIO SPECIES
• Gram-negative short curved bacilli

• Fish in stream appearance

• Darting motility – V. cholerae

• Special media – TCBS media

VIBRIO CHOLERAE
y Gram-negative short curved rod that has characteristic
y
darting or actively motile nature – polar flagella
y It is aerobic which grows slightly in the range of alkaline
y
medium (6.4 to 9.6)
y NaCl concentration is required for growth of all vibrios; for
y
V. cholerae it is 0.5 to 1% that is optimum for growth. Hence
called as halotolerant vibrio.
y It can grow well in basal media; to identify it from commensal
y
bacteria from stool – a special media called TCBS –
Thiosulphate citrate bile salt agar medium is used – yellow Figure 1: Curved shaped V. cholerae (Courtesy: CDC)
colored colonies indicate V. cholerae.

Classification of V. cholerae
y Two different types of classification:
y
 Heiberg grouping

 Gardner and Venkatraman’s classification

Figure 2: Gardner and Venkatraman classification
y Most common biotype in India is El tor vibrios; most common serotype is Ogawa
y
y O-139 strain is Bengal strain
y
Characteristics of El Tor Vibrio Plating media: Alkaline bile salt agar (BSA), Gelatin
Unit 2     Bacteriology



taurocholate trypticase tellurite agar (GTTA), Thiosulfate


y Much hardier than classical Vibrio – survives in the environ-
citrate bile salt sucrose (TCBS)
y

ment much longer


Š Identification is by TCBS media – sucrose fermenting
y Causes mild infections
Š

colonies gives yellow colour – identified as V. cholerae


y

y More chance for carriers


• Each subgroups and serovars has characteristic sera –
y

y Secondary attack rate is less


cholera isolates need to be agglutinated with specific


y

sera to identify the serotypes


Table 1:  Differentiation of classical and El Tor Vibrios
• Any isolate obtained should be sent to National Institute

Test Classical El Tor of Cholera and enteric disease located at Kolkata


cholera • Cholera is a notifiable disease

Hemolysis – +
VP test – +
Chick erythrocyte agglutination – +
Polymyxin B sensitivity + –
Group IV phage susceptibility + –
El Tor phage 5 susceptibility – +

Cholera
Pathogenesis of Cholera
y Cholera is a toxin mediated disease
y

y IP – 24 hours
y

y Cholera toxin is a potent enterotoxin regulated by toxin


y

coregulated pilus (TCP)


y It has two units A (A1 and A2) and B
y

y A1 fragment → ADP ribosylation of GTP binding protein →


y

activation of adenyl cyclase → activation of cAMP


y This leads to outpouring into small intestinal lumen because
y

of activation of secretory chloride transport system in crypt


cells Figure 3: Yellow colored colonies of V. cholerae in TCBS agar
y More water moves passively leading to watery diarrhea →
y

(Courtesy: CDC)
massive water and electrolytes depletion
y Another toxin called endotoxin has no role in pathogenesis
Treatment
y

of cholera
y Administration of ORS – fluids and electrolytes replacement
y

Clinical Features is the mainstay of treatment


y y Sudden onset of painless watery diarrhea y Antimicrobials is not must for cure but it decreases the
y

y y No fever duration and volume of fluid loss


 Erythromycin
y y Muscle cramps 

 Azithromycin
y y Characteristic rice water stool 

 Tetracycline
y y Complications: Hypovolaemic shock, muscular cramps, 

 Doxycycline
renal failure, pulmonary oedema, cardiac arrhythmias and 

paralytic ileus y Prophylaxis:


y

 Killed oral whole cell vaccines




Laboratory Diagnosis  Live oral vaccines




y Stool microscopy: Darting motility can be seen


Table 2:  Drug of choice for various age groups
y

y Rapid bed side diagnosis is by visualization of darting motility


y

in microscopy followed by addition of antiserum and look for Drug of choice


inhibition of motility – it needs dark field microscope
y Stool sample should be in specific media:
y
Adults Macrolides or doxycycline
 Holding or transport media: Venkatraman-Ramakrish-


Children Cotrimoxazole
nan (VR) medium, Cary Blair medium, Autoclaved sea
water Pregnancy Furazolidone
 Enrichment media for Vibrio cholerae: Alkaline peptone
Chemoprophylaxis Tetracycline


water (pH = 8.6) and Monsur’s taurocholate tellurite


136
peptone water (pH = 9.2)
HALOPHILIC VIBRIOS

Chapter 14     Vibrio


Vibrio parahemolyticus • • Inhabits shrimps and crabs
• • Capsulated vibrio
• • Bipolar staining
• • Peritrichous flagella
• • Wagatsuma agar – Kanagawa phenomenon
• • Causes watery diarrhea in those who consume raw crabs or Shrimps
Vibrio vulnificus • Causes primary sepsis in patients with cirrhosis or hemochromatosis; also causes wound infections

• History of contact with sea water seen


Vibrio alginolyticus • • Most halophilic among all


• • Can grow in salt concentration of even >10%
• • Causes otitis, eye and wound infections
• • DOC: Tetracycline

High Yield
Recap of all Toxins
Enterotoxins Cytotoxins Neurotoxins
Cholera toxin Shigella dysenteriae type 1 Staph aureus
Vibrio parahaemolyticus Enterohemorrhagic E. coli Bacillus cereus
E. coli  LT and ST of ETEC, EAEC and EHEC Clostridium difficile (Toxin B) Cl. botulinum toxin
•• Clostridium difficile (Toxin A)
•• Aeromonas
•• Rota virus (NSP4)
•• Campylobacter jejuni

Toxins Mechanism of action


Diphtheria toxin • ADP-ribosyl transferase.

(Corynebacterium diphtheriae) • Inactivation of ribosomal elongation factor eEf2 resulting from ADP-ribosylation during protein

synthesis
• Leads to cell death.

Cholera toxin • ADP-ribosyl transferase.


(Vibrio cholerae) • ADP-ribosylation of regulatory protein G5 of adenylate cyclase, resulting in permanent


activation of this enzyme and increased levels of cAMP


• Increased secretion of electrolytes.

Tetanus toxin • Metalloprotease. Proteolytic cleavage of protein in components from the neuro exocytosis

(Clostridium tetani) apparatus in the synapses of the anterior horn that normally transmit inhibiting impulses to the
motor nerve terminal.
Membrane toxins • Phospholipase.

Alpha toxin
(Clostridium perfringens)
Listeriolysin • Pore formation in membranes.

(Listeria monocytogenes)
Superantigen toxins • Stimulation of secretion of cytokines in T cells and macrophages.

Toxic shock syndrome toxin-1


(TSST-1)
(Staphylococcus aureus)

137
MULTIPLE CHOICE QUESTIONS
Unit 2     Bacteriology

1. Cholera is caused by: (PGI Nov 2014) 13. Cholera toxin binds to which receptors in the intestine:
a. Vibrio cholerae – 01  (Recent Pattern July 2016)
b. Vibrio cholerae – 0139 a. Sphingosine through A subunit
c. Vibrio parahaemolyticus b. Sphingosine through B subunit
d. NAG vibrio c. GMI gangliosides through A subunit
2. True about El Tor vibrio:  (Recent Pattern Dec 2016) d. GMI gangliosides through B subunit
a. More SAR b. VP reaction (+) ve 14. Phage encoded exotoxin of vibrio cholerae resembles
c. Low carrier rate d. More severe which toxin of E. coli:  (Recent Pattern Nov 2015)
3. The characteristic features of El Tor Cholera are all ex- a. Heat labile toxin b. Heat stable toxin
cept: (Recent Pattern Nov 2014) c. Shiga like toxin d. Verocytotoxin
a. More of subclinical cases 15. True about cholera:  (Recent Pattern Dec 2012)
b. Mortality is less a. Gram-negative rod
c. Secondary attack rate is high in family b. Associated with fever
d. El Tor vibrio is harder and able to survive longer c. Causes painful watery diarrhea
4. Not true about El Tor vibrio 01:  (All India 2010) d. It is an achlorhydria which renders and individual
a. Animals are the only reservoir susceptible to disease
b. Epidemiologically indistinguishable from V cholera 16. The endotoxin of the following Gram-negative bacteria
0139 does not play any part in the pathogenesis of the natural
c. Humans acts as a vehicle for spread disease: (AIIMS Nov 2012, Recent Pattern 2012)
d. The efficacy of vaccine against El Tor vibrio is great a. E. coli b. Klebsiella
5. Transport medium for cholera: c. Vibrio cholerae d. Pseudomonas
 (Recent Pattern Dec 2013) 17. Which of the following about cholera is true:
a. LJ medium b. Cary-Blair medium a. Invasive  (Recent Pattern Dec 2014)
c. MYPA medium d. Stuart’s medium b. Endotoxin is released
6. Optimal percentage of NaCl for V cholerae:  c. Recent infections in India are of classical type
 (Recent Pattern Dec 2015) d. Vibriocidal antibody titre measures prevalence
a. 1% b. 2% 18. All of the following statements about El Tor vibrios are
c. 3% d. 4% true, except:  (All India 2010)
7. Which organism grows in alkaline pH?  a. Humans are the only reservoir
 (Recent Pattern Dec 2014) b. Can survive in ice cold water for 2 – 4 weeks
a. Vibrio b. Klebsiella c. Killed by boiling for 30 seconds
c. Pseudomonas d. E. coli d. Enterotoxin can have direct effects on other tissues
8. Cholera toxin is due to:  (Recent Pattern Dec 2012) besides intestinal Epithelial cells
a. Chromosome b. Plasmid 19. Invasive infection is caused by all except: 
c. Phage d. Transposons  (Recent Pattern Dec 2012)
9. Which of the following stimulate adenylate cyclase with a. V. cholerae b. Neisseria
G – protein coupled action:  (Recent Pattern Dec 2013) c. Streptococci d. H. influenzae
a. Shiga toxin b. Cholera toxin 20. “Darting motility” is shown by: 
c. Diphtheria toxin d. Pseudomonas toxin  (Recent Pattern Dec 2013)
10. Cholera toxin effects are mediated by stimulation of a. Proteus b. Vibrio
which of the following second messengers:  c. Serratia d. E. coli
 (Recent Pattern 2012) 21. Halophilic vibrio which causes wound infection at sea
a. cAMP b. cGMP coast is: (Recent Pattern July 2016)
c. Ca ++ – calmodulin d. IP3 / DAG a. Vibrio vulnificus
11. In the small intestine, cholera toxin acts by:  b. Vibrio parahaemolyticus
 (Recent Pattern Nov 2014) c. Vibrio mimicus
a. ADP ribosylation of the G regulatory protein d. Vibrio cholerae
b. Inhibition of adenyl cyclase 22. Which of the following Vibrios is most commonly
c. Activation of GTPase associated with ear infections:  (Recent Pattern 2012)
d. Active absorption of NaCl a. V. alginolyticus b. V. parahaemolyticus
12. Vibrio cholerae acts by disrupting which of the following c. V. vulnifcus d. V. fluvialis
structures:  (AIIMS May 2015) 23. True about vibrio vulnificus:  (Recent Pattern Dec 2015)
a. Hemidesmosome a. Causes diarrhea commonly
b. Gap junctions b. Halophilic
c. Zona occludens c. Drug of choice is penicillin
138
d. Zona adherens d. Produces Shiga toxin
24. Kanagawa phenomenon is seen in:

Chapter 14     Vibrio


27. At a birthday party, oysters, hamburger and potato salad
a. Pseudomonas aeruginosa  (Recent Pattern Dec 2015) were served. After 24 hours, 50 people had diarrhea.
b. Vibrio parahemolyticus Which is the most likely organism causing this type of
c. Shigella sonnei clinical presentation?  (AIIMS Nov 2018)
d. Proteus mirabilis a. Salmonella enteritidis b. Vibrio parahemolyticus
25. True about Vibrio parahemolyticus:  c. Yersinia enterocolitica d. Staphylococcus aureus
 (Recent Pattern Dec 2015) 28. 28. A 23-year-old man presented with an erythematous
a. Polar flagella b. Nonhalophilic vibrios edematous patch with centrally grouped vesicles on the
c. Non-capsulated d. Requires NaCl left ankle. The skin lesion occurred following a coral
injury while he was skin-scuba diving at an island 2
26. Which of the following halophilic vibrio: (PGI May 2018)
days prior to his presentation. Which is the causative
a. V. alginolyticus b. V. parahaemolyticus organism?  (JIPMER Nov 2018)
c. V. cholerae d. V. damsela a. Streptococcus pyogenes b. Vibrio vulnificus
e. V. vulnificus c. Alkaligenes faecalis d. Pseudomonas aeruginosa

ANSWERS AND EXPLANATIONS


1. Ans. (a) Vibrio cholerae – 01; (b) Vibrio cholerae – 5. Ans.  (b) Cary Blair medium
0139; (d) NAG vibrio
Ref: Ananthanarayan and Paniker’s Textbook of Microbiol-
Ref: Ananthanarayan and Paniker’s Textbook of Microbiol- ogy – 10th ed – Page 310
ogy – 10th ed – Page 312 • Holding or transport media used for Cholera vibrios

• Gardner and Venkatraman gave a serological


• are Venkatraman-Ramakrishnan medium, Cary-Blair
classification in which, cholera and biochemically medium and Autoclaved sea water.
similar vibrios possessing a common flagellar (H)
antigen were classified as Group A and rest as Group B 6. Ans.  (a) 1%
vibrios. This Group A has 139 serogroups, which have Ref: Ananthanarayan and Paniker’s Textbook of Microbiol-
been classified on the basis of ‘O’antigen. ogy – 10th ed – Page 310
• Earlier, only serogroup O-1 was isolated from epidemic
• NaCl (0.5-1%) is required for optimal growth of

cholera and was called pathogenic or agglutinable •

vibrios. But later in 1992, serogroup O-139 (non-O-1 V. cholerae, though high concentrations (6% and above)
vibrios) earlier called as non-pathogenic/NAG/non- are inhibitory.
agglutinable vibrios was identified as the causative
7. Ans.  (a) Vibrio
agent in epidemic.
Ref: Ananthanarayan and Paniker’s Textbook of Microbiol-
2. Ans.  (b) VP reaction (+) ve ogy – 10th ed – Page 310
Ref: Ananthanarayan and Paniker’s Textbook of Microbiol- • Growth of V. cholerae is better in an alkaline medium,

ogy – 10th ed – Page 312 (Table 32.4), Page 314 the range of pH being 6.4 - 9.6 (optimum 8.2).
• In case of El Tor vibrio, the severity of illness is much
8. Ans.  (c) Phage

less,with a large proportion of mild and asymptomatic


infections, mortality is low and the carrier rate is high. Ref: Ananthanarayan and Paniker’s Textbook of Microbiol-
• The El Tor vibrio is much harder than the classical
• ogy – 10th ed – Page 314
vibrios, capable of surviving in the environment for • Vibrios multiplying on the intestinal epithelium produce

much longer. an enterotoxin called Cholera toxin, the production of


which is determined by a filamentous phage integrated
3. Ans.  (c) Secondary attack rate is high in family
with the bacterial chromosome.
Ref: Ananthanarayan and Paniker’s Textbook of Microbiol-
ogy – 10th ed – Page 314 9. Ans.  (b) Cholera toxin

Ref: Already explained in Q.2 Ref: Ananthanarayan and Paniker’s Textbook of Microbiol-
ogy – 10th ed – Page 314
4. Ans.  (a) Animals are the only reservoir • Cholera toxin molecule of approx. 84000 MW consists of

1 A and 5 B subunits. The B (binding) units attach to GM1


Ref: Ananthanarayan and Paniker’s Textbook of Microbiol- ganglioside receptors on the surface of jejuna epithelial
ogy – 10th ed – Page 313 cells. A(active) fragments into A1 and A2. A2 binds A1 to B.
• Natural infection with cholera occurs only in humans. The A1 fragment causes prolonged activation of cellular
139

There are no animal reservoir.


adenylate cyclase and accumulation of cAMP, leading to 16. Ans.  (c) Vibrio cholerae
Unit 2     Bacteriology

outpouring of large quantities of water and electrolytes


Ref: Ananthanarayan and Paniker’s Textbook of Microbiol-
into the intestinal lumen. The toxin also inhibits
ogy – 10th ed – Page 314
intestinal absorption of sodium and chloride.
• Shiga toxin acts by inhibition of protein synthesis.

• Cholera vibrios also possess lipopolysaccharide O

• Diphtheria toxin inhibits polypeptide chain elongation



antigen (LPS, endotoxin), which apparently plays no
in the presence of nicotinamide adenine dinucleotide role in pathogenesis but is responsible for immunity
by inactivating the elongation factor EF-2. induced by killed vaccines.
• Pseudomonas produces Exotoxin A, the mechanism of
17. Ans.  (d) Vibriocidal antibody titre measures prevalence

which is similar to diphtheria toxin.


Ref: Ananthanarayan and Paniker’s Textbook of Microbiol-
10. Ans.  (a) cAMP ogy – 10th ed – Page 313
Ref: Ananthanarayan and Paniker’s Textbook of Microbiol- • In the small intestine, vibrios cross the protective layer

ogy – 10th ed – Page 314 of mucus and reach the epithelial cells, to which, they
• Already explained in Q.10

adhere and colonize with the help of special fimbria
called Toxin co-regulated pilus (TCP). They remain
11. Ans.  (a) ADP ribosylation of the G regulatory protein attached to the epithelium but do not damage or invade
the cells. The changes induced are biochemical rather
Ref: Ananthanarayan and Paniker’s Textbook of Microbiol-
than histological.
ogy – 10th ed – Page 314
• Cholera vibrios possess lipopolysaccharide endotoxin in

• Already explained in Q.10



their cell membrane. It is not released by the organism.
• Recent infections in India are of serovar O-139 type.
12. Ans.  (c) Zona occludens

• Serological testing is not used for diagnosis in cholera


Ref: Ananthanarayan and Paniker’s Textbook of Microbiol- cases rather they are helpful in assessing the prevalence.
ogy – 10th ed – Page 314 The tests available are agglutination using live or
killed vibrio suspensions, indirect hemagglutination,
• Vibrio cholera acts by disrupting the zona occludens.
vibriocidal test and antitoxin assay.

13. Ans.  (d) GMI gangliosides through B subunit


18. Ans.  (c) Killed by boiling for 30 seconds
Ref: Ananthanarayan and Paniker’s Textbook of Microbiol-
Ref: Ananthanarayan and Paniker’s Textbook of Microbiol-
ogy – 10th ed – Page 314
ogy – 10th ed – Page 314
• Already explained in Q.10
• Cholera toxin (Enterotoxin) exhibits other biological

14. Ans.  (a) Heat labile toxin effects besides effect on intestinal epithelial cells like
activation of lipolysis in rat testicular tissue, elongation
Ref: Ananthanarayan and Paniker’s Textbook of Microbiol- of Chinese hamster ovary cells in culture, histological
ogy – 10th ed – Page 313 changes in adrenal tumor cell culture and vero cells. It
• Cholera toxin (CT) is very similar to heat labile toxin

is called permeability factor because it increases skin
(LT) of E. coli in structural, chemical, biological and capillary permeability demonstrated by skin blueing
antigenic properties, though CT is far more potent than test.
LT in biological activity. • Humans are the only reservoir.

• Samples stored in cold may harbour vibrios for more


15. Ans.  (d) It is an achlorhydria which renders and than 2 weeks.


individual susceptible to disease
19. Ans.  (a) V. cholerae
Ref: Ananthanarayan and Paniker’s Textbook of Microbiol-
ogy – 10th ed – Page 313 Ref: Ananthanarayan and Paniker’s Textbook of Microbiol-
• Vibrio cholera is a gram-negative curved rod.

ogy – 10th ed – Page 313
• Cholera is an acute diarrheal disease presenting with

• Already explained in Q.18

profuse, painless, watery diarrhea and copious effortless


vomiting, which may lead to hypovolemic shock and 20. Ans.  (b) Vibrio
death in less than 24hrs. Its not associated with fever. Ref: Ananthanarayan and Paniker’s Textbook of Microbiol-
The stool has characteristic inoffensive sweetish odour, ogy – 10th ed – Page 309
termed as rice water stools.
• Vibrio cholera are actively motile (swarm of gnats) with
• Vibrios are highly susceptible to acids and gastric

a single sheathed polar flagellum. Motility is of darting


acidity provides an effective barrier against small doses


type and can be seen when an acute cholera stool or a
of cholera vibrios. Hence, achlorhydria predisposes to
young culture is examined under microscope.
cholera.
140
21. Ans.  (a) Vibrio vulnificus 25. Ans.  (d) Requires NaCl

Chapter 14     Vibrio


Ref: Ananthanarayan and Paniker’s Textbook of Microbiol- Ref: Ananthanarayan and Paniker’s Textbook of Microbiol-
ogy – 10th ed – Page 318 ogy – 10th ed – Page 317
• Vibrio vulnificus - halophilic, VP negative, ferments
• • Vibrio parahaemolyticus is a halophilic bacteria that

lactose but not sucrose, salt tolerance of <8%. grows on media containing NaCl up to 8% but 10%
• It causes two types of illnesses – (1) Wound
• concentration is inhibitory. Unlike other vibrios, it
infection following contact of open wounds with produces peritrichous flagella and are capsulated
sea water. (2) Septicemia with high mortality, when showing bipolar staining.
immunocompromised host ingests the vibrio through
oysters which penetrates the gut mucosa and enters 26. Ans.  (a,b,e) a. V. alginolyticus; b. V. parahaemolyticus;
bloodstream, without causing gastrointestinal e. V. vulnificus
manifestations. Ref: Ananthanarayan and Paniker’s Textbook of Microbiol-
ogy – 10th ed – Page 317
22. Ans.  (a) V. alginolyticus
Halophilic vibrios (Salt tolerant)
Ref: Ananthanarayan and Paniker’s Textbook of Microbiol- • Vibrio parahemolyticus

ogy – 10th ed – Page 318 • Vibrio vulnificus


• Vibrio alginolyticus is associated with infections of eyes,


• • Vibrio alginolyticus

ears and wounds in human beings exposed to sea water.


27. Ans.  (b) Vibrio parahemolyticus
23. Ans.  (b) Halophilic
Ref: Jawetz T.B of medical microbiology – 27th ed – page 256
Ref: Ananthanarayan and Paniker’s Textbook of Microbiol- • Whenever a case of food poisoning is given: two findings

ogy – 10th ed – Page 318 are important: (1) Food type (2) Incubation period; Here
• Already explained in Q.22
• sea food clearly gives a clue that halophilic vibrios (salt-
loving) – Vibrio parahemolyticus is the answer;
24. Ans.  (b) Vibrio parahemolyticus • Vibrio parahaemolyticus is a halophilic bacterium

Ref: Ananthanarayan and Paniker’s Textbook of Microbiol- that causes acute gastroenteritis after ingestion of
ogy – 10th ed – Page 318 contaminated seafood such as raw fish or shellfish.
After an incubation period of 12–24 hours, nausea
• Strains of Vibrio parahaemolyticus isolated from

and vomiting, abdominal cramps, fever, and watery to


environmental sources (water, fish, crabs or oysters) are bloody diarrhea occur
nearly always non-hemolytic on high salt blood agar
(Wagatsuma agar), while strains from human patients 28. Ans.  (b) Vibrio vulnificus
are almost always hemolytic due to presence of heat
stable hemolysin. This is called Kanagawa phenomenon Ref: Q.21 explanation
and in laboratories, if strains isolated are Kanagawa
positive, are inferred to be pathogenic to humans.

141
15 Pseudomonas, Acinetobacter
and Burkholderia
PSEUDOMONAS AERUGINOSA y Lipopolysaccharide: Produces endotoxin, sepsis syn-

y
drome— fever, shock, oliguria, DIC, leucopenia and metabol-
• Gram-negative motile bacilli ic abnormalities
y Exotoxin A: Causes tissue destruction by inhibiting protein

• It produces mucoid slime layer

y

• Obligate aerobe; colonies have a gun-metal appearance and synthesis, interrupts cell activity and macrophage response.
y Enterotoxin: Interrupts normal GI activity leading to diar-

gives a fruity odor

y
• It is a nonlactose fermenter; oxidase positive rhea.
y Exoenzyme S: Inhibits protein synthesis.

• Has characteristic pigment production

y
y Phospholipase C: Destroys cytoplasmic membrane and pul-

y
y Pseudomonas are Gram-negative straight or slightly curved monary surfactant, inactivates opsonins.
y
rods arranged singly, in small bundles or short chains, non- y Elastase: Cleaves immunoglobulins and compliment com-
y
sporing, non-acid-fast, strict aerobes, motile by polar flagella ponents, disrupts neutrophil activity
y Pigment is of many types: y Leukocidin: Inhibits neutrophil and lymphocyte function
y
y
 Pyocyanin: Demonstration of the presence of blue y Pyocyanins: Suppress other bacteria and disrupt respiratory
y

phenazine pigment is absolute confirmation of ciliary activity, causes oxidative damage to the tissues.
P. aeruginosa and is the major diagnostic test.
 Pyoverdin: The yellow/green pigment produced by most
Clinical Manifestations

strains giving the characteristic blue-green appearance of y Pyogenic infections (Blue pus)
y
infected pus or cultures. (Blue Pus) y Urinary tract infection (UTI)
y
 Pyocyanin, pyoverdin pyorubin easily identified on y Ecthyma gangrenosum
y

nutrient or sensitivity test agars and pyomelanin requires y Infection in burns patients
y
growth medium containing 1% tyrosine. y Infection in neutropenic patients
y
y Chronic P. aeruginosa infection in cystic fibrosis patients
y
Table 1: Characteristics of pigments y ICU/ nosocomial infections
y

y Acute pneumonia
y
Pyocyanin (M/c) Most important pigment
Green color Laboratory Diagnosis
Pyoverdin Greenish yellow y P. aeruginosa grows readily on ordinary media; P. aeruginosa
y
strains exhibit a moth-eaten type of colonial lysis with metallic
Pyorubin Red color
sheen known as iridescence
Pyomelanin Black color y Special media: Cetrimide agar.
y
y Demonstration of pigment production (10% do not produce
y P. aeruginosa is mostly found in moist environments: It is
y
pigment)
y
ubiquitously seen everywhere in hospital thus it is a major
y Biochemical reactions like oxidate test, OF test.
cause of nosocomial infections
y
y It can even grow in Dettol or cetrimide; it is highly resistant Minimum requirements for definitive identification of
y
to commonly used disinfectants. P. aeruginosa are:
• Gram-negative rod

Virulence Factors • Oxidase-positive

• Typical smell (fruity grape-like odor or corn tortilla)
y Alginate: Capsular polysaccharide allows the infecting

• Recognizable colony morphology
y
bacteria to adhere to lung epithelial cell surfaces and form

ƒ On blood or chocolate agar appears as large colonies with
biofilms and protect the bacteria from antibiotics and host
ƒ
metallic sheen, mucoid, rough, or pigmented (pyocyanin)
immune system
and often β-hemolytic
y Pili: Surface appendages that allows adherence of the
y
organism to GM-1 ganglioside receptoron host epithelial
surfaces. Remember
y Neuraminidase: Facilitating binding of pili by removing sialic • On MacConkey, appears as lactose-nonfermenting with

y
acid residues from GM-1 ganglioside receptors green pigmentation or metallic sheen
y Capsule: The presence of polysaccharide capsule protects it

Chapter 15     Pseudomonas, Acinetobacter and Burkholderia


y

against phagocytosis
y Fimbriae: Fimbriae facilitate the adhesion to the human
y

epithelial cells
y Protein S layers and Slime also potentially enhance the
y

virulence of the organism


y Certain strains of Acinetobacter have been shown to produce
y

siderophores and iron-repressible outer membrane receptor


proteins
y Enzymes: Enzymes butyrate esterase, caprylate esterase and
y

leucine arylamidase potentially involved in damaging tissue


lipids
y Bacteriocin production may enhance the survival of
y

Acinetobacter.
y Biofilm: The excess polysaccharide formation in A.
y

Figure 1: Greenish pigmented colonies of Pseudomonas baumannii leads to difficulty in antibiotic penetration and
(Courtesy: Dr.M.Gomathi, Chengalpattu Medical College, Tamil Nadu) the differences in cell physiology in biofilm increases the drug
resistance.

Treatment
y Treatment varies depending upon neutropenic or nonneu-
y
Clinical Features
tropenic host It can cause:
 Neutropenic host: Cefepime

y Pneumonia especially in mechanically ventilated patients
y

 Nonneutropenic host: Ceftazidime or cefepime plus amik-



(VAP)
acin (or) Piptaz, IMP, MRP or doripenam y Blood stream infections
y

 MDR Pseudomonas (but needs to get confirmed with AST



y Wound infections
y

testing) – Colistin is the DOC. y Meningitis


y

y UTI
y

ACINETOBACTER
Treatment
• • Gram-negative bacilli y Selection of antimicrobials for Acinetobacter is challenging –
y

• • Non-motile as it has intrinsic and acquired resistance genes


• • Oxidase negative y TMP-SMX and amikacin can be given
y

• • Nonfermenter y Meropenam in sensitive cases


y

• • Produces characteristic pale pink colonies in MacConkey agar y If carbapenam resistance strains occur then treatment is
y

tigecycline and colistin


y Acinetobacter is strict aerobic, Gram-negative coccobacilli
y

rods, widely distributed in nature and hospital environments


y Gram negative rods or coccobacilli, catalase positive, oxidase
y
High Yield
negative, nonmotile, nonsporing and may be capsulated. Drug resistance in Pseudomonas and Acinetobacter:
y Single most species A. baumannii is the leading cause of
y • Multidrug Resistant (MDR): The isolate resistant to at least

nosocomial infections nowadays three classes of antimicrobial agents including all penicillin,
y A. baumannii has the characteristic to get acquired resistance
y cephalosporins, fluoroquinolones and aminoglycosides.
determinants easily • Extensive Drug Resistant (XDR): The isolate will be resistant to

y Acinetobacter species is ubiquitously found in water, soil and


y carbapenems in addition to the above-mentioned drugs.
in environment • Pan Drug Resistant (PDR): The isolate will be resistant to all

y Fecal carriage is also seen in healthy and hospitalized indi-


y the available drugs, including polymyxins and tigecycline.
viduals
y The most common cause of nosocomial infection is
BURKHOLDERIA
y

colonization of organism leading to spread of infection


y These organisms have high rate of colonization of trachea.
y

Respiratory tract is the most common site for A. baumanii


Burkholderia Cepacia
infections in ICU with a mortality rate approaching 70%. y It causes rapidly fatal syndrome called as cepacia syndrome
y

— respiratory distress and septicemia occurs


y It colonizes in lung and causes chronic granulomatous
y

Virulence Factors disease (cystic fibrosis like condition)


y LPS is most potent and causes inflammation of lungs
y Lipopolysaccharide: Because of lipopolysaccharide, Acine-
y

y It also causes VAP, catheter–associated infections and wound


y

tobacter O antigen display a marked hydrophobicity with the


y

infection.
ability to grow on hydrophobic substrates. 143
y Colonies are smooth and glistening, forming non-lactose STENOTROPHOMONAS MALTOPHILIA
Unit 2     Bacteriology

fermenting colonies on MacConkey agar and yellow


pigmented colonies on blood agar. y Originally classified as Pseudomonas maltophilia, it is an
y

y It is weakly oxidase positive, catalase positive, motile, oxidizes


y obligate aerobe and an important nosocomial pathogen
all sugars, decarboxylates lysine, ONPG negative, acetamide associated with substantial morbidity and mortality in
negative and resistant to Polymixin B. immunosuppressed patient in intensive care unit and
y It is intrinsically resistant to many antibiotics; inherent
y pulmonary source of the isolate.
resistant to polymyxin B is used in diagnosis y The most common site for recovery of Stenotrophomonas
y

y DOC: TMP-SMX, Meropenem and Doxycycline.


y maltophilia is the respiratory tract.
y It is one among the most common causes of wound infections
y

due to trauma.
Burkholderia Pseudomallei y It is frequently isolated from patients with ventilator support
y

y Gram negative, motile with safety pin appearance


y
in ICU.
y Causative agent for melioidosis
y
y It is an important pathogen in cystic fibrosis patients
y

y Humans get infection by inoculation, inhalation and ingestion


y
y It is also isolated from wound infections, bacteremia,
y

y Most virulent among pseudomonads


y
pneumonia, endocarditis, urinary tract infections, meningitis
y Infection is more common in diabetics and renal disease
y
and peritonitis
patients y S. maltophilia is a motile rod, with polar multitrichous flagella
y

y It causes pneumonia, septicemia, acute pulmonary infection


y
and distinguished from pseudomonads by oxidase negative.
mimicking TB y S. maltophilia is susceptible to colistin and polymyxin. This
y

y DOC: Ceftazidime followed by carbapenem or TMP-SMX


y
property is used to distinguish it from B. cepacia.
y The antibiotic susceptibility pattern can be a clue to the
Burkholderia Mallei
y

identification of S. maltophilia. The most active agents are


y Gram negative, nonmotile
y
trimethoprim sulphamethoxazole, colistin and quinolones.
y Causative agent for disease called glanders (equine disease)
y
y Like other nonfermenters it is intrinsically resistant to many
y

y Occupational disease for veterinarians and horse handlers


y
common antibiotics like aminoglycosides, carbapenems and
y Polysaccharide capsule is the virulence factor
y
many betalactam agents.
y Organism transmitted from animals to humans by inoculation
y

y Regional lymphadenopathy occurs leading to febrile illness


y

and septicemia
y DOC: Cephalosporins, Carbapenems and Macrolides
y

144
MULTIPLE CHOICE QUESTIONS

Chapter 15     Pseudomonas, Acinetobacter and Burkholderia


1. Pseudomonas is which type of bacteria:  8. Glander’s disease is caused by: 
 (Recent Pattern Dec 2013)  (Recent Pattern Dec 2014)
a. Anaerobic b. Microaerophilic a. Burkholderia mallei
c. Strict aerobe d. Obligate anaerobe b. Diphtheria
2. An organism grown on agar shows green colored colo- c. Mumps
nies, likely organism is: (Recent Pattern Nov 2015) d. Burkholderia pseudomallei
a. Staphylococcus b. Streptococcus 9. Which of the following causes Meliodosis? 
c. Pseudomonas d. E.coli a. Burkholderia mallei  (Recent Pattern Dec 2014)
3. Which of the following is incubated at 40 – 44 degrees? b. Burkholderia pseudomallei
 (Recent Pattern Dec 2015) c. Burkholderia cepacia
a. Vibrio cholerae d. None
b. Pseudomonas aeruginosa 10. Which of the following is nonmotile: 
c. Vibrio parahaemolyticus a. Pseudomonas aeruginosa (Recent Pattern Dec 2016)


d. E. coli b. Burkholderia mallei


4. Bacteria that can grow even in the presence of antiseptic: c. Burkholderia pseudomallei
 (Recent Pattern Dec 2016) d. None
a. Staphylococcus b. Streptococcus 11. Causative agent for meliodosis is: 
c. E. coli d. Pseudomonas  (Recent Pattern Dec 2013)
5. Most common organism associated with cystic fibrosis: a. Pseudomonas pseudomallei
 (Recent Pattern June 2014) b. Pseudomonas aureginosa
a. Pseudomonas mucoid c. Pseudomonas capecia
b. Pseudomonas non mucoid d. Pseudomonas mallei
c. Listeria 12. Burkholderia cepacia is intrinsically resistant to:
d. Neisseria  (Recent Pattern 2018)
6. Pseudomonas exhibits protein synthesis by inhibiting: a. Ciprofloxacin
 (Recent Pattern Dec 2015) b. Ceftriaxone
a. RNA polymerase b. EF -2 c. Polymyxin B
c. Transpeptidase d. Reverse transcriptase d. Ampicillin
7. A cystic fibrosis patient presented with an episode of 13. Organism(s) commonly causing infection in cystic
pneumonia. On sputum culture, mucoid colonies of fibrosis patients: (PGI May 2017)
pseudomonas were seen. What does this indicate? a. Burkholderia cepacia
 (AIIMS Nov 2014) b. Pseudomonas aeruginosa
a. It formed a biofilm on bronchial walls c. Staphylococcus aureus
b. It underwent a mutation d. Burkholderia pseudomallei
c. It is resistant to most of the antibiotic e. Streptococcus pyogenes
d. There is a mistake with the culture technique

ANSWERS AND EXPLANATIONS


1. Ans.  (c) Strict aerobe 3. Ans.  (b) Pseudomonas aeruginosa
Ref: Ananthanarayan and Paniker’s Textbook of Microbiol- Ref: Ananthanarayan and Paniker’s Textbook of Microbiol-
ogy – 10th ed – Page 320 ogy – 10th ed – Page 320
• Pseudomonas is an obligate aerobe which when grown
• • P. aeruginosa grows in a wide range of temperature,

in broth forms a surface pellicle and dense turbidity. It 6-42°C, optimum being 37°C.
shows oxidative metabolism and is oxidase positive.
4. Ans.  (d) Pseudomonas
2. Ans.  (c) Pseudomonas
Ref: Ananthanarayan and Paniker’s Textbook of Microbiol-
Ref: Ananthanarayan and Paniker’s Textbook of Microbiol- ogy – 10th ed – Page 321
ogy – 10th ed – Page 320 • Pseudomonas shows high degree of resistance to

• P. aeruginosa produces a number of pigments:


• chemicals agents like common antiseptics and
ƒ Pyocyanin: Bluish green
ƒ disinfectants. E.g. Quarternary ammonium compounds,
ƒ Fluorescein (Pyoverdin): Greenish yellow
ƒ chloroxylenol and hexachlorophene. It can grow
ƒ Pyorubin: Red
ƒ profusely in Dettol bottles kept for use in hospitals.
ƒ Pyomelanin: Brown
ƒ

145
5. Ans.  (a) Pseudomonas mucoid 9. Ans.  (b) Burkholderia pseudomallei
Unit 2     Bacteriology

Ref: Ananthanarayan and Paniker’s Textbook of Microbiol- Ref: Ananthanarayan and Paniker’s Textbook of Microbiol-
ogy – 10th ed – Page 320 ogy – 10th ed – Page 323
• The two most common infections caused by
• • Melioidosis, is a glander’s like disease caused by Burk-

Pseudomonas in community are suppurative otitis holderia pseudomallei (formerly called, Pseudomonas
and respiratory tract infections in cystic fibrosis pseudomallei) which resembles B. mallei, but differs in
patients. The strain responsible for the latter, are being motile.
usually mucoid in nature, which have abundance of
extracellular polysaccharides composed of alginate 10. Ans.  (b) Burkholderia mallei
polymers. This forms a loose capsule (glycocalyx) in Ref: Ananthanarayan and Paniker’s Textbook of Microbiol-
which microcolonies of the bacillus are enmeshed and ogy – 10th ed – Page 323
protected from host defences. In other words, there is
biofilm formation on the bronchial walls. • Already explained in Q.8

6. Ans.  (b) EF -2 11. Ans.  (a) Pseudomonas pseudomallei

Ref: Ananthanarayan and Paniker’s Textbook of Microbiol- Ref: Ananthanarayan and Paniker’s Textbook of Microbiol-
ogy – 10th ed – Page 322 ogy – 10th ed – Page 323

• Exotoxin A produced by Pseudomonas acts by inhibiting



• Already explained in Q.9

protein synthesis similar to diphtheria toxin. The active


12. Ans.  (c) Polymyxin B
fragment inhibits polypeptide chain elongation in the
presence of NAD by inactivating the elongation factor Ref: T.B of diagnostic microbiology – Connie R Mahon –
EF-2. Page 492
• Burkholderia cepacia is intrinsically resistant to amino-
7. Ans.  (a) It formed a biofilm on bronchial walls

glycosides and polymyxin


Ref: Ananthanarayan and Paniker’s Textbook of Microbiol- • 100% resistant to polymyxin B helps in diagnosis

ogy – 10th ed – Page 320 • In addition many strains have acquired resistance to

• Already explained in Q.5 almost all groups of antibiotics


• Hence for treatment: Ideally disk diffusion test with only

8. Ans.  (a) Burkholderia mallei the following drugs are used:


ƒ Ceftazidime
ƒ

Ref: Ananthanarayan and Paniker’s Textbook of Microbiol- ƒ Meropenam


ƒ

ogy – 10th ed – Page 323 ƒ Minocycline


ƒ

• Glander’s is a disease primarily of equine animals but


• ƒ TMP-SMX
ƒ

capable of being transmitted to other animals and


humans. This is caused by Burkholderia mallei, which 13. Ans.  (a) Burkholderia cepacia; (b) Pseudomonas
is an aerobe, facultative anaerobe, nonmotile, Gram aeruginosa; (c) Staphylococcus aureus
negative bacillus. Ref: Ananthanarayan and Paniker’s Textbook of Microbiol-
ogy – 10th ed 322
• Chronic P. aeruginosa infection in cystic fibrosis patients

is the most common organism is CF patients;


• Next in order are Burkholderia and Staph. aureus

146
Haemophilus, Francisella 16
and Pasteurella
HAEMOPHILUS y Capsular polysaccharide is the most important virulence

y
factor
Haemophilus Influenzae y Based on the capsule: Pitman classified H. influenzae into six

y
y Small, Gram negative, nonmotile bacilli types: a to f
y
y It has characteristic growth requirements y Type b H. influenzae is unique in chemical structure by

y
y
y It needs two accessory growth factors called X and V having pentose sugars while others have hexose sugars.
y
y It is aerobic and grows only in chocolate agar. y Hib antigen induces IgM, IgG and IgA antibodies in human

y
y
body; hence this antigen is being used for vaccination.
Table 1:  Differences between factor V and factor X y Other strains who donot have any capsule antigens are called

y
as nontypable strains
FACTOR V FACTOR X
It is not clotting factor five It is not factor ten Pathogenicity
Factor V – codes for NAD Factor X codes for hemin Table 2:  Invasive and noninvasive infections
Heat labile Heat stable
Invasive infections Noninvasive infections
• H. influenzae acts a primary • Infections are caused
Satellitism


pathogen and causes secondary to some other
y H. influenzae depends on factor V and X for its growth invasive infections infections
y
y When S. aureus is streaked across a plate in a straight line on • Mostly seen in children • Usually seen in adults
y


blood agar that was inoculated with H. influenzae culture or • Caused by capsulated strains • Caused by noncapsulated or


specimen – colonies grow well near the midline and fades • The most important is nontypable strains

toward the periphery – due to availability of factor V that is meningitis • Causes otitis media,

produced in staphylococcus growth • It also causes laryngo sinusitis, exacerbations of

y This phenomenon is called a satellitism. epiglottitis, conjunctivitis, chronic bronchitis
y
bacteremia, pneumonia,
arthritis, endocarditis and
pericarditis

Clinical Features
y Meningitis is the most-common manifestation seen in
y
children less than 2 years of age
y Fevers and altered CNS manifestations are seen
y
y The most common complication is subdural effusion.
y
y Epiglottitis is a life-threatening infections that is caused in 2–7
y
years old
y Cellulitis is also caused especially in head and neck
y
y It causes pneumonia in infants.
y
Figure 1: Satellitism (Courtesy: CDC/ Dr. Mike Miller) Table 3:  Diseases caused by H. influenzae in specific age
group
Antigenic Nature H. influenzae in age group Disease
y Three major antigens are:
Infants Pneumonia
y
 Capsular polysaccharide
< 2years Meningitis

 Outer membrane proteins

 Lipooligosaccharides 2-7 years Epiglottitis

Laboratory Diagnosis Treatment
Unit 2     Bacteriology

y Specimens are usually CSF in meningitis and sputum in


y y Any one can be taken from the following drugs:
y

pneumonia  Single dose of 1 g of azithromycin




y Culture is done in:


y  Ceftriaxone IM single dose


 Chocolate agar
  Ciprofloxacin bd for 3 days


 Satellitism demonstration in blood agar




 Levinthal’s medium shows clear transparent medium with




Table 4: Important points in all species of Haemophilus


translucent colonies
 Filde’s peptic digest agar

Haemophilus • Gram negative, capsulated, pleomorphic

y Serological tests: Latex agglutination test in CSF and urine.


y
influenzae bacilli
(Pfeiffer’s • Required factor X and V for growth

bacillus) • Factor X – hemin/porphyrin


Treatment

• Factor V – NAD/NADP

y DOC: Ceftriaxone or cefotaxime


y
• Does not grow on blood agar; grows only

y Administration of glucocorticoids to patients with Hib


y
in chocolate agar
meningitis reduces the incidence of neurological sequelae • Culture media – Levinthal’s medium

y Immunoprophylaxis:
y
• Satellitism – due to high concentration of

 Two conjugate vaccines are available



factor V in Staph aureus – released in the
Š All children should be immunized with Hib conjugate
Š
medium – helps in the growth of
with first dose within 2 months of age (HibPRP vaccine) H. influenzae
Š There is no vaccine available for nontypable strains
Š
• Major virulence factor – Capsulated

Š Chemoprophylaxis – for contacts - Rifampicin


Š
polysaccharide
• Invasive – meningitis – Most commonly by
Remember

capsulated stains of group b – (Hib) – in <2


Childhood otitis media: years of age
• Nontypable H. influeanzae (Most common)

• Most common cause of acute epiglottitis

• Streptococcus pneumoniae

• Noninvasive – Otitis media and sinusitis –

• Moraxella catarrhalis

caused by noncapsulated – nontypeable
strains
Haemophilus Ducreyi
Haemophilus • Requires only Factor V
y It is the causative agent for chancroid

parainfluenzae
y

y y Gram-negative bacilli that needs only Factor X


y y Infection is acquired through sexual route Haemophilus • Also knows as Koch’s Weeks bacillus

y y Incubation period is 4–7 days aegyptius • Causes Brazilian purpuric fever


y y An initial papular lesion develops which then becomes • Pink eye – Conjunctivitis

pustule and ruptures formed ulcers; these ulcers are painful


Haemophilus • Bipolar staining
Remember

ducreyi • School of fish / Rail road track appearance


Painful ulcers – Chancroid Haemophilus ducreyi • Chocolate agar enriched with isovitale X

and fetal calf serum with Vancomycin


Painless ulcers - Chancre Treponema pallidum • Causes Chancroid or soft sore – painful

and non indurated


• DOC – single dose azithromycin
High Yield

• Probable diagnosis of chancroid are made under following


Table 5: Growth factors in Haemophilus species


conditions:

ƒ One or more painful genital ulcers


ƒ

Species Growth factors


ƒ No evidence of Treponema pallidum as seen by dark field
Haemophilus influenzae, H. aegyptius, Factor X and V
ƒ

examination
ƒ No evidence of syphilis done by serological tests (7 days
ƒ
H. haemolyticus
after ulcer onset)
ƒ Clinical evidence of chancroid Haemophilus parainfluenzae, Factor V
H. parahemolyticus, H. paraphrophilus
ƒ

ƒ Negative test for Herpes


ƒ

H. ducreyi, H. aphrophilus Factor X

148
FRANCISELLA TULARENSIS y Lab diagnosis is mainly by agglutination testing. Microagglu-

Chapter 16     Haemophilus, Francisella and Pasteurella


y

tination and tube agglutination tests are used.


y Gram-negative bacilli, pleomorphic with bipolar staining
y Treatment: Aminoglycosides, tetracyclines, rifampin
y

y It is a class A bioterrorism agent


y

y It is a zoonotic infectious agent causing tularemia


PASTEURELLA MULTOCIDA
y

y Vectors: Ticks
y

y Inhalation or ingestion also causes infection


y

y Gram-negative, non-motile coccobacilli


y

y After inoculation into the skin, it multiplies locally and then


y

y Bipolar staining
y

produces a papule. It forms an ulcer with a black base and y Oxidase and catalase positive
y

regional lymphadenopathy; it leads to bacteremia and infects y Most common organism in human wounds inflicted by cat
y

CNS also. bite


y It can present in different forms:
y

y A history of animal bite followed by redness, swelling and


y

 Ulceroglandular


pain suggests Pasteurella infection


 Glandular


y It is susceptible to most antibiotics


y

 Pulmonary


y DOC is penicllin G; alternative drugs are tetracyclines and


y

 Oropharyngeal


FQs.
 Typhoidal


149
MULTIPLE CHOICE QUESTIONS
Unit 2     Bacteriology

1. About H. influenza all true, except:  (AIIMS Nov 2010) 9. Chancroid is caused by:  (Recent Pattern Dec 2014)
a. Requires factor X and V for growth a. H. ducreyi b. T. pallidium
b. Rarely presents as meningitis in children less than 2 c. Gonococcus d. HSV
months of age 10. School of fish appearance is characteristic of: 
c. Capsular polypeptide protein is responsible for  (Recent Pattern July 2016)
virulence a. Bordetella pertussis
d. M.C invasive disease of H. influenza is meningitis b. Yersinia enterocolitica
2. The major antigenic determinant of H. influenza is: c. Haemophilus ducreyi
 (Recent Pattern Dec 2014) d. Legionella
a. M protein 11. Pasteurella multocida is transferred by: 
b. Capsular polysaccharide  (Recent Pattern June 2014)
c. Catalase a. Aerosols b. Animals
d. Coagulase c. Sexual d. Transplacental
3. Agar media used for H. influenza - 12. Most common mode of transmission of Pasturella mul-
 (Recent Pattern Dec 2015) tocida is:  (Recent Pattern Dec 2016)
a. Blood agar b. Chocolate agar a. Animal bites or scratches
c. Tryptose agar d. BYCE agar b. Aerosols or dust
4. Satellitism is seen in cultures of: c. Contaminated tissue
 (Recent Pattern Dec 2016) d. Human to human
a. Hemophilus b. Streptococcus 13. True statement about genital ulcer: (PGI Nov 2017)
c. Klebsiella d. Proteus a. Donovanosis- Painful and multiple lesion
5. Not caused by non typable Hib :  (AIIMS Nov 2014) b. Granuloma inguinale is donovanosis
a. Meningitis b. Otitis media c. Haemophilus ducreyi causes chancroid
c. Puerperal sepsis d. Exacerbation of COPD d. Systemic feature common in primary syphilis and
6. True about H. influenza: (Recent Pattern Dec 2012) H. ducreyi infection
a. Grown on sheep blood agar and CO2 e. Primary syphilis-Painful and indurated papule
b. It is not capsulated 14. A patient is presenting with multiple painful ulcers and
c. Invasive strain is most common discharging inguinal lymphadenopathy. Which of the
d. Gram positive following finding is suggestive of this disease:
7. Brazilian purpuric fever is caused by:   (PGI Nov 2017)
 (Recent Pattern Dec 2016) a. Demonstration of school of fish appearance of bacilli
a. Bordetella pertussis b. Culture of H. ducreyi from an aspirate of suppurative
b. Hemophilus aegypticus lymph nodes
c. Hemophilus ducreyi c. Microscopic identification of Donovan bodies in tissue
d. Hemophilus parainfluenzae smear
8. Which of the following hemophilus doesn’t require Fac- d. Evidence of Treponema pallidum infection by dark-
tor V: (Recent Pattern Dec 2015) field examination of ulcer exudate
a. H. parainfluenzae b. H. influenza e. Herpes simplex virus in the ulcer exudate
c. H. ducreyi d. H. aegypticus

ANSWERS AND EXPLANATIONS


1. Ans.  (c) Capsular polypeptide protein is responsible for 2. Ans.  (b) Capsular polysaccharide
virulence
Ref: Ananthanarayan and Paniker’s Textbook of Microbiol-
Ref: Ananthanarayan and Paniker’s Textbook of Microbiol- ogy – 10th ed – Page 334
ogy – 10th ed – Page 334 • Already explained in Q.1

• There are three main surface antigens: (a) The major


antigenic determinant of capsulated strains is the 3. Ans.  (b) Chocolate agar


capsular polysaccharides based on which H. influenzae Ref: Ananthanarayan and Paniker’s Textbook of Microbiol-
strains have been classified into six capsular types – a ogy – 10th ed – Page 336
to f. Type b contains pentose sugars ribose and ribitol
instead of hexoses and hexosamines as in the other • Chocolate agar is made by heating blood agar due

five serotypes. Its called Polyribosylribitol phosphate to which the V factor is released from within the
antigen. (b) Outer membrane protein antigens. erythrocytes and it becomes the best agar media for
(c) Lipooligosaccharides (LOS). growing H.influenzae.
150
4. Ans.  (a) Hemophilus 9. Ans.  (a) H. ducreyi

Chapter 16     Haemophilus, Francisella and Pasteurella


Ref: Ananthanarayan and Paniker’s Textbook of Microbiol- Ref: Ananthanarayan and Paniker’s Textbook of Microbiol-
ogy – 10th ed – Page 334 ogy – 10th ed –Page 337
• Staphylococcus aureus synthesizes factor V in excess of
• • Chancroid or soft sore caused by H. ducreyi, is a disease

their requirements and is released into the surrounding characterized by tender nonindurated irregular ulcers
medium. Hence, when S. aureus is streaked across on the genitalia.
blood agar on which specimen containing H. influenzae
has been inoculated, the colonies of H. influenzae 10. Ans.  (c) Haemophilus ducreyi
will be large and well developed alongside the streak Ref: Ananthanarayan and Paniker’s Textbook of Microbiol-
and smaller farther away. This phenomenon is called ogy – 10th ed –Page 337
satellitism.
• On Gram staining, the H. ducreyi bacilli may be arranged

5. Ans.  (c) Puerperal sepsis in small groups or whorls or in parallel chains, giving a
‘school of fish’ or ‘rail road track’ appearance.
Ref: Ananthanarayan and Paniker’s Textbook of Microbiol-
ogy – 10th ed – Page 335 11. Ans.  (b) Animals
• The infections caused by H. influenzae are meningitis;

Ref: Ananthanarayan and Paniker’s Textbook of Microbiol-


laryngoepiglottitis (croup); pneumonia; suppurative ogy – 10th ed –Page 331
lesion like arthritis, endocarditis, pericarditis, otitis me-
dia; cellulitis; acute exacerbation of chronic bronchitis, • The bacilli are carried in upper respiratory tract of dogs,

bronchiectasis and COPD. cats, rats, cattle’s and sheep which may be transmitted to
humans by animal bites.
6. Ans.  (c) Invasive strain is most common
12. Ans.  (a) Animal bites or scratches
Ref: Ananthanarayan and Paniker’s Textbook of Microbiol-
ogy – 10th ed – Page 335 Ref: Ananthanarayan and Paniker’s Textbook of Microbiol-
ogy – 10th ed –Page 331
• H. influenzae is Gram-negative, nonmotile, nonsporing,

often capsulated bacteria. It is grown on chocolate agar


13. Ans.  (b) Granuloma inguinale is donovanosis;
and incubated in 5–10% CO2 at 37˚C.
(c) Haemophilus ducreyi causes chancroid
7. Ans.  (b) Hemophilus aegypticus Ref: Ananthanarayan and Paniker’s Textbook of Microbiol-
Ref: Ananthanarayan and Paniker’s Textbook of Microbiol- ogy – 10th ed –Page 404
ogy – 10th ed – Page 336 • Option A and B: Granuloma inguinale otherwise called

• Brazilian purpuric fever is caused by Haemophilus



as Donovanosis is caused by Klebsiella granulomatis; it
aegyptius in which conjunctivitis proceeds to fulminant causes painless papules
septicaemia in infants and children with high fatality. • Option C: Chancroid has painful lesions caused by

H.ducreyi
8. Ans.  (c) H. ducreyi • Option D: Systemic features are seen only in tertiary

syphilis
Ref: Ananthanarayan and Paniker’s Textbook of Microbiol- • Option E: Syphilis has painless lesions

ogy – 10th ed –Page 334 Table 35.1


14. Ans.  (a) Demonstration of school of fish appearance of
Growth requirements bacilli
Organism X V CO2
Ref: Ananthanarayan and Paniker’s Textbook of Microbiol-
H. influenzae + + _ ogy – 10th ed –Page 336
H. ducreyi + _ Variable • Multiple painful ulcers with inguinal nodes clearly sug-

gests chancroid
H. aegyptius + + _ • H. ducreyi is very difficult to isolate by culture; most

common diagnosis is direct staining and microscopic


identification

151
17
Brucella and Bordetella
BRUCELLA Lab Diagnosis
• Gram-negative coccobacilli In Humans

• Nonmotile, noncapsulated y Blood culture is the most definitive method of diagnosis

y

• Strict aerobes y All cultures should be incubated and observed for a period of

y

• Capnophilic (needs CO2 for growth) 3 weeks before declaring it as negative

y Castaneda’s bottle with both solid and liquid medium

y
y Four species of Brucella: B. abortus, B. suis, B. melitensis, increases the chance of isolation
y
B. canis y Serological test includes standard agglutination test (SAT)

y
y Brucellosis is a zoonotic disease acquired from animals and – in this test the blocking antibodies in the sera should be
y
animal products (milk) removed by prior treatment with saline or by heating. A titer of
y It is called mediterranean fever, malta fever, undulant 160 is considered as positive test; these blocking antibodies
y
fever, Bang’s disease and Gibraltar fever are of IgA type.
y It is an intracellular organism y ELISA tests to detect IgM and IgG available.
y
y
y Common route of infection is through infected milk or
y
contact with animals. The organism enters the lymphatic In Animals
channels and spread to bloodstream
y It leads to systemic infections y Milk ring test
y
y Rose Bengal card test
y
y Histological finding includes proliferation of mononuclear
y
y Rapid plate agglutination test
y
cells, exudation of fibrin, coagulation necrosis and fibrosis
y
Clinical Features Treatment
y Clinical presentation includes fever, weakness, malaise. y DOC includes tetracyclines, rifampin, TMP-SMX and amino-
y
glycosides
y
y Acute brucellosis usually caused by B. melitensis— fever
y Treatment should be given for at least 6–8 weeks.
y
for prolonged duration followed by irregular spikes is seen.
y
Associated features are arthralgia, nocturnal sweats, anorexia
and constipation. BORDETELLA
y Chronic brucellosis is a low grade infection that is present for
Bordetella Pertussis
y
years with malaise and arthralgia.
y Depending upon the species, clinical features differs: y Gram-negative coccobacilli, strict aerobe, capsulated, non-
y
y
motile
Table 1:  Presentation of various species of Brucella y Bipolar metachromatic granules are seen
y
y It gives characteristic thumb print appearance in culture
Species Presentation
y
films
B. abortus • Mild disease without suppurative y Media used for Culture : (not grown in basal media)
y

complications  Bordet Gengou glycerine potato blood agar medium

• Non caseating granulomas of the RES are  Regan Lowe medium


seen  Charcoal blood agar

y Growth in Bordet-Gengou medium gives certain characteristic
B. canis • Mild disease
y
appearances:

B. suis • Chronic suppurative disease  Confluent growth gives ‘Aluminium paint appearance’


B. melitensis • More acute disease and severe  Individual colonies looked like ‘bisected pearls’ or


‘mercury drops’
y IP varies from 1 to 4 weeks
y
Virulence Factors Treatment

Chapter 17     Brucella and Bordetella


y y Pertussis toxin y DOC: Erythromycin is given to reduce infectivity
y

y y Filamentous hemagglutinin
y y Pertactin Antimicrobial therapy for pertussis: (any one)
y y Adenylate cyclase • Erythromycin 1–2 g in three divided doses for 14 days

y Heat labile toxin


• Clarithromycin 500 mg in two divided doses for 7 days
y

y LPS

y y Piracy of adhesins • Azithromycin 500 mg day 1 followed by 250 mg for 5 days


• Cotrimaxozole in two divided doses for 14 days


Clinical Features
y IP is for 2 weeks; infection is initiated by attachment of
y

Immunoprophylaxis
organism in the ciliated epithelial cells of nasopharynx,
y Acellular pertussis vaccine: PT, FHA, agglutinogens (1,2,3),
y Three stages of disease as:
y

pertactin and fimbrial 2 and 3 antigens


y

 Catarrhal stage
y Whole cell pertussis vaccine have complications like


 Paroxysmal stage
y

postvaccinal encephalopathy especially after seven years




 Convalescent stage
y Both whole cell and acellular has 90% protection rate


y 100 days cough – Pertussis


y

y Children under four years who have contacts need to receive


y

y Most infective stage is catarrhal stage; secondary attack rate


y

a booster dose and chemoprophylaxis with erythromycin.


y

is 90%
y Characteristic whoop upon inhalation occurs in paroxysmal
Table 2:  Difference between whole cell and acellular
y

stage – hence named as whooping cough


y B. pertussis is a common cause of prolonged cough in
y
pertussis vaccine
adults 4–6 weeks Whole cell pertussis Acellular pertussis vaccine
y Post tussive vomiting occurs at the end of each episode
y

vaccine
y These classical symptoms are mostly seen in children; in
y

adults it is not so typical features for pertussis Part of DTP vaccine; Consists of purified bacterial
y Complications: Bronchiectasis, subconjunctival hemorrhage,
y
Merthiolate killed bacterial components; PT, FHA,
encephalopathy, ataxia; Secondary infections with S. pneu- suspension agglutinogens, pertactin and
moniae and H. influenzae is most commonly seen. fimbrial antigens; can be
combined with other vaccines
Stage of maximum infectivity Catarrhal stage (1st stage) More immunity production Less immunity protection
Stage of whooping cough Paroxysmal stage (2nd stage) than acellular
More side effects Eg: Less side effects
Lab Diagnosis Postvaccinal encephalopathy
y Nasopharyngeal swabs or aspirates are the ideal choice of
y
DTP vaccine DTaP vaccine
specimen
y Swabs should be taken with dacron or rayon tipped
y

Bordetella Parapertussis
y Cough plate method can also be employed to collect
y It may also produce whooping cough
y

secretions
y

y This can grow in basal media producing larger colonies


y Specimens should be cultured or done PCR to identify the
y

y It may produce pigment in culture media


y

agent
y

y Produce milder disease.


y Differential diagnosis: Mimicking the same presentation is
y

caused by adenoviruses and Mycoplasma (named as Pseu-


do whooping cough) Bordetella Bronchiseptica
y y Seen in respiratory tract of animals
y y Motile bacilli
y y It can cause chronic respiratory tract infections in humans
y y It has beta lactamase inherently that makes it resistant to
penicllins and cephalosporins.

153
MULTIPLE CHOICE QUESTIONS
Unit 2     Bacteriology

Brucella Bordetella
1. Mode of transmission of Brucella is:  7. “Thumb print appearance” in culture film smear is
 (Recent Pattern Aug 2013) seen in:  (Recent Pattern Dec 2013)
a. Air b. Water a. Bacillus anthracis b. Brucella species
c. Milk d. Aerosol c. Bordtella pertussis d. Clostridium welchii
2. Brucella melitensis is commonly found in: 8. Culture media for pertussis is: 
 (PGI Nov 2015)  (Recent Pattern Dec 2014)
a. Pig b. Camel a. LJ medium b. Chocolate medium
c. Sheep d. Goat c. Wilson blair medium d. Bordet – Gengou medium
e. Reindeer 9. Acellular pertussis vaccine contains: (AIIMS May 2011)
3. All are true about Brucella, except:  (AIIMS May 2011) a. Pertactin, flagillary hemagglutinin, cytotoxin, endotox-
a. B. abortus is capnophilic in
b. Transmission by aerosol can occur occasionally b. Pertactin, flagillary hemagglutinin, fimbriae, endotoxin
c. Pasteurization destroys it c. Pertactin, fimbriae, cytotoxin, pertussis toxin
d. 2ME is used to detect IgA d. Flagillary hemagglutinin, pertussis toxin, fimbriae
4. A farmer who handles goats and sheep is presenting 10. The usual incubation period of pertussis is: 
with fever, muscle and joint pain and rash. The most  (Recent Pattern July 2016)
likely diagnosis is:  (Recent Pattern July 2016) a. 7 – 14 days b. 3 – 5 days
a. Brucellosis b. Malaria c. 21 – 25 days d. Less than 3 days
c. Glanders d. Melioidosis 11. Whooping cough is caused by: 
5. Milk ring test is seen in:  (Recent Pattern Dec 2013)  (Recent Pattern Nov 2014)
a. Brucellosis b. Bacteriodes a. B. pertussis b. H. influenzae
c. Tuberculosis d. Salmonellosis c. Pneumococcus d. Meningococcus
6. A 30-year-old woman who has unpasteurized cheese 12. Mouse is used for pathogenicity testing in: 
often, suffers from fever, fatigue, sweating for a month.  (Recent Pattern Dec 2012)
There is a history of loss of appetite and weight. On a. M. tuberculosis b. C. diphtheriae
examination, hepatosplenomegaly is present and blood c. B. pertussis d. Brucella
count revealed pancytopenia. Which of the following 13. Whole cell pertusis vaccine used to prevent
causative agent is responsible for this condition?  (JIPMER Nov 2017)
 (PGI pattern) a. Infection of Bordetella parapertussis
a. Brucella melitensis b. Francisella tularensis b. Infection of Bordetella pertussis
c. Pasteurella multocida d. Yersinia pestis c. Both
e. Bordetella pertussis d. None of the above

ANSWERS AND EXPLANATIONS


1. Ans.  (c) Milk 3. Ans.  (d) 2ME is used to detect IgA
Ref: Ananthanarayan and Paniker’s Textbook of Microbiol- Ref: Ananthanarayan and Paniker’s Textbook of Microbiol-
ogy – 10th ed – Page 348 ogy – 10th ed – Page 345, 348, 346
• The most important vehicle of infection is raw milk. Milk
• • Brucellae are strict aerobes but B. abortus is capnophilic.

products, meat from infected animals, raw vegetables or Many strains require 5-10% CO2 for growth.
water supplies contaminated by the feces or urine of • The modes of infection are ingestion, contact, inhalation

infected animals may also be responsible. or accidental inoculation.


• Brucellae are killed by pasteurization.

2. Ans.  (a) Pig ; (b) Camel; (c) Sheep; (d) Goat; (e) Reindeer
4. Ans.  (a) Brucellosis
Ref: Ananthanarayan and Paniker’s Textbook of Microbiol-
ogy – 10th ed – Page 348 Ref: Ananthanarayan and Paniker’s Textbook of Microbiol-
• Human brucellosis is acquired from animals, directly

ogy – 10th ed – Page 347
or indirectly. Goats, sheep, cattle, buffaloes and swine • The clinical features are suggestive of brucellosis.

are common sources. In some parts of the world, also Human infections may be of 3 types – Latent infection;
154 transmitted by dogs, reindeer, caribou, camels, yaks. Acute or subacute brucellosis; chronic brucellosis.
5. Ans.  (a) Brucellosis 9. Ans.  (d) Flagillary hemagglutinin, pertussis toxin,

Chapter 17     Brucella and Bordetella


fimbriae
Ref: Ananthanarayan and Paniker’s Textbook of Microbiol-
ogy – 10th ed – Page 349 Ref: Ananthanarayan and Paniker’s Textbook of Microbiol-
• A sample of whole milk is mixed well with a drop of

ogy – 10th ed – Page 343
stained brucella antigen and incubated in waterbath • Acellular vaccine contains Pertussis toxin, Filamentous

at 70˚C for 40–50 mins. If antibodies are present in the hemagglutinin, Agglutinogens 1,2,3 associated with
milk, the bacilli are agglutinated and rises with the fimbriae, and Pertactin.
cream to form a blue ring at the top, leaving the milk
unstained. If antibodies are absent, no ring is formed 10. Ans.  (a) 7 – 14 days
and milk remains uniformly blue. Ref: Ananthanarayan and Paniker’s Textbook of Microbiol-
ogy – 10th ed – Page 341
6. Ans.  (a) Brucella melitensis
• Incubation period is 1–2 weeks.

Ref: Review of medical microbiology – 13th edition – Levinson


– page 395 11. Ans.  (a) B. pertussis
• Brucella is a Gram negative rod which causes zoonotic

Ref: Ananthanarayan and Paniker’s Textbook of Microbiol-


infections ogy – 10th ed – Page 341
• Species: B. melitensis, B. abortus, B. suis
• Bordetella pertussis causes 95% of whooping cough

• Enters human body through milk and milk products,



direct contact with sheep, goat, cows cases.


• Localizes in the reticulo endothelial system
12. Ans.  (c) B. pertussis

• Imported cheese from unpasteurized milk is the source


of infection for B. melitensis Ref: Ananthanarayan and Paniker’s Textbook of Microbiol-


• Symptoms include fever, fatigue, sweating for prolonged
• ogy – 10th ed – Page 340
period. There will be a history of loss of appetite and • Antibody of pertussis toxin can protect mice against

weight. On examination, hepatosplenomegaly will intranasal, intraperitoneal or intracerebral challenge.


be present and blood count revealed pancytopenia.
Osteomyelitis is also a usual complication. Symptoms 13. Ans.  (b) Infection of Bordetella pertussis
are more severe and prolonged with B. melitensis.
• Lab diagnosis: Growth in culture with 10% CO2;

Ref: Ananthanarayan and Paniker T.B of microbiology –
castenada method of culture with solid and liquid 10th ed – Page 343
medium; • Whole cell pertussis vaccine is prepared from a smooth

• Standard agglutination test is also diagnostic; a rise in



Phase I strain of Bordetella pertussis
titre with paired serum or single serum with 1:160 titre • Detoxication with 0.2% merthiolate – is the method of

is diagnostic; inactivation
• Treatment: Tetracycline and rifampicin

• Alum absorbed vaccine provides better protection

• Its usually given along with Diptheria and tetanus toxoid


7. Ans.  (c) Bordtella pertussis as DPT vaccine


• Children under four years who have contacts with
Ref: Ananthanarayan and Paniker’s Textbook of Microbiol-

pertussis patient should receive a booster dose even if


ogy – 10th ed – Page 339
they had get previous immunisation
• In culture films, the Bordetella pertussis bacilli tend

• Complications: Post vaccinal encephalopathy


to be arranged in loose clumps, with clear spaces in • To avoid complications – Acellular vaccine is preferred

between, giving a ‘thumbprint appearance’.

8. Ans.  (d) Bordet – Gengou medium


Ref: Ananthanarayan and Paniker’s Textbook of Microbiol-
ogy – 10th ed – Page 339
• Complex media are required for isolation of B. pertussis.

E.g. Bordet Gengou glycerine potato blood agar, Regan


lowe media, Charcoal blood agar.

155
18
Mycobacterium
MYCOBACTERIUM TUBERCULOSIS Secondary TB
y Occurs in adults (latent infection – reactivation)

y
Acid fast organism y It mainly affects the upper lobe

y
Gram-positive bacilli y It leads to necrosis, tissue destruction and cavity formation

y
Slender rods y They expectorate plenty of bacilli leading to spread of infec-

y
Has characteristic presence of mycolic acid in cell wall tion

• Slow growing, nonmotile, and noncapsulated y Widespread dissemination occurs in immunodeficient indi-

y
viduals
y Mycobacterium tuberculosis differs from other organisms by
y
not being visualized in Gram staining
y It needs special staining called as acid-fast staining
y
y Discovered by Robert Koch
y
y Acid fastness is because of presence of unsaponifiable lipid-
y
rich waxy mycolic acid that is seen in the cell wall
y M. tuberculosis is both acid (20–25% sulphuric acid) and
y
alcohol (3% hydrochloric acid) fast

Remember
Acid fast organisms:
• Mycobacterium

• Nocardia

• Rhodococcus

• Legionella micdadei

• Coccidiean parasites

Figure 1: HPE of a TB granuloma (Courtesy: Dr S.Jamuna Rani,
MD (Pathology), Associate professor, Tagore Medical College and
Pathogenesis Hospital, Chennai)
y Mode of infection is by droplet nuclei (mainly the smallest
y
droplets <5–10 μm in diameter)
y After entering the respiratory tract – the risk of developing Clinical Manifestations
y
disease mainly depends upon individual’s innate immunity y TB is classified based on the clinical features as pulmonary,
y
and cell-mediated immunity extrapulmonary or both.
y Clinical illness that develops first, mainly in children is called y It classically presents as low grade fever, cough with
y
y
primary TB expectorations, weight loss, loss of appetite, hemoptysis,
y Bacilli that remains in the body for years and being persist – cervical lymph nodes enlargement.
y
later in life can go for reactivation leading to secondary TB y Systemic extrapulmonary infections can affect any site and
y
y It is estimated that up to 10% of infected people will develop may present as:
y
TB in their lifetime.  Pleural TB

 Lymphadenopathy
Primary TB

 Genitourinary TB

 Skeletal TB
y It is seen in children; the alveolar macrophages that engulf

 TB meningitis
y
the TB bacilli make it multiply intracellularly; this leads to

 Tuberculoma
localized subpleural infection affecting lower lobe or lower

 Gastrointestinal TB
part of upper lobe named as Ghon focus

 Pericardial TB
y Ghon focus along with the hilar lymphadenopathy is called

y
Primary complex. (Occurs after 1–2 months after infection).
y Around 2 to 6 months, the lesion heals leaving a calcified
y
nodule.
Laboratory Diagnosis Liquid media used are Middlebrooks 7H9 media (used in

Chapter 18     Mycobacterium


 

automated cultures like MGIT)


Sputum Microscopy   Liquid media are also helpful for antimicrobial suscep-
y Sputum should be collected and concentrated using 4% N
y
tibility testing in Mycobacterial growth indicator tube
acetyl cysteine or 2% NaCl (Petroff’s method) (MGIT)
y To identify the bacilli in microscopy 104 bacilli/mL should be
Remember
y

present in the sputum; hence concentration techniques will


improve the chance of detection Culture media commonly used are:
y Ziehl-Neelsen staining method using strong carbol fuchsin
y
• Lowenstein Jensen media (solid media)

with intermittent heating, followed by 25% H2SO4 (according • Middle brook 7H10 and 7H11 media (solid media)

to RNTCP) and counterstaining with methylene blue is done. • Middle brook 7H9 broth (liquid media)

Figure 2: Image of Ziehl Neelsen staining (Courtesy: CDC/ Ronald


W. Smithwic)
  Fluorescent staining using Auramine rhodamine or
acridine orange and counting done by LED microscope is Figure 3: Growth of M.tb colonies in LJ media
most sensitive as it is done in high power field and it covers (Courtesy: Dr Vanathi S, Asst Prof, Dept of Microbiology KAP
many areas Viswanathan medical college, Trichy)

Table 1: RNTCP grading of tuberculosis smear in AFB


Automated Systems of TB Culture

Number of AFB Grading Number of fields y BACTEC MGIT


y

examined y BACTEC 9000MB


y

≥10 AFB/ OIF 3+ 20 y BacT/ALERT


y

1–10 AFB/OIF 2+ 50
High Yield
10–99 AFB/100 OIF 1+ 100
• Newer method of cultivation of M.tb  BACTEC MGIT –
1–9 AFB/100 OIF Scanty 100

Automated Mycobacteria growth indicator tube – medium


No AFB Negative 100 used in this system is 7H9 Middlebrook medium with
fluorometric detection based on oxygen consumption by the
Sputum Culture microorganism for its growth
• MGIT also detects resistance to Pyrazinamide – because this is

High Yield added in the medium

Collection of sputum: (RNTCP) y Another system used is ESP system – recently this helps to
y

• Day 1: Patient should provide on the spot sputum sample



detect drug susceptibility of mycobacteria also.
• Day 2: Patient should bring an early morning sample
Antimicrobial Susceptibility Testing

y Gold standard for diagnosis of TB is culture which can detect


y

y Phenotypic methods in LJ media:


y

even 10–100 bacilli/mL  Absolute concentration method




y Limitation of culture is generation time for TB bacilli is 14 – 15


y

 Resistance ratio method




hours hence culture takes 1 month to show visible colonies  Proportion method


y Media used:
y

y Automated methods
y

 Solid media like Lowenstein Jensen media, Dorset egg


y Genotypic methods – detect genes that code for resistance


media, Petragnani medium


y

(E.g. GeneXpert detects rifampicin resistance)


 Most commonly used in LJ media (sterilized by inspissation


157
as it contains egg)
Molecular Methods y Time period to diagnose is less than 2 hours
Unit 2     Bacteriology

y It helps to identify rifampicin resistance


Polymerase chain reaction (PCR)
y


y WHO and RNTCP recommended diagnostic tool


Ligase chain reaction (LCR)


y

 

  Transcription-mediated amplification (TMA) Table 3:  Ideal method for diagnosis of Tuberculosis
  RFLP
  IS fingerprinting Tuberculosis Methods used to diagnose TB
  Line probe assay Pulmonary TB Sputum culture

Immunodiagnosis TB lymphadenopathy HPE


y Skin sensitivity testing:
y
TB meningitis CSF culture
 Mantoux test: For screening and to estimate the preva-

TB Genitourinary Early morning urine repeated
lence of M.tb collection and culture
 WHO advocates PPD-RT-23 with Tween 80


Disseminated / Miliary Biopsy and culture of bone marrow


 Mantoux test is the m/c used tuberculin test


TB and liver tissue and other sites


 Induration should be measured
TB with HIV Sputum microscopy is usually


≤ 5 mm Negative negative; Xpert MTB/RIF assay is


sensitive
6 – 9 mm Equivocal
≥ 10 mm Positive Treatment
  Uses of tuberculin test: For diagnosis of active infection in Table 4:  Treatment as per RNTCP Guidelines
infants and young children; it measures prevalence and
incidence of infection but not the disease First-line drugs Second-line drugs
• Isoniazid (H) • Ethionamide
IGRA: Interferon Gamma Release Assays
• •

• • Rifampicin (R) • • Thiacetazone (T)


y It is a test done with whole blood to diagnose Mycobacterium
y • • Pyrazinamide (Z) • • Para aminosalicylic acid
tuberculosis infection • • Ethambutol (E) • • Bedaquiline
y They do not differentiate latent TB infection from active
y

• • Streptomycin (S) • • Amikacin


tuberculous disease • • Capreomycin
y It measures T cell release of IFN gamma
y

• Cycloserine
y It is very useful in vaccinated populations as it is not affected

• Ciproloxacin
y

by vaccines like it happens in tuberculin test


• Rifabutin
y Two tests approved are:

• Kanamycin
y

 QuantiFERON – TB Gold test


 T-SPOT: TB test


DOTS–Short Term Regimens


Table 2:  Difference between QuantiFERON TB Gold test y y Rapid response
and T-SPOT y y Lower failure rate
y Lesser chances for resistance
QFT-GT T-SPOT
y

y y Better patient compliance


Process whole blood within 16 Process peripheral mononu-
hours clear cells within 8 hours Category I – new HRZE thrice a week for 2 months followed
Antigens used: ESAT-6, CFP-10 Antigens used: ESAT-6 & CFP-10 patients by HR daily or thrice a week for 4 months
& TB7.7 (single mixture of all) (Separate) Category II – HRZES thrice a week for 2 months
Concentration of IFN gamma is Number of IFN gamma Previously treated followed by HRZE for 1 month followed
measured producing cells are measured patients by HRE thrice a week for 5 months

GeneXpert MTB – RIF Assay Drug-Resistant TB


y It is a molecular rapid test method for the diagnosis of
y y DOTS plus refers to the programs that have components for
y

pulmonary and extrapulmonary TB MDR TB diagnosis and treatment


y Principle used in Real time PCR
y y Proper antimicrobial susceptibility testing should be done
y

y It is a cartridge-based assay
y to coin the term MDR TB or XDR TB

158
Table 5:  Treatment of drug-resistant TB

Chapter 18     Mycobacterium


Definition Treatment
MDR TB : Kanamycin + ofloxacin +
Multi drug resistant TB – Ethionamide + pyrazinamide +
Resistant to rifampicin and ethambutol + cycloserine (6–9
isoniazid months) followed by
XDR TB: Ofloxacin + ethionamide +
Extensively drug resistant ethambutol + cycloserine for 18
TB – resistant to rifampicin, months
isoniazid and any
fluoroquinolones and at least
one injectable second-line
drugs
Figure 4: Cigarbundle appearance of M.leprae (Courtesy: CDC)

High Yield Pathogenesis


• Newer Drugs for MDR TB:

y Virulence factor of lepra bacilli is – PGL – Phenolic glyco-


ƒ Bedaquiline
y

lipid - 1
ƒ Delamanid
ƒ

y Spreads by droplet infection – low pathogenicity but highly


y

infectious
Immunoprophylaxis y Incubation period is very long may be up to 2–5 years
y

y Disease primarily infects skin, peripheral nerves and nasal


y BCG vaccine – Bacille Calmette Guerin vaccine
y

mucosa but any organ can be affected


y

y It is developed from live attenuated strain of M. bovis


y Based on the presentation, leprosy is classified into:
y

y Intradermal injection is given at birth; vaccine should not be


y

 Lepromatous
y

given to immunosuppressed babies and not be given above




 Tuberculoid
age of 2 years


 Dimorphic
y It undergoes series of reactions from papule to vesicle finally


 Indeterminate
y

forming a scar


 Pure neuritic type (seen only in Indian type)


y The immunity ranges from 0–60% and may last for 10–15 years


y It mainly protects from miliary and disseminated serious TB


Table 6:  Differences between tuberculoid leprosy and
y

y Complication of vaccine:
lepromatous leprosy
y

 Locally abscess or ulcerations may occur




 Enlargement and suppuration of draining lymph nodes




Tuberculoid leprosy Lepromatous leprosy


 Generally fever with mediastinal adenitis may occur
Paucibacillary disease Multibacillary disease


Minimally infectious; less severe Highly infectious


Chemoprophylaxis
y Izoniazid is given for prophylais in those who has
y
Skin lesions are asymmetric in Symmetrical skin lesions
 Children who have contact with active TB (usually in the

distribution
same house) Nerves involved are ulnar, All organs systems are
 Infants of mother with active TB


posterior auricular, peroneal and involved except lungs and


 Latent TB


posterior tibial nerve CNS


 HIV patients having contact with active TB.
CMI present CMI absent


MYCOBACTERIUM LEPRAE Granulomatous lesions Non granulomatous

y Acid-fast organism but it differs from M.tb – needs 5% H2SO4


y
Lepromin test positive Lepromin test negative
in acid-fast staining method (because it is less acid fast than
M.tb)
y Not alcohol fast
y
Lab Diagnosis
y Bacilli are seen singly or in groups sometimes the bacilli
y
y Microscopy is the best method to diagnose in resource-poor
y

appears grouped together by a lipid-like substance called glia areas:


– this group is called globi;  Slit skin smear technique should be followed or skin


y Each globus appears as parallel rows and gives cigar-bundle


y punch biopsy or nerve biopsy specimens can be used
appearance;  Smears are graded based on the number of bacilli

159
Table 7:  Grading of smears Treatment
Unit 2     Bacteriology

Table 9:  Doses of drugs in multibacillary and


Bacilli per field Grading
paucibacillary leprosy
1 – 10 bacilli in 100 fields 1+
Drug Multibacillary Paucibacillary
1 – 10 bacilli in 10 fields 2+ Rifampicin 600 mg once a month 600 mg once a month
1 – 10 bacilli per field 3+ Dapsone 100 mg daily self- 100 mg daily self-
administered administered
10 -100 bacilli per field 4+
Clofazimine 300 mg once a month
100 – 1000 bacilli per field 5+ or 50 mg daily
>1000 bacilli or clumps and globi in every field 6+ Treatment 12 months 6 months
duration
y Bacteriological index is calculated by adding all the grading in
y

all the smear divided by the number of smears ROM regimen: Rifampicin, Ofloxacin and Minocycline
y Morphological index is calculated as the percentage of solid
High Yield
y

fragmented granular bacilli (SFGB) out of the total number


of bacilli
Lepra Reactions
y Criteria for calling solid rods are:
• Type 1: Downgrade and reversal reactions – occurs in border-
y

 Uniform staining of entire organism


line leprosy; type IV DTH hypersensitivity – neuritis occurs;




 Parallel sides


ƒ Treatment is corticosteroids with dapsone or rifampicin


ƒ

 Rounded ends


• Type 2: Erythema nodosum leprosum – type III hypersensitivity:


 Length 5 times than that of width



ƒ Treatment is glucocorticoids, thalidominde, clofazimine
ƒ

y Methods of cultivation:
y
and chloroquine
 It is not possible to cultivate lepra bacilli in bacteriological


media or in tissue culture


 Generation time: Average: 12–13 days

ATYPICAL MYCOBACTERIA
 Foot pad of mice: Intradermal inoculation granuloma

y It is also called as nontuberculous mycobacteria or mycobac-
y

develops standard procedure for experimental work teria other than tuberculous (MOTT) or anonymous myco-
 Nine banded armadillo: Highly susceptible to infections
 bacteria
 Artificial culture media: ICRC, Bombay – human fetal


spinal ganglion cell culture, adapted for growth on LJ Table 10:  Classification of atypical mycobacteria
media
y Test to detect CMI: Lepromin test
y
Runyon classification Species
 0.1 mL of lepromin is injected in the forearm of patient and

Photochromogens M. kansasii, M. marinum
observed on 48 hours and 21 days
Scotochromogens M. scrofulaceum, M. gordonae
 Two reactions are noted as follows:


Non-photochromogens M. avium, M. intracellulare,


Table 8:  Early and late reactions in lepromin test M. ulcerans, M. xenopi
Rapid growers M. fortuitum, M. chelonae,
Early reaction Late reaction M. smegmatis, M. phlei
Fernandez reaction Mitsuda reaction
Seen within 24–48 hours Develops after 7–10 days Table 11:  Important points to be remembered
Disappears in 3–4 days after injection and reaches
M. kansasii • Causes chronic pulmonary disease (DD: TB)
maximum in 3 to 4 weeks

M. marinum • Fish tank granuloma or swimming pool


Seen as redness and induration Test is read on 21st day

granuloma
in the inoculated site If a nodule of more than 5 mm
If the redness is more than is seen- then test is positive M. scrofulaceum • Cervical adenitis in children

10 mm – then the test is M. gordonae • Tap water scotochromogen


positive M. avium • Called as Battey bacillus


It indicates whether or not a It indicates cell-mediated intracellulare • Most common NTM isolated from lung disease

person has been previously immunity complex • Causes pulmonary disease and disseminated

sensitized to lepra bacilli lesions in AIDS patients


It is superior than late reaction M. fortuitum • Chronic abscess

This reaction is due to DTH to This reaction is due to bacillary M. chelonae • Injection site abscess

160 soluble constituents of lepra component of antigen M. ulcerans • Buruli ulcer


bacilli M. vaccae • Immunomodulator



MULTIPLE CHOICE QUESTIONS

Chapter 18     Mycobacterium


Mycobacterium Tuberculosis 11. Live TB bacillis culture is by: (Recent Pattern Nov 2015)
a. Tinsdale medium
1. True about M. tuberculosis is: (PGI November 2014)
b. MGIT
a. Strict aerobes b. Gram negative
c. MYPA medium
c. Thin cell wall d. Curved rod
d. BYCE agar
2. Acid fastness of tubercle bacilli is attributed to:
12. GenXpert used for getting diagnosis of TB in:
(Recent Pattern Dec 2014)
(Recent PatternJuly 2015)


a. Presence of mycolic acid




a. 1-2 hours
b. Integrity of cell wall
b. 5 hours
c. Both of the above
c. 10 hours
d. None of the above
d. 20 hours
3. Primary complex of M. bovis involves:
13. Fastest method for diagnosis of TB:
 (Recent Pattern Dec 2016)
(Recent Pattern Dec 2015)
a. Tonsil and lung


a. Gene expert
b. Tonsil and intestine
b. LJ medium
c. Tonsil and skin
c. TB MGIT
d. Skin and intestine
d. BACTEC
4. Which of the following statements regarding Gamma
14. To notify a slide as AFB negative minimum how many
release assays for diagnosis of tuberculosis is true:
fields should be checked: (Recent PatternJuly 2015)
(Recent Pattern 2012)
a. 20 b. 100


a. First generation Quanti FERON-TB assay used ESAT-6


c. 50 d. 200
b. Second generation Quanti FERON-TB (Gold) assay
15. Radiomimetic BACTEC detect growth of M. tuberculosis
used ESAT-6 and CPF-10
in how much time: (Recent Pattern Dec 2015)
c. These tests can distinguish M.tuberculosis and M.bovis
a. 1 week b. 2-3 weeks
d. None of the non-tuberculosis mycobacteria give a
c. 4-8 weeks d. >10 weeks
positive reaction with the test
16. Tuberculin test is positive if induration is:
5. Reactivation of TB is almost exclusively a disease of-
(Recent Pattern Dec 2013)
 (Recent Pattern Dec 2014)


a. > 2mm b. > 5mm


a. Bone b. Lymph nodes
c. > 7mm d. > 10mm
c. Lung d. Brain
17. Tuberculin test is: (Recent Pattern Nov 2015)
6. The factor which promotes virulence of M. tuberculosis:
a. Intramuscular b. Intradermal
a. Wax D
c. Subcutaneous d. None
b. Cord factor
18. Negative Mantoux test is when induration is:
c. Muramyl dipeptide
(Recent PatternJuly 2016)
d. Mycolic acid


a. <5 mm b. <10 mm
7. Rapid examination of tubercle bacilli is possible with:
c. <15 mm d. <20 mm
a. Ziehl-Neelsen stain
19. Positive tuberculin test means:
b. Kin Young stain
(Recent PatternJuly 2015)
c. Auramine Rhodamine stain


a. Resistance to TB
d. Giemsa stain
b. Susceptibility to TB
8. Mycobacterium tuberculosis grows in LJ media in:
c. Hypersensitivity
(Recent Pattern Dec 2015)
d. None of the above


a. 10-14 days b. 2-3 weeks


20. Which type of pulmonary TB is most likely to give
c. 4-8 weeks d. >10 weeks
sputum positive: (Recent Pattern Dec 2015)
9. Drug resistance in tuberculosis is due to:
a. Fibronodular
(Recent Pattern Nov 2014)
b. Pleural effusion


a. Transformation
c. Cavitary
b. Transduction
d. None
c. Conjugation
21. XDR-TB is defined as resistance to:
d. Mutation
(Recent Pattern Dec 2016)
10. Role of malachite green in LJ medium:


a. INH plus rifampicin


(Recent Pattern Nov 2015)
b. Fluoroquinolones plus INH plus amikacin


a. Increase growth of M.tuberculosis


c. Fluoroquinolones plus rifampicin plus kanamycin
b. Inhibits growth of other bacteria
d. Fluoroquinolones plus INH plus rifampicin plus
c. Nutritive value
d. As an indicator
amikacin 161
22. Organism identified by interferons: 34. Mycobacterium leprae can be grown on:
Unit 2     Bacteriology

 (Recent Pattern July 2016)  (Recent Pattern August 2013)


a. Staphylococcus b. Leptospira a. LJ medium
c. Campylobacter d. M. tuberculosis b. Robertson’s cooked meat medium
23. Test done for Mycobacterium tuberculosis by whole c. Foot pad of mice
blood is: (Recent Pattern 2018) d. Sabouraud’s agar
a. IGRA b. BACTEC 35. Hansen’s bacillus is cultured in:
c. Culture d. GenXpert  (Recent Pattern Dec 2016)
a. LJ medium
b. Robertson’s cooked meat medium
Atypical Mycobacteria
c. Foot pad of mice
24. True about Mycobacterium other than tuberculosis: d. Sabouraud’s agar
a. Causes disseminated infection 36. True about Mycobacterium leprae:
b. Occurs in persons with normal immunity (Recent Pattern July 2016)
c.


Causes decreased efficacy of BCG due to cross immu- a. Transmitted by droplet infection
nity b. Phenolic glycolipid(PGL) is virulence factor
d. Person to person transmission is seen c. Generation time 12-13 days
25. Pigment producing atypical mycobacteria: d. All are true
 (Recent Pattern Dec 2012) 37. Maximum lepra bacilli are seen in:
a. M. fortuitum and M. chelonae (Recent Pattern Dec 2015)
b.


M. xenopi and MAC a. TT b. LL


c. M. gordonae and M. szulgai c. BT d. BB
d. M. ulcerans 38. ENL is seen in: (Recent Pattern Dec 2012)
26. Which of the following is photochromogenic- a. Lepromatous leprosy
 (Recent Pattern Dec 2014) b. Tuberculoid leprosy
a. M. kansasii b. M. scrofulaceum c. Indeterminate leprosy
c. M. intracellulare d. M. avium d. Pure neuritic leprosy
27. Battey bacillus is: (Recent Pattern July 2015) 39. Urine LAM is used for diagnosis of: (AIIMS Nov 2017)
a. Klebsiella pneumonia a. Mycobacterium tuberculosis
b. Mycobacteria paratuberculosis b. Mycobacterium leprae
c. Klebsiella ozaenae c. Mycoplasma
d. Mycobacteria intracellulare d. Listeria monocytogenes
28. Fish tank granuloma is seen in: 40. Identify the following organism: (AIIMS Nov 2017)
 (Recent Pattern Dec 2014)
a. M. fortuitum b. M. kansasii
c. M. marinum d. M. leprae
29. Buruli ulcer is caused by: (Recent Pattern Dec 2012)
a. Streptococcus b. Spirillium minus
c. M. ulcerans d. Brucella
30. Tuberculosis complex includes all, except:
 (Recent Pattern Nov 2014)
a. M. tuberculosis b. M. bovis
c. M. kansasii d. M. microti
31. Which of the following is not a pathogenic mycobacteria:
 (Recent Pattern Dec 2012)
a. M. kansasii b. M. scrofulaceum
c. M. cheloneae d. M. smegmatis
32. Which of the following is not a rapidly growing atypical
mycobacteria causing lung infection:(AIIMS May 2010)
a. M. chelonae b. M. fortuitum a. Mycobacteria b. Nocardia
c. M. abscessus d. M. kansasii c. Actinomyces d. Corynebacterium
41. Leprosy as a zoonoses is prevalent among: 
M. Leprae  (PGI Nov 2018)
33. Which of the following is true regarding globi in a patient a. Nude mice b. Nine banded armadillo
with lepromatous leprosy: (Recent Pattern July 2015) c. Lion tailed macaques d. Chimpanzee
a. Consists of lipid laden macrophages e. Bats
b. Consists of macrophages filled with AFB
162 c. Consists of neutrophils filled with bacteria
d. Consists of activated lymphocytes
ANSWERS AND EXPLANATIONS

Chapter 18     Mycobacterium


1. Ans.  (a) Strict aerobes 6. Ans.  (b) Cord factor
Ref: Ananthanarayan and Paniker’s Textbook of Microbiol- Ref: Ananthanarayan and Paniker’s Textbook of Microbiol-
ogy – 10th ed – Page 351 ogy – 10th ed – Page 354
• M. tuberculosis is a strict aerobe.

• A factor called cord factor is also responsible for the

• They are slow growing, nonmotile. Noncapsulated and



virulence of M. tuberculosis.
nonsporing • Mycolic acid is a long chain fatty acid present in the

cellwall of Mycobacteria. It plays a role in pathogenesis


2. Ans.  (a) Presence of mycolic acid and along with peptidoglycan, is responsible for
Ref: Ananthanarayan and Paniker’s Textbook of Microbiol- granuloma formation)
ogy – 10th ed – Page 352
7. Ans.  (c) Auramine Rhodamine stain
• M. tuberculosis resists decolorization with 20% sulphuric

acid when stained with carbolfuchsin by Ziehl-Neelsen Ref: Ananthanarayan and Paniker’s Textbook of Microbiol-
method or by fluorescent dyes like auramine O or ogy – 10th ed – Page 358
rhodamine. • When several smears are to be examined, it is more

• Acid fastness has been ascribed to the presence of an



convenient to use fluorescent microscopy. Smears
unsaponifiable lipid-rich (mycolic acid) wax material in stained with auramine phenol or auramine rhodamine
the cell wall or to a semipermeable membrane around fluorescent dyes and examined under UV illumination.
the cell. • Bacilli appear as bright rods against a dark background.

Because of the contrast, the bacilli can be seen even


3. Ans.  (b) Tonsil and intestine under high power objective, enabling large areas to be
Ref: Ananthanarayan and Paniker’s Textbook of Microbiol- screened rapidly.
ogy – 10th ed – Page 357
8. Ans.  (c) 4-8 weeks
• M.bovis infection primarily infects cervical and

mesenteric lymphnodes from where it can spread to Ref: Ananthanarayan and Paniker’s Textbook of Microbiol-
bone, joints and other extrapulmonary sites. ogy – 10th ed – Page 352
• Human infection with M.bovis occurs by consuming

• Colonies of M. tuberculosis appears in 2–8 weeks of


raw milk. aerobic incubation at 37 C in LJ media.


• Negative report should be given only after 12 weeks of
4. Ans.  (b) Second generation Quanti FERON-TB (Gold)

incubation.
assay used ESAT-6 and CPF-10
Ref: Ananthanarayan and Paniker’s Textbook of Microbiol- 9. Ans.  (d) Mutation
ogy – 10th ed – Page 258 Ref: Ananthanarayan and Paniker’s Textbook of Microbiol-
• IGRA is a test done in serum

ogy – 10th ed – Page 362
• Helps in diagnosis of latent TB infection
• Drug resistance in tuberculosis is due to mutations, with

• Antigens used are: ESAT6, CFP10, TB7.7 of M.tubercu-


with an approximate rate of 1 in 108 cell divisions.


losis
• It measures T cell release of IFN gamma

• Combination therapy can prevent emergence of drug

• It cannot distinguish between active and latent



resistance.
infections
• It is very useful in vaccinated populations as it is not

10. Ans.  (b) Inhibits growth of other bacteria
affected by vaccinees like of the tuberculin test
Ref: Ananthanarayan and Paniker’s Textbook of Microbiol-
• Two tests approved are:QuantiFERON – TB Gold test;
ogy – 10th ed – Page 352

T-SPOT: TB test
• LJ medium is composed of coagulated hen’s eggs,

5. Ans.  (c) Lung mineral salt solution, asparagines and malachite green.
Ref: Ananthanarayan and Paniker’s Textbook of Microbiol- • Malachite green inhibits the growth of other bacteria

ogy – 10th ed – Page 356 and makes the media selective for M. tuberculosis.
• Reactivation of latent tubercular infection mainly affects

the upper lobe of the lungs.


• Lymph node involvement is unusual..

163
11. Ans.  (b) MGIT 17. Ans.  (b) Intradermal
Unit 2     Bacteriology

Ref: Ananthanarayan and Paniker’s Textbook of Microbiol- Ref: Ananthanarayan and Paniker’s Textbook of Microbiol-
ogy – 10th ed – Page 353 ogy – 10th ed – Page 360
• MGIT is Mycobacteria growth indicator tube.
• • In mantoux test, 0.1 mL of PPD containing 5 TU is

• It is a rapid growth detection method, which uses 7H9


• injected intradermally (between layers of skin and
Middlebrook medium with fluorometric detection not subcutaneously) on flexor aspect of forearm with a
technology via O2 consumption. tuberculin syringe, raising a wheal.
• Only live bacteria can consume O2. So the bacilli has to
• • The site is examined after 48-72 hours for induration.

be alive for it to be detectable by MGIT.


18. Ans.  (a) <5 mm
12. Ans.  (a) 1-2 hours
Ref: Ananthanarayan and Paniker’s Textbook of Microbiol-
Ref: tbfacts.org ogy – 10th ed – Page 360
• Gene expert can directly test sputum samples and give

• Refer to answer no.16.


the results in less than 2 hours.


• Gene expert test works by detecting DNA in TB bacteria.
• 19. Ans.  (c) Hypersensitivity
It can also detect the genetic mutations associated with
Ref: Ananthanarayan and Paniker’s Textbook of Microbiol-
resistance to Rifampicin.
ogy – 10th ed – Page 361
• It detects both live and dead bacteria. So cannot be used

for monitoring the treatment response. • A positive tuberculin test indicates hypersensitivity

to tuberculoprotein, denoting infection with tubercle


13. Ans.  (a) GenXpert bacilli or prior immunization with BCG.
Ref: Ananthanarayan and Paniker’s Textbook of Microbiol- • The test becomes positive 4-6 weeks after infection or

ogy – 10th ed – Page 359 immunization.


• Tuberculin allergy wanes gradually and disappears after
• GenXpert results are available in 1-2 hours.

4-5 years in the absence of subsequent contact with


• The use of liquid media with radiometric growth

mycobacteria.
detection such as BACTEC 460 gives results within 2–3 • In endemic areas, allergy is maintained by repeated

weeks. contacts with the bacilli.


• MGIT detects bacterial growth about one week prior to

that of LJ medium. 20. Ans.  (c) Cavitary


• LJ medium detects bacterial growth in 2–8 weeks.

Ref: Ananthanarayan and Paniker’s Textbook of Microbiol-


14. Ans.  (d) 200 ogy – 10th ed – Page 356 and 357

Ref: Ananthanarayan and Paniker’s Textbook of Microbiol- • Bacillary shedding in the sputum is higher in cavitation

ogy – 10th ed – Page 358 because the necrotic material breaks out into the airways
leading to expectoration of bacteria laden sputum..
• When Ziehl-Neelsen staining is performed and sputum

• Bacillary shedding is relatively scanty in lesions that do


smears are examined, a negative report should not be not communicate with the airways.
given till at least 300 fields have been examined, taking
about 10 minutes. 21. Ans.  (d) Fluoroquinolones plus INH plus rifampicin
• The minimum number of fields to be examined to give a

plus amikacin
negative Ziehl Neelsen sputum smear report is 300.
Ref: Ananthanarayan and Paniker’s Textbook of Microbiol-
15. Ans.  (b) 2-3 weeks ogy – 10th ed – Page 363
Ref: Ananthanarayan and Paniker’s Textbook of Microbiol- • XDR-TB (Extensively drug resistant MTB) is defined as

ogy – 10th ed – Page 359 MDR-TB that is resistant to isoniazid and rifampicin
plus any fluoroquinolone and atleat one of the three
• The use of liquid media with radiometric growth
injectable second line drugs (amikacin, kanamycin or

detection such as BACTEC 460 gives results within 2–3


capreomycin).
weeks.

16. Ans.  (d) >10 mm 22. Ans.  (d) M. tuberculosis

Ref: Ananthanarayan and Paniker’s Textbook of Microbiol- Ref: Ananthanarayan and Paniker’s Textbook of Microbiol-
ogy – 10th ed – Page 360 ogy – 10th ed – Page 360
• Mantoux or tuberculin test is considered positive if the

• Interferon gamma release assay uses M. tuberculosis

site of injection of PPD shows induration of > or = 10 mm antigen CFP 10 which reacts with T-lymphocytes to
in 48-72 hours. release gamma interferon which can be detected
• Induration of < or = 5 mm is taken as negative and • This test is not very specific for pulmonary TB
164

• It is no longer recommended.

6–9 mm as equivocal. •
23. Ans.  (a) IGRA 29. Ans.  (c) M. ulcerans

Chapter 18     Mycobacterium


Ref: Mosby’s manual of diagnostic and laboratory tests – Ref: Ananthanarayan and Paniker’s Textbook of Microbiol-
Page 772 ogy – 10th ed – Page 368
• Interferon-Gamma Release Assays (IGRAs) are whole-
• • M. ulcerans causes Buruli ulcer.

blood tests that can aid in diagnosing Mycobacterium • Ulcers are usually seen in legs or arms and are believed

tuberculosis infection. to follow infection through minor injuries.


• They do not help differentiate latent tuberculosis
• • After incubation period of a few weeks, indurated

infection from tuberculosis disease. nodules appear which break down forming indolent
• Two IGRAs that have been approved by FDA are
• ulcers which slowly extend under the skin.
ƒ QuantiFERON®-TB Gold In-Tube test (QFT-GIT);
ƒ

ƒ T-SPOT®.TB test (T-Spot)


ƒ
30. Ans.  (c) M. kansasii
Ref: Ananthanarayan and Paniker’s Textbook of Microbiol-
24. Ans.  (c) Causes decreased efficacy of BCG due to cross
ogy – 10th ed – Page 351 and 366
immunity
• M.kansasii is atypical mycobacteria.

Ref: Ananthanarayan and Paniker’s Textbook of Microbiol- • M.tuberculosis complex includes M.tuberculosis,

ogy – 10th ed – Page 369 M.bovis, M.africanum, M.microti, M.canetti, M.caprae


• Sensitisation with environmental mycobacteria is
• and M.pinnipaedi,
believed to interfere with the protective response to
BCG vaccination. 31. Ans.  (d) M. smegmatis
• This may be one of the reasons for wide variation in

Ref: Ananthanarayan and Paniker’s Textbook of Microbiol-


protective effect of the vaccine observed in field trials in ogy – 10th ed – Page 368
various parts of the world.
• M.smegmatis is commonly considered as a saprophyte

25. Ans.  (c) M. gordonae and M. szulgai in smegma but is seldom seen in that location. It is a
frequent isolate from soft tissue infections following
Ref: Ananthanarayan and Paniker’s Textbook of Microbiol- trauma or surgery.
ogy – 10th ed – Page 367 • Other three species given as options are pathogenic.

• M. gordonae and M.szulgai are scotochromogenic and


form pigmented colonies even in dark. 32. Ans.  (d) M. kansasii


Ref: Ananthanarayan and Paniker’s Textbook of Microbiol-
26. Ans.  (a) M. kansasii
ogy – 10th ed – Page 366
Ref: Ananthanarayan and Paniker’s Textbook of Microbiol- • M.kansasii is not a rapid grower but a photochromogen.

ogy – 10th ed – Page 366 • Other three species given as options are rapid growers.

• M. kansasii, M. marinum, M. asiaticum and M. simiae


are photochromogenic. 33. Ans.  (b) Consists of macrophages filled with AFB
• They do not form pigment in the dark, but when young

Ref: Ananthanarayan and Paniker’s Textbook of Microbiol-


culture is exposed to light for one hour in the presence ogy – 10th ed – Page 371
of air and reincubated for 24-28 hours, a yellow-orange
pigment appears. • Acid fast lepra bacilli found inside macrophages.

• The globi appear in Virchow’s lepra cells or foamy cells


27. Ans.  (d) Mycobacteria intracellulare which are large undifferentiated histiocytes.
• The bacilli are seen as agglomerates being bound

Ref: Ananthanarayan and Paniker’s Textbook of Microbiol- together by a lipid like substance known as glia.
ogy – 10th ed – Page 367
• M. intracellulare is commonly known as the Battey

34. Ans.  (c) Foot pad of mice
bacillus Ref: Ananthanarayan and Paniker’s Textbook of Microbiol-
• It was first identified as a human pathogen at the Battey

ogy – 10th ed – Page 372


state hospital for Tuberculosis, Georgia, USA and hence
the name. • Lepra bacilli are not cultivable in artificial bacteriological

media or tissue culture.


28. Ans.  (c) M. marinum • Foot pad of mice can be inoculated intradermally with

lepra bacilli. Lepra bacilli could multiply in foot pad of


Ref: Ananthanarayan and Paniker’s Textbook of Microbiol- mice when kept at 20°C.
ogy – 10th ed – Page 367
• M. marinum causes a warty skin lesion called swimming

pool or fish tank granuloma.

165
35. Ans.  (c) Foot pad of mice • Crops of tender, inflamed subcutaneous nodules appear
Unit 2     Bacteriology

with fever, lymphadenopathy and arthralgia.


Ref: Ananthanarayan and Paniker’s Textbook of Microbiol-
• This is an arthus type response to antigens released from
ogy – 10th ed – Page 372

dead lepra cells.


• M. leprae is Hansen’s bacillus.

• Lepra bacilli are not cultivable in artificial bacteriological


• 39. Ans.  (a) Mycobacterium tuberculosis
media or tissue culture.
Ref: Manual of clinical microbiology – 11th edition – Page
• Foot pad of mice can be inoculated intradermally with
558

lepra bacilli. Lepra bacilli could multiply in foot pad of


mice when kept at 20°C. • LAM – Lipoarabinomannan

• LAM antigen is a lipopolysaccharde present in myco-


36. Ans.  (d) All are true bacterial cell wall–released from metabolically active
cells or degenerating cells with patients having TB
Ref: Ananthanarayan and Paniker’s Textbook of Microbiol-
• Urine based lateral flow tests are helpful in diagnosis of
ogy – 10th ed – Page 371 and 373

TB especially TB HIV patients


• The mode of transmission of lepra bacilli may be through

droplet transmission or entry through skin. 40. Ans.  (a) Mycobacteria


• Phenolic glycolipid (PGL) is a polyketide-derived viru-

• The image shows Acid fast staining with blue background


lence factor produced by Mycobacterium tuberculosis,

and clumps of pink bacilli


Mycobacterium leprae, and other mycobacterial patho- • Bacilli are clustered together and single at some places

gens. • Acid fastness rules out Actinomyces and Corynebacte-


• Generation time of M.leprae is 12-13 days on average.


• rium
• Nocardia appears as branching filaments, long and

37. Ans.  (b) LL slender


Ref: Ananthanarayan and Paniker’s Textbook of Microbiol- • Hence from the above scenario – this image may be a

ogy – 10th ed – Page 373 culture smear of Mycobacteria – showing many bacilli
• It may be atypical mycobacteria
• In LL (lepromatous type) of leprosy, bacilli are seen in

• Only with culture and biochemical reactions we can


large numbers. This is therefore known as multibacillary confirm the organism; this image gives a presumptive
disease. evidence of Mycobacteria
• Bacilli are shed in large numbers in nasal and oral

secretions. Bacillemia is common. The lepromatous 41. Ans.  (b) Nine banded armadillo
type is more infective than other types.
Ref: https://wwwnc.cdc.gov/eid/article/21/12/15-0501_article
38. Ans.  (a) Lepromatous leprosy • Nine-banded armadillos (Dasypus novemcinctus) are

Ref: Ananthanarayan and Paniker’s Textbook of Microbiol- naturally infected with Mycobacterium leprae.
ogy – 10th ed – Page 374 • Hence they are zoonotic transmitters of infection.

• Erythema nodosum leprosum is type 2 lepra reaction


which occurs in LL and BL types (Lepromatous leprosy),


usually a few months after institution of chemotherapy.

166
19
Spirochaetes
SPIROCHAETES SPECIES y It is actively motile with rotator movement around the long

y
axis and backward and forward movement and flexion of the
y Spirochaetes are motile bacteria that has a characteristic body
y
twisted appearance y It cannot be seen by light microscope; needs dark ground

y
y A unique feature is the presence of endoflagella (axial microscope;
y
filament) – they are polar flagella that is situated between the y Staining methods:

y
outer membrane and cell wall  Silver staining is used to visualize the organism


 Staining films: Fontana’s method


 Staining tissue sections – Levaditi’s method


Cultivation
 It does not grow in artificial culture media


Cultivable strains are nonpathogenic treponemes –
Reiter’s strain
 Virulent strains can be maintained by serial passage in

testicules in rabbit – Nichol’s strain.

Figure 1: Spirochaetes endoflagella


Pathogenicity
y Spirochaetes have many pathogens classified as genera:
y IP of Syphilis – 10 to 90 days with average of 21 days
y
 Treponema
y
y Syphilis is transmitted by:

 Borrelia
y
 Venereal route

 Leptospira

 Congenital syphilis through maternal transfer


 Blood transfusion
Table 1: Pathogenic spirochaetes


y T. pallidum can penetrate the intact mucous membrane
y
Organisms Disease Caused or through the minute abrasions in the skin and enters the
lymphatics and blood.
Treponema pallidum-Veneral Syphilis
Treponema endemicum – Nonveneral Bejel Clinical Features
Clinical manifestations falls into three groups
Treponema pertunae – Nonveneral Yaws
Table 2: Clinical manifestations of syphilis
Treponema carateum – Nonveneral

Pinta
Primary • Chancre at the site of entry
Borrelia burgdorferi Lyme’s disease

syphilis • Painless, avascular chancre called as hard

Borrelia recurrentis Relapsing fever chancre
• Regional lymphadenopathy

Borrelia vincenti Vincent angina • By then the treponema enters the lymph and

blood stream
Leptospira interrogans Weil’s disease
Secondary • It presents after an asymptomatic phase of

TREPONEMA PALLIDUM syphilis around 1–3 months following primary syphilis
• Because of multiplication and dissemination of

y Gram negative, motile by endoflagellum Treponema through the blood – papular skin
y
y The causative agent of syphilis, Treponema pallidum, was rashes, mucous patches in the oropharynx and
y
discovered in the year 1905 by Schaudinn and Hoffmann. condylomata occurs
y The word pallidum refers to the pale staining nature of • Highly infectious during this stage

y
syphilis. It is a thin, delicate spirochaete of this size about
10 µm× 0.1 to 0.2 µm with ten regular spirals. contd...
Important Points to Be Remembered In
Unit 2     Bacteriology

Tertiary • Patient either develops cardiovascular or


syphilis neurological manifestations Serological Tests For Syphilis


• Aneurysms, gummatous lesions occur
Veneral Disease Research Laboratory (VDRL)

ƒ Tabes dorsalis
ƒ

ƒ General paralysis of insane


y It is an example of slide flocculation test
ƒ

ƒ Late benign syphilis – Gumma


y

y VDRL antigen is not prepared from T. pallidum; hence it is


ƒ

Congenital • Transmission occurs from mother to fetus


called as nonspecific test


syphilis • Manifestation in infant occurs only after

y How then an antigen prepared?


y

4 months of gestation  Antigen that is similar to T. pallidum is cardiolipin antigen




• Rhinitis (snuffles), mucocutaneous lesions,


that is derived from bovine heart which is then added with


bone changes, hepatosplenomegaly, anemia, cholesterol and lecithin
lymphadenopathy  This antigen is reconstituted just before doing the test


• Saddle nose deformity


y Serum sample from the patient is collected with universal


y

• Interstitial keratitis

precaution
• Hutchinson’s teeth

y VDRL test is done in a slide with 12 concentric rings


y

• Shaber shins

y Serum needs to be inactivated by heating (for inactivation


y

of inhibitors and complement)


Laboratory Diagnosis y Drops of serum (that has antibodies) are placed in the rings
y

y Collecting specimens from patients of syphilis are highly


y
and antigen added and kept in a VDRL rotator and properly
infectious; hence utmost care should be done mixed.
y Scrapings from chancre can be seen directly under dark
y
y When antibodies are present – it gives flocculation (clumps) –
y

ground microscopy after silver staining this can be visualized only under low power microscope
y Wet films are prepared from the exudates and it is examined
y
y Unique feature of VDRL:
y

 It gets negative after treatment so helps in prognosis


under dark ground illumination. T. pallidum is identified by its 

 Only test helpful to diagnose neurosyphilis (CSF)


slender spiral structure and its slow motility. A concentration 

of 104 /mL in the exudates is needed to visualize the


T. pallidum in a wet mount. Remember
y Direct fluorescent antibody test for T. Pallidum (DFA – TP) :
y • The sensitivity of VDRL in diagnosing Primary syphilis is

Smears from the infective lesions is fixed with acetone. DFA- 78%, secondary syphilis is 100%, latent syphilis is 95%, late
TP test is done using the fluorescent tagged anti – T. pallidum syphilis is 71%.
antiserum. • The specificity of the test is 98 to 100%.

High Yield Rapid Plasma Reagin Test


The direct microscopic techniques become positive 1–3 weeks y Latex agglutination test
y

before serological test and are most useful in the diagnosis of y Commercially prepared antigen helps to detect antibodies
y

primary, secondary and early congenital syphilis. y No need of pre heating of serum (instead the kit has choline
y

chloride in it – which inactivate the serum)


y No need to have low power microscope as the agglutination is
y

visible to naked eye


y Limitation – cannot test CSF
y

Biological False Positive Reactions


y BFP reactions – when nonspecific treponemal tests are
y

positive and treponemal specific tests are negative – which


are not due to technical errors are termed as BFP
y 1% of all reports are due to BFP
y

y BFP is due to certain causes:


y

 Acute BFP: Infections, injuries, and inflammations




 Chronic BFP: SLE, Collagen diseases, Rheumatoid arthritis




 Lepromatous leprosy, relapsing fever, malaria, TPE, IMN,




Figure 2: T. pallidum in dark ground microscopy (Courtesy: CDC) viral hepatitis, and pregnancy
y Serological tests available are:
y
y Pregnant females doing VDRL test: BFP may occur hence titer
y

value is important to confirm


Nonspecific test or Reagin VDRL, RPR – test measures y Titer value ≥1:8 is termed as VDRL reactive
antibody or standard test IgG and IgM
y

y Titer value less than that may be BFP – hence it needs


y

Group specific test Complement fixation test confirmation by specific tests or follow-up with another
sample
168 Species specific test TPI, TPHA, FTA-ABS
y BFP antibody is of IgM type while in syphilitic antibody is
y

of IgG type
y It occurs in three stages:

Chapter 19     Spirochaetes


Remember
y

 Localized infection – annular skin lesions (Erythema mi-




• Gold standard test is FTA ABS



grans)
• Most sensitive test is TPHA

 Disseminated infection: Fever, lymphadenopathy, head


• Most specific test is FTA ABS



ache, myalgia, and arthralgia
 Persistent infection: Chronic arthritis, polyneuropathy,
• Prognostic test is VDRL



encephalopathy, and acrodermatitis


• VDRL and FTA-ABS are sensitive in neurosyphilis;
y Laboratory diagnosis:

VDRL positive is specific; If negative confirm with FTA-ABS; y

 Media used for culture of B. burgdorferi is BSK Modified


• Treponema isolation from chancre can be done only during


Kelley’s medium

primary syphilis
 ELISA test helps in diagnosis
• IgM antibody testing in babies helps in diagnosis of


 False positive serological tests can occur with trepo-


congenital syphilis


nemes being FTA ABS positive but VDRL is negative


• The sensitivity of TPHA is 85 to 100% and the specificity is

98 to 100%.
LEPTOSPIRA
y They are actively motile bacilli with Characteristic hooked
Treatment of Syphilis
y

ends
y Benzathine penicillin G - 2.4 mU i.m single dose is given in
y
y They cannot be seen under light microscope
y

primary and secondary stages of syphilis. y Classification of Leptospira


y

y In patients with history of allergy to penicillin, Tetracycline


L. interrogans Pathogenic leptospira which has 200 serotypes
y

HCl – 500 mg qid or doxycycline – 100 mg bd can be given.


y In tertiary stage with neurosyphilis, aqueous crystalline
y
L. biflexa Saprophytic group that is not pathogenic
penicillin G 18 to 24 mU/d IV is given.
Pathogenesis
BORRELIA y Important factors that are needed for infection of leptospiro-
y

y Gram-negative motile bacilli that exhibits twisting motility


y
sis- (3 R)
y It has several species:
y
 Rats


 Borrelia burgdorferi causing lyme disease



 Rice fields


 Borrelia recurrentis causing epidemic relapsing fever



 Rain


 Borrelia duttoni causing endemic relapsing fever



y Leptospirosis is a zoonotic disease
y

 Borrelia vincenti causing Vincent’s angina



y Humans get infected by contact with water contaminated
y

with urines of infected animals


Relapsing Fever y Incubation period is usually in the range of 2 – 26 days
y

y Different serogroups causes different type of illness:


Table 3: Relapsing fever caused by Borrelia species
y

Epidemic relapsing fever Endemic relapsing fever Table 4: Disease caused by different serogroups of

B. recurrentis B. duttoni, B. hermsii, B. turicatae


Leptospira
Transmitted by louse Transmitted by Tick L.interrogans serogroups Diseases
Icterohaemorrhagiae Weils disease
Hemorrhage and CNS Less common
features more common Canicola Canicola fever
Doxycycline – single dose Doxycycline – 1 week Hebdomadis Seven-day fever
Fortbragg Pretibial fever
Vincent’s Angina Grippotyphosa Marsh fever
y Leptotrichia buccalis formerly called as Fusobacterium fusi-
y

forme
y Anaerobe, normal oral flora
y
Clinical Features
y Causes acute necrotizing gingivostomatitis: Vincents’ angina
y
y It classically presents with biphasic fever
y

y Porphyromonas gingivalis and endodontalis causes dental


y
y Acute leptospiremic phase has fever for around 3–10 days
y

infections duration - in this phase, organisms can be isolated and


cultured
y Followed by immune phase that has resolution of symptoms.
Lyme Disease y

 In this phase, antibodies start to appear and hence




y Lyme disease is caused by Borrelia burgdorferi


y

serological tests are done


y Vector for disease is ticks
169
y
y Mild disease: Presents as an acute febrile illness with nausea, y Culture media:
Unit 2     Bacteriology

y y

vomiting, abdominal pain, conjunctival suffusion and myalgia  Solid media: Korthof’s media, Stuart’s media, Fletcher’s


y Severe disease presents as Weils disease (Classical triad)


y
media
 Hemorrhage

 Liquid media: EMJH media


 Jaundice

 Animal inoculation in guinea pigs


 Acute kidney injury



y Serological tests form the main stay of diagnosis nowdays
y

 ELISA
y Complications of leptospirosis is: 

 Macroscopic slide agglutination test (MSAT)


y

 Multiorgan failure


 Microscopic agglutination test (MAT): Available only in




 Septic shock


reference laboratories


 Acute kidney injury


 A single antibody titer of 1:200 to 1:800 (depending upon


y Loss of magnesium in urine is characteristically seen in


y

the endemic titer) – clinches the diagnosis


leptospiral nephropathy

Treatment
Diagnosis (Given for 7 days)
y Lab parameters in CBC shows leukocytosis, increased ESR
y
y Mild leptospirosis – doxycycline / amoxycyllin/ampicillin
y

y In the acute febrile phase – examination of blood under dark


y
y Severe leptospirosis – penicillin IV or ceftraixone or cefotaxime
y

ground microscopy by Fontana’s stain shows leptospiral y Chemoprophylaxis: doxycycline or azithromycin


y

170
MULTIPLE CHOICE QUESTIONS

Chapter 19     Spirochaetes


Treponema Nonvenereal Treponematoses
1. Stain for treponema: (Recent Pattern Dec 2013) 12. Bejel is caused by:  (Recent Pattern Dec 2014)
a. Fontana’s b. Acid – facid a. T. pallidum
c. Methenamine – silver d. PAS b. T. endemicum
2. Syphilis was identified by:  (Recent Pattern Dec 2012) c. T. pertunae
a. Fraenkel b. Nicolaicu d. T. carateum
c. Schaudinn & Hoffmann d. Ogston 13. Pinta is caused by which subspecies of treponema:
3. In a syphilis patient, site which does not help in isolation  (Recent Pattern June 2014)
of organism:  (Recent Pattern Dec 2012) a. Pertenue
a. Gumma b. Chancre b. Pallidum
c. Mucosal patch d. Maculopapular rash c. Endemicum
4. Incubation period of syphilis:  d. Carateum
 (Recent Pattern Dec 2014) 14. All of the following statements about “yaws” are true
a. 1 hour – 5 hour except:  (All India 2011)
b. 24 hour - 48 hour a. Caused by Treponema pertenue
c. 1 days – 10 days b. Transmitted nonvenereally
d. 10 days – 90 days c. Secondary yaws can involve bone
5. Tabes dorsalis is seen in:  (Recent Pattern Dec 2012) d. Late stages of yaws involve heart and nerves
a. Primary syphilis 15. Yaws is caused by:  (Recent Pattern Dec 2012)
b. Secondary syphilis a. T. pallidum
c. Tertiary syphilis b. T. refrigens
d. Latent syphilis c. T. pertunae
6. True about primary chancre:  (PGI May 2015) d. T. carateum
a. Painless ulcer
b. Painless lymphadenopathy
c. Covered with exudate
Borrelia
d. Indurated lesion 16. Kelly’s medium is used in the isolation of: 
e. Organism can be cultured from exudative fluid  (Recent Pattern Nov 2014)
7. Diagnostic test for neurosyphilis is:  a. Leptospira b. Borrelia
 (PGI May 2013, 2010) c. Bartonella d. Brucella
a. VDRL b. FTA – ABS 17. “Erythema chronicum migrans” is caused by: 
c. TPI d. RPR  (Recent Pattern Dec 2013)
8. Confirmatory test for syphilis:  a. Toxoplasma gondii
(Recent Pattern Dec 2012) b. Toxocara canis
c.


a. VDRL b. FTA – ABS B. burgdorferi


c. RPR d. None d. Stronglyoids stercoralis
9. Most sensitive test in syphilis:  18. Lyme’s disease is –  (Recent Pattern Dec 2013)
(Recent Pattern Dec 2016) a. A mite – borne illness caused by B. burgdorferi
b.


a. VDRL b. TPHA A louse – borne illness caused by B. burgdorferi


c. RPR d. FTA – ABS c. A tick – borne illness caused by B. burgdorferi
10. Most sensitive test for Treponema:  d. Nonvector borne illness caused by B. burgdorferi
 (Recent Pattern Dec 2015)
a. VDRL b. RPR Relapsing Fever
c. FTA – ABS d. Kahn
19. True about B. recurrentis:  (Recent Pattern Dec 2012)
11. Non-treponemal serological tests–is/are true:
a. Causes leptospirosis
(PGI pattern)
b. Vector borne disease
a. Are helpful in definitive identification of Treponema
c. Water borne disease
pallidum infection
d. Transmitted by tick
b. Measure antibodies against Treponema pallidum
20. Louse born recurring fever:  (Recent Pattern Dec 2012)
c. Not helpful in monitoring the treatment; prognosis
a. B. recurrentis
d. Biological false positive results occur which needs to be
b. B. parkeri
ruled out by serial dilutions
c. B. duttoni
e. Measures antibodies against lipids released from
d. B. burgdorferi
damaged cells
171
Leptospira 31. Identify the organism from the given image:
Unit 2     Bacteriology

(AIIMS Nov 2017)


21. Which of the following is transmitted by rat urine:
 (Recent Pattern Dec 2012)
a. Listeria b. Leptospira
c. Legionella d. Mycoplasma
22. Reservoir of leptospira:  (Recent Pattern Dec 2012)
a. Cat b. Rat
c. Monkey d. Dog
23. Weil’s disease is caused by:  (Recent Pattern Dec 2013)
a. Listeria b. Mycoplasma
c. Legionella d. Leptospira
24. Most common form of leptospirosis: 
 (Recent Pattern Dec 2016)
a. Icteric form b. Weil’s disease
c. Anicteric form d. Hepatorenal form
25. Fletcher’s medium containing rabbit serum is used for:
 (Recent Pattern July 2016)
a. Streptococcus b. Bacillus anthracis a. Vibrio
c. Leptospira d. Borrelia b. Leptospira
26. Not used in leptospirosis:  (AIIMS May 2010) c. Helicobacter
a. Dark field illumination d. Salmonella
b. Microscopic agglutination test 32. Which of the following days of disease and incubation
c. Weil’s felix reaction period is/are correctly matched: (PGI May 2017)
d. Macroscopic agglutination test a. Measles: 4-5 days
27. Which of the following statements about leptospirosis is b. Chicken pox: 3-20 days
true:  (All India 2011) c. Bubonic plague: 2-5 days
a. Fluoroquinolones are the drug of choice d. Leptospirosis: 4-20 days
b. Rats are prime reservoirs e. Hepatitis A: 45-180 day
c. Person to person transmission is common 33. True about leptospirosis: (PGI May 2017)
d. Hepatorenal syndrome may occur in up to 50% of the a. It is a zoonotic disease
patients b. Transmission may occurs through cuts and abraded
28. Culture medium used for leptospira for laboratory diag- skin
nosis:  (Recent Pattern July 2016) c. It is an arboviral disease
a. Skirrow’s medium b. EMJH medium d. Rodents are the most important reservoir
c. BYCE agar d. Pike’s medium 34. Which of the following is/are true about Syphilis: 

29. Leptospira canicola infection usually presents as:   (PGI May 2017)
 (Recent Pattern July 2014) a. VDRL is the one of the most widely used nontreponemal
a. Jaundice antibody tests
b. Lobar pneumonia b. Tabes dorsalis causes hypertonia
c. Aseptic meningitis c. Jarisch Herxheimer reaction is due to IgE mediated
d. Diarrhoea and vomiting action
30. A sewage worker presents to the emergency department d. Primary and secondary syphilis are treated by penicillin
with fever and jaundice. Laboratoty findings reveal 35. Which of the following disease is/are caused by family
an elevated BUN and serum creatinine suggestive Spirochetes? (PGI May 2017)
of renal failure. Which of the following antibiotics is a. Lyme’s disease
recommended?  (All India 2011) b. Yaws
a. Erythromycin c. Syphilis
b. Penicllin G d. Pinta
c. Cotrimaxozole e. Leptospira
d. Ciprofloxacin

172
ANSWERS AND EXPLANATIONS

Chapter 19     Spirochaetes


1. Ans.  (a) Fontana’s 7. Ans.  (a) VDRL
Ref: Ananthanarayan and Paniker’s Textbook of Microbiol- Ref: Ananthanarayan and Paniker’s Textbook of Microbiol-
ogy – 10th ed – Page 378 ogy – 10th ed – Page 211
• Fontana’s method is used for staining films of specimens

suspected to harbor Treponema. 8. Ans.  (b) FTA – ABS


• Levaditi’s method is used for staining tissue sections.

Ref: Ananthanarayan and Paniker’s Textbook of Microbiol-


ogy – 10th ed – Page 382
2. Ans.  (c) Schaudinn and Hoffmann
• The confirmatory tests for syphilis are Fluorescent

Ref: Ananthanarayan and Paniker’s Textbook of Microbiol- Treponemal antibody absorption test (FTA-ABS) and
ogy – 10th ed – Page 378 TPHA (T. pallidum Hamagglutination assay).
• Treponema pallidum, the causative agent of syphilis was
• • They are helpful in identifying biological false positive

discovered by Schaudinn and Hoffmann in the chancres reactions.


and inguinal lymph nodes of syphilitic patients. • FTA-ABS test can be positive in Lyme disease (False

positive).
3. Ans.  (a) Gumma
9. Ans.  (d) FTA – ABS
Ref: Ananthanarayan and Paniker’s Textbook of Microbiol-
ogy – 10th ed – Page 380 Ref: Ananthanarayan and Paniker’s Textbook of Microbiol-
• Latent syphilis presents with cardiovascular lesions

ogy – 10th ed – Page 382
including aneurysms, gumma (chronic granulomata) • FTS-ABS is the most sensitive test for Syphilis.

and meningovascular manifestations. • Frequency of reactive serological tests in untreated


• Tertiary lesions contain only few spirochaetes and


• syphilis (in percentage)
serological tests only help in diagnosis.
• Tertiary lesions are manifestations of delayed hypersen-

Stage VDRL/RPR FTA-ABS TPHA
sitivity. Primary 70-80 85-100 65-85

4. Ans.  (d) 10 days – 90 days Secondary 100 100 100


Latent/ Late 6--70 95-100 95-100
Ref: Ananthanarayan and Paniker’s Textbook of Microbiol-
ogy – 10th ed – Page 379 10. Ans.  (c) FTA – ABS
• The incubation period of syphilis is 10-90 days.
Ref: Ananthanarayan and Paniker’s Textbook of Microbiol-

• The infective dose is small – as few as 60 treponemes are


ogy – 10th ed – Page 382

capable of infecting 50% of human volunteers.


• Refer answer no.3

5. Ans.  (c) Tertiary syphilis


11. Ans.  (d and e) Biological false positive results occur
Ref: Ananthanarayan and Paniker’s Textbook of Microbiol- which needs to be ruled out by serial dilutions, and
ogy – 10th ed – Page 380 Measures antibodies against lipids released from
• Late tertiary or quarternary syphilis presents with
• damaged cells
neurological manifestations such as tabes dorsalis or
Ref: Jawetz Microbiology – page 325 – 27th edition
general paralysis of insane several decades after the
initial infection. • Non treponemal tests are used as screening tests for

syphilis
6. Ans.  (All options are correct) • Although not sensitive in early syphilis, widely available

• Antigens in this are cardiolipin, cholesterol and purified


Ref: Ananthanarayan and Paniker’s Textbook of Microbiol-

lecithin
ogy – 10th ed – Page 379
• Cardiolipin–obtained from beef heart or liver

• The primary lesion in syphilis is a chancre at the site of



• Syphilitic antibodies are called as Reagin antibodies-is a

entry of the Spirochaete. In most of the cases the chancre mixture of IgM and IgG antibodies which is reactive with
is genital. The chancre is a paimless, relatively avasular, above antigens
indurated lesion. • Tests include – VDRL, RPR and TRUST

• It is covered by a glairy exudate which is rich in



• On serial dilutions, quantitative results can be obtained

spirochaetes. The regional lymphnodes rae swollen, which is helpful in monitoring the treatment
discrete, rubbery and non tender. • VDRL test is the standard test for neurosyphilis

173
12. Ans.  (b) T. endemicum appears as an expanding annular skin lesion (erythema
Unit 2     Bacteriology

migrans or EM).
Ref: Ananthanarayan and Paniker’s Textbook of Microbiol-
• It then proceeds onto disseminated infection (weeks
ogy – 10th ed – Page 384

later) and persistent infection (months or years later).


• Bejel is another name for endemic syphilis.

• It is caused by T. endemicum.
• 18. Ans.  (c) A tick – borne illness caused by B.burgdorferi
• It is non venereal syphilis that most commonly affects
Ref: Ananthanarayan and Paniker’s Textbook of

children.
Microbiology – 10th ed – Page 387
13. Ans.  (d) Carateum • B. burgdorferi is transmitted by the bite of Ixodid ticks.

• Ixodes dammini and related species are the vectors. The


Ref: Ananthanarayan and Paniker’s Textbook of Microbiol-

borrelia grows mainly in the midgut of the tick.


ogy – 10th ed – Page 384
• Infection occurs by regurgitation of the gut contents

• The causative agent for pinta (carate, mal del pinto) is


during biting.
T.carateum.
• The primary lesion is a papule which does not
• 19. Ans.  (b) Vector borne disease
ulcerate but develops into a lichenoid or psoriaform
Ref: Ananthanarayan and Paniker’s Textbook of Microbiol-
patch. It affects only skin. Secondary skin lesions are
ogy – 10th ed – Page 385
characterized by hypo or hyperpigmentation.
• Relapsing fever is a vector borne disease.

14. Ans.  (d) Late stages of yaws involve heart and nerves • B. recurrentis causes epidemic relapsing fever which

is louse-borne (transmitted from person to person)


Ref: Ananthanarayan and Paniker’s Textbook of Microbiol-
through body lice.
ogy – 10th ed – Page 384
• Endemic or tick borne relapsing fever is caused by

• Yaws is also known as Frambesia, pian, parangi.


B. duttoni, B. hermsii and B. parkeri.


• It is caused by T. pertenue.

• The primary lesion is an extragenital papule which


• 20. Ans.  (a) B. recurrentis
enlarges and ulcerates to produce a granuloma.
Ref: Ananthanarayan and Paniker’s Textbook of Microbiol-
Secondary and tertiary manifestations follow but
ogy – 10th ed – Page 385
cardiovascular and neurological involvement is rare.
• Destructive gummatous lesions of the bones are

• B. recurrentis causes epidemic relapsing fever which

common. is louse-borne (transmitted from person to person)


through body lice.
15. Ans.  (c) T. pertunae
21. Ans.  (b) Leptospira
Ref: Ananthanarayan and Paniker’s Textbook of Microbiol-
ogy – 10th ed – Page 384 Ref: Ananthanarayan and Paniker’s Textbook of Microbiol-
ogy – 10th ed – Page 390
• Non venereal syphilis and causative agents

• Leptospirosis is transmitted from rate urine.


Non venereal syphilis Causative agent

• Pathogenic leptospires survive for long periods in the


Endemic syphilis T.endemicum convoluted tubules of kidneys in natural hosts, multiply


Yaws T.pertenue and are shed in urine.
• Animal carriers excrete upto 100 leptospires per ml of
Pinta T.carateum

urine.
16. Ans.  (b) Borrelia
22. Ans.  (b) Rat
Ref: Ananthanarayan and Paniker’s Textbook of Microbiol-
Ref: Ananthanarayan and Paniker’s Textbook of Microbiol-
ogy – 10th ed – Page 387
ogy – 10th ed – Page 390
• Borrelia burgdorferi is a fastidious bacterium which can
• Several animals act as carriers of Leptospirosis. Rats are

be grown in modified Kelly’s (BSK) medium.


particularly important as they are ubiquitous and carry


• B. burgdorferi is the causative agent of Lyme disease or
the most pathogenic serotype icterohemorrhagiae.

Lyme borreliosis.
23. Ans.  (d) Leptospira
17. Ans.  (c) B.burgdorferi
Ref: Ananthanarayan and Paniker’s Textbook of Microbiol-
Ref: Ananthanarayan and Paniker’s Textbook of Microbiol-
ogy – 10th ed – Page 388
ogy – 10th ed – Page 387
• Weil’s disease is caused by Leptospira.
• Erythema chronicum migrans or Erythema migrans is a

• It is a severe fatal illness with hepatorenal damage.


skin lesion found in the early stage of Lyme disease.


• Most of the time, it is caused by serogroup icterohemor-


• Lyme disease occurs in 3 stages. After an incubation

174 •

rhageae.
period of 3-30 days, the first stage of localized infection
24. Ans.  (c) Anicteric form • The given clinical scenario is Leptospirosis. Leptospires

Chapter 19     Spirochaetes


are sensitive to penicillin and tetracyclines. But


Ref: Ananthanarayan and Paniker’s Textbook of Microbiol-
treatment should be started early in the disease course
ogy – 10th ed – Page 388
to be effective.
https://www.medicalnewstoday.com/articles/246829.php • Penicillin is given as IV, 1–2 million units 6 hourly for

• Leptospirosis is classified into two clinical types – icteric



seven days in serious cases.
and non-icteric.
• Non icteric type accounts for most (90%) of the cases.

31. Ans.  (b) Leptospira
Only 5-10% of patients present with icteric type. Ref: Manual of clinical microbiology – 11th edition – Page
1028; Ananthanarayan and Paniker T.B of microbiology –
25. Ans.  (c) Leptospira
10th ed – Page 377
Ref: Ananthanarayan and Paniker’s Textbook of Microbiol- • All members of spirochetes has the characteristic pres-

ogy – 10th ed – Page 388 ence of endoflagellum (axial filament)


• Leptospires grow in media enriched with rabbit serum.

• Endoflagella are nothing but polar flagella that winds

• Several liquid and semisynthetic solid media such as



around the helical protoplasmic cylinder which is
Korthof’s, Stuart’s and Fletcher’s media are used for situated between the outer membrane and cell wall
culture of Leptospira. • Endoflagella are seen in:

• Semisynthetic media, such as EMJH (Ellinghausen,



ƒ Treponema
ƒ

McCullough, Johnson, Harris) are commonly used. ƒ Borrelia


ƒ

ƒ Leptospira
ƒ

26. Ans.  (c) Weil felix reaction • Characteristic feature of Leptospira:


ƒ The helical conformation is right-handed


Ref: Ananthanarayan and Paniker’s Textbook of Microbiol-
ƒ

ƒ The cells have pointed ends, either or both of which


ogy – 10th ed – Page 389, 417
ƒ

are usually bent into a distinctive hook.


• Dark ground microscopy, microscopic agglutination

ƒ Two axial filaments (periplasmic flagella) with polar


ƒ

test and macroscopic agglutination test are used for insertions are located in the periplasmic space
diagnosis of Leptospirosis.
• Weil-Felix reaction is a heterophile agglutination test

32. Ans.  (c) Bubonic plague: 2-5 days; (d) Leptospirosis:
used for the diagnosis of Rickettsial diseases. 4-20 days

27. Ans.  (b) Rats are prime reservoirs Ref: Incubation period of each disease is explained in re-
spective Chapters
Ref: Ananthanarayan and Paniker’s Textbook of Microbiol-
ogy – 10th ed – Page 390 33. Ans.  (a) It is a zoonotic disease; (b) Transmission may
• Several animals act as carriers of Leptospirosis. Rats are

occurs through cuts and abraded skin; (d) Rodents are
particularly important as they are ubiquitous and carry the most important reservoir
the most pathogenic serotype icterohemorrhagiae. • Q.22 and Q.23

28. Ans.  (b) EMJH medium 34. Ans.  (a) VDRL is the one of the most widely used
Ref: Ananthanarayan and Paniker’s Textbook of Microbiol- nontreponemal antibody tests; (d) Primary and
ogy – 10th ed – Page 388 secondary syphilis are treated by penicillin
• Several liquid and semisynthetic solid media such as

Ref: Ananthanarayan and Paniker’s Textbook of Microbiol
Korthof’s, Stuart’s and Fletcher’s media are used for ogy – 10th ed – Page 382
culture of Leptospira • The most commonly used method of diagnosis is VDRL
• Semisynthetic media, such as EMJH (Ellinghausen,

and RPR; Tertiary syphilis causes hypotonia;


McCullough, Johnson, Harris) are commonly used. • The Jarisch-Herxheimer reaction (JHR) is a transient

immunological phenomenon seen commonly in


29. Ans.  (c) Aseptic meningitis
patients during treatment for syphilis, and it manifests
Ref: Ananthanarayan and Paniker’s Textbook of Microbiol- clinically with short-term constitutional symptoms
ogy – 10th ed – Page 389 such as fever, chills, headache and myalgias, besides
• Serotype canicola infection usually presents as aseptic

exacerbation of existing cutaneous lesions - it is not type
meningitis and icterohaemorrhagiae presents with I (IGE mediated)
abdominal symptoms.
35. Ans.  (a) Lyme’s disease; (b) Yaws; (c) Syphilis; (d) Pinta;
30. Ans.  (b) Penicllin G (e) Leptospira
• Already explained Q.15
Ref: Ananthanarayan and Paniker’s Textbook of Microbiol-

ogy – 10th ed – Page 391 175


20
Rickettsia and Chlamydia
RICKETTSIACEAE
y This family has five important genus
y
 Rickettsia

 Orientia

 Ehrlichia

 Coxiella

 Bartonella

y They are obligate intracellular parasites
y
y Transmitted by blood sucking arthropod vectors
y
Table 1:  Epidemiological factors and disease caused by family Rickettsiaceae
Genus Species Vector Disease
R. prowazekii Human body louse Epidemic typhus or Brill Zinsser disease
R. typhi Rat flea Endemic typhus
R. rickettsii Ticks Rocky mountain spotted fever
Rickettsia R. conorii Ticks Boutonneuse fever
R. australis Ticks Australian tick typhus
R. siberica Ticks Siberian tick typhus
R. akari Mites Rickettsial pox
Orientia O. tsutsugamushi Trombiculid mite Causes scrub typhus or chigger borne typhus
E. chaffeensis Human monocytic ehrlichiosis
Ehrlichia E. sennetsu Resembles glandular fever
E. equi Human granulocytic ehrlichiosis
Coxiella C. burnetii Q Fever – caused by Coxiella burnetii – acute and chronic endocarditis forms
Primary reservoir – Bandicoot
Transmitted from bandicoot to bandicoot or cattle by – Ticks – (Ixodid ticks)
From bandicoot to humans – no vector
The only organism that survives by the holder method of pasteurization – Coxiella
burnetti – flash method is effective
Bartonella B. hensalae Cat scratch disease, bacillary angiomatosis, bacillary peliosis, Bacteremia and endocarditis
B. quintana Trench fever, Chronic bacteremia, endocarditis, bacillary angiomatosis
B. bacilliformis Bartonellosis (Carriion’s disease), Oroya fever, Verruga peruana

Epidemic Typhus
y Causative agent: R. prowazekii
y
y Vertebrate host: Humans
y
y Human body louse: Pediculus humanus corporis
y
Pathogenesis
 Lice become infected by feeding on rickettsia infected patients -multiply in the gut of lice appear in feces in 3 to 5 days of

infection and gets transmitted from person to person
 Occasionally by aerosols of dried louse feces through inhalation or through the conjunctiva, infection is acquired

Clinical Features Scrub Typhus (Chigger Borne Typhus)

Chapter 20     Rickettsia and Chlamydia


y y Incubation period is 5 to 15 days
 y Causative agent: Orientia tsutsugamushi
y

y y Fever with chills
 y First observed in Japan



y

y y Characteristic rash on 4th and 5th day y Vector: Trombiculid mites - Mite islands

y

y y Stuporous and delirious
 y Humans bitten by mite larvae (Chiggers)


y

y y Case fatality: 40% y Transovarian transmission in mites


y

y Zoonotic tetrad is: O. Tsutsugamushi, chiggers, rats and sec-


y

Recrudescent Typhus ondary and transitional forms of vegetation


y Incubation period: 1 to 3 weeks
y Latent infection in whom recovered from epidemic typhus
y

y Characteristic: Eschar, regional lymphadenopathy and macu-


y

y Reactivation occurs and causes illness


y

lopapular rash
 is seen in patients


y

y Hence this group do not have extra human reservoir – it stays


y Three antigenic types: Karp, Gilliam and Kato
y

in human body itself as a latent infection


y

Genus Ehrlichia
Endemic Typhus y Small, Gram negative, obligate intracellular bacteria
y

y Causative agent: R. typhi


y
y Tick borne 

y

y Host: Rats, Humans


y
y Cytoplasm of infected phagocytic cells grow within phago-
y

y Vector: Flea – Xenopsylla cheopis


y
somes as mulberry-like clusters called as Morula 


Pathogenesis Pathogenicity
y Rickettsia multiplies in the gut of the flea and shed in feces -
y
Three infections:
gets transmitted to humans y Resembles glandular fever: Causative agent is Ehrlichia
y

y Humans acquire the disease:


y
sennetsu. Endemic in Japan. Causes lymphoid hyperplasia
 By the bite of infected fleas, when their saliva or 
feces is

and atypical lymphocytosis. No arthropod vector. Human
rubbed in 
 infection is suspected to be caused by ingestion of fish with
 Through aerosols of dried feces 


flukes
 By ingesting food recently contaminated with infected rat

y Human monocytic ehrlichiosis: Caused by Ehrlichia
y

urine or flea feces 
 chaffeensis. Transmitted by Amblyomma ticks. Deer and


y Human infection is a dead end
y
rodents are reservoir hosts. Human disease with leukopenia,
y Person to person transmission does not occur
y
thrombocytopenia and elevated liver enzymes
y Human granulocytic ehrlichiosis: Either identical with or
y

Tick Typhus closely related to equine pathogen Ehrlichia equi. Transmitted


by ticks. Deer, cattle and sheep are the suspected reservoir.
y Transovarian transmission occurs in ticks – it acts as both
y

Leucopenia and thrombocytopenia are seen in patients


vector and reservoir
y Transmitted to human by tick bite
y
Treatment
Rickettsial Pox y Doxycline for ehrlichiosis
y

y Self-limited, nonfatal, vesicular exanthem


Bartonella
y

y y Similar to chicken pox



y y Vesicular or varicelliform rickettsiosis
 y Tiny GNB
y

y y Causative agent: R. akari y Transmitted by arthropods


y

y y Reservoir : Domestic mouse
 y Pathogenic strains



y

y y Vector: Mite (Transovarial transmission)  B. bacilliformis





y y Pathogenesis in humans:  B. quintana





 Transmitted to humans by arthropod vectors through bite




 B. henselae


or feces
 Rickettsiae multiply locally and enter the blood


Bartonella Bacilliformis
 Localized chiefly in the vascular endothelial cells leading


to thrombus formation and occlusion of vascular lumen y y Causes Oroya fever


y y It has high mortality
y Late sequel is verruga peruana
Clinical Features y
y

y 1885 – Medical student named Daniel Carrion has inoculated


y Acute febrile illness, septicemia with maculopapular rash and
y

himself with material from verruga and developed Oroya


fever fever and died, hence named as Carrion’s disease
y The long survival of rickettsiae in various organs and
y

lymphatic tissues in infected humans and animals is a Bartonella Quintana


distinctive feature in its pathogenesis.
y During first world war: It caused trench fever or five-day
177
y

fever (Millions of cases)


y No animal reservoir
 CHLAMYDIAE
Unit 2     Bacteriology

y Transmitted by body louse



y Chlamydiae are obligate intracellular bacterial pathogen.
y

y Ability to grow in blood agar.


y

y The genus Chlamydia has three species: Chlamydia


y

trachomatis, Chlamydia psittaci and Chlamydia pneumoniae.


Bartonella Henselae
y Febrile illness with lymphadenopathy
y

y It usually occurs following a cat scratch – hence named as Cat


y
Chlamydia Trachomatis
scratch disease y Chlamydia trachomatis has 15 serologic variants. Serotypes
y

y Isolated from the blood of patients in blood media after


y
D to K are mostly associated with urethritis, cervicitis,
prolonged incubation endometritis, salpingitis, infertility and repeated abortions.
y Demonstrated in lymph node biopsy smears and sections by
y
y Chlamydia trachomatis presents as asymptomatic infection
y

Warthin-Starry staining and hence if left untreated it leads to PID and infertility.
y Associated with HIV infected and immunodeficient persons
Morphology
y

causing - Bacillary angiomatosis


y Chlamydiae possess both DNA and RNA and they have cell
y

Laboratory Diagnosis of Rickettsial Disease wall.


y A unique feature of the species in Chlamydiae is their complex
y Specimen: Smears from infected tissues and Blood:
y

reproductive cycle
y

 Pleomorphic coccobacilli 

y There are two forms in the developmental cycle namely


 Nonmotile, noncapsulated 

y

elementary body (EB) and reticulate body (RB)




 Gram negative, but do not stain well 
in routine Gram




staining y The EB is the infective form transmitted from one person to


y

y Cultivation:
y
another. EBs gets attached to the target cells and then enter
 Unable to grow in cell-free media

the cells inside a phagosome
 Growth occurs in the cytoplasm of infected cells

y Within 8 hours of entry into the cell, the EBs gets converted
y

 Optimum temperature: 32 – 35°C



into RBs and gets adapted to intracellular survival and starts
 Cultivated in yolk sac of developing embryo

its multiplication
 HeLa, Hep-2, Detroit 6 continuous cell line

y They undergo binary fission and produce enormous
y

y Neil Mooser Reaction:


y replicates within the intracellular membrane body called
 R. typhi and R. prowazekii – similar phenotypic features;
 inclusion body
hence to differentiate following test is performed y After 24 hours, the RB gets converted to EB and the inclusion
y

Š When male guinea pigs are inoculated intraperitoneally


Š
then lyses, releasing EBs from the cell.
with blood from a case of endemic typhus or with a
culture of R. typhi, it develops fever and characteristic Laboratory Diagnosis
scrotal inflammation. 

Š Scrotum enlarged and testes cannot be pushed back
Š
y The endocervix is the preferred site of specimen collection.
y

into the abdomen because of inflammatory adhesions y Endocervical swabs are collected under sterile precautions
y

between the layers of the tunica vaginalis. and immediately stored in the transport medium
y Chlamydial inclusion bodies can be seen in direct microscopic
High Yield
y

examination. It is readily stained by Giemsa or Papanicolaou


Weil-Felix Test stains
• Agglutination test in which sera are tested for agglutinins to y Culture: As the organism is intracellular, it can be isolated only
y

in living cells. Cell lines helpful for the growth of Chlamydiae


the O antigens of certain nonmotile Proteus strains OX 19,


OX2, OX K are McCoy (Mouse fibroblast line), HeLa 229 (Human cervical
• Due to sharing of antigen by ricketssiae and certain strain of carcinoma), BHK 21 (Baby hamster kidney)
y Enzyme Immunoassay and Immunofluorescence are the

proteus y

two widely used serological tests; but they are not routinely
Results of Weil Felix test done for the diagnosis of uncomplicated genital infections
Diseases OX - 19 OX – 2 OX – K and it is not recommended for the screening of asymptomatic
RMSF + + – individuals.
Rickettsial pox – – – y Molecular methods: The gold standard test for diagnosing
y

Chlamydia trachomatis infection is the detection of nucleic


Epidemic typhus + – –
acid in samples i.e., Nucleic Acid Amplification Test(NAAT).
Endemic typhus + – –
Brill zinsser disease +/– – – Treatment
Scrub typhus – – +
y Tetracycline 500 mg four times daily for a period of 7 days or
Trench fever – – –
y

single 1g dose of azithromycin is equally effective. Till now, no


drug resistance has been reported.
178
Table 2:  Diseases caused by Chlamydia species

Chapter 20     Rickettsia and Chlamydia


Species Disease Natural Mode of Serotypes Diagnosis Treatment
host transmission
C. trachomatis Urethritis, Humans Sexual contact; > 15 Giemsa stain – Doxycyline,
Pneumonitis, LGV, Perinatal inclusion bodies; cell Erythromycin
NGU, Trachoma transmission culture and NAAT
C. pneumoniae Atypical pneumonia Humans Respiratory droplets 1 Serological test Doxycycline

C. psittaci Psittacosis Birds Inhalation of dried 1 Serological test Doxycycline


bird feces

179
MULTIPLE CHOICE QUESTIONS
Unit 2     Bacteriology

RICKETTSIACEAE 14. Eschar is seen in all the Ricketssial diseases, except: 


 (Recent Pattern 2016)
Rickettsial Diseases a. Rickettsial pox b. Scrub typhus
1. Indian tick typhus is caused by:  (Recent Pattern 2016) c. Endemic typhus d. Indian tick typhus
a. R. conorii b. R. rickettsia 15. Most serious form of Rickettsial disease:
c. R. prowazekii d. R. typhi a. Q -Fever  (Recent Pattern 2016)
2. ‘Query fever’ is caused by:  (Recent Pattern 2013) b. Scrub typhus
a. Franciseella b. Pseudomonas c. Rocky mountain spotted fever
c. Ricketssia typhi d. Coxiella burnetti d. Trench fever
3. Mite is a vector for:  (PGI May 2013) 16. All are true regarding scrub typhus, except:
a. R. conorii b. R. rickettsia  (Recent Pattern Dec 2013)
c. R. prowazekii d. R. typhi a. Spread by mites
e. R. tsutsugamushi b. Caused by R. tsutsugamushi
4. Patient came form Nagaland and shows positive test c. Tetracycline is effective
with OXK antigen. Diagnosis is:  d. Caused by bite of adult mite on vertebral host
a. Scrub typhus  (Recent Pattern Dec 2013) 17. True about scrub typhus is:  (Recent Pattern AUG 2013)
b. Trench fever a. Incubation period is 3–4 days
c. Epidemic typhus b. Transmitted by larva of trombiculid mite
d. Endemic typhus c. Caused by Rickettsia typhi
5. Which rickettsiae are able to grow in cell free media? d. Eschar is diagnostic
a. R. Ricketsii b. R. Quintana 18. What is trench fever:  (Recent Pattern 2016)
c. R. Typhi d. R. Tsutusugamushi a. 5: days fever b. Q: fever
6. Weil Felix reaction for scrub typhus shows positivity for: c. Indian tick typhus d. Boutonneuse fever
 (Recent Pattern 2012) 19. Bartonella Quintana causes:  (Recent Pattern 2016)
a. OXK + OX19 b. OXK a. Scrub typhus b. Trench fever
c. OX: 2 d. OX: 19 c. Epidemic typhus d. Endemic typhus
7. Ricketsial disease with positive Weil-Felix reaction by 20. All are true about tick typhus, except: 
only OX -19:  (Recent Pattern Nov 2015)  (Recent Pattern Dec 2015)
a. Scrub typhus b. Epidemic typhus a. Tick borne
c. Q: Fever d. Trench fever b. Mice is reservoir
8. Weil-Felix reaction is based on the principle of sharing c. Recrudescence is common
common antigen between:  (Recent Pattern Dec 2013) d. Transmitted by bite
a. Rickettsia and Chlamydia 21. All are true about B. quintana, except: 
b. Streptococcus and Staphylococcus a. Causes trench fever  (Recent Pattern 2012)
c. Rickettsia and Proteus b. Tick is the vector
d. Rickettsia and Pseudomonas c. Not detected by Weil-felix reaction
9. Tunica reaction is seen in:  (Recent Pattern Nov 2014) d. Recurrence is common
a. Plague b. H. influenzae 22. Which of the following is a Ricketssial disease– 
c. R. mooseri d. B. Anthrax  (Recent Pattern 2016)
10. True about weil-felix reaction are all except: 
a. Pinta b. Boutonneuse fever
 (Recent Pattern 2016) c. Lyme’s disease d. Erythma migrans
a. OX: K antigen of Proteus vulgaris is used 23. Boutonneuse fever is caused by: 
b. It is an agglutination test  (Recent Pattern Dec 2013)
c. Scrub typhus is OX: K positive a. Rickettsia japonica b. Rickettsia sibirica
d. Q: fever shows negative results c. Rickettsia conorii d. Rickettsia australis
11. Tunica reaction is positive in: (Recent Pattern Dec 2015) 24. Brill: Zinsser disease is:  (Recent Pattern 2016)
a. R. typhi b. R. prowazekii a. Recrudescent of R. prowazekii infection
c. R. akari d. R. tsutsugamushi b. Recrudescent of R. conorii infection
12. Rickettsial infections cause 30% mortality due to: c. Recrudescent of R. typhi infection
a. Endocarditis  (Recent Pattern Nov 2015) d. Recrudescent of R. akari infection
b. Endothelial injury 25. Which of the following statements about bacillary
c. Hemodynamic instability angiomatosis is correct? (PGI pattern 2017)
d. Renal failure a. It is caused by Bartonella bacilliformis
13. The primary site of multiplication of rickettsial organ- b. It is typically confined to skin only
isms is in the:  (Recent Pattern Dec 2014) c. Cats are the reservoir for the etiological agent
a. Endothelial cells of small vessels d. The major differential diagnosis is Kaposi sarcoma
b. Parenchymal cells of liver e. Involvement of liver and spleen is termed as peliosis
180 c. Adventitia of all blood vessels hepatitis
d. Media of artieries
26. Which is not transmitted by arthropod:  39. Inclusion body containing glycogen is seen in: 

Chapter 20     Rickettsia and Chlamydia


 (Recent Pattern Dec 2014)  (Recent Pattern Dec 2015)
a. Rickettsia prowazekii b. Rickettsia akari a. Chlamydia trachomatis
c. Coxiella burnetti d. Rickettsia ricketii b. Chlamydia psittaci
27. “Brill: Zinsser” disease is:  (All India Dec 2013) c. Chlamydia pneumoniae
a. Q: fever d. All of the above
b. Endemic flea: borne typhus fever 40. L1, L2, L3 serovars of Chlamydia trachomatis cause:
c. Epidemic louse born typhus  (Recent Pattern 2016)
d. Rocky mountain spotted fever a. NGU b. LGV
28. Ehrlichia chaffeensis is causative agent of:  c. Trachoma d. Inclusion conjunctivitis
 (Recent Pattern 2012) 41. Trachoma is caused by which serotype of chlamydia
a. HGE b. HME trachomatis:  (Recent Pattern 2016)
c. Glandular fever d. None a. D to K b. L1 L2 L3
29. Which of the following is used for Rickettsia: c. A, B, C d. All of the above
42. Chlamydia pneumoniae causes:  (Recent Pattern 2016)
 (Recent Pattern 2012)
a. Atherosclerosis b. LGV
a. VDRL b. Paul: Bunnel test
c. Inclusion conjunctivitis d. Trachoma
c. Rose: waler test d. Weil felix reaction
43. Chlamydia trachomatis serovars D: K cause:
30. Morula form is seen in which infection: 
a. Trachoma  (Recent Pattern Dec 2014)
(Recent Pattern 2016)
b. Arteriosclerosis


a. Mycoplasma hominis b. Bartonella Quintana c. Urethritis


c. Chlamydiae d. Ehrlichia d. Lymphogranuloma venereum
31. Orientia tsutsugamushi causes  (CET 2018) 44. Fries test is useful for diagnosis of: 
a. RMSF b. Scrub typhus  (Recent Pattern Dec 2013)
c. Epidemic typhus d. Ricketssial pox a. Rickettsia b. Mycoplasma
c. Sarcoidosis d. Chlamydia
45. A 25 year old sexually active women presents with
CHLAMYDIAE purulent vaginal discharge and dysuria for 7 days
32. How does chlamydia differ from other usual bacteria:  after an unprotected sexual intercourse with a new
a. Lack cell wall  (Recent Pattern Dec 2015) partner. Which of the following is the most sensitive
b. Contains inclusion body investigations to determine the etiological agent? 

c. Cannot grow in free culture media a. An enzyme immunoassay  (AIIMS Nov 2014)
d. None of the above b. Gram stain
33. Taxonomically chlamydia is a:  c. A polymerase chain reaction for nucleic acids
 (Recent Pattern Aug 2013) d. Culture on sensitive medium
a. Fungus b. Virus 46. HP body is seen in:  (Recent Pattern 2016)
c. Bacteria d. Nematode a. Chlmaydia trachomatis b. Chlmaydia pneumoniae
34. Which is false about Chlamydia trachomatis:  c. Chlmaydia psittaci d. Mycoplasma hominis
a. It is biphasic  (All India 2010) 47. Which of the following is used for isolation of chlamydia–
b. Elementary body is metabolically active  (All India 2010)
c. Reticulate body divides by binary fission a. ELISA b. Microimmunoflorescence
d. Inside the cell it evades phagolysosome c. Yolk sac inoculation d. Immunoflorescence
35. Inclusion body is seen in:  (Recent Pattern 2012) 48. Levinthal Coles Lille bodies are seen in: 
a. Chlamydia b. Rickettsiae  (Recent Pattern 2016)
c. Mycoplasma d. H. Pylori a. Kala azar b. LGV
36. True about Chlamydia is:  (Recent Pattern 2016) c. Psittacosis d. Chicken pox
a. Replicative form is elementary body 49. Which of the following is/are Tick–borne disease:
b. Cell wall contains N acetyl muramic acid and peptido-  (PGI May 2017)
glycan a. Murine typhus b. Epidemic typhus
c. Infective form to host cell is elementary body c. Lyme’s disease d. Tularemia
d. All of the above are correct e. Trench fever
37. Which of the following has only one serotype:  50. Incubation period of Lymphogranuloma venerum:
 (Recent Pattern Dec 2013)  (AIIMS May 2018)
a. C. pneumoniae b. C. psittaci a. 1-2 days b. 1-3 weeks
c. C. trachomatis d. None c. 6 weeks d. 1-2 years
38. All of the following are true about Chlamydia except:  51. LGV is caused by:  (CET 2018)
a. Causes trachoma  (Recent Pattern Dec 2014) a. Chlamydia trachomatis
b. Gram positive b. Neisseria gonorrhea
c. Are also called basophilic viruses c. Calymmatobacterium granulomatis
181
d. Causative organism of psittacosis d. Treponema pallidum
ANSWERS AND EXPLANATIONS
Unit 2     Bacteriology

1. Ans.  (a) R. conorii


Ref: Ananthanarayan and Paniker’s Textbook of Microbiology – 10th ed – Page 413
• Indian tick typhus is caused by R. conorii

Table:  Human diseases caused by Rickettsia species


Group Species Disease Vector Vertebrate Distribution
reservoir
Typhus R. prowazekii Epidemic typhus Louse Humans Worldwide
Brill-zinsser disease
R. typhi Endemic typhus Rat flea Rat Worldwide
R. felis Endemic typhus Cat flea Opossum USA
R. rickettsii Rocky mountain Rabbit, dog, small North America
Spotted fever spotted fever Tick rodents
group
R. siberica Siberian tick typhus Wild animals, cattle Russia, Mongolia
Indian tick typhus ? rodents India
R. conori
Fever boutonneuse Dog, rodents Mediterranean
South Aftrican tick Dog, rodents South Africa
typhus
Kenyan tick typhus Rodents Kenya
R. australis Queensland tick typhus Bush rodents N Australia
R. japonica Oriental spotted fever ? Japan
R. akari Rickettsial pox Gamasid mite Mouse USA, Russia
Scrub typhus O. tsutsugamushi Scrub typhus Trombiculid mite Small rodents, birds East Asia, pacific islands,
Australia

2. Ans.  (d) Coxiella burnetti • It is a heterophile agglutination test in which patient sera

are tested for agglutinins (antibodies) to the O antigens


Ref: Ananthanarayan and Paniker’s Textbook of Microbiol- of certain non-motile Proteus strains OX 19, OX 2 and
ogy – 10th ed – Page 418 OX K.
• Query fever or Q fever is caused by Coxiella burnetii.
• • Agglutination occurs with OX K only in scrub typhus.

• As the causative agent of the disease was unknown



• In Epidemic and Endemic typhus, agglutination occurs

during the first investigation of the disease by Derrick, it only with OX 19.
was named as “Query fever”. • Rickettsiae of the spotted fever group agglutinate with

• It is a zoonoses which is transmitted among animals by



OX19 and OX 2 only.
Ixodid ticks.
5. Ans.  (b) R. Quintana
3. Ans.  (e) R. tsutsugamushi Ref: Ananthanarayan and Paniker’s Textbook of Microbiol-
ogy – 10th ed – Page 419
Ref: Ananthanarayan and Paniker’s Textbook of Microbiol-
• R. quintana differs from other members of the genus
ogy – 10th ed – Page 415

Rickettsiae by its ability to grow in cell-free culture


• R. tsutsugamushi causes scrub typhus.

media such as blood agar and was separated into a new


• The vectors are trombiculid mites of genus Leptotrom-

genus Rochalimaea.
bidium. • In subsequent taxonomical shift, it has been reclassified

as Bartonella and named B. quintana.


4. Ans.  (a) Scrub typhus
Ref: Ananthanarayan and Paniker’s Textbook of Microbiol- 6. Ans.  (b) OXK
ogy – 10th ed – Page 417 Ref: Ananthanarayan and Paniker’s Textbook of Microbiol-
182 • Weil felix reaction is done for the diagnosis of Rickettsial
• ogy – 10th ed – Page 417
diseases. • Refer answer no.1

7. Ans.  (b) Epidemic typhus 13. Ans.  (a) Endothelial cells of small vessels

Chapter 20     Rickettsia and Chlamydia


Ref: Ananthanarayan and Paniker’s Textbook of Microbiol- Ref: Ananthanarayan and Paniker’s Textbook of Microbiol-
ogy – 10th ed – Page 417 ogy – 10th ed – Page 413
• Weil-felix reaction is done for the diagnosis of Rickettsial

• Rickettsiae on entering into the human body multiply

diseases. locally and enter blood.


• It is a heterophile agglutination test in which patient sera

• They become localized chiefly in the vascular

are tested for agglutinins (antibodies) to the O antigens endothelial cells, which enlarge, degenerate and cause
of certain non-motile Proteus strains OX 19, OX 2 and thrombus formation, with partial or complete occlusion
OX K. of the vascular lumen.
• In Epidemic and Endemic typhus, agglutination occurs

only with OX 19. 14. Ans.  (c) Endemic typhus


8. Ans.  (c) Rickettsia and Proteus Ref: https://www.sciencedirect.com/topics/medicine-and-
dentistry/murine-typhus
Ref: Ananthanarayan and Paniker’s Textbook of Microbiol-
ogy – 10th ed – Page 417 • There is no eschar associated with endemic or flea borne

or murine typhus. This makes diagnosis difficult.


• Weil-felix reaction is a heterophile agglutination test in

• Eschar is a typical presentation of other Rickettsial


which patient sera are tested for agglutinins (antibodies) diseases.


to the O antigens of certain nonmotile Proteus strains
OX 19, OX2 and OX K. 15. Ans.  (c) Rocky mountain spotted fever
9. Ans.  (c) R. mooseri Ref: Ananthanarayan and Paniker’s Textbook of Microbiol-
Ref: Ananthanarayan and Paniker’s Textbook of Microbiol- ogy – 10th ed – Page 415
ogy – 10th ed – Page 416 • Rocky Mountain spotted fever is the most serious form

of rickettsial diseases with the highest mortality.


• R. typhi was previously called R. mooseri.

• It is transmitted by Dermacentor andersoni and related


• Neil-Mooser reaction or Tunica reaction is an animal

species of ticks.

inoculation test to differentiate R. typhi and R. prowazekii.


• Blood sample collected from patient is inoculated intra

16. Ans.  (d) Caused by bite of adult mite on vertebral host


peritoneally in a male guinea pig. Scrotal inflammation
occurs - Scrotum becomes enlarged and testes cannot be Ref: Ananthanarayan and Paniker’s Textbook of Microbiol-
pushed back into the abdomen because of inflammatory ogy – 10th ed – Page 415
adhesions between the layers of tunica vaginalis (Tunica • It is the larvae of the mite that bites the vertebrate host to

reaction or scrotal reaction). spread the agent of scrub typhus.


• R. typhi, R. conori and R. akari develop fever and

• The mite larvae are called chiggers and hence the name

testicular inflammation (positive tunica reaction). chigger-borne typhus.


• R. prowazekii produces only fever and no scrotal

inflammation. 17. Ans.  (c) Caused by Rickettsia typhi


10. Ans.  (a) OX – K antigen of Proteus vulgaris is used Ref: Ananthanarayan and Paniker’s Textbook of Microbiol-
ogy – 10th ed – Page 415
Ref: Ananthanarayan and Paniker’s Textbook of Microbiol-
ogy – 10th ed – Page 417 • Scrub typhus is caused by O. tsutsugamushi.

• The vectors are trombiculid mites of genus Leptotrom-


• For weil-felix reaction, OX 19 and OX 2 of Proteus •

bidium. Humans are infected when they are bitten by


vulgaris and OX K of P.mirabilis are used.


the mite larvae (chiggers).
11. Ans.  (a) R . Typhi • Patients typically develop a characteristic eschar at the

site of mite bite, with regional lymphadenopathy and


Ref: Ananthanarayan and Paniker’s Textbook of Microbiol- maculopapular rash.
ogy – 10th ed – Page 416
• Neil-mooser reaction or Tunica reaction is an animal

18. Ans.  (a) 5 – day fever
inoculation test to differentiate R. typhi and R.prowazekii. Ref: Ananthanarayan and Paniker’s Textbook of Microbiol-
• R. typhi, R. conori and R. akari develop fever and ogy – 10th ed – Page 419

testicular inflammation (positive tunica reaction).


• R. prowazekii produces only fever and no scrotal • Trench fever first occurred among the soldiers who

fought in the trenches of Europe


inflammation.
• It is also called the five day fever

12. Ans.  (b) Endothelial injury • Caused by Bartonella quintana.


Ref: Ananthanarayan and Paniker’s Textbook of Microbiol-


ogy – 10th ed – Page 414 19. Ans.  (b) Trench fever

• Fatalities occur in 20% of untreated cases of Rickettsial Ref: Ananthanarayan and Paniker’s Textbook of Microbiol-
183

diseases. This can be explained by the damage to the ogy – 10th ed – Page 419
vascular endothelium. • Refer answer no 7.

20. Ans.  (b) Mice is reservoir 25. Ans.  (d & e) (d) The major differential diagnosis is
Unit 2     Bacteriology

Kaposi sarcoma; (e) Involvement of liver and spleen is


Ref: Ananthanarayan and Paniker’s Textbook of Microbiol-
termed as peliosis hepatitis
ogy – 10th ed – Page 415
• In tick typhus, ticks act both as the vector and reservoir.

Ref: Jawetz Microbiology – page 306 – 27th edition
• The rickettsiae are transmitted transovarially in ticks.

• Bacillary angiomatosis is typically a disease of immuno-

• The infection may be transmitted to vertebrate hosts by



suppressed individuals, AIDS
any of the larval stages or by the adult tick. Transmission • Clinically it presents as a enlarging red papule surround-

to humans is primarily by bite as the rickettsiae invade ed by erythema which is similar to Kaposi sarcoma
the salivary glands of the tick. • Involvement of liver and spleen leads to proliferation of

cystic blood filled space – peliosis hepatitis


21. Ans.  (b) Tick is the vector • Diagnosis : silver staining – pleomorphic bacilli

Ref: Ananthanarayan and Paniker’s Textbook of Microbiol- • Treatment : oral erythromycin or doxycycline

ogy – 10th ed – Page 419 • Cats: Bartonella hensale – Trench fever


• B. quintana, causing trench fever or five-day fever is


26. Ans.  (c) Coxiella burnetti


transmitted by the body louse and not by the tick.
Ref: Ananthanarayan and Paniker’s Textbook of Microbiol-
22. Ans.  (b) Boutonneuse fever ogy – 10th ed – Page 412
Ref: Ananthanarayan and Paniker’s Textbook of Microbiol- • Coxiella burnettii is not transmitted by arthropod vector.

ogy – 10th ed – Page 413 • Coxiella infection to human mainly


• Boutonneuse fever is a rickettsial disease caused by


27. Ans.  (c) Epidemic louse born typhus


R.conori belonging to the spotted fever group.
• Pinta and Lyme disease are spirochaete diseases caused

Ref: Ananthanarayan and Paniker’s Textbook of Microbiol-
by Treponema carateum and Borellia burgdorferi ogy – 10th ed – Page 414
respectively. • Recrudescent typhus is also called Brill-Zinsser disease.

• In some patients who recover from epidemic typhus


23. Ans.  (c) Rickettsia conorii

(Caused by R. prowazekii), the rickettsiae may remain


Ref: Ananthanarayan and Paniker’s Textbook of Microbiol- latent in the lymphoid tissues or organs for years.
ogy – 10th ed – Page 413 • Reactivation of such latent infection leads to

• Diseases caused by Rickettsiae of spotted fever group



recrudescent typhus.
Organism Disease caused 28. Ans.  (b) HME
R. rickettsii Rocky mountain spotted fever Ref: Ananthanarayan and Paniker’s Textbook of Microbiol-
R. siberica Siberian tick typhus ogy – 10th ed – Page 416
• Human monocytic Ehrlichiosis (HME) is caused by
Indian tick typhus

E. haffeensis. It is transmitted by Amblyomma ticks.


R. conori
Fever boutonneuse • Ehrlichiae are small, gram-negative obligate intracellular

South African tick typhus bacteria which have an affinity towards blood cells.

Kenyan tick typhus 29. Ans.  (d) Weil-felix reaction


R. australis Queensland tick typhus Ref: Ananthanarayan and Paniker’s Textbook of Microbiol-
R. japonica Oriental spotted fever ogy – 10th ed – Page 417
• Weli-felix test is a serological test (heterophile aggluti-
R. akari Rickettsial pox

nation test) that is used for the diagnosis of Rickettsial


infections.
• All rickettsiae of spotted fever group are transmitted by

• VDRL test is used to diagnose syphilis.


tick except R. akari (rickettsial pox) which is transmitted • Paul bunnel test is used to diagnose infectious

by mite. mononucleosis (Causative: Ebstein barr virus)


• Rose waller test is a haemagglutination slide test for the
24. Ans.  (a) Recrudescent of R prowazekii infection

detection of rheumatoid Factor in serum samples. It was


Ref: Ananthanarayan and Paniker’s Textbook of Microbiol- previously used for diagnosis of rheumatoid arthritis.
ogy – 10th ed – Page 414
30. Ans.  (d) Ehrlichia
• Recrudescent typhus is also called Brill-Zinsser disease.

• In some patients who recover from epidemic typhus



Ref: Ananthanarayan and Paniker’s Textbook of Microbiol-
(Caused by R.prowazekii), the rickettsiae may remain ogy – 10th ed – Page 416
latent in the lymphoid tissues or organs for years. • Ehrlichia grows in the cytoplasm of the cells infected

• Reactivation of such latent infection leads to recrudes- phagocytic cells forming mulberry-like clusters called
184 •

cent typhus. morula.


31. Ans.  (b) Scrub typhus 37. Ans.  (a) C. pneumoniae

Chapter 20     Rickettsia and Chlamydia


Ref: Ananthanarayan and Paniker’s Textbook of Microbiol- Ref: Ananthanarayan and Paniker’s Textbook of Microbiol-
ogy – 10th ed – Page 413 ogy – 10th ed – Page 423
https://www.cdc.gov/pneumonia/atypical/cpneumoniae/
32. Ans.  (b) Contains inclusion body about/history.html
Ref: Ananthanarayan and Paniker’s Textbook of Microbiol- • C. pneumoniae has only one serotype.

ogy – 10th ed – Page 423 • Serotypes and diseases caused by various species of

• Chlamydia has cell wall but they contain only small



Chlamydiae
amounts of peptidoglycan in their cell wall. Species Serotype Disease
• They contain intracellular inclusions which are visible

Endemic blinding
under the light microscope. Other bacteria do not have C. trachomatis A,B,Ba,C
trachoma
intracellular inclusions. Inclusion conjunctivitis
D, E, F, G, H,
C. trachomatis Genital chlamydiasis
33. Ans.  (c) Bacteria I, J, K
Infant pneumonia
Ref: Ananthanarayan and Paniker’s Textbook of Microbiol- Lymphogranuloma
ogy – 10th ed – Page 423 C. trachomatis L1, L2, L3
venerum
• Chlamydiaare classified as bacteria belonging to the

C. psittaci Many serotypes Psittacosis
genus Chlamydia, in the family Chlamydiaceae, under Only one Acute respiratory
the order Chlamydiales. C. pneumoniae
serotype disease
• Though they are filterable and fail to grow in cell free

media like viruses, they are classified as bacteria because 38. Ans.  (b) Gram positive
they possess the following features.
ƒ They have both DNA and RNA
ƒ
Ref: Ananthanarayan and Paniker’s Textbook of Microbiol-
ƒ They have cell wall and ribosomes
ƒ
ogy – 10th ed – Page 423 and 424.
ƒ They replicate by binary fission
ƒ • Chlamydial cell wall resembles that of Gram negative

ƒ They are susceptible to antibiotics.


ƒ bacteria.
• Chlaydia are Gram negative but are stained better by

34. Ans.  (b) Elementary body is metabolically active Giemsa, Castaneda, Macchiavello and Gimenez stains.
Ref: Ananthanarayan and Paniker’s Textbook of Microbiol-
39. Ans.  (a) Chlamydia trachomatis
ogy – 10th ed – Page 423
• Elementary body is the infective form, which is

Ref: Ananthanarayan and Paniker’s Textbook of Microbiol-
extracellular. It does not actively replicate. ogy – 10th ed – Page424
• It is the reticulate form that actively replicates. The
• • C. trachomatis inclusion bodies contain glycogen and

elementary body following attachment to the susceptible can be stained by Lugol’s iodine.
epithelial cell is endocytosed into the host cell and lies • C. psittaci inclusion bodies contain no glycogen and

within the endosome. therefore cannot be stained with Lugol’s iodine.


• After 8 hours the elementary body undergoes transfor-

mation into the reticulate body which begins to repli- 40. Ans.  (b) LGV
cate. Ref: Ananthanarayan and Paniker’s Textbook of Microbiol-
ogy – 10th ed – Page 423
35. Ans.  (a) Chlamydia
• Serotypes and diseases caused by various species of

Ref: Ananthanarayan and Paniker’s Textbook of Microbiol- Chlamydiae


ogy – 10th ed – Page 422
Species Serotype Disease
• Refer answer no.1
C. trachomatis A,B,Ba,C Endemic blinding

36. Ans.  (c) Infective form to host cell is elementary body trachoma
C. trachomatis D, E, F, G, H, Inclusion conjunctivitis
Ref: Ananthanarayan and Paniker’s Textbook of Microbiol- I, J, K Genital chlamydiasis
ogy – 10th ed – Page 423 Infant pneumonia
• Elementary body is the infective form, which is extracel-

C. trachomatis L1,L2, L3 Lymphogranuloma


lular. venerum
• The elementary body following attachment to the
C. psittaci Many Psittacosis

susceptible epithelial cell is endocytosed into the host


cell and lies within the endosome. serotypes
C. Only one Acute respiratory disease
pneumoniae serotype 185
• PCR is the most sensitive and specific method for
Unit 2     Bacteriology

41. Ans.  (c) A, B, C •

detection of the causative agent.


Ref: Ananthanarayan and Paniker’s Textbook of Microbiol- • Another advantage of molecular methods is that non

ogy – 10th ed – Page 423 invasive sample can be used, thus simplifying specimen
• Refer answer no.1

collection and transport.

42. Ans.  (a) Atherosclerosis 46. Ans.  (a) Chlmaydia trachomatis

Ref: Ananthanarayan and Paniker’s Textbook of Microbiol- Ref: Ananthanarayan and Paniker’s Textbook of Microbiol-
ogy – 10th ed – Page 429 ogy – 10th ed – Page 426
https://www.ncbi.nlm.nih.gov/pubmed/14706098 • The characteristic inclusions of C. trachomatis are

• Recent reports have linked C. pneumoniae with athero- Halberstaedter- Prowazek or HP bodies.
• They may be demonstrated in conjunctival scrapings

sclerosis and its clinical effects like coronary, carotid •

and cerebral arterial disease. after staining with Giemsa, Casteneda or Macchiavello
• Studies of C. pneumoniae pathogenesis have shown

methods
that the organism can infect many cell types associated
47. Ans.  (c) Yolk sac inoculation
with both respiratory and cardiovascular sites, including
lung epithelium and resident alveolar macrophages, Ref: Ananthanarayan and Paniker’s Textbook of Microbiol-
circulating monocytes, arterial smooth muscle cells and ogy – 10th ed – Page 425
vascular endothelium. • Chlamydia can be isolated by inoculation into embryo-
• Infected cells have been shown to exhibit characteristics

nated eggs.
associated with the development of cardiovascular • Chlamydia can grow in the yolk sac of 6-8 day old chick

disease (e.g. secretion of proinflammatory cytokines embryos.


and procoagulants by infected endothelial cells and • The group specific antigen as well as the elementary and

foam cell formation by infected macrophages). inclusion bodies can be demonstrated in the yolk sac.
43. Ans.  (c) Urethritis 48. Ans.  (c) Psittacosis
Ref: Ananthanarayan and Paniker’s Textbook of Microbiol- Ref: Ananthanarayan and Paniker’s Textbook of Microbiol-
ogy – 10th ed – Page 427 ogy – 10th ed – Page 428
• In C. psittaci infection, infected cells, including alveolar
• Inclusion conjunctivitis

D, E, F, G, H, macrophages from patients, and mouse brain, yolk sac


C.trachomatis • Genital chlamydiasis


I, J, K and cell cultures show inclusion bodies (Levinthal-

• Infant pneumonia
Cole-Lillie or LCL bodies).

• Genital chlamydiasis manifests in men as urethritis


49. Ans.  (c) Lyme’s disease; (d) Tularemia

(non gonococcal urethritis), epididymitis, proctitis,


conjunctivitis and Reiter’s syndrome (Triad of recurrent Ref: Ananthanarayan and Paniker’s Textbook of Microbiol-
conjunctivitis, polyarthritis and cervicitis or urethritis) ogy – 10th ed – Page 386
• Women develop acute urethral syndrome, bartholinitis,

• Epidemic typhus and trench fever are louse borne;


mucopurulent cervicitis, endometritis, salpingitis, pelvic • Tick borne are lyme’s disease and Tularemia

inflammatory disease, conjunctivitis, perihepatitis (Fitz-


Hugh Curtis syndrome) and Reiter’s syndrome. 50. Ans.  (b) 1-3 weeks

44. Ans.  (d) Chlamydia Ref: Harrison’s T. B. of internal medicine – 19th edition –
page 1166
Ref: Ananthanarayan and Paniker’s Textbook of Microbiol-
• LGV – Lymphogranuloma venerum is caused by
ogy – 10th ed – Page 428

Chlamydia trachomatis – LGV serovars


• Currently, the Frei intradermal test is only of historical

• C. trachomatis strains are capable of causing chronic,


interest. clinically inapparent, asymptomatic infections.


• The test was based on a positive hypersensitivity to

• The duration of the chlamydial growth cycle is around


an intradermal standardized antigen obtained from 48–72 hrs


bubo pus, which indicated past or present chlamydial • Hence the incubation period of sexually transmitted

infection (lymphogranuloma venereum). chlamydial infections is relatively long—generally 1–3


weeks.
45. Ans.  (c) A polymerase chain reaction for nucleic acids
Ref: Ananthanarayan and Paniker’s Textbook of Microbiol- 51. Ans.  (a) Chlamydia trachomatis
ogy – 10th ed – Page 425 Ref: Harrison’s T. B. of internal medicine – 19th edition –
186 • A purulent vaginal discharge with dyauria following
• page 1166
unprotected sex can be due to gonococci / Chlamydia. For explanation please refer Q. 49.
Helicobacter and 21
Campylobacter
HELICOBACTER PYLORI y Humans are the primary reservoirs of H. pylori

y
y Transmission of infection occurs via feco oral route
y Gram-negative spiral rod

y
y It causes many diseases:
y
y It is motile bacilli: lophotrichous

y
 Gastritis
y
y Grown in chocolate agar under microaerophilic conditions


 Peptic ulcers
y
y It has characteristic feature of abundant production of


 Gastric adenocarcinomas
y
Urease.


 MALT B cell lymphomas


Remember Remember
Powerful Urease producing organism is Helicobacter pylori
Colonization of H. pylori has a protective role in following
followed by Proteus
disease:
• GERD
Pathogenesis

• Barrett’s Oesophagus

y The unique feature of H. pylori is that it has the ability to • Adenocarcinoma of Oesophagus and cardia
y

establish lifelong colonization in stomach of untreated
humans
y Factors that are responsible for colonization are:
y
 Bacteria has an inhibitor protein that inhibits the acid

production
 Neutralization of gastric acid occur because of urease

production that converts urea to ammonia and making
alkaline
 Localized tissue damage occurs due to urease end prod-

ucts like mucinase and phospholipase
y Virulence genes:
y
 vacA

 cagA

Figure 2: Effects of H.pylori colonisation
Laboratory Diagnosis
y H. pylori is detected by histologic examination of gastric
y
biopsy specimens
y Stain that can be used is Warthin-Starry silver stain (most
y
sensitive)
y Antigen detection for presence of urease activity
y
y Nucleic acid based tests (PCR)
y
y Important screening test is serology but almost 90% has
y
antibodies present in the body – it can be used only as
epidemiological tool

Figure 1: Pathogenesis of H. pylori


Table 1:  Tests for diagnosis of H. pylori y Primary sites of infection are jejunum and ileum
Unit 2     Bacteriology

 Antigenic cross reactivity occurs between surface




Invasive tests Biopsy urease test lipooligosaccharides of some strains of Campylobacter


Histology with silver stain and peripheral nerve ganglion leading to GBS.
Most specific is culture  Another immune-mediated late complication that may


Non invasive test C13 or C14 urea breath test is the most occur is reactive arthritis that occurs in HLA – B27
accurate test phenotype.
Stool antigen test
Serology Table 2:  Diseases caused by various species of
PCR Campylobacter
Species Disease
Treatment
C. jejuni • Gastroenteritis
y Regimen I – OCM (7-14 days) (or)

• Extra-intestinal infection
y

y Regimen II – OCA (7-14 days)


• Guillain Barre syndrome


y

{O – omeprazole; C – Clarithromycin; M – Metronidazole; A –


• Reactive arthritis
Amoxicillin)

• • Bacteremia
C. coli • Gastroenteritis
CAMPYLOBACTER

• Extraintestinal infections

y Gram negative, S or comma-shaped bacilli (Sea gull bacilli)


y C. fetus • Vascular infections

y Motile bacilli with polar flagellum – it also exhibits darting


y • Meningoencephalitis

motility (Ref: Greenwood) • Septic abortion


y Growth needs microaerophilic conditions


y

y Thermophilic bacteria - Growth at 42°C is characteristic of


y

Campylobacter
Laboratory Diagnosis
y Microscopy: Observation of characteristic, thin, S shaped or-
y

ganisms in stool specimen is useful in presumptive evidence


Pathogenesis of Campylobacter infection
y Campylobacter is the cause of bacterial gastroenteritis in
y
y Antigen detection by commercial immunoassay which has a
y

developed countries sensitivity of 80–90%


y Many different species are seen:
y
y Culture of Campylobacter needs some special media like
y

 C. jejuni

 Transport medium: Cary Blair transport medium


 C. coli

 Skirrow’s medium


 C. upsaliensis

 Butzler’s medium


 C. fetus

 Campy BAP selective medium


y It is a zoonotic infection which enters human body through


y

poultry products
y Infections are acquired by ingestion of contaminated food,
y
Treatment
unpasteurized milk or contaminated water y Fluid and replacement management; usually self-limiting;
y

y Person-to-person transmission is not common


y
y DOC in severe cases is erythromycin (5–7 days)
y

y Infectious dose is the most important factor in producing


y

infection

188
MULTIPLE CHOICE QUESTIONS

Chapter 21     Helicobacter and Campylobacter


1. Urease breath test is used for which organism: 5. Skirrow’s medium is used for – 
 (Recent Pattern 2018) (Recent pattern Aug 2013)
a. Haemophilus influenzae a. Clostridium tetani
b. Campylobacter jejuni b. Corynebacterium diphtheriae
c. Helicobacter pylori c. Campylobacter jejuni
d. Ureaplasma urealyticum d. Helicobacter pylori
2. Campylobacter culture media are –  6. Rapid urease positive test is done in – 
 (PGI May 2014)  (Recent pattern Nov 2013)
a. Schaedler’s agar b. CVA medium a. H. pylori
c. Regan – Lowe medium d. Skirrow medium b. S. aureus
e. Campylobacter blood agar c. Klebsiella
3. Regarding Campylobacter jejuni not true is –  (AI 2011) d. Staphylococcus
a. Most common cause of Campylobacteriosis 7. Non invasive test for H. pylori –  (PGI Nov 2014)
b. Poultry is the cause of the disease a. Rapid urease test
c. Human is the only reservoir b. Urease breath test
d. Causes Guillain barre syndrome c. Stool antigen assay
4. Seven sheathed flagella is seen in –  d. Stomach aspiration culture
 (Recent pattern Dec 2012 ) e. Biopsy
a. V. cholera b. H. pylori
c. Ps aeruginosa d. Spirochetes

ANSWERS AND EXPLANATIONS


1. Ans.  (c) Helicobacter pylori 4. Ans:  (b) H. pylori
Ref: Ananthanarayan and Paniker’s Textbook of Microbiol- Ref: Essentials of Medical Microbiology – 1st ed –page 786
ogy – 10th ed – Page 407 • H.pylori has five to seven sheathed polar flagella

Methods to detect Helicobacter pylori are: • Spirochetes is motile by endoflagella


• Gram staining – Curved GNB


• • Vibrio and Pseudomonas has single sheathed polar

• HPE with Warthin Starry silver stain


• flagellum
• Culture – microaerophilic in nature – grows in chocolate

agar under above conditions 5. Ans:  (c) Campylobacter jejuni; (d) Helicobacter pylori
• Rapid or biopsy urease test which helps to detect the

Ref: Ananthanarayan and Paniker T.B. of Microbiology – 9th


presence of urease in biopsy specimens by using a broth ed – page 399
that has urea sensitivite but invasive
• Non invasive tests are: Urea breath test which is the most

• Skirrow’s medium can be used for both Helicobacter

sensitive, quick and accurate method and it helps in the and Campylobacter
monitoring of treatment
6. Ans:  (a) Helicobacter pylori
2. Ans:  (b) CVA medium; (d) Skirrow medium Ref: Ananthanarayan and Paniker T.B. of Microbiology – 9th
(e) Campylobacter blood agar ed – page 400
Ref: Ananthanarayan and Paniker T.B. of Microbiology – 9th Diagnostic tests done for H.pylori are:
ed – page 399 Invasive tests Biopsy urease test
• Media used for Campylobacter are:
• Histology with silver stain
ƒ Skirrow medium
ƒ Most specific is culture
ƒ Butzler’s medium
ƒ

Noninvasive C13 or C14 urea breath test is the most


ƒ Columbia blood agar
ƒ

test accurate test


ƒ CVA agar
ƒ

Stool antigen test


Serology
3. Ans:  (c) Human is the only reservoir
PCR
Ref: Harrison T.B. of Internal Medicine 19th ed – page 1059
7. Ans:  (b) Urease breath test; (c) Stool antigen assay
• Campylobacter jejuni is a zoonotic infection

• Animals are the main reservoir of C.jejuni • Already explained Q.6



189
22 Mycoplasma and
Legionella
MYCOPLASMA
y Smallest free living bacteria
Remember
Causes of Primary atypical pneumonia are:
y
y Unique feature: They do not have a cell wall
• Mycoplasma pneumoniae
y
y There are several species under Mycoplasma namely:


• Chlamydia pneumoniae
y
 Mycoplasma pneumoniae


• Legionella pneumophila

 Mycoplasma genitalium



 Mycoplasma hominis

 Ureaplasma urealyticum
Remember

y They can be filterable by bacterial filters
y
y They grow on artificial cell-free media Urogenital Mycoplasmas:
y
y They are facultative anerobic – exception is M. pneumoniae • M. hominis
y

which is a strict aerobe • M. genitalium

• U. urealyticum
Pathogenesis and Clinical Features

• U. parvum

Table 1: Diseases caused by Mycoplasma species

Organism Site of Disease that are caused Laboratory Diagnosis
infection y Organism does not have cell wall: cannot be seen with Gram
y
M. pneumoniae Respiratory • Pharyngitis stain
y Culture needs PPLO broth: growth is very slow

infection • Tracheobronchitis
y
y It gives typical fried egg appearance: Agar is cut and placed

• Pneumonia
y
on a slide – then colonies are seen with the help of hand lens

• Arthritis
– this is called as Dienes method

• Pericarditis
y Other test used is tetrazolium reduction test

• Hemolytic anemia
y
y PCR has excellent sensitivity

• GBS
y
y Serological tests:

• Skin lesions
y
 Complement fixation test

• Encephalitis

 EIA

M. genitalium Genito • Non gonococcal urethritis

 Cold agglutination test

urinary (NGU)

 Streptococcus MG test
infection • PID


M. hominis Both • Pyelonephritis Table 2: Sensitivity and specificity of the test done in


respiratory • Postpartum fever Mycoplasma infection

and • Systemic infections
Tests done Sensitivity Specificity

genitourinary
infection Culture < 60% 100%
U. urealyticum Both • NGU PCR 65 – 90% 90 – 100%

• Pyelonephritis Serology 55 – 100% 55 – 100%

• Spontaneous abortion

• Premature birth
Treatment

y M. pneumoniae is also called Eaton agent (discovered by y Mycoplasma pneumoniae has no cell wall – so cell wall
y
Eaton)
y
inhibitors are not used
y Pneumonia is caused by M. pneumoniae is also called: y DOC is macrolides and FQs, doxycycline
y
 Eaton agent pneumonia
y
y For NGU - doxycycline

 Primary atypical pneumonia
y

 Walking pneumonia

LEGIONELLA Table 3:  Comparison between Pontiac Fever and

Chapter 22     Mycoplasma and Legionella


y Gram-negative rod, slender and pleomorphic, Noncapsulated
y
Legionnaires disease
– motile by polar or subpolar flagella Pontiac fever Legionnaires disease
y Many species are there namely:
• Mild, acute and self • Incubation period is 2-10 days
y

 Legionella pneumophila
• •

limiting condition • The symptoms are usually




 Legionella micdadei

• Flu like illness atypical – hence called atypical




y Appears as coccobacilli •

• No pneumonia pneumonia
y

y It needs special media for growth •

• Fever, myalgia, head ache • It also causes diarrhea


y

y It is a facultative intracellular bacteria • •

occurs • Relative bradycardia is seen


y

Legionella micdadei:
• Acid-fast organism

Laboratory Diagnosis
• Called as Pittsburgh pneumonia agent

y Definitive diagnosis for Legionella is isolation of organism
y

from respiratory secretions (most sensitive and specific)


y Media used for culture is BCYE agar
Clinical Features
y

y Direct fluorescent antibody test can be done


y

y Legionella enters the lungs through aspiration or direct


y
y ELISA can be done
y

inhalation y Urinary antigen detection for Legionella by rapid tests are


y

y Source of infection is mainly cooling towers, air conditioners


y
available
y It causes:
Treatment
y

 Pontiac fever


 Legionnaires’ disease (Pneumonia)



y DOC are macrolides and quinolones.
y

191
MULTIPLE CHOICE QUESTIONS
Unit 2     Bacteriology

Mycoplasma 10. Not true about Legionella –  (Recent Pattern July 2016)
a. Grows readily in simple culture media
1. About mycoplasma true statement is: 
b. Motile by polar flagella
(Recent Pattern June 2014)
c. Most important species is L.pneumophila


a. Acid fast b. Commensal in mouth


c. Beta hemolysis d. Sporing d. BCYE is selective media
2. Fried egg colony is seen in culture of:  11. Legionnaire’s disease is caused by –
 (Recent Pattern Dec 2013) a. Motile gram positive  (Recent Pattern Dec 2012)
a. Mycoplasma b. Trachoma b. Motile gram negative
c. Legionella d. Hemophilus c. Nonmotile gram positive
3. Dienes stain is used for:  (Recent Pattern July 2015) d. Nonmotile gram negative
a. Camplyobacter b. Rickettsiae 12. True about Legionella – (Recent Pattern Dec 2016 )
c. Helicobacter d. Mycoplasma a. Most common mode of transmission is aerosol inhala-
4. True about mycoplasma:  (Recent Pattern 2012) tion
a. Obligate intracellular organism b. There is no man to man transmission
b. Penicillin is effective treatment c. Prolonged carrier are common
c. Requires cholesterol for growth d. All are true
d. Have thick cell wall 13. Legionella causes –  (Recent Pattern July 2016)
5. Ureaplasma is naturally resistant to:  a. Pontiac fever b. Myocarditis
 (Recent Pattern 2012) c. Diarrhea d. All of the above
a. Tetracycline b. Erythromycin 14. Pontiac fever is caused by – 
c. Cephalosporins d. Chloramphenicol  (Recent Pattern Dec 2012, Nov 2011)
6. True about mycoplasma:  (Recent Pattern 2012) a. Legionella b. Listeria
a. Has cell wall b. Cause NGU c. Scrub typhus d. Leptospira
c. Multidrug resistant d. Penicillin effective 15. In Pontiac fever, which antigen is seen in urine – 
7. Pathogenic mycoplasma is:  (Recent Pattern Dec 2013)  (Recent Pattern Dec 2015)
a. M. hominis b. M. penetrans a. Lipopolysaccharide – 1 b. Lipopolysaccharide - 2
c. M. primatum d. M. faucium c. Lipopolysaccharide - 4 d. Lipopolysaccharide – 6
8. Mycoplasma is intrinsically resistant to: 16. In Pontiac fever , which antigen is seen in urine – 
 (Recent Pattern 2018)  (Recent Pattern Dec 2015)
a. Aminoglycosides b. Ceftazidime a. Group specific antigen of Legionella serogroup
c. Macrolides d. Doxycycline – 1 (LP1)
b. Group specific antigen of Legionella serogroup
Legionella – 2 (LP2)
9. Which of the following is a method for acquiring infec- c. Group specific antigen of Legionella serogroup
tion by Legionella pneumophila –  (AIIMS May 2013) – 4 (LP4)
a. Consumption of contaminated water with presence of d. Group specific antigen of Legionella serogroup
Legionella – 6 (LP6)
b. Breathing of aerosols of environmental water sources 17. An elderly male patient presented with fever, chest
such as air conducting units pain and dry cough, sputum cultured on charcoal yeast
c. Consumption of contaminated meat and meat prod- medium, the organism is –  (AIIMS Nov 2011)
ucts a. H. influenzae b. Moraxella catarrhalis
d. Close contact with carriers of Legionella c. Legionella d. Burkholderia cepacia

ANSWERS AND EXPLANATIONS


1. Ans.  (b) Commensal in mouth 2. Ans.  (a) Mycoplasma
Ref: Ananthanarayan and Paniker’s Textbook of Microbiol- Ref: Ananthanarayan and Paniker’s Textbook of Microbiol-
ogy – 10th ed – Page 393 ogy – 10th ed – Page 393
• Mycoplasma can live independently with no cell wall.
• • Mycoplasma can be cultivated in fluid or solid media.

• They are the smallest prokaryotic microbial cells and can


• • On solid media, colonies appear after incubation for 2-6

be isolated from normal human and animal respiratory days and about 10-600 µm in size. The colony is typically
mucosa. biphasic, with a fried egg appearance.
192 • In the oral cavity, M. pneumoniae, M. oralis, M. salivalis,

and M. nominis can be isolated.


3. Ans.  (d) Mycoplasma 9. Ans.  (b) Breathing of aerosols of environmental water

Chapter 22     Mycoplasma and Legionella


sources such as air conducting units
Ref: Ananthanarayan and Paniker’s Textbook of Microbiol-
ogy – 10th ed – Page 394 Ref: Ananthanarayan and Paniker T.B. of Microbiology – 9th
• Colonies of mycoplasma are best studies after staining

ed- page 401
by Dienes method. Modes of transmission of Legionella are:
• A block of agar, containing the colony is cut and placed
• • Inhalation of droplets from aerosolized water

on a slide. It is covered with a cover slpi on which an • Aspiration


alcoholic solution of methylene blue and azure has been


dried. 10. Ans.  (a) Grows readily in simple culture media
Ref: Ananthanarayan and Paniker T.B. of Microbiology – 9th
4. Ans.  (c) Requires cholesterol for growth
ed- page 401
Ref: Ananthanarayan and Paniker’s Textbook of Microbiol- • Legionella is a fastidious organism which grown on

ogy – 10th ed – Page 393 special media like BCYE agar


• The cells of mycoplasma are bound by a soft trilaminar
• • It is a motile organism

membrane containing sterols. • Most common species is Legionella pneumophila


• They require cholesterol for growth. Media for growth


of mycoplasma are enriched with 20% horse or human 11. Ans.  (b) Motile gram negative
serum. Ref: Ananthanarayan and Paniker T.B. of Microbiology – 9th
• The high concentration of serum acts as a source of

ed- page 401


cholesterol and lipids.
• Legionnaire’s disease is caused by L.pneumophila

5. Ans.  (c) Cephalosporins • L.pneumophila is a gram negative coccobacilli


• It is motile by polar flagellum


Ref: Ananthanarayan and Paniker’s Textbook of Microbiol-


ogy – 10th ed – Page 394 12. Ans.  (b) There is no man to man transmission
• Ureaplasma is devoid of cell wall.

Ref: Harrison’s T.B. of Medicine – 19th ed – page 1016


• Cephalosporins are antibiotics that act on cell wall. So
• Most common mode of transmission is aspiration

Ureaplasma is intrinsically resistant to cephalosporins •

and other antibiotics that act on the cell wall. • Man to man transmission never occurs

6. Ans.  (b) Cause NGU 13. Ans.  (d) All of the above

Ref: Ananthanarayan and Paniker’s Textbook of Microbiol- Ref: Harrisons T.B. of Medicine – 18th ed – page 1238
ogy – 10th ed – Page 396 • Legionella causes:

ƒ Pulmonary infections like Pontiac fever and Legion-


• Mycoplasma can cause genital infections.

ƒ

• After Chlamydia trachomatis, the T strains of myco-



naire’s disease
plasma (Ureaplasma urealyticum) are the second most ƒ Extrapulmonary infections like myocarditis, pericar-
ƒ

common cause of non gonococcal urethritis (NGU). ditis and endocarditis

7. Ans.  (a) M. hominis 14. Ans.  (a) Legionella


• Already explained Q.13

Ref: Ananthanarayan and Paniker’s Textbook of Microbiol-


ogy – 10th ed – Page 393 15. Ans:  (a) Lipopolysaccharide – 1
• M. hominis amd M. gentalium cause genital tract infec-
Ref: Jawetz – Medical Microbiology – 27th edition

tions and are pathogenic.


• M. pneumoniae causes pneumonia.

• Legionella is classified based on the group specific

• U. urealyticum causes non gonococcal urethritis (NGU). lipopolysaccharide


• LP-1 is the most common infecting organism – in urine

8. Ans.  (b) Ceftazidime LP-1 can be detected

Ref: Jawetz T.B of medical microbiology – 27th edition – 16. Ans.  (a) Group specific antigen of Legionella serogroup
Page 31 – 1 (LP1)
• Mycoplasma are cell wall–lacking bacteria containing

• Already explained Q.15

no peptidoglycan
• Mycoplasma lack a target for cell wall–inhibiting antimi-
• 17. Ans.  (c) Legionella
crobial agents (e.g. penicillins and cephalosporins) and
Ref: Harrison’s T.B. of Medicine 19th edition – page 1016
are therefore resistant to these drugs
• DOC for Mycoplasma is Macrolides • Charcoal agar culture clearly indicates the organism is

193

Legionella
• Selective medium for Legionella is BCYE agar

23
Miscellaneous Bacteria
LISTERIA MONOCYTOGENES STREPTOBACILLUS
y Gram-positive bacillus, nonsporing, and motile Table 2:  Differences between streptobacillary rat bite
y
y Characteristically motile at 25°C but nonmotile at 37°C -
fever and spirillary rate bite fever
y
exhibits tumbling motility
y It is facultative intracellular pathogen Streptobacillary rat bite Spirillary rat bite fever
y
y It shows beta hemolysis in sheep blood agar fever
y
y Infection is acquired by consumption of undercooked meat
Streptobacillus moniliformis Spirillum minus
y
y Clinically, it presents disease mainly in neonatal age
Also called as Haverhill fever Called sudoku
y
y Listeriolysin O – Important virulence factor
y
y Culture media: Tryptose phosphate, Thioglycollate broth Gram negative, highly Gram negative, spirally coiled,
y
(4°C) - cold enrichment: PALCAM agar – selective medium pleomorphic, nonmotile motile
for Listeria.
Cultivable by artificial media Non cultivable

Clinical Features KLEBSIELLA GRANULOMATIS


Table 1:  Clinical features of listeriosis
y Earlier called as Calymmatobacterium granulomatis
y
Early onset neonatal Late onset neonatal y Gram negative rod
y
listeriosis listeriosis y It causes a veneral disease named as donovanosis or granu-
y
loma inguinale
Presents within 5 days of birth More than five days of birth
y Lab diagnosis: Demonstration of donovan bodies in Giemsa
y
Infection is acquired from Acquired from environment stain from lesions in the genital area
maternal genital flora y It has characteristic safety pin appearance
y
Most common is neonatal Most common is neonatal y DOC: Tetracycline
y
sepsis meningitis
Mortality rate is higher Mortality rate is lower HACEK BACTERIA
It does not cause nosocomial Nosocomial outbreaks are seen y This group of organism usually reside in the oral cavity and
y
outbreaks causes infections of mouth
y They cause bacterial endocarditis in native valve or
Miliary microabscesses – Not seen
y
prosthetic valves
Granulomatosis infantiseptica
y Eikinella corrodens is associated with infections following
y
y In adults it can cause UTI, meningitis and bacteremia human bites
y
H Haemophilus sp.,
Laboratory Diagnosis
A Aggregatibacter
y Microscopy is not useful in diagnosis
C Cardiobacterium hominis
y
y Culture of specimens with cold enrichment media helps in
E Eikenella corrodens
y
diagnosis
y It gives positive CAMP reaction K Kingella kingae
y
y Characteristically motile at 25°C but nonmotile at 37°C-ex-
y
hibits tumbling motility ACTINOMYCES
y Close DD : Corynebacterium during lab diagnosis.
y Species comes under Actinomyces are A. israelii, A. odonto-
y
y
lyticus, A. viscosus etc.,
Treatment y It causes actinomycosis, which is usually a polymicrobial
y
y Ampicillin, Cotrimoxazole and Gentamicin; Cephalosporins condition
y
–not recommended y It is a slowly progressive lesion that usually presents as mass
y
lesion in mucosal areas
y Growth of actinomycetes in the body characteristically Treatment

Chapter 23     Miscellaneous Bacteria


y

produces grains or sulfur granules, which helps in diagnosis


y Penicillin, macrolides, doxycycline
y Classical triad:
y

 Chronic lesions with mass


NOCARDIA


 Sinus tract which shows sulfur granules




 Relapsing in nature after a short course of therapy




y The fibrotic walls of the mass lesions is characteristically


y
y y Gram positive filamentous bacilli – causes mycetoma
woody in nature y y It is an acid fast bacilli - with 1% H2SO4
y y It is cultivable in LJ media
y Most common species causing human disease – N.asteroides
Clinical Manifestations y

y y Pneumonia is the m/c manifestation


y y Oral – cervicofacial actinomycosis y Extrapulmonary – m/c – brain abscess
y Thoracic diseases
y

y Organism is resistant to peicillin


y

y Abdominal actinomycosis
y

y DOC: TMP-SMX
y

y Pelvic actinomyosis – typically occurs in IUCD users


y

Laboratory Diagnosis
y Microscopic identification of sulfur granules in pus or in tis-
y

sues
y FDG – PET – hypermetabolism
y

y Culture needs anaerobic conditions


y

Figure 2: Acid fast staining showing Nocardia spp

TROPHERYMA WHIPPLEI
y y Causes Whipple’s disease
Figure 1: HPE of Actinomycosis (Courtesy: Dr S. Jamuna Rani, y y It is a Gram positive bacilli
MD (Pathology), Associate professor, Tagore Medical College and y y It has a tropism for myeloid cells
Hospital, Chennai) y y It can cause Whipple’s disease, Behcet syndrome, IRIS

NONSPORING ANAEROBES

Table 3:  Nonsporing anaerobes


Gram-positive cocci Gram-negative Gram-positive Gram-negative bacilli
cocci bacilli
Peptostreptococcus Veillonella • • Eubacterium • • Bacteroides
Peptococcus • • Propionibacterium • • Prevotella
• • Lactobacillus • • Porphyromonas
• • Mobiluncus • • Fusobacterium
• • Bifidobacterium • • Leptotrichia
• • Actinomyces
• • Clostridium (endospore)

195
Table 4:  Important points to be remembered
Unit 2     Bacteriology

Organisms Features
Lactobacillus • Normally present in mouth, adult vagina

• It is called as Doderlein’s bacilli


• It is nonpathogenic

Mobiluncus • Causes bacterial vaginosis along with


Gardnerella vaginalis
Bacteroides • Most common anaerobic infection

• Normally seen in intestinal tract and


Figure 3: Clue cells in a vaginal discharge

female genital tracts


(Courtesy: Dr A.V.M. Balaji, Sr. Assistant Professor of Microbiology,
• Causes brain abscess, cellulitis and

Stanley Medical College, Chennai)


abdominal abscess and other infections
• Treatment: Metronidazole

y Other methods are: Amsel criteria and Nugents criteria
y

y Amsel criteria are as follows: (At least presence of three out


y

Porphyromonas • Periodontal disease



of these four criteria is enough to make the diagnosis of BV)
• Gingivitis

 Homogenous vaginal discharge


 Whiff test positive


Prevotella • It produces colored colonies 

 Clue cells

• Causes wound infections 

 Vaginal pH > 4.5


Fusobacterium • Liver abscess



y Nugents scoring is by observing the following counts in
y

necroporum microscopy and making the score interpretation:


Leptotrichia • Vincent’s angina

Table 5:  Nugent scoring


Lacto- Score Gardnerella, Score Curved Score Sum
BACTERIAL VAGINOSIS bacilli Bacteroides GNB

y Bacterial vaginosis (BV) is caused by abnormal growth of


y
≥ 30 0 0 0 0 0 0
bacteria in the vaginal flora with the replacement of normal 5 – 30 1 <1 1 <1 1 3
indigenous microbial flora. 1–4 2 1–4 2 1–4 1 5
y Predisposing factors for the bacterial vaginosis are having
<1 3 5 – 30 3 5 – 30 2 8
y

multiple sexual partners, when having a new sexual partner


or when doing repeated douching. 0 4 ≥ 30 4 ≥ 30 2 10
y Clinical presentations are homogenous thin vaginal discharge
y

Table 6:  Interpretation of Nugent Score


with fishy odor and lower abdomen pain.
y Causative agents for bacterial vaginosis are:
y
Score (N) Additional criteria Report
 Gardnerella vaginalis

0–3 Smear not consistent
 Mobiluncus spp.,


4–6 When clue cells are not with Bacterial


 Peptostreptococci


present vaginosis
 Prevotella
4–6 When clue cells are present Smear consistent with


 Porphyromonas.
Bacterial vaginosis


≥7
Laboratory Diagnosis
y Smear from vaginal discharge usually reveals clue cells, which
y
Treatment
are the vaginal epithelial cells studded with coccobacilli. The drug regimen for treating BV is oral metronidazole 500 mg
twice a day for a period of 7 days.

196
MULTIPLE CHOICE QUESTIONS

Chapter 23     Miscellaneous Bacteria


1. Anaerobic Gram-positive bacillus: 6. With reference to Bacteroides fragilis the following
 (Recent Pattern June 2014) statements are true, except:  (AIIMS Nov 2011, 2012)
a. Bacteroides a. B. fragilis is not uniformly sensitive to metronidazole
b. Propionibacterium b. B. fragilis is the same frequent anaerobe isolated from
c. Fusobacterium clinical samples
d. Leptotrichia c. Shock and disseminated intravascular coagulation are
2. All are non–sporing anaerobes of medical importance, common in Bacteroides bacteremia
except:  (Recent Pattern Dec 2013) d. The lipopolysaccharide formed by B. fragilis is struc-
a. Clostridia turally and functionally different from the conventional
b. Actinomyces endotoxin
c. Bacteroides 7. Bacteroides are susceptible to: 
d. Fusobacterium  (Recent Pattern July 2016)
3. Ideal urine specimen for anaerobic culture should be: a. Ceftriaxone b. Erythromycin
 (Recent Pattern Nov 2015) c. Metronidazole d. Sulfonamides
a. First few drops at morning 8. True about Bacteroides fragilis is: 
b. Mid stream urine sample  (Recent Pattern July 2016)
c. Sample by Foley’s catheter a. Produce endotoxin similar to other Gram-negative
d. Sample by suprapubic aspiration bacteria
4. All are true about anaerobic infection, except:  b. Normal stool sample contains 105 organisms per gram
 (Recent Pattern Dec 2016) c. Most important virulence factor is capsular polysac-
a. Exudates and swabs are best for culture charide
b. Most infections are endogenous d. All of the above are correct
c. Specimen for UTI is suprapubic aspiration 9. True about Actinomycosis: (PGI May 2017)
d. They are normally found on skin and GIT a. Caused by Madurella mycetomatis
5. Best specimen for anaerobic culture: b. Caused by anaerobic or microaerophilic bacteria
 (Recent Pattern Dec 2015) c. Cervicofacial is the most common site affected
a. Pus aspirated in vial d. Sulphur granules are present in lesion
b. Exudates from wound e. Actinomyces is non-acid fast
c. Swab from wound 10. Not true about Donovanosis: (PGI May 2017)
d. Mid – stream urine a. Caused by Klebsiella granulomatis
b. Associated with pseudobuboes
c. Caused by Leishmania donovani
d. Drug of choice is Miltefosine
e. Drug of choice is sodium stibogluconate

ANSWERS AND EXPLANATIONS


1. Ans.  (b) Propionibacterium 3. Ans.  (d) Sample by suprapubic aspiration
Ref: Ananthanarayan and Paniker’s Textbook of Microbiol- Ref: Ananthanarayan and Paniker’s Textbook of Microbiol-
ogy – 10th ed – Page 273 ogy – 10th ed – Page 276
• Anaerobic gram-positive bacteria are
• • Only specimens collected by aspiration are acceptable

ƒ Clostridium (spore forming)


ƒ for performing anaerobic culture.
ƒ Eubacterium
ƒ • Suprapubic aspirate is the only urine specimen suitable

ƒ Propionibacterium
ƒ for anaerobic culture.
ƒ Lactobacillus Non-sporing anaerobes
ƒ

ƒ Mobiluncus
ƒ
4. Ans.  (a) Exudates and swabs are best for culture
ƒ Bifidobacterium
ƒ

Ref: Ananthanarayan and Paniker’s Textbook of Microbiol-


ƒ Actinomyces
ƒ

ogy – 10th ed – Page 276


2. Ans.  (a) Clostridia • Only specimens that collected by aspiration are

acceptable.
Ref: Ananthanarayan and Paniker’s Textbook of Microbiol- • Swabs are highly unsatisfactory. But when they are to be

ogy – 10th ed – Page 273 used, they should be sent in RCM or Stuart’s transport
• Clostridium produces spores. All others given as options
• medium.
are nonsporing 197
5. Ans.  (a) Pus aspirated in vial • The endotoxin of B. fragilis is different from that of other
Unit 2     Bacteriology

gram negative bacilli.


Ref: Ananthanarayan and Paniker’s Textbook of Microbiol-
ogy – 10th ed – Page 276 9. Ans.  (b) Caused by anaerobic or microaerophilic
• Pus and other fluids for anaerobic culture may be
• bacteria; (c) Cervicofacial is the most common site
collected in sealed vials gassed out with carbondioxide, affected; (d) Sulphur granules are present in lesion; (e)
or in small bottles with air tight caps filled to capacity to Actinomyces is non-acid fast
remove air and transported quickly.
Ref: Ananthanarayan and Paniker’s Textbook of Microbiol-
6. Ans.  (a) B. fragilis is not uniformly sensitive to ogy – 10th ed – Page 399
metronidazole • Species comes under Actinomyces are A. israelii, A.

odontolyticus, A. viscosus etc.,


Ref: Ananthanarayan and Paniker’s Textbook of Microbiol-
• It causes actinomycosis, which is usually a polymicrobial
ogy – 10th ed – Page 275

condition
• B. fragilis is universally susceptible to metronidazole

• It is a slowly progressive lesion that usually presents as

and so susceptibility testing is not carried out for mass lesion in mucosal areas; most common form is
metronidazole. cervico facial actinomycosis; it is non acid fast
• B. fragilis is not susceptible to penicillin.

10. Ans.  (c) Caused by Leishmania donovani; (d) Drug


7. Ans.  (c) Metronidazole of choice is Miltefosine; (e) Drug of choice is sodium
Ref: Ananthanarayan and Paniker’s Textbook of Microbiol- stibogluconate
ogy – 10th ed – Page 275 Ref: Ananthanarayan and Paniker’s Textbook of Microbiol-
• The bacteroides species is susceptible to metronidazole

ogy – 10th ed – Page 404
and usually to clindamycin and chloramphenicol. • Earlier called as Calymmatobacterium granulomatis

• B. melaninogenicus is suscepticle to penicillin B. fragilis



• Gram shaped negative rod

is not susceptible. • It causes a veneral disease named as donovanosis or


granuloma inguinale
8. Ans.  (c) Most important virulence factor is capsular • Lab diagnosis: Demonstration of donovan bodies in
polysaccharide

Giemsa stain from lesions in the genital area


Ref: Ananthanarayan and Paniker’s Textbook of Microbiol- • It has characteristic safety pin appearance DOC:

ogy – 10th ed – Page 275 Tetracycline


• Capsular polysaccharide is the most important virulence

factor.

198
HIGH YIELDING FACTS TO BE REMEMBERED IN BACTERIOLOGY

Chapter 23     Miscellaneous Bacteria


Catalase positive and negative organisms Catalase positive: Staphylococcus and Micrococcus; Catalase negative:
Streptococcus, Pneumococcus, and Enterococcus
Surrogate marker for identifying MRSA Cefoxitin disc
Most common cause of native valve endocarditis, Native valve endocarditis – S. aureus; Prosthetic valve – S. epidermidis; i.v
prosthetic valve, endocarditis in i.v. drug users and drug users – S. aureus; Subacute endocarditis – Viridans streptococci
subacute endocarditis
Treatment for MRSA Vancomycin is the DOC; Others given are Teicoplanin, linezolid, quinupristin-
dalfopristin, tigecycline. (for VRSA) Daptomycin is given for endocarditis and
complicated skin infections (for VRSA)
Typing of Streptococcus – methods? Lancefield’s sero grouping: based on carbohydrate antigen; Serotyping
based on M protein: Griffith typing
Antigenic cross-reaction among streptococcal antigens Cell wall M protein – Myocardium; Cell wall C carbohydrate – cardiac valves;
and human antigens? Cytoplasmic membrane – Glomerular vascular intima ; Peptidoglycan – skin
antigens; Hyaluronic acid – Synovial fluid
Strains causing acute rheumatic fever and post ARF – most of the strains of S. pyogenes; PSGN – Pyodermal strains – 49,53-
streptococcal glomerulonephritis? 55, 59-61; Pharyngitis strains – 1,12
Use of serology in diagnosing Streptococcal infections? ASO antibodies - > 200 todd units – diagnostic of most of the streptococcal
infections except pyoderma and PSGN, Where antiDNase B Ab titer is
helpful
Features of GBS? (Group B Streptococci) S. agalactiae, CAMP test positive, Most common cause of neonatal sepsis
and meningitis due to colonization in maternal vagina
Waterhouse Friderichsen syndrome? Seen in meninogococcemia; Severe form of fulminant meningococcemia –
large purpuric rashes, shock, DIC, bilateral adrenal hemorrhage and multi
organ failure
Vaccination of N. meningitidis Polyvalent vaccine – A,C,Y, W135; no vaccine for Group B meningococcus
Watercan perineum is seen in Gonorrhea; In females it can cause Fitz Hugh Curtis syndrome
Agents causing non-gonococccal urethritis Chlamydia trachomatis (M/c), Ureaplasma urealyticum, M. hominis, Herpes
virus, Cytomegalovirus, Candida albicans, T. vaginalis
DOC for carriers of Meningococcus Cefriaxone (DOC), Rifampicin and Ciprofloxacin
Metachromatic granules or Volutin granules are seen in: C. diptheriae, C. xerosis, Gard. vaginalis, Spirillium, Enterobacter aerogenes
Which strain is used for toxin production in Diphtheria Park William 8 Strain
vaccine
Most common form of Diphtheria and other clinical Respiratory: Faucial diphtheria (Mc) that leads to Pseudomembrane
types formation, Cutaneous type, Systemic complication which include
mycocarditis and neurological symptoms
Diagnosis of Diphtheria Albert stain – to see metachromatic granules; Loeffler’s serum slope to
isolate C. diptheriae. Potassium tellurite agar is also helpful;
Toxin demonstration of Diphtheria is by: In vivo – Guinea pig inoculation; In vitro – Elek’s gel precipitation test,
Detection of tox gene by PCR, ELISA or ICT tests
Name few Diphtheroids and its importance C. ulcerans: Respiratory diphtheria; C. pseudotuberculosis – Preisz-Nocard
bacilli; C. minutissimum: Erythrasma (Coral red – Woods’ lamp); C. jeikeium
– MDR species; C. parvum - immunomodulator
Obligate aerobe with non bulging spore is: Bacillus
Obligate aerobe with bulging spores is: Clostridium
Types of human anthrax M/c - Cutaneous anthrax – Hide Porter’s disease (Malignant pustule);
Pulmonary anthrax – Wool Sorter disease (Hemorrhagic pneumonia);
Intestinal anthrax
199
Unit 2     Bacteriology

Special features of B. anthracis Gram staining – Bamboo-stick appearance, Medusa head appearance in
colony, Gelatin stab – Inverted fir tree, String of pearl appearance in solid
media with penicillin
Media for B. cereus MYPA agar (Mannitol, Egg yolk, Phenol red, Polymyxin), PEMBA agar
Emetic type of B. cereus – features: Preformed toxin – IP 1 – 6 hours; MC food – Chinese fried rice
Location of Spores in Clostridium sp Clostridium tetani: Spherical and terminal spore (Drumstick appearance); Cl.
tertium – Oval and terminal spore (Tennis racket appearance);
C. bifermentans – Central and oval spore
Agents of Gas gangrene Cl. perfringens, Cl. novyi, Cl. septicum, C. histolyticum, Cl. sporogenes, C.
fallax
Action of Tetanus toxin is mediated by: Tetanospasmin – acts presynaptically at the inhibitory neuron terminals and
prevents release of GABA and glycine – spastic muscle contraction
Toxins of Botulinum and its MOA: A,B,C1,C2,D,E,F and G – all are neurotoxins except C2 which produces
enterotoxins; blocks the release of Ach in NM junction – flaccid paralysis
Agent of Pseudomembranous colitis and treatment Clostridium difficile; Prolonged usage of antimicrobial drugs Ceftriaxone -
Treatment is Vancomycin (DOC)
Agent of Vincent’s angina and Lemierre’s syndrome? Vincent’s angina – Fusobacterium fusiformis and Borrelia vincenti;
Lemierre’s syndrome – Fusobacterium necrophorum
Acid fastness of MTB is due to: Mycolic acids in the cell wall
Concentration methods of sputum Petroff’s method by 4% NaOH, N-acetyl-L-Cysteine + 2% NaOH
Media for MTB and how to sterilize it: EGG based media – LJ media, Dorset egg media, Petragnani media; As it
contains egg, it should be sterilized by inspissation
Diagnosis of Extrapulmonary TB Molecular methods; Elevated ADA in pleural fluid; CSF – cobweb coagulum
Interpretation of tuberculin test ≥10 mm – Positive ; 6-9 mm – Equivocal; < 5 mm - Negative
What is IGRA Interferon Gamma Release Assay – sensitized T lymphocytes of suspected
individuals – when exposed to highly specific M.tb antigen such as CFP10
and ESAT6 - releases high level of IFN gamma (QuantiFERON-TB Gold assay)
Drug susceptibility testing methods for M.tb Phenotypic methods: Proportion method – gold standard; Absolute
concentration method; Resistance ratio method
MDR TB and XDR TB - Definition MDR TB – Resistance to Isoniazid and Rifampicin with or without resistance
to first line drugs; XDR –TB – MDR TB + Resistant to Fluoroquinolones and at
least one injectable aminoglycosides
Types of leprosy Lepromatous, Tuberculoid, Indeterminate, Borderline and Pure neuritic type
Lepra reactions: Type I – DTH type IV hypersensitivity and Type II – Type III immune complex
mediated; Treatment for Type 1 is glucocorticoids ; whereas type 2 is
Steroids, Thalidomide and Clofazimine
Uses of lepromin test Does not diagnose leprosy; used to assess the CMI; helps in classifying the
lesions; assessing prognosis; assessing resistance to leprosy
Treatment of paucibacillary and multibacillary leprosy Paucibacillary – Dapsone 100 mg daily plus Rifampicin 600 mg given once
a month under supervision; Multibacillary – Dapsone 100 mg daily +
Rifampicin 600 mg given once a month + Clofazimine 300 mg once a month
followed by 50 mg daily
Cause of swimming pool granuloma and Buruli ulcer Swimming pool granuloma – Myco. marinum; Buruli ulcer – M. ulcerans
Examples for Lactose and Non lactose fermenters LF – Escherichia, Klebsiella, Citrobacter; NLF – Salmonella, Shigella, Proteus,
Morganella, Providencia, Yersinia
Serotypes of UPEC (Uropathogenic E. coli) responsible for O1, O2, O4, O6, O7 and O75
UTI’s
Kass concept for significant bacteriuria ≥105 CFU/ml of urine
Ideal culture media for UTI CLED is better than Mac
200
Chapter 23     Miscellaneous Bacteria
Types of diarrheagenic E. coli EPEC (pathogenic), ETEC (toxigenic), EIEC (invasive), EHEC (hemorrhagic),
EAEC (aggregative)
Organisms that causes swarming Proteus, Clostridium tetani, Vibrio parahemolyticus, Serratia
Most common Shigella species in world and in India In world – S. sonnei
In India – S. flexneri
Infective doses of Shigella, Salmonella and V. cholerae Shigella – 10 to 100 bacilli, Salmonella - 103 to 106 and V. cholerae 106 to 108
Pigment produced by Serratia Prodigiosin – red coloured – Pseudohemoptysis
Vector for Plague Xenopsylla cheopis (north India) – most efficient vector; Xenopsylla astia
(south India)
Most common type of Plague Bubonic plague (Inguinal Lymph Node is the M/c)
Mesenteric adenitis and Terminal ileitis in children is Mesenteric adenitis - Yersinia pseudotuberculosis and Terminal ileitis -
caused by Y. enterocolitica
Diagnosis of enteric fever BASU – Blood culture (1st week); Antibody titre by Widal (2nd week); Stool
and Urine culture (3-4 weeks)
Diagnosis of Salmonella carriers Stool and Urine culture; Serum for detection of antibodies to Vi antigen;
Sewage culture
MDR Salmonella Resistant to Chloramphenicol, Ampicillin, Cotrimaxazole; DOC –Ciprofloxacin
Typhoral vaccine Live attenuated vaccine; After 6 years of age; Given as enteric coated
capsules; Booster given every three years
Cause of Gastroenteritis by NTS (Non typhoidal Salmonella gastroenteritis; Also NTS causes osteomyelitis which is common
salmonella) in sickle cell anemia patients
Name few examples for nonhalophilic and halophilic nonhalophilic – Vibrio cholerae and halophilic vibrios – V. parahaemolyticus,
vibrios V. alginolyticus, V. vulnificus
Serotypes of V. cholerae Ogawa (M/c), Inaba, Hikojima
Mechanism of Cholera toxin Fragment B is the binding unit which binds to Ganglioside receptor ;
Increase in adenylate cyclase; Increase in cAMP
Which is called as Bengal strain of V. cholerae O139; first isolated in chennai
What are all the selective media for V. cholerae? Alkaline bile salt agar, Monsur’s GTTA medium, TCBS agar
What is the characteristic motility pattern of V. cholerae? Darting motility
What is kanagawa phenomenon? Production of beta hemolysis in Wagatsuma agar – by V. parahaemolyticus
Clinical diseases caused by Pseudomonas sp., VAP, Malignant otitis externa, Shanghai fever, Swimmer’s ear, Burns
infections (M.c), Ecthyma gangrenosum, Green nail syndrome
Melioidosis is caused by Burkholderia pseudomallei; the disease is also called as Vietnam time bomb
The most virulent serotype of H. influenzae is: H. influenzae serotype b (Hib)
Accessory growth factor necessary for H. influenzae and Factor X (Hemin) and Factor V (NAD) – Exhibit satellitism
its characteristics:
Only drug of choice of Hib invasive infection Cephalosporins
Brazilian purpuric fever is caused by Haemo.aegyptius – Koch’s Week’s bacillus; causes pink eye syndrome
Chancroid is caused by Haemo.ducreyi – School of fish appearance or rail road track appearance
HACEK group Haemophilus sp., Aggregatibacter actinomycetemcomitans,
Cardiobacterium hominis, Eikenella corrodens, Kingella kingae
Which phase is highly infectious in pertussis? Catarrhal phase
Media for isolation of Bordetella pertussis Regan and Lowe medium, Border-Gengou medium
Gold standard serological test for Brucellosis Standard Agglutination test
Causative organism for Syphilis, Yaws, Pinta and endemic Syphilis – Treponema pallidum, Yaws – Treponema pertenue, Pinta –
syphilis T. carateum and endemic syphilis – T. endemicum
Specific serological tests for identification of syphilis TPI, TPHA, FTA-ABS (gold std), TPIA
201
Unit 2     Bacteriology

Biological false positive reactions Positive nontreponemal test Eg; VDRL, but negative treponemal test Eg;
TPHA – BFP antibody is IgM type
Ideal test for monitoring treatment of syphilis VDRL
Diagnosis of neuro syphilis VDRL
First serological test to be positive in syphilis FTA-ABS
Relapsing fever is caused by Borrelia recurrentis – epidemic RF; B. duttonii, B. hermsii – endemic RF
Lyme disease is caused by Borrelia burgdorferi
Weil’s disease is caused by Leptospira interrogans – causes hepatorenal hemorrhagic syndrome
Diagnosis of leptospirosis MAT – Microscopic agglutination test – gold standard
Which is called as Eaton agent Mycoplasma
Freid egg appearance is a feature of Mycoplasma
What is Diene’s method and Diene’s phenomenon Diene’s method – used to study colonies in Mycoplasma; Diene’s
phenomenon – to detect swarming of proteus
Mycoplasma causing genital infections Mycoplasma hominis
Obligate intracellular organisms Chlamydia and Rickettsiae
Q fever is caused by Coxiella burnetti
Five-day fever or trench fever is caused by Bartonella quintana
Neil-mooser reaction is positive in Rickettsia typhi
Reiter’s syndrome is caused by C. trachomatis, Ureaplasma urealyticum, Salmonella sp, Shigella sp,
Campylobacter sp
Atypical pneumonia is caused by Chlamydia pneumoniae (TWAR strain)
Levinthal cole Lillie bodies are seen in Chlamydia psittaci

202
3

VIROLOGY
Unit Outline
Chapter 24 Introduction and General Properties of
Viruses
Chapter 25 Bacteriophages
Chapter 26 Poxviruses
Chapter 27 Herpesviruses
Chapter 28 Adenovirus
Chapter 29 Picornaviruses
Chapter 30 Orthomyxoviruses
Chapter 31 Paramyxovirus
Chapter 32 Arthropod- and Rodent-Borne Viral
Infections
Chapter 33 Rhabdovirus
Chapter 34 Hepatitis Virus
Chapter 35 Human Immunodeficiency Virus
Chapter 36 Miscellaneous Viruses
Introduction and General 24
Properties of Viruses
CHARACTERISTICS OF VIRUSES CLASSIFICATION OF VIRUS
y Viruses are obligate intracellular parasites – because they
y
do not have the enzymes to synthesize protein and nucleic Table 2:  Classification of DNA viruses based on envelope
acid on its own
y They are the smallest infectious agents of size from 20 to 300 nm Enveloped DNA viruses (HHP) Nonenveloped
DNA viruses (PAP)
y
in diameter.
y No proper cellular organization Herpes viruses Parvoviridae
y
y Possess only one type of nucleic acid either RNA or DNA
Hepadnaviridae – Hepatitis B virus Papovaviridae
y
y Division: By binary fission and they need other cells for
y
multiplication and they are inert when seen extracellular Poxviridae Adenoviridae
y All viruses cannot be seen under light microscope - Exception:
y
Poxvirus (largest) Table 3:  Classification of RNA viruses based on envelope
y They need electron microscope to get visualized.
y
Enveloped RNA viruses Nonenveloped RNA viruses
TERMINOLOGIES USED IN VIROLOGY (PCR)
y Virion: Extracellular infectious virus particle Togaviridae Picornaviridae
y
y Capsid: Protein coat which covers nucleic acid of virion Flaviviridae Caliciviridae
y
 Main function is to prevent the nucleic acid by nucleases
Coronaviridae Reoviridae

 It also helps to introduce the viral genome into the host

cells Rhabdoviridae
 Based on the capsid symmetry – Three groups of viruses
Filoviridae

are seen
Š Icosahedral Paramyxoviridae
Š
Š Helical Orthomyxoviridae
Š
Š Complex
Bunyaviridae
Š
y Envelope: Outer covering of the virus
y
 Lipid layer – means the covering is from the host cell Arenaviridae

 Protein layer - means it is from the virus itself
Retroviridae

y Peplomers: Certain spikes seen in the outer layer which are
y
the protein subunits
 Hemagglutinin SYMMETRY

 Neuraminidase
y The nucleocapsids of the viruses are either in helical or

y Viriods: Subviral agent that has no virion and looks smaller
y
icosahedral symmetry.
y
than viruses
y Helical symmetry means the capsids are arranged like the
y Prion: Unconventional virus-like agents, mainly composed
y
spiral stepped staircase around the central genome.
y
of proteins and having no nucleic acid
y Icosahedral symmetry means the capsids are arranged to
y
form a solid with 20 equal triangular side which encloses the
STRUCTURE OF VIRUS genome.
y Enveloped viruses: Susceptible to lipid solvents (Ether, Chlo- y The large poxviruses have complex structures not falling into
y
y
roform and Bile salts) and biocides either category.
Table 1:  Different shapes of viruses y Icosahedral symmetry: All DNA viruses except poxvirus; Non-
y
enveloped RNA viruses; Retrovirus, Flavivirus, Togaviridae
Shapes Viruses
y Helical symmetry – Rest of the RNA viruses
Bullet shaped Rabies virus
y
y Segmented nucleic acid is seen in (BORA):
y
Space vehicle shaped Adeno virus  Bunyaviridae

 Orthomyxoviridae – Influenza virus has 8 segments of RNA
Filamentous Ebola virus

 Reoviridae
Brick shaped Pox virus

 Arenaviridae

Rod shaped Tobacco mosaic virus
Unit 3     Virology

Molluscum bodies Molluscum contagiosum


Guarnieri bodies Vaccinia
Bollinger bodies Fowl pox
Both intracytoplasmic and intranuclear inclusion bodies
Warthin Finkeldey cells - Measles virus

MULTIPLICATION OF VIRUS
y Viral replication undergoes process, which follows the follow-
y

Figure 1: Virus with icosahedron symmetry ing steps:


 Adsorption (Attachment)


 Penetration


 Uncoating


 Biosynthesis


 Maturation


 Release


Figure 2: Virus with helical symmetry

Table 4: Types of nucleic acid present in the viruses


Nucleic acid viruses


ss DNA virus Parvo virus
ds DNA virus All other DNA viruses
ss RNA virus All other RNA viruses
ds RNA virus Reo viridae

INCLUSION BODIES
y In virus-infected cells, some structures are seen with unique
y

size, shape and location called as Inclusion bodies


y They can be seen with light microscope
y

y It can be either intranuclear or intracytoplasmic


y

y It can be either acidophilic or basophilic


y

y All of the inclusion bodies are acidophilic except Adenovirus-


y

Basophilic

Table 5: Inclusion bodies of the viruses


Intranuclear inclusion bodies


Cowdry type A Herpes, Yellow fever
Cowdry type B Polio virus
Intracytoplasmic inclusion bodies
Figure 3: Process of viral replication
Negri bodies Rabies
y DNA viruses synthesize in the host cell nucleus
Paschen bodies Variola
y

y RNA viruses synthesize in the cytoplasm


y

206 y Eclipse phase: Virus cannot be demonstrated during the


y

Contd... phase from penetration to maturation


y von Magnus phenomenon: During the viral replication, a Table 6:  Types of cell culture

Chapter 24     Introduction and General Properties of Viruses


y

few proportion is not formed properly—called incomplete


viruses due to defective assembly – in such cases – when we Cell culture types Examples
perform hemagglutination test, the titer will be high – but the Primary cell culture • • Monkey kidney cell lines
infectivity in such cases is low • • Human embryonic kidney cell line
y Abortive infection: Certain host cells are nonpermissive
y
• • Human amnion cell line
cells – virus when entered in such cells – will not yield proper • • Chick embryo cell cultures
daughter cells—non infectious cells Diploid cell culture • WI-38
y Defective virus: Virus is genetically defective in undergoing

• HL-8
multiplication—needs another virus for it multiplication

– E.g: Hepatitis D virus, Adeno-associated satellite viruses Continuous cell lines • • HeLa
(Dependent viruses) • • HEp-2
• • KB
• The nucleic acid of all DNA viruses except parvoviruses is

• • McCoy
double stranded (but note that hepadnavirus DNA is partly • • Detroit 6
single stranded when not replicating) • • Chang C/I/L/K
• The nucleic acid of all RNA viruses except reoviruses is single

• • Vero
stranded. • • BHK-21
y After the virus is grown in cell culture, it can be identified by
y

CULTIVATION OF VIRUS the effects it produces in the cell lines – termed as cytopathic
effect (CPE).
y Viruses are grown in cell cultures or in eggs under strict
y

incubatory conditions.
y For some viruses, animal isolation is till practiced.
y
Table 7:  Cytopathic effect of viruses

Virus CPE
Commonly Employed Methods
Measles virus Syncytium formation
y Animal inoculation e.g: suckling mice for Coxsackie virus
y

y Embryonated eggs
y
Herpes virus Discrete focal degeneration
 Inoculation on the chorioallontoic membrane—pock


formation Adenovirus Large granular clumps (like grapes)


 Inoculation into the allantoic cavity – influenza and few


SV40 Cytoplasmic vacuolation


paramyxoviruoes – also for vaccine production
 Amniotic sac inoculation: Influenza

Enterovirus Crenation and degeneration of cells
 Yolk sac inoculation: Chlamydia and Rickettsia mainly


y Tissue culture – Cell culture is the most common method


y

employed now How to Detect a Virus that Has Grown in Cell


y Cell culture types:
y

Culture
 Primary cell culture:


Š It is made by dispersing the cells with trypsin from


Š
y Cytopathic effect as described above
y

freshly removed host tissues y Metabolic inhibition – virus after grown make the medium
y

Š These types of cell-cultured viruses cannot grow for


Š
change in color (by indicator)
more than a few passages in culture y Hemadsorption
y

Š E.g. Monkey kidney, human embryonic kidney, human


Š
y Interference
y

amnion and chick embryo cell cultures. y Transformation


y

 Diploid cell strains:



y Immunofluorescence.
y

Š These are secondary cultures that has extended some


Hemagglutination
Š

more passage (maximum up to 50 passages) which can


retain the viability y Haemagglutination is a method that is used for viral assays
y

Š E.g. WI-38, HL-8


Š
especially influenza virus
 Continuous cell lines :

y When red blood cells are added to serial dilutions of viral
y

Š These cultures are capable of more prolonged growth


Š
suspension – the highest dilution at which the virus produces
Š These cell lines are usually derived from malignant cell
Š
agglutination of RBCs is called as hemagglutination titre
lines
Š E.g. HeLa, HEp-2, KB, McCoy, Detroit 6, Chang C/I/
Š

L/K, Vero, BHK-21

207
y Interferon is secreted and bind to nearby cell receptors. It
Unit 3     Virology

Viruses that cause Haemagglutination are:


y

produces an antiviral state by prompting the synthesis of


• Influenza virus
other proteins

• Parainfluenza virus
y Major drawbacks of interferons were species specific. Now it

• Newcastle disease virus


y

has been overcome by adding a viral inducer


• Mumps, Measles and Rubella


y Uses: Helpful in the prophylaxis and treatment of viral

• Togavirus
y

infections. e.g. Hepatitis viruses


• Enterovirus

• Rhinovirus
Biological Effects of Interferons

• Rabiesvirus

• Reovirus
• y Antiviral effects: Induction of resistance to infection
y

y Antimicrobial effects: Resistance to intracellular infections,


y

E.g. Toxoplamsa, Chlamydia and Malaria


Elution y Cellular effects – Inhibition of cell growth and proliferation;
y

y Found only in myxoviruses


y inhibition of DNA and protein synthesis; Increased expression
y It is because of receptor called neuraminidase
y of MHC antigens on cell surfaces.
y This causes reversal of hemagglutination
y

Table 9:  Properties of interferon


Interference
Properties Interferon Interferon Interferon
y When a virus that does not produce cytopathic effect is grown
y

alpha beta gamma


along with virus producing cytopathic effect – the growth of
first one is inhibited by second – interference is mediated by Source Leucocytes Fibroblasts T cells and NK
Interferons cells
Induced by Virus Virus Antigen
Interferons: (IF) Subtypes 20 1 1
y Interferons are secreted by host cell when induced by virus;
y

Glycosylation No Yes Yes


belongs to cytokines
y Nonviral inducers also produce interferons
y
Functional form Monomer Dimer Tetramer
y IF produced by one host cell will act on near by host cells and
y
Mechanism of Inhibits Inhibits MHC antigens
protect them from viral infection action protein protein activates
y Species specific
y
synthesis synthesis T cells, NK
y Types:
y
cells and
 Alpha interferon (IFN-α)

macrophages
 Beta interferon (IFN-β)


 Gamma interferon (IFN-γ)




VIRAL ASSAYS
y Both IFN-α and IFN-β belong to type I or viral interferons
y

y IFN-γ is of type II or immune interferon


y
Two types: Quantitative assay and Quantal assay
y RNA viruses are strong inducers of interferon than DNA
y
y Quantitative assay
y

 Plaque assay
viruses 

 Pock assay


y Quantal assay
Table 8:  Features of interferons
y

Interferons Features Plaque Assay


Alpha interferon (IFN-α) Known as leucocyte interferon y Viral suspension is prepared and added to a monolayered
y

(Type I or viral interferon) Produced by leucocytes on viral culture cells in bottle or Petri dish
stimulation y Some time is allowed for the virus to get absorbed
y

It is a nonglycosylated protein y After viral replication – each viral particle gives rise to localized
y

Totally – 16 antigenic subtypes are focus of infected cells that can be seen with the naked eye
seen y This focus area is called Plaques
y

Beta interferon (IFN-β) Also known as fibroblast interferon Pock Assay


(Type I or viral interferon) Produced by fibroblasts and y Viruses form pocks in the chorio allantoic membrane and it
epithelial cells
y

can be assayed by counting the number of pocks formed on


It is a glycoprotein CAM.
Gamma interferon (IFN-γ) Formerly known as immune
(Type II or immune interferon Quantal Assay
interferon) Produced by T lymphocytes on y With the help of serial dilution of viral suspensions and
stimulation by antigens or mitogens
y

208 some statistical methods – accurately the infectivity can be


It is a glycoprotein calculated.
This is more towards
immunoregulatory functions
y This is done in animals or eggs or even in tissue culture. Disadvantages

Chapter 24     Introduction and General Properties of Viruses


y

y A titer is calculated based on death of animal or


y There is small risk of reversion to virulence at any time (Eg:
y

hemagglutination or CPE.
y

Vaccine derived poliomyelitis)


y It is called as ID50 (50% Infectious dose)
y While preparing vaccine chances of contamination is there
y

y The method that is used is Reed and Muench statistical


y

y Live vaccines cannot be given to immunodeficient people


y

method.
y

y Two live vaccines when given simultaneously may lead to


y

interference in immunity.
LABORATORY DIAGNOSIS OF VIRAL
INFECTIONS Killed Vaccines
y Microscopy: Electron microscope and Fluorescent micro-
y
y These vaccines are prepared by inactivating viruses with heat,
y

scope phenol, formalin or beta propiolactone


y Isolation of virus: Cell culture technique
y
y UV radiation method of inactivation is not an ideal method
y

y Serology: Antigen and antibody detection


y
(more chance of reactivation is there)
y Molecular method: Conventional PCR and RT-PCR (Reverse
y

transcriptase) Advantages
y Killed vaccines are stable and safe
VIRAL VACCINES
y

y Reversal is almost zero


y

y Most viral infections usually produce long-lasting immunity


y
y Killed vaccines can be combined with other vaccines.
y

y Viral vaccines can be of Live or Killed


y

Disadvantages
Live Viral Vaccines y They need booster doses for producing effective immunity
y

y Local immunity is not produced


y These are more effective means of producing immunity than
y

killed vaccines
Table 10:  Examples for live and killed viral vaccines
Advantages Live viral vaccines Killed viral vaccines
y A single dose is sufficient
y

Oral polio virus vaccine (OPV) Inactivated polio vaccine (IPV)


y It can be administered through natural route of infection
Yellow fever vaccine (17D) Rabies vaccine
y

E.g. (Oral polio virus) OPV – thereby local immunity can be


achieved Varicella zoster vaccine Japanese encephalitis vaccine
y It produces more number and wide spectrum of
y

MMR vaccine (Mumps, Measles, Influenza (subunit vaccine)


immunoglobulins (humoral immunity). Rubella)
y It also induces cell-mediated immunity
Influenza (attenuated and
y

y The immunity is long lasting


recombinant vaccines)
y

y Vaccine production is cheap and economical.


y

209
MULTIPLE CHOICE QUESTIONS
Unit 3     Virology

1. False about viruses is: (Recent Pattern Dec 2013) 15. Which is a enveloped virus: (Recent Pattern Dec 2012)
a. Ribosomes absent b. Mitochondria absent a. Dengue virus b. Norwalk virus
c. Motility absent d. Nucleic acid absent c. Hep A virus d. Adenovirus
2. Virus most sensitive to inactivation by biocides: 16. Smallest DNA virus is:
 (Recent Pattern Dec 2015) a. Herpes virus b. Adenovirus
a. Adenovirus b. Herpes virus c. Parvovirus d. Poxvirus
c. Parvovirus d. Polio virus 17. Ebola virus belongs to: (Recent Pattern July 2015)
3. Appearance Cowdry type A inclusion bodies: a. Picornaviridae b. Togaviridae
 (Recent Pattern Nov 2014) c. Flaviviridae d. Filoviridae
a. Granular b. Circumscribed 18. In which of the following has genome double stranded
c. In polio d. None nucleic acid: (PGI Nov 2013)
4. Lipschutz bodies are seen in: a. Orthomyxoviruses b. Poxviruses
 (Recent Pattern Dec 2016) c. Parvoviruses d. Reoviruses
a. Hodgkin’s disease b. Viral hepatitis 19. The virus with smallest genome is:
c. Herpes d. Influenza a. Reovirus b. Parvovirus
5. Bollinger bodies are seen in: (Recent Pattern Dec 2014) c. Picornavirus d. HIV
a. Chickenpox b. Cowpox 20. HPV belongs to: (Recent Pattern Dec 2012)
c. Smallpox d. Fowlpox a. Papovavirus b. Parvovirus
6. Which the following is always present in a virus? c. Herpesvirus d. Poxvirus
 (Recent Pattern July 2016) 21. Segmented genome is found in all, except:
a. Enzymes b. Envelope  (Recent Pattern Dec 2013)
c. RNA or DNA d. All of the above a. Influenza virus b. Reovirus
7. Not true about viral envelope: c. Bunyavirus d. Rhabdovirus
 (Recent Pattern July 2015) 22. Pocks producing virus can be grown in:
a. Lipid derived from host cells  (Recent Pattern July 2016)
b. Protein is derived from virus itself a. Organ culture
c. Dissolve in solvent b. Chorioallantoic membrane
d. Propagates in next generation c. Cell culture
8. Cowdry type A inclusion bodies are seen in: d. Continuous cell line
 (Recent Pattern Nov 2015) 23. Syncytium formation is a property of:
a. HBV b. Herpes virus  (Recent Pattern Dec 2015)
c. Adenovirus d. Pox virus a. Herpes virus b. Adenovirus
9. Both intracellular and cytoplasmic inclusion is seen in: c. Measles virus d. Rabies virus
 (Recent Pattern Dec 2012) 24. Suckling mice is used for isolation of:

a. Pox virus b. Herpes virus  (Recent Pattern Dec 2012)


c. Measles virus d. Mumps virus a. Coxsackie virus b. Pox
10. Capsid of viral structure is: (Recent Pattern Dec 2016) c. Herpes d. Adenovirus
a. Extracellular infectious particle 25. Virus quantification is done by:
b. Protein coat around nucleic acid  (Recent Pattern Dec 2016)
c. Envelop around a virus a. Egg inoculation b. Hemadsorption
d. None of the above c. Plaque assay d. Electron microscopy
11. New castle virus belongs to family: 26. Human fibroblast cell line is used for cultivation of:
 (Recent Pattern June 2014)  (Recent Pattern Nov 2014)
a. Paramyxovirus b. Calcivirus a. Adenovirus b. Poliovirus
c. Flavivirus d. Filovirus c. HIV d. Measles
12. Which is not a DNA virus? (Recent Pattern Dec 2013) 27. Which of the following is primary cell line:
a. Parvovirus b. Papovavirus a. Chick embryo fibroblast (Recent Pattern Dec 2012)
c. Poxvirus d. Rhabdovirus b. Hela cells
13. Nonenveloped SS-RNA virus is: c. Vero cells
 (Recent Pattern Dec 2012) d. WI-38
a. Picornavirus b. Poxvirus 28. Influenza virus culture is done on:
c. Retrovirus d. Bunyavirus  (Recent Pattern Dec 2012)
14. True about herpes virus is: (Recent Pattern Aug 2013) a. Chorioallantoic membrane
a. Circular double stranded DNA virus b. Allantoic cavity
b. Linear double stranded DNA virus c. Yolk sac
210 c. Circular double stranded RNA virus d. All
d. Circular single stranded RNA virus
29. Hep-2 cells a type of: (Recent Pattern Dec 2012) 33. von Magnus phenomenon is: (Recent Pattern Dec 2016)

Chapter 24     Introduction and General Properties of Viruses


a. Primary cell cultures b. Diploid cell strain a. Abnormal replicative cycle
c. Continuous cell lines d. Explant culture b. Virus with low infectivity
30. Hemagglutination done by all virus, except: c. Virus with high hemagglutination
 (Recent Pattern July 2015) d. All of the above
a. Influenza b. Rubella 34. Beta interferon is produced because of:
c. Measles d. HPV a. Bacteria infected cells  (AIIMS Nov 2018)
31. Integration of viral genome into host cell chromosome b. Virus infected cells
can leads to: (Recent Pattern Dec 2016) c. Mycoplasma infected cells
a. Malignancy b. Latency d. Parasite-infected cells
c. Altered growth d. All of the above
32. Property of elution is found in:
 (Recent Pattern Dec 2013)
a. Myxovirus b. Togavirus
c. Parvovirus d. Adenovirus

ANSWERS AND EXPLANATIONS


1. Ans.  (d) Nucleic acid absent 5. Ans.  (d) Fowlpox
Ref: Ananthanarayan and Paniker’s Textbook of Microbiol- Ref: Ananthanarayan and Paniker’s Textbook of Microbiol-
ogy – 10th ed – Page 434 ogy – 10th ed – Page 450
• Viruses do not have proper cellular organization. They

• Large inclusion bodies – named as Bollinger bodies –

do not have ER or ribosomes seen in fowl pox


• Possess only one type of nucleic acid either RNA or DNA.
6. Ans.  (c) RNA or DNA

2. Ans.  (b) Herpes virus Ref: Ananthanarayan and Paniker’s Textbook of


Ref: Ananthanarayan and Paniker’s Textbook of Microbiol- Microbiology – 10th ed – Page 451
ogy – 10th ed – Page 436 Ref: Already explain Answer. 1
• Enveloped viruses – susceptible to lipid solvents (Ether,

Chloroform, Bile salts) and biocides 7. Ans.  (d) Propagates in next generation
• Among the given option all other are nonenveloped

Ref: Ananthanarayan and Paniker’s Textbook of Microbiol-


viruses except Herpes virus (Ref: Above table in text) ogy – 10th ed – Page 438
3. Ans.  (a) Granular • Envelope: Outer covering of the virus

ƒ Lipid layer – means the covering is from the host cell


ƒ

Ref: Ananthanarayan and Paniker’s Textbook of Microbiol- ƒ Protein layer - means it is from the virus itself
ƒ

ogy – 10th ed – Page 451 ƒ During viral replication – multiple steps happens-
ƒ

• Cowdry type inclusion bodies are intranuclear and are



during that viral envelope uncoating done and new
of two types envelope is synthesized for each progeny.
• Cowdry type A – size is variable, granular appearance –
8. Ans.  (b) Herpes virus

E.g. Herpes virus, Yellow fever virus


• Cowdry type B – more circumscribed and multiple – E.g.
• Ref: Ananthanarayan and Paniker’s Textbook of Microbiol-
Adenovirus, Polio virus ogy – 10th ed – Page 451
• Already explained Q.3
4. Ans.  (c) Herpes

9. Ans.  (c) Measles virus


Ref: Ananthanarayan and Paniker’s Textbook of Microbiol-
ogy – 10th ed – Page 451 Ref: Ananthanarayan and Paniker’s Textbook of Microbiol-
• Cowdry type A – size is variable, granular appearance

ogy – 10th ed – Page 451
seen in Herpes virus • Both intra cellular and intra nuclear inclusion bodies are

• It is intra nuclear and called as Lipschutz bodies


• seen in MEASLES – Warthin Finkeldey cells.

211
Unit 3     Virology

10. Ans.  (b) Protein coat around nucleic acid • Filoviridae – long, filamentous, enveloped viruses

• ss RNA
Ref: Ananthanarayan and Paniker’s Textbook of Microbiol-

• Marburg and Ebola virus


ogy – 10th ed – Page 434

• Capsid - Protein coat which covers nucleic acid of virion


• 18. Ans.  (b) Poxviruses (d) Reoviruses
ƒ Main function is to prevent the nucleic acid by nucle-
Ref: Ananthanarayan and Paniker’s Textbook of Microbiol-
ƒ

ases
ƒ It also helps to introduce the viral genome into the
ƒ
ogy – 10th ed – Page 446
host cells Examples for types of nucleic acid
ƒ Based on the capsid symmetry – Three groups of vi-
ss DNA virus Parvovirus
ƒ

ruses are seen


Š Icosahedral
Š ds DNA virus All other DNA viruses
Š Helical
Š

ss RNA virus All other RNA viruses


Š Complex
Š

ds RNA virus Reoviridae


11. Ans.  (a) Paramyxovirus
19. Ans.  (b) Parvovirus
Ref: Ananthanarayan and Paniker’s Textbook of Microbiol-
ogy – 10th ed – Page 446 Ref: Ananthanarayan and Paniker’s Textbook of Microbiol-
• Paramyxovirus – RNA virus – includes Para influenza

ogy – 10th ed – Page 446
virus, Respiratory syncytial virus, Mumps, Measles virus Ref: Already explained Q.16
and Newcastle disease virus
20. Ans.  (a) Papovavirus
12. Ans.  (d) Rhabdovirus
Ref: Ananthanarayan and Paniker’s Textbook of Microbiol-
Ref: Ananthanarayan and Paniker’s Textbook of Microbiol- ogy – 10th ed – Page 446
ogy – 10th ed – Page 444
• Papova viridae – small, non enveloped, ds DNA viruses
• Rhabdovirus is a RNA virus (Ref: Above table in text)

• Papilloma virus and Polyoma virus belongs to this


13. Ans.  (a) Picornavirus


21. Ans.  (d) Rhabdovirus
Ref: Ananthanarayan and Paniker’s Textbook of Microbiol-
Ref: Ananthanarayan and Paniker’s Textbook of Microbiol-
ogy – 10th ed – Page 444
ogy – 10th ed – Page 447
• Picornavirus is a RNA virus which is nonenveloped (Ref:
• Segmented nucleic acid is seen in (BORA)

Above table in text)


ƒ Bunyaviridae
ƒ

14. Ans.  (b) Linear double stranded DNA virus ƒ Orthomyxoviridae – Influenza virus has 8 segments
ƒ

of RNA
Ref: Ananthanarayan and Paniker’s Textbook of Microbiol- ƒ Reoviridae
ƒ

ogy – 10th ed – Page 444 ƒ Arenaviridae


ƒ

• Nucleic acid can be either circular or linear


• Herpes virus belongs to linear ds DNA.



22. Ans.  (b) Chorioallantoic membrane
Ref: Ananthanarayan and Paniker’s Textbook of Microbiol-
15. Ans.  (a) Dengue virus
ogy – 10th ed – Page 440
Ref: Ananthanarayan and Paniker’s Textbook of Microbiol- • Inoculation on the chorioallontoic membrane – pock

ogy – 10th ed – Page 444 formation


• Already explained in the table – it is a member of

• Inoculation into the allantoic cavity – Influenza and few


flaviviridae paramyxoviruses – also for vaccine production


• Amniotic sac inoculation – Influenza
16. Ans.  (c) Parvovirus

• Yolk sac inoculation – Chlamydia and Rickettsia mainly


Ref: Ananthanarayan and Paniker’s Textbook of Microbiol-


ogy – 10th ed – Page 446 23. Ans.  (c) Measles virus
• Smallest virus – Parvovirus
• Ref: Ananthanarayan and Paniker’s Textbook of Microbiol-
• Largest virus – Pox virus
• ogy – 10th ed – Page 442
• After the virus is grown in cell culture, it can be identified
17. Ans.  (d) Filoviridae

by the effects it produces in the cell lines – termed as


Ref: Ananthanarayan and Paniker’s Textbook of Microbiol- cytopathic effect (CPE)
ogy – 10th ed – Page 447 • Measles virus produces syncytium formation as CPE
212

Chapter 24     Introduction and General Properties of Viruses
24. Ans.  (a) Cox sackie virus 30. Ans.  (d) HPV
Ref: Ananthanarayan and Paniker’s Textbook of Microbiol- Ref: Ananthanarayan and Paniker’s Textbook of Microbiol-
ogy – 10th ed – Page 440 ogy – 10th ed – Page 437
• Animal inoculation was the earliest method used for
• • Viruses that cause Haemagglutination are:

cultivation of virus • Influenza virus, Para influenza, Newcastle disease virus,


• Nowadays it has been replaced by cell culture


• Mumps, Measles, Rubella, Toga virus, Entero virus,
• But for certain viruses – animal inoculation is needed
• Rhino virus, Rabies and Reo virus
• Eg: Coxsackie virus – suckling mice

31. Ans.  (d) All of the above


25. Ans.  (c) Plaque assay
Ref: Jawetz – 27th edition – Page 428
Ref: Ananthanarayan and Paniker’s Textbook of Microbiol- • When virus enters the host cells – integration of viral

ogy – 10th ed – Page 443 genomes into the host chromosome occurs; This can
• Two types of viral assays: Quantitative assay and Quantal
• leads to:
assay • Viral persistence – Persistent or chronic infections

• Quantitative assay
• • Latent infection

ƒ Plaque assay
ƒ • Changes in the host cell surface

ƒ Pock assay
ƒ • Malignancy

26. Ans.  (b) Poliovirus 32. Ans.  (a) Myxovirus


Ref: Ananthanarayan and Paniker’s Textbook of Microbiol- Ref: Ananthanarayan and Paniker’s Textbook of Microbiol-
ogy – 10th ed – Page 441 and (Source: Internet) ogy – 10th ed – Page 436
• Human fibroblast cell lines: belong to diploid cell strains
• ƒ Elution – found only in myxo viruses that possess
ƒ

• These cell lines are used for the cultivation of


• neuraminidase
ƒ Coxsackie virus
ƒ

ƒ CMV
ƒ
33. Ans.  (d) All of the above
ƒ Herpes virus 1, 2
ƒ

Ref: Ananthanarayan and Paniker’s Textbook of Microbiol-


ƒ Echo virus
ƒ

ogy – 10th ed – Page 439


ƒ Entero virus
• Von magnus phenomenon – During the viral replication,
ƒ

ƒ Polio virus
ƒ

ƒ Rhino virus
a few proportion is not formed properly – called as
incomplete viruses due to defective assembly – in such
ƒ

27. Ans.  (a) Chick embryo fibroblast cases – when we perform hemagglutination test – the
titre will be high – but the infectivity is such cases is low
Ref: Ananthanarayan and Paniker’s Textbook of Microbiol-
ogy – 10th ed – Page 442 34. Ans.  (b) Virus infected cells
• Primary cell culture – E.g. Monkey kidney, Human

Ref: Ananthanarayan and Paniker`s T.B of microbiology –


embryonic kidney, Human amnion and chick embryo 10th ed – page 451
cell cultures
• Interferons are secreted by the host cell when induced

28. Ans.  (b) Allantoic cavity by virus; belongs to cytokines


• Types:

Ref: Ananthanarayan and Paniker’s Textbook of Microbiol- ƒ Alpha interferon (IFN-α)


ƒ

ogy – 10th ed – Page 440 ƒ Beta interferon (IFN-β)


ƒ

Ref: Already explained in Answer 22. ƒ Gamma interferon (IFN-γ)


ƒ

29. Ans.  (c) Continuous cell lines


Ref: Ananthanarayan and Paniker’s Textbook of Microbiol-
ogy – 10th ed – Page 440
• Continuous cell lines – HeLa, HEp-2, KB, McCoy, Detroit

6, Chang C/I/L/K, Vero, BHK-21

213
25
Bacteriophages
y Head of the phage has core of nucleic acid – dsDNA which is

y
surrounded by a protein coat called capsid
y Tail is made of core with contractile sheath and prongs and

y
tail fibers.

LIFE CYCLE
1. Virulent or Lytic cycle



2. Temperate or Lysogeny cycle



Virulent or Lytic Cycle
A virulent phage uses the cellular apparatus of its bacterial host
for multiplication, typically resulting in cell lysis (for obligatory
Figure 1: Bacteriophage lytic phages) and release of progeny virions
y Bacteriophages are the viruses that infect bacteria
y
y Discovered by Twort and d’ Herelle Temperate Cycle or Lysogeny Cycle
y
Temperate phages have alternative replication cycles: A
MORPHOLOGY productive, lytic infection or a reductive infection, in which the
y Bacteriophage that infect E. coli is called as T even phages phage remains latent in the host, establishing lysogeny.
y
y Phages are tadpole shaped with hexagonal head and
y
cylindrical tail

Figure 2: Lytic and lysogenic cycle of bacteriophage


y Phage conversion

Chapter 25    Bacteriophages


Terminologies y

• Prophage: When bacteriophage enters the bacterium – the y Cloning


y

y Phage typing

nucleic acid gets integrated – that integrated phage nucleic y

acid is called prophage – seen in lysogenic cycle


• Transfection: Infection of a bacterium by the naked phage

Phage Typing
nucleic acid is called transfection  Phage typing is helpful in identification of following
• Eclipse phase: The interval between the entry of the phage

bacteria:
nucleic acid into the bacterial cell and the appearance of first  Salmonella
infectious intracellular phage particle is known as eclipse  Bacillus anthracis
phase  Staphylococcus
• Latent period: The interval between the infection of a

 Vibrio cholera
bacterial cell by a phage to the first release of a infectious  Why the river Ganges is pure: It is because of the presence
phage particle is called as latent period of bacteriophages named as vibrio phages which clears
• Lysogenic conversion: Useful in toxin production of diphtheria

the V. cholerae infection
bacilli by beta phages  Protype of phage is: ’T’ even phages that infect E. coli
 Phage assay: When a lawn culture of bacterium is done
and phage is applied on that - phage will cause lysis of the
Role of Phages bacterium forms plaques. This plaques assay is helpful for
y Convenient model for study of virus-host interactions
y preparing the viable phages by titrating the count.
y Transduction
y

215
MULTIPLE CHOICE QUESTIONS
Unit 3     Virology

1. Which of the following statement is true about 6. Prophage is defined as: (Recent Pattern July 2016)
bacteriophage? (AIIMS May 2011) a. Insertion of viral nucleic acid into bacteria by
a. It is a bacterium bacteriophage
b. It helps in transformation b. First cycle of division of bacterial nucleic acid
c. It imparts toxigenicity to bacteria c. Last cycle of division of bacterial nucleic acid
d. It transfers only chromosomal gene d. Integrated temperate bacteriophage genome into
2. What is integrated temperate phage genome? bacterial chromosome
 (Recent Pattern July 2016) 7. Prophage is seen in which cycle of bacteriophage:
a. Prophage b. Telophage  (Recent Pattern Nov 2014)
c. Transphage d. Metaphage a. Virulent b. Lytic
3. Lysis of bacterial colony in culture is seen by: c. Temperate d. None
 (Recent Pattern Dec 2013) 8. Most common type of phage: (Recent Pattern June 2014)
a. Pox b. HSV a. Helical b. Complex
c. Bacteriophage d. CMV c. Icosahedral d. None of the above
4. Lambda phage which of the following is true: 9. Lysogenic conversion is: (Recent Pattern Dec 2014)
a. It causes mad cow disease (AIIMS May 2011) a. New properties in a bacterium due to integration of
b. Lysogenic to lytic conversion cannot occur phage genome
c. Lysogenic form incorporates in host DNA and remain b. Transfer of DNA from one bacterium to another by
dormant bacteriophage
d. Lytic phase incorporates in host DNA proliferate and c. Transfer of free DNA
causes rupture of cell d. Transfer of genome during physical contact
5. Phage typing can be done for:(Recent Pattern Dec 2013)
a. Salmonella b. Streptococcus
c. Shigella d. Pseudomonas

ANSWERS AND EXPLANATIONS


1. Ans.  (c) It imparts toxigenicity to bacteria helpful for preparing the viable phages by titrating the
count
Ref: Ananthanarayan and Paniker’s Textbook of Microbiol-
ogy – 10th ed – Page 462 4. Ans.  (c) Lysogenic form incorporates in host DNA and
• Bacteriophage is a virus that infects the bacteria
• remain dormant
• Helps in transduction
Ref: Ananthanarayan and Paniker’s Textbook of Microbiol-

• During lysogenic conversion – toxigenicity is acquired –


ogy – 10th ed – Page 464

E.g. Diphtheria
• It transfers chromosomal DNA, plasmid, episomes

• Mad cow disease – caused by prions

• Lambda phage is a bacteriophage


2. Ans.  (a) Prophage • In lytic cycle – phage enters the bacterial cell –

penetration –synthesis – maturation – assembly - release


Ref: Ananthanarayan and Paniker’s Textbook of Microbiol-
of the mature progeny by lysis of the cell occurs
ogy – 10th ed – Page 464
• In lysogenic cycle – Temperate phages makes a good

• Prophage: When bacteriophage enters the bacterium –



relationship with the bacterium – enters inside and stays
the nucleic acid gets integrated – that integrated phage there without causing Lysis
nucleic acid is called as prophage – seen in lysogenic
cycle 5. Ans.  (a) Salmonella

3. Ans.  (c) Bacteriophage Ref: Ananthanarayan and Paniker’s Textbook of Microbiol-


ogy – 10th ed – Page 465
Ref: Ananthanarayan and Paniker’s Textbook of Microbiol-
• Phage typing: Helpful in
ogy – 10th ed – Page 465

ƒ Salmonella
ƒ

• Phage assay: When a lawn culture of bacterium is done



ƒ Bacillus anthracis
ƒ

and phage is applied on that - phage will cause lysis of ƒ Staphylococcus


ƒ

the bacterium – forms plaques – This plaques assay is ƒ Vibrio cholera


ƒ

216
• Icosahedral head – ds DNA with a protein coat called

Chapter 25    Bacteriophages


6. Ans.  (d) Integrated temperate bacteriophage genome •

into bacterial chromosome capsid


• Tail has a sheath with terminal base plate

Ref: Ananthanarayan and Paniker’s Textbook of Microbiol-


ogy – 10th ed – Page 464 9. Ans.  (a) New properties in a bacterium due to integra-
Ref: Already explained Q.2 tion of phage genome
Ref: Ananthanarayan and Paniker’s Textbook of Microbiol-
7. Ans.  (c) Temperate ogy – 10th ed – Page 464
Ref: Ananthanarayan and Paniker’s Textbook of Microbiol- • Prophage: When bacteriophage enters the bacterium –

ogy – 10th ed – Page 464 the nucleic acid gets integrated – that integrated phage
Ref: Already explained Q.2 (Lysogenic cycle also called as nucleic acid is called as prophage – seen in lysogenic
temperate cycle) cycle
• During this lysogenic integration – some new properties

are added
8. Ans.  (c) Icosahedral
E.g. useful in toxin production of diphtheria bacilli by
Ref: Ananthanarayan and Paniker’s Textbook of Microbiol- beta phages.
ogy – 10th ed – Page 462

217
26
Poxviruses
y Poxviruses are the largest viruses among all groups and it can y Variola major causes Smallpox
y
y
be seen under light microscopy y Variola minor causes Alastrim

y
y They have characteristic brick shape y These two viruses are similar antigenically; But their charac-
y
y
y Medically important among this group are: teristics and pathogenesis are different.
y
 Variola

 Vaccinia
Pathogenesis of Smallpox

 Orf

 Paravaccinia
Portal of entry of variola virus is mucous membranes of upper

 Molluscum contagiosum respiratory tract

Properties of Poxviruses:
• Brick or oval-shaped virion

• ds DNA

• Linear DNA

• Replication: Cytoplasmic

• Can be seen under light microscope

FEATURES OF POXVIRUS
y Brick shaped
y
y Biconcave nucleoid DNA
y
y Enveloped
y
y Resistant to 50% glycerol and 1% phenol
y
y Inactivated by formalin
y
y Though enveloped, virus is not inactivated by ether
y
y Cultivation: Chick embryo, tissue culture in monkey kidney,
Figure 1: Pathogenesis of Smallpox
y
HeLa and chick embryo tissue culture.

CLASSIFICATION Clinical Features


y Incubation period is 10–14 days
Table 1:  Classification of poxviruses
y
y Up to 5 days fever and malaise occurs
y
Ortho pox virus • Variola major y By 6–9 days of infections – mouth ulcerations starts to occur
y
y It is then followed by macules → papules → vesicles and

• Variola minor
y

• Vaccinia finally pustules

• Buffalopox y It leaves behind crusts and scarification
y

• Monkeypox y Global eradication of smallpox was declared on May 8, 1980

y
• Cowpox y Last case was a Bangladeshi woman on 1975

y
Parapox virus • Orf • Why Smallpox is successfully eradicated?

• Pseudocowpox

ƒ No animal reservoir

• Bovine papular stomatitis
ƒ
ƒ Only humans are infected

ƒ
Molluscipox virus • Molluscum contagiosum ƒ No carrier state
ƒ

Yata pox virus • Tanapox ƒ Use of freeze dried vaccine
ƒ
ƒ Technique of vaccination: multiple puncture, easy and

• Yabapox
ƒ
simple

VARIOLA • In world, only in two places last stocks of Smallpox are kept

under high security
y Variola virus attacks only humans and monkeys and causes
ƒ CDC, Atlanta, USA
y
smallpox
ƒ
ƒ Russia
y Two types: Variola major and minor
ƒ
y
Treatment of Smallpox Molluscum Contagiosum

Chapter 26     Poxviruses


y Mainly supportive treatment
y y y Most commonly affects children
y Methisazone was a chemotherapeutic agent given for
y
y y Lesions are pinkish or pearly white wart like nodules in skin
Poxviruses. y y Areas commonly affected are: Face, arms, back and buttocks
y y It is spread by direct contact or using towels and sharing pools
Vaccinia y y Molluscum contagiosum is a benign epidermal tumor-like
condition that is seen only in humans
y It is a vaccine virus that was used for smallpox vaccination
y

y y Nowadays it is mostly seen in young adults - because it is


y It is an artificial virus
y

sexually transmitted – seen in genitals – DD: Herpes virus


y Nowadays, it is very helpful as a vector in recombinant
y

y y This virus is a very poor immunogen; Antibodies are not


technology for the preparation of vaccines (but the drawback produced up to the level – secondary infections are more
is that it causes pathogenesis in humans; hence used only in common
animals) y y Diagnosis: Molluscum bodies (inclusion bodies)
y y This virus cannot be grown in eggs, tissue culture or animals
Table 2:  Comparison between variola and vaccinia virus (non cultivable)

Variola virus Vaccinia virus Poxviruses that does not grow in eggs
Natural virus causing smallpox Artificial virus for smallpox Paravaccinia – Milker’s node
vaccination Tanapox – affects only monkeys
In chick embryo: small, shiny, In chick embryo – larger, Molluscum contagiosum
white, convex, non-necrotic, irregular, flat, grayish,
non-hemorrhagic pocks are necrotic lesion are formed
formed
Do not produce plaques in chick Produce plaques in chick
embryo tissue culture embryo tissue culture
Elementary bodies – Paschen Eosinophilic inclusion bodies
bodies – Guarnieri bodies
Lesions can be produced In monkeys, calves, sheep and
experimentally only in monkeys rabbits

MISCELLANEOUS POXVIRUSES
Cowpox Figure 2: Umbilicated nodules in molluscum contagiosum
y Infection occurred during milking of cows from the udder
y

and teats of cows DIFFERENCES IN POXVIRUSES


y Lesions resemble that of primary vaccinia
y

y To differentiate it from vaccinia and variola virus chorio al- y Pocks formed in chorioallontoic membranes helps in growth
y

of certain viruses and differentiation also


y

lantoic membrane inoculation is done. Hemorrhagic lesions


y Ceiling temperature – highest temperature above which
are seen
y

pocks are not produced


y Based on differences in ceiling temperature also, Poxviruses
Milker’s Node
y

can be differentiated
y Called paravaccinia
y

y Humans get the infection during milking of cows


y
CAM Variola Monkey Vaccinia Cowpox Camel
y This virus does not grow in eggs
y
pox pox
Pocks Small, Small, Large, Hemorrhagic Small,
Orf white pink white white
y Causes contagious pustular dermatitis
Ceiling 37.5– 39 41 39.5 38.5
y

y Disease of sheep and goats, transmitted to humans


y

temperature 38.5
y Single lesion with a central ulcer is seen.
y

219
MULTIPLE CHOICE QUESTIONS
Unit 3     Virology

1. Which virus cannot be cultivated? 8. Inclusion bodies of vaccinia is known as:


a. Vaccinia (Recent Pattern Dec 2014)  (Recent Pattern Dec 2014)
b. Variola a. Guarnieri bodies b. Negri bodies
c. Molluscum contagiosum c. Asteroid bodies d. Schuffner bodies
d. Cowpox 9. Brick-shaped virus: (Recent Pattern Dec 2014)
2. Smallpox belongs to which class of poxviruses: a. Chicken pox
 (Recent Pattern Dec 2012) b. Smallpox
a. Parapoxvirus b. Capripoxvirus c. CMV
c. Leporipoxvinus d. Orthopoxvirus d. EBV
3. Guarnieri inclusion body is seen in:  10. A child presented with umbilicated pearly lesions in the
a. Fowl pox  (Recent Pattern Jun 2014)
face following this; mother also had developed the same
b. HSV
lesion. Identify the virus causing a lesion. 
c. Molluscum contagiosum
(AIIMS Nov 2018)
d. Vaccinia


4. Following virus is of pox virus: 


 (Recent Pattern Nov 2014)
a. Variola b. Coxsachie
c. ECHO d. HSV
5. Umblicated nodules are produced by:

 (Recent Pattern Dec 2012)


a. Poxvirus b. Enterovirus
c. Rhinovirus d. Myxovirus
6. Which pox wont grow in egg, animal cells:
 (Recent Pattern Dec 2012)
a. Cowpox b. Vaccinia
c. Variola d. Molluscum
7. Men is the primary host for which poxvirus:
 (Recent Pattern June 2014)
a. Monkey pox b. Orf a. Pox virus b. HPV
c. Tanapox d. Molluscum contagiosum c. Varicella-zoster virus d. Herpes simplex virus

ANSWERS AND EXPLANATIONS

1. Ans.  (c) Molluscum contagiosum 3. Ans.  (d) Vaccinia


Ref: Ananthanarayan and Paniker’s Textbook of Microbiol- Ref: Ananthanarayan and Paniker’s Textbook of Microbiol-
ogy – 10th ed – Page 471 ogy – 10th ed – Page 468
• Molluscum virus cannot be grown in eggs, tissue culture

Intra nuclear inclusion bodies


or animals (non cultivable
Cowdry type A Herpes, Yellow fever
2. Ans.  (d) Orthopoxvirus Cowdry type B Polio virus
Ref: Ananthanarayan and Paniker’s Textbook of Microbiol- Intra cytoplasmic inclusion bodies
ogy – 10th ed – Page 467
Negri bodies Rabies
Orthopox virus Variola, vaccinia, cowpox, Paschen bodies Variola
Monkeypox, rabbitpox, buffalopox
Molluscum bodies Molluscum contagiosum
Parapox virus Orf, paravaccinia,
Guarnieri bodies Vaccinia
Capripox virus Diseases that causes animals
Leporipox virus Bollinger bodies Fowl pox
Avipoxvirus Both intra cytoplasmic and intra nuclear inclusion bodies
220 Suipoxvirus Warthin-Finkeldey cells - Measles virus
4. Ans.  (a) Variola

Chapter 26     Poxviruses


8. Ans.  (a) Guarnieri bodies
Ref: Ananthanarayan and Paniker’s Textbook of Microbiol- Ref: Ananthanarayan and Paniker’s Textbook of Microbiol-
ogy – 10th ed – Page 467 ogy – 10th ed – Page 468
Ref: Already explained Answer 2 Ref: Already explained Answer 3

5. Ans.  (a) Poxvirus 9. Ans.  (b) Smallpox


Ref: Ananthanarayan and Paniker’s Textbook of Microbiol- Ref: Ananthanarayan and Paniker’s Textbook of Microbiol-
ogy – 10th ed – Page 471 ogy – 10th ed – Page 468
• Nodulated lesions are produced mostly by Poxviruses

• Virion of vaccinia and variola virus is brick shaped


• E.g. Molluscum contagiosum causes wart like nodules

• The elementary bodies are called as Paschen bodies.


in skin
10. Ans.  (a) Pox virus
6. Ans.  (d) Molluscum
Ref: Ananthanarayan and Paniker’s T.B of microbiology –
Ref: Ananthanarayan and Paniker’s Textbook of Microbiol- 10th ed – page 471
ogy – 10th ed – Page 471
Above picture shows umbilicated pearly lesions – which is
Ref: Already explained Answer 1 clinically diagnosed as Molluscum contagiosum

7. Ans.  (d) Molluscum contagiosum


Ref: Ananthanarayan and Paniker’s Textbook of Microbiol-
ogy – 10th ed – Page 471
• Humans are the only susceptible hosts for molluscum

virus.
• It mostly affects children

• But nowadays it has become a sexually transmitted


disease also.

221
27
Herpesviruses
INTRODUCTION HERPES SIMPLEX VIRUS
y Enveloped DNA viruses
y
y Icosahedral capsid Table 3:  Comparison between Herpes simplex virus
y
y ds DNA 1 and 2
y
y The linear DNA has a sequence arrangement which is unique
y
for herpesviruses Herpes simplex virus 1 Herpes simplex virus 2
y Replication occurs in host cell nucleus. Affects the upper part of the Lower part of the body –
y
body especially lips – oral cavity especially genitals
CLASSIFICATION Trigeminal ganglia Sacral ganglia

Table 1:  Official and common names of Herpesvirus Reacts with specific monoclonal Reacts with specific monoclo-
antibodies of HSV 1 nal antibodies of HSV 2
Official name Common name
Chick embryo – smaller pocks Larger pocks resembling
Herpesvirus type 1 HSV – 1 – Herpes simplex virus 1 variola
Herpesvirus type 2 HSV – 2 – Herpes simplex virus 2
Do not replicate well in chick Replicate well in chick embryo
Herpesvirus type 3 Varicella Zoster virus embryo fibroblast cells fibroblast cells
Herpesvirus type 4 Epstein Barr virus Not temperature dependent Infectivity is more
Herpesvirus type 5 Cytomegalovirus temperature sensitive
Herpesvirus type 6 Human B cell lymphotropic virus Neurovirulence is seen Higher comparatively than
Herpesvirus type 7 R K virus type 1
Herpesvirus type 8 Rhadinovirus Antiviral agent effect seen Higher resistant than type 1
Restriction endonuclease effect Unique to type 2
PROPERTIES specific
Table 2:  Properties of Herpes viruses
Pathogenesis
Subfamilies Replicative Host range Infection
cycle y HSV virus enters through abrasions in the skin and then
y
enters cutaneous nerve fibers and transported to ganglia.
Alpha Short (12– Variable Latent infection
y Virus replication occurs at the site of infection and then
herpesviruses 18 hours) with rapid CPE
y
through the local nerve endings, it gets transported to dorsal
Beta Slow Narrow Cytomegaly with
root ganglia where further replication continues
herpesviruses (>24 hours) slow CPE
y HSV causes cytolytic infections that lead to necrosis of
Gamma Variable Narrow Latent infection in
y
infected cells
herpesviruses lymphoid tissues
Characteristic histopathologic changes are:
Examples of Subfamilies • Ballooning of infected cells

y Alpha herpes viruses: HSV 1, 2, VZV • Production of Cowdry type A intranuclear inclusion bodies
y

y Beta herpes viruses: CMV, HSV 6, 7 • Margination of chromatin
y

y Gamma herpes viruses: EBV, HSV 8 • Formation of multinucleated giant cells
y

Note: None of the herpes viruses show antigenic cross reactivity y Further recurrence is more common
except HSV 1 and 2
y
y Antibodies cannot prevent future recurrence
y
y Cell-mediated immunity is important to prevent from HSV y Molecular methods: Polymerase Chain Reaction method is

Chapter 27     Herpesviruses


y y

infection sensitive and specific in the diagnosis. It is the widely used


y Typical lesions: Thin walled, umbilicated vesicles with
y method for diagnosis of HSV meningitis.
superficial ulcers
y Usually self-limiting, but in immunosuppressed it causes var-
y

HSV Meningitis
ious presentations
y The most common cause of sporadic, fatal encephalitis
y

HSV-1 (Oropharyngeal HSV) Trigeminal ganglia and meningitis in children is HSV


y It has a high mortality rate.
HSV-2 (Genital HSV) Sacral ganglia
y

Neonatal Herpes
Clinical Features y Newborn acquires herpes infections from mother through
y

Oropharyngeal Disease natural vaginal labor


y Three categories of neonatal herpes are:
y Primary HSV-1 infections initially present as asymptomatic
y

 Lesions localized to eye, skin and mouth


y

y Incubation period is 3–5 days




 Encephalitis with or without localized skin involvement


y

y In children, it usually presents as symptomatic disease and




 Disseminated disease involving multiple organs including


y

presents as buccal and gingival mucosal ulcerations




CNS
y In adults, it manifests as pharyngitis and tonsillitis
y Cause of mortality is disseminated disease with viral pneu-
y

y When there is a recurrent lesion, it presents as cluster of


y

monitis or intravascular coagulopathy


y

vesicles seen at the corner of lip.


Clinical findings in HSV infection:
Keratoconjunctivitis • Fever blister or Herpes febrilis

y HSV infections in the eye lead to severe keratoconjunctivitis


y • Herpetic whitlow

y It produces classical dendritic keratitis and corneal ulcers


y • Eczema herpeticum

y Recurrent keratitis may lead to permanent blindness.


y • Kaposi’s varicelliform eruption

• Gingivostomatitis

Genital Herpes • Most commonly affected site – buccal mucosa


• Dendritic ulcers in cornea


y Genital infections are usually caused by HSV-2

• HSV encephalitis – most common sporadic acute viral


y

y Exposure to HSV through oral or genital sexual contact


encephalitis
y

permits the entry of the virus through the skin abrasions or


• Postnatal infections are mostly due to HSV 1
minor trauma. The virus then enters the epidermal cells and

multiplies rapidly.
y First episode herpetic lesions presents with fever, pain,
y
Laboratory Diagnosis
itching, painful micturition, discharge per vaginum and y y Tzanck smear: Multinucleated giant cells
associated inguinal lymphadenopathy. But the presentation y y Intranuclear type A inclusion bodies (Lipschutz bodies)
is usually subclinical. y y Virus culture – human diploid fibroblasts cell lines
y Once infected the patient presents subclinical infection
y
y y ELISA
and later recurrent infections are more common when the y y PCR
immune condition of the patient gets altered.

Laboratory Diagnosis of Genital Herpes


y Direct Microcopy: The base of the genital lesion is scraped
y

off and the smear is stained with Giemsa or Toluidine blue


stain. The smear reveals multinucleated giant cells with intra-
nuclear inclusion called Cowdry type A bodies.
y Virus isolation: Scrapings or Swabs collected and transported
y

in a suitable condition are inoculated in the cell culture


lines like human diploid fibroblast lines, Vero cell lines and
cytopathic effects can be seen as early as 16 hours.
y Serology: IgM Antibodies to HSV infections occurs in 4 to
y

7 days after infection and reaches a peak in 2 to 4 weeks. It


can be detected by neutralization tests, immunofluorescent
technique and Enzyme Linked Immunosorbent Assay
(ELISA). The drawback of serological tests is IgG antibodies
persist for lifelong with fluctuations and it is helpful only for
research purposes.
Figure 1: Tzanck smear with multinucleated giant cells
(Courtesy: CDC/ Joe Miller) 223
Treatment Treatment
Unit 3     Virology

y Acyclovir (200 mg 5 times/day or 400 mg tid) is the drug of


y
y Prophylaxis: Live varicella vaccine – Oka strain – subcategory
y

choice for herpetic lesions. injection – two doses given at 1 month apart for adults and
y It is a nucleoside analog which inhibits the viral DNA
y
single dose for children
synthesis y Gamma globulin of high VZV antibody titers helps
y

y The drug suppresses the severity of the clinical manifestations


y
development of severe illness – especially given for adults in
and it reduces the recurrent infections household contacts
y However, virus remains latent in the sensory ganglia
y
y Antiviral compounds like: Acyclovir, Valacyclovir, Famciclovir
y

throughout the life. Acyclovir resistant strains have been y Acyclovir prevents the development of systemic illness in im-
y

identified. munosupporessed patients. It also prevents latent infections.

VARICELLA ZOSTER VIRUS Shingles


y VZV causes a highly contagious disease
y
y It is a latent infection of VZV–
y

y It causes chicken pox, shingles or zoster infection (latent)


y
y Virus remains in the sensory ganglia for years and later
y

present as herpes zoster or shingles in adults


Pathogenesis y Reactivation occurs due to stimuli like fever or immunosup-
y

pression
y Virus enters via respiratory route or conjunctiva
y

y Severe neuritic pain occurs


y

y After initial multiplication in the regional lymph nodes, virus


y

y Typically affects → areas innervated by spinal cord segments


y

goes to liver and spleen and do secondary multiplication of D3 to L2 and ophthalmic branch of trigeminal nerve
y Secondary viremia transports the virus by infected mono
y

y Ramsay hunt syndrome → Herpes zoster affecting facial


y

nuclear cells to skin – at that time rash starts developing. nerve → leads to eruption in the lympanic membrane,
y Epithelial cells start swelling, leading to ballooning
y

external auditory canal and facial palsy


degeneration and accumulation of tissue fluids leading to
vesicle formation.

Clinical Features
y Incubation period is usually 10–21 days
y

y Earliest features that coincides with primary viremia are fever


y

and malaise
y It is then followed by lesions that goes into → rash in trunk
y

and spreads
y Stages of rash → Macule → Papule → Vesicle → Pustule and
y

scab
y Rash is centripetal in distribution; it stays for 5 days.
y

y Following varicella → Reye’s syndrome can occur in children


y

who takes salicylates


y Complications:
y

 Pneumonia


 Encephalitis


Varicella pneumonia is the most common complication seen in


neonates, adults and immunocompromised patients
Figure 2: Herpes zoster infection [Courtesy: CDC/National
y Chicken pox in pregnancy:
y
Institute of Allergy and Infectious Diseases (NIAID)]
 Early trimester – Fetal varicella syndrome – cicatrizing


skin lesions, hypoplasia of the limbs, chorioretinitis and


CNS defects
 Late trimester or near delivery – Congenital varicella


 Neonatal varicella – Give VZV antiserum or chemotherapy




immediately after birth

Laboratory Diagnosis
y y Direct fluorescent antigen detection gives rapid diagnosis
y y PCR assays are sensitive and specific
y y Tzanck smears can be seen – multinucleated giant cells Figure 3: Primary and latency of HSV and VZV
y VZV PCR is the ideal method to diagnose VZV encephalitis
224
y
CYTOMEGALOVIRUS

Chapter 27     Herpesviruses


y It is also called as salivary gland viruses
y

y Named as megalo because it causes enlargement of infected


y

cells and produce prominent intranuclear inclusions (Owl’s


eye inclusions)
y Largest of the herpesvirus family; it has the largest genetic
y

content of the human herpesviruses


y It has a strict host specificity E.g.: Human cytomegaloviruses
y

can be grown only in human fibroblast cultures.

Pathogenesis Figure 4: Owl’s eye inclusion bodies (Courtesy: CDC/ Rosalie B.


y CMV is transmitted from person to person by close contacts
y
Haraszti, MD)
y Incubation period is around 4–8 weeks
y

y Virus causes systemic infections and present as infectious


y
Treatment
mononucleosis like syndrome in healthy individuals y Ganciclovir and Foscarnet
y Immune status of individual has a direct correlation with the
y

y Ganciclovir: A nucleoside helps to treat life-threatening CMV


presentation of disease in CMV
y

infections
y Infections in immunosuppressed hosts are more severe than
y

y In bone marrow transplantation patients, Ganciclovir therapy


healthy hosts.
y

prevents disseminated CMV infections


y Glycoprotein: Present on the surface of cytomegalovirus →
y

y Cidofovir and Foscarnet are given for CMV retinitis and


acts as receptor of immunoglobulin (Fc portion)
y

Ganciclovir resistant strains.


• Routes of transmission:
EPSTEIN BARR VIRUS

ƒ Salivary secretions
ƒ

ƒ Sexual contact
ƒ

y Unique herpes virus that has association and causation with


y

ƒ Blood transfusion
ƒ

carcinomas
ƒ Organ transplantation
ƒ

y There are two major types of EBV, type A and B


y

Clinical Features Pathogenesis


y Intrauterine infection of CMV – fetal death or cytomegalic in-
y y Major target cell for EBV is the B lymphocytes
y

clusion disease – leads to hepatosplenomegaly, microceph- y Only humans have B cells that have receptors – CD21 for this virus
y

aly, jaundice, thrombocytopenic purpura, hemolytic anemia y The virus infects the B cells
y

y Most important cause for microcephaly in children is CMV


y  Causes polyclonal activation of B cells


y Infection in children is mainly due to transfusion of CMV


y  Transformation of T cells – Atypical lymphocytes


infected blood (Post-transfusion mononucleosis) y Immortalization or transformation of lymphocytes occurs,


y

y Heterophile, antibody negative, infectious mono nucleosis is


y which leads to malignancies pathogenesis of EBV infection
due to cytomegalovirus
Table 4:  Type of hosts and their clinical features
Type of hosts Clinical features
Normal healthy hosts Infectious mononucleosis like syndrome
Immunocompromised Pneumonia, colitis, retinitis, hepatitis
hosts and even disseminated diseases
Congenital and Cytomegalic inclusion disease
perinatal infections

Laboratory Diagnosis
y PCR assays are the main stay of diagnosis from samples like
y

blood, CSF and urine: It gives the quantitative viral load to


estimate the severity of disease
y Shell vial cultures: Done unique for CMV virus: This allow
y

detection of CMV antigen using fluorescent antibodies before


the development of CPE (hence it helps in rapid diagnosis)
y Microscopy showing: Owl’s eye inclusions
y

y ELISA for screening blood donors and organ donors – detects


y

225
IgG antibodies and IgM antibodies Figure 5: Pathogenesis of EBV infection
Clinical Features y It spreads through saliva
Unit 3     Virology

y There are two variants – A and B


y Incubation period varies from 30–50 days
y

 Variant A: has no clear role in disease


y

y Source of infection – saliva (Kissing disease)




 Variant B: Causes exanthema subitum or roseola


y

y Most of the humans get subclinical infection – around 95% of




infantum or sixth disease; infection is most commonly


y

world’s population have antibodies to EBV


seen in 6 months to 3 years of age. Fever along with
y Typical illness presents as self-limiting and last for 2–4 weeks
widespread maculopapular rashes are seen in 10% of
y

Clinical illness cases.


• Infectious mononucleosis – Glandular fever y Both the variants has a clinical role in causing bone marrow
y

suppression following organ transplantation


• Oral hair leukoplakia in AIDS


• Malignancies:

ƒ Burkitt’s lymphoma
ƒ
HUMAN HERPES VIRUS 7
ƒ Lymphomas in AIDS and transplant patients
y It also spreads through saliva
ƒ

ƒ Hodgkins lymphoma
y

y Shares the same CD4 receptor as that of HIV


ƒ

ƒ Nasopharyngeal carcinoma (Especially in Chinese)


y

y Hence seen in HIV infection – coinfection leads to extreme


ƒ

ƒ Anaplastic gastric carcinoma


y

immune suppression
ƒ

ƒ T/NK cell lymphoma


y It has an associated with exanthem subitum and febrile
ƒ

• Duncan’s syndrome or XLP – X linked lymphoproliferative y

convulsions

disorder.

Laboratory Diagnosis HUMAN HERPES VIRUS 8


y Seen in patients with Kaposi’s sarcoma and also without
y EBV can be isolated from saliva, peripheral blood or lymphoid
y

kaposi
y

tissues – but it needs lots of time and facilities therefore not


y Also called Kaposi sarcoma associated herpesvirus
performed as routine diagnostic part
y

y Virus has genes related to cellular regulatory genes involved in


y Peripheral smear from IMN patients shows Atypical lym-
y

cell proliferation, apoptosis and host responses – Molecular


y

phocytes (T cells): Accounts for 20% of the total lymphocytes


piracy of cell regulatory genes
seen
y Kaposi sarcoma is defined as multifocal pigmented sarcoma –
y CBC shows prominent leukocytosis
y

it is a mucocutaenous neoplasm
y

y Serological tests:
y Kaposi sarcoma was the most common tumour seen in HIV
y

 Paul-Bunnel test (Heterophile agglutination test)


y

infected before the advent of HAART therapy




 Differential agglutination test


y HHV 8 is also found in lymphoma cells from AIDS related


 ELISA
y

B cell lymphomas termed as Body cavity associated B




 Definitive diagnosis: IgM antibody to VCA – Viral capsid


lymphomas or primary effusion lymphoma


antigen
y It is also associated with Castleman’s disease which is
 EBNA: Antibody to the EB nuclear antigen
y

a lymphoproliferative disorder that is localized in the




y Molecular assays (PCR) can detect virus in blood, body fluids


mediastinum, mesenterium and in peripheral lymph nodes
y

and tissues

Infectious mononucleosis syndrome caused by HHV8 is associated with:


• Kaposi sarcoma
• EBV

• Body cavity associated B lymphomas or primary effusion


• CMV

lymphoma

• Toxoplasmosis
• Castleman’s disease

• Non infectious stimuli


Treatment HERPESVIRUS SIMIAE


y Acyclovir has little value in treatment
y Also called B virus
y

y Rituximab is helpful in EBV associated B cell tumors


y

y Infection occurs due to monkey bite


y

y Causes fatal ascending myelitis


HUMAN HERPES VIRUS 6
y

y y Official name is cercopithecine herpes virus 1


y Human herpes virus 6 was first isolated from B cells
y

y But it infects CD4 T cells; Receptor for HHV 6 is CD46


y

226
MULTIPLE CHOICE QUESTIONS

Chapter 27     Herpesviruses


1. Lipschutz bodies are seen in: (Recent Pattern Dec 2016) 17. Shingles are seen in: (Recent Pattern Dec 2012)
a. Hodgkin’s disease b. Viral hepatitis a. IMN b. Herpes zoster
c. Herpes d. Influenza c. Chicken pox d. Small pox
2. Herpes simplex virus is: (Recent Pattern Dec 2014) 18. Burkitts’s virus is: (Recent Pattern Dec 2012)
a. Single stranded DNA b. Double stranded DNA a. EBV b. HPV
c. Single stranded RNA d. Double stranded RNA c. HIV d. HAV
3. HSV affects: (Recent Pattern June 2014) 19. A patient with sore throat has positive Paul Bunnell test
a. Basal ganglia b. Temporal lobe The causative organism is: (Recent Pattern Dec 2012)
c. Cerebellum d. Pons a. EBV b. Herpes virus
4. Herpes simplex virus causes all, except: c. Adenovirus d. CMV
 (Recent Pattern Dec 2013) 20. Treatment of Herpes zoster: (Recent Pattern Dec 2013)
a. Encephalitis b. Pharyngitis a. Zidovudin b. Valacyclovir
c. IMN d. Whitlow c. Ribavarin d. Nevirapine
5. Kaposi sarcoma is caused by: (Recent Pattern Dec 2013) 21. Rash of chicken pox is: (Recent Pattern July 2016)
a. Human herpes virus-2 b. Human herpes virus-4 a. Deep seated b. Centripetal distribution
c. Human herpes virus-6 d. Human herpes virus-8 c. Monomorphic d. Umblicated
6. Which of the following belongs to Herpesviridae: 22. Varicella zoster remains latent in: (Recent Pattern 2010)
 (Recent Pattern Dec 2016) a. Lymphocytes b. Monocytes
a. Variola b. Adenovirus c. Trigeminal ganglion d. Plasma cells
c. HPV d. RK virus 23. Congenital varicella zoster infection causes:
7. HHV-6B causes.  (Recent Pattern July 2016)
a. Carcinoma cervix b. Carcinoma endometrium a. Microcephaly b. Limb hypoplasia
c. Clear cell carcinoma d. Focal encephalitis c. IUGR d. All of the above
8. Human B-cell lymphotropic virus belongs to: 24. Most common infection in transplant recipients:
 (Recent Pattern July 2016) (Recent Pattern 2018)
a. Picornavirus b. Pox virus a. Cytomegalovirus b. Epstein Barr virus
c. Reovirus d. Herpes virus c. Hepatitis virus d. Polyoma virus
9. Castleman disease is associated with: 25. Shingles caused by infection with: (Recent Pattern 2018)
 (Recent Pattern July 2015) a. Varicella zoster virus b. Herpes simplex virus
a. HSV b. CMV c. Human papilloma virus d. Ebstein Barr virus
c. EBV d. HHV – 8 26. Herpes zoster multiply in: (Recent Pattern 2018)
10. Varicella zoster attacks: (Recent Pattern June 2014) a. Dorsal root ganglion b. Peripheral nerve
a. B-cell b. T cells c. Epithelium of skin d. Pharyngeal epithelial cells
c. NK d. Neurons 27. A neonate presented with periventricular calcification
11. Owl’s eye intranuclear inclusion body is seen in: on CT brain; Best method of diagnosis of etiological
 (Recent Pattern Aug 2013) agent is: (AIIMS Nov 2017)
a. Herpes zoster b. Herpes simplex a. Biopsy b. CSF
c. CMV d. EBV c. Blood d. Urine
12. Most common complication of chicken pox: 28. A 52-year-old man has undergone lung transplantation.
 (Recent Pattern Dec 2015) Two months after transplantation, he developed
a. Bacterial infection b. Meningitis pulmonary symptoms and diagnosed as having bilateral
c. Pneumonia d. Nephritis diffuse interstitial pulmonary pneumonitis and bron-
13. Lymphocytosis with atypical lymphocytes are seen in chiolitis. Which of the following etiological agent is
infection with. (Recent Pattern Dec 2012) responsible in this condition? (PGI pattern 2017)
a. HSV b. HBV a. Cytomegalovirus
c. EBV d. RSV b. Herpes Simplex virus
14. EBV action in nasopharynx is through: c. Epstein Barr virus
 (Recent PatternNov 2015) d. Varicella zoster virus
a. CD 3 b. CD 4 e. Rhino virus
c. CD 8 d. CD 21 29. HHV-8 is associated with all, except: (AIIMS May 2018)
15. Infectious mononucleosis syndrome is caused by: a. Kaposi sarcoma 
 (Recent Pattern July 2016) b. Adult T cell leukemia
a. HPV b. Adenovirus c. Castleman’s disease
c. HSV-1 d. CMV d. Primary effusion lymphoma
16. Paul Bunnell test for: (Recent Pattern Dec 2012) 30. Herpes zoster oticus is caused by:  (CET 2018)
a. Malta fever b. Typhus fever a. EBV b. CMV
227
c. Enteric fever d. Infectious mononucleosis c. VZV d. HSV-1
ANSWERS AND EXPLANATIONS
Unit 3     Virology

1. Ans.  (c) Herpes


Official name Common name
Ref: Ananthanarayan and Paniker’s Textbook of Microbiol- Herpesvirus type 1 HSV – 1 – Herpes simplex virus 1
ogy – 10th ed – Page 472
Herpesvirus type 2 HSV – 2 – Herpes simplex virus 2
• Herpes viruses produce Cowdry type A intra nuclear
Herpesvirus type 3 Varicella Zoster virus

inclusion bodies – named as Lipschutz bodies


Herpesvirus type 4 Epstein Barr virus
2. Ans.  (b) Double stranded DNA
Herpesvirus type 5 Cytomegalovirus
Ref: Ananthanarayan and Paniker’s Textbook of Microbiol- Herpesvirus type 6 Human B cell lymphotropic virus
ogy – 10th ed – Page 472
Herpesvirus type 7 R K virus
• Enveloped DNA viruses
Herpesvirus type 8 Rhadinovirus

•• Icosahedral capsid
• ds DNA
7. Ans.  (d) Focal encephalitis

•• The linear DNA has a sequence arrangement which is


unique for herpes viruses Ref: Ananthanarayan and Paniker’s Textbook of Microbiol-
ogy – 10th ed – Page 483
3. Ans.  (b) Temporal lobe
• Human herpes virus 6 – has two variants A and B

Ref: Ananthanarayan and Paniker’s Textbook of Microbiol- • HHV – 6 B – causes exanthema subitum or roseola infan-

ogy – 10th ed – Page 475 tum or sixth disease


• The most common sporadic acute viral encephalitis in • In older age groups: It leads to focal encephalitis

• In immunodeficiency: It causes pneumonia and dis-


most of the world is HSV encephalitis •

• HSV usually affects the temporal lobe; it also affects



seminated disease
sacral ganglion leading to sacral autonomic dysfunction
8. Ans.  (d) Herpes virus
– leading to transverse myelitis or Guillian Barre
syndrome Ref: Ananthanarayan and Paniker’s Textbook of Microbiol-
ogy – 10th ed – Page 473
4. Ans.  (c) IMN
• Already explained Q.6

Ref: Ananthanarayan and Paniker’s Textbook of Microbiol-


ogy – 10th ed – Page 475 9. Ans.  (d) HHV – 8

Infections caused by Herpes simplex viruses are as follows: Ref: Ananthanarayan and Paniker’s Textbook of Microbiol-
• Fever blister or Herpes febrilis

ogy – 10th ed – Page 483
• Herpetic whitlow

• HHV: 8 causes a heterogenous group of lympho prolifer-

• Eczema herpeticum
• ative disorder – Castleman disease
• Kaposi’s varicelliform eruption

• It is more of association than causation

• Gingivostomatitis

• It is more commonly seen in HIV patients

• Most commonly affected site – buccal mucosa


10. Ans.  (d) Neurons

• Dendritic ulcers in cornea


• HSV encephalitis: Most common sporadic acute viral



Ref: Ananthanarayan and Paniker’s Textbook of Microbiol-
encephalitis ogy – 10th ed – Page 477
IMN: Infectious mono nucleosis – caused by Epstein Barr • Latent infection of Varicella Zoster: Virus remains in

virus the sensory ganglia for years and later present as herpes
zoster or shingles in adults
5. Ans.  (d) Human herpes virus-8
• Reactivation occurs due to stimuli like fever or immu-

Ref: Ananthanarayan and Paniker’s Textbook of Microbiol- nosuppression


ogy – 10th ed – Page 483 • Severe neuritic pain occurs – called as Herpes zoster or

Shingles
Human herpes virus 8: Seen in patients with Kaposi’s
sarcoma and also without Kaposi; Also called as Kaposi 11. Ans.  (c) CMV
sarcoma associated herpes virus
Ref: Ananthanarayan and Paniker’s Textbook of Microbiol-
6. Ans.  (d) RK virus ogy – 10th ed – Page 480
Ref: Ananthanarayan and Paniker’s Textbook of Microbiol- • Cytomegalovirus causes enlargement of infected cells

ogy – 10th ed – Page 473 and produce prominent intra nuclear inclusions (Owl
228 eye inclusions)
18. Ans.  (a) EBV

Chapter 27     Herpesviruses


12. Ans.  (c) Pneumonia
Ref: Ananthanarayan and Paniker’s Textbook of Microbiol- Ref: Ananthanarayan and Paniker’s Textbook of Microbiol-
ogy – 10th ed – Page ogy – 10th ed – Page 481
• Following varicella → Reye’s syndrome can occur in

• EBV causes Burkitt lymphoma – hence named as

children who takes salicylates Burkitt’s virus


• Complications: Pneumonia and encephalitis. Usually
19. Ans.  (a) EBV

occurs in older age children and adults


• Pneumonia is the most common complication

Ref: Ananthanarayan and Paniker’s Textbook of Microbiol-
• In pregnancy – it causes fetal varicella syndrome

ogy – 10th ed – Page
Ref: Already explained Q.16
13. Ans.  (c) EBV
Ref: Ananthanarayan and Paniker’s Textbook of Microbiol- 20. Ans.  (b) Valacyclovir
ogy – 10th ed – Page 483
Ref: Ananthanarayan and Paniker’s Textbook of Microbiol-
• The virus infects the B cells

ogy – 10th ed – Page 476


ƒ Causes polyclonal activation of B cells
• Topical infections – Idoxuridine
ƒ

ƒ Transformation of T cells – Atypical lymphocytes


• Encephalitis – intravenous acyclovir


ƒ

14. Ans.  (d) CD 21 • More effective oral agents – Valaciclovir and famciclovir

Ref: Ananthanarayan and Paniker’s Textbook of Microbiol- 21. Ans.  (b) Centripetal distribution
ogy – 10th ed – Page 482
Ref: Ananthanarayan and Paniker’s Textbook of Microbiol-
• EBV affects only humans

ogy – 10th ed – Page 477


• Because only humans have B cells that have receptors –
• Varicella zoster virus enters via the respiratory route →

CD21 for this virus •

lesions starts → rash in trunk and spreads


15. Ans.  (d) CMV • Stages of rash → Macule → Papule → Vesicle → Pustule

and scab
Ref: Ananthanarayan and Paniker’s Textbook of Microbiol- • Rash is centripetal in distribution

ogy – 10th ed – Page 483


22. Ans.  (c) Trigeminal ganglion
Infectious mononucleosis syndrome
EBV Ref: Ananthanarayan and Paniker’s Textbook of Microbiol-
CMV ogy – 10th ed – Page 478
Toxoplasmosis • Virus remains in the sensory ganglia for years and later

Non infectious stimuli present as herpes zoster or shingles in adults


• Typically affects → areas innervated by spinal cord

16. Ans.  (d) Infectious mononucleosis segments of D3 to L2 and ophthalmic branch of


trigeminal nerve
Ref: Ananthanarayan and Paniker’s Textbook of Microbiol-
ogy – 10th ed – Page 482 23. Ans.  (d) All of the above
•• Standard diagnostic test for IMN is Paul Bunnel test
•• Heterophile agglutination test Ref: Ananthanarayan and Paniker’s Textbook of Microbiol-
•• These antibodies agglutinate sheep erythrocytes ogy – 10th ed – Page 479
•• An agglutination titre of 100 or above suggests IMN • Varicella infection in early trimester – Fetal varicella

syndrome – cicatrizing skin lesions, hypoplasia of the


17. Ans.  (b) Herpes zoster limbs, chorioretinitis and CNS defects
Ref: Ananthanarayan and Paniker’s Textbook of Microbiol- 24. Ans.  (a) Cytomegalovirus
ogy – 10th ed – Page 478
• Latent infection of Varicella zoster virus remains in the

Ref: Mandell’s principle of infectious disease – 8th edition –
sensory ganglia for years and later present as herpes Page 4618
zoster or shingles in adults • Cytomegalovirus belongs to Herpesviridae

• Reactivation occurs due to stimuli like fever or immu-



• It is the largest virus among the herpes virus

nosuppression • It is the most common organism causing intrauterine


• Severe neuritic pain occurs



infection
• It is the most common pathogen complicating organ

transplantation

229
25. Ans.  (a) Varicella zoster virus 28. Ans.  (a) Cytomegalovirus
Unit 3     Virology

Ref: Ananthanarayan and Paniker’s Textbook of Microbiol- Ref: Jawetz Microbiology – page 471 – 27th edition
ogy – 10th ed – Page 478 • Cytomegalovirus, a type of herpes group of virus is the

• Latent infection of Varicella zoster virus remains in the


• most common cause of congenital infections.
sensory ganglia for years and later present as herpes • It has the largest genetic content among the herpes

zoster or shingles in adults group of viruses


• Reactivation occurs due to stimuli like fever or immu-
• • It has a cell surface glycoprotein, which acts as an Fc

nosuppression receptor that can bind non specifically with the Fc


• Severe neuritic pain occurs
• portion of the immunoglobulin
• CMV infection forms a characteristic cytopathic effect

26. Ans.  (a) Dorsal root ganglion – perinuclear cytoplasmic inclusions in addition to
Ref: Ananthanarayan and Paniker’s Textbook of Microbiol- intranuclear inclusions
ogy – 10th ed – Page 478 • Normal hosts – infectious mononucleosis like illness

• Immunosuppressed, post transplantation patients –


• Varicella zoster infection usually affects children

pneumonia, bronchiolitis, hepatitis, colitis, disseminat-


• When it affects adults – it is mainly due to latent infection

ed infection
of zoster • Congenital infection: IUGR, hepatomegaly, splenomeg-
• This latency is called as Herpes zoster or shingles

aly, chorioretinitis, thrombocytopenia, microcephaly,


• This is mainly due to reactivation of latent infection from

calcifications.
the dorsal root ganglion • Diagnosis: CMV: Owl eye shaped inclusion bodies;
• The dermatomes from T3 to L3 and trigeminal branch of

serological tests, molecular diagnosis


ophthalmic nerve are commonly involved • Treatment: Ganciclovir

27. Ans.  (b) CSF 29. Ans.  (b) Adult T cell leukemia
Ref: Jawetz T.B of medical microbiology – 27th edition – Ref: Ananthanarayan and Paniker’s Textbook of Microbiol-
page 473 ogy – 10th ed – Page 483
• Many women infected previously with CMV show

HHV8 is associated with:


reactivation and begin to excrete the virus from the • Kaposi sarcoma

cervix during pregnancy. • Body cavity associated B lymphomas or primary effusion


• At the time of delivery through the infected birth canal,

lymphoma
infants may become infected, although they possess high • Castleman’s disease.

titers of maternal antibody acquired transplacentally.


• These infants begin to shed virus at about 8–12 weeks of

30. Ans.  (c) VZV
age. They continue to excrete the virus for several years.
• Hence urine and blood samples are not good choice of

Ref: Ananthanarayan and Paniker’s Textbook of Microbiol-
diagnosis ogy – 10th ed – Page 478
• Best specimen is CSF to isolate the organism
• For explanation please refer Q. 26.
• DD: Toxoplasmosis

230
28
Adenovirus
INTRODUCTION y When adenoviruses enters through GIT, the infections are

y
usually subclinical (exception: types 40, 41)
y Adenoviruses are named because it was first isolated from y Adenoviruses usually elicit long lasting immunity
y
adenoids

y
y Cytopathic effect that is seen in cultures unique to adenovi-
y Adenoviruses have the classical icosahedral symmetry; they

y
ruses are: Grape like clusters
y
are so unique by having a structure called as fiber projecting
from the vertices
y Surface of adenoviruses has visible knobs called as capsomer- CLINICAL FEATURES
y
es
y Adenoviruses are the causative agents for 5% of the acute
y The capsomeres are surrounded by pentons at 12 vertices and

y
respiratory diseases in children
y
at remaining places by hexons
y Most of the infections are mild and self-limiting
y Hexons are group-specific antigens y
y They can cause conjunctivitis in summer camps in children
y
y Pentons are type-specific antigens
y
y A more serious disease is epidemic keratoconjunctivitis
y
y It has a specific structure resembling space-vehicle
y
seen in adults, which are highly contagious and may lead to
y
appearance
keratitis and opacification.
y Adenovirus replicates only in cells of epithelial origin
y In transplant recipients, it may cause respiratory diseases
y
y
Table 1:  Serotypes causing adenoviral infections
Respiratory disease in children 1, 2, 5, 6
Sore throat, Febrile cold, Pneumonia 3, 4, 7, 14, 21
Pharyngoconjunctival fever 3, 7, 14
Acute respiratory disease (occurs in military 4, 7, 21
recruits)
Follicular or swimming pool conjunctivitis 3, 7
Epidemic keratoconjunctivitis or Shipyard eye 8, 19, 37
Diarrhea 40, 41
Acute hemorrhagic conjunctivitis 11, 21
Infection in bone marrow transplants 34, 35
y Diarrheal illness is caused by fastidious adenoviruses
y
Figure 1: Electron microscopic image of adenovirus (Serotypes 40, 41)
 Can be demonstrated in electron microscopy but it cannot
(Courtesy: CDC/ Dr. G. William Gary, Jr.)

be grown in conventional cell culture methods
 Special culture methods: Trypsinized monkey kidney
PATHOGENESIS

cells; Transformed human embryonic kidney cells
y Adenovirus also causes mesenteric adenitis and intussus-
y Adenovirus is host specific
y
ception in children.
y
y Human adenoviruses will grow only in human tissues origin y Adenoviruses are useful as vectors in gene therapy (onco-
y
cell culture
y
lytic therapy with viral vectors)
y It usually affects the epithelial cells of the respiratory tract,
y
eye, GIT and Urinary tract
y They do not spread beyond the regional lymph nodes; hence
LABORATORY DIAGNOSIS
y
the infections are usually localized. y Samples are collected based on the clinical presentation
y
y Duration of adenovirus shedding differs in each clinical
y
illness:
Table 2:  Duration of shedding of virus in different type of y Fastidious adenoviruses (40, 41) can be detected by:
Unit 3     Virology

illness  Direct examination of fecal extracts in electron microscopy




 ELISA 

Type of illness and samples Duration of shedding of  Latex agglutination assay




virus
Respiratory samples in adults 1 – 7 days TREATMENT
Respiratory samples in children 3 – 6 weeks
y There is no specific drug for adenoviruses
y

Pharyngoconjunctival fever 3 – 14 days y Mostly infections are self limiting and not life threatening
y

Keratoconjunctivitis 2 weeks
Immunocompromised patients 2 – 12 months ADENO-ASSOCIATED VIRUSES
– blood, urine, throat and stool
y y Defective or Dependoviruses
y Virus isolation needs human cells – most commonly used is
y
y y Belongs to parvoviridae family
primary human embryonic kidney cells y y Four types are seen
y Shell vial technique (similar to CMV) can be employed
y
y y Types 1, 2, 3 are human origin and type 4 is simian origin
y PCR helps in diagnosis of Adenovirus in respiratory samples
y
y y Unable to replicate on own and they will replicate in cells
infected with adenovirus

MULTIPLE CHOICE QUESTIONS

1. Adenovirus causes all, except: (Recent Pattern Dec 2012) 2. Pharyngoconjunctival fever is caused by:
a. Hemorrhagic cystitis a. Adenovirus 3 and 7 (Recent Pattern Dec 2012)
b. Diarrhea b. Adenovirus 11, 21
c. Respiratory tract infection c. Adenovirus 40, 41
d. IMN d. Adenovirus 8, 19

ANSWERS AND EXPLANATIONS

1. Ans.  (d) IMN 2. Ans.  (a) Adenovirus 3 and 7


Ref: Ananthanarayan and Paniker’s Textbook of Microbiol- Ref: Ananthanarayan and Paniker’s Textbook of Microbiol-
ogy – 10th ed – Page 487 ogy – 10th ed – Page 487
• IMN - Infectious mononucleosis otherwise called as

Serotypes causing adenoviral infections


glandular fever – caused by Ebstein Barr virus, which
belongs to Herpes virus family Respiratory disease in children 1, 2, 5, 6
Diseases caused by adenovirus are: Sore throat, Febrile cold, Pneumonia 3, 4, 7, 14, 21
Respiratory disease in children Pharyngoconjunctival fever 3, 7, 14
Sore throat, Febrile cold, Pneumonia Acute respiratory disease (occurs in 4, 7, 21
Pharyngoconjunctival fever military recruits)

Acute respiratory disease (occurs in military recruits) Follicular or swimming pool conjunctivitis 3, 7

Follicular or swimming pool conjunctivitis Epidermic keratoconjunctivitis or Shipyard 8, 19, 37


eye
Epidermic keratoconjunctivitis or Shipyard eye
Diarrhea 40, 41
Diarrhea, mesenteric adenitis, intussusception
Acute hemorrhagic conjunctivitis 11, 21
Acute hemorrhagic conjunctivitis
Infection in bone marrow transplants 34, 35

232
29
Picornaviruses
y Picornaviruses have a very large group of viruses and Antigenic Structure
y
they include the two most important human pathogens:
y Based on the antigenic nature – Poliovirus has three types 1,
Enteroviruses and Rhinoviruses

y
2, 3
y The most serious of picorna virus is poliomyelitis.
y VP1 – has the major antigenic site – Each type of poliovirus
y
y
has a specific antibody based upon the VP1 (Type specific
PROPERTIES OF PICORNAVIRUS antibodies)
y Virion is icosahedral in symmetery; nonenveloped Table 2:  Types and characteristic of poliovirus
y
y It has ss linear RNA (++ sense) Types of Characteristics Prototype
y
y Proteins are made up of four major polypeptides, which are poliovirus strains
y
cleaved from a large precursor protein
y Surface capsid proteins are VP1 and VP3 – these are the major Type 1 Most common, Most common Brunhilde and
cause for epidemics Mahoney
y
antibody-binding sites
y VP4 is an internal protein Type 2 Endemic Lansing and MEFI
y
y Replication: Intracytoplasmic. Type 3 Leon and Saukett
y
y C antigen: Coreless or capsid antigen
CLASSIFICATION
y
y D antigen: Dense antigen
y
Table 1:  Classification of picornavirus y D antigen can be converted to C antigen by heating the virus
y
at 56°C
Enterovirus Poliovirus
Coxsackie virus A
Coxsackie virus B Pathogenesis
Echovirus y Polio virus enters the human body by means of oral ingestion
y
Enterovirus y Virus enters the nasopharynx and started multiplying. From
y
Rhinovirus Common cold virus there it goes to the alimentary tract
y Then it enters the regional lymph nodes and thus primary
Hepatovirus Hepatitis A virus
y
viremia sets in
Parechovirus y From the reticuloendothelial system – virus enters the blood
y
stream – thus secondary viremia sets
Aphthovirus Veterinary virus
y The most important of all is - virus enters the axons to the
Cardiovirus
y
CNS
y It starts multiplying in the neurons and destroys them
y
POLIOVIRUS y Poliovirus does not multiply in the muscle in vivo
y
y First change that occurs in the CNS is degeneration of nissl
Enders, Weller and Robbins was awarded Nobel prize for their
y
bodies – Chromatolysis
discovery that polio viruses can be grown in nonneural cell y Most commonly affected area – anterior horns of the spinal
culture
y
cord
General Properties y Flaccid paralysis
y
y Poliovirus structure has the typical enterovirus - that consists
Clinical Features
y
of four viral proteins – VP1 – VP4
y Symmetry – Icosahedral y Mild disease:
y
y
y Polioviruses are inactivated when heated at 55°C for 30  It is the most common form of the disease
y

minutes. Polioviruses are not affected by ether or sodium  These are called inapparent infections (90–95% will pre-

deoxycholate. sent like this)
y Polio viruses have a very restricted host range. They usually  Fever, malaise, drowsiness, headache, nausea and vomit-
y

infect humans, chimpanzees. Hence for growing them – Pri- ing occurs
mary or continuous cell lines cultures from human or mon- y Abortive poliomyelitis or minor illness
y
key is needed.
y Nonparalytic poliomyelitis– This can present as aseptic Oral Polio Vaccine
Unit 3     Virology

meningitis
y Paralytic poliomyelitis or major illness:
y
y Trivalent vaccine: Type 1, 2, 3 strains are present in the vaccine
y

 This form leads to biphasic fever with headache with



y Due to removal of type 2 strains after pulse polio immunization
y

meningitis – now WHO has introduced bivalent vaccine containing


 Presenting complaint is the flaccid paralysis with lower

type 1 and 3
motor neuron damage. y Attenuation needs to be done to prepare OPV
y

 Maximal recovery occurs within 6 months, if not residual



y Hence to differentiate whether wild virus has been converted
y

paralysis is seen. to attenuation strain – following markers are used:


y Chance of paralysis is higher in the following conditions:
y

 d marker
 Pregnancy


 rct 40


 Tonsillectomy during epidemics




 MS


 Triple vaccine injection with alum conjugation




 McBride’s intratypic antigenic marker




 Severe trauma during the incubation period.




 Best method is SEQUENCING (To differentiate wild virus




Laboratory Diagnosis from vaccine virus)


y Best method: Virus isolation in tissue culture
Vaccine Failure
y

y Virus can be isolated from throat swabs taken soon after the
y

onset of illness y Sometimes OPV can lead to VDPV – Vaccine derived polio
y

y But rectal swabs or stool samples are collected over longer


y

viral infection
periods y Original strain of OPV is changed and converted to VDPV –
y Specimen should be submitted immediately to the lab; if
y

called circulating VDPV – cVDPV


delaying – it needs to be frozen.
y Cultures using cell lines are done: cell lines used or human or
y
• Why vaccine failure occurs? Especially in tropical countries like

monkey cell lines. India:


y Cytopathic effects are seen in 3–6 days
y
ƒ Because of interference with other enteric viruses which are
ƒ

y To differentiate vaccine virus or wild virus – it needs real time


y

present in the gut


RT PCR ƒ Diarrheal diseases
ƒ

ƒ Breastfeeding immediately before or after the vaccine


Table 3:  Chance of isolation of virus in feces
ƒ

ƒ An inhibitor in the saliva


ƒ

First week 80%


Second week 50% Polio Eradication
Third week 25% y The last polio case in India was reported on January 13, 2011
y

from the state of West Bengal, and was a case of wild polio
virus-1 (WPV 1)
Polio Vaccination y Polio eradication in India was possible because of PPI – Pulse
y

Table 4:  Features of OPV and IPV polio immunization - Children less than 5 years of age are
given OPV on a single day – to achieve herd immunity
Features Oral polio Inactivated y GPEI – Global polio eradication initiative has devised a
vaccine (OPV) polio vaccine
y

plan named as Polio Eradication and Endgame strategic plan


(IPV) (2013-2018)
Type of vaccine Live attenuated Killed
Route given Orally Intramuscularly Endgame Strategic Plan
Immunity Mucosal intestinal Only systemic is y It is a comprehensive, long-term strategy to deliver a polio-
y

immunity and good; free world by 2018.


systemic y The plan is mainly for the eradication of all polio disease,
y

Possibility of reversal Yes No whether caused by wild poliovirus or circulating vaccine-


to virulent form derived poliovirus.
y Objectives of the plan are:
Any interference with Yes No
y

 Detect and interrupt all poliovirus transmission


other group of enteric


 Strengthen immunization systems and withdraw oral


viruses


polio vaccine
Vaccine-derived Yes No  Contain poliovirus and certify interruption of transmission


poliomyelitis  Plan polio’s legacy




Can be given to No Yes y Objective 2 of the endgame plan calls for the introduction
y

immunosuppressed of at least one dose of inactivated polio vaccine (IPV) into


patients? routine immunization schedules, strengthened routine
Storage Sub zero No need of hitech immunization services, and withdrawal of oral polio vaccine
234 temperature storage facility (OPV) in a phased manner, which began with the switch from
storage is must and transport trivalent to bivalent OPV in April 2016.
COXSACKIEVIRUSES y Herpangina:

Chapter 29     Picornaviruses


y

 It is a severe febrile pharyngitis seen in small children




y Coxsackieviruses are larger group of enteroviruses: which are


y

 It is mostly self-limiting.


divided into two types: A and B y Hand-Foot and mouth disease (HFMD):
y

y Now they have been classified as HEV A, B, C


y

 Characteristic lesions are seen in oral and pharyngeal area




y Coxsackie virus B is the most commonly identified causative


y

– ulcerations.
agents of viral heart disease in humans  A vesicular rash is seen in the palms and soles


y Unique feature of this group of viruses are: Infects only


y

 DD: Herpes viruses and poxviruses




suckling mice not adult mice - hence isolation can be done  But HFMD has vesicles without crusts (this helps to


only by inoculation in suckling mice differentiate from herpes and pox)


y Infection of suckling mice produces some pathological
y

 It is most commonly caused by Coxsackie A16 and B1.




changes; Based upon that Coxsackieviruses are classified into Nowadays Coxsackie A6 is also emerging as a cause of
A and B severe HFMD (which has characteristic clinical feature of
nail shedding)
Table 5:  Classification based on the pathological y Pleurodynia:
y

changes produced by the suckling mice  It is also called as epidemic myalgia




 It is usually caused by Group B viruses


Coxsackievirus Group A Coxsackievirus Group B


 Clinical features are fever, stabbing chest pain followed by




24 serotypes 6 serotypes malaise, headache and anorexia.


y Myocarditis:
In suckling mice, produces Produces patchy focal
y

 Coxsackie B viruses are the major cause of primary myo-


generalized myositis and flaccid myositis, spastic paralysis,


cardial disease in adults as well as children


paralysis – death within a week necrosis of brown fat with
y Generalized disease of infants:
pancreatitis, hepatitis,
y

 Very serious condition seen in infants where simultane-


myocarditis, encephalitis


ously multiple organs are affected


Clinically in humans, it causes It causes  This disease is typically caused by group B coxsackie virus.


• Herpangina or Vesicular
• • Epidemic myalgia or

pharyngitis pleurodynia or Bornholm


• Aseptic meningitis (A7)
• disease
Laboratory Diagnosis
• HFMD – Hand Foot Mouth
• • Myocarditis

y Virus can be isolated from throat in initial phase and then
y

disease (A16, A9, B1-3) • Pericarditis



from stool specimens
• Minor respiratory infections
• • Juvenile diabetes (B4)

y In Coxsackie virus A21 infections, largest amount of viruses
y

• Orchitis

are shedded in nasal secretions
• Post viral fatigue

y Specimens are inoculated into suckling mouse – signs of
y

syndrome illness are seen.


y Tissue culture: only few serotypes grow
y

 Group A – types 7 and 9; Group B (all types)  Grow well




in monkey kidney tissue cultures


 Group A – type 21 – grows only in HeLa cells


y Rapid detection can be achieved by molecular assays


y

Treatment
y There is no vaccine or antiviral drug available currently
y

y Treatment would be symptomatic.


y

ECHOVIRUSES
y ECHO – Entero cytopathogenic human orphan viruses
y

y 34 serotypes are there; many viruses are grouped together


y

Figure 1: Feature of HFMD in a child (Courtesy: National Center for in the past because all of them infect the human enteric tract
Immunizations and Respiratory Diseases) and they all can be recovered only by inoculation in certain
tissue cultures.
y Among the 34 types – type 10 has been identified as reovirus
Clinical Features
y

and type 28 is rhinovirus and they are removed


y Aseptic meningitis:
y
y Unique feature: All echoviruses infect only humans; they
y

 It is caused by all the types of group B coxsackieviruses and



are not pathogenic to laboratory animals.
group A coxsackieviruses (most commonly A7 and A9).
 Fever, malaise, headache, nausea and abdominal pain
235


occurs.
Pathogenesis and Clinical Findings RHINOVIRUS
Unit 3     Virology

y It enters by feco-oral route


y

y Rhinoviruses are called common cold virus


y Though thought as orphan viruses, they are the most
y

y They are the most common causative agents recovered from


common cause of aseptic meningitis
y

upper respiratory tracts


y It causes respiratory diseases in children, gastroenteritis,
y

y More than 150 serotypes are present


y

paralysis and hepatic necrosis y Unique feature: More acid labile and more heat stable than
y

y Most common lesion is rashes in children


y

other enteroviruses
y It has the property of Hemagglutination – agglutinate human
y

y Three groups: H, M and O


y

erythrocytes.
Table 7:  Groups and characteristics of rhinovirus
Diagnosis
Groups of rhinovirus Characteristics
y To diagnose the causal role of Echo viruses for a clinical
y

illness: H Grow only in human cells


 The virus should be recovered at a rate higher than the


M Grow both in human and monkey cells


recovery from healthy individuals in the same locality at
the same time O Grow only in nasal or tracheal ciliated
 Antibodies should be seen against the virus during the

epithelium
course of disease y Virus enters the human through nasal ciliated epithelial
 Virus needs to be isolated from body fluids or tissues
y

cells; these viruses are infectious only for humans and


manifesting lesions like CSF in aseptic meningitis. chimpanzees.
y After entering the upper respiratory tract, high titers of viral
y

ENTEROVIRUS shedding is seen in nasal secretions. Replication is confined


y New enteroviruses are added – which includes types 68-71
y
only to the surface epithelium of the nasal mucosa.
y Clinical features are: Malaise, headache, nonproductive
y

cough and secondary bacterial infections that may complicate


Table 6:  Newly added enterovirus
the illness
Types Diseases y Isolation is by human cell culture – MRC5 or W138.
y

EV 68 Pneumonia and bronchitis


EV 69 No human disease
EV 70 Acute hemorrhagic conjunctivitis (also caused by
Coxsackie virus A24)
EV 71 Meningitis and encephalitis, HFMD

236
MULTIPLE CHOICE QUESTIONS

Chapter 29     Picornaviruses


1. All are about polio, except: (Recent Pattern Dec 2013) 10. Herpangina is caused by: (Recent Pattern Dec 2012)
a. 99% nonparalytic b. Flaccid paralysis a. Enterovirus b. Rhinovirus
c. Aseptic meningitis d. 90% paralytic c. Myxovirus d. Rabies virus
2. Diagnosis of polio: (Recent Pattern Dec 2014) 11. Coxsackie A virus does not cause:
a. Detection of poliovirus in stool  (Recent Pattern Dec 2013)
b. Serology a. Herpangina
c. Limb wasting b. Hand, foot and mouth disease
d. AFP c. Laryngotracheobronchitis
3. The virus, which spreads by both hematogenous and d. Aseptic meningitis
neural route is: (Recent Pattern Dec 2014) 12. Juvenile diabetes is caused by:
a. Rabies virus b. Varicella zoster virus  (Recent Pattern Dec 2013)
c. Polio virus d. E.B. virus a. Adenovirus b. Enterovirus -72
4. Which viral disease is transmitted by orofecal route: c. Coxsackie virus A d. Coxsackie virus B
 (Recent Pattern Aug 2013) 13. Enterovirus species that is causing conjunctivitis:
a. Dengue b. Polio virus  (Recent Pattern 2018)
c. Hepatitis B d. Influenza virus a. Enterovirus 68 b. Enterovirus 69
5. Most common mode of transmission of polio virus: c. Enterovirus 70 d. Enterovirus 71
 (Recent Pattern Dec 2013) 14. Which causative organism is responsible for this
a. Droplet infection b. Fecal-oral route disease: (AIIMS Nov 2017)
c. Blood transfusion d. Vertical transmission
6. Coxsackie virus is: (Recent Pattern Dec 2012)
a. Herpesvirus b. Poxvirus
c. Enterovirus d. Myxovirus
7. Epidemic pleurodynia is caused by:
 (Recent Pattern July 2016)
a. Enterovirus -70 b. Coxsackie – A virus
c. Coxsackie – B virus d. Enterovirus
8. Enterovirus – 72 is: (Recent Pattern Dec 2012)
a. Hepatitis A b. Hepatitis E
c. Hepatitis G d. Hepatitis C
9. Mode of spread of enterovirus is:
 (Recent Pattern Aug 2013) a. Coxsackie virus b. Human herpes virus 7
a. Vector mediated b. Droplet infection c. Pox virus d. Molluscum contagiosum
c. Feco-oral route d. Skin contact

ANSWERS AND EXPLANATIONS

1. Ans.  (d) 90% paralytic • When a case of Acute flaccid paralysis is seen, feces

should be collected immediately and after a second


Ref: Ananthanarayan and Paniker’s Textbook of Microbiol- sample after 14 days
ogy – 10th ed – Page 493 • Virus isolation by tissue culture is the best method

• Poliomyelitis presents as aseptic meningitis or flaccid


3. Ans.  (c) Polio virus

paralysis
• Most of the cases with poliovirus infection (90-95%)

Ref: Ananthanarayan and Paniker’s Textbook of Microbiol-


becomes inapparent infections or minor illness and ogy – 10th ed – Page 492
only 1% will go for paralysis
• Polio virus enters the human body by means of oral

2. Ans.  (a) Detection of polio virus in stool ingestion (Faeco oral route)
• Virus enters the nasopharynx and started multiplying.

Ref: Ananthanarayan and Paniker’s Textbook of Microbiol- From there it goes to the alimentary tract
ogy – 10th ed – Page 493 • Then it enters the regional lymph nodes and thus

primary viremia sets in


237
• From the reticulo endothelial system – virus enters the • Herpangina or vesicular pharyngitis – caused by Group
Unit 3     Virology

• •

blood stream – thus secondary viremia sets A Coxsackie virus


• The most important of all is - virus then enters the axons
• • It is a severe febrile pharyngitis with vomiting and head

to the CNS ache


• It starts multiplying in the neurons and destroys them
• • Presence of small vesicles in the posterior pharyngeal

wall
4. Ans.  (b) Polio virus
11. Ans.  (c) Laryngotracheobronchitis
Ref: Ananthanarayan and Paniker’s Textbook of Microbiol-
ogy – 10th ed – Page 492 Ref: Ananthanarayan and Paniker’s Textbook of Microbiol-
Ref: Already expla.ined Q.3 ogy – 10th ed – Page 497

Coxsackievirus Group A Coxsackievirus Group B


5. Ans.  (b) Fecal-oral route
Clinically in humans, it It causes
Ref: Ananthanarayan and Paniker’s Textbook of Microbiol- causes • Epidemic myalgia or

ogy – 10th ed – Page 491 • Herpangina or Vesicular


• pleurodynia or Bornholm
Ref: Already explained Q.3 pharyngitis disease
• Aseptic meningitis (A7)
• • Myocarditis

6. Ans.  (c) Entero virus • HFMD – Hand Foot


• • Pericarditis

Mouth disease (A16, A9, • Juvenile diabetes (B4)


Ref: Ananthanarayan and Paniker’s Textbook of Microbiol- B1-3) • Orchitis


ogy – 10th ed – Page 491 • Minor respiratory


• • Post viral fatigue

• Coxsackie virus belongs to Picornavirus family



infections syndrome
• Two groups are there Group A and Group B

12. Ans.  (d) Coxsackie virus B


7. Ans.  (c) Coxsackie – B virus
Ref: Ananthanarayan and Paniker’s Textbook of Microbiol-
Ref: Ananthanarayan and Paniker’s Textbook of Microbiol- ogy – 10th ed – Page 498
ogy – 10th ed – Page 497 Ref: Already explained Q.11
• Group B Coxsackie virus cause epidemic myalgia or

pleurodynia or Bornholm disease 13. Ans.  (c) Enterovirus 70


• Febrile illness which causes severe pain in chest and
Ref: Ananthanarayan and Paniker’s Textbook of Microbiol-

abdomen
ogy – 10th ed – Page 498
8. Ans.  (a) Hepatitis A Types Diseases
Ref: Ananthanarayan and Paniker’s Textbook of Microbiol- EV 68 Pneumonia and bronchitis
ogy – 10th ed – Page 545
• Hepatitis A virus belongs to Hepato virus genera of

EV 69 No human disease
Picornaviridae EV 70 Acute hemorrhagic conjunctivitis (also caused
• It was originally designated as Enterovirus A; but now it’s

by Coxsackie virus A24)
a separate genera – Hepato virus
EV 71 Meningitis and encephalitis, HFMD
9. Ans.  (c) Feco-oral route
14. Ans.  (a) Coxsackie virus
Ref: Ananthanarayan and Paniker’s Textbook of Microbiol-
ogy – 10th ed – Page 492 Ref: Ananthanarayan and Paniker T.B of microbiology –
• All entero viruses enters the body through oral ingestion

10th ed – page 497
by feco oral route • Coxsackie virus Group A – causes HFMD

E.g. Polio virus, Coxsackieviruses, Echo virus • HFMD – Hand foot mouth disease

• Presence of clusters of papulo vesicular lesions on the


10. Ans.  (a) Enterovirus skin and oral mucosa


• Occurs as sporadic cases and also as outbreaks
Ref: Ananthanarayan and Paniker’s Textbook of Microbiol-

• Causative agents: Coxsackie A16, A9, B1-3


ogy – 10th ed – Page 497

238
30
Orthomyxoviruses
CHARACTERISTICS
y Enveloped, helical RNA viruses
y
y Unique feature: Eight-segmented RNA
y
y Genetic reassortment is commonly seen in members of same
y
genus
y Virus envelope has hemagglutinin and neuraminidase (HA
y
and NA)
y Influenza virus comes under orthomyxovirus
y
ANTIGENIC STRUCTURE
y Internal antigen: Ribonucleoprotein or RNP antigen and
y
M protein antigen (Both protein are type specific)
y Surface antigens: hemagglutinin and neuraminidase
Figure 1: Electron microscopic image of Influenza virus
y
y RNP antigen is stable and there is no antigenic variations
(Courtesy: CDC/ Erskine. L. Palmer, Ph.D.; M.L. Martin)
y
y Based on the types of RNP – influenza viruses are classified
y
into three types – Type A, B and C
y Major cause of epidemics and pandemics is due to influenza PATHOGENESIS AND CLINICAL FINDINGS
y
A
y Virus enters through respiratory tract ; IP is 1 to 3 days
y
Table 1:  Surface antigens y Causes Flu – symptoms are fever, headache, generalized
y
myalgia, respiratory symptoms, abdominal pain and vomiting
Haemagglutinin Neuraminidase y Current classification of influenza strains – H1N1, H2N2 and
y
Responsible for hemagglutination Responsible for elution H3N2
y Most important complication of influenza is pneumonia due
Fifteen distinct subtypes H1 – H15 Nine different subtypes
y
to bacterial super infection (Staphylococcus aureus)
N1- N9 y Cardiac complication: Myocarditis
y
Types A and B agglutinate y Influenza B: Affects children – associated with Reye’s
y
erythrocytes of fowls, humans, syndrome and Gastric flu
guinea pigs y Influenza type C: Endemic throughout the world: It causes
y
Type C agglutinates only fowl mild and asymptomatic infections
erythrocytes y Swine flu – H1N1 (Reassortant between swine virus with
y
Eurasian swine virus) – Swine origin influenza (S-OIV)
y Antigenic variations: y When influenza virus infects one individual – he develops
y
 Antigenic shift – Major antigenic change due to novel
y
antibodies; if again he gets infection with another strain;

virus strain that has come because of mutation antibodies develop not only against this strain but also against
 Antigenic drift – minor change, gradual sequential change
the previous one; This is called original antigenic sin.

in antigenic structure that occurs at intervals
 Antigenic variations are the highest in influenza type A,

less in type B and not seen in type C
AVIAN FLU y Isolation of virus – inoculation in amniotic cavity
Unit 3     Virology

y Recent method of detection – Real time and Multiplex PCR –


y

y The last recent pandemic outbreak was in Hong Kong of


y

detects the virus within a day


chicken flu in 1997 with a new strain H5N1 – caused human
cases with confirmed deaths also
y All human cases were shown to have spread directly from
y
Immunoprophylaxis
chickens y Vaccines:
y

y Wild aquatic birds – repertoire of genes of all influenza strains


y
 Inactivated vaccines: Subunit vaccine and recombinant


y Viruses do not cause any disease/ do not undergo mutational


y
vaccine
changes in birds  Live attenuated cold adapted vaccine – given intranasally


y Birds shed the virus in feces


y
before;
y Fecal contamination of ponds, lakes → spread to domestic
y
 Live vaccines – prepared by using temperature sensitive


birds → spread to pigs → recombination takes place in mutants; these mutants will grow in the nasopharyngeal
pigs between swine and avian strains. mucosa where the temperature is usually 32-34°C but will
y Unique features: Once an antigenic variant emerged, it
y
die in lungs where the temperature is 37°C
completely displaced pre-existing strain
New Flu vaccines: (Source: https://www.cdc.gov/flu/protect/
keyfacts.html)
Recent pandemic – H1N1 2009 Flu: Trivalent flu vaccines:
(Source: CDC) • Standard-dose trivalent shots that are manufactured using

y 2009 H1N1 was first detected in the United States in April


y
virus grown in eggs.
2009. • One trivalent vaccine formulation can be given with a jet

y This virus was a unique combination of influenza virus genes


y
injector, for persons aged 18 through 64 years.
never previously identified in either animals or people. • A high-dose trivalent shot, approved for people 65 and older.

y The virus genes were a combination of genes most closely


y
• A recombinant trivalent shot that is egg-free, approved for

related to North American swine-lineage H1N1 and Eurasian people 18 years and older, including pregnant women.
lineage swine-origin H1N1 influenza viruses • A trivalent flu shot made with adjuvant approved for people

CDC had identified some notable features about the 2009 65 years of age and older
H1N1 virus: Quadrivalent flu vaccines:
y Researchers had confirmed earlier testing that the 2009
• Quadrivalent flu shots approved for use in different age
y

H1N1 influenza virus was a quadruple-reassortant virus, •

meaning that it contained virus genes that originated from groups, including children as young as 6 months.
four different influenza virus sources. • A quadrivalent vaccine, which is injected into the skin instead

y Testing of a number of the virus samples submitted to CDC of the muscle and uses a much smaller needle than the
regular flu shot. It is approved for people 18 through 64 years
y

showed that they were very similar, which means they likely
originated from the same source. of age.
y Laboratory testing showed that the 2009 H1N1 influenza virus • A quadrivalent flu shot containing virus grown in cell culture,

which is approved for people 4 years of age and older.


y

did not have any 1918-like markers that had been associated
with increased risk of severe disease. • A recombinant quadrivalent flu shot approved for people 18

y Testing also did not find genetic markers that were previously
y
years of age and older, including pregnant women.
associated with high death rates in people infected with the
avian influenza A (H5N1) virus in other countries. Chemoprophylaxis
y Amantadine
y

FDA Approved Treatment During Pandemic y Rimantadine


y

 Mechanism of action of above drugs – block the viral M2


y Oseltamivir to treat children younger than 1 year of age and to 

protein
y

help prevent influenza in children 3 months to 1 year of age,


 Viral M2 protein is absent in type B virus
and


 Hence chemoprophylaxis is effective only for Type A


y Oseltamivir and Zanamivir to treat patients who are sympto- 

infections
y

matic for more than two days before initiation of treatment, or


who had complicated illness requiring hospitalization.
Treatment
Laboratory Diagnosis y y Oseltamivir
y Zanamivir
y Specimen – Nasopharyngeal swab y

y Amantadine
y

y Virus antigen can be demonstrated directly by IMF


y

y Rimantadine
y

240
MULTIPLE CHOICE QUESTIONS

Chapter 30     Orthomyxoviruses


1. Influenza virus belongs to which family: 13. Avian flu is: (Recent Pattern June 2014)
 (Recent Pattern Aug 2013) a. Type A b. Type B
a. Paramyxovirus b. Orthomyxovirus c. Type C d. Type D
c. Bunyaviridae d. Togaviridae 14. All are true about influenza virus except:
2. A 70-year-old lady refused to take influenza vaccine and a. Influenza A has non human hosts
developed influenza. She died due to pneumonia 1 week b. Influenza B has only human hosts
after contracting influenza. Which is the most common c. Influenza C has both human and non human hosts
cause of acute post influenza pneumonia:
d. Usually non human influenza do not cause human
(AIIMS Nov 2014)
infection


a. Cytomegalovirus b. Legionella
15. Elution is mediated by:
c. Staphylococcus aureus d. Measles
3. Influenza virus has: (Recent Pattern Dec 2013) a. Hemagglutination
a. 5 segments of SS RNA b. 8 segments of DS DNA b. Neuraminidase
c. 8 segments of SS DNA d. 8 segments of SS RNA c. Matrix protein
4. True about influenza vaccine: (Recent Pattern Dec 2016) d. Ribonucleoprotein
a. Live vaccine is used most commonly 16. Amantidine acts by:
b. Live vaccine is given by nasal drops a. Blocking HA
c. Killed vaccine is given intramuscular in deltoid b. Blocking viral M2 protein
d. All are correct c. Blocking ribonucleoprotein
5. At present following types of influenza viruses are circu- d. Blocks Neuraminidase
lating in the world, except: (Recent Pattern Dec 2013) 17. Identify the following organism by its replication:
a. H1N1 b. H3N2  (AIIMS Nov 2018)
c. H5N1 d. C virus
6. H1N1 is a type of: (Recent Pattern Aug 2013)
a. SARS virus b. Influenza type A virus
c. Influenza type B virus d. Influenza type C virus
7. Antigenic shift: (Recent Pattern Dec 2012)
a. Occurs every 2-3 years
b. Gradual change over time
c. Results from genetic recombination
d. Occurs in all influenza viruses
8. False about antigen drift: (Recent Pattern Dec 2013)
a. Causes pandemic
b. Occurs due to mutation
c. Occurs more frequently
d. Affected by previous antibodies
9. Which influenza virus shows both antigenic shift as well
as antigenic drift? (Recent Pattern Dec 2016)
a. Type -A b. Type -B
c. Both d. Avian influenza
10. H1N1 is: (Recent Pattern Dec 2015)
a. Swine flu b. SARS
c. Avian d. Chicken pox
11. Which influenza strain, not of human origin and can
cause pandemic? (Recent Pattern Dec 2016)
a. H1N1 b. H2N2
c. H5N1 d. H9N1
12. The avian influenza (H5N1)has the potential to cause
human infections with high mortality. However, it’s not
caused any pandemics. The probable reason for this
could be: (AIIMS Nov 2014)
a. Contains avian genes
b. Contains segmented RNA
c. No human-to-human transmission
d. Very effective infection of wild birds, not domestic a. Hepatitis virus b. Influenza virus
poultry c. HIV d. Herpes virus
241
ANSWERS AND EXPLANATIONS
Unit 3     Virology

1. Ans.  (b) Orthomyxovirus Types of influenza viruses circulating in the world:


• Influenza type A – H1N1

Ref: Ananthanarayan and Paniker’s Textbook of Microbiol- • Influenza type A – H3N2


ogy – 10th ed – Page 502 • Influenza type A – H5N1


• Myxo viruses are divided into orthomyxoviruses and


• • Influenza type B

paramyxovirus
• Influenza comes under orthomyxoviruses

7. Ans.  (c) Results from genetic recombination
Ref: Ananthanarayan and Paniker’s Textbook of Microbiol-
2. Ans.  (c) Staphylococcus aureus
ogy – 10th ed – Page 505
Ref: Jawetz – medical microbiology – 27th edition – Page 572 • Antigenic shift – occurs due to genetic recombination of

• Pneumonia complicating influenza infections can be


• human with animal or avian strains
viral, secondary bacterial, or a combination of the two. • It is an abrupt change that occurs over 10-15 years

• Increased mucous secretion helps carry agents into the



• Causes pandemics

lower respiratory tract. • Occurs only in Influenza A virus


• Bacterial pathogens are most commonly in order of


Staphylococcus aureus, Streptococcus pneumoniae, 8. Ans.  (a) Causes pandemic


and H influenzae
Ref: Ananthanarayan and Paniker’s Textbook of Microbiol-
• A synergistic effect between virus and bacteria may be
ogy – 10th ed – Page 505

that some Staph. aureus strains secrete a protease able


to cleave the influenza HA, thereby allowing production • Antigenic drift – caused only periodic epidemics

of much higher titers of infectious virus in the lungs. • Due to point mutation

• Occurs more frequently 2-3 years


3. Ans.  (d) 8 segments of SS RNA


9. Ans.  (a) Type -A
Ref: Ananthanarayan and Paniker’s Textbook of Microbiol-
ogy – 10th ed – Page 503 Ref: Ananthanarayan and Paniker’s Textbook of Microbiol-
ogy – 10th ed – Page 505
• Virus core has ribonucleoprotein in helical symmetry

• Negative sense ss RNA genome – segmented – eight



• Antigenic variations are highest in influenza type A, less

pieces in type B and not seen in type C


• Viral RNA dependent RNA polymerase is present

10. Ans.  (a) Swine flu


4. Ans.  (b) Live vaccine is given by nasal drops Ref: Ananthanarayan and Paniker’s Textbook of Microbiol-
Ref: Park – T.B of community medicine – 23rd ed – Page 153- ogy – 10th ed – Page 509
155
Most common cause of swine flu H1N1
• Live vaccine is given by intra nasal instillation
Most common cause of avian flu H5N1

• Most commonly used vaccine for influenza is killed


vaccine – given by subcutaneous route


11. Ans.  (c) H5N1
5. Ans.  (d) C Virus Ref: Ananthanarayan and Paniker’s Textbook of Microbiol-
Ref: Harrison 19th edition page 1210 ogy – 10th ed – Page 509
• Avian influenza – caused by type A virus – Various HN

• H5N1 – avian flu – can cause pandemics in human

combinations can cause this


12. Ans.  (c) No human-to-human transmission
• Most common strain causing avian flu – H5N1 –

pandemic – causing high mortality rate Ref: Ananthanarayan and Paniker’s Textbook of Microbiol-
• Other strains are H7N7, H9N2

ogy – 10th ed – Page 508
• DOC for avian flus is oseltamivir

• Avian flu – H5N1 – endemic among birds and poultry in


Europe and Asia


6. Ans.  (b) Influenza type A virus
• All human cases are acquired directly from birds

Ref: Ananthanarayan and Paniker’s Textbook of Microbiol- • No human to human transmission


ogy – 10th ed – Page 509 • But there is a possibility of sporadic transmission to


humans that causes mutation or genetic recombination


with other strains – leads to novel strains

242
13. Ans.  (a) Type A 16. Ans.  (b) blocking viral M2 protein

Chapter 30     Orthomyxoviruses


Ref: Ananthanarayan and Paniker’s Textbook of Microbiol- Ref: Ananthanarayan and Paniker’s Textbook of Microbiol-
ogy – 10th ed – Page 502 ogy – 10th ed – Page 510
• Influenza type A – human and non human hosts
• • Amantadine and rimantadine – blocks the viral M2 pro-

• Influenza type B and C – only human viruses


• tein

14. Ans.  (c) Influenza C has both human and non human 17. Ans.  (b) Influenza virus
hosts
Ref: Jawetz T. B of medical microbiology – 27th edition –
Ref: Ananthanarayan and Paniker’s Textbook of Microbiol- page 565-569
ogy – 10th ed – Page 504 • The image clearly indicates that it is negative sense sin-

• Already explained Q. 13
• gle stranded RNA virus
• The single-stranded, negative-sense RNA genomes of

15. Ans.  (b) Neuraminidase influenza A and B viruses occur as eight separate seg-
Ref: Ananthanarayan and Paniker’s Textbook of Microbiol- ments; influenza C viruses contain seven segments of
ogy – 10th ed – Page 510 RNA, lacking a neuraminidase gene
• To begin with, it is positive strand then gets converted
• Haemagglutination: important characteristics of influ-

into negative strand (note the replication occurs from


enza virus positive to negative – to give negative strands)
• HA is followed after a time by detachment of this virus

• Spikes surrounding the viral particles suggests H and N


from the cell surface – this is called as elution spikes


• Elution – Reversal of hemagglutination

• Hence, the answer is Influenza


• This process is mediated by neuraminidase (also called

sialidase)

243
31
Paramyxoviruses
y Paramyxoviruses are the group of viruses which are most y Parotid swelling is the first sign; Other salivary glands are also
y
y
important causes of respiratory infections in infants and involved
young children. y Non suppurative parotitis followed by Epididymo orchitis,

y
y All the members of this group initiates infection through testicular atrophy can occur
y
respiratory tract. y 10% can go for meningitis, aseptic meningitis and meningo

y
encephalitis
y Other complications: Arthritis, oophoritis, nephritis, pancre-
PROPERTIES OF PARAMYXOVIRUS

y
atitis, thyroiditis and myocarditis
y Virion is spherical and sometimes pleomorphic
y
y Genome is made up of linear, ss RNA which is nonsegmented, Laboratory Diagnosis
y
negative sense
y Envelope contains viral glycoproteins (G, H, HN) and Fusion y RT-PCR method is the very sensitive method to detect mumps
y
genome sequences in clinical samples.
y
(F) proteins.
y Unlike orthomyxoviruses, these are antigenically stable and y Clinical samples that gives appropriate detection of virus are:
y
 Saliva
y
particles are labile; but they are highly infectious

 Cerebrospinal fluid
y Replication: Intracytoplasmic

 Urine
y

y In urine samples, virus can be recovered for up to 2 weeks
VIRUSES THAT COMES UNDER
y
y Primary monkey kidney cell line is the preferred culture lines.
y
PARAMYXOVIRUS Cytopathic effect shows cell rounding and giant cell formation
suggestive of mumps virus.
• Rubulavirus – Mumps virus y Rise in titer for antibodies is must for serological demonstra-

y
• Parainfluenza virus tion of virus.

• Newcastle disease virus
Treatment

• Pneumovirus: Respiratory syncytial virus
y There is no specific therapy

• Morbillivirus: Rubeola - Measles
y
y Vaccine: Jeryl Lynn strain – live attenuated vaccine

y
• Nipah and Hendra viruses y Now combination vaccine is given – MMR vaccine (Effective

y
• Human Metapneumovirus for ten years)

MUMPS VIRUS PARAINFLUENZA VIRUS
y Parainfluenza viruses are ubiquitous and they cause common
y Mumps virus usually causes mild childhood diseases
y
respiratory illnesses in all age groups.
y
y It is a classical example of paramyxovirus that contains
y There are four types of Parainfluenza viruses:
y
HN (Haemagglutination/Neuraminidase) and F (Fusion)
y
 Types 1, 2, 3, 4
proteins

Pathogenesis
Pathogenesis and Clinical Findings y Parainfluenza virus replicates in the respiratory epithelia
y
y Man is the natural host y Viremia is uncommon
y
y
y Infection is acquired by droplets – Respiratory route y Infections are usually confined to respiratory tract only
y
y
y Most infectious period is 4–6 days before the onset of y Exception is Type 3: where infection may spread deeper
y
y
symptoms and after a week from that. to the lower trachea and bronchi causing pneumonia or
y IP is 2-3 weeks bronchiolitis.
y
Clinical Findings y Important cause of otitis media in young children

Chapter 31     Paramyxoviruses


y

y RSV infections causes one third of respiratory infection in


y

Table 1:  Clinical findings in the infection of Parainfluenza bone marrow transplant patients.
virus y It can cause nosocomial infections in nurseries and in
y

pediatric hospitals.
Parainfluenza type 1 Sendai virus and HA-2 virus belongs to
this group; Sendai virus is also called
as hemagglutinating virus of Japan and Laboratory Diagnosis
Influenza virus type D y Isolation of virus by cell culture: HeLa, Hep-2; Virus growth
y

can be seen by development of giant cells and syncytia in


Parainfluenza type 2 Causes acute laryngotracheobronchitis
or croup; also known as Croup inoculated cultures
associated or CA virus y Preferred method for diagnosis is nucleic acid amplification
y

method; Especially in adult specimens – NAAT is very helpful


Parainfluenza type 3 Named as HA-1; causes shipping fever y Detection of viral antigens can be done in nasopharyngeal
y

Parainfluenza type 4 Causes mild respiratory infection in epithelium specimens


children y Detection of virus in an individual is strongly suggestive
y

of causative agent (as this virus is not seen in healthy


y Most common complication of parainfluenza virus is otitis
y

individuals)
media
y Immunocompromised children are more affected with severe
Treatment
y

infections.
y Supportive care like removal of nasal secretions, administra-
y

tion of oxygen helps in treatment


NEWCASTLE DISEASE VIRUS y Antiviral drug – Aerosilised Ribavirin is approved for the
y

y It is also known as Ranikhet virus


y treatment of lower respiratory tract disease in infants at high
y It causes poultry infection – spreads to humans – causes con-
y risk for severe disease.
junctivitis
y Infection in humans is usually an occupational disease.
y

MEASLES
Laboratory Diagnosis y Measles is an acute, highly contagious disease caused by
y

Rubeolavirus
y Preferred method for diagnosis is nucleic acid amplification
y

y Causative agent: RNA virus of Paramyxoviridae: Only one


y

method serotype is seen


y Detection of viral antigens can be done in nasopharyngeal
y

epithelium specimens.
y Cell culture used for isolation of virus – LLC-MK2
y
Pathogenesis
y A rise in titer in ELISA and hemagglutination assays help in
y
y Source of infection: Cases of measles; humans are the only
y

diagnosis. natural hosts for measles viruses.


y Route of infection: Droplets
y

y The virus enters the human body via the respiratory tract,
RESPIRATORY SYNCYTICAL VIRUS: (RSV) y

where it multiplies locally. Then infections spread to regional


y It is also called Chimpanzee coryza agent (CCA)
y
lymphoid tissue.
y RSV is the most common cause for lower respiratory tract
y
y Primary viremia disseminates the virus and replication occurs
y

illness in infants and young children in reticuloendothelial cells; then secondary viremia sets in.
y Does not have HA (Haemagglutination/Neuraminidase)
y
y IP – 10 to 14 days
y

y It has only fusion (F) protein: Responsible for characteristic


y
y Infective period: 4 days before and 5 days after the appearance
y

syncytial cytopathic changes, hence the name respiratory of rash


syncytical virus. y Infectivity is maximum during prodromal stage
y

y Secondary attack rate is 80%


y

y Maternal antibodies help preventing the infection up to


Pathogenesis and Clinical Findings y

6 months, hence most common age group affected are >


y RSV spreads by large droplets and direct contact
y

6 months - < 3 years; That’s why vaccination is given at 9th


y RSV enters through nasal tract and replication occurs initially
y

month; During epidemics even 6 months old children are


in the epithelial cells of nasopharynx also vaccinated.
y Incubation period is 3 to 5 days; it then spreads to lower
y

respiratory tract causing bronchiolitis and pneumonia


y Most common cause of bronchiolitis and pneumonia – in
y
Clinical Features
less than 1 year child (infants) y An initial phase of prodromal symptoms occur. It has fever,
y

y Children who had RSV bronchiolitis and pneumonia can


y
sneezing, coughing, redness of eye;
y Koplik’s spots: Clustered whitish lesions in buccal mucosa
present with recurrent episodes of wheezing illness for many y

245
years seen opposite to 1st and 2nd molar tooth
y After that, rash appears – maculopapular rash
Unit 3     Virology

Strategic Plan for Measles Elimination and Rubella in South


y Complications: Otitis media (Most common), Croup, Bron-
y

East Asia Region (2014–2020) by WHO


chitis, Giant cell pneumonia WHO advocates a 3 phase strategy for measles elimination. The
y Most serious complication – Meningoencephalitis – After
y

3 phases are:
years, rarely it can cause Subacute sclerosing panencepha- • Catch up: This is a nationwide campaign to vaccinate
litis (SSPE)

all children between the ages of 9 months and 14 years


regardless of measles history or vaccination status
Modified measles
• Keep up: This is the routine vaccination schedule in which the
• It is seen in partially immunized children (infants) where

aim is to vaccinate at least 95% of children in each successive


residual maternal antibody is present.


birth cohort
• Incubation period is prolonged
• Follow-up: It is conducted every 2–4 years after the catch

• Prodromal symptoms are decreased


up phase. All children born after the catch up phase are


• Koplik spots not seen


vaccinated.

• Rashes are mild


• Most common complication of measles is otitis media


NIPAH AND HENDRA VIRUS


• Neurological sequelae of measles is subacute sclerosing
y Both are biosafety level 4 agents

panencephalitis (SSPE)
y

y Both are termed as zoonotic paramyxoviruses;


y

y Fruit bats are the natural hosts for both Nipah and Hendra
Complications of Measles
y

viruses.
y y Most common: otitis media y Nipah virus can cause severe encephalitis; direct transmission
y

y y Pneumonia from pigs to humans


y y Croup y Hendra virus is an equine virus which has caused hendra
y

y y Gastroenteritis viral encephalitis in veterinary clinical staffs.


y y Cervical adenitis
y y Acute encephalitis NIPAH VIRUS
y SSPE
y Nipah virus (NiV) is a member of the family Paramyxoviridae,
y

genus Henipavirus.
Laboratory Diagnosis y Transmission of Nipah virus to humans may occur:
y

y Smear from lesions - Multinucleated giant cells – Warthin-


y
 After direct contact with infected bats


Finkeldey cells (Both intranuclear and intracytoplasmic  After contact with infected pigs


inclusion bodies)  Contact with other NiV infected people.




y Measles antigen can be directly detected in respiratory


y
y Person-to-person transmission of Nipah virus in Bangladesh
y

epithelial cells and India is regularly reported.


y The most abundant viral proteins in infected cells is
y
y Another common mode is consumption of raw date palm sap
y

antibodies to the nucleoprotein contaminated with infectious bat excretions.


y Isolation of virus can be done in monkey or human kidney
y

cells or lymphoblastoid cell lines: They have the characteristic Clinical Features
cytopathic effect of multinucleated giant cells
Infection with Nipah virus is associated with encephalitis
y Serological tests need to demonstrate a fourfold titer in
(inflammation of the brain). After exposure and following an
y

antibodies that are increased between the acute phase and


incubation period of 5 to 14 days, illness presents with 3-14 days
convalescent phase of sera
of fever and headache, followed by drowsiness, disorientation
y Suppression of DTH – hence Mantoux will be negative during
and mental confusion. It can progress to coma within 24-48 hours.
y

infection

Treatment and Prophylaxis Diagnosis


Virus isolation attempts and real time polymerase chain reaction
y Immunization: Edmonstan- Zagreb strain; combinations are
(RT-PCR) from throat and nasal swabs, cerebrospinal fluid, urine,
y

used now MMR vaccine


and blood should be performed in the early stages of disease
y Vitamin A treatment in developing countries has decreased
y

the mortality and morbidity of measles.


Treatment
Supportive care; Ribavirin has shown some effects in vitro.
Recent outbreak: Kerala,India (May, 2018)

246
Chapter 31     Paramyxoviruses
Pathogenic transmission of Nipah virus

HUMAN METAPNEUMOVIRUS
y In children – respiratory illness
y

y In adults – causes lymphomas and leukemias


y

247
MULTIPLE CHOICE QUESTIONS
Unit 3     Virology

1. Not true about paramyxoviruses: 10. Warthin Finkeldey cells are seen in:
 (Recent Pattern Dec 2016) a. Mumps b. HSV
a. Belong to family myxoviridae c. Measles d. Influenza
b. Are DNA viruses 11. Croup is caused by:
c. Have linear nucleic acid a. Parainfluenza type 1 b. Parainfluenza type 2
d. Antigenically stable c. Parainfluenza type 3 d. Parainfluenza type 4
2. Myxoviruses include: (PGI Nov 2014) 12. Treatment of RSV:
a. Orthomyxoviruses b. Influenza a. Immunoglobulin b. Foscarnet
c. Measles d. Polio c. Ribavirin d. Amantidine
e. HSV 13. Human metapneumo virus causes:
3. Paramyxoviruses enters the body via: a. Lymphoma in adults b. Leukemia in children
 (Recent Pattern July 2015) c. Gastric flu d. Meningitis
a. Blood 14. Strain used for mumps:
b. Respiratory route a. Jeryl Lynn strain b. Edmonstron Zagreb strain
c. Conjunctiva c. Kolar strain d. 17D
d. Fecal-oral route 15. Which virus is called as Influenza virus type D?
4. Paramyxovirus includes: (Recent Pattern Dec 2012) a. Parainfluenza type 1 b. Parainfluenza type 2
a. Retrovirus b. Poliovirus c. Parainfluenza type 3 d. Parainfluenza type 4
c. Parainfluenza d. Rabies 16. Warthin Finkeldy cells are seen in:
5. Incubation period of measles is:  (Recent Pattern 2018)
 (Recent Pattern July 2016) a. Mumps b. Measles
a. 18-72 hours b. 10-14 days c. Rubella d. Adeno virus
c. 3-4 days d. 20-25 days 17. A man on contact with following gets fever, malaise
6. Most common viral disease affecting parotid glands: and get admitted, in Bangladesh. Which is the possible
 (Recent Pattern Dec 2013) etiological agent?  (AIIMS Nov 2018)
a. Mumps b. Measles
c. Rubella d. Varicella
7. Most common cause of viral pneumonia in infant is:
 (Recent Pattern Dec 2013)
a. Rhinovirus b. RSV
c. Reovirus d. CMV
8. Virus lacking hemagglutinin and neuraminidase but
having membrane fusing protein is:
 (Recent Pattern Dec 2015)
a. RSV b. CMV
c. HSV d. EBV
9. All of the following have only intracytoplasmic inclusion
bodies except:
a. RSV b. Parainfluenza a. Ebola virus b. Nipah virus
c. Measles d. Mumps c. Newcastle disease virus d. Rabies

ANSWERS AND EXPLANATIONS

1. Ans.  (b) Are DNA viruses Myxoviruses:


• Orthomyxovirus – Influenza
Ref: Ananthanarayan and Paniker’s Textbook of Microbiol-

• Paramyxovirus – Measles, mumps, RSV, Parainfluenza,


ogy – 10th ed – Page 512

Newcastle, Nipah, Hendra, Human metapneumovirus


• Paramyxoviruses are RNA virus which do not have

antigenic variations 3. Ans.  (b) Respiratory route


• While orthomyxoviruses have antigenic variations
Ref: Ananthanarayan and Paniker’s Textbook of Microbiol-

2. Ans.  (a) Orthomyxoviruses, (b) Influenza, (c) Measles ogy – 10th ed – Page 513-515
• All paramyxoviruses spreads by droplet infection –
Ref: Ananthanarayan and Paniker’s Textbook of Microbiol-

248 through respiratory route


ogy – 10th ed – Page 502
4. Ans.  (c) Parainfluenza • Parainfluenza type 2 – called as croup associated virus

Chapter 31     Paramyxoviruses


– CA virus
Ref: Ananthanarayan and Paniker’s Textbook of Microbiol-
• Causes acute laryngotreacheobronchitis or croup
ogy – 10th ed – Page 513

Ref: Already explained Q.2 12. Ans.  (c) Ribavirin


Ref: Ananthanarayan and Paniker’s Textbook of Microbiol-
5. Ans.  (b) 10-14 days
ogy – 10th ed – Page 517
Ref: Ananthanarayan and Paniker’s Textbook of Microbiol- • Main management is supportive care for RSV infections

ogy – 10th ed – Page 518 • Continous aerosol administration of ribavirin helps to


• IP is 10-14 days

decrease the illness severity
• Infective period – 4 days before and 5 days after the

appearance of rash 13. Ans.  (a) Lymphoma in adults


Ref: Ananthanarayan and Paniker’s Textbook of Microbiol-
6. Ans.  (a) Mumps
ogy – 10th ed – Page 520
Ref: Ananthanarayan and Paniker’s Textbook of Microbiol- Human metapneumovirus causes:
ogy – 10th ed – Page 513 • In children – respiratory illness

• Parotid swelling is the first classical sign of mumps;



• In adults – causes lymphomas and leukemias

Other salivary glands are also involved


• Non suppurative parotitis followed by Epididymo

14. Ans.  (a) Jeryl Lynn strain
orchitis, testicular atrophy can occur Ref: Ananthanarayan and Paniker’s Textbook of Microbiol-
ogy – 10th ed – Page 515
7. Ans.  (b) RSV
• Jeryl Lynn strain of mumps virus – attenuation done

Ref: Ananthanarayan and Paniker’s Textbook of Microbiol- by passaging in eggs and growing in chick embryo
ogy – 10th ed – Page 514 fibroblast culture – live attenuated vaccine
• Respiratory synctial virus is the most common cause of

• Given after one year of age

pneumonia and bronchiolitis (Lower respiratory tract • Contraindicated in – pregnancy, immunodeficiency,


infections) in children of less than one year – infants allergic to egg protein and neomycin

8. Ans.  (a) RSV 15. Ans.  (a) Parainfluenza type 1

Ref: Ananthanarayan and Paniker’s Textbook of Microbiol- Ref: Ananthanarayan and Paniker’s Textbook of Microbiol-
ogy – 10th ed – Page 514 ogy – 10th ed – Page 515
• It has only fusion (F) protein – responsible for

• Sendai virus (Parainfluenza type 1) – grows well in eggs

characteristic syncytial cytopathic changes and with the infected allantoic fluid – shows high titres
• Does not have HA (Haemagglutination/Neuraminidase)

of hemagglutinin – which resembles that of influenza
virus
9. Ans.  (c) Measles • Hence its is called as influenza virus type D

Ref: Ananthanarayan and Paniker’s Textbook of Microbiol- 16. Ans.  (b) Measles
ogy – 10th ed – Page 513
• All of the paramyxoviruses have intracytoplasmic

Ref: Ananthanarayan and Paniker’s Textbook of Microbiol-
inclusion bodies except Measles which has both ogy – 10th ed – Page 518
intranuclear and intracytoplasmic inclusion bodies. • Multinucleate giant cells – seen in lymphoid tissues of

patients of measles
10. Ans.  (c) Measles • Termed as Warthin-Finkeldey cells

Ref: Ananthanarayan and Paniker’s Textbook of Microbiol- 17. Ans.  (b) Nipah virus
ogy – 10th ed – Page 518
• Multinucleate giant cells – seen in lymphoid tissues of

Ref: CDC - https://www.cdc.gov/vhf/nipah/index.html
patients of measles • Transmission of Nipah virus to humans may occur after

• Termed as Warthin Finkeldey cells



direct contact with infected bats, infected pigs, or from
other NiV infected people.
11. Ans.  (b) Parainfluenza type 2 • Other modes are by drinking raw date palm sap that is

infected with fruit bat excretions which have Nipah virus


Ref: Ananthanarayan and Paniker’s Textbook of Microbiol-
• Conversely, person-to-person transmission of Nipah
ogy – 10th ed – Page 515

virus in Bangladesh and India is regularly reported.

249
32 Arthropod- and Rodent-
Borne Viral Infections
y Arboviruses are arthropod (or) hematophagous insect (vector)
ARTHROPOD-BORNE VIRAL
y
borne viruses
y All are RNA viruses INFECTIONS
y
y Arboviruses are transmitted by blood sucking vectors from
y
one host to another MOSQUITO-BORNE ARBOVIRAL ILLNESS
y Viruses multiply in the tissue of vector without any evidence
Chikungunya Virus
y
of disease or damage
y All arboviruses produces three main clinical entities: y Chikungunya is a mosquito-borne viral disease first described
y
y
 Fevers with or without rashes during an outbreak in southern Tanzania

 Hemorrhagic fevers y It is a RNA virus that belongs to the alphavirus genus of the

y
 Encephalitis family Togaviridae

y The name “chikungunya” meaning “to become contorted”,
IMPORTANT ARBOVIRUSES
y
and describes the stooped appearance of sufferers with joint
pain (arthralgia).
Table 1:  Important Species of arboviruses y Three major genotypes of virus:
y
 West African
Family Genera Important Species

 East/Central/South African (ECSA)
Togaviridae Alphavirus Chikungunya, Western

 Asian
encephalitis virus

Flavivirus Japanese Encephalitis (JE), West Pathogenesis
Flaviviridae Nile fever, Yellow fever, Dengue,
y Vector is Aedes aegypti
KFD, and Zika virus
y
y No animal reservoir
y
Bunyavirus y During interepidemic periods: The virus is maintained
y
Phlebovirus Sandfly fever, Rift valley fever through nonhuman primates (Sylvatic cycle)
Bunyaviridae y Infection is through bite of Aedes mosquitoes. After the bite of
Nairovirus Crimean Congo hemorrhagic
y
an infected mosquito, onset of illness occurs usually between
fever
4 and 8 days but can range from 2 to 12 days.
Hantavirus
Reoviridae Orbivirus Colorado tick fever Clinical Features
Arenaviridae Arenavirus Lassa fever y An abrupt onset of fever frequently accompanied by joint
y
pain is seen in most of the patients
Filoviridae Marburg Ebola
y Other common signs and symptoms include muscle pain,
virus
y
headache, nausea, fatigue and rash
Table 2:  Disease caused by arbovirus illness and the y Most of the patients recover fully after a month, but in some
y
vector cases arthralgia remains for months or even years
Mosquito-borne Chikungunya • Typical biphasic fever
Dengue

• Crippling joint pain with maculopapular rash
Yellow fever

• Conjunctivitis
JE

• Lymphadenopathy
Zika virus

Tick-borne KFD Laboratory Diagnosis
Crimean Congo hemorrhagic fever
y Serological tests confirm the presence of IgM and IgG anti-
Colorado tick fever
y
chikungunya antibodies.
Rodent-borne Hanta virus y IgM antibody levels are highest 3 to 5 weeks after the onset of
y
Lassa fever illness and persist for about 2 months.
Hemorrhagic fever with renal syndrome
y IgM/IgG ELISA in paired sera Laboratory Diagnosis

Chapter 32     Arthropod- and Rodent-Borne Viral Infections


y

y RT PCR
y Increased neutrophil count
y

y Pleocytosis with normal or raised sugar levels, slightly raised


Treatment
y

proteins in CSF.
y No vaccine is available
y

y No specific antiviral drug available


y

Vaccines
y Supportive measures like antipyretics, analgesics and fluids.
y Formalin in activated mouse brain vaccine – Nakayama strain
y

y Live attenuated vaccine from china – JE strain SA 14-14-2


West Nile Fever Virus
y

y Vaccine licensed in India: Recommended for children in


y

y West Nile fever is caused by member of Japanese B encephalitis


y
endemic area
group of flavi viruses  Inactivated vero cell culture derived – SA -14-14-2


y It causes dengue-like syndrome in humans


y
 Inactivated vero cell culture derived kolar strain


y 80% of the West Nile fever infections are asymptomatic; only


y

20% presents with fever and among them <1% presents as


neuroinvasive illness
y In older people, it more commonly causes encephalitis.
y

Genetic deficiency causing nonfunctional variant of chemokine


receptor – CCR5 has increased risk factor for symptomatic West
Nile infections
y In India, virus isolated from encephaltis patients from Ra-
y

jasthan to Karnataka
y West Nile fever spreading through organ transplantation was
y

documented in US in 2002, hence screening of blood donors,


transplantation donors are done in US.
y It is most leading cause of arboviral encephalitis in United
y

States.

Japanese Encephalitis
y Japanese B encephalitis is the leading cause of viral encepha-
y

litis in Asia Figure 1: Transmission cycle of Japanese encephalitis

Pathogenesis
y Vector is Culex tritaeniorhynchus and Culex vishnui (Indian
y
Yellow Fever
vector) y The virus is endemic in tropical areas of Africa and Central
y

y Herons are the reservoir host


y
and South America
y Pigs are the amplifier hosts
y
y Yellow fever is an acute viral hemorrhagic disease transmitted
y

y Humans are dead end


y
by infected mosquitoes. The “yellow” in the name refers to
y In Japan to distinguish from encephalitis A – this has been
y
the jaundice that affects some patients.
named as Japanese B encephalitis y Yellow fever does not exist in India – India is a receptive area
y

y Encephalitis A is a disease seen in Japan area –also called as


y
disease can attack any time in India.
encephalitis lethargica or Von Economo’s disease (This is
different from JE) Pathogenesis and Clinical Findings
y Endemic areas in India – Tamil Nadu and Andhra Pradesh
y

y Vector is Aedes aegypti


y

y Incubation period 3-6 days


Clinical Features
y

y There are two cycles:


y

y Abrupt onset of fever, headache, vomiting occurs – then after


y
 Sylvatic cycle


1–6 days – encephalitis with nuchal rigidity, convulsions,  Urban cycle




altered sensorium and coma occurs y Clinical features are fever with chills, headache, nausea and
y

y Most JEV infections are mild and asymptomatic;


y
vomiting
y Those who have severe infections can go for 30% case fatality.
y
y Complications: Jaundice (hence the term yellow fever), albu-
y

y Of those who survive, 20%–30% suffer permanent intellectual,


y
minuria, hemorrhage – hepatic or renal failure
behavioral or neurological problems such as paralysis, y Differential diagnosis are: severe malaria, leptospirosis, viral
y

recurrent seizures or the inability to speak. hepatitis (especially fulminant forms), other hemorrhagic
fevers, infection with other flaviviruses (such as dengue
hemorrhagic fever), and poisoning. 251
DHF is associated with hemorrhagic manifestations,
Unit 3     Virology


Why yellow fever is not in India yet? 

• Aedes aegypti carries dengue virus plasma leakage resulting from an increased vascular
permeability, and thrombocytopenia (<100, 000 platelets/

• Presence of antibodies in the human body for arbovirus which


mm3)

gives cross protection


• In Africa: Yellow fever attacks mostly West coast  Common clinical warning signs for DSS include a rapidly
rising haematocrit, intense abdominal pain, persistent

• In India – Aedes mosquitoes are seen mostly in the East coast


vomiting, and narrowed or absent blood pressure

Laboratory Diagnosis Complications occur more commonly:


• In secondary infections who have non neutralizing heterologous
y Polymerase chain reaction (PCR) testing in blood and urine

antibodies from previous infection


y

can sometimes detect the virus in early stages of the disease.


• In infants with maternal antibodies.
y In later stages, testing to identify antibodies is needed.

Prophylaxis Mechanism of Secondary Infections Causing


y Non-neurotrophic - 17D vaccine – given s.c – is must for trav-
y
DHF/DSS
ellers to or from endemic areas y The most commonly known theory is immune enhancement
y

y Valid for 10 years, from 10th day of vaccination


y
hypothesis
y In India 17D vaccine is manufactured in – Central Research
y
y This states that when a person experiencing a second infection
y

Institute, Kasauli with a heterologous dengue virus (means different serotype)


serotype have a significantly higher risk for developing DHF
and DSS.
Dengue Virus y Preexisting heterologous dengue antibody recognizes the
y

y Dengue is an acute febrile disease caused by the mosquito-


y
infecting virus and forms an antigen-antibody complex,
borne dengue viruses, consisting of four serotypes (DENV 1 which is then bound to and internalized by immunoglobulin
to 4), that are members of the flaviviridae family Fc receptors on the cell membrane of leukocytes, especially
y Dengue virus is an enveloped, single-stranded positive-sense
y
macrophages.
RNA virus. y This antibody is present, but virus is not neutralized – leads
y

y Four serotypes – DEN 1, DEN 2, DEN 3, and DEN 4


y
to a process known as antibody-dependent enhancement
(ADE), enhances the infection and replication of dengue
Pathogenesis virus in cells of the mononuclear cell lineage.
y Vector is Aedes aegypti
Laboratory Diagnosis
y

y These mosquitoes are day biting; There are two peaks of


y

biting activity, early morning for 2 to 3 hrs after daybreak and y Because of bone marrow suppression – leukopenia occurs
y

in the afternoon for several hours before dark y Decreased platelet count in critical phase
y

y After a person is bitten by an infective mosquito, the virus


y
y Increased hematocrit
y

undergoes an incubation period of 3 to 14 days (average, 4 to


7 days), after which the person may experience acute onset Day 1 up to Day 10 NS1 antigen detection
of fever.
Day 5 up to three months IgM positive
y Also called as ‘Break bone fever’
y

y Clinical presentation is similar to that of Chikungunya fever


y
Using paired sera IgG positive
y Incubation period 3-14 days
y Serological tests used are:
y

 Hemagglutination-inhibition test
Clinical Features


 Complement fixation test




y Two phases of clinical features;


y  Neutralization test


 Febrile phase (high grade fever): Biphasic fever or saddle


  Immunoglobulin M (IgM) capture enzyme-linked im-


back fever, headache, retrobulbar pain, photophobia, munosorbent assay (MAC-ELISA) – Most common used
pain in back and limbs (hence called as break bone fever), method now
lymphadenopathy and maculopapular rash  Indirect immunoglobulin G ELISA


 A macular-papular recovery rash appears 3–5 days after




the onset of fever, and it usually starts on the trunk before Dengue Vaccine
spreading peripherally
• World’s first Dengue vaccine – Dengvaxia- by Sanofi Pasteur –
 This febrile phase lasts for 2–7 days

first licensed in Brazil




 Then it goes to critical phase with rapid decrease in




platelet count – leads to plasma leakage – lasts for 3–8 days


y Complications: Dengue Hemorrhagic Fever (DHF) and
y

Dengue Shock Syndrome (DSS)

252
Table 3:  Duration of illness and types of diagnostic tests

Chapter 32     Arthropod- and Rodent-Borne Viral Infections


Treatment
y Supportive management; antipyretics
y
Time Duration of illness Types of Diagnostic tests
y Fluid replacement mainly crystalloids
y
< 7 days Nucleic acid testing
y In case of severe bleeding manifestations platelet transfu-
≥ 7 days Serological tests
y

sion may be needed.

Treatment
Zika Virus y Supportive measures
y

y Zika virus is a flavivirus that was first isolated in 1947 from a


y
y WHO in collaboration with UNICEF developed a Zika virus
y

febrile rhesus macaque monkey in the Zika Forest of Uganda vaccine target product profile (TPP) for use in an emergency,
y In 1954, the first 3 cases of human infection were reported in
y
or in a future outbreak scenario.
Nigeria
y Zika virus was first reported in May 2015 in continental South
y
TICK-BORNE ARBOVIRUSES
America in Brazil where epidemic was set in
y Zika virus is a positive-sense, single-stranded RNA virus
y
Kyasanur Forest Disease (KFD)
y Transmitted by Aedes mosquito; it can also be transmitted
y
y Kyasanur Forest Disease Virus (KFDV) was first isolated
y

through sexual route during an outbreak of febrile disease in 1957 in people living
y Usually it causes mild asymptomatic illness or subclinical
y
in the Kyasanur forest area of the Shimoga district in the
infections Karnataka
y In pregnancy, it is associated with microcephaly in babies
y
y It is endemic in Karnataka and Kerala and Tamil Nadu
y

y Zika virus is associated with microcephaly and Guillain


y
y Kyasanur Forest disease virus is member of the family
y

Barre syndrome Flaviviridae


y Vector is Ticks – Haemaphysalis spinigera
y

y Reservoir is forest birds and small mammals


y

y It usually infects monkeys


y

y It is also called as Tick-borne hemorrhagic fever or monkey


y

fever
Clinical features: Fever, headache, conjunctivitis, myalgia
y Case fatality rate – 5%
y

y The KFD is placed in the biosafety risk group-4 category


y

as per the guidelines of the Center for Disease Control and


Prevention
y A killed vaccine was given in a small field trial
y

Crimean Congo Hemorrhagic Fever


y Caused by Nairo virus
y

y Vector is ticks: Hyalomma


y

y Most commonly seen in Eastern Europe, central Asia and


y

Africa

Colorado Tick Fever Virus


y An illness due to arbovirus
y

y Vector is ticks - Dermacentor andersoni


y

Figure 2: Mode of spread of Zika virus y Self-limiting fever without rash.


y

Laboratory Diagnosis Wanowrie Virus


y Zika virus has been detected in whole blood, urine,
y
y Not grouped yet
y

cerebrospinal fluid, amniotic fluid, semen and saliva y Isolated from Hyalomma ticks and from brain of a young girl
y

y Zika virus that is present in urine and semen can be seen for
y
who died of fever in Sri Lanka.
longer periods than in whole blood or saliva
y Whole blood, serum collected in a dry tube and/or urine
Bhanja Virus
y

collected from patients presenting with onset of symptoms ≤


7 days should be subjected for nucleic acid tests y Not grouped yet
y

y Whole blood collected in a dry tube and serum collected from


y
y Isolated from Haemaphysalis ticks
y

patients presenting with onset of symptoms ≥ 7 days need to y Human infections are seen in Yugoslavia
253
y

be done with serological tests.


SANDFLY-BORNE ARBOVIRUS Seoul virus
Unit 3     Virology



 Puumala virus
Phlebotomus


 Prospect Hill virus




y Vector is Sandfly
y
y y Transmitted from rodent to rodent and rodent to humans by
y Called as sandfly fever or Pappataci fever or three day fever
y
inhalation of virus from the dried excreta
y Self limiting fever – humans are the only hosts
y
y y Diseases caused are:
 Epidemic nephritis


 Epidemic hemorrhagic fever


RODENT-BORNE VIRUSES


 Hantavirus pulmonary syndrome: caused by newly




y Hanta virus – has four serogroups


y
identified virus – Sin Nombre virus of Hanta virus
y Hosts – Wild rodents
y
y y Hanta virus is responsible for HFRS – hemorrhagic fever with
y Four serogroups are:
y
renal syndrome – endemic or epidemic nephrosonephritis
 Hantaan virus

y y Diagnosis: IgM ELISA

MULTIPLE CHOICE QUESTIONS

1. Which of the following is not common in India: 11. Which of the following are true regarding KFD:
a. Japanese B encephalitis (Recent Pattern Dec 2013) a. It is zoonosis b. Affects monkeys
b. Lassa fever c. Caused by bacteria d. Caused by rickettsia
c. KFD 12. Kolar strain is used for vaccination of:
d. Dengue a. Dengue b. Yellow fever
2. Hemmorrhagic fever is not caused by: c. JE d. KFD
 (Recent Pattern Dec 2012) 13. A 35-years-old female with h/o fever for five days,
a. Yellow fever b. KFD headache, back ache with rash in the back and fore arm
c. Japanese encephalitis d. Dengue fever presents with decreased platelet count; Which test is
3. Hemorrhagic fever is caused by: helpful at this stage for diagnosis?
 (Recent Pattern Dec 2012) a. NS1 antigen detection b. IgM ELISA
a. West-Nile fever b. Sandfly fever c. IgG ELISA d. PCR
c. Ebola virus d. All of the above 14. Zika virus is associated with:
4. Acute hemorrhagic fever with renal involvement is a. Guillian-Barre syndrome b. Macrocephaly
caused by: (Recent Pattern Jul 2016) c. Genital defects d. Dysplasia
a. KFD b. Yellow fever 15. Crimean Congo hemorrhagic fever is transmitted by:
c. Hanta fever d. JE a. Mosquito b. Tick
5. Amplifier host is: (Recent Pattern Dec 2012) c. Sandfly d. Rodent
a. Pig in JE b. Dog in rabies 16. Viral hemorrhagic fever which have occurred in India:
c. Man in JE d. Cattle in JE a. Ebola (Recent Pattern 2018)
6. JE virus life cycle in nature run between: b. Crimean congo hemorrhagic fever
 (Recent Pattern Aug 2013) c. Yellow fever
a. Pigs-Mosquito b. Cattle-Birds d. Marburg
c. Pigs-human d. Bird-Pigs 17. Regarding Zika virus, which of the following is/are
7. Number of dengue virus serotypes: accurate? (PGI pattern 2017)
 (Recent Pattern Dec 2012) a. Zika virus transmitted through bite of Aedes mosquito
a. 1 b. 2 b. Route of transmission seen are breast milk and sexual
c. 3 d. 4 c. Concentration of virus is more in Semen than blood
8. Diagnosis of Dengue fever can be made earliest by: d. Zika virus first identified in Brazil
 (Recent Pattern Jul 2015) e. Virus not spread through saliva
a. Viral culture b. NS -1 antigen detection 18. Which of the following causes hemorrhagic fever: 
c. IgG antibody detection d. Nucleic acid test  (PGI May 2018)
9. Break bone fever is caused by:(Recent Pattern Dec 2015) a. Lassa fever b. Yellow fever
a. Yellow fever b. Japanese encephalitis c. Crimean-Congo fever d. West Nile fever
c. Dengue fever d. KFD e. Kyasanur forest disease
10. Following are true of kyasanur forest disease, except: 19. Which of the following strains of Ebola is non pathogenic
a. Transmitted by soft tick to humans:  (JIPMER Nov 2018)
b. Caused by retrovirus a. Zaire b. Sudan
c. Incubation period is 3–8 days
254 c. Tai forest d. Reston
d. Killed vaccine available
ANSWERS AND EXPLANATIONS

Chapter 32     Arthropod- and Rodent-Borne Viral Infections


1. Ans.  (b) Lassa fever 7. Ans.  (d) 4
Ref: Ananthanarayan and Paniker’s Textbook of Microbiol- Ref: Ananthanarayan and PanikerJuls Textbook of Microbi-
ogy – 10th ed – Page 524 ology – 10th ed – Page 527
• Lassa fever – rodent borne infection
• • DEN1, DEN2, DEN3, DEN4

• Seen in Western Africa


• • There are four different serotypes

• Infection with one serotype does not protect another


2. Ans.  (c) Japanese encephalitis serotype and hence secondary infections are more
Ref: Ananthanarayan and PanikerJuls Textbook of Microbi- dangerous
ology – 10th ed – Page 524
8. Ans.  (b) NS -1 antigen detection
Encephalitis virus Hemorrhagic fever virus Ref: Ananthanarayan and PanikerJuls Textbook of Microbi-
Eastern equine encephalitis Chikungunya ology – 10th ed – Page 529
Western equine encephalitis Dengue types 1-4 • NS1 – Non structural glycoprotein – detected in the
Venezuelan equine Yellow fever

blood as early as three days upto 10 days of the fever


encephalitis Kyasanur Forest Disease
St.Louis encephalitis Omsk hemorrhagic fever 9. Ans.  (c) Dengue fever
West Nile Crimean Congo hemorrhagic
Japanese encephalitis fever Ref: Ananthanarayan and PanikerJuls Textbook of Microbi-
Murray Valley encephalitis Hanta virus pulmonary ology – 10th ed – Page 529
RSSE complex syndrome with HFRS • Dengue fever causes severe pain in the back and limbs

Loupin ill Lassa fever – Break bone fever


Powassan
California 10. Ans.  (b) Caused by retrovirus
Ref: Ananthanarayan and PanikerJuls Textbook of Microbi-
3. Ans.  (c) Ebola virus
ology – 10th ed – Page 530
Ref: Ananthanarayan and PanikerJuls Textbook of Microbi- • KFD – caused by Flavi virus

ology – 10th ed – Page 524, 562 • Vector is Ticks – Haemaphysalis spinigera (Soft tick)

• Sandfly fever – self-limiting three day fever



• Tick borne hemorrhagic fever or monkey fever

• West Nile fever – encephalitis group



• Clinical features – Fever, headache, conjunctivitis,

• Ebola – belongs to Filovirus – causes hemorrhagic fever



myalgia
• A killed vaccine was given in a small field trial

4. Ans.  (c) Hanta fever


11. Ans.  (b) Affects monkeys
Ref: Ananthanarayan and PanikerJuls Textbook of Microbi-
ology – 10th ed – Page 532 Ref: Ananthanarayan and PanikerJuls Textbook of Microbi-
• Hanta virus is responsible for HFRS – hemorrhagic

ology – 10th ed – Page 530
fever with renal syndrome – endemic or epidemic Ref: Already explained Q.10
nephrosonephritis
• It is a rodent borne infection
• 12. Ans.  (c) JE

5. Ans.  (a) Pig in JE Ref: Ananthanarayan and PanikerJuls Textbook of Microbi-


ology – 10th ed – Page 528
Ref: Ananthanarayan and PanikerJuls Textbook of Microbi-
Vaccine licensed in India: recommended for children in
ology – 10th ed – Page 527
endemic area
• Herons act as the reservoir hosts

• Inactivated vero cell culture derived – SA -14-14-2


• Pigs act as the amplifier hosts


• Inactivated vero cell culture derived kolar strain.


• Human infection is the dead end.


13. Ans.  (a) NS1 antigen detection


6. Ans.  (d) Bird-Pigs
Ref: Ananthanarayan and PanikerJuls Textbook of Microbi-
Ref: Ananthanarayan and PanikerJuls Textbook of Microbi- ology – 10th ed – Page 529
ology – 10th ed – Page 532
• Clinical picture suggests that the patient is in febrile

• Natural cycle runs between birds like herons and


phase of Dengue fever going to critical phase as the


amplifier is pigs through culex mosquitoes platelet count is falling down.
• Man is an incidental dead end host

255
• NS1 antigen detection is helpful to diagnose dengue • A majority of those infected with Zika virus disease
Unit 3     Virology

• •

fever from Day 3 up to Day 10. either remain asymptomatic (up to 80%) or show mild
symptoms of fever, rash, conjunctivitis, bodyache,
14. Ans.  (a) Guillian-Barre syndrome joint pains. Zika virus infection should be suspected in
Ref: Ananthanarayan and PanikerJuls Textbook of Microbi- patients reporting with acute onset of fever, maculo-
ology – 10th ed – Page 530 papular rash and arthralgia.
• The viral load in semen is 100, 000 times more than both
• Zika virus causes microcephaly

blood and urine.Zika virus has been found in breast milk


• It is associated with neurological complications and

• There is no vaccine or drug available


Guillian-Barre syndrome.
18. Ans.  (a) Lassa fever; (b) Yellow fever; (c) Crimean-
15. Ans.  (b) Tick Congo fever; (e) Kyasanur forest disease
Ref: Ananthanarayan and PanikerJuls Textbook of Microbi- Ref: Ananthanarayan and PanikerJuls Textbook of Microbi-
ology – 10th ed – Page 531 ology – 10th ed – Page 524
Tick-borne arboviral • Kyasanur forest disease

illness are: • Crimean Congo hemorrhagic fever



19. Ans.  (d) Reston
• Colorado tick fever

Ref: https://www.cdc.gov/vhf/ebola/about.html
16. Ans.  (b) Crimean congo hemorrhagic fever Different species in Ebola:
• Ebola virus (species Zaire ebolavirus)

Ref: Ananthanarayan and PanikerJuls Textbook of Microbi- • Sudan virus (species Sudan ebolavirus)

ology – 10th ed – Page 531 • Taï Forest virus (species  Taï Forest ebolavirus,

• CCHF is one of the deadly hemorrhagic fevers that are


• formerly Côte d’Ivoire ebolavirus)
endemic to Africa, Asia and Europe • Bundibugyo virus (species Bundibugyo ebolavirus)

• Outbreaks have occurred in Gujarat and antibodies have


• • Reston virus (species Reston ebolavirus)

been detected in human and animal sera. • Bombali virus (species Bombali ebolavirus)

Among these, only four - Ebola, Sudan, Taï Forest, and


17. Ans.  (a) Zika virus transmitted through bite of Aedes Bundibugyo viruses causes infection in humans. Reston
mosquito, (c) Concentration of virus is more in Semen is non pathogenic for humans – it causes infections in
than blood primates.
Ref: https://www.cdc.gov/zika/
• Zika virus disease is an emerging viral disease trans-

mitted through the bite of an infected Aedes mosquito


(Aedes agypti).

256
33
Rhabdovirus
RHABDO VIRUS – TWO GENERA Table 2:  Comparison between Glycoprotein and
nucleoprotein
y Vesiculo virus: Vesicular stomatitis virus
y
y Lyssa virus Glycoprotein Nucleoprotein
y
• Mediates binding of virus to • Induces complement
Table 1:  Lyssavirus



acetylcholine receptors in neural fixing antibodies
Genotype Virus Infects tissues • These antibodies are


• Induces heamagglutination not protective
1 Rabies Warm blooded animals


inhibiting antibodies • Group specific antigen
2 Lagos bat/ Natal bat Bat/Cat


• Induces neutralizing protective • Antiserum agains


3 Mokola Shreq/cat/dog/humans antibodies Nucleoprotein – helps
• Stimulates cytotoxic T cell in diagnostic tests
4 Duvenhage Human/bat

immunity
5 European bat Bat/human • Serotype-specific antigen
Lyssavirus type I

• Purified GP – safe subunit vaccine

6 European bat Bat/human
Lyssavirus type II
7 Australian bat Bat/human
Lyssavirus

RABIES VIRUS
Characteristics
y Enveloped helical RNA virus – ssRNA, linear and non-seg- Figure 1: Structure of rabies virus
y
mented, negative sense RNA
y Bullet-shaped virus Table 3:  Comparison between street and fixed viruses
y
y Envelope is made of lipoprotein, which has knob-like spikes
Street virus Fixed virus
y
(Glycoprotein)
y There are five major proteins – among them envelope • Virus isolated from • Virus that undergoes


y
glycoprotein is most important natural human or intracerebral passages in rabbits
y Replication is intracytoplasmic. animal infection – has certain changes – becomes
• Long and variable IP
y
fixed virus

• IP 1 to 12 weeks • More neurotropic
Antigenic Properties


• Negri bodies are seen in • Short and fixed IP


y Only single serotype is seen in rabies virus infected dead animals • IP 6–7 days
y

y But when isolated from different animals or sources, there are • Negri bodies is not demonstrable
y

certain strain differences seen • Used for vaccine production

y G glycoprotein is the major factor seen in rabies virus, which
y
is most responsible for neuroinvasiveness and pathogenicity. Pathogenesis
y Most important region is amino acid position 333. When this y Source of infection: Rabid animals – almost any animal can
y
amino acid is substituted, it leads to loss of virulence.
y
transmit rabies; Most common are Dogs and Bats
y Virus enters through the bite of rabid animal - it remains in
y
the bitten area for 48–72 hours
y Incubation period: 1–3 months
Unit 3     Virology

y Rabies virus multiplies in the muscle or connective tissues


y

and then spreads through nerve endings; it then travels in


axoplasm to reach CNS
y Movement of virus in axons is passive –3 mm/hour
y

y The virus then spreads peripherally to salivary glands and


y

other tissues.
y Highest amount of viral load is seen in submaxillary salivary
y

gland
y It spreads centripetally from the axon to the neuronal bodies
y

y First neurological sign: Cerebral dysfunction, anxiety,


y

confusion and agitation


y It progresses to hallucinations, hydrophobia, aerophobia
y

and photophobia
y Fatal disease: Death occurs due to respiratory arrest during
y

seizures
y Humans are dead end; no human-to-human transmission
y
Figure 2: Negri bodies in brain biopsy
y Unusual way of transmission among humans is corneal
y
(Courtesy: CDC/ Dr Daniel P Perl)
grafts
Immunoprophylaxis
Clinical Features
y Clinical findings occurs in three phases:
y
Table 4:  Types of vaccines
 Short prodromal phase
 Neural vaccines Non-neural vaccines
 Acute neurologic phase


 Coma

• Semple vaccine
• • • Duck egg vaccine
y Prodromal phase lasts for up to 10 days with malaise,
y
• Beta propiolactone
• •
• Live attenuated chick embryo vaccine
headache, anorexia, photophobia and fever vaccine • • Tissue culture vaccine
y It is followed by acute neurologic phase showing nervous-
y
• Suckling mouse brain
• • • Subunit vaccine (in experimental
ness, hallucinations, sympathetic overactivity – lacrimation, vaccine stage)
salivation and increased sweating
y Most of patients have hydrophobia and aerophobia
y
y In India, tissue culture vaccines are used:
y

y This phase is followed by spasm of throat muscles, seizures,


y
 Human diploid cell vaccine (HDC)


coma and death  Purified chick embryo cell vaccine (PCEC)




y Mostly cause of death is cardiorespiratory arrest


y
 Purified Vero cell vaccine


Table 5:  Vaccination schedule


Laboratory Diagnosis
Pre-exposure prophylaxis 0, 7, 21 or 0, 28, 56; Booster after
y Diagnosis:
y

1 year and then after 5 years


 Antemortem: Corneal smears, Skin biopsy and saliva –


demonstration of viral antigens by IMF Post-exposure prophylaxis 0, 3, 7, 14, 30 and optionally 90;
 Postmortem: Demonstration of Negri bodies


y Negri bodies:
y Table 6:  Categories of exposure to virus
 Intracytoplasmic inclusion bodies seen in brain biopsy
Category I Touching or licking on No vaccine


 Absent in 20% of rabies (Hence absence of Negri bodies


intact skin


does not rule out Rabies)


 Seller’s technique

Category II Nibbling of uncovered Vaccine
 Most abundant in hippocampus and cerebellum

skin; minor scratches or
 Brain specimen: fixed with 50% glycerol saline and

abrasions without bleeding
Zenker’s fixative
y Tissue culture – rapid and sensitive method – cell lines used Category III Single or multiple Vaccine +
transdermal bite/ exposure Immunoglobulin
y

are WI38, BHK 21, CER – can isolate in 2-4 days of inoculation
y Antibody demonstration in CSF by ELISA
y
to bats
y RT PCR
y

258
Passive immunization

Chapter 33     Rhabdovirus


y y Human rabies immunoglobulin (HRIG) - recommended in category III exposures
y y Dose 20 IU/kg body weight
y y Should be given before vaccine or simultaneously in opposite side; never after that
y y Vaccine failure was common in the past during neural vaccines; But nowadays its not seen

MULTIPLE CHOICE QUESTIONS

1. Which antigen of rabies virus stimulates antibody 11. Number of doses of HDCV vaccine requires for pre-
production: (Recent Pattern Jun 2014) exposure prophylaxis:
a. Lipoprotein b. Glycoprotein a. 7 b. 5
c. Phosphoprotein d. Intranuclear protein c. 3 d. 1
2. Street rabies virus cause: (Recent Pattern Dec 2013) 12. What is the correct recommended schedule (on days)
a. Natural rabies for postexposure treatment of person who has been
b. Laboratory passage in rabbit vaccinated for rabies previously with HDC:
c. Fatal encephalitis in 6 days a. 0, 3 and 7
d. Negri bodies not seen b. 0, 3, 7 and 14
3. Speed of rabies virus in axon is: c. 0, 3, 7, 14 and 28
 (Recent Pattern Aug 2013) d. 0 and 3
a. 1 mm per hour b. 3 mm per hour 13. Which of the following antigen is suited for subunit
c. 5 mm per hour d. 7 mm per hour vaccine production in rabies:
4. Negri bodies are found in: (Recent Pattern Dec 2013) a. Nucleoprotein
a. Rubella b. Rabies b. Glycoprotein
c. HSV d. IMN c. Glycolipid
5. Negri bodies are found in: (Recent Pattern Dec 2012) d. RNA dependent RNA polymerase
a. Hypothalamus b. Hippocampus 14. All of the following about rabies are correct except:
c. Midbrain d. Medulla a. Can be transmitted through corneal grafts
6. A patient presented to the hospital with severe hydro- b. Transmitted from human to human by inhalation
phobia. You suspect rabies obtain corneal scrapings c. Transmitted by bite of rabid dog
from the patients. What test should be done on this spec- d. Transmitted by entering into bat caves
imen for a diagnosis of rabies: 15. Identify the following intracytoplasmic inclusion iso-
a. RT-PCR for rabies virus (Recent Pattern Nov 2016) lated from brain biopsy:
b. Negri bodies
c. Antibodies to rabies virus
d. Indirect immunofluorescense
7. Site of injection of cell culture rabies vaccine:
 (Recent Pattern Dec 2013)
a. Gluteus b. Subcutaneous
c. Deltoid d. Anterior abdominal wall
8. Neurological complication following rabies vaccine is
common with:
a. HDCS vaccine b. Chick embryo
c. Semple vaccine d. Duck egg vaccine
9. Which anti-rabies vaccine is given in India:
a. Duck cell vaccine
b. Chick fibroblast vaccine
c. HDCV
d. Sheep brain vaccine
10. Schedule of intradermal rabies vaccine is: a. Paschen bodies b. Negri bodies
 (Recent Pattern Aug 2013) c. Molluscum bodies d. Guarnieri bodies
a. 2-2-0-1-0-1 b. 8-0-4-0-1-1
c. 8-4-4-1-0-1 d. 2-0-2-0-1-1

259
ANSWERS AND EXPLANATIONS
Unit 3     Virology

1. Ans.  (b) Glycoprotein • Region: Adults – Deltoid; Children – anterolateral aspect


of thigh
Ref: Ananthanarayan and Paniker’s Textbook of Microbiol- • Gluteal regions should be avoided as it will lead to less

ogy – 10th ed – Page 535 immunogenicity due to decreased absorption


• Two important antigens are responsible for virulence

• Glycoprotein and Nucleoprotein



8. Ans.  (c) Semple vaccine
• Glycoprotein induces protective antibodies while

Ref: Ananthanarayan and Paniker’s Textbook of Microbiol-


nucleoprotein does not ogy – 10th ed – Page 539
2. Ans.  (a) Natural rabies • Two types of rabies vaccines: Neural and non-neural

vaccines
Ref: Ananthanarayan and Paniker’s Textbook of Microbiol- • Semple vaccine is a type of neural vaccine which has

ogy – 10th ed – Page 535 neurological complications


• Two strains of rabies viruses are Street and fixed viruses
9. Ans.  (c) HDCV

• Street virus causes natural rabies


• Fixed virus helps in vaccine production.



Ref: Ananthanarayan and Paniker’s Textbook of Microbiol-
ogy – 10th ed – Page 539
3. Ans.  (b) 3 mm per hour
• In India, tissue culture vaccines are used:

Ref: Ananthanarayan and Paniker’s Textbook of Microbiol- ƒ Human diploid cell vaccine (HDC)
ƒ

ogy – 10th ed – Page 537 ƒ Purified chick embryo cell vaccine (PCEC)
ƒ

• Rabies virus from the site of entry gains path to neurons



ƒ Purified Vero cell vaccine
ƒ

– travels at a speed of 3 mm/hour in axoplasm


10. Ans.  (b) 8-0-4-0-1-1
4. Ans.  (b) Rabies Ref: Park 20th edition – page 243
Ref: Ananthanarayan and Paniker’s Textbook of Microbiol- • Intradermal schedule has two different types:

ogy – 10th ed – Page 537 ƒ 2 site schedule – one dose given at each of the two
ƒ

• Negri bodies are characteristic inclusion bodies seen in sites on days 0, 3 and 7 and at one site on days 28 and
90

rabies
ƒ 8 site schedule :
• Intracytoplasmic inclusion bodies seen in brain biopsy
ƒ

Š Day 0 – vaccine given at 8 sites


• Absent in 20% of rabies (Hence absence of Negri bodies


Š

Š Day 7 – vaccine given at 4 sites


does not rule out Rabies)


Š

Š Day 28 – given at 1 site


• Most abundant in hippocampus and cerebellum
Š

Š Day 90 – given at 1 site


ƒ This is simply written as:


5. Ans.  (b) Hippocampus
ƒ

Š Days 0, 3, 7, 14, 28, 90


Š

Ref: Ananthanarayan and Paniker’s Textbook of Microbiol- Š Vaccine 8-0-4-0-1-1


Š

ogy – 10th ed – Page 537


11. Ans.  (c) 3
Ref: Already explained Q.4
Ref: Ananthanarayan and Paniker’s Textbook of Microbiol-
6. Ans.  (a) RT-PCR for rabies virus ogy – 10th ed – Page 540
• Pre exposure prophylaxis – three doses of vaccine given
Ref: Ananthanarayan and Paniker’s Textbook of Microbiol-

on days 0, 7, 21 or days 0, 28, 56


ogy – 10th ed – Page 538; Harrison 18th edition Page 195
• Reverse Transcriptase (RT-PCR) – Sensitive method to
• 12. Ans.  (a) 0, 3 and 7
detect rabies in CSF, saliva, corneal scrapings and urine.
Ref: Park 20th edition page 244
• Another method which is not quite sensitive is used

where PCR is not available – Direct fluorescence • Persons who have already vaccinated – should receive

antibody technique three doses of i.m HDCV vaccine on days 0, 3, 7

7. Ans.  (c) Deltoid 13. Ans.  (b) Glycoprotein

Ref: Ananthanarayan and Paniker’s Textbook of Microbiol- Ref: Ananthanarayan and Paniker’s Textbook of Microbiol-
ogy – 10th ed – Page 540 ogy – 10th ed – Page 537
• Vaccine should be given either intramuscular or subcu- Glycoprotein:
• Induces heamagglutination inhibiting antibodies

taneous
260

• Induces neutralizing protective antibodies



• Serotype specific antigen • Exposure to bats in caves need rabies vaccination

Chapter 33     Rhabdovirus


• •

• Purified GP – safe subunit vaccine


• • Humans are dead end; no human to human transmission

• Unusual way of transmission among humans is corneal


14. Ans.  (b) Transmitted from human to human by grafts


inhalation
15. Ans.  (b) Negri bodies
Ref: Ananthanarayan and Paniker’s Textbook of Microbiol-
ogy – 10th ed – Page 541 Ref: Ananthanarayan and Paniker’s Textbook of Microbiol-
• Source of infection is bite of any animals most common

ogy – 10th ed – Page 538
dogs and bats Ref: Already explained Answer 4

261
34
Hepatitis Virus
y Hepatitis virus includes group of viruses belonging to differ- y Histological changes seen are:
y
y
ent family that causes infections in liver  Necrosis of hepatocytes in the periportal areas


y All of the hepatitis viruses are RNA viruses except Hepatitis B  Proliferation of Kupffer and other endothelial cells
y

virus – DNA virus y There are no features of chronicity or malignancy in HAV.

y
Table 1:  Characteristics of hepatitis virus Clinical Features
y The incubation period is 2–6 weeks.
HAV HBV HCV HDV HEV

y
y Most of the infections are asymptomatic or mild in children;
Picornaviridae Hepadnavirus Flavivirus Deltavirus Calcivirus

y
y Clinical illness usually starts with a few days of malaise, loss of

y
RNA DNA RNA Defective RNA appetite, vague abdominal discomfort, and fever.
RNA y Soon afterwards, jaundice becomes apparent showing darker
y
urine and pale feces. Severe jaundice may cause itching.
Fecal-oral Percutaneous, Percuta- Percutane- Fecal-oral
y Hepatitis A is nearly always self-limiting, but relapses have
route vertical and neous ous
y
been reported.
sexual route
y The severity of illness is less in children than in adults.
y
Children Any age Adults Any age Young y Complications such as fulminant hepatitis, fortunately rare,
y
adults are seen mainly in older people.
No carrier Carrier Carrier No No
Table 2:  Difference in clinical presentations in children
Nil Oncogenic Oncogenic Nil Nil
and adults
Vaccine Vaccine Nil HBV Nil
available available vaccine HAV in children HAV in adults
95% subclinical infections Only 10% subclinical infections
HEPATITIS A VIRUS 5-20% icteric disease 75-90% icteric disease
y HAV was previously termed as Enterovirus 72 >98% complete recovery >98% complete recovery
y
y It is the most common cause of acute hepatitis in children
Lab Diagnosis
y
y Early stage of jaundice: Virus can be identified in the feces by
Properties of HAV
y
IEM.
y It is the only virus among hepatitis group that can be y During the acute phase: Liver function tests show raised
y
cultivated in vitro
y
serum bilirubin and transaminases and a depressed
y Only one serotype is seen; there is no antigenic cross reactivity prothrombin level.
y
seen with other hepatitis viruses. y Specific diagnosis is made by an ELISA test for specific IgM.
y HAV is stable to treatment with 20% ether, acid pH and heat;
y
y
its infectivity is maintained even for a month after being dried. Prophylaxis
y Once infected, antibody to HAV is produced and it is
y HAV vaccine – conjugate vaccine available
y
protective;
y
y Passive immunization by immunoglobulin also given
y Most of the population has exposure to HAV
y
y
HEPATITIS B VIRUS
Pathogenesis
y In 1965 – Blumberg et al., discovered Australia antigen – now
y HAV is acquired through fecal-oral route.
y
termed as Hepatitis B surface antigen
y
y The virus replicates primarily in the hepatocytes, from which y About one third of world’s population: has HBV infection;
y
it passes through the bile duct to the intestine, and is shed in
y
HBV infection is a global health problem.
large quantities in the feces. y India accounts for 10 to 15% of the HBV carriers all over the
y
world. The prevalence of HBV carriers in India is around 40
million as estimated by various studies.
Properties of HBV

Chapter 34     Hepatitis Virus


Antigens Characteristics
y Two linear strands of DNA – one strand is complete and
y

• Can only be demonstrated in hepatocytes


another one is incomplete (plus strand) – viral DNA by IMF


polymerase (codes for P gene) • Antibodies to HBcAg can be detected by

y HBV has DNA dependent DNA polymerase and RNA


y

ELISA
dependent reverse transcriptase
HBeAg (C gene • Soluble nucleocapsid antigen
y HBV has an enveloped, 42 nm spherical particle

– Pre C region) • Gene for HBcAg and HBeAg are same


y

y Three types of particles


• Antigen is the detectable marker of high


y

 Most abundant – Spherical particle - Hepatitis B surface


infectivity


antigen – Australia antigen (22 nm)


 Filamentous or tubular (22 nm)

HBxAg (X gene) • Transactivating effects on viral and some

 Complete HBV – Dane particle (42 nm) – contains three



cellular genes
antigens: • Causes enhancement of viral replication

Surface antigen HBsAg


Core antigen HBcAg
Nucleocapsid antigen HBeAg

Figure 2: Antigenic subtypes of HBsAg

HBV Viral Replication

Figure 1: Hepatitis B genome structure

Table 3: Antigenic structure


Antigens Characteristics
HBsAg (S gene) • HBsAg has antigenic diversity:

• Group specific antigen – termed as ’a’


• Two pairs of type specific antigens ’d-y’ and


‘w-r’
• Based on the different combinations – it
Figure 3: HBV viral replication

has four types: adw, adr, ayw, ayr


• Subtypes do not have a difference in

Reason for Viral Persistence and


immunity
• Subtypes have difference only in

Hepatocellular Carcinoma
geographical distribution y Hepatitis B virus is the only virus that produce genome DNA
y

• Major antigen

by reverse transcription with mRNA as template
HBcAg (C gene – • To expose this antigen – mild detergent

y Following entry into the host cells, the virion RC DNA is
y

C region) treatment is used to disrupt the viral released into the nucleus for conversion into CCC DNA
envelope - covalently closed circular DNA, which is the first viral
• Not secreted – does not circulate in the

product to be made in a newly infected cell. CCC DNA then
blood serves as the viral transcriptional template for the synthesis of 263
all viral RNAs by the host RNA polymerase II.
Contd...
y The prevalence of HCC closely parallels the prevalence of y Super carriers – high titer of HBsAg along with HBeAg + DNA
Unit 3     Virology

y y

persistent HBV infection with the highest incidences in Asia polymerase + HBV in circulation
and in central and southern Africa. y Simple carriers – low infectivity and low HBsAg
y

y In most of the tumors, HBV DNA is integrated into the cell


y

genome, and in most cases, the virus DNA has undergone • Prevalence of HBV in India – 2 to 7% (Intermediate group)

rearrangements, including deletions. • 350 million carriers are seen all over world

y The P and C gene ORFs have generally been destroyed, but


y
• 45 million carriers are in India

the S and X ORFs are often intact.


y The only viral gene product that is consistently present in the
y
Laboratory Diagnosis
tumor cells is the X protein. y Serological markers helps to identify the stage of hepatitis
y

infection
Mutations y Indicator of active viral replication and infectivity – HBeAg,
y

HBV DNA and HBV viral load


y Precore mutants:
y HBV DNA identified by molecular methods – reflects the
y

 HBeAg mutation gives rise to precore mutants


y

degree of viral replication in liver; to assess the progress of




 These patients show serological tests negative for HBeAg


patients with chronic hepatitis under anti viral chemotherapy.


but shows positive in anti-HBe and anti-HBc


y Escape mutants:
y

 HBsAg mutation is called as escape mutants.




 It is usually seen in infants born to HBeAg positive mother,




liver transplant patients who have received both active


and passive immunization
 Mutation occurs in the group specific ~a~ determinant


 Because of this mutation – HBV vaccine does not protect;




HBV vaccine is targeted against this group specific a


determinant
 Hence, these mutants escape protection from vaccination


termed as escape mutants.

Mode of Infection
y Parenteral transmission – Even 0.0001 mL of infected blood
y

causes infection (more dangerous than HIV)


y Perinatal transmission
y
Figure 4: Serological markers of HBV virus infection
y Sexual transmission
Table 4: Interpretation of serological markers for
y

diagnosis of HBV
Clinical Features
y IP is 1-6 months
y
HBsAg HBeAg Anti Anti Anti Interpretation
y Acute infection may be subclinical, especially in young
y
HBc HBs HBe
children + + IgM – – Acute HBV infection;
y Hepatitis, arthralgia, urticaria, polyarteritis or glomerulone-
y

highly infectious
phritis are the features seen in adults
+ + IgG – – Late/chronic HBV
y 90-95% recover from the mild illness
infection or carrier state;
y

y 1-10% goes for chronicity leading to chronic hepatitis and


highly infectious
y

development of hepatocellular carcinoma.


y HCC may result from integration of viral genome into the
y
+ – IgG – +/– Late/Chronic HBV
DNA of the hepatocytes, but the pathogenesis is not so clear. infection or carrier state;
y But HCC arise after a chronic infection with continuing
y
low infectivity
production of complete virions that has been in progress for – +/– IgM – +/– Seen rarely in early acute
at least 2 years. HBV infection; infectious
– – IgG +/– +/– Remote HBV infection;
Carriers infectivity nil or very low
y A carrier is a person who has detectable HbsAg in blood for
y
– – – + – Immunity following HBV
>6 months vaccine
y Rate of carriers are after an infection:
y

 5-10% of adults


Prophylaxis of HBV
 30% of children
264 y Currently used vaccine is prepared by cloning the S gene in


 90% of neonates
y

bakers yeast

y Given IM – deltoid or anterolateral aspect of thigh in children y Complications:

Chapter 34     Hepatitis Virus


y y

– 0, 1, 6 months  Glomerulonephritis


y It should not be given in gluteal as absorption in muscle is low


y  Vasculitis


leading to poor immune response  Type 2 cryoglobulinemia




y Seroconversion rate is around 90%


y y Hepatitis C becomes chronic in about 80 per cent of those
y

y Immunoglobulin given for babies born to carrier mothers and


y infected
for those who had exposure with known postive HBV persons y After many years, cirrhosis of the liver occurs in 10–20 per
y

cent of patients
Treatment y HCC, which develops in 1–5 per cent of those with chronic
y

infection has a very poor prognosis.


y IFN-alpha2b is given subcutaneously at a dose of 5 million
y

units daily or 10 million units thrice weekly for 16 to 24 weeks.


y Oral lamivudine is given as 100 mg/day for a period of 12 to
y Laboratory Diagnosis
18 months and it is effective in reducing the HBV DNA levels. y ELISA and immunoblot tests are available for detecting
y

antibodies to core, envelope and NS3 and NS4 non-structural


HEPATITIS C VIRUS proteins.
y ELISA has higher sensitivity when compared to the immuno-
y

y HCV is a member of family Flaviviridae, which has a positive


y

blot rapid card tests.


sense ssRNA. y The limitations of immunoassays are inability to distinguish
y

y WHO estimates that 3% of the world’s population is infected


y

among acute, chronic or resolved infections.


with HCV. y Nucleic acid based assays are helpful in detecting the
y

y Occult HBV infections (33%) are seen in patients with chronic


y

presence of circulating HCV RNA and for genotyping.


HCV liver disease, where HBsAg cannot be detectable in y The viral load assays are helpful in the monitoring of antiviral
y

serum because of its lower levels. therapy.


y The co-infections of Hepatitis infections with HIV are more
y

common.
Treatment
Properties of HCV y Drugs used are: Interferon alpha, Sofosbuvir, Ribavirin
y

y Interferon-Alpha2a or 2b given at the dose of 3 million units


y Most common cause of post-transfusion hepatitis in
y

thrice weekly for 12 to 24 months is effective.


developed countries y Ribavirin can also be given orally at a concentration of 1000-
y

1200 mg daily for 6 to 12 months.


HCV – causes chronic form of hepatitis; causes hepatocellular
carcinoma
• Carriers – 200 million worldwide

HEPATITIS D VIRUS
• Prevalence in India – 12.5 million cases

y Hepatitis D virus is an enveloped virus with RNA genome


y

y Six different genotypes are present


y
y Classified under Delta virus
y

y The distributions of the major genotypes are related to various


y
y It does not have polymerase in its virion; cannot replicate
y

risk groups, response to antiviral therapy and geographical on its own; called as Defective virus (Defective RNA virus –
areas of prevalence. dependent virus)
y There is no cross-protection between the various genotypes
y
y HDV can replicate only when co-infected with HBV
y

y This genotypic variation and no cross protection makes


y
y HDV transmitted as like HBV – blood and blood borne
y

difficulty in making vaccine development. products, vertical transmission


y It can either gets infected along with HBV (Co-infection) or
y

after infection with HBV (Super infection)


Mode of Spread of HCV y Lab diagnosis – detection of delta antigen of IgM antibody to
y

y Parenteral transmission
y
delta antigen in blood
y Screening of blood and blood products for HBV and HCV is
y
y Experimental model for HDV - Woodchuck
y

important y No specific therapy; no vaccination


y

y Immunization against Hepatitis B virus will reduce the


y

Clinical Features hepatitis caused by HDV

y The incubation period is about 8 weeks.


HEPATITIS E VIRUS
y

y Only very few individuals gets symptoms that too very mild
y

like anorexia and nausea


y Jaundice is uncommon in HCV
y
Properties
y When jaundice occurs, symptoms and biochemical changes
y
y HEV is a calcivirus – has a single stranded positive sense RNA
y

are identical to other forms of hepatitis y HEV – non A non B hepatitis (NANB) – enterically transmitted
y

y Alanine aminotransferase levels begin to increase shortly – causes fulminant hepatic failure in pregnant females (last
265
y

before symptoms and a 10-fold elevation can be detected. trimester)


y Also called as epidemic NANB; It is also a common cause of HEPATITIS G VIRUS
Unit 3     Virology

acute sporadic hepatitis in Asian countries.


y Animal reservoir – Pigs
y
y Hepatitis G virus: Found in patients with acute, chronic
y

y Sporadic infections are zoonotic in nature: hence it is called


y
and fulminant hepatitis, hemophiliacs, patients who
as Zoonotic hepatitis infections. had undergone multiple transfusions and hemodialysis,
intravenous drug addicts and blood donors

Clinical Features Miscellaneous viruses that causes hepatitis are:


y Infection occurs through fecal-oral route • Epstein Barr virus

• Cytomegalovirus
y

y Incubation period around 6 weeks •

• Herpes simplex virus


y

y Infections are seen in adults than in children; person to •

• Mumps virus
y

person transmission rate is lower than HAV. •

y Women who are infected in the last trimester of pregnancy – • ECHO virus

• Rubella
y

develops for fulminant hepatitis leading to mortality •

• Yellow fever

Laboratory Diagnosis
y Virus like particles can be seen from feces by Immunoelectron
y

microscope
y ELISA for IgG and IgM antibodies
y

266
MULTIPLE CHOICE QUESTIONS

Chapter 34     Hepatitis Virus


1. Which hepatitis virus causes acute illness: 14. Super carrier of HBV shows following serum markers:
 (Recent Pattern Dec 2015) a. HBsAg (Recent Pattern Dec 2015)
a. HBV b. HCV b. HbsAg + HBV DNA
c. HAV d. All of the above c. HbsAg + HBeAg + HBV DNA
2. Virus causing hepatocellular carcinoma belongs to:
d. Anti-HBsAg + HBV DNA
 (Recent Pattern Dec 2015) 15. First antibody to appear in hepatitis:
a. Hepadnaviridae b. Enterovirus  (Recent Pattern Dec 2014)
c. Calcivirus d. None a. IgM anti - HBe b. IgG anti - HBe
3. Which is SS RNA unenveloped virus: c. IgM anti - HBc d. IgM anti - HBs
 (Recent Pattern Dec 2012) 16. The serological marker of acute Hepatitis B infection is:
a. HBV b. HEV  (Recent Pattern Dec 2013)
c. HCV d. None a. HBsAg + HBeAg b. HBsAg + Core antibody
4. Hepatitis A virus is best diagnosed by: c. HBsAg d. HBcAg
 (Recent Pattern Nov 2015) 17. Infectivity of HBsAg is best/commonly diagnosed by:
a. IgM antibodies in serum b. Isolation from stool  (Recent Pattern Dec 2012)
c. Culture from blood d. Isolation from bile a. HBeAg b. HbsAg
5. True about hepatitis A virus: (Recent Pattern Dec 2013) c. HBV DNA d. Anti HBsAg
a. Causes carcinoma 18. Active replication in Hepatitis B infection is indicated
b. Helps HDV replication by: (Recent Pattern Dec 2013)
c. Common cause of hepatitis in children a. HBeAg b. HBsAg
d. Causes chronic hepatitis c. HBcAg d. Anti - HBsAg
6. Hepatitis A is transmitted by: (Recent Pattern Dec 2014) 19. Presence of HBeAg in patients with hepatitis indicates:
a. Blood route b. Inhalation  (Recent Pattern Dec 2014)
a. Simple carriers b. Late convalescence
c. Fecal-oral route d. All
c. High infectivity d. Carrier status
7. Which is not true about hepatitis B virus: 
20. Which of the following does not indicate Hepatitis B
a. DNA virus  (Recent Pattern Dec 2013)
replication: (Recent Pattern July 2016)
b. Transmitted by fecal-oral route
a. HBsAg b. HBeAg
c. Can be transmitted from mother to child
c. HBV DNA d. Viral copies
d. Contains reverse transcriptase
21. Which of the following antigen is found within the nuclei
8. DNA polymerase of HBV is encoded by which of the
of infected hepatocytes and not usually in the periphery
following: (Recent Pattern Dec 2015)
circulation in Hepatitis B infection:
a. S gene b. C gene
(Recent Pattern Dec 2013)
c. P gene d. X gene


a. HBeAg b. HBcAg
9. Reverse transcriptase is a RNA dependent DNA poly- c. Anti – HBc d. HBsAg
merase. Which of these use it: 22. Best means of giving hepatitis B vaccine is:
(Recent Pattern May 2015)

 (Recent Pattern Nov 2014)


a. Hepatitis A virus b. Hepatitis B virus a. Subcutaneous b. Intradermal
c. Hepatitis C virus d. Hepatitis D virus c. Intramuscular deltoid d. Intramuscular gluteal
10. E antigen (HBeAg) of hepatitis B virus is a product of 23. True about HCV is: (Recent Pattern July 2016)
which gene: (Recent Pattern Dec 2015) a. Most common mode of transmission is by needle
a. S b. C puncture
c. P d. X b. Diagnosis is established by viral isolation from blood
11. Which is the longest DNA of hepatitis B virus: c. Chances of perinatal transmission are directly corre-
 (AIIMS Dec 2013) lated with degree of maternal viremia
a. P gene b. X gene  d. Associated with polyarteritis nodosa (PAN)
c. S gene d. C gene 24. Most common cause of chronic hepatitis:
12. HBV is associated with all of the following except:  (Recent Pattern July 2016)
 (Recent Pattern Nov 2014) a. HEV b. HAV 
a. Hepatic cancer b. Chronic hepatitis c. HBV d. HCV
c. Hepatic adenoma d. Cirrhosis 25. Hepatitis C virus is: (Recent Pattern Dec 2012)
13. Serological testing of patient shows HBsAg, IgM, HBeAg a. Togavirus b. Flavivirus
positive. The patient has: (Recent Pattern Dec 2012) c. Filovirus d. Retrovirus
a. Chronic hepatitis B with low infectivity 26. Commonest hepatotropic virus causing increased
b. Acute hepatitis B with high infectivity chronic carrier state is: (Recent Pattern Dec 2013)
c. Chronic hepatitis with high infectivity a. HEV b. HAV
267
d. Acute on chronic hepatitis c. HBV d. HCV
27. HDV is: (Recent Pattern Dec 2014) 30. Hepatitis E usually affects: (Recent Pattern Nov 2015)
Unit 3     Virology

a. SS-RNA virus b. SS-DNA virus a. Children


c. DS-RNA virus d. DS-DNA virus b. Adults
28. HBV and HDV false is: (Recent Pattern Dec 2012) c. Old age
a. Both can infect simultaneously d. Toddlers
b. HDV causes more serious infection d/t super infection 31. Hepatitis E clinically resembles:
c. HDV cannot infect in absence of HBV  (Recent Pattern Nov 2014)
d. DNA viruses a. Hepatitis A b. Hepatitis B
29. A pregnant women in Bihar presents with fulminant c. Hepatitis C d. Hepatitis D
hepatic failure. The most likely etiological agent is: 32. Cryoglobulinemia is associated with:
a. Hep. E (AI Dec 2013)  (Recent Pattern 2018)
b. Hep. B a. HAV b. HBV
c. Sepsis c. HCV d. HDV
d. Acute fatty liver of pregnancy

33. Identify the virus:  (AIIMS Nov 2017)

a. HIV b. Hepatitis c. Influenza d. Herpes simplex

34. There are 3 to 5% healthy hepatitis B carriers in India 35. Regarding Hepatitis D virus (HDV), which of the follow-
who are asymptomatic. They have the risk of developing ing is/are correct?  (PGI pattern)
HCC in future because: (AIIMS Nov 2017) a. Alpha interferon can eradicate the latent stage caused
a. They are unable to mount inflammation against the by HDV
virus. b. Immunization against Hepatitis B virus will reduce the
b. Virus can integrate with host DNA and form hepatitis caused by HDV
complementary DNA c. HDV has RNA in its genome and it has no polymerase
in its virion
c. There is a risk of elevation of transaminases
d. Laboratory diagnosis of HDV is done by growing the
d. Liver parenchymal cells are in a state of high prolifera- cells in co-infection with HBV
tion e. It is a non-enveloped virus
36. Which is the dangerous hepatitis virus during
pregnancy?  (CET 2018)
a. Hepatitis A b. Hepatitis B
268 c. Hepatitis D d. Hepatitis E
ANSWERS AND EXPLANATIONS

Chapter 34     Hepatitis Virus


1. Ans.  (c) HAV 8. Ans.  (c) P gene
Ref: Ananthanarayan and Paniker’s Textbook of Microbiol­ Ref: Ananthanarayan and Paniker’s Textbook of Microbiol­
ogy – 10th ed – Page 546; Harrison T.B of internal medicine ogy – 10th ed – page 548
– 18th ed – Page 2559
Gene Coding
• Acute onset is caused by HAV and HEV
S gene HBsAg

• Insidious onset is by HBV, HCV, HDV


C gene – C region HBcAg


2. Ans.  (a) Hepadnaviridae
C gene – Pre C region HBeAg
Ref: Ananthanarayan and Paniker’s Textbook of Microbiol­ P gene DNA polymerase
ogy – 10th ed – page 546
X gene HBxAg
• Hepatitis viruses that cause hepatocellular carcinoma

are HBV and HCV 9. Ans.  (b) Hepatitis B virus


• HBV – Hepadnavirus

• HCV – Flavivirus

Ref: Ananthanarayan and Paniker’s Textbook of Microbiol­
ogy – 10th ed – page 546
3. Ans.  (b) HEV • HBV has DNA dependent DNA polymerase and RNA

Ref: Ananthanarayan and Paniker’s Textbook of Microbiol­ dependent reverse transcriptase


ogy – 10th ed – page 555
10. Ans.  (b) C
• Hepatitis A virus and Hepatitis E virus are the non

enveloped RNA viruses Ref: Ananthanarayan and Paniker’s Textbook of Microbiol­


ogy – 10th ed – page 549
4. Ans.  (a) IgM antibodies in serum • Already explained Q.8

Ref: Ananthanarayan and Paniker’s Textbook of Microbiol­


11. Ans.  (a) P gene
ogy – 10th ed – page 546
• HAV – Diagnosis is by IgM anti HAV antibody which

Ref: Ananthanarayan and Paniker’s Textbook of Microbiol­
starts appearing around 2 weeks and disappears after ogy – 10th ed – page 548
3-4 months • Already explained Q.8

5. Ans.  (c) Common cause of hepatitis in children 12. Ans.  (c) Hepatic adenoma
Ref: Ananthanarayan and Paniker’s Textbook of Microbiol­ Ref: Ananthanarayan and Paniker’s Textbook of Microbiol­
ogy – 10th ed – page 545 ogy – 10th ed – page 547
• Type A hepatitis – infectious hepatitis

• HBV infections resolves in 90% of individuals

• Most common cause of hepatitis in children



• Remaining will go for chronicity – that leads to chronic

• Children – self-limiting

hepatitis – inflammation and fibrosis – leads to cirrhosis
• Cirrhosis after years may end up in Hepatocellular

6. Ans.  (c) Fecal-oral route carcinoma


Ref: Ananthanarayan and Paniker’s Textbook of Microbiol­ • Hepatic adenoma has not be associated with HBV

ogy – 10th ed – page 545


13. Ans.  (b) Acute hepatitis B with high infectivity
• Transmission of HAV is only by fecal-oral route

• Other hepatitis virus which gets transmitted by fecal-



Ref: Ananthanarayan and Paniker’s Textbook of Microbiol­
oral route is HEV ogy – 10th ed – page 552
• IgM – denotes acute infection

7. Ans.  (b) Transmitted by feco-oral route • HbeAg positive – denotes high infectivity – means there

Ref: Ananthanarayan and Paniker’s Textbook of Microbiol­ is active replication of virus


ogy – 10th ed – page 547
14. Ans.  (c) HbsAg + HBeAg + HBV DNA
• Mode of transmission of HBV is through blood and

blood products, needle prick, perinatal transmission Ref: Ananthanarayan and Paniker’s Textbook of Microbiol­
and sexual route ogy – 10th ed – page 549
• Only HAV and HEV gets transmitted by fecal-oral route

• Super carriers – high titer of HBsAg along with HbeAg +

HBV DNA
269
15. Ans.  (c) IgM anti - HBc 23. Ans.  (c) Chances of perinatal transmission are directly
Unit 3     Virology

correlated with degree of maternal viremia


Ref: Ananthanarayan and Paniker’s Textbook of Microbiol­
ogy – 10th ed – page 550 Ref: Ananthanarayan and Paniker’s Textbook of Microbiol­
• HbcAg – cannot be demonstrable in blood

ogy – 10th ed – page 553; Harrison T.B of internal medicine
• The earliest antibody marker to be identified is Anti

19th ed – page 2009
HbcAg – IgM • Transmission is based on the degree of maternal viremia

• After six months IgG starts appearing


• – this can be detected with the help of PCR for HCV RNA
• But the Most common mode of transmission is blood

16. Ans.  (a) HBsAg + HBeAg transfusion


Ref: Ananthanarayan and Paniker’s Textbook of Microbiol­
24. Ans.  (d) HCV
ogy – 10th ed – page 550
Acute hepatitis has the following serological markers: Ref: Ananthanarayan and Paniker’s Textbook of Microbiol­
• HbsAg

ogy – 10th ed – page 552
• IgM anti HbcAg
• • About 50-80% of the persons infected with HCV goes for

• HbeAg
• chronicity

17. Ans.  (a) HBeAg 25. Ans.  (b) Flavivirus


Ref: Ananthanarayan and Paniker’s Textbook of Microbiol­ Ref: Ananthanarayan and Paniker’s Textbook of Microbiol­
ogy – 10th ed – page 550 ogy – 10th ed – page 552
• Already explained Q.13
• • HCV – belongs to hepacivirus in the family of flaviviridae

18. Ans.  (a) HBeAg 26. Ans.  (d) HCV


Ref: Ananthanarayan and Paniker’s Textbook of Microbiol­ Ref: Ananthanarayan and Paniker’s Textbook of Microbiol­
ogy – 10th ed – page 550 ogy – 10th ed – page 552
• Already explained Q.13
• • Already explained Q.24

19. Ans.  (c) High infectivity 27. Ans.  (a) SS-RNA virus
Ref: Ananthanarayan and Paniker’s Textbook of Microbiol­ Ref: Ananthanarayan and Paniker’s Textbook of Microbiol­
ogy – 10th ed – page 550 ogy – 10th ed – page 553
Ref: Already explained Q.13 • HDV – ssRNA virus; Defective or dependovirus.

20. Ans.  (a) HBsAg 28. Ans.  (d) DNA viruses

Ref: Ananthanarayan and Paniker’s Textbook of Microbiol­ Ref: Ananthanarayan and Paniker’s Textbook of Microbiol­
ogy – 10th ed – page 550 ogy – 10th ed – page 553
Markers of HBV replication are: • HDV - Defective RNA virus – dependent virus

• HbeAg • Needs HBV for replication


• It can either gets infected along with HBV (Co-infection)


• HBV DNA – viral load/viral copies



or after infection with HBV (Super infection)


21. Ans.  (b) HBcAg • HBV is a DNA virus; HDV is a RNA virus.

Ref: Ananthanarayan and Paniker’s Textbook of Microbiol­ 29. Ans.  (a) Hep. E
ogy – 10th ed – page 548
Ref: Ananthanarayan and Paniker’s Textbook of Microbiol­
• HbcAg is core antigen; It is not demostrated in the
ogy – 10th ed – page 554

circulation because it is enclosed within the HbsAg


envelope • HEV – unique feature is clinical severity and high case

fatality rate around 20-40% in pregnant women


22. Ans.  (c) Intramuscular deltoid • Severity is more in the last trimester.

Ref: Ananthanarayan and Paniker’s Textbook of Microbiol­ 30. Ans.  (b) Adults
ogy – 10th ed – page 552
Ref: Ananthanarayan and Paniker’s Textbook of Microbiol­
• Vaccine is given IM in the deltoid or anterolateral aspect
ogy – 10th ed – page 554

of thigh in children – 0, 1, 6 months


• Not to be given in gluteal - poor immune response

• Hepatitis E virus affects adults whereas HAV affects

• Seroconversion – 90%

children.
270
31. Ans.  (a) Hepatitis A 34. Ans.  (b) Virus can integrate with host DNA and form

Chapter 34     Hepatitis Virus


complementary DNA
Ref: Ananthanarayan and Paniker’s Textbook of Microbiol­
ogy – 10th ed – page 554 Ref: John Carter – Virology Principles and Applications –
• HAV and HEV - Both gets infected by fecal-oral route

page 290
• Because of clinical features – both looks same.
• • HBV is the most significant agent for hepatocellular

carcinoma
32. Ans.  (c) HCV • The prevalence of HCC closely parallels the prevalence

Ref: Essentials of Medical Microbiology – Apurba Sastry – of persistent HBV infection with the highest incidences
Page 116 in Asia and in central and southern Africa.
• In most of the tumors, HBV DNA is integrated into the
• Cryoglobulinemia is a condition in which the blood

cell genome, and in most cases, the virus DNA has


contains cryoglobulins – that is IgM antibodies that are undergone rearrangements, including deletions.
directed against Fc portion of IgG • The P and C gene ORFs have been generally destroyed,
• It is associated with multiple myeloma and Hepatitis C

but the S and X ORFs are often intact.


virus infection. • The only viral gene product that is consistently present

in the tumor cells is the X protein


33. Ans.  (b) Hepatitis
Ref: Review of medical microbiology – Levinson – 13th ed – 35. Ans.  (b) Immunization against Hepatitis B virus will
Page 750 reduce the hepatitis caused by HDV; (c) HDV has RNA in
its genome and it has no polymerase in its virion
• Hepatitis B virus is the only virus that produce genome

DNA by reverse transcription with mRNA as template Ref: Review of Medical Microbiology – 13th edition – Levin­
• Another identification here is cccDNA – covalently

son – page 760
closed circular DNA • Hepatitis D virus is an enveloped virus with RNA genome
• Following entry into the host cells, the virion RC DNA

• It does not have polymerase in its virion; cannot replicate


is released into the nucleus for conversion into CCC

on its own; called as defective virus


DNA, which is the first viral product to be made in a • HDV can replicate only when co-infected with HBV
newly infected cell. CCC DNA then serves as the viral

• HDV transmitted as like HBV – blood and blood borne


transcriptional template for the synthesis of all viral

products, vertical transmission


RNAs by the host RNA polymerase II. • Lab diagnosis – detection of delta antigen of IgM
• Though not integrated into the host chromosomes, the

antibody to delta antigen in blood


episomal HBV CCC DNA functions equivalently to that • No specific therapy; no vaccination
of the integrated retroviral provirus and is the molecular

• Immunization against Hepatitis B virus will reduce the


basis of HBV persistence.

hepatitis caused by HDV.

36. Ans.  (d) Hepatitis E


Ref: Ananthanarayan and Paniker’s Textbook of Micro­
biology – 10th ed – page 554
For explanation please refer Q. 29.

271
35 Human
Immunodeficiency Virus
INTRODUCTION
y Family Retroviridae has many viruses that can affect different
y
host species
y All retroviruses have an outer envelope of lipid and viral
y
proteins
y The envelope encloses the core which has other proteins
y
y The core has two molecules of viral RNA – single stranded
y
positive sense RNA (ss RNA)2

STRUCTURE OF HIV
y HIV is a lentivirus that infects and destroys cells in the
y
immune system.
y Lentiviruses are a subgroup of retroviruses; the word lenti
y
means slow, since HIV produces a slow viral infection. It
comes under lentiviruses Figure 1: Structure of HIV
y HIV is an enveloped virus
y
y The virus envelope is composed of two phospholipid layers
GENOMIC STRUCTURE OF HIV
y
derived from the host cell membrane
y The envelope has an envelope coated protein, glycoprotein y HIV genome has nine genes and long terminal repeat (LTR)
y
(gp) 160.
y
regions at either end of the genome
y Gp160 is composed of two subunits, gp120 and gp41. y There are three structural genes namely env, gag and pol
y
y Gp 120 is an external protein and it contains sites that bind
y
 Envelope (env) coding for envelop of HIV;
y
CD4 cells and coreceptors on the surface of human CD4 T

 Group-specific antigens (gag) coding for capsid and
cells.

matrix;
y gp41 is membrane-bound protein.  DNA polymerase (pol) coding for integrase, protease and
y
y Inside the viral envelope there is a layer called the matrix,

reverse transcriptase enzymes.
y
which is made from the protein p17. y Regulatory genes that are needed for viral replication are tat,
y The viral core or capsid is made up of protein p24.
y
rev, nef
y
y Inside the core there are three enzymes required for HIV y Accessory genes needed for viral replication and regulations
y
replication:
y
are vif, vpr, vpx, vpu
 Reverse transcriptase (RT)
y Importance of genes:

 Integrase
y
 For diagnostic testing, the detection of antibodies and

 Protease

viral proteins (Env, Gag and Pol) are often used

y Within the core is HIV genetic material which consists of two  During the window period of detection, the p24 (Gag)
y
positive strands of single-stranded Ribonucleic Acid (RNA).

protein is used as a diagnostic for HIV infection.
Table 1: Types of genetic structure and genes

Chapter 35     Human Immunodeficiency Virus


Genetic structure types Genes


Structural genes Env, gag and pol
Regulatory genes Tat, rev, nef
Accessory genes Vif, vpr, vpx and vpu

Figure 3: Classification of HIV


Figure 2: Genomic structure of HIV (Source: Hivbook.com 2007) Table 3: Subtypes of HIV virus and their geographical

Table 2: Gene product and their description



distribution

Gene products Descriptions Subtypes Geographical distribution


gp160 Precursor of envelope glycoproteins A, C, D Africa

gp120 Outer envelope glycoproteins of virion C India and China


p66 Reverse transcriptase and Rnase H from E, C, B Asia
polymerase gene product E* Thailand
p55 Precursor of core proteins, polyprotein from
gag gene * E is now actually a recombinant with A – AE forms or Circulating
recombinant forms (CRFs)
p51 Reverse transcriptase, RT
gp41 Transmembrane envelope glycoproteins, TM
LIFE CYCLE OF HIV
p32 Integrase , IN
y HIV virus enters the host cells with the help of CD4 receptors
p24 Nucleocapsid core protein of virion
y

present on the T lymphocytes


p17 Matrix core protein of virion y It can also attack other cells like macrophages and dendritic
y

cells
y HIV-1 needs coreceptor for entry into cells
CLASSIFICATION OF HIV y

y Coreceptors involved are: CXCR4 and CCR5


y

y There are two HIV types:


y

 HIV-1
Receptor for HIV virus CD4 in T cells
 HIV-2

Receptor of HIV virus in gut Integrin α-4 β-7


y HIV-1 is the most prevalent type throughout the world.
Coreceptor for macrophage tropic strains of CCR5
y

y HIV-2 has limited geographic distribution especially in


HIV-1
y

African countries.
y HIV -1 is further subdivided into three main groups with an
y
Coreceptor for lymphocyte tropic strains of CXCR4
additional group HIV-1
y Groups are M, N, O and P
y The first step in fusion involves the high-affinity attachment
y

y P is a group seen only in women living in Paris.


y

of viral gp120 to a CD4 molecule.


y

y Group M is the most common in world. It is further subdivided


y Fusion of the virus envelope with host cell membranes and
y

into subtypes as given in the following figure.


y

releases the viral genetic material (RNA) in the protoplasm of


• Most common type in the world is – HIV-1

the host cell occurs.
• Most common group in the world is – Group M

y Then reverse transcription occurs where single-strand HIV
y

• Most common subtype seen in India – Subtype C



RNA molecules are converted to double-strand c-DNA
molecules. (by reverse transcriptase)
273
y These DNA molecules enter the nucleus
Unit 3     Virology

y The HIV enzyme “integrase” integrates the HIV DNA with the host cell’s DNA. The integrated HIV DNA is called provirus. The
y

provirus may remain inactive for several years, producing few or no new copies of HIV. (Reason for latency)
y When host cell is activated, transcription occurs producing copies of HIV RNAs and mRNAs. These are moved from nucleus to
y

cytoplasm
y Then viral assembly occurs and new viral particles are expelled by budding
y

y During budding the HIV envelope also acquires host membrane proteins and a lipid bi-layer.
y

Figure 4: Life cycle and replication of HIV

MODES OF TRANSMISSION OF HIV Difference of Pathogenesis in HIV-2


y Transmission occurs through:
y
y HIV-2 is less easily transmitted and less infectious
y

 Sexual intercourse (vaginal, anal or oral)



y It has a longer incubation period
y

 Parenteral route (IV drug use)



y Immunosuppression in HIV-2 infected persons is significantly
y

 Unsafe blood transfusion



slower than in HIV-1 infected persons.
 Organ transplantation

y Progression of HIV-2 is slower and infection leads to a signifi-
y

 Vertical transmission from mother to child



cantly lower plasma viral load.
y The virus can be found in blood and body fluids like semen,
y

vaginal secretions, breast milk, cerebrospinal fluid, amniotic CLINICAL STAGES OF HIV INFECTION
fluid and synovial fluid containing blood.
y Acute infection phase/primary infection or window period:
y

 Immediately after the infection, there is wide virus


PATHOGENESIS OF HIV INFECTION


dissemination and seeding of lymphoid organs.


y After the entry of virus into the body, viral replication starts, at
y  This period usually lasts for 3–6 weeks and terminates


both the sites of entry and at the draining lymph nodes within with appearance of an adaptive immune response to HIV.
72 hours y Chronic asymptomatic phase:
y

y Lymphocytes are immediately activated once the infection is


y  Between six to twelve months after infection, the host’s


acquired. immune response establishes the plasma virus load set


y HIV replicates more in these activated cells.
y point. The plasma virus load set point is an important
y Once there is an active infection, CMI and humoral immunity
y determinant in the progression of the HIV disease
starts exhibiting leading to neutralizing HIV antibodies.  The chronic infection phase may last for 7–10 years.


y HIV antibodies are first detected by IgM ELISA around 3


y  The virus becomes largely sequestered in lymphoid tissue


weeks after infection and continues to replicate there during the years of clinical
y The period from virus entry into the cells till the detectable
y latency
levels of HIV is called as window period (Antibody tests are y AIDS
y

negative)  The advanced stage of HIV infection is characterized by




y This seronegative period may ranges from 3 weeks to 3


y an increase in the plasma viral load. Dysfunction in the
months. immune system occurs.

274
Clinical AIDS is defined as a CD4 cell count of less than

Chapter 35     Human Immunodeficiency Virus



• Cytomegalovirus disease (other than liver, spleen, or nodes),


200/mm3 and/or the appearance of AIDS defining


onset at age >1 month


illnesses.
• Cytomegalovirus retinitis (with loss of vision)

• Encephalopathy attributed to HIV


Table 4:  Classification of HIV infection

• Herpes simplex: chronic ulcers (>1 month’s duration) or


Group I Acute HIV syndrome bronchitis, pneumonitis or esophagitis (onset at age >1 month)
• Histoplasmosis, disseminated or extrapulmonary
Group II Asymptomatic infection

• Isosporiasis, chronic intestinal (>1 month’s duration)


Group III Persistent generalized lymphadenopathy • Kaposi’s sarcoma


Group IV Other diseases • Lymphoma, Burkitt’s (or equivalent term)


 Subgroup A Constitutional disease • Lymphoma, immunoblastic (or equivalent term)


 Subgroup B Neurologic disease • Lymphoma, primary of brain


 Subgroup C Secondary infectious disease • Mycobacterium avium complex or Mycobacterium kansasii,


 Subgroup C1 Specified infectious disease like disseminated or extrapulmonary


P. carinii, Cryptosporidiosis, toxoplasmosis, • Mycobacterium tuberculosis of any site, pulmonary,

strongyloidosis disseminated or extrapulmonary


 Subgroup C2 Oral hairy leukoplakia, Salmonella, bacteremia, • Mycobacterium, other species or unidentified species,

nocardiosis, TB, thrush disseminated or extrapulmonary


 Subgroup D Kaposi sarcoma, lymphoma • Pneumocystis jirovecii (previously known as Pneumocystis

 Subgroup E Other conditions carinii) pneumonia


• Pneumonia, recurrent

• Progressive multifocal leukoencephalopathy


PROGRESSION OF HIV TO AIDS

• Salmonella septicemia, recurrent


y Based on the median time period for conversion of HIV to


y
• Toxoplasmosis of brain, onset at age >1 month

AIDS, the persons infected with HIV are classified as: • Wasting syndrome attributed to HIV

 Typical progressors


 Rapid progressors


LABORATORY DIAGNOSIS
 Long term nonprogressors (LTNP)


 Subset of LTNP – Elite controllers



y Serological tests:
y

y Elite Controllers: Also known as natural controllers are


y
 Enzyme linked immunosorbent assays (ELISAs)


a subset of LTNPs. Their immune system, despite being  Rapid tests




infected with HIV, has been able to successfully suppress the  Western blots


virus to an undetectable level (HIV RNA below 50 copies/mL) y NAATs – sensitive tests for detection of HIV infection – viral
y

for many years in the absence of ART load can be detected

DIAGNOSIS OF HIV IN INFANTS


y Diagnosis in a child less than 18 months cannot be done
y

using antibody based assays as maternal antibodies may be


present in the infant’s circulation.
y Up to the age of 18 months, the diagnosis of HIV infection can
y

only be reliably made by DNA PCR

Figure 5: Different type of progressions of HIV to AIDS


AIDS defining opportunistic illness are: (Ref: Harrison’s 19th ed)
• Multiple or recurrent bacterial infections

• Candidiasis of bronchi, trachea or lungs


• Candidiasis of esophagus

• Cervical cancer (invasive)


• Coccidioidomycosis,

• Extrapulmonary Cryptococcosis

275
• Cryptosporidiosis, chronic intestinal (>1 month’s duration)

Figure 6: Laboratory methods of HIV diagnosis


Table 5:  Serological markers helpful for diagnosis of HIV y Non-nucleoside reverse transcriptase inhibitor (NNRTI):
Unit 3     Virology

infection inhibits the HIV reverse transcriptase enzyme by binding to it


y Protease inhibitors (PI): binds to the active site of the protease
y

Early infection P24 antigen and prevents maturation.


Acute (Seroconversion stage) Anti HIV IgM and IgG
Partial illness gp120, gp41, gp160 antibodies, POST EXPOSURE PROPHYLAXIS
Anti HIV IgG, western blot
y When an occupational exposure occurred, nature of exposure,
y

Carriers and asymptomatic P24 antigen, gp41, gp160, HIV status of the individual should be known for PEP
gp120 antibodies, Anti HIV IgG, y Immediately after a needle stick injury or blood spill, wash
y

western blot, the area with soap and water thoroughly


Persistent generalized Anti HIV IgG, western blot y Contact the casualty medical officer and start PEP within 72
y

lymphadenopathy hours of exposure


AIDS All antibodies decline; Anti HIV
IgG, western blot are positive Table 7:  Levels of exposure – NACO guidelines
Mild exposure Mucus membrane/non-intact skin with
small volume, e.g., a superficial wound
Table 6:  NACO strategies for testing different categories
(erosion of the epidermis) with a plain or
of samples
low calibre needle; contact with the eyes
Strategies Samples or mucous membranes; subcutaneous
injections following small-bore needles
Strategy I To screen blood/blood products, organ, tissues
and sperms Moderate Mucus membrane/non-intact skin with
exposure large volumes or percutaneous superficial
Strategy IIA For surveillance
exposure with solid needle (e.g., a cut or
Strategy IIB For diagnosis–to determine the HIV status of needle stick injury penetrating gloves).
clinically symptomatic suspected AIDS cases
Severe exposure Percutaneous with large volume, e.g. an
Strategy III For diagnosis in asymptomatic individuals accident with wide bore needle (>18G)
visibly contaminated with blood; a deep
VACCINE RESEARCH IN HIV wound (hemorrhagic wound and/or very
painful); transmission of a significant
y Modified whole virus has been tried
y

volume of blood; an accidental injury with


y Subunits based on envelope glycoprotein is under
y

material, which has previously been used


development intravenously or intra-arterially.
y Target cell protection by anti-CD4 antibody or genetically
y

engineered CD4
Dosages of the Drugs for PEP for adults and adolescents:
y PEP - vaccine
FDC of Tenofovir ( TDF) 300 mg plus Lamivudine (3TC) 300 mg
y

plus Efavirenz (EFV) 600 mg once daily for 4 weeks


TREATMENT Tests needed to be done after exposure:
There are three major classes of antiretroviral drugs available: • HIV, Anti HCV, HbsAg

y Nucleoside/Nucleotide reverse transcriptase inhibitor


y
• Baseline, 3rd month and 6th month

(NRTI): acts as a DNA chain terminator

276
MULTIPLE CHOICE QUESTIONS

Chapter 35     Human Immunodeficiency Virus


1. Which of the following is a lentivirus? 11. Cell fusion of HIV with target cell is done by:
 (Recent Pattern July 2016)  (Recent Pattern July 2016)
a. HIV b. HBV a. gp 120 b. gp 41
c. HCV d. Rabies virus c. p 24 d. p 18
2. Which HIV virus is more dangerous? 12. Most common genital lesion in HIV patient is:
a. HIV-1 (Recent Pattern Dec 2016)  (Recent Pattern Dec 2016)
b. HIV-2 a. Chlamydia b. Herpes
c. Both are same c. Syphilis d. Candida
d. It depends on host factors 13. Most common opportunistic infection in HIV globally:
3. HIV envelop is formed by: (Recent Pattern Dec 2016)  (Recent Pattern Nov 2015)
a. Host cell a. P. jiroveci b. Candida
b. Virus c. M.tuberculosis d. Cryptococcus
c. Both 14. True about HIV all, except: (Recent Pattern Dec 2016)
d. None a. PML caused by JC virus
4. Nef gene in HIV is used for: (Recent Pattern Dec 2012) b. CNS lymphoma is the most common CNS tumor
a. Enhancing the expression of genes c. CMV is the most common cause of retinitis
b. Enhancing viral replication d. Most common cause of seizure is candida
c. Decreasing viral replication 15. Oral hairy leukoplakia is caused by:
d. Maturation  (Recent Pattern Dec 2013)
5. Subtype of HIV most common in India: a. HSV b. EBV
 (Recent Pattern Dec 2013) c. CMV d. HPV
a. A b. B 16. A person has unprotected sex 3 weeks back. To rule out
c. C d. D HIV infection the best test is: (Recent Pattern Dec 2016)
6. Mother to child transmission of HIV: a. P24 antigen assay b. ELISA
 (Recent Pattern Dec 2013) c. Western blot d. Lymphnode biopsy
a. 25% b. 50% 17. Window period in HIV infection:
c. 60% d. 75%  (Recent Pattern Dec 2015)
7. Most common mode of transmission of HIV after sexual a. 1 – 2 weeks b. 4 – 8 weeks
transmission: (Recent Pattern Dec 2016) c. 8 – 12 weeks d. >12 weeks
a. Blood and blood products 18. Most sensitive test for HIV infection:
b. Occupational  (Recent Pattern Dec 2013)
c. Perinatal a. Western blot b. ELISA
d. Breast feeding c. Agglutination d. CFT
8. The chance that a health worker gets HIV from an 19. False about p24 is: (Recent Pattern Dec 2012)
accidental needle prick is: (Recent Pattern Dec 2015) a. Seen after 3 weeks of infection
a. 1% b. 10% b. Can’t be seen in first week
c. 95% d. 100% c. Can’t be detected after seroconversion
9. The receptor through which M-tropic HIV strains bind: d. Detected by ELISA
 (Recent Pattern Dec 2013) 20. Which of the following is included in AIDS related
a. CCR5 b. CXR4  complex? (Recent Pattern 2018)
c. CXCR5 d. Any of the above a. Vaginal candidiasis b. Invasive Ca cervix
10. Function of gp120 of HIV envelop: c. Cyclospora infection d. Herpes zoster virus
 (Recent Pattern July 2016) 21. Mean transformation time for HIV to AIDS is:
a. Cell fusion a. 7.5 years (Recent Pattern 2018)
b. Cell penetration b. 10 years
c. Attachment of CD4 receptor c. 15 years
d. Integration of nucleic acid d. 12 years

277
ANSWERS AND EXPLANATIONS
Unit 3     Virology

1. Ans.  (a) HIV 7. Ans.  (a) Blood and blood products


Ref: Ananthanarayan and Paniker’s Textbook of Microbiol- Ref: Ananthanarayan and Paniker’s Textbook of Microbiol-
ogy – 10 th ed – Page 574 ogy – 10 th ed – Page 586
HIV, the causative agent of AIDS belongs to the lentivirus The second mode of transmission of HIV next only to sexual
subgroup of family Retroviridae.
route is through blood and blood products. Screening of
2. Ans.  (a) HIV-1 blood donors is mandatory now, which must include p24
antigen screening.
Ref: Ananthanarayan and Paniker’s Textbook of Microbiol-
ogy – 10 th ed – Page 576 8. Ans.  (a) 1%
HIV type 1 is more virulent. HIV 2 is less virulent than
Ref: Ananthanarayan and Paniker’s Textbook of Microbiol-
HIV1. The original isolates of HIV and the related strains
ogy – 10 th ed – Page 586
prevalent all over the world belong to HIV type 1.
The chance of HIV infection per exposure is 0.5-1% for nee-
3. Ans.  (c) Both dle stick injuries.
Ref: Ananthanarayan and Paniker’s Textbook of Microbiol- Types of exposure Approximate
ogy – 10 th ed – Page 575 chance of HIV
The lipoprotein envelope of HIV consists of lipid part and infection per
protein part. exposure
The lipid part is derived from the host cell and the protein Sexual intercourse 0.1-1%
part is virus encoded. Blood and blood products transfusion >90%
4. Ans.  (c) Decreasing viral replication Tissue and organ donation (semen, 50-90%
cornea, bone marrow and kidney)
Ref: Ananthanarayan and Paniker’s Textbook of Microbiol-
ogy – 10 th ed – Page 575 Injections and injuries (needle stick/ 0.5 – 1%
surgical wounds)
Nef is negative factor gene which decreases or down
regulates viral replication. Mother to baby 30%
Non structural and regulatory genes
9. Ans.  (a) CCR5
Gene Function
Ref: Ananthanarayan and Paniker’s Textbook of Microbiol-
tat (trans activating gene) Enhances expression of all viral
ogy – 10 th ed – Page 577
genes
Binding of HIV to CD4 receptor requires the participation
nef (Negative factor gene) Down regulates viral replication of a co-receptor molecule, which has been identified as
rev (Regulator of virus gene) Enhances expression of structural CC4 for T-cell tropic HIV strains and CCR5 for macrophage
proteins tropic (M-tropic) strains.
vif (viral infectivity factor Influences infectivity of viral
10. Ans.  (c) Attachment of CD4 receptor
gene) particles
vpu (only in HICV-1) and vpx Enhance maturation and release Ref: Ananthanarayan and Paniker’s Textbook of Microbiol-
(only in HIV-2) of progeny virus from cells. ogy – 10 th ed – Page 577
vpr Stimulates promoter region of Specific binding of the virus to the CD4 receptor is by the
the virus envelope glycoprotein gp120.

LTR sequences Contains the sequences thet 11. Ans.  (b) gp 41


give promoter, enhancer and
integration signals Ref: Ananthanarayan and Paniker’s Textbook of Microbiol-
ogy – 10 th ed – Page 577
5. Ans.  (c) C Following attachment, cell fusion is necessary for HIV
Ref: Ananthanarayan and Paniker’s Textbook of Microbiol- infection to take place.
ogy – 10 th ed – Page 575 Fusion is brought about by transmembrane gp41.
In India and China, subtype C is the most prevalent.
12. Ans.  (b) Herpes
6. Ans.  (a) 25% Ref: Ananthanarayan and Paniker’s Textbook of Microbiol-
Ref: Ananthanarayan and Paniker’s Textbook of Microbiol- ogy – 10 th ed – Page 579
278 ogy – 10 th ed – Page 586 Herpes simplex viral infection is the most common genital
Mother to child transmission of HIV is 30%. infection in HIV patients.
13. Ans.  (a) P. jiroveci 20. Ans.  (b) Invasive Ca cervix

Chapter 35     Human Immunodeficiency Virus


Ref: Ananthanarayan and Paniker’s Textbook of Microbiol- Ref: Harrison’s T.B of internal medicine – 19th edition – Page
ogy – 10 th ed – Page 578 1215
HIV/AIDS; National Institute of Allergy and Infectious • AIDS defining opportunistic illness are:
Diseases National Institutes of Health Bethesda, Maryland

• Multiple or recurrent bacterial infections


20892

• Candidiasis of bronchi, trachea, or lungs


Pneumocystis carinii pneumonia is the most common

• Candidiasis of esophagus
opportunistic infection in HIV patients.

• • Cervical cancer (invasive)


14. Ans.  (d) Most common cause of seizure is candida • • Coccidioidomycosis,
• • Extrapulmonary Cryptococcosis
Ref: Ananthanarayan and Paniker’s Textbook of Microbiol- • • Cryptosporidiosis, chronic intestinal (>1 month’s dura-
ogy – 10 th ed – Page 579 tion)
The typical CNS opportunistic infections are toxoplasmosis • • Cytomegalovirus disease (other than liver, spleen, or
and cryptococcosis. nodes), onset at age >1 month
Lymphomas of the CNS are more common. • • Cytomegalovirus retinitis (with loss of vision)
• • Encephalopathy attributed to HIV
15. Ans.  (b) EBV • • Herpes simplex: chronic ulcers (>1 month’s duration) or
Ref: Ananthanarayan and Paniker’s Textbook of Microbiol- bronchitis, pneumonitis, or esophagitis (onset at age >1
ogy – 10 th ed – Page 578 month)
Oral hairy leukoplakia is caused by Epstein-Barr virus • • Histoplasmosis, disseminated or extrapulmonary
(EBV) infection of the oral mucosa. It most often occurs in • • Isosporiasis, chronic intestinal (>1 month’s duration)
association with HIV infection. It has been less frequently • • Kaposi’s sarcoma
described in immunosuppressed patients, especially • • Lymphoma, Burkitt’s (or equivalent term)
following organ transplantation, and is rare in immune • • Lymphoma, immunoblastic (or equivalent term)
competent individuals. • • Lymphoma, primary of brain
• Mycobacterium avium complex or Mycobacterium
16. Ans.  (a) P24 antigen assay

kansasii, disseminated or extrapulmonary


Ref: Ananthanarayan and Paniker’s Textbook of Microbiol- • • Mycobacterium tuberculosis of any site, pulmonary,
ogy – 10 th ed – Page 580 disseminated, or extrapulmonary
P24 antigen is detectable as ealy as 2 weeks following HIV • • Mycobacterium, other species or unidentified species,
infection and is the earliest diagnostic marker to appear in disseminated or extrapulmonary
blood. • • Pneumocystis jirovecii (previously known as Pneumo-
P24 antigen capture assay can be used to detect p24 cystis carinii) pneumonia
antigen. • • Pneumonia, recurrent
• Progressive multifocal leukoencephalopathy
17. Ans.  (b) 4 – 8 weeks

• • Salmonella septicemia, recurrent


Ref: Ananthanarayan and Paniker’s Textbook of Microbiol- • • Toxoplasmosis of brain, onset at age >1 month
ogy – 10 th ed – Page 580 • • Wasting syndrome attributed to HIV
Window period is the period of serological negativity.
21. Ans.  (b) 10 years
The patient is infected and is infectious but undetectable
by the serological tests. Ref: Zuckermann principles and practice of clinical virology
Seroconversion refers to the appearance of IgM antibodies – 5th edition – Page 730
in the patient’s serum, following the initial period of
• Primary HIV infection entails symptomatic fever and
viremia and p24 antigenemia.

lymphadenopathy in about 50% of infections.


IgM antibodies appear in about 4-6 weeks following
• Following seroconversion, there follows an asymptom-
infection, which constitutes the window period.

atic phase of infection


18. Ans.  (b) ELISA • CD4 cell counts decline rapidly and patients progress

clinically in 5 years (rapid progressors), or a protracted


Ref: Ananthanarayan and Paniker’s Textbook of Microbiol-
course where the CD4 cell counts remain in the
ogy – 10 th ed – Page 582
normal range and viral load low or undetectable with
ELISA is the test which is most commonly used for asymptomatic disease for 15 years (long-term non-
screening of HIV in patients because it is more sensitive. progressors).
• These differences probably represent the limits of a
19. Ans.  (a) Seen after 3 weeks of infection

normal distribution
Ref: Ananthanarayan and Paniker’s Textbook of Microbiol- • The time to progression to AIDS following infection by

ogy – 10 th ed – Page 580 HIV is variable but averages about 10 years.


P24 antigen can be seen in 2 weeks following HIV infection. 279
It disappears before appearance of IgM antibodies.
36
Miscellaneous Viruses
RUBELLA VIRUS – GERMAN MEASLES y BK virus can grow in a wide range of primary and continuous

y
cell cultures
y Belongs to Togaviridae: Enveloped RNA virus
y
y Infection is acquired from a clinical or subclinical case; no
PARVOVIRIDAE
y
carriers
y Gets transmitted by either droplet infection or vertical trans-
Parvo Virus B19 (Smallest Virus)
y
mission
y It will affect the children of 3–10 years y Non enveloped ss DNA
y
y
y Around 40% of the females are susceptible to infection; Hence y Smallest virus with smallest genome
y
y
vaccination is must to avoid congenital syndrome y Parvo virus B19 – causes infection in children

y
y Depending upon the trimester – infection rate differs y Parvoviruses has a special tropism for erythroid progenitor
y
y
y Damage to fetus occurs most during first trimester cells in the adult bone marrow and fetal liver
y
y It binds to the P antigen in the blood cell surface
Congenital rubella syndrome: Cardiac defects, cataract and
y
y Infection is acquired through respiratory route
deafness
y
y It causes Slapped cheek disease or fifth disease or erythema
y
Expanded rubella syndrome – includes hepatosplenomegaly, infectiosum in children
thrombocytopenic purpura, myocarditis and bone lesions y Causes aplastic crisis in children with sickle cell anemia
y
y In adults it can cause arthropathy and aplastic anemia.
Diagnosis
y
y In pregnancy, it leads to nonimmune fetal hydrops
y
IgM antibody: Acute infection; IgG means that person is immune
to Rubella infection
y Vaccine strain: RA 27/3
y
y MMR vaccine: Those who are vaccinated should avoid getting
y
pregnant for three months

PAPOVAVIRIDAE
HUMAN PAPILLOMA VIRUS
y Belongs to Papovaviridae
y
y Small, nonenveloped, DNA tumor viruses
y
Table 1:  HPV types and clinical illness
HPV 6, 11 Intraepithelial neoplasia
HPV 16, 18 Cervical cancer
HPV 1, 2, 3, 4 Verruca vulgaris
HPV 6, 11 Condyloma acuminatum Figure 1: Slapped cheek appearance caused by Parvo virus B19
y Recombinant vaccine: (Courtesy: CDC)
y
 Bivalent vaccine: HPV 16, 18
Disease conditions associated with parvovirus:

 Quadrivalent vaccine: Gardesil – HPV 6, 11, 16, 18
• Erythema infectiosum or fifth disease or slapped cheek

 Nonavelent vaccine: HPV 6, 11, 16, 18, 31, 33, 45, 52

disease in children

y Vaccine is to be given for female population aged 9 to 26 years
• Arthralgia or Polyarthropathy in adults
y

• Transient aplastic crisis
POLYOMAVIRUS

• Pure red cell aplasia

y JC virus – grows only in human fetal glial cell cultures • Hydrops fetalis
y

y BK virus – isolated in kidney transplant patients -nephropathy • Papular purpura
y

y Human rotavirus does not grow in cell cultures, except Group
REOVIRIDAE

Chapter 36     Miscellaneous Viruses


y

A few strains have been made change to grow in serial culture.


ROTAVIRUS
y Member of Reoviridae
y
Prophylaxis
y Has 11 segments of dsRNA – characteristic feature is genetic
y
y Vaccine: Rotarix and Rotateq
y

reassortment y Both are oral live attenuated vaccines


y

y Vaccines should be given before 32 weeks of age – to avoid


y

intussusception
Rotarix (2 doses) P1 A (8) G 1
Rotateq (3 doses) G1, G2, G3, G4 and P (8)

Table 2:  Other gastroenteritis viruses


Virus Characteristics
Norwalk virus • • Family - calciviridae
• • Causes epidemics
• • Associated with consumption of raw oysters
• • Causes Adult diarrhea
Adenovirus • Types 40, 41 causes diarrhea

Figure 2: Electron microscopic appearance of rotavirus • Difficult to grow in culture


(Courtesy: CDC/ Dr. Erskine Palmer)


Astrovirus • Star-shaped virus

y Double walled viruses – Cart-wheel appearance


y
• Epidemics of diarrhea in children

y Most common cause of diarrheal disease in infants and


Coronavirus • Virus isolated in human feces
y

children •

y Rotaviruses can be classified into five species Group A to E


y
• Association with diarrhea unknown

based on the antigenic structural protein called VP6.


y Major antigens are VP6, VP4 and VP7
y

SLOW VIRAL DISEASE


y Group A affects children
y Some diseases in humans take slow or chronic persistent type
y

y Group B – Adult diarrhea rotavirus (ADRV)


y

of infections called as slow viral diseases.


y

y It also causes – SIDS, intussusception, necrotizing enterocolitis


y They usually affect the central nervous system
y

and DM.
y

Pathogenesis and Clinical Findings Characteristics


y Long incubation period
y Rotaviruses infect the cells in the villi of small intestine
y

y Course of illness last for months to years


y

y Destruction of enterocytes leading to loss of villi and diarrhea


y

y Involves CNS
y

y Secretion of Enterotoxin – NSP4 (a viral encoded protein) –


y

y Absence of immune response


y

leads to triggering of signal transduction pathway


y

y Genetic predisposition
y This leads to activation of enteric nervous system
y

y Invariable fatal termination


y

y Diarrhea caused by rotaviruses are due to impaired sodium


y

and glucose absorption


y It will take 3–8 weeks for restoration into normal villi.
y Classification
y Incubation period is 1–3 days
y

y Symptoms seen are watery diarrhea, fever, abdominal pain


y
Table 3:  Classification of slow viral diseases
and vomiting.
Group A Lenti viruses - HIV
Group A rota viruses – most common cause of diarrhea in
Group B Prions
children
Group B viruses - causes diarrhea in adults Group C Subacute sclerosing polioencephalitis (SSPE
Progressive multifocal leucoencephalopathy (PML)

Laboratory Diagnosis
Group A
y Direct detection of viral particles in stool by electron
y Two animal viruses namely Visna and Maedi viruses are the
y

microscopy
y

agents that causes slow infections


y IgM and IgG ELISA (Most common method employed)
y Lentiviruses like HIV usually take up to 10 year’s time for
y

y Most sensitive method – RT PCR


y

developing disease complex named AIDS


y

281
Group B-Prions y Three distint strains:
Unit 3     Virology

y Prions are proteinaceous infectious materials


Zaire CFR 90%
y

y They do not have DNA/RNA


y

y The normal prion protein is PrPc gets converted to abnormal


y
Sudan CFR 50%
prion protein - PrPsc and causes diseases Reston
y Prions are highly resistant to routine sterilization methods
y

y It can be disinfected only by:


y
y Recent outbreak – West Africa – 2013 to 2016
y

 90% phenol

y Vaccine: rVSV – ZEBOV
y

 2 N NaOH


 10% Sodium dodecyl sulfate




 Autoclaving at 121°C for 1 hour




Prion Diseases
y Subacute spongiform viral encephalopathies (Sequel to
y

measles virus)
y Creutzfeldt Jakob disease
y

y Kuru
y

y SSPE
y

y Progressive multifocal leukoencephalopathy


y

Basic Features for Prion Diseases Are


y Neurodegeneration and spongiform changes
y Figure 3: Filamentous Ebola virus
y Amyloid plaques are seen
y
(Courtesy: CDC/ Frederick A. Murphy)
y As the agents are not considered antigenic – the host do not
y

mount immune or inflammatory response CORONA VIRUS


y Prions are excreted in urine.
y

y Enveloped RNA viruses


y

y Two groups:
Human Spongiform Encephalopathies
y

 Acid labile viruses – causes cold




y Two main encephalopathies associated with prions are


y
 Acid stable viruses – causes gastroenteritis


 Kuru

y Types of corona virus:
y

 Classic Cueutzfeldt- Jakob disease (CJD)



 229E


y Kuru is not seen nowadays. It is spreaded once by old custom


y
 OC43


among tribals by eating dead relatives (Cannibalism)  SARS – CoV




y CJD is developed because of transformation of normal prion


y
 NL63 – New Haven corona virus


to abnormal prion proteins  HKU 1




y CJD is manifested by progressive dementia, ataxia, myoclonus


y
 MERS – CoV – previously named as novel coronal virus 2012


and death is usually seen within 6 months to 1 year y Severe acute respiratory syndrome (SARS) – spreads by
y

y Two variants of CJD are:


y
inhalation of the virus
 Gerstmann-Straussler-Scheinker (GSS) syndrome

y Virus undergoes high degree of mutation, vaccine is impossible
y

 Fatal familial insomnia



y MERS-CoV – Middle east respiratory syndrome corona
y

y CJD can also be transmitted through organ transplantation.


y
virus – first detected in Saudi Arabia in 2012; It causes
Blood there is no proof for blood transfusion. severe respiratory illness; Epidemics occurs; No vaccine till
now; Severe cases can go for acute renal failure.

VIRAL HEMORRHAGIC VIRUS ONCOGENIC VIRUSES


Arenavirus Causes south American hemorrhagic fever Retrovirus
Lassa fever (transmitted through rodents) Human papilloma virus
Filoviruses Marburg and Ebola virus belongs to this group Polyoma virus
BK and JC virus
EBOLA VIRUS Molluscum contagiosum
Adenovirus
y Named Ebola virus: after Ebola river, where the first cases
EBV
y

were noticed
y Mode of infection: person to person transmission through
y
HSV 1 and 2
blood and body fluids CMV
282
y Causes fever with hemorrhagic illnes – highly fatal
y HBV and HCV
MULTIPLE CHOICE QUESTIONS

Chapter 36     Miscellaneous Viruses


1. Parvo virus B19 has predilection for:  14. True about prions: (AIIMS 2008)
 (Recent Pattern 2018) a. Virus coded
a. Erythroid progenitor cells b. Causes misfolding of proteins
b. Myeloid progenitor cells c. Cleaves proteins
c. Haematopoietic stem cells d. Defect in synthesis of proteins
d. Lymphoid progenitor cells 15. Rota virus is detected by: (AIIMS 2002)
2. Which HPV types causes Condyloma acuminata? a. Antigen in stool
 (AIIMS Nov 2016) b. Antibody in serum
a. 1 and 4 b. 6 and 11 c. Demonstration of virus
c. 16 and 18 d. 31 and 33 d. Stool culture
3. Human papilloma virus is associated with all of the 16. Vaccination causing intussuception: (PGI pattern 2017)
cancers except: (AIIMS May 2015) a. Rotavirus b. Parvovirus
a. Cancer base of tongue c. IPV d. BCG
b. Tonsillar carcinoma e. Measles
c. Nasopharyngeal cancer 17. Genetic Reassortment is typically seen in:
d. Recurrent respiratory papilloma  (AIIMS Nov 2010)
4. Oropharyngeal cancer is associated with: (AIPG 2014) a. Herpes virus b. Hepadna virus
a. EBV b. HPV c. Rotavirus d. Astro virus
c. HSV d. HBV 18. True about Human papilloma virus: (PGI May 2017)
5. Quadrivalent vaccine for HPV has all except: a. Belongs to family papovaviridae
 (PGI May 2013) b. DNA virus
a. Type 7 b. Type 11 c. RNA virus
c. Type 16 d. Type 18 d. Enveloped
e. Type 26 e. Causes anal warts
6. Slapped cheek appearance is seen in: (PGI May 2016) 19. All are true about Ebola virus infection except:
a. Rubella b. Rubeola  (PGI May 2017)
c. Parvo virus B19 d. HSV – 6 a. Air droplet is most common mode of transmission
7. Parvovirus infection is associated with:  b. Haemorrhagic manifestation may occur
 (PGI June 2008) (AIIMS May 2008) c. Thai forest type – most common species in epidemics
a. Hydrops fetalis b. Aplastic anemia d. Presents as sudden onset of fever and sore throat
c. Abortion d. Sixth disease e. Case fatality rate may be high as 70%
e. Hemophagocytic syndrome 20. True about Human papillomavirus (HPV):
8. All are true about SARS except: (Recent Pattern 2012)  (PGI Nov 2017)
a. Epidemic in India b. Spreads by droplet a. ssDNA containing virus
c. Diagnosed by PCR d. Caused by SARS CoV b. Icosahedral symmetry
9. SARS is a type of: (Recent Pattern 2014) c. HPV type 16 and 18 causes cervical cancer
a. Corona virus b. Lenti virus d. Recently nine valent vaccine passed phase 3 clinical
c. Calci viridae d. Hepadna viridae trial
10. Prions are: (AIPG 2004) e. 5 genera of HPV identified
a. Infectious proteins b. Made up of virus particles 21. True about Ebola virus: (PGI Nov 2017)
c. It is a nuclear material d. Can be cultured a. Flavivirus
11. Prions consists of: (AIIMS 2007) b. Icosahedral symmetry
a. DNA and RNA c. Predominantly found in Africa
b. DNA, RNA and proteins d. Spreads by direct contact with the blood or body fluids
c. RNA only e. Currently there is no specific treatment for this disease
d. Proteins only 22. Non immune hydrops is caused by:  (CET 2018)
12. Which of the following is not prion associated disease a. Parvovirus b. HPV
 (AIIMS 2003) c. Ehrlichia d. Rubella virus
a. Scrapie b. Kuru 23. Laryngeal papilloma is caused by  (CET 2018)
c. CJD d. Alzhiemers disease a. HPV 6 and 11 b. HPV 16 and 18
13. True about prion diseases are all, except: (AIIMS 2010) c. HPV 31 and 32 d. HPV 6 and 18
a. Myoclonus is seen in 10% of the patients
b. Caused by infectious protein
c. Brain biopsy is diagnostic
d. Manifests commonly as dementia
283
ANSWERS AND EXPLANATIONS
Unit 3     Virology

1. Ans.  (a) Erythroid progenitor cells • Parvo virus infection is associated with

ƒ Erythema infectiosum or fifth disease or slapped


Ref: Ananthanarayan and Paniker’s Textbook of Microbiol-
ƒ

cheek disease in children


ogy – 10th ed – Page 558
ƒ Arthralgia or Polyarthropathy in adults
ƒ

••Parvo virus B19 causes Erythema infectiosum ƒ Transient aplastic crisis


ƒ

••Most common route is respiratory ƒ Pure red cell aplasia


ƒ

••It infects the precursors of RBC’s ƒ Hydrops fetalis


ƒ

••In children it causes rashes on the cheeks giving an ƒ Papular purpura


ƒ

appearance of slapped cheek – hence called as slapped


cheek disease ; other wise called as fifth disease 8. Ans.  (a) Epidemic in India
• It also causes transient aplastic crisis
Ref: Ananthanarayan and Paniker’s Textbook of Microbiol-

2. Ans.  (b) 6 and 11 ogy – 10th ed – Page


• First time the outbreak of SARS happened in China in
Ref: Ananthanarayan and Paniker’s Textbook of Microbiol-

2002
ogy – 10th ed – Page
• India escaped at that time

• HPV types 6, 11, 40, 42 have very low oncogenic potential


• They are the causes for benign conditions


• 9. Ans.  (a) Corona virus
• Eg: Anogenital condylomas, larngeal papilloma,
Ref: Ananthanarayan and Paniker’s Textbook of Microbiol-

dysplasia and intraepithelial neoplasia


ogy – 10th ed – Page
3. Ans.  (c) Nasopharyngeal cancer • SARS is a type of corona virus

Ref: Ananthanarayan and Paniker’s Textbook of Microbiol- 10. Ans.  (a) Infectious proteins
ogy – 10th ed – Page
Ref: Jawetz medical microbiology – 27th ed – page 615
• Nasopharygeal carcinoma is caused by EBV

• Rest all others are associated with HPV



• Prions are infectious protein particles that does not have

nucleic acid
4. Ans.  (b) HPV • Prions are highly resistant

• They have abnormal folding proteins – causes diseases


Ref: Harrisons T.B of medicine 18th ed, chapter 185

• Most of the head and neck cancers are caused by HPV.


• 11. Ans.  (d) Proteins only
The most common head and neck cancer is oral cancer.
Ref: Jawetz medical microbiology – 27th ed – page 615
5. Ans.  (a) Type 7, (e) Type 26 Ref: Q.10
Ref: Ananthanarayan and Paniker’s Textbook of Microbiol-
12. Ans.  (d) Alzhiemers disease
ogy – 10th ed – Page
• Bivalent vaccine – HPV 16, 18

Ref: Jawetz medical microbiology – 27th ed – page 615
• Quadrivalent vaccine - Gardesil – HPV 6, 11, 16, 18

• Prion causes the following diseases:

• Nonavalent vaccine – HPV 6, 11, 16, 18, 31, 33, 45, 52



ƒ Kuru
ƒ

ƒ CJD
ƒ

6. Ans.  (c) Parvo virus B19 ƒ GSS


ƒ

Ref: Ananthanarayan and Paniker’s Textbook of Microbiol- ƒ Fatal familial insomnia


ƒ

ogy – 10th ed – Page ƒ Scrapie


ƒ

ƒ Mad cow disease


• In children, parvo virus causes rashes on the cheeks
ƒ

ƒ Spongiform encephalopathy

giving an appearance of slapped cheek – hence called as


ƒ

slapped cheek disease 13. Ans.  (a) Myoclonus is seen in 10% of the patients
• It is other wise called as fifth disease

Ref: Harrison T.B of internal medicine – 18th ed – Page 3554


7. Ans.  (a) Hydrops fetalis, (b) Aplastic anaemia, • Most of the patients of CJD has myoclonus that occurs

(c) Abortion (e) Hemophagocytic syndrome on and off throughout the illness
Ref: Ananthanarayan and Paniker’s Textbook of Microbiol- • Most common human prion disease is sporadic CJD

ogy – 10th ed – Page

284
• Mode of infection: person to person transmission

Chapter 36     Miscellaneous Viruses


14. Ans.  (b) Causes misfolding of proteins •

through blood and body fluids


Ref: Harrison T.B of internal medicine – 18th ed – Page 3443 • Causes fever with hemorrhagic illnes – highly fatal.

• Prion is a infectious protein particle


• The normal prion protein is PrPc gets converted to



20. Ans.  (a) ssDNA containing virus; (b) Icosahedral
abnormal prion protein - PrPsc and causes diseases symmetry; (c) HPV type 16 and 18 causes cervical
cancer; (d) Recently nine valent vaccine passed phase
15. Ans.  (a) Antigen in stool 3 clinical trial

Ref: Harrison T.B of internal medicine – 18th ed – Page 1592 Ref: Jawetz medical microbiology - 27th ed - page 631
• Diagnosis of rota virus is by:

• • HPV is a ssDNA virus with icosahedral symmetry
ƒ Genotyping of viral nucleic acid – the most sensitive
ƒ
• • HPV 6,11 are benign
method • • HPV 16,18 causes malignant illness - cervical cancer
ƒ Direct visualisation of fecal particles in electron
ƒ
• • Nonavalent vaccine is on use now
microscopy • • there are totally 16 genera and 82 types of HPV based on
ƒ Serological methods
ƒ
genetic homology
ƒ They do not grow in cell culture except few strains of
ƒ

group A 21. Ans:   (c) Predominantly found in Africa; (d) Spreads


by direct contact with the blood or body fluids;
16. Ans.  (a) Rotavirus (e) Currently there is no specific treatment for this
disease
Ref: Harrison T.B of internal medicine – 18th ed – Page 1592
• First rotavirus vaccine was introduced in the year 1998 Ref: Ananthanarayan and Paniker T.B of microbiology -
10th ed - page 562

• It was withdrawn then because of association with


intussuception • Ebola virus belongs to filoviridae; They are predom-


• Then recently multivalent bovine human reassortment



inantly seen in Africa and spreads person to person
rotavirus vaccines were prepared through blood and body fluids; Currently there is no
specific management
17. Ans.  (c) Rotavirus • Vaccines are under trail

Ref: Harrison T.B of internal medicine – 18th ed – Page 1591 22. Ans.  (a) Parvo virus
• Genetic reassortment is characteristic of rota virus and
Ref: Ananthanarayan and Paniker’s Textbook of Micro­

influenza virus
biology – 10th ed – Page 558
18. Ans.  (a) Belongs to family papovaviridae; (b) DNA • Parvovirus causes:

virus; (e) Causes anal warts ƒ Slapped cheek disease or fifth disease in children
ƒ

• Already explained Q.2



ƒ Non immune fetal hydrops in pregnancy
ƒ

ƒ Belongs to Papovaviridae
ƒ
ƒ Aplastic anaemia in adults
ƒ

ƒ They are Small, nonenveloped, DNA tumor viruses


23. Ans.  (a) HPV 6 and 11
ƒ

19. Ans.  (a) Air droplet is most common mode of transmis- Ref: Jawetz medical microbiology - 27th ed - page 631
sion; (c) Thai forest type – most common species in ep-
idemics • HPV 6 and 11 causes benign lesions – papillomas

• HPV 16 and 18 causes malignant tumours


Ref: Ananthanarayan and Paniker T.B of microbiology -


10th ed - page 562
• It is named Ebola virus: after Ebola river, where the first

cases were noticed

285
HIGH YIELDING FACTS TO BE REMEMBERED IN VIROLOGY
Unit 3     Virology

Both intracytoplasmic and intranuclear inclusion bodies are Measles – Warthin-Finkeldey cells
seen in
DNA viruses Hepadna, Herpes, Adeno, Parvo, Papova, Pox
Segmented nucleic acid is seen in Bunya viridae, Orthomyxoviridae, Reoviridae, Arenaviridae
Smallest and largest DNA virus Parvovirus and Pox virus
Smallest and largest RNA virus Picornavirus and Paramyxovirus
Most commonly employed method for virus isolation Cell culture
Viruses causing hemagglutination Influenza, Measles, Rubella, Coxsackie, Rhinovirus, Rabies,
Parainfluenza, Toga, Entero, Echo, Reo
Microscopy of herpes lesions is by Tzanck smear – Giant multinucleated cells
What is Shingles? Reactivation of latent infections from the dorsal root ganglion – VZV –
Herpes zoster
Which is the reservoir of EBV Memory B cells
Malignancies associated with EBV Burkitt lymphoma, Hodgkin’s disease, T cell lymphoma, Nasopharyngeal
carcinoma
Most common viral organism causing intrauterine infection is Cytomegalovirus
Space vehicle shaped virus Adenovirus
Epidemic keratoconjunctivitis is caused by Adenovirus (Shipyard eye)
Condyloma acuminate is caused by which of the HPV HPV 6, 11
serotypes
Erythema infectiosum is caused by Parvovirus – Slapped cheek disease or fifth disease
Outbreaks of vaccine-derived polio virus is due to OPV type 2
Hand foot mouth disease is caused by Coxsackie virus
Acute hemorrhagic conjunctivitis is caused by Enterovirus 70
Unique feature of Influenza virus Antigenic drift and antigenic shift
Hecht’s pneumonia is caused by Measles
Most common cause of bronchiolitis Respiratory Syncytial Virus
Major vector for the transmission of KFD – Kyasanur Forest Ticks
Disease
Earliest detection for Dengue virus illness is NS1 antigen detection
Only live vaccine that can be given during pregnancy when Yellow fever vaccine
needed
Inclusion bodies seen in rabies and its location Negri bodies; most abundant in cerebellum and hippocampus
Vaccination schedule for rabies Day 0, 3, 7, 14, 28
Which group is called as adult diarrhea rotavirus? Group B virus
The most common cause of diarrhea in infant and children Rotavirus
Strains used in Rota teq and Rotarix G1, G2, G3, G4, P(8); G1 P (8)
Genetic reassortment is seen in Rota virus
The only DNA hepatitis virus is Hepatitis B virus
Enterically transmitted non A and non B hepatitis virus is Hepatitis E virus
Three types of particles in HBV Dane particles – complete hep B, Spherical particle which is the most
common, Filamentous or tubular particle
The first marker to appear in serum in HBV infection Hepatitis B surface antigen HBsAg
Qualitative and Quantitative marker for viral replication in HBeAg and HBV DNA
HBV
Most common cause of post-transfusion hepatitis and Hepatitis C virus
chronic hepatitis
286 Examples for defective viruses HDV and Adenoassociated viruses
4

PARASITOLOGY
Unit Outline
Chapter 37 Introduction to Parasitology
Chapter 38 Flagellates–I
Chapter 39 Hemoflagellates
Chapter 40 Leishmania
Chapter 41 Apicomplexa
Chapter 42 Toxoplasma, Ciliate Protozoa
Chapter 43 Coccidian Intestinal Parasites
Chapter 44 Helminthology Cestodes
Chapter 45 Trematodes
Chapter 46 Nematodes
Chapter 47 Filarial Nematodes
Introduction to 37
Parasitology
INTRODUCTION Intestinal Amoebae
y A parasite is an organism that lives on another organism for y Group of Amoeba that attacks the intestine with unique

y
y
nutrition morphological features of naked cytoplasm and lobose
y Types of parasites: pseudopodium comes under intestinal Amoeba
y Based on the pathogenicity: Amoebae are classified into
y
 Ectoparasites: The Parasites that live on the outer surface

y

of the host E.g. Lice—this condition is called as infestation. pathogenic and nonpathogenic Amoebae
 Endoparasites: Parasites that live within the host,–e.g.

Plasmodium— this condition is called as infection.
High Yield
Table 1:  Categories of parasites • How to differentiate pathogenic and nonpathogenic Amoeba:

ƒ By zymodeme pattern
Obligate parasites These parasites cannot exist without host ƒ
ƒ Zymodeme: It is a group of Amoeba strains that share the
e.g. Toxoplasma
ƒ
same electrophoretic pattern and mobility for different
Facultative parasites These parasites can live freely also enzymes
e.g. Naegleria ƒ Enzymes used are:
ƒ
Accidental parasites They attack human as unusual host, e.g. – L–malate
Echinococcus – NADP+ oxidoreductase
– Glucose phosphate isomerase
Aberrant parasites Some parasites when entering the host
– Hexokinase
(called as paratenic host)—enters a
– Phosphoglucomutase
site where they cannot live or develop
ƒ Based on these–Amoeba which come under seven
further—called as aberrant, e.g. Toxocara
ƒ
populations namely 2, 6, 7, 11, 12, 13, 14 are pathogenic
causing larva migrans
(total zymodemes = 24)
y Organisms that gives shelter and nutrition to parasite are
y
called as hosts
y Types of hosts: Pathogenic Amoeba Entamoeba histolytica
y
 Definitive host: Host that harbors adult parasite; Host Nonpathogenic E. dispar, E.coli, E.hartmanni,

where sexual reproduction occurs Amoeba E. moshkovskii, E. gingivalis, E. polecki
 Intermediate host: Host that has the asexual forms of

parasites
y Parasites are broadly classified as: ENTAMOEBA HISTOLYTICA
y
 Protozoa
y Lives in large intestine (Caecum) of man

 Helminths
y
y Three morphological forms:

y
 Trophozoite
PROTOZOOLOGY

 Precyst

y All protozoa are unicellular eukaryotes  Cyst
y

y Single cell performs all the functions y Definitive host: Human beings
y
y
y Most of the protozoa are < 50 μm size except Balantidium y Infective form: Mature quadrinucleate cyst
y
y
coli which is >100 μm y Source of infection: Contaminated food by ingestion (fecal
y
y All the protozoan nucleus are vesicular except B. coli oral route), anal sexual transmission, through vectors like
y
y Stages of protozoa are trophozoites and cyst. cockroaches and flies.
y
Unit 4     Parasitology

Figure 1:  E. histolytica


(Courtesy: CDC)

Figure 2: Cyst of Entamoeba histolytica


(Courtesy: CDC/ Dr LLA Moore, Jr.)

Pathogenesis
y Both trophozoites and cysts can be seen in the intestinal
y

lumen but only trophozoites can invade the tissue


y Trophozoites attach to the colonic mucous and epithelial
y

cells by their Gal/GalNAc lectin and then invades the mucous


membrane of large intestine–erodes and forms flask shaped
ulcers
y These ulcers when treated gets healed; if left untreated can go
y

for necrosis and perforation


y Rarely, intestinal infection can produce a mass lesion named
y

as ameboma in the bowel lumen which causes obstructions.


y From there, it spreads to extraintestinal sites–liver–lungs–
y

brain and all other sites


y Leads to Amoebic liver abscess (ALA), pulmonary amoebia-
y

sis, cerebral amoebiasis.


Figure 3: Life cycle of Entamoeba histolytica

290
Clinical Features y Medium used for isolation of E. histolytica are:

Chapter 37     Introduction to Parasitology


y

 Boeck and Drbohlav medium


y Intestinal Amoebaisis­— the most common type is asympto-


 Balamuth’s medium
y

matic cyst passage.




 Diamond’s medium
y Symptomatic amoebic colitis can present as lower abdominal


 Philip’s medium
y

pain, mild diarrhea, malaise, weight loss.




 Jones medium
y Fecal material is full blown with bad odor; no blood or scanty


blood is seen. Invasive form Trophozoite


y Sometimes patients may go for toxic megacolon leading to
y

bowel dilation. Infective form Cyst

Complications
Table 2:  Identification of the morphological forms
y The most common form of extra intestinal amoebiasis is
y

Amoebic liver abscess (ALA); patients have characteristic Trophozoite Cyst


fever with right upper quadrant pain. Some has pleural
• Invasive and growing stage • Infective form
effusion. In case of endemicity and PUO are seen, Amoebic
• •

• It is motile with the help of • Immature cyst has glycogen


liver abscess must be ruled out.
• •

pseudopodium mass and chromatoid bodies


y The most common complication of ALA is pulmonary
• Nucleus is eccentric in • Mature cyst is quadrinucleate
y

amoebiasis (Pleuropulmonary involvement). Manifesta-


• •

position
tions seen are sterile effusions, or spread from liver due to
• Karyosome is central in
rupture.

position
y Genital and cerebral involvement also can occur in extreme
• Nuclear membrane has fine
y

cases.

chromatin granules
• It has ingested RBCs (called

as erythrophagocytosis)

Serological Test
y Mainly used to diagnose invasive amoebiasis–extra intestinal
y

y Indirect hemagglutination test


y

y ELISA–most sensitive and specific test: Detects stool antigen


y

(Copro antigen)-antigen positive means recent infection


y Serum showing antibody titer of 1:128 is diagnostic of ALA
y

y But antibody detection is not helpful to diagnose recent or


y

past infection

Additional Lab Features


Figure 4:  HPE in intestinal amoebiasis showing many trophozoites y Helps to differentiate from bacillary dysentery
(Courtesy: CDC/Dr Mae Melvin)
y

y Stool microscopy: Few pus cells, RBCs agglutinated, Charcot


y

Leyden crystals are present


Diagnosis by Wet Mount y Blood smear: No/mild leucocytosis
y

y Saline preparation: Motile trophozoites seen


y
y WHO recommendation for diagnosis of intestinal amoebiasis–
y

y Iodine preparation: Trophozoites (motility can not be seen)


y
specific tests needs to be done; Hence stool microscopy is
and cysts seen not confirmative.

Culture of E. Histolytica Treatment


y Polyxenic culture: Means culture is done in the presence of
y
y Amoebic dysentery: Metronidazole, emetine hydrochloride
y

a bacteria—Boeck and Drbohlav egg serum medium (first y ALA: Metronidazole, Dihydroemetine, chloroquine
y

medium used for culture of E. histolytica) y Asymptomatic cyst passers: Metronidazole, diloxanide furo-
y

y Axenic culture–bacteria free culture: Its uses are


y
ate, tetracycline
 Pathogenicity of Amoeba can not be studied

y Metronidazole is the DOC for ameobic liver abscess.
y

 In vitro anti Amoeba drug susceptibility




 Antigen preparation


291
Nonpathogenic Amoebae
Unit 4     Parasitology

Remember
Table 3: Characteristics of non-pathogenic amoebae

• Entamoeba histolytica vs Entamoeba dispar Vs Entamoeba


moshkovskii:
Organisms Characteristics • In stool examination–both looks similar morphologically

Entamoeba • Largest Amoebae—commensal in large


• • Hence always the report should be given as Trophoites

coli intestine or Cyst suggestive of Entamoeba histolytica/Entamoeba


• No invasion

dispar/Entamoeba moshkovskii
• Never contain ingested RBCs

• Differentiation is done based on:

• Until recently E. coli was considered



ƒ Clinical features
ƒ

nonpathogenic–but few cases of diarrhea in ƒ Zymodeme pattern


ƒ

children has been described now. ƒ Agglutination with concanavalin A (positive in


ƒ

E. histolytica)
Entamoeba • Asymptomatic carriers

dispar • No invasion
Free Living Amoebae

• Morphologically trophozoites and cyst are


similar to E. histolytica y Pathogenic free living Amoebae are seen in soil and water–it
y

Entamoeba • Morphologically trophozoites and cyst are



does not have human carrier state; no vector
moshkovskii similar to E.histolytica y These are neuropathogenic
y

y Four species:
Entamoeba • It is called as small E. histolytica
y

 Naegleria fowleri

hartmanni • But it is a misnomer–it does not look




 Acanthamoeba

morphologically like 

 Balamuthia mandrillaris
E. histolytica 

 Sappinia
Entamoeba • First Amoeba described in humans


gingivalis •• It has only trophozoite stage Table 4: Characteristics of free-living amoebae


• No ingested RBCs
Free living Characteristics

• Commensal that is present in the margins


Amoebae

of gum
Entamoeba • Amoeba that infects pigs and rarely humans Naegleria • Infection is acquired while swimming in

fowleri contaminated water


polecki
• Amoeboid trophozoite enters through nasal
Endolimax • Trophozoite: Nucleus contains large irregular

mucosa—goes to brain
nana karyosome–eccentric in position • It causes primary amoebic

• Spherical cyst–quadrinucleate; chromatoid



meningoencephalitis
bodies and glycogen masses are absent • Very fatal—patients die within a week

Iodamoeba • Most common Amoeba of pigs • Wet mounts of CSF shows motile amoeboid

trophozoites

butschlii • Trophozoites: Large, irregular, round


• This free living Amoebae can be cultures on

karyosome–central in position–surrounded

a NNA (non-nutrient agar plate) with a lawn


by refractile globules
culture of
• Cyst: Large vacuole that has closely packed

E. coli done already (which serves as a


glycogen mass source of food for Amoebae)
• Treatment: Amphotericin B

Acanthamoeba • Infection is acquired through skin abrasions


or through cornea lens (Contact lens);


inhalation
• Most common species is

A. culbertsoni
• It causes Granulomatous amoebic

encephalitis and keratitis


• In GAE–wet mount of CSF shows–

trophozoites
• In keratitis–Corneal scrapings shows–cysts

• Fatal condition–no appropriate treatment


Balamuthia • Infection is transmitted through respiratory


mandrillaris tract or skin lesions


• It causes GAE

• Presence of multiple nucleoli in


trophozoites helps in diagnosis


• Not reported in India yet

292 Sappinia • Only one case of Sappinia causing amoebic


Figure 5:  Illustration showing Entamoeba coli encephalitis has been reported


(Courtesy: CDC)
Chapter 37     Introduction to Parasitology
MULTIPLE CHOICE QUESTIONS

Entamoeba 13. A 23 year old male presented with abdominal pain and
bloody diarrhea of one week duration. The following
1. All the following amoeba live in the large intestine
colonic biopsy is diagnostic of infection with
except:
a. E. coli b. E. nana
c. E. gingivalis d. I.butschlii
2. Culture media used for entamoeba histolytica
a. Blood agar b. Philip’s medium
c. CLED medium d. Trypticase serum
3. Non pathogenic amoeba is/are
a. Entamoeba histolytica
b. E. coli
c. Acanthamoeba
d. E. Hartmani
e. Balamuthia
4. Amoebiasis is transmitted by all except
a. Cockroach b. Faeco oral
a. Giardiasis b. Amoebiasis
c. Vertical transmission d. Oro-rectal
c. Enterobius d. Severe bacterial infection
5. The pathogenicity of Entamoeba histolytica is indicated
by
a. Zymodeme pattern b. Size Free Living Amoeba
c. Nuclear pattern d. ELISA test 14. Acute Primary Amoebic meningoencephalitis true is
6. Which is true of trophozoites of E. histolytica (Recent Pattern 2017)
(PGI 2017) a. Meningitis caused by acanthamoeba species is acute in
a. Has eccentric karyosome nature
b. Nuclear membrane with chromatin b. Diagnosis is by demonstration of trophozoite in CSF
c. Shows erythrophagocytosis c. Caused by feco-oral transmission
d. Presence of bacteria inside cell d. More common in tropical climate
7. Amoebiasis is not transmitted by 15. A 15 year old girl residing in a village recently returned
a. Feco-oral route from a vacation visiting her friends in another village.
b. Sexual transmission she complained of severe headache and fever, was diag-
c. Blood and blood products nosed as a case of pyogenic meningitis and admitted to
d. Vector transmission the hospital. She died 5 days later. Which of the follow-
8. The main reservoir for Entamoeba histolytica is ing organism should be considered in the diagnosis?
a. Man b. Dirty water (AIIMS 2017)
c. Soil d. Ponds a. Entamoeba histolytica
9. Charcot-Leyden crystals are seen in: b. Naegleria fowleri
a. Bacillary dysentery c. Toxoplasma gondii
b. Amoebic dysentery d. Falciparum malaria
c. Giardiasis 16. A patient presents with headache, high fever and
d. Cholera meningismus. Within 3 days he become unconscious.
10. Stain with parasite having Charcot-Leyden crystals but Most probable causative agent (Recent Pattern 2018)
no pus cells a. Naegleria fowleri
a. Giardia b. Taenia b. Acanthamoeba castellani
c. E.histolytica d. Trichomonas c. Entamoeba histolytica
11. Most common extrahepatic complication of amoebic d. Trypanosoma cruzi
hepatitis is 17. A 30 year old patient presented with features of acute
a. Meningitis meningoencephalitis in the casualty. His CSF on
b. Lung abscess wet mount microscopy revealed motile unicellular
c. Nephritis microorganism. The most likely organism is
d. Encephalitis (AIIMS 2017)
12. Regarding amoebic liver abscess all are true except a. Naegleria fowleri
a. Trophozoite in stool are essential for clinical diagnosis b. Acanthamoeba catellini
b. Mostly asymptomatic c. Entamoeba histolytica
c. More common in male than female d. Trypanosoma cruzi
293
d. It rarely affects brain eye and skin
18. Which of the following amoebae does not have 20. Which of the following is true regarding Naegleria
Unit 4     Parasitology

neuropathogenic effect (Recent Pattern 2017) fowleri  (Recent Pattern Nov 2018)
a. Naegleria a. Fresh water living 
b. Acanthamoeba b. Hot springs
c. Dientamoeba c. Salt water living 
d. Balamuthia d. Desert living
19. Cerebral amoebiasis is not caused by
(Recent Pattern 2017)
a. Nagleria b. Acanthamoeba
c. Dientamoeba d. Balamuthia

ANSWERS AND EXPLANATIONS


1. Ans.  (c) E.gingivalis • Zymodeme–it is a group of amoeba strains that share the

same electrophoretic pattern and mobility for different


Ref: T.B of Medical Parasitology–S.C.Parija–4th edition–
enzymes
Page 41
• Entamoeba gingivalis is an amoeba of human mouth –
• 6. Ans.  (b) Nuclear membrane with chromatin;
gums (c) Shows erythrophagocytosis
• It has only trophozoite stage
Ref: T.B of Medical Parasitology–S.C.Parija–4th edition–

• Rest all three are intestinal amoebae


Page 24

2. Ans.  (b) Philip’s medium • Trophozoites is the invasive form of the parasite

• It has peripheral chromatin and central karyosome


Ref: T.B of Parasitology–Rajesh Karyakarte–Page 221

• The cytoplasm has a clear ectoplasm and granular


Culture media used for cultivation of E. histolytica: endoplasm


• Boeck and Drbohlav medium

• Characteristic feature is presence of ingested RBCs

• Balamuth’s medium

• Diamond’s medium
• 7. Ans.  (c) Blood and blood products
• Philip’s medium
Ref: T.B of Medical Parasitology–S.C.Parija–4th edition–

• Jones medium
Page 33

3. Ans.  (b) E. coli ; (d) E. Hartmani • Already explained Q.4


Ref: T.B of Medical Parasitology–S.C.Parija–4th edition– 8. Ans.  (a) Man


Page 40
Ref: T.B of Medical Parasitology–S.C.Parija–4th edition–
• Non pathogenic amoebae:
Page 33

ƒ E. coli
ƒ

ƒ E. hartmanni
ƒ
• Reservoir of infection is infected persons especially cyst

ƒ E. polecki
ƒ
carriers
ƒ Endolimax nana
ƒ
• There are no animal reservoirs

ƒ Iodamoeba botschlii
9. Ans.  (b) Amoebic dysentery
ƒ

ƒ Dientamoeba fragilis
ƒ

Ref: T.B of Medical Parasitology–S.C.Parija–4th edition–


4. Ans.  (c) Vertical transmission Page 37
Ref: T.B of Medical Parasitology–S.C.Parija–4th edition– • Stool microscopy in amoebiasis-Few pus cells, RBCs

Page 33 agglutinated, Charcot Leyden crystals are present


• Amoebiasis is transmitted by:
10. Ans.  (c) E. histolytica

ƒ Feco oral route


ƒ

ƒ Homosexual – anal contact


ƒ
Ref: T.B of Medical Parasitology–S.C.Parija–4th edition–
ƒ Flies
ƒ

Page 37
ƒ Cockroaches
• Already explained Q.9
ƒ

5. Ans.  (a) Zymodeme pattern


11. Ans.  (b) Lung abscess
Ref: T.B of Medical Parasitology–S.C.Parija–4th edition–
Ref: Harrison’s T.B of Internal Medicine–19th edition–
Page 24
Page 1365
• Pathogenic and nonpathogenic amoeba are differenti-
• The most common form of extra intestinal amoebiasis is

294 ated by Zymodeme pattern


Amoebic liver abscess


• The most common complication of ALA is pulmonary 17. Ans.  (a) Nagleria fowleri

Chapter 37     Introduction to Parasitology


amoebiasis (Pleuropulmonary involvement)


Ref: T.B of Medical Parasitology–S.C.Parija–4th edition–
12. Ans.  (a) Trophozoite in stool are essential for clinical Page 44; Harrison T.B of Internal Medicine–19th edition–
diagnosis Page 1367
• Patients is diagnosed with acute meningoencephalitis
Ref: Harrison’s T.B of Internal Medicine–19th edition–

with CSF shows unicellular motile organism–hence it is


Page 1366
amoeba–trophozoite stage
• Most of the amoebic liver abscess cases have no

• This confirms the diagnosis of Naegleria fowleri

symptoms and signs of colitis


• When Trophozoites in the stool are searched also – most
• 18. Ans.  (c) Dientamoeba
cases are not available
Ref: T.B of Medical Parasitology–S.C.Parija–4th edition–
13. Ans.  (b) Amoebiasis Page 43
• The image shows flask shaped ulcer seen in intestinal

• Free living amoeba which are neuropathogenic are:

amoebiasis ƒ Naegleria fowlerii


ƒ

• It clearly indicates amoebiasis



ƒ Acanthamoeba
ƒ

ƒ Balamuthia mandrillaris
ƒ

14. Ans.  (b) Diagnosis is by demonstration of trophozoite ƒ Sappinia


ƒ

in CSF
19. Ans.  (c) Dientamoeba
Ref: T.B of Medical Parasitology–S.C.Parija–4th edition–
Page 44 Ref: T.B of Medical Parasitology–S.C.Parija–4th edition–
Page 43
• Acute primary amoebic meningoencephalitis is caused
• Already explained Q. 5

by Naegleria fowleri •

• Infection is acquired through contaminated water from


20. Ans.  (a) Fresh water living 

pools and lakes during swimming


• The trophozoites enter via the nasal mucosa

Ref: Medical parastiology – Parija – 4th edition – Page 43
• Diagnosis is by wet mount of CSF – showing trophozoites

• Naegleria fowleri are free living amoebae


• Found in freshwater or brackish water like lakes, river or


15. Ans.  (b) Nagleria fowleri

ponds and in the soil


Ref: T.B of medical parasitology–S.C.Parija–4th edition– • They are sensitive to extremes of pH and certain envi-

Page 44 ronments like aridity


• Most fatal condition among all neuropathogenic free

• They cannot survive in sea water

living amoeba infections are Naegleria fowleri–in spite


of treatment–mortality rate is higher

16. Ans.  (a) Nagleria fowleri


Ref: T.B of medical parasitology–S.C.Parija–4th edition–
Page 44
• Already explained Q.2

295
38
Flagellates–I
Table 1:  Flagellates
Intestinal flagellate Giardia lamblia
Chilomastix mesnili
Retortamonas intestinalis
Enteromonas hominis
Trichomonas hominis
Dientamoeba fragilis
Oral flagellate Trichomonas tenax
Genital flagellate Trichomonas vaginalis
Hemoflagellates Leishmania spp and Trypanosoma sp

GENERAL FEATURES
y All flagellates except Dientamoeba fragilis (has no flagella)
y
possess flagella Figure 1:  Figure showing trophozoite of Giardia
y Dientamoeba and Trichomonas vaginalis–do not have (Courtesy: CDC/ Dr Mae Melvin)
y
cystic stage
y All flagellates have both trophozoite and cyst stage except
y
above two

GIARDIA LAMBLIA
y This flagellate lives in duodenum and upper jejunum (Small
y
intestine)
y Trophozoite is tennis racket shaped and actively motile; it
y
has a central axostyle
y Cyst stage is the infective stage
y
y Definitive host: Human beings
y
y No intermediate host
y
y Infective form: Mature cyst
y
y Source of infection:
y
 Contaminated food/water by ingestion

 Person to person transmission

 Homosexual-transmission Figure 2: Figure showing cysts of Giardia in stool sample

y Giardia cysts are highly resistant to standard concentra- (Courtesy: CDC)
y
tions of chlorine y String test or Entero test: A gelatin capsule with a nylon
y With the help of sucking disc: Trophozoites infect the duodenal
y
string attached is inserted up to duodenum and jejunum and
y
epithelium–causes interference in absorption of fats and
remained there for 4–6 hours–after removal of the capsule–it
vitamins leads to malabsorption
is examined for trophozoites by microscopy
y The parasite causes Giardiasis: Diarrhea, abdominal pain,
y ELISA: Done on stool samples to detect the antigen (Copro
y
steatorrhea, malabsorption, flatulence
y
y G.intestinalis or G.lamblia: One of the first intestinal antigen)
y Most sensitive method of diagnosis is Duodenal sampling by
y
pathogens to infect infants
y
endoscopy–followed by staining with Giemsa or Trichrome
Diagnosis y Treatment: Metronidazole or Tinidazole
y
y Stool examination:
TRICHOMONAS VAGINALIS
y
 Direct wet mount shows falling leaf motility of trophozo-

ites seen in liquid stools; y Flagellate having only trophozoites; no cystic stage
y
 In formed stools – cyst can be seen y It is noninvasive

y
y Definitive host–humans; Man is the only reservoir of infec- y Treatment: Metronidazole; It is very important to treat the

Chapter 38     Flagellates–I


y y

tion sexual partner with a complete course


y Lives in vagina, cervix, urethra and prostate (adheres to the
y

mucosa) Table 2:  Miscellanous Flagellates


y Route of transmission: Sexual and through fomites (sharing
y

towels, blades) Flagellate Characteristics


y Most common sexually transmitted disease in humans
y
Chilomastix mesnili • Flagellate located in cecum and colon

y In females: It causes trichomoniasis–Strawberry vagina


y
• Trophozoites and cyst can be

because of inflammation that leads to itching, greeny demonstrated in formed stool


discharge with an abnormal odor • Cytostome present

y In males–it causes–non gonococcal urethritis


y
• It is harmless commensal

• Cysts are lemon shaped


Diagnosis
y Saline wet mount of vaginal or urethral discharge shows-Pear
y

Retortamonas • Rare commensal located in intestines


shaped trophozoites (Infective form) with jerky movements– intestinalis • Also known as Embadomonas

twitching motility intestinalis


y Stain used in Giemsa stain and Acridine orange stain to look
y

for trophozoites Enteromonas hominis • Commensal lives in intestine


• Harmless

• Cytostome is absent

Trichomonas tenax • Present in oral cavity–in gums


• Nonpathogenic

Trichomonas hominis • Present in lower GIT–in cecum


• Nonpathogenic

Dientamoeba fragilis • Flagellate that does not have


flagellum
• Inhabits large intestine

• No cystic stage

• Transmitted by feco oral route;


Figure 3: Trophozoites of Trichomonas vaginalis

Enterobius vermicularis acts as a


(Courtesy: CDC) vector for this amoeba
y Gold standard method: Culture in Diamond’s medium–
y • Non invasive

most sensitive method • Acts as a chronic irritant–causes


y ELISA and DFA tests to identify the antigen


y dientamoebiasis–diarrhea,
y PCR
y abdominal pain and oral pruritus
y Vaginal pH estimation by pH paper and Whiff or amine odor
y

test helps in preliminary diagnosis–normal pH excludes the


diagnosis of trichomoniasis

297
MULTIPLE CHOICE QUESTIONS
Unit 4     Parasitology

Giardia Lamblia Trichomonas Vaginalis


1. True about Giardiasis (Recent Pattern 2017) 9. A pap smear was prepared from the vagina and it
a. Only cyst is infective showed the following findings. Identify the organism
b. Reside in cecum responsible. (Recent Pattern 2017)
c. only man to man transmission
d. Exist in one phase
2. How many pairs of flagella does Giardia lamblia possess
(Recent Pattern 2018)
a. One b. Two
c. Three d. Four
3. The normal habitat of giardia is
(Recent Pattern 2017)
a. Duodenum and jejunum
b. Stomach
c. Cecum
d. Ileum
4. A 4 year old child presents with acute watery diarrhea
and abdominal cramps. Stool microscopy reveals
trophozoites with falling leaf motility. The etiological
agent is ? (Recent Pattern 2018)
a. Entamoeba histolytica
b. Giardia lamblia
a. Trichomonas vaginalis
c. Trichomonas tenax
b. Neisseria gonorrhea
d. Balantidium coli
c. Chlamydia
5. A patient is presenting with abdominal pain, steatorrhea
d. Treponema pallidum
and normal LFT; Most likely infecting organism
10. The cystic form of all are seen in man except
(Recent Pattern 2017)
(Recent Pattern 2018)
a. Shigella b. E.histolytica
a. E.histolytica
c. Giardia d. Enterotoxigenic E.coli
b. Giardia
6. Giardiasis true is all except (Recent Pattern 2017)
c. Trichomonas
a. Diarrhea with steatosis
d. Toxoplasma
b. Bloody diarrhea
11. Gold standard method of diagnosis for trichomoniasis
c. Metronidazole is the drug of choice
(Recent Pattern 2017)
d. Absent fever
a. PCR
7. A case of giardiasis presents with
b. Microscopy
(Recent Pattern 2018)
c. Giemsa stain
a. Nausea and vomiting
d. Culture
b. Abdominal pain
12. All are true regarding trichomoniasis except:
c. Steatorrhea and flatulence
(Recent Pattern 2018)
d. All of the above
a. Man to man transmission occurs
8. Identify the following egg: (Recent Pattern Nov 2017)
b. Reservoir for infection is lice
c. Fomites act as source of infection sometimes
d. Sexual mode is the most common method of transmis-
sion
13. True regarding treatment of trichomoniasis:
(Recent Pattern 2017)
a. It is important to give full course of treatment for
partner
b. Metronidazole can be given in pregnancy to avoid fetal
transmission
c. Diloxanide is the drug of choice
d. No drug resistant strains have been isolated till now

a. Entamoeba dispar b. Giardia lamblia


298
c. Balantidium coli d. Entamoeba histolytica
ANSWERS AND EXPLANATIONS

Chapter 38     Flagellates–I


1. Ans.  (a) Only cyst is infective 7. Ans.  (d) All of the above
Ref: T.B of Medical Parasitology–S.C.Parija–4th edition– Ref: T.B of Medical Parasitology–S.C.Parija–4th edition–
Page 56 Page 59
• Intestinal flagellate that most commonly causes diarrhea
• • Already explained Q.5

is Giardia intestinalis or Giardia lamblia


• There are two morphological forms namely trophozoites

8. Ans.  (b) Giardia lamblia
and cyst Ref: T.B of Medical Parasitology – Parija – 4th ed – Page 173
• Mature cyst is the infective stage
Giardia lamblia:

• It lives in small intestine


• Lives in duodenum and upper jejunum

• It is transmitted through contaminated food, water and


• Trophozoite is tennis racket shaped and actively motile;


from man to man.


it has a central axostyle


• Cyst stage – infective form; oval shaped – thick cyst wall
2. Ans.  (d) Four •

surrounds
Ref: T.B of Medical Parasitology–S.C.Parija–4th edition–
Page 56 9. Ans.  (a) Trichomonas vaginalis
• Trophozoite of Giardia–tennis racket shaped or pear
• Ref: T.B of Medical Parasitology–S.C.Parija–4th edition–
shaped Page 69
• It has two axonemes, two median bodies and four pairs

• Pap smear shows some kite shaped trichomonas–it


of flagella is very difficult to see the flagella in pap smear and


• Four pairs–two pairs of lateral, one pair of ventral and

so the classical pear shaped is lost during pap smear
one pair of caudal flagella preparation; Surrounding reactive squamous cells are
• It has two nuclei

seen. This feature is suggestive of trichomoniasis

3. Ans.  (a) Duodenum and jejunum 10. Ans.  (c) Trichomonas


Ref: T.B of Medical Parasitology–S.C.Parija–4 th
edition– Ref: T.B of Medical Parasitology–S.C.Parija–4th edition–
Page 56 Page 65
• Giardia lives in the small intestine of humans–
• • Flagellates that do not have cystic stage is–Trichomonas

duodenum, jejunum and upper ileum and Dientamoeba

4. Ans.  (b) Giardia lamblia 11. Ans.  (d) Culture


Ref: T.B of Medical Parasitology–S.C.Parija–4th edition– Ref: T.B of Medical Parasitology–S.C.Parija–4th edition–
Page 56 Page 69
• Already explained Q.1

• Culture is the gold standard and most sensitive for

• Features of diarrhea in children with stool microscopy


• diagnosis of Trichomonas vaginalis
showing falling leaf motility is characteristic of Giardiasis
12. Ans.  (b) Reservoir for infection is lice
5. Ans.  (c) Giardia
Ref: T.B of medical parasitology–S.C.Parija–4th edition–
Ref: T.B of Medical Parasitology–S.C.Parija–4th edition– Page 68
Page 59 • There is no reservoir for infection for trichomonas other

• Symptoms suggest features of Giardiasis–foul smelling



than man
diarrhea, bloating, abdominal cramps, flatulence, • Man is the only definitive host where the trichomonas

steatorrhoea, malabsorption, weight loss harbours the genital area


• Normal LFT to rule out amoebiasis–that may spread
13. Ans.  (a) It is important to give full course of treatment

causing amoebic hepatitis


for partner
6. Ans.  (b) Bloody diarrhea Ref: T.B of Medical Parasitology–S.C.Parija–4th edition–
Ref: T.B of Medical Parasitology–S.C.Parija–4 th
edition– Page 70
Page 59 • Metronidazole is the DOC for trichomoniasis

• It is must to treat the partner


• Bloody diarrhea is a feature of invasive organisms like

• Drug is contraindicated in pregnancy because of


Entamoeba histolytica, Shigella


teratogenicity
• Giardiasis is not invasive–hence no blood is seen

• In some patients, recently drug resistant strains of trich-


• 299
omonas has been isolated
39
Hemoflagellates
Table 1: Important Hemoflagellates Table 2: Stages of Hemoflagellates


Leishmania • Leishmania donovani Amastigotes and Promastigotes Leishmania

• Leishmania tropica Amastigote Trypanosoma cruzi

• Leishmania mexicana Epimastigotes and Trypomastigotes Trypanosoma brucei and

• Leishmania braziliensis cruzi

Trypanosoma • Trypanosoma cruzi
TRYPANOSOMA

• Trypanosoma brucei gambiense

• Trypanosoma brucei rhodesiense y Flagellate that infects humans and are found in blood and

y
lymph nodes
y They are elongated in shape with a central nucleus and
FEATURES OF HEMOFLAGELLATES
y
kinetoplast–which contains DNA and mitochondria
y They all have a kinetoplast–which is a DNA carrying organelle y An undulating membrane originates from the kinetoplast
y
y
y They are pleomorphic–variety of stages are seen morphologi- y A single flagellum is seen at the anterior end–organism is
y
y
cally motile
y They live in blood and tissue Trypanosoma cruzi Chagas disease (American trypanosomiasis)
y
y Presence of undulating membrane
Trypanosoma Sleeping sickness (African trypanosomiasis)
y
brucei

Figure 1: Life cycle of Trypanosoma cruzi


(Courtesy: CDC/ Alexander J. da Silva, PhD, Melanie Moser)
TRYPANOSOMA CRUZI

Chapter 39     Hemoflagellates


y Zoonotic disease–seen in South America
y

y Not seen in India


y

y Three main developmental forms are:


y

 Amastigotes


 Trypomastigotes


 Epimastigotes


y Definitive host: Humans and other vertebrates


y

y Intermediate host: Reduviid bugs


y

Table 3:  Location of morphological forms


Amastigotes Reticulo endothelial system, muscles,
mononuclear phagocytes
Non multiplying Peripheral blood Figure 2: Amastigotes of Trypanosoma cruzi
trypomastigotes (Courtesy: CDC/Dr AJ Sulzer)
Multiplying Reduviid bug Treatment
trypomastigotes
y Nifurtimox
y

Epimastigotes Reduviid bug y Benznidazole


y

y Gentian violet
y Infective form to man is metacyclic trypomastigotes
y

y Apart from vector transmission, Trypanosoma is transmitted


TRYPANOSOMA BRUCEI
y

by:
 Blood transfusion


y Trypanosoma brucei has two subspecies: gambiense and


y

 Transplacental transfusion


rhodesiense
 Organ transplantation


y Seen in Africa; Not in India


y

 Laboratory accidental inoculation




y Morphological forms are trypomastigotes and epimastigotes


y

(No amastigotes)
Clinical Features Trypomastigote Humans and other vertebrates
y Chaga’s disease is most commonly seen in infants and chil-
y Epimastigote Tsetse fly
dren y Definitive host: Humans and other vertebrates
y At the bite area–a localized edema and erythema occurs
y

y Intermediate host: Tsetse fly (Glossina)


y

named as Chagoma
y

y Infective form: Metacyclic trypomastigote


y Followed by local lymphadenopathy
y

y In Acute Chaga’s disease–when the site of inoculation is in


y

Clinical Features
conjunctiva–it leads to unilateral painless edema of the eye
named as Romana’s sign Table 4:  Clinical Features of different subspecies of T.brucei
y Complications of acute disease are:
y

T.brucei gambiense T.brucei rhodesiense


 Myocarditis
• Causes West African • Causes East African


 Meningoencephalitis
• •

trypanosomiasis trypanosomiasis


y In Chronic Chaga’s disease–it causes: (Mega disease)


• Winterbottom sign is seen– • Less common
y

 Cardiomyopathy–heart block
• •

cervical lymphadenopathy lymphadenopathy


 Mega colon


 Megaoesophagus

y After the bite from the fly a chancre develops at the site–then
y

 Colopathy

leads to hemolymphatic spread followed by CNS spread
 Sudden death occurs due to ventricular fibrillation

y CNS stage leads to sleeping sickness
y

Table 5:  Differentiating features of West African and East


Diagnosis African trypanosomiasis
y Wet mount of anticoagulated blood–shows motile trypomas-
Features West African East African
y

tigotes
y Thick or thin blood smear preparation is done–to see the
y
trypanosomiasis trypanosomiasis
morphology clearly Causative agent T. brucei.gambiense T. brucei.rhodesiense
y Blood culture done in NNN medium
y Vector Tsetse flies Tsetse flies
y The only confirmatory method to diagnose chronic chagas
y

Reservoir Humans Antelope and cattle


disease is xenodiagnosis (Using bugs); Serological methods Type of illness Chronic CNS disease Acute CNS disease
also help to diagnose chronic chagas disease
Lymphadenopathy Prominent Minimal
y Animal inoculation–intraperitoneally into mice 301
Parasitemia Low High
y
Lab Diagnosis
Unit 4     Parasitology

y Lymph node specimens and peripheral smear show trypomastigotes


y

y Culture can be done in liquid media


y

y IHA, IFA and ELISA are most commonly used method for diagnosis
y

Figure 3: Trypomastigotes of T. brucei


(Courtesy: CDC/ Dr. Mae Melvin)

Treatment
Table 6:  Treatment of T. brucei infection
Organism Normal CSF Abnormal CSF
T. brucei gambiense First drug: Pentamidine Eflornithine
Alternative drug: (or)
Suramin Nifurtimox-Eflornithine combination
T. brucei rhodesiense First drug: Melarsoprol
Suramin
Alternative drug:
Pentamidine

New drug (November, 2018)


FDA has approved orally administered fexinidazole to replace the current treatment regimen for Trypanosoma brucei gambiense
infections causing sleeping sickness.

302
MULTIPLE CHOICE QUESTIONS

Chapter 39     Hemoflagellates


1. Protozoa associated with megaesophagus: 7. The following vector takes part in the life cycle of African
(Recent pattern 2017) trypanosomiasis. Identify it: (AIIMS 2017)
a. Trypanosome b. Amoeba a. Glossina b. Reduviid bug
c. Giardia d. Gnathostoma c. House flea d. Mite
2. Vector for T. cruzi is: (Recent pattern 2017) 8. Which form of the following flagellate is infective for
a. Reduviid bug b. Tsetse fly mammalian host? (AIIMS 2017)
c. Sand fly d. Hard tick
3. Amastigote form of which parasite is found in human:
(PGI pattern 2017)
a. Trypanosoma cruzi
b. Trypanosoma brucei
c. Trypanosoma gambiense
d. Trypanosoma rhodesinse
e. Trypanosoma rangelii
4. Winter bottom sign in sleeping sickness refers to:
(Recent pattern 2017)
a. Unilateral conjunctivitis
b. Posterior cervical lymphadenopathy
c. Narcolepsy
d. Transient erythema
5. Tsetse fly transmits: (Recent pattern 2017)
a. Trypanosoma brucei b. T.cruzi a.
Metacyclic trypomastigote
c. Kala-azar d. Oriental sore b.
Long slender form
6. Infective form of T. brucei: (Recent pattern 2017) c.
Short stumpy form
a. Amastigote b. Trypomastigote d.
Intermediate form
c. Egg d. none

ANSWERS AND EXPLANATIONS


1. Ans.  (a) Trypanosome 4. Ans.  (b) Posterior cervical lymphadenopathy
Ref: T.B of Medical Parasitology–S.C.Parija–4th edition– Ref: T.B of Medical Parasitology–S.C.Parija–4th edition–
Page 100 Page 104
• In Chagas disease caused by T. cruzi–esophagus, stom-
• • Cervical lymphadenopathy is a feature of west african

ach and colon are dilated leading to mega esophagus, sleeping sickness caused by T. brucei gambiense
megacolon and other organs are enlarged–called as • There are enlarged, non tender and mobile posterior

mega disease cervical lymph nodes called as Winter bottom sign

2. Ans.  (a) Reduviid bug 5. Ans.  (a) Trypanosoma brucei


Ref: T.B of Medical Parasitology–S.C.Parija–4th edition– Ref: T.B of Medical Parasitology–S.C.Parija–4th edition–
Page 95 Page 95, 102
• Reduviid bug is the vector and intermediate host for
• • Vector for trypanosomiasis:

T.cruzi ƒ T. cruzi–Reduviid bug


ƒ

• Unique feature of disease is R.J Chagas first discovered


• ƒ T. brucei–Tsetse fly
ƒ

T.cruzi in Reduviid bug rather than in man


6. Ans.  (b) Trypomastigote
3. Ans.  (a) Trypanosoma cruzi
Ref: T.B of Medical Parasitology–S.C.Parija–4th edition–
Ref: T.B of Medical Parasitology–S.C.Parija–4th edition– Page 102
Page 94 • Infective form for trypanosoma is metacyclic trypomas-

• Amastigote forms are mostly seen in:


• tigote
ƒ Leishmania
ƒ

ƒ Trypanosoma cruzi
ƒ

• T. cruzi resembles the amastigotes of Leishmania–hence


it is called Leishmanial form 303


7. Ans.  (a) Glossina
Unit 4     Parasitology

Ref: Medical Parasitology–3rd edition–D.R.Arora–page 59


• The invertebrate host involved in the life cycle of African trypanosomiasis is Glossina genus termed as tsetse fly.

8. Ans.  (a) Metacyclic trypomastigote


Ref: Medical Parasitology–3rd edition–D.R.Arora–page 59
•• The above peripheral blood film shows Trypanosoma brucei gambiense
•• It occurs in three forms: long slender with flagellum, intermediate form and short stumpy form without flagellum.
•• Metacyclic trypomastigote is the infective form for man
•• Short stumpy form is the infective form for tsetse fly

304
40
Leishmania
INTRODUCTION
y Leishmaniasis is a zoonotic infection
y
y Vector is sand fly (Phlebotomus)
y
y There are several species in Leishmania
y
Leishmania donovani • Visceral leishmaniasis

• Post kala azar dermal leishmaniasis

Leishmania tropica • Cutaneous leishmaniasis

Leishmania mexicana • Cutaneous leishmaniasis

Leishmania braziliensis • Mucocutaneous leishmaniasis

(Espundia)

LEISHMANIA DONOVANI
y Morphological forms are Amastigotes and Promastigotes
y
Figure 1: Amastigotes of L. donovani in microscopy
Amastigotes (LD bodies) Humans
CDC/Dr Francis W Chandler
Promastigotes Sand fly
y Culture can be done by NNN medium
y
y Definitive host: Humans; Intermediate host: Sand fly y Animal inoculation done in Chinese and golden hamster
y
y
y Humans are the only reservoir; no animal reservoirs (Indian (intraperitoneal inoculation)
y
kala azar)–while in other countries zoonotic spread is there y Serodiagnosis:
y
y Infective form: Promastigote  Complement fixation test
y

y It causes visceral leishmaniasis or Kala azar  Direct agglutination test
y

y Clinical features are–Fever, Anemia, hepatosplenomegaly,  Rapid immunochromatographic test (RDT)–for detec-
y

weight loss and lymphadenopathy tion of antibodies against rK39 antigen–rk39 dipstick
y Additional features are–leucopenia, thrombocytopenia, test is widely used now and sensitive.
y
hypergammaglobulinemia  Napier`s aldehyde test–1–2ml of the serum from patient

y After treatment of the primary disease–due to partially treated is added with 40% formalin; a milky white jellification
y
or untreated cases–recurrence occurs as non ulcerative is seen means that indicates positive test. But this test is
lesions even up to 20 years–named as post kala azar dermal nonspecific.
leishmaniasis (PKDL)  Antimony test

y PKDL: y Leishmanin skin test: Montenegro test (used for epidemio-
y
y
 Indian form–usually present after years, with nodule and logical studies)–this test is usually positive 4 to 6 weeks after

no ulceration without nerve involvement onset in case of Cutaneous and mucocutaneous leishmania-
 African form–presents suddenly, ulceration occurs, nerve sis.

involvement occurs
y In India–endemic areas are Bihar, West Bengal, Eastern UP High Yield
y
Case definition for kala azar
Lab Diagnosis • A case of Kala azar is defined as a person from an endemic

area who has fever for more than 14 days duration with
y Smears prepared from spleen, BM, lymph nodes or peripheral
splenomegaly and has positive RDT test or a biopsy.
y
smear shows amastigotes seen inside macrophages (LD
bodies)
y Most sensitive method to detect LD bodies in splenic smear Treatment
y
y But the most common method of diagnosis is bone marrow
y Sodium stibogluconate (SSG)
y
aspiration and microscopy
y
y When resistant strains to SSG are seen: y Aldehyde test is negative in cutaneous leishmaniasis
Unit 4     Parasitology

y y

 Amphotericin B deoxycholate
 y Skin test is positive in cases treated for ulcers but skin test is
y

 Miltefosine
 usually negative in diffuse cutaneous leishmaniasis
y Treatment of PKDL: SSG + Rifampicin (4 months)
y

LEISHMANIA BRAZILIENSIS
LEISHMANIA TROPICA y Most dangerous form of cutaneous leishmaniasis is
y

y Morphology is identical to that of donovani


y
mucocutaneous leishmaniasis also called as Espundia is
y L. tropica causes cutaneous leishmaniasis otherwise called
y
caused by braziliensis
as Delhi boil, Aleppo boil y Amastigotes are seen in the skin and the nasal mucosal
y

y The difference between tropica and donovani is tropica do


y
membranes
not invade organs y Ulcers starts progressing and may lead to perforation or
y

y Bite of the sandfly region develops a papule followed by


y
severe destruction of the nasal septum
ulceration and scar y Nasal tip may collapse named as tapir nose
y

y This ulcer is called as oriental sore


y
y This condition is not seen in India
y

y From the localized site–amastigotes starts moving and cause


y
y Slit skin smear helps to visualize the amastigotes
y

satellite lesions–finally metastasise to face and extremities– y Leishmanin test is positive


y

leading to diffuse cutaneous leishmaniasis y Serological tests are helpful


y

y DOC–Pentavalent antimonials
y

Lab Diagnosis LEISHMANIA MEXICANA


y Full thickness skin biopsy–shows amastigotes
y Zoonotic disease
y

y Culture is needed when no parasites are seen in microscopy


y

y Causes bay sore or Chiclero ulcer


y

306
MULTIPLE CHOICE QUESTIONS

Chapter 40     Leishmania


1. Amastigote forms are seen in (PGI 2017) a. Plasmodium vivax
a. Leishmania donovani b. Toxoplasma gondii b. Leishmania
c. Leishmania major d. Entamoeba c. Microfilaria
e. T.brucei d. None
2. Nasopharyngeal leishmaniasis is caused due to? 6. In a case of Kala azar aldehyde test becomes positive
(Recent pattern 2017) after (Recent pattern 2018)
a. Leishmania brazilensis b. Leishmania tropica a. 2 weeks
c. Leishmania chagasis d. Leishmania donovani b. 4 weeks
3. Drug not used in visceral leishmaniasis c. 8 weeks
(AIIMS May 2018) d. 12 weeks
a. Sitamaquine b. Paromomycin 7. Leishmania is cultured in ……………….media
c. Miltefosine d. Hydroxychloroquine (Recent pattern 2017)
4. Which of the following is most severely affected in Kala a. Chocolate agar b. NNN
azar (Recent pattern 2017) c. Tellurite d. Sabourauds
a. Spleen b. Liver 8. Espundia is caused by (Recent pattern 2018)
c. Adrenal gland d. Bone marrow a. L.donovani b. L.tropica
5. 40 year old male from Bihar complains of abdomen pain, c. L.mexicana d. L.braziliensis
having hepatosplenomegaly, smear on stain shows 9. Oriental sore is caused by (Recent pattern 2017)
(Recent pattern 2017) a. L.donovani b. L.tropica
c. L.mexicana d. L.braziliensis
10. Infective stage for kala azar (Recent pattern 2017)
a. Amastigotes
b. Promastigote
c. Trypomastigote
d. Epimastigote
11. A patient presenting from West Bengal with fever,
lymphadenopathy. Serological test showed rk39 posi-
tive. What is the treatment of choice? 
 (AIIMS May 2018)
a. Sodium stibogluconate b. Artemesinin
c. Chloroquine d. Dapsone

ANSWERS AND EXPLANATIONS


1. Ans.  (a) Leishmania donovani; (c) Leishmania major 3. Ans.  (d) Hydroxychloroquine
Ref: T.B of Medical Parasitology–S.C.Parija–4 th
edition– Ref: T.B of Medical Parasitology–S.C.Parija–4th edition–
Page 75 Page 83, Harrison T.B of internal medicine 19th edition–
• Hemoflagellates that have amastigote form are:

Page 1390
ƒ Leishmania sp (all species)
ƒ • Drugs used in the treatment of kala azar or visceral leish-

ƒ Trypanosoma cruzi
ƒ maniasis are:
ƒ Sodium stibogluconate
ƒ

2. Ans.  (a) Leishmania brazilensis ƒ Amphotericin B


ƒ

Ref: T.B of medical parasitology–S.C.Parija–4 th


edition– ƒ Paromomycin
ƒ

ƒ Pentamidine
Page 92 ƒ

ƒ Miltefosine
• Nasopharyngeal leishmaniasis is a part of mucocutane-
ƒ

ƒ Sitamaquine

ous leishmaniasis
• Classical features of nasal septum perforation occurs

and tip of the nose gets collapsed named as Espundia


• This condition is caused by Leishmania braziliensis

307
4. Ans.  (a) Spleen 9. Ans.  (b) L.tropica
Unit 4     Parasitology

Ref: T.B of Medical Parasitology–S.C.Parija–4th edition– Ref: T.B of medical parasitology–S.C.Parija–4th edition–
Page 78 Page 86
• Splenomegaly is the most important sign seen in kala
• • Oriental sore is caused by L.tropica

azar • It presents as an ulcerative lesion in the skin – this ulcer


is called as oriental sore


5. Ans.  (b) Leishmania
10. Ans.  (b) Promastigote
Ref: T.B of Medical Parasitology–S.C.Parija–4th edition–
Page 81 Ref: T.B of medical parasitology–S.C.Parija–4th edition–
• Above image shows macrophages

Page 76
• Within the macrophages–arrow points show amastigotes
• • Bite of infected sandfly with promastigotes causes kala

of L. donovani called as LD bodies azar

6. Ans.  (d) 12 weeks 11. Ans.  (a) Sodium stibogluconate


Ref: T.B of parasitology–Chaterjee–12 edition–Page 61
th
Ref: T.B of medical parasitology–S.C.Parija – 4th edition–
• Napier’s aldehyde test principle is based on increase in

Page 83, Harrison T.B of internal medicine 19th edition–
the serum gamma globulins Page 1390
• Visceral leishmaniasis has hypergammaglobulinemia
• • Rapid immunochromatographic test (RDT) is done

• Serum of the patient is added with two drops of 40%


• for detection of antibodies against rK39 antigen of
formalin–if jellification occurs then the test is positive Leishmania donovani - rk39 dipstick test is widely used
• The test is positive in:
• now and sensitive.
ƒ Visceral leishmaniasis or Kala azar
ƒ • Leishmania donovani causes kala-azar

ƒ African trypanosomiasis or T.brucei infection


ƒ • Drugs used in the treatment of kala azar or visceral

ƒ S. japonicum infection
ƒ leishmaniasis are:
ƒ Sodium stibogluconate
ƒ

7. Ans.  (b) NNN ƒ Amphotericin B


ƒ

Ref: T.B of medical parasitology–S.C.Parija–4 th


edition– ƒ Paromomycin
ƒ

Page 82 ƒ Pentamidine
ƒ

ƒ Miltefosine
• Leishmania donovani is cultured in NNN medium
ƒ

ƒ Sitamaquine
• NNN – Novy, McNeal and Nicolle medium
ƒ

• The amastigote form in the blood when in culture with


NNN medium gets converted into promastigote form

8. Ans.  (d) L.braziliensis


Ref: T.B of medical parasitology–S.C.Parija–4th edition–
Page 92
• Already explained Q.2

308
41
Apicomplexa
INTRODUCTION Stages of Life Cycle in Features
human
Order Haemosporida Order Piroplasmida
• But exception in P. falciparum-
• Plasmodium • Babesia


live in brain capillaries and other


• Haemoproteus • Entopolypoides organs by forming aggregates


• Leucocytozoon (only young ring forms are seen
in peripheral blood)

• Pigments are seen in late
PLASMODIUM


trophozoite and schizont stage
y Malarial parasites are called as Plasmodium Gametogony • Merozoites inside the red cells
y
y There are five species in Plasmodium


develop into gametocytes (male
y
 Plasmodium falciparum
and female)

 Plasmodium vivax
• Male gametocytes are called as

 Plasmodium malariae


microgametocytes

 Plasmodium ovale
• Female gametocytes are called as

 Plasmodium knowlesi


macrogametocytes

Exoerythrocytic • Occurs in P. vivax and P. ovale
Hosts

schizogony • Some of the sporozoites in liver

gets converted into dormant
Definitive host Female Anopheles mosquito (Sexual cycle
stage called as hypnozoites –
happens)
this is responsible for relapse of
Intermediate host Man (Asexual cycle happens) malaria

Life Cycle of Malarial Parasite Cycle in Mosquito


y Infective form for humans–sporozoites (from mosquito) y When mosquito bites human–the gametocytes are ingested
y
y
y Infective form for mosquito–Gametocyte by them
y
y Inside the stomach of mosquito–they are converted into
y
Table 1:  Stages of life cycle in human gametes–zygote–ookinete–oocyst

Stages of Life Cycle in Features


human Routes of Transmission of Malaria
Pre erythrocytic • Occurs in liver parenchymal cells y Bite of infected female anopheles mosquito
y
y Blood transfusion

schizogony • Sporozoites gets converted into
y
y Maternal to fetus

• P. falciparum–6 days merozoites
y
y Contaminated syringes

• P. vivax–8 days • When the cycle is complete–these
y


• P. ovale–9 days merozoites are liberated from the
Table 2:  Incubation period of species

• P. malariae–13 to 16 liver to blood

days
Species Incubation period
Erythrocytic schizogony • Merozoites in the blood–invade
P. falciparum 12 days (9-14)

• P. falciparum–36 to 48 RBCs

hours • Converted into young ring form- P. vivax 14 days (8-17)

• P. vivax–48 hours trophozoites–then schizonts–then P. ovale 17 days (16-18)

• P. ovale–48 hours merozoites
P. malariae 28 days (18-40)

• P. malariae–72 hours • During this phase after a time


period they die out (RBC life cycle)
(Contd...)
Unit 4     Parasitology

Figure 1: Life cycle of Plasmodium

Pathogenesis of Malaria  Benign tertian malariae–P. vivax and P. ovale


 Malignant tertian malariae–P. falciparum
y Human infection begins with bite of female anopheline


 Quartan malariae–P. malariae


y

mosquito that inoculates sporozoites into man.




 Quotidian malariae–P. knowlesi


y These sporozoites are carried via the bloodstream to the liver.


y Microcytic or normocytic hypochromic anemia; Pernicious


y

They invade the hepatic parenchymal cells and starts asexual


y

anemia
reproduction.
y Splenomegaly (tropical splenomegaly or hyper reactive
y This amplification process is called as intrahepatic or
y

malarial splenomegaly seen in endemic areas)


y

pre-erythrocytic schizogony or merogony (sporozoites →


y Chronic or repeated infections with P. malariae causes
Merozoites)
y

immune mediated injury to renal glomeruli causing quartan


y The infected hepatocytes bursts and releases these merozoites
malarial nephropathy.
y

into the blood stream.


y P. falciparum causes:
y These merozoites then go and invade the RBCs and multiply
y

 Cerebral malaria
y

inside them.


 Black water fever


y Because of multiplication - the parasite density reaches as 50/


μl of blood and that leads to symptomatic stage of infection.


y In some species like P. vivax and P. ovale, some intrahepatic
y
Table 3:  Features of various species of Plasmodium
forms remain inside for even years and termed as Hypnozoites Features P. vivax P. falciparum P. ma- P. ovale
(dormant forms)–these forms are responsible for relapse of lariae
infection.
y After entering into the blood stream, the merozoites gets
y
Diagnos- Trophozo- Ring forms Trophozo- Trophozo-
converted into trophozoites. Parasite gets attached to the tic forms ites Gametocytes ites ites
RBCs by erythrocyte surface receptor. seen in Schizonts Schizonts Schizonts
y By the end of intra-erythrocytic cycle–the parasite eats 2/3rd
y
peripher- Gameto- Gameto- Gameto-
of the hemoglobin and occupies the cells named as Schizont. al smear cytes cytes cytes
The RBC then ruptures releasing the merozoites and this
invades new RBC and repeats the cycle. Infected Enlarged Normal Normal Enlarged
y But some of the blood forms instead of going to schizont
y

RBC
stage, gets entered into sexual reproduction phase and gets
converted to gametocytes. Dots Schuffner’s Maurer’s dots Ziemann’s James’
y These micro and macro gametocytes are taken by mosquitoes
y
dots dots dots
where sexual reproduction occurs leading to oocyte. Others Relapse Recrudescence - Relapse

Clinical Features
310 y Febrile paroxysms: Based on the periodicity of malarial fever–
y

fever is divided into:


Plasmodium Knowlesi

Chapter 41     Apicomplexa


y It is also known as monkey malarial parasite
y

y This species is seen in South East Asia and South America


y

y The main hosts are long tailed and pig tailed macaques
y

y The fever is typically quotidian


y

y Parasite looks similar to P. malariae–hence it may be misdi-


y

agnosed
y But the disease is more severe than malaria
y

y Treatment: Chloroquine
y

Lab Diagnosis
y Microscopy of peripheral smear–thick and thin smear method
y

y Conventional light microscopy is the gold standard for


y

confirmation of malaria (Ref: Parija)


y Thick smear is helpful to easy screening of malarial parasite
Figure 2: Schizont of P. vivax
y

y Thin smear is useful for species identification


(Courtesy: CDC/ Dr Mae Melvin)
y

y Fluorescence method using acridine orange staining is also


y

helpful
y QBC–Quantitative buffy coat:
y

 Principle is based on acridine orange which stain the




nucleic acid in the parasites


 More sensitive method than thick blood smear–can


detect as low as 3–4 parasites/uL of blood


y Serodiagnosis:
y

 Immunochromatographic tests–rapid diagnostic tests–




detects HRP-2 and pLDH and aldolase


 ELISA 

y PCR–helpful to diagnose P. knowlesi


y

Remember
Plasmodium vivax Schuffner’s dots
Figure 3: Gametocyte (banana shaped) of P. falciparum
Plasmodium falciparum Maurer’s dots
(Courtesy: CDC/ Dr. Mae Melvin)
Plasmodium malariae Ziemann’s dots
Remember Plasmodium ovale James’ dots
Manifestation of Severe Falciparum Malaria
• Cerebral coma

• Acidosis

• Severe normochromic/normocytic anemia


• Renal failure

• Pulmonary edema/ARDS

• Hypoglycemia

• Hypotension/shock

• DIC

• Convulsions

• Hemoglobinuria

• Hyperparasitemia

• Jaundice

High Yield
Why Plasmodium Falciparum is most Pathogenic?
• > 2,50,000 to 3,00,000/mL parasites of blood

• 30–40% of total RBC’s are parasitized


• Invasion of erythrocytes of all ages


• Ability to do cytoadherence that leads to sequestration of the


parasite Figure 4: Schizont and ring forms of Plasmodium vivax


• Extreme diversity of PfEMP1 antigen–which cannot be
• (Courtesy: Dr S. Jamuna Rani, Associate professor of pathology,
targeted by immune system Tagore Medical College & Hospital, Chennai) 311
• Massive cytokine production leading to end organ disease

Chemoprophylaxis
Unit 4     Parasitology

Remember y For all chloroquine sensitive areas–prophylaxis with chloro-


Laboratory manifestations seen in severe falciparum
y

quine
malaria:
y Chloroquine resistant areas–Atovaquone-Proguanil or Doxy-
• Hypoglycemia
y

cycline

• Hyperlactatemia
y For P. vivax–Primaquine

• Acidosis
y

• Elevated serum creatinine


Treatment Failure

• Elevated total bilirubin


• Elevated liver enzymes



y Early treatment failure:
y

• Elevated muscle enzymes



 Development of danger signs or severe malaria on Day 1,2


• Elevated urate

or 3 in the presence of parasitemia
• Leukocytosis

 Parasitemia higher on Day 2 than Day 0


• Severe anemia

y Late clinical failure:
y

• Coagulopathic features

 Development of danger signs on day 4 to day 28


• Hyperparasitemia

y Late parasitological failure:
y

 Presence of parasitemia on any day between day 7 and




day 28
Table 4:  Diagnosis of malaria
Standard methods for Features High Yield
diagnosis of malaria
Initiatives to Reduce Malaria
Thick blood film Sensitive method that will • Global Technical Strategy for malaria: (2016–2030)

detect even 0. 0001% of • Roll back malaria strategy by WHO: 1998


parasitemia; but it needs • National malaria control programme


experience to look the parasites.

Thin blood film Rapid and specific method BABESIOSIS


but it can detect only 0. 05% y Babesiosis is an emerging Tick borne zoonotic illness
y

parasitemia. y Causative agent is Babesia; There are many species namely:


y

 Babesia microti
PfHRP2 dipstick test Rapid and inexpensive tests. 

 Babesia bovis
Help in field surveys. Detects 

 Babesia divergens
only plasmodium falciparum 

y Vector: Ixodid ticks; it can also be occasionally transmitted


Plasmodium LDH dipstick Rapid tests; can detect all species
y

through transfusion of blood or blood products.


test y Babesia microti is the most common transfusion transmitted
y

Microtube concentration Sensitive tests and superior to pathogen.


methods (with Acridine thick films; helps to identify y Morphological forms:
y

orange staining) and process large number  Merozoites




of samples; but it needs  Trophozoites




fluorescence microscopy.  Sporozoites




y Sporozoites are the infective form


y

Table 5:  Treatment


Definitive host: Hard ticks
Chloroquine sensitive–for all Chloroquine (dosage–10 mg/kg)
species Intermediate host: Humans and animals
P. vivax and P. ovale–to avoid In addition to chloroquine +
relapse Primaquine (Primaquine should Pathogenesis
not be given in G6PD deficiency) y Pathogen is seen inside RBCs. Rupture of infected RBCs leads
y

P. falciparum Artesunate + Sulfadoxine/ to anemia.


Pyrimethamine y Ruptured RBCs generates cell debris and this gets accumu-
y

lated in kidney causing renal failure.


MDR P. falciparum Artemether-Lumefantrine (or)
y Babesia microti infections are usually asymptomatic
Artesunate–Mefloquine (or)
y

y Infection with Babesia bovis leads to hemolytic anemia,


Dihydroartemisinin–piperaquine
y

thrombocytic anemia an haemoglobinuria (resembling


Second line drugs Tetracycline falciparum)
Doxycycline
Clindamycin
312
Lab Diagnosis

Chapter 41     Apicomplexa


Remember
Clinical suspicion for Babesiosis:
• H/o travel to Babesia endemic area (north eastern and mid western USA)

• H/o fever in late spring or summer


• H/o blood transfusion within 6 months


y y Complete blood count shows anemia, thrombocytopenia, elevated reticulocyte counts and elevated LDH levels.
y y LFT: Liver enzymes elevated
y y Urine analysis: Hemoglobinuria, proteinuria, excess urobilinogen, elevated BUN, serum creatinine.
y y Specific diagnosis is done by Peripheral smear–Demonstration of ring forms; pigments are absent; no gametocytes; maltese cross
pattern (merozoites are arranged in tetrads) helps to differentiate from malaria

Figure 5: Babesia in blood smear


(Courtesy: CDC/Dr. George Healy)
y Intraperitoneal inoculation in golden hamsters or gerbil
y

y Serology: Indirect fluorescent antibody test


y

y PCR (helps in confirmation)


y

Table 6:  Treatment of Babesiosis


Species and type of infection DOC
Babesia microti – mild to moderate illness Atovaquone + Azithromycin
Babesia microti – severe illness Clindamycin + Quinine + Exchange transfusion
Babesia divergens Immediate complete exchange transfusion + clindamycin +
Quinine

313
MULTIPLE CHOICE QUESTIONS
Unit 4     Parasitology

Plasmodium 12. Which of the following is true about P. falciparum


(Recent pattern 2017)
1. Causative agent of malaria (Recent pattern 2017)
a. James dots are seen
a. Protozoa
b. Accole forms are seen
b. Mosquito
c. Relapses are frequent
c. Bacteria
d. Longest incubation period
d. Virus
13. Ziemann’s stippling of erythrocyte is caused by which
2. The scientist who discovered the transmission of malar-
species of plasmodium (Recent pattern 2018)
ia by anopheline mosquito (Recent pattern 2017)
a. Vivax b. Falciparum
a. Laveran
c. Malariae d. Ovale
b. Paul muller
14. Stages seen in peripheral smear of falciparum malaria
c. Ronald ross
(PGI May 2018)
d. Pampana
a. Schizont b. Gametocyte
3. Malarial parasite was discovered by
c. Accole d. Ring form
(Recent pattern 2017)
e. Trophozoite
a. Ronald ross
15. Stages of falciparum not seen in PBS is
b. Paul muller
(Recent pattern 2017)
c. Laveran
a. Schizont b. Gametocyte
d. Pampania
c. Ring form d. Double ring
4. In malaria, sexual cycle is (Recent pattern 2017)
16. Malarial pigment is formed by (Recent pattern 2017)
a. Sporozoite to gametocyte
a. Parasite b. Bilirubin
b. Gametocyte to sporozoite
c. Hemoglobin d. Any of the above
c. Occurs in human
17. A patient presents with fever. Peripheral smear shows
d. Responsible for relapse
band across the erythrocytes. Diagnosis is
5. In malaria pre-erythrocytic schizogony occurs in
(AIIMS 2017)
(Recent pattern 2018)
a. P. falciparum b. P. vivax
a. Lung b. Liver
c. P. ovale d. P. malariae
c. Spleen d. Kidney
18. Band form of P. malariae is (Recent pattern 2017)
6. Infective form of mosquito in plasmodium falciparum
a. Schizont stage b. Trophozoite stage
(AIIMS 2017)
c. Merozoite stage d. Gametocytic stage
a. Merozoites b. Sporozoites
19. Relapse in seen in:  (Recent Pattern Nov 2017)
c. Gametocytes d. Trophozoites
a. Plasmodium vivax and falciparum
7. Which form of the malarial parasite is present in saliva
b. Plasmodium ovale and falciparum
of an infective mosquito (Recent pattern 2018)
c. Plasmodium vivax and ovale
a. Ring form b. Schizont
d. Plasmodium vivax and malariae
c. Gametocyte d. Sporozoite
20. All are true about Plasmodium knowlesi all except:
8. Transfusion associated malaria has a shorter incubation
(Recent Nov 2018)
period because of: (AIIMS 2017)


a. Monkeys are the host b. Chloroquine sensitivity


a. Trophozoites b. Sporozoites
c. Central Africa d. Quotidian malaria
c. Female Gametocyte d. Merozoites
21. Ziemann dots are seen in:  (Recent pattern Nov 2017)
9. Malaria carriers contain (Recent pattern 2017)
a. Plasmodium malariae
a. Trophozoite b. Gametocytes
b. Plasmodium ovale
c. Merozoites d. Trophozoites
c. Plasmodium vivax
10. Recrudescences are seen in which malaria
d. Plasmodium falciparum
(Recent pattern 2018)
a. P vivax b. P ovale
Babesiosis
c. P malariae d. P falciparum
11. Why are schizont and late trophozoite stages of 22. Babesiosis is transmitted by (Recent pattern 2017)
plasmodium falciparum not seen in peripheral blood a. Tick b. Mites
smear? (AIIMS 2017) c. Flea d. Mosquito
a. They are sequestered in the spleen 23. Maltese cross is characteristic feature of
b. Due to adherence to the capillary endothelium,they are (Recent pattern 2017)
not seen in peripheral blood a. Cryptococcus neoformans
c. Due to antigen-antibody reaction and removal b. Babesia microti
d. They are seen in mosquito blood c. Blastomycosis
314 d. Penicillium marneffei
24. Infective form of humans for babesiosis is 26. Identify the vector that is involved in following diagnos-

Chapter 41     Apicomplexa


(Recent pattern 2017) tic image: (Recent pattern 2018)
a. Sporozoites
b. Merozoites
c. Gametocytes
d. Trophozoites
25. Which of the following is true about Malaria: 
 (PGI May 2017)
a. Chloroquine resistance occurs in India
b. Relapses is usual for vivax and ovale malaria
c. Sexual cycle occurs in mosquito
d. Not a public problem in India
a. Female anopheles mosquito
b. Ixodid tick
c. Glossina d. Reduvid bug

ANSWERS AND EXPLANATIONS

1. Ans.  (a) Protozoa 5. Ans.  (b) Liver


Ref: T.B of Medical Parasitology–S.C.Parija–4th edition– Ref: T.B of Medical Parasitology–S.C.Parija–4th edition–
Page 111 Page 113
• Malaria is caused by Plasmodium–which is a protozoan
• • Pre erythrocytic and exo erythrocytic schizogony occurs

parasite in liver
• Female anopheles mosquito acts as a vector
• • Erythrocytic schizogony occurs inside RBCs

2. Ans.  (c) Ronald ross 6. Ans.  (c) Gametocytes


Ref: T.B of Medical Parasitology–S.C.Parija – 4th edition– Ref: T.B of Medical Parasitology–S.C.Parija–4th edition–
Page 109 Page 114
• Ronald Ross demonstrated the life cycle of malarial
• • Infective form for man-Sporozoites

parasite in mosquito • Infective form for mosquito – Gametocytes


• He was given Nobel prize


7. Ans.  (d) Sporozoite


3. Ans.  (a) Ronald ross
Ref: T.B of Medical Parasitology–S.C.Parija–4th edition–
Ref: T.B of Medical Parasitology–S.C.Parija–4 th
edition– Page 114
Page 109 • Already explained Q.6

• Already explained Q.2


8. Ans.  (a) Trophozoites


4. Ans.  (b) Gametocyte to sporozoite
Ref: T.B of Medical Parasitology–S.C.Parija–4th edition–
Ref: T.B of Medical Parasitology–S.C.Parija–4th edition– Page 120
Page 114
Mosquito borne malaria Blood transfusion
• Asexual cycle occurs in humans

malaria
• Sexual cycle occurs in female anopheles mosquito
• Infective stage is • Infective stage is

• Gametocytes are in the infective form for anopheles • •

sporozoite trophozoite

from man; these gets converted to sporozoites and


become infective for humans • Long incubation period
• • Short incubation period

Gametocytes → Sporozoites (Mosquito) • Pre-erythrocytic and exo-


• • Pre-erythrocytic and exo-

Sporozoites → Gametocytes (Man) erythrocytic schizogony erythrocytic schizogony


present absent

315
9. Ans.  (b) Gametocytes 14. Ans.  (b) Gametocyte; (c) Accole; (d) Ring form
Unit 4     Parasitology

Ref: T.B of Parasitology–Chaterjee–12th edition–Page 113 Ref: T.B of Medical Parasitology–S.C.Parija–4th edition–
• In endemic areas, malarial parasites are residing in

Page 114,115
humans and act as as carriers (reservoirs) • The diagnostic forms of falciparum that are seen in

• Carriers harbors gametocytes


• peripheral smear are early ring forms and gametocytes
• Rings forms when seen attached to the margin of a red

10. Ans.  (d) P falciparum blood cell is called as accole


Ref: T.B of Medical Parasitology–S.C.Parija–4th edition–
15. Ans.  (a) Schizont
Page 119
• Relapse is seen in P.vivax and P.ovale

Ref: T.B of Medical Parasitology–S.C.Parija–4th edition–
• Recrudescence is seen in P.falciparum

Page 114
• Already explained Q.14

11. Ans.  (b) Due to adherence to the capillary


endothelium, they are not seen in peripheral blood 16. Ans.  (c) Hemoglobin
Ref: T.B of Medical Parasitology–S.C.Parija–4 th
edition– Ref: T.B of Medical Parasitology–S.C.Parija–4th edition–
Page 114; Page 115
• In P. falciparum, the erythrocytic schizogony takes place
• • Malarial parasite resides in RBC and feeds on the

within 48 hours hemoglobin


• This takes places in the capillaries of the internal organs
• • These hemoglobin feed becomes pigments inside the

• Hence the schizonts and merozoites are not seen in


• parasite
peripheral blood
• Only young ring forms and gametocytes can be seen

17. Ans.  (d) P.malariae
Ref: T.B of Medical Parasitology–S.C.Parija–4th edition–
12. Ans.  (b) Accole forms are seen
Page 112
Ref: T.B of Medical Parasitology–S.C.Parija–4th edition– • Band form trophozites are characteristic of P malariae

Page 112
• P.falciparum parasite attaches to the margin or edge of

18. Ans.  (b) Trophozoite stage
the host cell, nucleus and near the cytoplasm too Ref: T.B of Medical Parasitology–S.C.Parija–4th edition–
• It is known as accole form

Page 112
• Already explained Q.17

19. Ans.  (c) Plasmodium vivax and ovale


Ref: T.B of Medical Parasitology – Parija – 4th ed – Page 173
• True relapse – caused by hypnozoites

• Specific to P.ovale and P.vivax


• Due to re emergence of blood stages from latent


hypnozoites

20. Ans.  (c) Central Africa


Ref: Harrison T.B of Internal Medicine – 19th edition – Page
1368-1371
• Plasmodium knowlesi is called as monkey malarial

parasite
 Accole form of falciparum parasite • This species is particularly seen on the island of

Borneo and, to a lesser extent, elsewhere in Southeast


13. Ans.  (c) Malariae Asia, where the main hosts, long-tailed and pig-tailed
macaques, are found.
Ref: T.B of Medical Parasitology–S.C.Parija–4th edition–
• It may cause severe malaria as indicated by its asexual
Page 112

erythrocytic cycle of about 24 hours, with an associated


P. vivax Schuffner’s dots fever that typically occurs at the same frequency (i.e. the
fever is quotidian).
P. falciparum Maurer’s dots
• Treatment of Plamodium knowlesi uncomplicated

P. malariae Ziemann’s stippling malaria is: Chloroquine (10 mg of base/kg stat followed
316 P. ovale James dots by 5 mg/ kg at 12, 24, and 36 h or by 10 mg/kg at 24 h and
5 mg/kg at 48 h)
21. Ans.  (a) Plasmodium malariae 24. Ans.  (a) Sporozoites

Chapter 41     Apicomplexa


Ref: Medical Parasitology – Arora – 3rd edition – Page 75 Ref: T.B of Medical Parasitology–S.C.Parija–4th edition–
• Infected red blood cells by the Plasmodium species

Page 138
undergoes alteration in size and shape • Infective form for humans is sporozoites

• In P. malariae – the RBC are of normal size with fine


stippling seen on prolonged staining – this stippling is 25. Ans: (a, b, c) (a) Chloroquine resistance occurs in
called Ziemann’s dots India; (b) Relapses is usual for vivax and ovale malaria;
(c) Sexual cycle occurs in mosquito
Plasmodium vivax Schuffner’s dots
• Already explained Q.4 and Q.10
Plasmodium falciparum Maurer’s dots

Plasmodium malariae Ziemann’s dots 26. Ans.  (b) Ixodid tick


Plasmodium ovale James’ dots Ref: Review of Medical Microbiology and Immunology – 13th
edition – Page 975
22. Ans.  (a) Tick
• The above figure showing maltese cross pattern of Ba-

Ref: T.B of Medical Parasitology–S.C.Parija–4th edition– besia


Page 138 • Babesiosis is a Tick borne zoonotic illness

• Definitive host is hard ticks (vector) • Pathogen is seen inside RBCs


• Sporozoites are the infective form


• Intermediate host is humans and animals •

• Peripheral smear: Demonstration of ring forms;


23. Ans.  (b) Babesia miroti pigments are absent; no gametocytes; maltese cross
pattern (merozoites are arranged in tetrads) helps to
Ref: T.B of Medical Parasitology – S.C.Parija – 4th edition – differentiate from malaria
Page 141
• In peripheral smear of babesiosis – merozoites are

arranged in tetrads and called as maltese cross pattern


• This is not so common condition

317
42 Toxoplasma,
Ciliate Protozoa
INTRODUCTION Diagnostic forms Features
y Toxoplasma is an obligate intracellular parasite that lives in Bradyzoites • These are tissue cysts or resting form in


y
the REC and nucleated cells humans
y Unique feature of this parasite is that all the stages are • Seen in chronic stage


• Commonly seen in brain, skeletal and
y
infective to humans


y Morphological forms: heart muscles
y
 Tachyzoites
Oocysts • Seen in cat and other felines

 Tissue cysts or Bradyzoites


• Not seen in humans

 Oocysts



Table 1:  Features of diagnostic forms y Definitive host: Cats (Sexual cycle occurs)
y
Diagnostic forms Features y Intermediate host: Humans and other mammals (Asexual
y
cycle occurs)
Tachyzoites • Actively multiplying form in humans
y Infecting form: Oocysts through cat feces

• Seen in acute stage
y

• This form causes invasion in all the cells

except RBCs
(Contd...)

Figure 1:  Life cycle of Toxoplasma


CLINICAL FEATURES CILIATE PROTOZOA

Chapter 42     Toxoplasma, Ciliate Protozoa


y Congenital toxoplasmosis: (Mother to fetus)
y

Balantidium Coli
 When infection occurs during pregnancy–it gets


transmitted to fetus (man to man transmission occurs like y It is the largest human protozoa
y

y Site of infection is large intestine of humans, pigs, monkeys


this)
y

y Morphological forms:
 Classical triad is: (3Cs)
y

 Trophozoite:


Š Invasive form
Š

• Chorioretinitis

Š Oval shaped with cilia all around it
Š

Š It has a micro and macro nucleus


Š

• Cerebral calcifications
• Š It ingests bacteria, RBC and fat
Š

 Cyst:
• Convulsions


Š Infective form
Š

Š Round in shape
Š

y Acquired toxoplasmosis: (Ingestion of undercooked meat,


y
Š Resting stage
Š

ingestion of food contaminated with cat feces, blood transfu- y Life cycle occurs in single host usually in pigs (natural host);
y

sion, organ transplantation, accidental inoculation) Humans are accidental host


y Humans acquire infection by ingestion of cysts contaminating
 Usually self-limiting in immunocompetent hosts–presents
y

food.


as lymphadenopathy y It causes ciliate dysentry–diarrhea with blood and mucous


 In immunocompromised patients like in AIDS patients–
y

(acute cases); in chronic cases it presents as diarrhea


presents as life threatening disease; presents as encepha- alternative with constipation;
litis y Balantidium invades the mucosa and submucosa causing
y

 Ocular toxoplasmosis infection presents as choroiretinitis


 ulcers named as Balantidum ulcers. They are found in cecum,
ascending colon and rectum. These ulcers do not invade the
muscular layer. Main complication of these ulcers is perforation.
LAB DIAGNOSIS
y Tissue samples like bone marrow aspiration shows tachyzoites
y
Lab Diagnosis
or bradyzoites depends upon the clinical stage y Microscopic examination of stool–wet mount–shows football
y

y Stain used is PAS or Giemsa–comma shaped tachyzoites seen


y
revolving motility of the trophozoites with characteristic
y Animal inoculation–intraperitoneally into mice
y
cilia around it
y Serological test: ELISA, IFA, IHA and latex agglutination tests y Balantidium grows well in:
y

 Robinson’s medium
y

y Sabin Feldman dye test:




 Balamuth’s medium
y

 Gold standard test for toxoplasmosis (highly sensitive




 Dobell’s medium


and specific test)




 Jone’s medium


 Done only in reference laboratories




 Test serum (contains antibodies) + live tachyzoites –




inactivated and killed (because of complement mediated


lysis)
 This is then stained with alkaline blue–which shows dead


tachyzoites as colorless and distorted–shows the test is


positive
y PCR helpful in detecting congenital infections in utero (by
y

amniotic fluid sampling)

TREATMENT
y y Pyrimethamine
y y Sulfadiazine Figure 2: Trophozoite of Balantidium coli
y y Clindamycin (Courtesy: CDC/ Dr L.L.A. Moore, Jr.)
y y Atovaquone y Biopsy from the large intestine and scrapings from ulcers also
y

y y Azithromycin shows the trophozoite


Treatment
y Tetracycline is the DOC; alternatively metronidazole can be
y

given.

319
MULTIPLE CHOICE QUESTIONS
Unit 4     Parasitology

1. Intermediate host for toxoplasma are all except: 8. Sabin Feldman dye test is used for diagnosis of:
(Recent pattern 2017)  (Recent pattern 2018)
a. Human b. Sheep a. Leishmaniasis b. Echinococcosis
c. Cat d. Pig c. Toxoplasmosis d. Balantidiasis
2. Oocyst of toxoplasma is found in (Recent pattern 2018)
a. Cat b. Dog Balantidium Coli
c. Mosquito d. Cow
9. All are true regarding Balantidium coli except:
3. Most common clinical feature of toxoplasmosis in an
a. Humans get infection from pigs (Recent pattern 2017)
immunocompetent adult: (Recent pattern 2018)
b. Trophozoite is the infective form
a. Encephalitis
c. It causes ulcers in the caecum
b. Lymphadenopathy d. Only ciliate that infect humans
c. Chorioretinitis 10. Revolving motility is seen in: (Recent pattern 2017)
d. Glaucoma a. Balantidium coli b. Trichomonas vaginalis
4. Sabin feldman Dye test is used to demonstrate infection c. Balamuthia mandrillaris d. Dientamoeba fragilis
with(Recent pattern 2017) 11. Largest human protozoan is (Recent pattern 2017)
a. Filaria b. Toxoplasma a. Entamoeba histolytica b. Entamoeba gingivalis
c. Histoplasma d. Ascaris c. Balantidium coli d. Naegleria fowleri
5. Congenital toxoplasmosis false is (AIIMS 2017) 12. Fecal smear showed the following finding; Identify:
a. IgA is better than IgM in detection  (Recent pattern Nov 2017)
b. Diagnosed by detection of IgM in cord blood
c. IgG is diagnostic
d. Cannot be diagnosed by antibodies
6. Which of the following is true regarding transmission of
Toxoplasma(Recent pattern Nov 2017)
a. Vertical
b. Ingestion of uncooked food with cyst
c. Organ transplantation
d. All of the above
7. In Toxoplasmosis the oocyst seen in________and
pseudocyst is seen in_______ respectively
(Recent pattern Nov 2017)
a. Tissue, felines b. Human, cats a. Balantidium coli b. Entamoeba dispar
c. Cats, human d. Tissue, feces c. Cryptosporidium d. Giardia

ANSWERS AND EXPLANATIONS


1. Ans.  (c) Cat 3. Ans.  (b) Lymphadenopathy
Ref: T.B of Medical Parasitology–S.C.Parija–4 th
edition– Ref: T.B of Medical Parasitology–S.C.Parija–4th edition–
Page 159 Page 162
• Definitive host is cat and other felines
• • Clinical presentation of toxoplasmosis is based on the

• Intermediate hosts are humans, mouse, sheep and pigs


• immunity of the host
• In immunocompetent hosts–it is benign condition and

2. Ans.  (a) Cat self-limiting–most common presentation is cervical


Ref: T.B of Medical Parasitology–S.C.Parija–4th edition– lymphadenopathy
Page 159
4. Ans.  (b) Toxoplasma
• There are three morphological forms in toxoplasma–

tachyzoites, bradyzoites and oocysts Ref: T.B of Medical Parasitology–S.C.Parija–4th edition–


• Sexual multiplication occurs in cat–oocyts are excreted

Page 165
in feces • Sabin Feldman dye test–used for the diagnosis of

• Asexual multiplication occurs in humans and other


• toxoplasmosis
320 animals–tachyzoites are active replicating forms • After infection is acquired–antibodies appear in the

• Bradyzoites are resting forms


• body following 2–3 weeks
• When live tachyzoites (active multiplying stage) of • Sabin Feldman dye test – used for the diagnosis of toxo-

Chapter 42     Toxoplasma, Ciliate Protozoa


• •

Toxoplasma is treated with serum of the patient (has plasmosis


antibodies) and alkaline methylene blue–the antibodies • After infection is acquired – antibodies appear in the

inactivate the live tachyzoites body following 2–3 weeks


• This looks as thin, distorted and colourless even in
• • When live tachyzoites (active multiplying stage) of

presence of alkaline blue as they are dead Toxoplasma is treated with serum of the patient (has
• If more than 50% are unstained then the test is positive
• antibodies) and alkaline methylene blue – the antibodies
• Gold standard method
• inactivate the live tachyzoites
• Done only in reference laboratories
• • This looks as thin, distorted and colourless even in

presence of alkaline blue as they are dead


5. Ans.  (c) IgG is diagnostic • If more than 50% are unstained then the test is positive

Ref: T.B of Medical Parasitology–S.C.Parija–4 th


edition– • Gold standard method

Page 166 • Done only in reference laboratories


• IgG antibodies can cross the placenta and so they are



9. Ans.  (b) Trophozoite is the infective form
not helpful in any of the congenital infections; it is from
maternal blood Ref: T.B. of Medical Parasitology–S.C.Parija–4th edition–
• IgM is diagnostic of any of the congenital infections

Page 171
• Natural host for Balantidium coli is pigs

6. Ans.  (d) All of the above • Infective form is cyst and invasive form is trophozoites

Ref: T.B of Medical Parasitology–S.C.Parija–4th edition–


10. Ans.  (a) Balantidium coli
Page 161
• Humans acquire infection by ingestion of food contami-

Ref: T.B. of Medical Parasitology–S.C.Parija–4th edition–
nated with sporulated oocysts and from uncooked meat Page 174
containing tissue cysts • Motile trophozoites are seen only in direct wet mount

• Humans are the intermediate host and cats are the


• examination of fresh liquid stools
definitive host • Rapid revolving motility (also called as football motility)

• Other modes of acquiring infection are


• is seen in Balantidium coli
ƒ Blood transfusion
ƒ • In Trichomonas vaginalis it is called as twitching motility

ƒ Organ transplantation
ƒ

ƒ Transplacental transmission
ƒ
11. Ans.  (c) Balantidium coli
ƒ Accidental inoculation of tachyzoites in laboratory
ƒ

Ref: T.B. of Medical Parasitology–S.C.Parija–4th edition–


Page 171
7. Ans.  (c) Cats, human
• The only ciliate and largest human protozoan is Balan-

Ref: T.B. of Medical Parasitology–S.C.Parija–4th edition– tidium coli


Page 159
• There are three stages in toxoplasma gondii—Tachy-

12. Ans.  (a) Balantidium coli
zoites, bradyzoites and oocysts Ref: T.B. of Medical Parasitology – Parija – 4th ed – Page 173
• Tachyzoites–active, multiplying trophozoites seen

during acute infection–seen in skeletal muscles, cardiac • Identification clue is: cilia all over the trophozoites
muscles and brain • • Balantidium coli – only ciliate known to infect humans
• Bradyzoites–resting form of the parasite-called as tissue • • Two forms: Trophozoite and cyst

cysts • Invasive form is Trophozoite – oval in shape – cilated –


• Oocysts are present only in cat and other felines–not in anterior end has mouth; macro and micro nucleus seen

humans • Trophozoite is capable of ingesting bacteria, RBC and


• Groups of proliferating tachyzoites inside a host cell is fats

called as pseudocyst • Contains many vacuoles


• • Causes ciliate dysentery or balantidiasis
8. Ans.  (c) Toxoplasmosis • • DOC: Tetracycline

Ref: T.B. of Medical Parasitology – S.C.Parija – 4th edition –


Page 165

321
43 Coccidian
Intestinal Parasites
COCCIDIAN PARASITES
y Cryptosporidium
y
y Cyclospora
y
y Isospora
y
CRYPTOSPORIDIUM
y Cryptosporidium parvum is an intestinal parasite that causes
y
diarrhea (especially in immunocompromised patients)
y Site–small intestine
y
y Morphological forms of this parasite are:
y
 Oocysts

 Sporozoite

 Trophozoite

 Merozoite

 Microgamont

 Macrogamont
Figure 1: Oocysts of Cryptosporidium parvum

y Infective form–Oocysts; there are two types of oocysts
(Courtesy: Department of microbiology, Chengalpattu Medical
y
 Thick walled oocysts that cause infections to others
College, Tamil Nadu)

 Thin walled oocysts that cause autoinfection

Mode of Infection Treatment
y Feco oral route y Anti diarrheal agents have no effect
y
y
y Zoonotic infections (contamination of oocysts from animal y Some benefits are there for paromomycin and spiramycin
y
y
feces in animal products) y In AIDS patients–Nitazoxanide has an effect
y
y It does not cause invasion after the mucosal layer in small
CYCLOSPORA
y
intestine–hence the diarrhea is non inflammatory/profuse/
non bloody y Species that cause intestinal infection is Cyclospora cayetan-
y The presentation of illness depends upon the immunity of
y
ensis
y
the individual y It affects the small intestine
y
 Immunocompetent persons–self-limiting illness y Humans get the infection by feco oral route through contam-

y
 Immunocompromised patients (AIDS)–life threatening ination with oocysts

diarrhea y Humans are the only source for infection
y
y Clinical condition resembles that of cryptosporidiosis
y
Diagnosis
y Stool microscopic examination–oocysts are seen Lab Diagnosis
y
y Acid fast staining using 0.5% H2SO4 (modified acid fast  Stool microscopy by acid fast staining–shows oocysts
y

staining)  Identification keys are:

y Identification keys are: Š Round to ovoid shape
y
Š
Š Oval shaped Š 8 to 10 um size
Š
Š
Š 4–5 um in diameter Š Variable acid fast
Š
Š Acid fast oocysts
Š
Š Auto fluorescence
Š
Š
Š Uniformly acid fast
Š
y ELISA to detect copro-antigen in stool
y
y Indirect fluorescent antibody tests
y
y PCR
y
Chapter 43     Coccidian Intestinal Parasites
Figure 2: Oocysts of Cyclospora
(Courtesy: CDC/DPDx-Melanie Moser) Figure 3: Oocyst of Isospora Belli
(Courtesy: Dr A Vijayalakshmi, HOD of microbiology. Chengalpattu
Treatment Medical College, Chengalpattu, Tamil Nadu)
y Self-limiting condition; if severe–TMP-SMX can be given
Treatment
y

ISOSPORA y TMP-SMX is the DOC


y

y Cystoisospora belli is a parasite that infects the small intestine


y

Table 1: Identification of coccidian parasites


of humans
y Humans gets infection by feco oral route through contamina- Coccidian Identification features
parasites
y

tion with mature oocysts


y It causes diarrhea–which is watery, profuse and foul smelling
y

Cyclospora Oocysts are of spherical or oval; Size is


y Resembles that of cryptosporidiosis
y

cayetanensis around 8–10 um; It has two sporocyts; Each


sporocyts has 2 sporozoites
Lab Diagnosis Acid fast oocyst; Partially acid fast
  Microscopic examination of acid fast staining of the fecal
smears shows Cryptosporidium Oocysts are of spherical or oval; Size is
Š Elliptical shaped oocysts; boat shaped (Variations in
Š
parvum around 4–5 um; It has 4 sporozoites
shape occurs) Acid fast oocyst
Š Acid fast oocysts
Cystoisospora Oocyst looks ellipsoidal (boat shaped);
Š

Š 20–33 um* 10–19 um size


Š

belli 20-33*10-19 um; Each sporocyts has 2


Š Each oocyst contains two sporoblasts–each has four
Š

sporozoites; Acid fast


sporozoites
Š Autofluorescence
Š

323
MULTIPLE CHOICE QUESTIONS
Unit 4     Parasitology

Coccidian Parasites 5. Auto infection is seen in: (Recent pattern 2017)


a. Cryptosporidium b. Cyclospora
1. In humans, cryptosporidiosis present as:
c. Cystisospora d. Microsporidia
(Recent pattern 2017)
a. Meningitis b. Diarrhea 6. A kinyoun positive oocyst was isolated from a HIV
patient with diarrhea; Identify the organism: 
c. Pneumonia d. Hepatitis
(AIIMS May 2018)
2. A HIV patient with malabsorption fever, chronic


diarrhea, with acid fast positive organism. What is the


causative agent? (AIIMS 2017)
a. Giardia b. Microsporidia
c. Isospora d. E. histolytica
3. A HIV positive individual having diarrheal episodes.
Stool examination reveals a oocyst of size 8-10 um. What
could be the diagnosis? (Recent pattern Nov 2017)
a. Cyclospora
b. Cryptococcus
c. Cryptosporidium parvum
d. Isospora
4. Drug of choice for cryptosporidiosis is:
(Recent pattern 2018)
a. TMP-SMX b. Nitazoxanide a. Cystisospora b. Cyclospora
c. Primaquine d. Niclosamide c. Cryptosporidium d. Microspora

ANSWERS AND EXPLANATIONS

1. Ans.  (b) Diarrhea • In patients with HIV–self-limiting diarrheal episodes are


more common with Cyclospora


Ref: T.B of Medical Parasitology–S.C.Parija–4th edition–
• While Cryptosporidium can cause life endangering
Page 147

diarrhea in immunocompromised patients


• Cryptosporidiosis presents the clinical manifestations
Coccidian Identification features

depending upon the immunity of the individual


parasites
• Immunocompetent individuals–it presents as self-
Cyclospora Oocysts are of spherical or oval; Size is

limiting diarrhea
cayetanensis around 8–10 um; It has two sporocyts;
• Immunosuppressed patients–life threatening diarrhea
Each sporocyts has 2 sporozoites

Acid fast oocyst; Partially acid fast


2. Ans.  (c) Isospora
Cryptosporidium Oocysts are of spherical or oval; Size is
Ref: T.B of Medical Parasitology–S.C.Parija–4th edition– parvum around 4–5 um; It has 4 sporozoites
Page 155 Acid fast oocyst
• Acid fast parasites that causes intestinal infections and

diarrhea in immunosuppressed patients are: Cystoisospora Oocyst looks ellipsoidal (boat shaped);
ƒ Cryptosporidium parvum
ƒ
belli 20-33*10–19 um; Each sporocyts has 2
ƒ Cyclospora cayetanensis
ƒ
sporozoites; Acid fast
ƒ Cystisospora belli
ƒ

3. Ans.  (a) Cyclospora 4. Ans.  (b) Nitazoxanide

Ref: T.B of Medical Parasitology–S.C.Parija–4th edition– Ref: T.B of medical parasitology–S.C.Parija–4th edition–
Page 146-155 Page 151
324
• Anti diarrheal agents have no effect in the treatment of • The thin walled oocysts which are seen in the intestinal

Chapter 43     Coccidian Intestinal Parasites


• •

cryptosporidial diarrhea lumen releases the sporozoites and they are responsible
• Some benefits are there for paromomycin and spiramy-
• for auto infection
cin
• In AIDS patients–Nitazoxanide has a symptomatic relief

6. Ans.  (a) Cystisospora
and effective Ref: Medical parasitology – Arora – 3rd edition – Page 97
5. Ans.  (a) Cryptosporidium • Clinical picture of HIV with diarrhea clearly suggests

that patient is suffering from cocciean parasite induced


Ref: T.B of medical parasitology–S.C.Parija–4th edition– infection
Page 145 • Oocysts seen in the image is identified as Cystisospora

• In Cryptosporidium parvum–there are two types of


oocysts
ƒ Thick walled oocysts that cause infections to others
ƒ

ƒ Thin walled oocysts that causes autoinfection


ƒ

325
44
Helminthology Cestodes
INTRODUCTION y Infective form: Plerocercoid larva

y
y Mode of infection: Ingestion of raw or inadequately cooked
y Helminths include worm like parasites

y
fish
y
y They are classified as:
y
 Platyhelminths (flat)–which includes Cestodes and Trem-
Remember

atodes
 Nemathelminths (Thread)–which includes Nematodes • It is the only human infecting tapeworm that has a life cycle



in aquatic life
Table 1:  Various Helminths • It is the only cestode that has two intermediate hosts


Cestodes Trematodes Nematodes • Only cestode which has operculated egg


• Diphyllobothrium Blood trematodes: • Ascaris
y Adult worm absorbs the Vit B12 from the stomach and causes


latum • Schistosoma lumbricoides y
B12 deficiency anemia–pernicious anemia

• Taenia solium haematobium • Necator
y Characteristics of anemia is hyperchromic, macrocytic-


• Taenia saginata • Schistosoma americanus
y
megaloblastic anemia with thrombocytopenia and mild


• Echinococcus mansoni • Strongyloides
leukopenia


granulosus • Schistosoma stercoralis
y Diagnosis is by demonstration of eggs in stool microscopy

• Hymenolepis nana japonicum • Trichuris trichiura
y
y Treatment: Praziquantel and Niclosamide


Hepatic trematodes: • Ancylostoma
y

• Fasciola hepatica duodenale
Tapeworm

• Fasciola gigantica • Enterobius


• Clonorchis sinensis vermicularis y These group of worms have a scolex or head; with a uterus

• Opisthorchis sp • Trichinella spiralis
y
having branches and genital pores


Intestinal y Larval forms of tapeworms are:
trematodes:
y
 Cysticercus
• Fasciolopsis buski

 Coenurus

• Heterophyes

 Unilocular hydatid cyst

heterophyes

 Multilocular hydatid cyst
• Metagonimus


yokogawai
• Watsonius
Taenia

watsoni y Two species of Taenia are:
y
• Gastrodiscoides  Taenia saginata


hominis  Taenia solium

Lung trematode: Table 2:  Features of T.saginata and T.solium
• Paragonimus
Features T.saginata T.solium

westermani
Name Beef or unarmed Pork or armed
CESTODES tapeworm tapeworm

Diphyllobothrium Latum Habitat Jejunum Upper jejunum


y Fish tapeworm Adult worm Tape like 5-10 m in Tape like 2-3 m in
length length
y
y Longest parasite seen in the intestine (jejunum and ileum)
y
of humans Scolex Large, quadrate with Small, globular with
y Egg: Bile stained, operculated 4 suckers-pigmented 4 suckers
y
y Larvae: Coracidium, Procercoid larva and Plerocercoid larva Head without Head with
y
y Definitive host: Human beings rostellum and rostellum armed
y
y Intermediate host: hooklets with double row of
y
 First–Cyclops hooklets

 Second–Fresh water fish
(Contd...)

Chapter 44     Helminthology Cestodes
Features T.saginata T.solium
Neck Long and thin Short and thick
Proglottids 1000-2000 800-900
Vaginal sphincter Present Absent
Ovary Two without Two with accessory
accessory lobes lobes
Clinical Mostly asymptomatic Larval form causes
manifestations intestinal taeniasis; intestinal taeniasis;
presentation may be Egg causes
abdominal pain and cysticercosis;
nausea

Egg of Tapeworm Figure 1:  Figure showing egg of Taenia


y y Bile stained (Courtesy: CDC/Dr. Mae Melvin)
y Outer–thin layer
Infective Form
y

y y Inner–oncosphere
y y Embryo–Hexacanth embryo y Egg of T. saginata–infective to cattle only
y

y y Egg of T. saginata and T. solium cannot be differentiated y Egg of T. solium–infective to pigs and human beings
y

y Larvae: Cysticercus bovis (T.saginata) and Cysticercus


y

cellulosae (T.solium)–infective to human beings

Figure 2:  Life cycle of Taenia solium

Cysticercosis Diagnostic Criteria for Human Cysticercosis


y Fatal systemic condition seen in T.solium
y
(Source: Modified from Harrison`s principles of internal
y Larvae is seen in muscle, subcutaneous tissue, eyes and brain
y
medicine–19th edition vol 2–Page 1431)
y Neurocysticercosis–most common cause for seizures in
y

India Absolute Criteria


y Three important features of neurocysticercosis are:
y

y Demonstration of cysticerci in histology or microscopy


y

 Convulsions


y Direct visualization of the organism in fundoscopy in the eye


y

 Intracranial hypertension


y Neurological features of cystic lesions as seen in radiography


y

 Psychiatric disturbances


y Other areas are: Thigh muscles, ocular cysticercosis


y

Major Criteria
y Diagnosis: CT, MRI-calcifications; Serological tests are most
y Radiological features suggestive of Neurocysticercosis (NCC)
y

helpful
y

y Demonstration of antibodies in ELISA for cysticerci


y Treatment: Praziquantel, Albendazole, Niclosamide
y

327
y
y Spontaneous resolution or resolution following therapy of the Echinococcus Granulosus
Unit 4     Parasitology

cystic lesions
y y Also known as dog tape worm; it is a zoonotic disease
y Definitive host: Dog
Minor Criteria
y

y y Intermediate host: Sheep, cattle, humans


y y Lesions similar to NCC in imaging studies y y Infective form: Eggs
y y Clinical features suggestive of NCC
y y Demonstration of antigens or antibodies in CSF for cysticerci Hydatid Cyst: (Larval Stage of the Tapeworm)
y Any evidence of cysticerci outside CNS
y It represents the scolex of adult worm
y

y Outer layer–Ectocyst–Cuticular layer


Table 3:  Lab diagnosis of Taenia infections y

y Inner layer–Endocyst–Germinal layer


y

Taenia solium Taenia saginata y Brood capsules are formed from germinal layer
y

• Intestinal taeniasis–
• • Intestinal taeniasis–

y When brood capsules are not formed–called as acephalocysts
y

diagnosed by demonstration diagnosed by demonstration y This causes unilocular hydatid disease


y

of gravid proglottids in the of gravid proglottids in the y Hydatid fluid:


y

 Antigenic in nature and highly toxic; if when secreted into


feces; feces;


the human body by rupture of cysts–it leads to anaphylaxis


• As eggs are morphologically • As eggs are morphologically
 Hydatid sand indicates brood capsules and protoscolices
• •

similar with T.saginata similar with T.saginata




which are floating in the hydatid fluid gives an appearance


it does not help in it does not help in of sand grains;
confirmatory diagnosis confirmatory diagnosis y Hydatid cyst can occur anywhere in the body; M/c is liver
y

• Neurocysticercosis–

followed by lung
diagnosed by imaging Eggs of both Taenia y Lab diagnosis:
y

 Wet mount of the hydatid fluid: Demonstration of proto-


methods like CT and MRI saginata and solium can be 

scolices
which shows the number, differentiating by acid fast
 Casoni’s test–immediate hypersensitivity test
site and stage of cysticerci staining


 Antigen detection by ELISA


(calcifications)


Eggs of T.saginata are acid  Antibody detection has low sensitivity


• CT is best method



fast  X-ray–helps in diagnosis of lung hydatid cyst, USG–helps


• Serodiagnosis like ELISA in diagnosis of liver and other organs


Eggs of T.solium are non acid

helps to identify the fast


antibodies in serum and CSF
• Postmortem diagnosis by

demonstration of cysticerci
in the brain biopsy tissue
Treatment: Treatment:
• Praziquantel
• • Praziquantel

• Niclosamide
• • Niclosamide

• Albendazole

• Praziquantel is the ideal


DOC + needs to be given


with steroids to avoid
inflammatory reactions Figure 3:  Hydatid cyst
caused by dead cysticerci (Courtesy: CDC/Dr Mae Melvin)
y Main modality of treatment is surgical removal of the cyst
y

Echinococcus followed by albendazole or mebendazole for a minimum of


three months
Table 4:  Species of Echinococcus
Species Disease Echinococcus Multilocularis
Echinococcus granulosus Hydatid disease or Cystic y Definitive host–foxes
y

echinococcosis y Intermediate host–Rodents


y

y Accidental host–Humans
Echinococcus multilocularis Alveolar echinococcosis or
y

y Mode of infection–humans ingesting food that is contami-


malignant hydatid disease
y

nated with feces of foxes


Echinococcus oligarthus y It causes multilocular cysts in liver–that does not have
y

Polycystic echinococcosis membrane and brood capsules–because of that the larvae


Echinococcus vogeli
328 spreading and causes liver necrosis
y Because of the spread of larvae to other sites like brain–it is

Chapter 44     Helminthology Cestodes


y

called as malignant hydatid disease

Table 5:  Differences between E.granulosus and


E.multilocularis
Echinococcus granulosus Echinococcus multilocularis
• Larvae has unilocular cyst
• • Larvae has multilocular cyst

• Definitive host–Dogs
• • Definitive host–Fox

• Intermediate host–Humans,
• • Intermediate host–Humans

sheep, cow and oriental rodents


• Causes hydatid cyst
• • Causes malignant hydatid

disease
Figure 4: Egg of Hymenolepis nana
(Courtesy: CDC/Dr. Moore)
Hymenolepis Species
y Two important species that cause infections in humans are:
y
Hymenolepis Diminuta
 Hymenolepis nana
y Usually affects rats and mice; humans are affected rarely


 Hymenolepis diminuta
y

y It is called as rat tapeworm




y Definitive host: Rats, Mice, Humans


Hymenolepis Nana
y

y y Intermediate host: Cockroach, beetles, flea


y The smallest intestinal cestode that infects human is H.nana
y y y Infection is most common in children; asymptomatic
y Hence it is called as dwarf tapeworm
y y y Demonstration of eggs in stool is diagnostic:
y Definitive host: Humans, rats and mice
y  Round in shape and larger than H.nana


y Intermediate host: Not needed


y  Polar filaments are absent


y Mode of infection: Ingestion of eggs


Table 6:  Differences between H.nana and H.diminuta
y

y The eggs that gets released from the host may hatch out in
y

the intestinal lumen and releases the embryo–which invades Hymenolepis nana Hymenolepis diminuta
the intestinal villi–it is called as internal autoinfection; Due • Egg is smaller in size
• • Egg is larger than nana

to bad hygiene–by feco oral route–own eggs cause infection • Polar filaments are present
• • Polar filaments are absent

called as external autoinfection;


y It most commonly affects children; mostly asymptomatic;
Dipylidium Caninum
y

Heavy infections may cause diarrhea or abdominal pain


y Egg of H.nana in stools is the identification method
y
y Parasite of dogs, cats and foxes
y

 Colorless, non bile stained



y It is called as double pored dog tapeworm
y

 Outer membrane is thin and inner membrane is embryo-



y Definitive host: Dogs, cat and humans (accidental)
y

phore which encloses an oncosphere with three pairs of y Intermediate host: Flea and louse
y

hooklets y Diagnosis is by: Demonstration of egg capsules that has eggs


y

 Thread like polar filaments is present on each two poles



in stools
y Treatment: Praziquantel is the DOC
y
y DOC: Niclosamide
y

329
MULTIPLE CHOICE QUESTIONS
Unit 4     Parasitology

1. Definitive host for Taenia saginata: 14. The following is the ovum of an helminth. What is true
(Recent pattern 2017) about helminth? (AIIMS 2017)
a. Man b. Pig
c. Cattle d. Cow
2. Man is both intermediate and definitive host for:
(Recent pattern 2018)
a. T. solium b. T. saginata
c. D. latum d. Dicroftis hominis
3. Consumption of uncooked pork is likely to cause which
of the following helminthic diseases: (AIIMS 2017)
a. Taenia saginata b. Taenia solium
c. Hydatid cyst d. Trichuris trichura
4. Commonest parasite of CNS in India is:
(Recent pattern 2018)
a. Schistosomiasis b. Cysticercosis
c. Trichinella spiralis d. Hydatid cyst
5. Intermediate host for hydatid disease:
(AIIMS 2017)
a. Man b. Dog a. Transmission is through ingestion of infected pork
c. Cat d. Foxes b. Both adult and larval stage are seen in humans
6. Most common site for hydatid cyst: c. The helminth causes a transient self-resolving infection
(Recent pattern 2018) in humans
a. Lung b. Liver d. Drug of choice for this condition is albendazole
c. Brain d. Kidney 15. Hydatid sand contains: (Recent pattern Nov 2017)
7. Least common site of calcified hydatid cyst is: a. Protoscolices
(Recent pattern 2018) b. Brood capsule
a. Lung b. Mediastinum c. Ectocyst
c. Extraperitoneal site d. Liver d. Germinal layer
8. The following infection resembles malignancy: 16. Adult worm of Echinococcus is found in:
(Recent pattern 2017) (Recent pattern 2018)
a. Echinococcus granulosa a. Dog b. Humans
b. E. multilocularis c. Domestic animals d. Felines
c. E. vogeli 17. All the following spread by auto infection except?
d. E. oligarthus  (Recent pattern Nov 2017)
9. Arc C-5 in countercurrent electrophoresis of serum is a. Taenia solium b. Strongyloides stercoralis
diagnostic of: (Recent pattern 2017) c. Hymenolepis nana d. Hymenolepis diminuta
a. Cysticercosis b. Cryptococcosis 18. Drug of choice for treatment of following egg: 
c. Hydatidosis d. Brucellosis  (AIIMS May 2018)
10. Hydatid disease is caused by: (Recent pattern 2018)
a. Echinococcus b. Tapeworm
c. Ascaris d. Clonorchis
11. Dwarf tapeworm refers to: (Recent pattern 2017)
a. Echinococcus
b. Loa loa
c. Hymenolepis nana
d. Schistosoma mansoni
12. All are true about Taenia solium except:
(AIIMS 2017)
a. Egg of T.solium causes neurocysticercosis (NCC)
b. Egg of T.solium is acid fast
c. Praziquantel is the DOC of NCC
d. NCC is the most common cause of seizures in India
13. All the following spread by auto infection except?
(Recent pattern 2018)
a. Taenia solium b. Strongyloides stercoralis a. Albendazole b. Praziquantel
330
c. Hymenolepis nana d. Hymenolepis diminuta c. Niclosamide d. Pyranteol pamoate
ANSWERS AND EXPLANATIONS

Chapter 44     Helminthology Cestodes


1. Ans.  (a) Man 8. Ans.  (b) E. multilocularis
Ref: T.B of Medical Parasitology–S.C.Parija–4th edition– Ref: T.B of Medical Parasitology–S.C.Parija–4th edition–
Page 191 Page 208

Species Definitive host Intermediate host • E. multilocularis causes multilocular cysts in liver–that

does not have membrane and brood capsules–because


Taenia saginata Humans Cattle of that the larvae spreads and causes liver necrosis
Taenia solium Humans Pigs, humans • Because of the spread of larvae to other sites like brain–

it is called as malignant hydatid disease


2. Ans.  (a) T.solium
9. Ans.  (c) Hydatidosis
Ref: T.B of Medical Parasitology–S.C.Parija–4th edition–
Page 195 Ref: T.B of Parasitology–Chaterjee–12th edition–Page 127
• Already explained Q.1
• • Serological tests are helpful in diagnosis of hydatid

disease
3. Ans.  (b) Tinea solium • Tests that are done are:

ƒ Indirect haemagglutination test


Ref: T.B of Medical Parasitology–S.C.Parija–4 th
edition– ƒ

ƒ Immunodiffusion in gel
Page 195 ƒ

ƒ ELISA
• Consumption of uncooked or undercooked pork–
ƒ

ƒ Western blot

infection of T.solium
• Consumption of uncooked or undercooked beef –

10. Ans.  (a) Echinococcus


infection of T.saginata
Ref: T.B of Parasitology–Chaterjee–12th edition–Page 201
4. Ans.  (b) Cysticercosis • Echinococcus granulosus is the causative agent for hy-

Ref: T.B of Medical Parasitology–S.C.Parija–4 th


edition– datid cyst
Page 197 • Hence it is also called as hydatid tape worm

• Cysticercosis is systemic disease caused by T.solium


11. Ans.  (c) Hymenolepis nana


• Neurocysticercosis is the most serious form seen in

brain Ref: T.B of Medical Parasitology–S.C.Parija–4th edition–


• It is the most commonest cause of seizures in India

Page 209
• Hymenolepis nana is the smallest cestode that causes

5. Ans.  (a) Man infection in humans


Ref: T.B of Medical Parasitology–S.C.Parija–4th edition– • Hence it is called as dwarf tapeworm

Page 203
12. Ans.  (b) Egg of T. solium is acid fast
• Definitive host for Echinococcus is Dogs and wild

carnivores Ref: T.B of Medical Parasitology–S.C.Parija–4th edition–


• Intermediate host is humans and domestic animals

Page 198-199
• Egg of Taenia solium and Taenia saginata are

6. Ans.  (b) Liver morphologically similar


Ref: T.B of Medical Parasitology–S.C.Parija–4th edition– • It can be differentiated by acid fast staining

Page 204 ƒ Egg of Taenia solium–non acid fast


ƒ

ƒ Egg of Taenia saginata–acid fast


• Cystic Echinococcosis caused by E.granulosis presents
ƒ

as hydatid cyst in organs 13. Ans.  (d) Hymenolepis diminuta


• Most common site for hydatid cyst is liver followed lung

• Other sites are spleen, heart, kidney, brain and bone



Ref: T.B of Medical Parasitology–S.C.Parija–4th edition–
Page 213
7. Ans.  (a) Lung • Eggs or larvae gets entered into the body from one cavity

• Complication of hydatid cysts are rupture and calcifica-


• to another and causes auto infection
tion • Parasites causing auto infection are:

• Among the cysts, mediastinal cysts undergo calcification


• ƒ Enterobius vermicularis
ƒ

most commonly and pulmonary cysts never undergo ƒ Taenia solium


ƒ

calcification mostly ƒ Strongyloides stercoralis


ƒ

331
ƒ Capillaria philippinensis 16. Ans.  (a) Dog
Unit 4     Parasitology

ƒ Hymenolepis nana
Ref: T.B of Medical Parasitology – S.C.Parija – 4th edition –
ƒ

ƒ Cryptosporidium parvum
Page 203
ƒ

14. Ans.  (b) Both adult and larval stage are seen in humans • Definitive host for Echinococcus is Dogs and wild

carnivores – adult worms live in definitive host


Ref: T.B of Medical Parasitology–S.C.Parija–4 th
edition–
• Intermediate host is humans and domestic animals
Page 209

••The given image shows egg of H.nana 17. Ans.  (d) Hymenolepis diminuta
• It does not need an intermediate host
Ref: Medical Parasitology – Arora – 3rd edition – Page 261

••Infections is acquired by ingestion of eggs


••Both adult and larval stage are seen in definitive host– • Eggs or larvae gets entered into the body from one cavity

humans to another and causes auto infection


• Treatment is praziquantel

• Parasites causing auto infection are:

ƒ Enterobius vermicularis
ƒ

15. Ans.  (a) Protoscolices ƒ Taenia solium


ƒ

ƒ Strongyloides stercoralis
Ref: T.B of Medical Parasitology–S.C.Parija–4th edition–
ƒ

ƒ Capillaria philippinensis
Page 202
ƒ

ƒ Hymenolepis nana
ƒ

• Hydatid cyst has three layers–Pericyst, Ectocyst and



ƒ Cryptosporidium parvum
ƒ

Endocyst
• Pericyst–outer most layer contains blood vessels and
• 18. Ans.  (b) Praziquantel
fibroblasts
Ref: T.B of Medical Parasitology–S.C. Parija–4th edition–
• Ectocyst–acellular, laminated, hyaline layer
Page 209 


• Endocyst–Germinal layer which gives rise to brood


capsules and scolices on inside • Above image clearly shows the following findings:

• When the embryos gets released from this layer and



ƒ Colorless, non bile stained
ƒ

floats freely in the fluid–it is called as hydatid sand–it has ƒ Outer membrane is thin and inner membrane
ƒ

daughter cysts with scolices. is 
embryophore which encloses an oncosphere with


three pairs of hooklets
• Thread like polar filaments is present on each two poles 


• Hence the egg is identified as Hymenolepis nana


• Treatment: Praziquantel is the DOC.


332
45
Trematodes
INTRODUCTION
y Trematodes have characteristic suckers
y
y They are also called as flukes
y
y All the trematodes have definitive host and two intermediate
y
hosts
 Definitive host: Humans and animals

 First intermediate host: Fresh water snail or molluscs

 Second intermediate host: Fish or Crab

y Infection with trematodes is mainly due to two mechanisms
y
Penetration of skin by Ingestion of metacercariae
cercariae
• Schistosoma (all species) • Clonorchis sinensis (Fish)


• Paragonimus westermani (Crab)
Figure 1: Egg of Schistosoma haematobium

• Fasciola hepatica (Vegetables or
(Courtesy: CDC/Dr D.S. Martin)

aquatic plants)
• Fasciolopsis buski (Vegetables or
Clinical Features

aquatic plants)
All trematodes have operculated eggs except Schistosoma sp., y A pruritic rash appears at the site of penetration
y
y Eggs get accumulated in the bladder–leads to inflammation
y
and egg granuloma in the ureter and bladder
SCHISTOSOMA SPECIES y This inflammatory granuloma leads to fibrosis and obstruc-
y
y All Schistosoma species are unisexual tion in the urinary tract
y
y Major species are: y Because of this continuous irritation–it leads to malignant
y
y
 S. haematobium tumors in the urinary bladder (Squamous cell carcinoma)

 S. mansoni y Acute infection causes–Katayama syndrome–fever, itching

y
 S. japonicum and lesions at the penetration site;

y Chronic infection leads to terminal hematuria, dysuria
y
Schistosoma Haematobium named as Endemic hematuria

y Major blood fluke that affects humans


Diagnosis
y
y The fluke live in the venous plexus that drains the urinary
y
bladder, ureter and pelvis y Nonoperculated, terminal spine egg seen in urine sediments
y
y Infective form is Cercariae–that enters the body through y Most sensitive and specific is–Bladder mucosal biopsy
y
y
skin penetration y Serological test is helpful in chronic cases and epidemiological
y
y Definitive host: Humans studies
y
y Intermediate host: Fresh water snails y Skin test used–Fairley`s test
y
y
y After entering the epidermis–the larvae spreads to the y DOC–Praziquantel and Metrifonate
y
y
capillaries in lung–liver–where they develop into adult worms
Table 1:  Clinical manifestation of various Schistosoma
y Adult worms moves to vesical venous plexus and lay eggs–
species
y
these eggs are passed in the urine
y Characteristics of Egg: Schistosoma species Clinical manifestation
y
 Eggs are oval shaped, yellowish brown in color S.haematobium Haematuria and dysuria

 Nonoperculated S.mansoni Dysentery

 Terminal spine seen in the posterior side is the unique
S.japonicum Katayama syndrome

key for identification
S.intercalatum Intestinal disorders
S.mekongi Hepatomegaly
Schistosoma Mansoni y Treatment: It is more resistant for treatment compared to
Unit 4     Parasitology

other species; Praziquantel is safe and effective;


y It causes intestinal schistosomiasis; also called as bilharziasis
y

y Adult worm lives in the mesenteric venous plexus draining


y
Table 2:  Identification of schistosoma eggs
the intestine
y Cercariae is the infective form
y
Schistosoma haematobium Terminal spine
y Heavy infection is seen in the age group of 10 to 14 years
y
Schistosoma mansoni Lateral spine
y Clinical features are:
y
Schistosoma japonicum Lateral small knob
 Katayama fever (M/c seen)


 Dysentery


FLUKES
 Symmer’s pipestem fibrosis (periportal fibrosis)
Fasciola Hepatica


 Portal hypertension


 Transverse myelitis


y y Common liver fluke


y Diagnosis is by characteristic demonstration of eggs in fecal
y

y y Definitive host: Sheep, humans


sample or rectal biopsy (most sensitive and specific is rectal y y Intermediate host: Snails
biopsy): y y Infective form: Metacercariae (aquatic plants that has this
 Nonoperculated egg


metacercariae is the source of infection)


 Yellowish brown


y y Pathogenesis is due to adult worm and migrating larvae


 Lateral spine seen near to the rounded posterior end


y y It causes fascioliasis
 Hepatomegaly, biliary obstruction, jaundice occurs


 Acute fascioliasis is due to migrating larvae




 Chronic fascioliasis is due to adult worms in bile ducts




y y Halzoun is a syndrome that occurs due to eating raw liver–


adult worm enters the posterior pharyngeal wall and causes
air way obstruction that leads to death
y y Diagnosis is by demonstration of egg: Bile stained, operculated
with a large unsegmented ovum

Figure 2: Egg of Schistosoma mansoni


(Courtesy: CDC)
y DOC–Praziquantel
y

Schistosoma Japonicum
y It causes oriental schistosomiasis; it is the most severe form
y

y Adult worms live in the superior mesenteric plexus


y

y Clinical features are:


y

 Katayama syndrome (M/c)




 Periportal fibrosis
 Figure 4: Egg of Fasciola hepatica
 Pulmonary hypertension
 (Courtesy: CDC)
 CNS schistosomiasis


y Serodiagnosis are of most helpful–ELISA is best method


y

y Diagnosis is by characteristic eggs:


y

y Serology is helpful as antibodies are positive before the eggs


y

 Oval shaped, bile stained egg




can be demonstrated in the stool sample


 Lateral knob seen


y Treatment: Bithionol; Triclabendazole


y

Table 3:  Habitat of fasciola species


Organism Habitat
Fasciola hepatica Bile tract
Fasciola gigantica Bile tract
Fasciolopsis buski Duodenum and jejunum

Fasciolopsis Buski
y Giant intestinal fluke–because it is the largest intestinal fluke
y

y Fluke lives in the duodenum and jejunum


y

y It is the most common intestinal trematode that causes


334
y

Figure 3: Egg of Schistosoma japonicum infections in humans


(Courtesy: CDC/Dr Moore) y Definitive host: Pigs and humans
y
y First Intermediate host: Snails Paragonimus Westermani

Chapter 45     Trematodes


y

y Second intermediate host: Fresh water plants


y It is also called as Oriental lung fluke
y

y Mode of infection: Ingestion of fresh water aquatic plants


y

y Definitive host–Man
y

y It causes fasciolopsiasis–worms causes obstruction and


y

y First intermediate host–Snails


y

ulceration
y

y Second intermediate host–Fresh water crab or crayfish


y Vit B12 malabsorption occurs
y

y Infective form–Metacercariae
y

y Demonstration of operculated eggs–helps in diagnosis


y

y Cases have been reported in Assam, Bengal, Tamil Nadu and


y

y Eggs of F.buski and F.hepatica are not distinguishable


y

Kerala in India
y

y Treatment: Praziquantel; Niclosamide


y Causes paragonimiasis in pulmonary system; it causes a
y

granulomatous reaction that leads to blood mixed sputum–


Remember consists of golden brown eggs; a fibrous tissue that may go for
Vitamin B12 Malabsorption cavitation in some cases;
• Diphyllobothrium latum (Cestode)

yy A patient with hemoptysis should be evaluated for P.westermani
• Fasciolopsis buski (Trematode)

also
y y Diagnosis is by demonstration of eggs in sputum
y y Treatment: Praziquantel

Figure 5: Egg of Fasciolopsis buski


(Courtesy: CDC/Dr Mae Melvin)

Remember
Parasites causing malignancy are: Figure 6: Eggs of trematodes
• Schistosoma haematobium–Bladder cancer
• (Courtesy: CDC/Dr Mae Melvin)
• Clonorchis sinensis–Cholangiocarcinoma
Remember

• Opisthorchis felineus–Cholangiocarcinoma

List of operculated eggs:


• Diphyllobothrium latum
Clonorchis Sinensis

• Fasciola hepatica

y Also called as Chinese or Oriental liver fluke


y
• Fasciola gigantica

y It lives in the biliary passage and pancreatic duct • Fasciolopsis buski


• Clonorchis sinensis
y

y Definitive host–Man, pig, dog or cat •

• Paragonimus westermanii
y

y First intermediate host–Snails •

• Gastrodiscoides hominis
y

y Second intermediate host–Fresh water fish •

• Watsonius watsoni
y

y Infective form–Metacercariae •

• Opisthorchis felineus
y

y The fluke proliferates in the biliary epithelium of bile ducts– •

• Opisthorchis viverrini
y

leads to dilatation of bile ducts


• Heterophyes heterophyes
y It is associated with:

• Metagonium yokogawai
y

 Bile duct carcinoma (cholangiocarcinoma)


 Pancreatic carcinoma


 Cholangitis

Table 4:  Summary of treatment
 Cholangiohepatitis


Name of trematode Drug of choice


y Lab diagnosis is by demonstration of eggs:
Blood fluke: Schistosomes (all species) Praziquantel
y

 Eggs are bile stained


Biliary fluke: Clonorchis sinensis Praziquantel


 Has a convex operculum


Opisthorchis viverrini


 A small knob is seen at the posterior end




 But egg of C.sinensis cannot be distinguished from



Liver fluke: Fasciola hepatica Ticlabendazole
Š Heterophyes heterophyes Fasciola gigantica
335
Š

Š Opisthorchis spp.,
Š

Intestinal fluke: Fasciolopsis buski Praziquantel


Š Metagonimus yokogawai
Š

Heterophyes heterophyes
y Treatment: Praziquantel
y

Lung fluke: Paragonimus westermani Praziquantel


Unit 4     Parasitology

MULTIPLE CHOICE QUESTIONS


1. Painless terminal hematuria is seen as one of the 4. Which is not a liver fluke: (Recent pattern 2018)
manifestations in the infection caused by: a. Paragonimus b. Whipworm
(Recent pattern 2017) c. Clonorchis sinensis d. Gnathostoma spinigerum
a. Schistosoma Japonicum e. Opisthotrichus
b. Schistosoma mansoni 5. Clonorchis sinensis infection is due to ingestion of:
c. Schistosoma haematobium (Recent pattern 2017)
d. Plasmodium falciparum a. Fish b. Pork
2. Katayama fever is caused by: (Recent pattern 2018) c. Snail d. Beef
a. F. hepatica b. C. sinensis 6. Clonorchis sinensis is: (Recent pattern 2017)
c. S. haematobium d. A. lumbricoides a. Tape worm b. Round worm
3. A man on return from a country complains of pain in ab- c. Thread worm d. Fluke
domen, jaundice, with increased alkaline phosphatase 7. Which of the following is a risk factor for bladder
and conjugated hyperbilirubinemia. USG shows block- carcinoma: (Recent pattern 2017)
age in biliary tree what could be the cause? a. Clonorchis sinensis
(AIIMS 2018) b. Opisthorchis sp
a. Fasciolopsis buski b. Clonorchis sinensis c. Schistosoma haematobium
c. Strongyloides d. Ancylostoma d. Fasciola hepatica

8. Which of the following parasite’s life cycle is shown below? (AIIMS 2017)

a. Fasciolopsis b. Fasciola hepatica


336 c. Paragonimus westermani d. Clonorchis sinensis
9. All of the following have operculated eggs except? 10. Following type of eggs are seen in:

Chapter 45     Trematodes


(Recent pattern Nov 2017) (Recent pattern 2017)
a. Clonorchis sinensis
b. Diphyllobothrium latum
c. Hymenolepis diminuta
d. Paragonimus westermanii

a. Schistosoma haematobium
b. Schistosoma mansoni
c. S.japonicum
d. Clonorchis sinensis

ANSWERS AND EXPLANATIONS


1. Ans.  (c) Schistosoma haematobium 4. Ans.  (a) Paragonimus; (b) Whipworm; (d) Gnathostoma
spinigerum
Ref: T.B of Medical Parasitology–S.C.Parija–4th edition–
Page 222 Ref: T.B of Medical Parasitology–S.C.Parija–4th edition–
• Painless terminal hematuria is a feature of urinary

Page 235
schistosomiasis • Liver flukes are:

• Urinary schistosomiasis is caused by S.haemotobium


• ƒ Fasciola hepatica
ƒ

• Chronic urinary infections lead to squamous cell


• ƒ Clonorchis sinensis
ƒ

bladder carcinoma ƒ Opisthorchis felineus


ƒ

2. Ans.  (c) S.haematobium 5. Ans.  (a) Fish


Ref: T.B of Medical Parasitology–S.C.Parija–4 th
edition– Ref: T.B of Medical Parasitology–S.C.Parija–4th edition–
Page 222, 227 Page 252
• Katayama fever is caused by any of the species of
• • Second intermediate host is fish

Schistosoma • The infection is due to ingestion of metacercaria from


• The most common causative pathogen in order are:


• inadequately cooked freshwater fish
ƒ S.mansoni
ƒ

ƒ S.japonicum
ƒ
6. Ans.  (d) Fluke
ƒ S.haematobium
ƒ

Ref: T.B of Medical Parasitology–S.C.Parija–4th edition–


• Fever, myalgia, arthralgia are the presenting symptoms

Page 252
in katayama fever
• Clonorchis sinensis belongs to trematodes inturn called

3. Ans.  (b) Clonorchis sinensis as flukes


• It is also called as Chinese liver fluke

Ref: T.B of Medical Parasitology–S.C.Parija–4th edition–


Page 252 7. Ans.  (c) Schistosoma haematobium
• Clonorchis sinensis is called Chinese liver fluke

Ref: T.B of Medical Parasitology–S.C.Parija–4th edition–


• The fluke proliferates in the biliary epithelium of bile

Page 222
ducts–leads to dilatation of bile ducts
• It is associated with:

• Already explained Q.1

ƒ Bile duct carcinoma (cholangiocarcinoma)


8. Ans.  (b) Fasciola hepatica
ƒ

ƒ Pancreatic carcinoma
ƒ

ƒ Cholangitis
ƒ Ref: T.B of Parasitology–Chaterjee–Page 187
ƒ Cholangiohepatitis
ƒ

• Fasciola hepatica: Sheep liver fluke comes under


Trematodes/flukes

337
• Adult worms reside in the biliary passages of the liver of ƒ Fasciolopsis buski
Unit 4     Parasitology

• ƒ

sheep, goat, cattle and man ƒƒ Clonorchis sinensis


• Infective form: Metacercariae encysted on water plants
• ƒƒ Paragonimus westermani
that are ingested by humans ƒƒ Gastrodiscoides hominis
• Definitive host: Sheep, goat, cattle and man
• ƒƒ Watsonius watsoni
• First intermediate host: Snails
• ƒƒ Opisthorchis felineus
• Second intermediate host: Aquatic vegetations
• ƒƒ Opisthorchis viverrini
ƒƒ Heterophyes heterophyes
9. Ans.  (c) Hymenolepis diminuta ƒƒ Metagonium yokogawai
Ref: Medical Parasitology–Arora–3 edition–Page 126,151
rd
10. Ans.  (a) Schistosoma haematobium
• In cestodes–the only helminth that has operculated egg

is Diphyllobothrium latum; It is a bile stained egg Ref: Medical Parasitology–Arora–3rd edition–Page 222
• In trematodes–all of them lay operculated eggs except
• • Characteristics of Egg:

Schistosomes ƒ Eggs are oval shaped, yellowish brown in color


ƒ

• List of operculated eggs:


• ƒ Nonoperculated
ƒ

ƒ Diphyllobothrium latum
ƒ ƒ Terminal spine seen in the posterior side is the unique
ƒ

ƒ Fasciola hepatica
ƒ key for identification
ƒ Fasciola gigantica
ƒ

338
46
Nematodes
INTRODUCTION Lymphatic nematodes • Wuchereria bancrofti


y Nematodes are cylindrical worms and looks elongated they • Brugia malayi


y
are called as roundworms. Conjunctival • Loa loa


Smallest nematode • Trichinella spiralis nematodes

• Strongyloides stercoralis

Largest nematode • Dracunculus medinensis TRICHINELLA SPIRALIS

y Based on the habitat of the nematodes – they are divided into y This worm is a common parasite of pig–causes infection to
y
y
humans (zoonosis)
Table 1:  Classification of Nematodes y Life cycle gets completed in a single host–Pig; Accidental host
y
Intestinal nematodes Small intestine: are humans
• Ascaris lumbricoides
Adult worm Lives in intestine

• Ancylostoma duodenale

• Necator americanus Larvae Lives in striated muscles

• Strongyloides stercoralis
y Infection is acquired by ingestion of encysted larvae which is

• Trichinella spiralis
y
present in raw pork or insufficiently cooked pork

• Capillaria philippinensis

Large intestine: y This larva enters the duodenum and develop into adult male
y
• Enterobius vermicularis and female within two days

• Trichuris trichiura y Gravid female directly gives rise to larva (No Egg Stage)

y
Subcutaneous tissue • Dracunculus medinensis y This larvae goes to muscles of tongue, mastication, intercostal
y

nematodes • Onchocerca volvulus muscle, eye muscles, thigh muscles and get encysted

• Loa loa y After few months–it goes for calcification after death
y

• Mansonella perstans y Infection in human is a dead end

y
• Mansonella ozzardi

(Contd...)

Figure 1: Illustration showing life cycle of Trichinella spiralis


y The larvae that are seeding in the muscles causes myositis–
Unit 4     Parasitology

leading to myalgia; periorbital edema and facial puffiness


y Myocarditis, bronchopneumonia, vascular thrombosis and
y

encephalitis occurs at later stage


y Lab diagnosis:
y

 Ideal specimen: Muscle tissue by biopsy




 Serological tests: ELISA, Bentonite flocculation test, West-




ern blot
 Antigen is named as Bachman antigen


Figure 3: Egg of Trichuris trichiura


Courtesy: CDC/B.G. Partin

Remember
Mucous plugs are seen in:
• Trichiruis trichiura

• Capillaria philippinensis

• Gnathostoma spinigerum

y Treatment: Mebendazole
y

y DD for Trichuris is Capillaria philippensis


y

Figure 2: Encystation of Trichinella spiralis larvae in muscle


(Courtesy: CDC)
y Treatment
y

 Thiabendazole


 Mebendazole


Remember A B
Soil transmitted helminths (STH) are:
Figures 4 A and B: Trichuris trichiura and Capillaria philippensis
• Ascaris lumbricoides

(Mucous plugs are prominent in trichuris;


• Hook worm

Size of Capillaria is smaller)


• Trichuris trichiura

• Strongyloides stercoralis
STRONGYLOIDES STERCORALIS

y This worm is also called as dwarf threadworm


Trichuris Trichiura
y

y Unique characteristic features:


y

y The worm has a characteristic whip like shape–hence called


y
 It has both parasitic and free living forms


as Whip worm  No male worm




y It lives in the large intestine


y
 Females reproduce itself and produce eggs


y Definitive host: Humans (the only host)


y
 Eggs give rise to larvae and develop into filariform larvae


y Infection is acquired by ingestion of soil with embryonated


y
y Infective form–Filariform larvae (enters by skin penetration)
y

eggs (has rhabditiform larvae) y Life cycle of Strongyloides:


y

y It usually affects children and remains asymptomatic


y
 Parasitic forms enters into human and continues its life


y Heavy infection causes rectal prolapse in children; appendi-


y
 Free living forms enters the soil and develop themselves


citis;  Autoinfection


y Lab diagnosis:
y
y When the larvae enters through the skin–it causes larva
y

 Demonstration of eggs in feces



currens–dermatitis
 Characteristics of egg:

y Larvae then migrates through capillaries–lungs–causing
y

Š Barrel shaped
Š
pneumonitis, bronchopneumonia, eosinophilia
Š Bile stained egg
Š
y Two important manifestations seen in immunocompromised
y

Š Mucous plugs at both ends–which are not bile stained


Š
(AIDS) patients are:
 Hyperinfection syndrome


 Disseminated strongyloidiasis
340 
Table 2:  Diseases caused by S. stercoralis

Chapter 46     Nematodes


Hyperinfection • After an initial infection with S.stercoralis–
Remember
Parasites causing auto infection are:

syndrome chronic infection persists


• Suddenly when that patient goes for • Enterobius vermicularis

• Taenia solium

immunocompromised state–it leads to •

hyperinfection (more multiplication) • Strongyloides stercoralis


• But it does not cause invasion • Capillaria philippinensis


• Hymenolepis nana

• So no extra intestinal manifestation can be •

• Cryptosporidium parvum

seen •

Disseminated • When an immunosuppressed patient


strongyloidiasis acquired infection with S.sterocoralis–it ASCARIS LUMBRICOIDES


leads to dissemination
y This worm is also called as round worm
• It affects extra intestinal organs
y

y Most common infection acquired by humans


• May lead to pulmonary and neurological


y

y Largest intestinal nematode infecting humans


complications
y

y Adult worm lives in the small intestine


y

y Lab diagnosis:
y
y Humans are the only definitive host
y

 Demonstration of rhabditiform larvae from the stool



y Infection is acquired by ingestion of food with embryonated
y

(Not Egg) eggs (only embryonated eggs are infective)


 When eggs are hatching out in the stool–it immediately

y Clinical manifestations:
y

forms rhabditiform larvae–hence from stool specimens  Intestinal ascariasis–few worms usually means asympto-


these are commonly seen matic; Heavy infections leads to:


 But infective form

Š Malabsorption
Š

Š Intussuception
Š

Š Intestinal obstruction
Š

 Pulmonary ascariasis: Due to migrating larvae that enters




the capillaries–leads to Loeffler’s syndrome


 Anemia 

y Lab diagnosis:
y

 Demonstration of eggs in stool samples;




Table 3:  Characteristics of of eggs in stool


Fertilized egg Decorticated egg Unfertilized egg
• Bile stained egg
• • Outer • • Ellipsoidal shape

• Oval to spherical
• albuminous coat • Heaviest of all

shaped is lost here helminthic eggs


• Surrounded by
• • When no male

Figure 5: Filariform larvae of Strongyloides stercoralis a thick outer worm is seen


(Courtesy: CDC/Dr Mae Melvin) albuminous coat in the infected
 Stool culture can be done to identify the larvae by following • A clear crescentic
• person–female
space at each produces


methods:
sides unfertilized egg
Š Harada Mori filter paper method
• Has an
Š

Š Baermann funnel method


unsegmented
Š Agar plate method
Š

ovum
  Enterotest can be done by checking the aspirations for
rhabditiform larvae
  Serodiagnosis by ELISA, IHA
  PCR

Remember
Enterotest is done for:
• Giardia lamblia

• Cystoisospora belli

• Strongyloides stercoralis

y Treatment
y

 Ivermectin is the first DOC for acute and strong strongyloi-




diasis; Second drug is albendazole


 Thiabendazole is the DOC for disseminated strongyloidia-
 Figure 6: Egg of Ascaris lumbricoides (Fertilized egg)
341
sis (Courtesy: CDC/Dr Mae Melvin)
y Demonstration of egg is difficult in following conditions:
Unit 4     Parasitology

Remember
y

 Extra intestinal infections




 When only male worms infect the individual



Hookworm that infect animals and accidentally humans:
y Treatment
y
• Ancylostoma ceylanicum

 DOC–Mebendazole

• Ancylostoma braziliense

 Pyrantel pamoate–causes spastic paralysis of the helminth



• Ancylostoma caninum

 Piperazine citrate–causes flaccid paralysis of the worm–




DOC for intestinal or biliary ascariasis Ancylostoma Duodenale


y It is also called as old world hook worm
ENTEROBIUS VERMICULARIS
y

y y It lives in the small intestine–jejunum


y This worm is also called as threadworm or pin worm
y
y y Humans are the only host
y Most commonly affects children
y
y y Infection is acquired by penetration of the filariform larvae
y Humans are the natural host
y
(third stage) through the skin–causes local reaction called
y Infection is acquired by ingestion of embryonated eggs
y
as Ground itch
through hands; through inhalation of ariborne eggs; y y Once the larvae enters the cavities–it causes cough, pharyn-
y Once eggs come out of human–it cause autoinfection by
y
gitis and dyspnea
hand to mouth transfer y y It then gets moulted to adults in small intestine–where it sucks
y Clinical features:
y
the blood and causes hypochromic microcytic anemia; also
 Perianal pruritus (nocturnal itching)–because the worms

causes hypoproteinemia
come out y y Lab diagnosis:
 Abdominal pain

 Demonstration of eggs in stool


 Excoriation

 To estimate the quantity of eggs–Kato Katz method is help-


y Laboratory diagnosis:
y
ful
 Demonstration of eggs from anal swab/perianal scrapings–

 For culture–Harada Mori culture method is used (Filari-


done by scotch tape swab method using NIH swab form larvae can be grown)
Š Plano convex shaped
Š
y y Treatment:
Š Not bile stained
Š
 Mebendazole


Š Very thin, hyaline egg shell


Š
 Ferrous sulfate for anemia


Š Coiled tad pole like larvae


Necator Americanus
Š

y Also called as new world hook worm


y

y Manifestations are similar to Ancylostoma except that the


y

severity of anemia is less here

Table 4:  Differences between Ancylostoma and Necator


Ancylostoma duodenale Necator americanus
Filariform larvae: Filariform larvae:
• has a conical head
• • has a rounded head

• no gap seen between


• • a gap seen between

esophagus and intestine esophagus and intestine


Adult worm: Adult worm:
• two pairs of claw like teeth
• • one pair of chitinous plates

on anterior side on anterior side


Figure 7: Egg of Enterobius vermicularis
• one pair of knob like teeth • one pair of plates on the
Courtesy: CDC
• •

on posterior side posterior side


y Treatment: Pyrantel pamoate is the DOC; alternate drugs are
y
• copulatory bursa has 13 rays
• • copulatory bursa has 12 rays

mebendazole Egg: Egg:


y All the family members and contacts need to get a course as
y
• oval shaped
• • oval shaped

prophylaxis • non bile stained


• • non bile stained

• surrounded by a thin hyaline


• • surrounded by a thin hyaline

membrane membrane
HOOK WORM • contains blastomeres
• • contains blastomeres

Pathogenesis: Pathogenesis:
Remember • Blood loss is around 0.15–
• • Blood loss is 0.03 mL/day

Hookworm that infect humans: 0.26 mL/day • Eggs released are 5000–

• Ancylostoma duodenale • Eggs released are 10000–


• 10000/day
25000 eggs/day

• Necator americanus

342 • More severe anemia


• • Less severe anemia

Chapter 46     Nematodes
High Yield
Important Points in Intestinal Nematodes:
• All intestinal nematodes are diagnosed by eggs in stool except

Strongyloides stercoralis which passes larvae in stool


• Perianal pruritus is hallmark of Enterobius vermicularis

• Iron deficiency anemia is seen in hook worm


• Biliary obstruction in heavy infections are seen with Ascaris


lumbricoides
• Small bowel infection is seen with round worm and hook

worm, Strongyloides.
• Large bowel infection is seen with Whip worm and Pin worm.

Table 6:  Differences between cutaneous and visceral


Figure 8: Egg of hookworm (Egg of Ancylostoma duodenale and
Necator americanus cannot be differentiated)
larva migrans
(Courtesy: CDC) Cutaneous larva migrans Visceral larva migrans
Table 5:  Miscellaneous nematodes • Ancylostoma braziliense (M/c)
• • • Toxocara canis
• Ancylostoma caninum (second
• • • Toxocara cati
Nematodes Characteristics M/c) • • Gnathostoma
Ancylostoma • Smallest hookworm • Necator americanus
• • • Anisakis
• Ancylostoma duodenale

ceylanicum • Commonly seen in cats


• Gnathostoma spinigerum

• Causes intestinal infection in


• Strongloides
humans

• Loa loa

Ancylostoma • Hookworm of dogs and cats



• Fasciola

braziliense • Causes creeping eruptions in



• Paragonimus

humans (cutaneous larva migrans)


Ancylostoma caninum • Dog hookworm

EOSINOPHILIC MENINGOENCEPHALITIS
• Causes cutaneous larva migrans

y Eosinophilic meningitis is defined as the presence of


y

Trichostrongylus • Also called as pseudo hook worm



more than 10  eosinophils/mm3  in the cerebrospinal fluid
• Egg similar to hook worm

(CSF)  and/or eosinophils accounting for more than 10% of
CSF leukocytes
Angiostrongylus • Rodent or rat lung worm
y Three important parasitic infections associated with eosino-

cantonensis • Causes Eosinophilic meningitis in


y

philic meningitis 

humans
Angiostrongylus • Causes abdominal angiostrongyliasis

• Angiostrongylus cantonensis

costaricensis and eosinophilic gastroenteritis • Baylisascaris procyonis


• Gnathostoma spinigerum

343
MULTIPLE CHOICE QUESTIONS
Unit 4     Parasitology

1. The cause of larva currens: (Recent pattern 2017) 7. Child is having perianal pruritus with following eggs due
a. Strongyloides stercoralis to: (Recent pattern 2017)
b. Necator americanus a. E.vermicularis
c. Ankylostoma duodenale b. Ascaris
d. H.nana c. Ancylostoma duodenale
2. A 40 year HIV positive male patient comes with ody- d. S stercoralis
nophagia and watery diarrhea. An endoscopy reveals 8. Larvae of Ascaris lumbricoides most commonly causes:
esophageal and gastric candidiasis. A wet mount of the (AIIMS 2017)
stool of the patient reveals following picture: a. Cardiac symptoms
(AIIMS 2017) b. Respiratory symptoms
c. Genitourinary symptoms
d. Cerebral symptoms
9. What is the most common clinical manifestation when
larvae of ascaris, hookworm and strongyloides migrates
through the body? (Recent pattern 2017)
a. Asymptomatic
b. Pneumonitis
c. Acute dermal reaction
d. Anemia
10. Cutaneous larva migrans is due to:
(Recent pattern 2017)
a. Ancylostoma braziliensis
b. Wucheria bancrofti
c. Brugia malayi
d. Dracunculus medinensis
a. Filariform larvae is infective for humans as shown in 11. Stool examination in a patient reveals the following
the diagram finding. What is the likely route of infection of this
b. Transmitted through contaminated food and water parasite? (AIIMS 2017)
usually
c. Females of these species show parthogenesis
d. Drug of choice is Triclabendazole
3. Ankylostoma enters the human body by:
(Recent pattern 2018)
a. Ingestion
b. Inhalation
c. Penetration of skin
d. Inoculation
4. Habitat of Nectator americanus: (PGI pattern 2017)
a. Jejunum
b. Ileum
c. Colon
d. Duodenum
e. Caecum
5. Visceral larva migrans is associated with:
(Recent pattern 2017)
a. Strongyloides stercoralis a. Ingestion of food contaminated with egg of larva
b. Ancylostoma braziliensis b. Insect bite
c. Toxocara canis c. Improperly cooked beef
d. Visceral leishmaniasis d. Swimming in dirty water pool
6. The hookworm thrives on: (Recent pattern 2017)
a. Whole blood
b. Plasma
c. Serum
d. RBC
344
12. The following are images of an intestinal nematode. 15. Barrel shaped eggs is/are seen in: (PGI May 2017)

Chapter 46     Nematodes




Which of these are true about it? (AIIMS 2017) a. Hook worm

b. Roundworm

c. Pin worm

d. Whipworm

e. Strongyloides stercoralis

16. Which of the following larva is/are found in stools:




 (PGI May 2018)


a. Strongyloides stercoralis b. Ankylostoma duodenale

c. Ascaris lumbricoides
d. Hymenolepis nana

e. Enterobius vermicularis

17. A patient who had undergone renal transplantation


three months before has presented with bloody


diarrhea. Stool sample shows the following organisms
on culture measuring 280 um. Which of the following is
true among the following: 

 (AIIMS Nov 2018 Video Question)


 For video scan this QR Code

a. Transmitted through percutaneous and autoinocula-


tion
b. Embryonated egg is the infectious stage for this worm

c. The adult worm usually lives in the small intestine


d. Triclabendazole is the drug of choice


13. Cutaneous larva migrans is caused by:


(Recent pattern 2018)


a. Ancylostoma braziliense b. Anisakiasis

c. Necator americanus
d. Ancylostoma catifera

14. A 48-year-old immunocompromised patient attended


the emergency block with complaints of abdominal


pain, vomiting and dyspnea. On examination and
baseline investigation, he was diagnosed as gastro- a. It causes Loeffler’s pneumonia
intestinal perforation with paralytic ileus. The most b. The video shows the filariform larvae of the parasite
common cause of this disseminated infection in c. It is acquired by contamination of food/water
immunocompromised patient is due to the following. d. They are monoecious since they undergo partheno-
Identify it:
 (PGI pattern 2017) genesis
a. Rhabditiform larvae of S. stercoralis

b. Filariform larvae of S. stercoralis


c. Adult female worm of S. stercoralis


d. Adult male worm of S. stercoralis


e. Egg of S. stercoralis

345
ANSWERS AND EXPLANATIONS
Unit 4     Parasitology

1. Ans.  (a) Strongyloides stercoralis 7. Ans.  (a) E.vermicularis


Ref: T.B. of Medical Parasitology–S.C.Parija–4th edition– Ref: T.B. of Medical Parasitology–S.C.Parija–4th edition–
Page 271 Page 287
• Strongyloides infection is acquired by penetration of
• • Clinical features of Enterobius vermicularis infection:

filariform larvae through skin ƒ Perianal pruritis (nocturnal itching) – because the
ƒ

• When it causes skin penetration–it produces localized


• worms come out
dermatitis at that site–leading to itching–named as larva ƒ Abdominal pain
ƒ

currens ƒ Excoriation
ƒ

2. Ans.  (c) Females of these species show parthogenesis 8. Ans.  (b) Respiratory symptoms
Ref: T.B. of Medical Parasitology–S.C.Parija–4 th
edition– Ref: T.B. of Medical Parasitology–S.C.Parija–4th edition–
Page 268 Page 291
• The above image shows Rhabdiform larvae of Strongy-
• • Migrating larvae of Ascaris causes pulmonary symptoms

loides stercoralis • This is the initial stage of infection


• Identification feature:
• • It causes eosinophilic pneumonia–Loeffler’s syndrome

ƒ Unsheathed
ƒ

ƒ Short mouth with double bulbed esophagus


ƒ
9. Ans.  (a) Asymptomatic
• Unique feature of Strongyloides is it does not have male

Ref: T.B of Medical Parasitology–S.C.Parija–4th edition–


worms Page 287–291
• Female worms reproduce themselves by parthenogenti-
• Usually the infections with nematodes are asymptomatic

cally releasing eggs •

when the larval migration occurs


3. Ans.  (c) Penetration of skin • The symptoms are mainly due to adult worm and when

heavy infections persists


Ref: T.B. of Medical Parasitology–S.C.Parija–4th edition–
Page 277 10. Ans.  (a) Ancylostoma braziliensis
• Penetration of skin by filariform larvae is by:

Ref: T.B. of Medical Parasitology–S.C.Parija–4th edition–


ƒ Hookworm
ƒ

Page 294
ƒ Strongyloides
• Cutaneous larva migrans is due to:
ƒ

4. Ans.  (a) Jejunum; (b) Ileum; (d) Duodenum ƒ Ancylostoma braziliense (M/c)
ƒ

ƒ Ancylostoma caninum (second M/c)


ƒ

Ref: T.B. of Medical Parasitology–S.C.Parija–4th edition– ƒ Necator americanus


ƒ

Page 275 ƒ Ancylostoma duodenale


ƒ

•• Adult hookworms live in the lumen of small intestine ƒ Gnathostoma spinigerum


ƒ

•• Most common site is jejunum ƒ Strongloides


ƒ

•• Less commonly in duodenum and ileum ƒ Loa loa


ƒ

•• If needed to choose one option –choose jejunum ƒ Fasciola


ƒ

ƒ Paragonimus
ƒ

5. Ans.  (c) Toxocara canis


11. Ans.  (a) Ingestion of food contaminated with egg of
Ref: T.B. of Medical Parasitology–S.C. Parija–4th edition–
larva
Page 294
• Visceral larva migrans is caused by:

Ref: T.B. of Medical Parasitology–S.C.Parija–4th edition–
ƒ Angiostrongylus cantonensis
ƒ
Page 265
ƒ A.costaricensis
ƒ
• The above image shows egg of Trichiuris trichirua

ƒ Toxocara canis
ƒ
• Identification is by:

ƒ Toxocara catis
ƒ
ƒ Barrel shaped egg
ƒ

ƒ Mucous plugs at both ends


ƒ Anisakine sp., ƒ

• Infection is acquired by ingestion of food that is contam-


ƒ

ƒ Gnathostoma spinigerum

inated with embryonated egg


ƒ

6. Ans.  (b) Plasma


12. Ans.  (b) Embryonated egg is the infectious stage for this
Ref: T.B. of Parasitology–Chaterjee–12th edition–Page 175 worm
• The main nutrition for hookworm is from plasma Ref: T.B. of Medical Parasitology–S.C.Parija–4th edition–
346

Page 265
• The above image shows adult male and female worms 15. Ans.  (d) Whip worm

Chapter 46     Nematodes


of Trichuris • Already explained Q.11


• Characteristic whip shaped hence called as whip worm

• Infection is acquired by ingestion of food that is


16. Ans.  (a) Strongyloides stercoralis; (b) Ankylostoma


contaminated with embryonated egg duodenale
13. Ans.  (a) Ancylostoma braziliense Ref: T.B of Medical Parasitology – S.C. Parija – 4th edition–
page 268
Ref: T.B of Parasitology – Chatterjee – 12th edition – Page 206
• Larvae seen in the stools are characteristic for:

Cutaneous larva migrans Visceral larva migrans • Strongyloides stercoralis


• Ancylostoma braziliense
• •• Toxocara canis • Hook worm – Ancylostoma duodenale and Necator

(M/c) •• Toxocara cati americanus


• Ancylostoma caninum
• •• Gnathostoma
(second M/c) •• Anisakis 17. Ans.  (b) The video shows the filariform larvae of the
• Necator americanus
• parasite
• Ancylostoma duodenale

• Gnathostoma spinigerum

Ref: T.B of Medical Parasitology – S.C. Parija – 4th edition–
• Strongloides

page 268
• Loa loa

• The clinical clue to identify the organisms are:


• Fasciola

ƒ Renal transplantation
• Paragonimus
ƒ

ƒ Diarrhea

• Considering the options – most common cause is taken



• The microbiological clue is shows larvae (only in two

as choice organisms – stool shows larvae, i.e.,); they are


ƒ Strongyloides stercoralis
14. Ans.  (a) Rhabditiform larvae of S.stercoralis
ƒ

ƒ Hook worm
ƒ

Ref: Medical Parasitology – 3rd edition – D. R. Arora – • Infections are more common in HIV, immunosup-

page 180 pressed and post-renal transplantation patients


• Patients with S.stercoralis hyperinfection presents with • Diagnosis: By demonstration of rhabditiform larvae

in freshly passed stool; Filariform larvae can be


above features of dissemination. It occurs in immuno-


compromised patients demonstrated by the culture of the stool by Harada Mori
• It is the smallest nematode and rhabditiform larvae is filter paper method.
• Treatment is Ivermectin and Thiabendazole

the most common form seen in the stool specimen. •

347
47
Filarial Nematodes
INTRODUCTION y Definitive host: Humans

y
y Intermediate host: Mosquitoes–Culex, Anopheles, Aedes

y
Table 1:  Nematodes and their associated manifestations y Wuchereria has a endosymbiotic bacteria called as Wol-

y
bachia
Lymphatic Subcutaneous Serous cavity
filariasis filariasis filariasis
• Wuchereria • Loa loa • Mansonella Pathogenesis



bancrofti • Mansonella perstans y Adult worms causes inflammatory damage to the lymphatics.

y

• Brugia malayi streptocerca • Mansonella y Infiltration of plasma cella, eosinophils and thickening of

y


• Brugia timori • Onchocerca ozzardi vessel walls leads to tortous lymph vessels and lymphedema.


volvulus y Clinical features of lymphatic filariasis (Bancroftian filariasis):
y
 Lymphangitis

 Obstruction of lymph nodes
WUCHERERIA BANCROFTI

 Lymph varices

y This worm lives in the lymphatic vessels–hence called as  Hydrocele
y

lymphatic filarial worm  Elephantiasis

y Important morphological forms are:  Chyluria
y

Adult worm Lives in the lymphatics y Acute adeno lymphangitis (ADL) is characterized by high
y
fever, lymphatic inflammation and local edema. Regional
Microfilaria Lives in peripheral blood, hydrocele fluid
lymph nodes are enlarged and entire lymphatic channel gets
(First stage larva) and chylous urine (blood and body fluids)
blocked.
Third stage larva Infective form y Genital involvement occurs manifesting as funiculitis,
y
epididymitis, scrotal pain and tenderness.
y Characteristics of microfilaria:
y In endemic areas, another syndrome complex presents as
y
 Key identification feature for diagnosis
y
high fever, chills, myalgia with head ache and edematous

 Transparent and colourless with a sheath
inflammatory plaques–termed as dermatolymphangioadenitis

 End of the tail does not contain nuclei
(DLA).

 Based on the periodicity of microfilaria circulation–
y Occult filariasis–means a hypersensitivity reaction that

infectivity differs–Microfilaria of Wuchereria has nocturnal
y
occurs to filarial antigens; In this condition microfilaria
periodicity
cannot be seen in peripheral blood; the most common
manifestation is TPE (Tropical pulmonary eosinophilia)
Table 2:  Differences between classical and occult
filariasis
Classical filariasis Occult filariasis
• It is due to adult worms • It is due to microfilariae


• Lymphangitis and lymph • Hypersensitivity reaction is


nodes are affected the feature
• Microfilariae in the • Microfilariae is not seen in


peripheral blood is present the peripheral blood
• Serological tests are not • High level of antibodies are


useful seen (Increased IgE)
y Laboratory diagnosis:
y
 Blood microscopy–peripheral blood smear

Figure 1: Figure showing microfilariae of Wuchereria bancrofti Š Collection of blood is during night (because of
Š
(Courtesy: CDC/Dr Mae Melvi) nocturnal periodicity)
Š DEC provocation test: DEC given at 2–8 mg/kg BW–after Table 4:  Other filarial nematodes

Chapter 47     Filarial Nematodes


Š

30 minutes–the capillary blood collected and checked


for microfilariae Brugia • Causes lymphatic filariasis called as Brugian

Š Sensitivity can be increased by concentration method malayi filariasis


• Identification is by presence of two distinct
Š

named as Knott’s concentration technique. •

Š QBC nuclei in the tail tip of microfilariae


• Genital involvement and chyluria are not seen
Š

 Immunodiagnosis–circulating antibodies are demonstrated


 Filarial skin test



Onchocerca • Causes river blindness

 PCR–helpful in chronic and occult filariasis



volvulus • Transmitted by the bite of infected blackfly

 The only noninvasive method helpful for detection of live



• Causes subcutaneous itchy nodules mainly

adult worms–is USG of scrotum over the bony prominences


y Treatment:
y
• Visual impairment occurs due to punctate

 Diethylcarbamazine (DEC) → 3–6 mg/kg for 3 weeks


 keratitis
 Ivermectin
 • Microfilariae are found in the skin; not seen

 Levamisole
 in the peripheral blood–it is demonstrated by
 Mebendazole
 skin snips
• Mazzotti patch test is helpful

TROPICAL PULMONARY EOSINOPHILIA Loa loa • Called as Eye worm


y Syndrome that is developed in individuals who are infected


y
• It produces transient subcutaneous swellings–

with lymphatic dwelling filarial species. calabar swellings


y Usually occurs in endemic areas • Microfilariae are sheathed and nuclei seen in

the tail tip


y

y Symptoms are mostly seen at night because of nocturnal


• Causes calabar swelling; conjunctival
y

periodicity of microfilariae. •

y Clinical features are nocturnal cough, weight loss, low grade granuloma; it causes proptosis;
meningoencephalopathy
y

fever, lymphadenopathy and high blood eosinophilia.


y Serum IgE levels and antifilarial antibody titers are markedly
y
Dracunculus • Also called as Guinea worm; Serpent worm or

elevated. medinensis Dragon worm


y DD: Churg Strauss syndrome, ABPA, Asthama, Loeffler`s
y
• Definitive host: Man

syndrome • Intermediate host: Cyclops


y Treatment: DEC 4–6 mg/kg for 14 days


y
• Mode of infection is through drinking water

that has cyclops


Table 3:  Important filarial organism
Dirofilariae • Zoonotic filarial infection
Filarial organism Vector Disease •

•• Primarily affects dogs, cats


Wuchereria Culex Bancroftian lymphatic •• Humans are accidental host
bancrofti filariasis •• It usually causes solitary pulmonary nodule
Brugia malayi Mansonia, Culex Brugian lymphatic
Table 5:  Identification of filarial nematodes
Brugia timori Mansonia, Culex, filariasis
Anopheles Sheathed Microfilaria Unsheathed Microfilaria
Onchocerca Simulum fly Subcutaneous nodules; • Wuchereria bancrofti
• • • Mansonella perstans
volvulus River blindness • Loa loa
• • • M.ozzardi
• Brugia malayi • M.streptocerca
Loa loa Chrysops fly Calabar swellings
• •

• • O.volvulus
Mansonella Mild clinical illness
perstans
Mansonella Culicoides
streptocerca
Mansonella
ozzardi

349
Unit 4     Parasitology

Microfilaria Identification
First look whether sheathed or unsheathed
Sheathed means: then look at tail tip

Sheathed Sheathed Sheathed


Tail tip no nuclei Tail tip has two-three nuclei Tail tip has dence nuclei fully
Wuchereria bancrofti Brugia malayi Loa loa

Microfilaria Identification
Unsheathed means:

Unsheathed Unsheathed Unsheathed; Very thin among all; tail tip


Body nuclei extends up to tip No tail tip nuclei nuclei seen
Mansonella perstans Mansonella ozzardi Onchocerca volvulus

350
MULTIPLE CHOICE QUESTIONS

Chapter 47     Filarial Nematodes


1. Wuchereria bancrofti, true is: (Recent pattern Dec 2012) 5. Calabar swelling is produced by:
a. Unsheathed (Recent pattern Nov 2014)
b. Tail tip free from nuclei a. Onchocerca volvulus
c. Non -periodic b. Loa loa
d. All c. Brugia malayi
d. Wuchereria bancrofti
2. Which is non lymphatic filariasis:
6. River blindness is caused by: (Recent pattern Dec 2013)
(Recent pattern Dec 2012)
a. Onchocera
a. Loa loa
b. Loa loa
b. Wuchereria bancrofti
c. Ascaris
c. Brugia malayi
d. B.malayi
d. Brugia timori
7. Which of the following is/are causes lymphatic filariasis:
3. All the filarial worms are found in blood except:
(PGI May 2018)
(Recent pattern Dec 2013)


a. Loa-Loa
a. W. bancrofti
b. Oncocerca volvulus
b. B. malayi
c. Wuchereria bancrofti
c. L. loa
d. Brugia malayi
d. O. volvulus
e. Brugia timori
4. This is a schematic diagram depicting body structure of
which of these helminthes? (AIIMS Nov 2015)

a. Loa loa b. Wuchereria bancrofti


c. Brugia malayi d. Onchocera volvulus

351
ANSWERS AND EXPLANATIONS
Unit 4     Parasitology

1. Ans.  (b) Tail tip free from nuclei 4. Ans.  (b) Wuchereria bancrofti
Ref: T.B of Medical Parasitology–S.C.Parija–4th edition– Ref: T.B of Medical Parasitology–S.C.Parija–4th edition–
Page 298 Page 298
• Wuchereria bancrofti is a filarial nematode that has

• Image clearly shows microfilaria of Wuchereria bancrofti

nocturnal periodicity • Wuchereria bancrofti is a filarial nematode that has


• It has sheathed microfilaria



nocturnal periodicity
• The tail is pointed and it if free from nuclei • It has sheathed microfilaria

• The tail is pointed if free from nuclei


2. Ans.  (a) Loa loa


5. Ans.  (b) Loa loa
Ref: T.B of Medical Parasitology–S.C.Parija–4th edition–
Ref: T.B of Medical Parasitology–S.C.Parija–4th edition–
Page 296
Page 316
• Filariasis is classified into:
• Loa Loa or eye worm causes transient subcutaneous

ƒ Lymphatic filariasis–W.bancrofti, Brugia malayi and


swellings known as calabar swellings


ƒ

Brugia timori
ƒ Subcutaneous filariasis: Loa loa, Mansonella strep-
ƒ
6. Ans.  (a) Onchocera
tocerca, Onchocerca volvulus
ƒ Serous cavity filariasis: Mansonella perstans and
Ref: T.B of Medical Parasitology–S.C.Parija–4th edition–
Page 313
ƒ

Mansonella ozzardi
• Onchocerca volvulus causes onchocerciasis or river

3. Ans.  (d) O.volvulus blindness


• It causes cutaneous filariasis

Ref: T.B of Medical Parasitology – S.C.Parija – 4th edition –


Page 297 7. Ans.  (c) Wuchereria bancrofti; (d) Brugia malayi;
(e) Brugia timori
Habitat Microfilariae Examples
Blood •• Wuchereria bancrofti Ref: T.B of Medical Parasitology – S.C.Parija – 4th edition –
•• Brugia malayi Page 297
•• Brugia timori • Lymphatic filariasis is caused by:

•• Loa loa ƒ Wucheria bancrofti


ƒ

•• Mansonella perstans ƒ Brugia malayi


ƒ

ƒ Brugia timori
Skin • Onchocerca volvulus ƒ

• Subcutaneous filariasis is caused by:


• Mansonella streptocerca •

ƒ Onchocerca volvulus

• Mansonella ozzardi
ƒ

Blood and skin •

ƒ Loa loa
ƒ

352
HIGH YIELDING FACTS TO BE REMEMBERED IN PARASITOLOGY

Chapter 47     Filarial Nematodes


Man acts as intermediate host in: Culture media used in parasitology
• Plasmodium

Leishmania and Trypanosoma NNN medium


• Babesia
Plasmodium RPMI medium

• Toxoplasma

• Echinococcus
• Nematodes like Strongyloides, Coproculture technique
• Sarcocystis
• hook worm Harada-Mori filter paper method
Man acts as both definitive and intermediate host in: Baermann technique
• Taenia solium
• Agar plate culture

Important vectors in parasitology Skin tests in parasitic diseases


Entamoeba histolytica Cockroaches Type I Hypersensitivity reactions:
Leishmania Sandfly • Hydatid disease–Casoni’s test

• Ascariasis
Trypanosoma cruzi Reduviid bugs

• Strongyloidiasis

Trypanosoma brucei Tsetse fly • Filariasis


Babesia Ixodid ticks • Schistosomiasis


• Trichinellosis–Bachmann test
Plasmodium Female anopheles

Type IV Hypersensitivity reactions:


Filarial nematodes Culex, anopheles • Leishmaniasis–Montenegro test

Dracunculus Cyclops • Trypanosomiasis


• Toxoplasmosis

Modes of transmission of parasitic infections


Ingestion of food, water and Entamoeba histolytica Arthropod borne diseases
vegetables Giardia lamblia Sandfly • Kala azar
Ascaris lumbricoides

• • Sandfly fever–three days fever


Enterobius vermicularis • • Oriental sore
Trichuris trichiura • • Oroya fever or Carrion’s disease
Ingestion of undercooked beef Taenia saginata Rat flea • Bubonic plague

• Endemic typhus

Ingestion of undercooked pork Taenia solium • Intermediate host for H.nana


Trichinella spiralis
Louse • Epidemic typhus

Ingestion of raw/undercooked Clonorchis sinensis • Trench fever


fish Diphyllobothrium latum • Epidemic relapsing fever


Ingestion of undercooked crab Paragonimus westermani Trombiculid mites • Scrub typhus


Ingestion of water plants Fasciola hepatica Itch mites • Scabies


Fasciolopsis buski Hard tick (Ixodid) • • Babesiosis


Skin penetration Strongyloides stercoralis • • KFD
Schistosoma species • • Tick typhus
• Q fever
Sexually transmitted agents Trichomonas vaginalis

• Tularemia
Entamoeba histolytica

(homosexuals MSM) Soft tick • Endemic relapsing fever


Giardia lamblia
Bile stained eggs
Culture media used in parasitology •• Ascaris lumbricoides • • Fasciolopsis buski
• Trichuris trichiura • Clonorchis sinensis
Entamoeba histolytica Boeck and Dr Bohlav’s medium
• •

• Taenia • Opisthorchis felineus


Balantidium coli Balamuth’s medium
• •

• Schistosoma japonicum • Diphyllobothrium latum


Robinson’s medium
• •

• Fasciola hepatica
Jone’s medium

Diamond’s medium Parasitic infections not in India: (As of 2017)


Giardia lamblia Diamond’s medium •• Trypanosoma spp • Onchocerca volvulus

Trichomonas vaginalis Modified Diamond’s medium • Leishmania braziliensis • Capillaria philippinensis


353
• •

TYM medium •• Balamuthia spp • Sappinia spp


contd... •• Plasmodium knowlesi


e s
No t
5

MYCOLOGY
Unit Outline

1 Chapter 48: Characteristics and Laboratory


Diagnosis of Fungi
Chapter 49. Superficial Mycoses
Chapter 50: Endemic/Systemic Mycoses
Chapter 51: Opportunistic Mycoses
Chapter 52. Miscellaneous Fungi
Characteristics and Laboratory 48
Diagnosis of Fungi
CHARACTERISTICS OF FUNGI Table 3:  Classification of fungi based on pathogenesis

y Fungi are Eukaryotes. Superficial mycoses Dermatophytosis, Candidiasis, Tinea,


y
y They have rigid cell walls. Piedra, Pityriasis versicolor
y
y They have chitin, mannan and other polysaccharides. Cutaneous mycoses Candidiasis
y
y True nuclei with nuclear membrane, paired chromosomes
Subcutaneous Mycotic mycetoma
y
are seen.
mycoses Chromoblastomycosis
y They are unicellular or multicellular
Rhinosporidiosis
y
y They divide sexually or asexually
Sporotrichosis
y
Deep or systemic or Blastomycosis
CLASSIFICATION visceral mycoses Cryptococcosis
Table 1:  Classification of fungi based on cell morphology Paracoccidioidomycosis
Coccidioidomycosis
Yeast Cryptococcus Histoplasmosis
Yeast like Candida sp. Candidiasis

Moulds Dermatophytes Opportunistic mycoses Mucor, Aspergillus, Candida, Penicillium


Aspergillus Mycotoxicoses Aspergillus, Claviceps purpurea
Mucor
Penicillium Important points to be noted in classification:
Rhizopus y Sexual spores: Ascospore, Zygospore, Basidiospore, Oospore
y
y Asexual spores (Conidia): Sporangiospores, Arthrospores,
Dimorphic fungi Blastomyces dermatitidis
y
Blastospores, Chlamydospores, Phialoconidia
(Two growth forms – Yeast and Paracoccidioides brasiliensis
Mould) Coccidioides immitis
Table 4:  Asexual spores with their example
Histoplasma capsulatum
Sporothrix schenckii Conidia Examples
Penicilliosis marneffei
Sporangiospores Mucor, Rhizopus
Arthrospores Coccidioides
Table 2:  Systemic classification based on sexual spore
formation Blastospores Cladosporium

Phycomycetes Aseptate hyphae, Endogenous asexual Chlamydospores Candida albicans


spores (Lower fungi) and sexual spores– Phialoconidia Aspergillus fumigatus
E.g. Rhizopus, Mucor
Ascomycetes Septate hyphae, exogenous asexual spores TERMINOLOGY
(Higher fungi) and sexual spores – Terminologies used in Mycology:
E.g. Dermatophytes y Yeast – Unicellular, reproduce by budding
y
Basidiomycetes Same as above E.g. Cryptococcus y Yeast like – Unicellular, reproduce by budding or by fission
y
y Moulds – Filamentous fungi – has hyphae – which is either
Fungi imperfecti No sexual spores
y
septate or aseptate, reproduce by spore formation
E.g: Candida, Coccidiodes, Paracoccidiodes
y Thermally dimorphic fungi – Grow as yeasts in 37°C, as
y
moulds in 25°C
y Coenocytic hyphae – Aseptate hyphae
Unit 5     Mycology

y Pseudohyphae – When yeast cell starts reproduce by budding → some of the buds fail to detach from the cells and becomes
y

elongated → This elongated budding cells are called as pseudohyphae as they look like hyphae E.g.: Candida albicans
y Mycelium – Tufts of hyphae
y

y Anamorph – Asexual form


y

y Telemorph – Sexual or perfect form


y

MORPHOLOGY OF FUNGI

Figure 1: Morphological features of fungi

Figure 3: Septate hyphae (Courtesy: CDC PHIL/Dr. Lucille Georg)


Figure 2: Budding yeast cells— oval shaped cells
(Courtesy: CDC PHIL/Dr. Libero Ajello)

358
Figure 4: Pseudohyphae
Chapter 48     Characteristics and Laboratory Diagnosis of Fungi
Figure 5: Systemic classification of fungi

Table 5:  Endemic and opportunistic mycoses Culture of Fungi


Endemic mycoses Opportunistic mycoses y Requirements for culture – Aerobic, lower pH than bacteria,
y

Temperature range 25-30°C


• Coccidioidomycosis • Candidiasis
y Exceptions
• •

• Histoplasmosis • Aspergillosis
y

 Deep mycotic organisms – grow even in 37°C


• •

• Blastomycosis • Cryptococcosis


 Asp.fumigatus – grow even in 50°C


• •

• Phaeohyphomycosis • Mucormycosis


y Agar used – SDA – Sabouraud dextrose agar – pH 5.4


• •

• Penicilliosis • Scedosporiosis
y

y Emmon’s modification (Neutral) SDA – pH 7.2


• •

• Sporotrichosis • Fusariosis
y

• •

• • Paracoccidioidomycosis •• Pneumocystosis

LABORATORY DIAGNOSIS OF FUNGI

Figure 7: A culture plate of SDA


y Other culture media used in fungal culture:
y

 Blood agar
Figure 6: Laboratory diagnosis of fungi 

 Brain heart infusion agar/broth


Microscopy


 Czapek Dox medium




y KOH mount – Specimens are first seen microscopically; After


y
 Bird seed agar


applying 10-40% KOH in the specimen – wait for few minutes  Corn meal agar


for the tissue debris to get removed so that fungal elements  CHROMagar


can be seen clearly y Special addition in culture media – Chloramphenicol – to


y

y 0.1% Calcofluor white – Fluorescent stain that binds to the


y
suppress bacterial contamination; Cycloheximide (Actid-
cellulose and chitin of the fungal cell wall ione) – to suppress saprophytic fungi
y Gram stain – helpful for yeasts and yeast like fungi and to
y

identify pseudohyphae (Candida sp is a Gram positive Serological Methods


budding yeast cell)
y Antigen detection – latex agglutination test – Cryptococcal
y India Ink stain – Negative staining procedure – for capsule of
y

antigen in CSF
y

Cryptococcus neoformans
y Antibody detection – ELISA, Immunodiffusion test, Aggluti-
y LPCB – Lactophenol cotton blue – for preparing mount from
y

nation test, Complement fixation test


y

the fungal colonies


y Immunohistochemistry – used in deep mycoses
359
y Tissue stains: H & E, GMS, PAS, Mayer’s Mucicarmine stain
y

y
Metabolites
Unit 5     Mycology

y Fungal metabolite detection – by Gas liquid chromatography


y

Skin Tests
y Some fungi elicit delayed type hypersensitivity reactions
y

For example: Histoplasma, blastomyces, cocciodiodes, paracocciodioides, dermatophytes, sporothrix and Candida

Special Stains
Table 6:  Recommended special stains for various fungi

Stains Uses
Periodic acid schiff (PAS) Recommended stain for fungi – stains only live fungi
Gomori methenamine silver (GMS) Stains both live and dead fungi
Mucicarmine stain For Cryptococcus and Rhinosporidium
Masson Fontana stain For pigmented fungi
H and E stain For tissue stain
Toluidine blue Pneumocystis carinii

360
MULTIPLE CHOICE QUESTIONS

Chapter 48     Characteristics and Laboratory Diagnosis of Fungi


1. Fungi are: (Recent Pattern Dec 2014) 10. All are dimorphic fungi except: (AIIMS May 2009)
a. Prokaryotes b. Eukaryotes a. Blastomyces dermatitidis
c. Plant d. Animalia b. Histoplasma capsulatum
2. Which of the following is a yeast? c. Penicillium marneffei
a. Candida (Recent Pattern Dec 2012) d. Phialophora
b. Rhizopus 11. Systemic infection is caused by all of the fungi except:
c. Mucor a. Cryptococcus (Recent Pattern Dec 2013)
d. Cryptococcus b. Histoplasma
3. Acute angled septate hyphae are seen in:  c. Dermatophytes
a. Aspergillus (Recent Pattern Dec 2013) d. Paracoccidioides
b. Mucor 12. pH of SDA is:
c. Penicillium a. 5.4 b. 7.4
d. Candida c. 7.2 d. 6.4
4. Budding reproduction in tissue is seen in: 13. In thermally dimorphic fungi, mycelial phase occurs in
a. Cryptococcus, Candida (Recent Pattern Nov2014) which temperature:
b. Candida, Rhizopus a. 25°C
c. Rhizopus, Mucor b. 37°C
d. Histoplasma, Candida c. 45°C
5. Ascospores are: (Recent Pattern Dec 2015) d. 15°C
a. Asexual spores b. Sexual spores 14. Arthrospores are seen in:
c. Conidia d. None of the above a. Coccidioides
6. All of the following fungi has sexual phase except: b. Histoplasma
 (Recent Pattern Dec 2016) c. Dermatophytes
a. Phycomycetes b. Fungi imperfecti d. Paracoccidioides
c. Basidiomycetes d. Ascomycetes 15. Special stain used to stain both live and dead fungi is:
7. Aseptate hyphae are not seen in: a. H and E
a. Rhizopus (Recent Pattern Dec 2012) b. PAS
b. Mucor c. Gomori methenamine silver
c. Aspergillus d. Toluidine blue
d. Absidia 16. Not a dimorphic fungus (AIIMS Nov 2017)
8. KOH mount is used to examine:  a. Blastomyces dermatitidis
 (Recent Pattern Aug 2013) b. Histoplasma capsulatum
a. Bacteria b. Virus c. Pneumocystis jirovecii
c. Fungus d. Parasite d. Penicillium marneffei
9. Culture medium used for isolation of fungus is: 17. Dimorphic fungi is/are: (PGI May 2017)
 (Recent Pattern Dec 2014) a. Histoplasma capsulatum
a. NNN medium b. Sporothrix schenckii
b. Potassium tellurite medium c. Malassezia furfur
c. Chocolate agar d. Cryptococcus neoformans
d. Sabouraud’s dextrose agar e. Aspergillus

ANSWERS AND EXPLANATIONS


1. Ans.  (b) Eukaryotes 2. Ans.  (d) Cryptococcus
Ref: Ananthanarayan and Paniker’s Textbook of Microbiol- Ref: Ananthanarayan and Paniker’s Textbook of Microbiol-
ogy – 10th ed – Page 593 ogy – 10th ed – Page 593
• Fungi – Eukaryotic Protista

Classification of fungi based on cell morphology


• Rigid cell walls
Yeast Cryptococcus

• Has chitin, mannan and other polysaccharides


• True nuclei with nuclear membrane, paired chromo-


• Yeast like Candida sp.,
somes
• Unicellular or multicellular

• Divide sexually or asexually


361
3. Ans.  (a) Aspergillus 9. Ans.  (d) Sabouraud’s dextrose agar
Unit 5     Mycology

Ref: Ananthanarayan and Paniker’s Textbook of Microbiol- Ref: Ananthanarayan and Paniker’s Textbook of Microbiol-
ogy – 10th ed – Page 613 ogy – 10th ed – Page 597
• Septate hyphae with acute angle dichotomous branch-
• • Agar used – SDA – Sabouraud dextrose agar – pH 5.4

ing is suggestive of Aspergillus species • Emmon’s modification (Neutral) SDA – pH 7.2


4. Ans.  (a) Cryptococcus, Candida 10. Ans.  (d) Phialophora


Ref: Ananthanarayan and Paniker’s Textbook of Microbiol- Ref: Ananthanarayan and Paniker’s Textbook of Microbiol-
ogy – 10th ed – Page 594 ogy – 10th ed – Page 595
• Yeast: Unicellular, reproduce by budding – E.g. Crypto-
• Examples for dimorphic fungi:
coccus • Blastomyces dermatitidis

• Yeast like: Unicellular, reproduce by budding or by


• • Paracoccidioides brasiliensis

fission E.g. Candida • Coccidioides immitis


• Histoplasma capsulatum

5. Ans.  (b) Sexual spores • Sporothrix schenckii


Ref: Ananthanarayan and Paniker’s Textbook of Microbiol- • Penicilliosis marneffei


ogy – 10th ed – Page 595


11. Ans.  (c) Dermatophytes
• Sexual spores: Ascospore, Zygospore, Basidiospore, Oo-

spore Ref: Ananthanarayan and Paniker’s Textbook of Microbiol-


• Asexual spores (Conidia): Sporangiospores, Arthro-

ogy – 10th ed – Page 595
spores, Blastospores, Chlamydospores, Phialoconidia Systemic mycoses are:
• Blastomycosis

6. Ans.  (b) Fungi imperfecti • Cryptococcosis


Ref: Ananthanarayan and Paniker’s Textbook of Microbiol- • Paracoccidioidomycosis


ogy – 10th ed – Page 595 • Coccidioidomycosis


• Histoplasmosis
• Based on sexual spore formation – Fungi are classified

• Candidiasis

into Ascomytes, Basidiomycetes, Phycomycetes and


Fungi Imperfecti 12. Ans.  (a) 5.4
• Fungi imperfecti is a special group which has no sexual

spores e.g.: Coccidioides, Paracoccidiodes Ref: Ananthanarayan and Paniker’s Textbook of Microbiol-
ogy – 10th ed – Page 595
7. Ans.  (c) Aspergillus • Already explained in Q.9

Ref: Ananthanarayan and Paniker’s Textbook of Microbiol-


13. Ans.  (a) 25°C
ogy – 10th ed – Page 594
• Moulds–Filamentous fungi–has hyphae – which is either

Ref: Ananthanarayan and Paniker’s Textbook of Microbiol-
septate or aseptate, reproduce by spore formation ogy – 10th ed – Page 595
• Septate hyphae – E.g. Aspergillus
• • Thermally dimorphic fungi – Grow as yeasts in 37°C, as

• Aseptate hyphae – E.g. Zygomycetes – Mucor, Rhizopus,


• moulds in 25°C
Absidia
14. Ans.  (a) Coccidioides
8. Ans.  (c) Fungus
Ref: Ananthanarayan and Paniker’s Textbook of Microbiol-
Ref: Ananthanarayan and Paniker’s Textbook of Microbiol- ogy – 10th ed – Page 595
ogy – 10th ed – Page 596
Conidia Examples
• Specimens are first seen microscopically; After applying
Sporangiospores Mucor, Rhizopus

KOH in the specimen – wait for few minutes for the


tissue debris to get removed so that fungal elements can Arthrospores Coccidioides
be seen clearly Blastospores Cladosporum
Chlamydospores Candida albicans
Phialoconidia Aspergillus fumigatus

362
15. Ans.  (c) Gomori methenamine silver 16. Ans.  (c) Pneumocystis jirovecii

Chapter 48     Characteristics and Laboratory Diagnosis of Fungi


Ref: Ananthanarayan and Paniker’s Textbook of Microbiol- Ref: Ananthanarayan and Paniker T.B of microbiology –
ogy – 10th ed – Page 597 10th ed – page 593

Periodic acid schiff (PAS) Recommended stain for fungi Dimorphic fungi:
– stains only live fungi • Blastomyces dermatitidis

• Paracoccidioides brasiliensis

Gomori methenamine Stains both live and dead • Coccidioides immitis


silver (GMS) fungi • Histoplasma capsulatum


Mucicarmine stain For Cryptococcus and • Sporothrix schenckii


Rhinosporidium • Penicilliosis marneffei


Masson Fontana stain For pigmented fungi


17. Ans.  (a) Histoplasma capsulatum; (b) Sporothrix
H and E stain For tissue stain schenckii
Toluidine blue Pneumocystis carinii • Already explained in Q. 16

363
49
Superficial Mycoses
INTRODUCTION Tinea Nigra
There are two types of superficial mycoses: y Localized infection of Stratum corneum

y
y Surface mycoses y Agent: Exophiala werneckii, Exophiala castellanii
y
y
y Cutaneous mycoses y Lesions: Brownish or black macular lesions
y
y
y Site: Palms
Table 1:  Different between surface and cutaneous

y
y Diagnosis: Skin scrapings – brownish, branched, septate hy-
mycoses

y
phae (Pigmented fungi)
Surface Mycoses Cutaneous Mycoses
Affects the dead layers Affects the cornified layer of Piedra
skin y Irregular nodules in hair shaft
y
No contact with living tissues Living tissues affected y Black piedra – Piedraia hortae
y
y White piedra – Trichosporon beigelii
No inflammation (Since it Inflammatory response and
y
does not affect living tissues) allergic response occurs
E.g. Pityriasis versicolor, e.g. Dermatophytes, Candida CUTANEOUS MYCOSES
Tinea nigra, Piedra albicans Dermatophytoses
y It is also called as Tinea or Ringworm
SURFACE MYCOSES
y
y Infection of skin, hair and nail (keratinized structures) are
y
Pityriasis Versicolor (Tinea Versicolor) caused by dermatophytoses
y Three genera:
y It affects stratum corneum
y
 Trichophyton
y
y Agent– Malassezia furfur: It causes Tinea versicolor, Folliculi-

 Microsporum
y
tis and seborrheic dermatitis

 Epidermophyton
y Lesions appear as macular areas of discoloration – depig-

y
mentation; DD - Vitiligo
y Sites: Skin of the chest, abdomen, upper limbs and back
Table 2:  C
 lassification of dermatophytes based on
Morphology
y
y Seborrheic dermatitis manifests as erythematous pruritic
y
scaly lesions seen in the eyebrows, moustache, nasolabial Genera Site of Macroconidia Microconidia
folds and scalp; scalp lesions seen in babies is called as cradle infection
cap.
y Diagnosis: Specimen – Skin scrapings – shows yeast like cells Trichophyton Skin, hair, nail Very few, Abundant tear
y
with short branched filaments pencil shaped drop shaped
y Culture: SDA with olive oil layer (lipophilic fungi) or cylindrical in clusters
y
y Treatment: Topical creams and lotions like selenium sulfide shaped
y
shampoo, ketoconazole shampoo, terbinafine cream and Microsporum Hair and skin Abundant Rare
ciclopirox cream. spindle
shaped

Epidermo- Skin and nail Abundant Absent


phyton club shaped

Figure 1: Skin lesions of Pityriasis versicolor


y Ectothrix: Arthrospores are seen as a sheath surrounding the

Chapter 49     Superficial Mycoses


y

hair – caused by Microsporum, T.rubrum, T.mentagrophytes


y Endothrix: Arthrospores are inside the hair shaft – caused by
y

T.schoenleinii, T.tonsurans, T.violaceum

Figure 2: Trichophyton rubrum (Bird in fence pattern of micro


conidia) (Courtesy: CDC PHIL/ Dr. Libero Ajello)

Table 3:  Classification of Dermatophytes based on


Source of Infection
Anthropophilic Human to human T.rubrum, T.tonsurans,
transmission T.violaceum, T.schoenleini,
M.audounii
Figure 3: Ectothrix and Endothrix type of hair infection
Zoophilic Animal to human T.mentagrophytes,
T.verrucosum, M.canis
Geophilic Soil to human M.gypseum, M.fulvum Clinical Manifestations
y Tinea capitis is seen mostly in children of age 3-7 years old;
y

y Pathogenicity: They grow only in keratinized layers of skin


y
they have well demarcated scaly patches in the hair and that
and its appendages (Keratinophilic fungi) leads to alopecia:
y Sometimes, after initiation of antifungal treatment →
y
 Kerion: Boggy lesion in the scalp – usually in children


hypersensitivity to fungal antigens occurs → Dermatophytids  Favus: Dense crusts develop in the hair follicles – leads to


(id reaction) alopecia


y Tinea corporis has well demarcated, annular, pruritic, scaly
y

Table 4:  Clinical features of dermatophytoses and their lesions that has central clearing.
etiological agents y Tinea cruris is seen in men; the perineal rash also called as
y

Jock’s itch – is erythematous and pustular without satellite


Clinical infection Region Causative organism lesions.
Tinea barbae Face and neck T. rubrum, y Tinea pedis is also more common in men than women with
y

(Barber’s itch) T. mentagrophytes, T. hyperkeratosis of soles.


verrucosum
Tinea corporis Body T.rubrum (M/c) and any
other
Tinea imbricata Concentric T.concentricum
rings of
papulosquamous
scaly patches in
body
Tinea capitis Scalp Trichophyton and
(Favus, kerion) Microsporum
Tinea cruris (Jock Groin and T.rubrum, E.floccosum
itch) perineum
Tinea pedis Foot T.rubrum, E.floccosum
(Athlete’s foot)
Tinea manuum Hand T.rubrum
Figure 4: Kerion
Tinea unguium Nails T.rubrum,
(Onychomycosis) T.mentagrophytes, 365
E.floccosum
Sporotrichosis
Unit 5     Mycology



 Rhinosporidiosis
 Entomophthoromycosis

Mycetoma
y Fungal mycetoma also called as Maduromycosis or Madura
y

foot
y Three types:
y

Table 5:  Types of mycetoma


Types of Mycetoma Organisms
Eumycetoma Madurella mycetomatis, M.grisea,
Exophiala sp., Acremonium sp.,
Figure 5: Favus (Courtesy: CDC PHIL) Aspergillus sp., Scedosporium,
Fusarium sp.,
Actinomycetoma Actinomadura sp., Streptomyces sp.,
Laboratory Diagnosis Nocardia sp.,
y Specimens: Skin, hair, nail
Botryomycosis Staphylococcus aureus and bacterial
y

y KOH examination of specimen: Reveals branching hyaline


cause
y

septate (nonpigmented) hyphae


y Wood’s lamp examination: Positive for Microsporum canis
y y Triad: Swelling of the foot, Multiple sinuses, Discharge of
y

and certain Trichophyton species— Direct examination granules


of hair with Wood’s lamp helps in visualization of affected y Granules or Grains are actually Micro colonies of the organ-
y

hair (Due to the presence of metabolites) – But not 100% of isms


dermatophytes can be diagnosed by Wood’s lamp y Color of the granules helps in presumptive identification of
y

y Culture: SDA with actidione and chloramphenicol


y
the organism
y Hair perforation test: Positive in T.mentagrophytes
Nocardia asteroides
y

Nocardia brasiliensis
Actinomadura madurae White to yellow granules
Streptomyces somaliensis
Scedosporium apiospermum
Madurella mycetomatis
Madurella grisea Brown to black
Exophiala jeanselmei
Actinomadura pelletieri Red

y Treatment: Surgery – Main mode of treatment


y

y Eumycetoma: Antifungal agents


y

Figure 6: Spindle shaped macroconidia of M.canis y Actinomycetoma: Dapsone, sulfonamides, cotrimoxazole,


y

(Courtesy: CDC/ Dr. Lucille K. Georg) Rifampicin

Chromoblastomycosis
Treatment y Fungal infection caused by pigmented fungi, also called as
y

y y Topical – Miconazole, Clotrimazole, Econazole dematiaceous fungi


y y Terbinafine cream y Called as verrucous dermatitis
y

y y Griseofulvin orally 100 mg tds y Most commonly affects the agricultural workers and
y

y Whitfield’s lotion – Benzoic acid woodcutters → After a trauma → soil fungi → Enters the
subcutaneous tissue of the feet and lower legs → causes
y

Warty, Cutaneous nodules


SUBCUTANEOUS MYCOSES y Agents causing: Fonsecaea sp., Exophiala dermatitidis, Phia-
y

y Predisposing risk factor – Trauma


y
lophora verrucosa, Cladophialophora
y Types are:
y
y Laboratory diagnosis: Lesions – KOH mount - show irregular,
y

 Mycotic mycetoma

dark brown with a septae yeast like bodies – Sclerotic bodies
 Chromoblastomycosis
y Treatment: Amphotericin B, Thiabendazole, Itraconazole
366
y

and Voriconazole
y Lab diagnosis: KOH / HPE – shows asteroid body

Chapter 49     Superficial Mycoses


y

y Culture: Yeast phase in 37deg C and mycelia phase in 25deg C


y

y Serology: Slide latex agglutination test – Antigen is peptido


y

– L- rhamno – D –mannan - ≥1:4 titre is positive (mainly


helpful in pulmonary sporotrichosis)
y Treatment: KI – Potassium iodide, Itraconazole, fluconazole,
y

cryotherapy
y Itraconazole is the DOC for lymphocutaneous sporotrichosis
y

(IDSA guidelines)
y Severe pulmonary or CNS spreads needs AmB lipid therapy
y

Rhinosporidiosis
y Polyps occurring in nose, mouth, eye rarely in genitals
y

Figure 7: Sclerotic bodies (Courtesy: CDC/ Dr. Libero Ajello) y Agent: Rhinosporidium seeberi
y

y Pseudofungi; belongs to hydrophilic fungi – DRIP clade of


y

aquatic protista
Phaeohyphomycosis y Endemic in Tamil Nadu, Kerala, Orissa, AP and Sri Lanka
y

y Infections caused by pigmented fungi are called phaeohy-


y
y Source of infection – Stagnant water like pool, lakes
y

phomycosis y Diagnosis – Spherules that contain numerous endospores


y

y Colored or melanin or pigmented fungi are called as


y
are seen
dematiaceous fungi E.g. Phialophora, Cladosporium, y It cannot be cultivated
y

Alternaria y Treatment – Surgery, to prevent recurrence – Dapsone


y

y Fungi enters the body through inoculation through the skin


y

by prick; usually seen in tropical countries and rural workers.


y Melanin is the virulence factor for all pigmented moulds.
y

y Brain abscess is caused by pigmented fungi - Cladophialophora


y

bantiana
y Subcutaneous lesions with abscess are caused by Exophiala
y

jeanselmei, E.spinifera, E.dermatitidis


y Microscopy of the specimen or culture shows Brown colored
y

hyphae
y Surgical removal of the lesions with antifungal therapy is the
y

treatment.

Sporotrichosis
y Causative agent: Sporothrix schenckii (Thermally dimor-
y

phic)
y Fungal infection affecting cutaneous, subcutaneous and
y

Figure 8: Spherules of Rhinosporidium seeberi


lymphatic tissue (Courtesy: Dr.S.Jamuna Rani, Associate Professor of Pathology,
y Most commonly affects gardeners, forest workers → After
y

Tagore Medical College & Hospital, Chennai, Tamil Nadu)


a minor trauma (Rose thorn prick) → nodules formed →
ulceration → necrosis → Fixed cutaneous sporotrichosis →
spread to lymphatics → lymphocutaneous sporotrichosis → Remember
systemic spread to the bones, joints and meninges Chromoblastomycosis – Sclerotic bodies
y DD: Nocardiosis, Tularemia, Non TB mycobacterial infection
Sporotrichosis – Asteroid body
y

and leishmaniasis
Rhinosporidiosis - Spherules

367
MULTIPLE CHOICE QUESTIONS
Unit 5     Mycology

1. Tinea cruris is caused by: (Recent Pattern Dec 2015) 14. Potassium iodide is used in the treatment of:
a. Trichophyton rubrum b. M.canis a. Rhinosporidiosis b. Chromoblastomycosis
c. T.verrucosum d. T.tonsurans c. Sporotrichosis d. Entomophthoromycosis
2. Trichophyton – Zoophilic species is: 15. Dapsone is used to treat recurrence of:
 (Recent Pattern Dec 2012) a. Rhinosporidiosis b. Sporotrichosis
a. T.tonsurans b. T.violaceum c. Entomophthoromycosis d. Candidiasis
c. T.schoenleinii d. T.mentagrophytes 16. A female from Himachal Pradesh presented with history
3. Pityriasis versicolor is caused by: of thorn prick, a year back has verrucous lesions in the
 (Recent Pattern Nov 2014) skin with following microscopic findings; Identify the
a. E.floccosum b. M.gypseum agent: (AIIMS 2017)
c. M.furfur d. T.tonsurans
4. Endothrix is caused by: (Recent Pattern Nov 2015)
a. E.floccosum b. M.gypseum
c. T.tonsurans d. M.canis
5. Black dot ring worm is caused by:
 (Recent Pattern Dec 2014)
a. Microsporum b. Trichophyton
c. Epidermophyton d. Candida
6. Most common cause of tinea capitis is:
 (Recent Pattern Dec 2013)
a. M.canis b. E.floccosum
c. T.tonsurans d. M.gypseum a. Blastomycosis b. Phaeohyphomycosis
7. Which of the following infects hair, skin and nails? c. Sporotrichosis d. Chromoblastomycosis
 (Recent Pattern Dec 2012) 17. A 21 years old man presented with following skin lesion.
a. Microsporum b. Trichophyton Identify the causative agent for this infection:
c. Epidermophyton d. Trichosporon (AIIMS Nov 2018)
8. White piedra is caused by: (MH 2011)
a. Piedra hortae b. Malassasia furfur
c. Hortaea werneckii d. Trichosporon beigelli
9. Color of granules produced by Actinomadura pelletieri
 (Recent Pattern Dec 2013)
a. Black b. Yellow
c. Red d. Brown
10. Granules discharged in mycetoma contains
 (Recent Pattern Dec 2013)
a. Bone spicules b. Fungal colonies
c. Pus cells d. Inflammatory cells
11. Rhinosporidium seeberi belongs to
a. Fungus b. Aquatic Protista
c. Protozoa d. Helminth
12. Sclerotic bodies are seen in:
a. Rhinosporidiosis b. Chromoblastomycosis a. Trichophyton rubrum
c. Candidiasis d. Entomophthoromycosis b. Microsporum spp
13. Rose Gardner’s disease is c. Epidermophyton
a. Rhinosporidiosis b. Chromoblastomycosis d. Aspergillus spp
c. Sporotrichosis d. Entomophthoromycosis

ANSWERS AND EXPLANATIONS

1. Ans.  (a) Trichophyton rubrum 2. Ans.  (d) T. mentagrophytes


Ref: Ananthanarayan and Paniker’s Textbook of Microbiol- Ref: Ananthanarayan and Paniker’s Textbook of Microbiol-
ogy – 10th ed – Page 601 ogy – 10th ed – Page 604
368 • Tinea cruris - T.rubrum, E.floccosum
• • Anthropophilic - T.rubrum, T.tonsurans, T.violaceum,

T.schoenleini, M.audounii
• Zoophilic – T.mentagrophyte, T.verrucosum, M.canis

Chapter 49     Superficial Mycoses


Nocardia asteroides
• Geophilic - M.gypseum, M.fulvum

Nocardia brasiliensis
3. Ans.  (c) M. furfur
Actinomadura madurae White to yellow granules
Ref: Ananthanarayan and Paniker’s Textbook of Microbiol-
Streptomyces somaliensis
ogy – 10th ed – Page 601
• Agent: Malassezia furfur

Scedosporium apiospermum
• Culture – SDA with olive oil layer (lipophilic fungi)

Madurella mycetomatis
4. Ans.  (c) T. tonsurans Madurella grisea Brown to black
Ref: Ananthanarayan and Paniker’s Textbook of Microbiol- Exophiala jeanselmei
ogy – 10th ed – Page 601 Actinomadura pelletieri Red
• Ectothrix: Arthrospores are seen as a sheath surround-

ing the hair – caused by Microsporum, T.rubrum, T.men- 10. Ans.  (b) Fungal colonies
tagrophytes
• Endothrix: Arthrospores are inside the hair shaft – Ref: Ananthanarayan and Paniker’s Textbook of Microbiol-
ogy – 10th ed – Page 605

caused by T.schoenleinii, T.tonsurans, T.violaceum


• Triad: Swelling of the foot, Multiple sinuses, Discharge

5. Ans.  (b) Trichophyton of granules


• Granules or Grains → Micro colonies of the organisms
Ref: Ananthanarayan and Paniker’s Textbook of Microbiol-

• Color of the granules helps in presumptive identifica-


ogy – 10th ed – Page 601

tion of the organism


• Another name for Dermatophytosis is Tinea or ring

worm 11. Ans.  (b) Aquatic Protista


• Most common cause for black dot ring worm is Tricho-
Ref: Ananthanarayan and Paniker’s Textbook of Microbiol-

phyton tonsurans
ogy – 10th ed – Page 607
6. Ans.  (a) M. canis • Pseudofungi; belongs to hydrophilic fungi – DRIP clade

of aquatic Protista
Ref: Ananthanarayan and Paniker’s Textbook of Microbiol-
ogy – 10th ed – Page 601 12. Ans.  (b) Chromoblastomycosis
• Most common mode of presentation of Tinea capitis is
Ref: Ananthanarayan and Paniker’s Textbook of Microbiol-

Ectothrix
• Most common organism causing Ectothrix is Microspo-

ogy – 10th ed – Page 605
rum canis • Chromoblastomycosis: Fungal infection caused by pig-

mented fungi, also called as dematiaceous fungi


7. Ans.  (b) Trichophyton • Lab diagnosis: Lesions – KOH mount - show irregular,

dark brown with a septae yeast like bodies – Sclerotic


Ref: Ananthanarayan and Paniker’s Textbook of Microbiol-
bodies
ogy – 10th ed – Page 601

Trichophyton Skin, hair, nail 13. Ans.  (c) Sporotrichosis

Microsporum Hair and skin Ref: Ananthanarayan and Paniker’s Textbook of Microbiol-
ogy – 10th ed – Page 606
Epidermophyton Skin and nail
• Causative agent: Sporothrix scheckii (Thermally dimor-

8. Ans.  (d) Trichosporon beigelli phic)


• Most commonly affects gardeners, forest workers →
Ref: Ananthanarayan and Paniker’s Textbook of Microbiol-

After a minor trauma (Rose thorn prick) – hence called


ogy – 10th ed – Page 600
as Rose Gardner’s disease
• Black piedra – Piedraia hortae

• White piedra – Trichosporon beigelii


• 14. Ans.  (c) Sporotrichosis

9. Ans.  (c) Red Ref: Ananthanarayan and Paniker’s Textbook of Microbiol-


ogy – 10th ed – Page 606
Ref: Ananthanarayan and Paniker’s Textbook of Microbiol-
• Treatment: KI – Potassium iodide, Itraconazole, fluco-
ogy – 10th ed – Page 605

nazole, cryotherapy

369
• Lab diagnosis: Lesions - show irregular, dark brown with
Unit 5     Mycology

15. Ans.  (a) Rhinosporidiosis •

a septae yeast like bodies – Sclerotic bodies or medlar


Ref: Ananthanarayan and Paniker’s Textbook of Microbiol- bodies or copper penny bodies
ogy – 10th ed – Page 607
• Treatment: Surgery, To prevent recurrence – Dapsone

17. Ans. (a) Trichophyton rubrum

Ref: Ananthanarayan and Paniker`s T.B of microbiology –


16. Ans.  (d) Chromoblastomycosis
10th ed – page 601
Ref: Ananthanarayan and Paniker T.B of microbiology – • Above image shows Tinea infection caused by

10th ed – page 605 dermatophytes. It is seen over the beard areas – so it is


• Fungal infection caused by pigmented fungi, also called
• named as Tinea barbae (Barber`s itch)
as dematiaceous fungi • All these species of Dermatophytes causes Tinea

• Called as verrucous dermatitis (because the lesions are


• infection – species depends upon the area of infection
verrucous) and microbiological diagnosis
• Most commonly occurs after a minor trauma → soil
• • Barber`s itch is caused by Trichophyton species namely

fungi → Enters the subcutaneous tissue of the feet T. rubrum, T. mentagrophytes


and lower legs → causes Warty, verrucous, cutaneous
nodules

370
Endemic/Systemic 50
Mycoses
CHARACTERISTICS
y Most of the infections are caused by soil fungi
y
y Systemic infections are caused by dimorphic fungi.
y
Blastomyces dermatitidis
Paracoccidioides brasiliensis
Coccidioides immitis
Histoplasma capsulatum

BLASTOMYCOSIS
Figure 1: Cutaneous lesions of Blastomycosis
y Causative agent: Blastomyces dermatitidis (Courtesy: CDC/ Dr. Lucille K. Georg)
y
y Telemorph stage (sexual stage): Ajellomyces dermatitidis
y Antigen detection in urine is more sensitive than in serum.
y
y Region: North America (North American blastomycosis)
y
y
y Source of infection is soil
y
y Mode of infection is inhalation fungal spores enters the body Treatment
y
through inhalation from the soil and then goes to lungs.
Alveolar macrophages and polymorphonuclear leukocytes Table 1:  Treatment of blastomycosis
are most needed for phagocytosis and killing of inhaled Patient condition Type of Treatment
conidia. disease
y Conidia upon entering inside the body, at body temperature
Immunocompetent Pulmonary Lipid AmB or Itraconazole
y
(37°C) – it gets converted into thick walled yeast which are
patient/life
difficult to kill by phagocytosis.
threatening disease Disseminated Lipid AmB or fluconazole
y This conidia to yeast phase conversion leads to expression of
CNS
y
protein called as BAD-1 (virulence factor)
y It causes acute pulmonary infection (resembles that of TB/ Disseminated Lipid AmB or itraconazole
y
histoplasmosis) Non - CNS
y Pulmonary presentation – may be asymptomatic; sometimes
y
consolidation and abscess can occur Immunocompetent Pulmonary or Itraconazole or
y Fungus then spreads to the blood and to various organs patient/ non-life disseminated fluconazole or
y
(Disseminated blastomycosis) threatening disease non CNS ketoconazole
y In cutaneous blastomycosis, a papule occurs followed by
y
nodule develops – leading to ulcerative lesions; Site is usually Immuno- All infections Lipid AmB or AmB
skin of face or hands; Two types of skin diseases are seen: compromised deoxycholate
 Verrucous blastomycosis patient

 Ulcerative blastomycosis

y It also causes osteomyelitis, which usually affects vertebrae, PARACOCCIDIOIDOMYCOSIS
y
sacrum, pelvis, skull and ribs.
y CNS blastomycosis presents as brain abscess. y Causative agent: Paracoccidioides brasiliensis
y
y
y Region: South America (South American blastomycosis)
Laboratory Diagnosis
y
y Source of infection is soil
y
y Body tissue/Culture at 37°C shows Yeast phase – single broad y Mode of infection is inhalation
y
y
bud with double contoured wall y Fungal spores are inhaled: Primary pulmonary infection –
y
y Culture at 25°C (Room temperature) shows Septate hyphae spreads through blood – to mucosa of nose, mouth, GIT, skin,
y
with round to oval conidia lymphatics
y Leads to ulcerative granuloma of the buccal and nasal Š Immunodiffusion test
Unit 5     Mycology

y Š

mucosa Š EIA (most frequently used screening technique)


Š

y Laboratory diagnosis:
y
y Organism is highly infective; bioterrorism agent; should be
y

 Body tissue/Culture at 37°C – Yeast phase – large, globose or



worked always in biological safety cabinet.
oval cells with multiple buds surrounding the mother cell
 Culture at 25°C (Room temp) – Septate hyphae with small


conidia
 Appearance: Mariner’s wheel or Pilot’s wheel or Cap-


tain’s wheel

COCCIDIOIDOMYCOSIS
y Causative agent: Coccidioides immitis
y

y Two species are there: C. immitis and C. posadasii


y

y Region: USA – called as Rift valley fever or Desert Rheuma-


y

tism
y Source of infection is soil
y

y Mode of infection is inhalation


y

y Arthrospores which are present in the dust are inhaled and it


y
Figure 2: Spherule of Coccidioides (Tissue stage)
enters the lungs (Courtesy: CDC/ Dr. Lucille K. Georg)
y 60% of the infected individuals are completely asymptomatic
y

and 40% have symptoms

Clinical Features
y Respiratory infection is mostly asymptomatic and it gives
y

lifelong immunity
y It presents as a flu like illness which is mild and self-limiting,
y

called as Desert rheumatism


y DD: Community acquired bacterial pneumonia
y

y In very few persons, there occurs a chronic progressive


y

disseminated disease – Coccidioidal granuloma (Fatal)


y Cutaneous manifestations of primary pulmonary
y

coccidioidomycosis are toxic erythema, erythema nodosum


and arthralgia.
Figure 3: Barrel shaped arthroconidia of Coccidioides
y Rarely, Coccidioidal meningitis can occur which is always
y

fatal.
Treatment
Table 2:  Clinical clue for primary pulmonary coccidioi- Table 3:  Treatment of Coccidioidomycosis
domycosis
Clinical type Treatment
• History of residence or travel to endemic area (western

Asymptomatic infection None


hemisphere)
• History of night sweats

Primary pneumonia None
• Profound fatigue

Diffuse pneumonia Amphotericin B followed by oral
• Peripheral blood eosinophilia

itraconazole for months
• Hilar or mediastinal lymphadenopathy in CXR
Pulmonary sequelae None, but when there is

• Failure to improve with antibiotics


chronic pneumonia start on oral

itraconazole
Laboratory Diagnosis Disseminated disease Lifelong triazole therapy for
 Body tissue/Culture at 37°C shows Spherules which are
meningitis


thick double layered wall filled inside with endospores


 Culture at 25°C (Room temperature) in blood agar shows
HISTOPLASMOSIS


colonies; these colonies when seen under microscope have


Hyphae that looks fragmented giving rise to arthrospores
y Causative agent: Histoplasma capsulatum
(Barrel shaped)
y

 Two types: H.capsulatum.var.capsulatum, H.capsulatum.


DTH skin test with coccidioidin – positive (used in endem-



var.duboisii


ic areas)
 Disease identified by Darling – hence called as Darling’s
Serological tests play a major role in diagnosis:



disease


372 Š Tube precipitin test


Š

Š Complement fixation test


Š
y Region: USA, Africa y Specimens from tissues shows yeast cells which are seen

Chapter 50     Endemic/Systemic Mycoses


y y

y H.capsulatum.var.duboisii – responsible for African histo-


y within phagocytic cells (intracellular)
plasmosis y Growth at 37°C produces yeast cells
y

y Source of infection is soil, rotting trees, birds


y y Room temperature – Spores with tubercles (finger like pro-
y

y Mode of infection is inhalation of microconidia


y jections)
y Inhalation of spores – pulmonary entry
y y Detection of histoplasma antigen in body fluids helps in di-
y

agnosis of PDH and acute diffuse pulmonary histopladmosis.


y Histoplasmin skin test helps in endemic area to know the
Clinical Features y

disease burden.
y In immunocompetent individuals with low level of exposure
y

– infections are usually mild and self-limiting


y Most of the adults in endemic area have 50-80% positive skin
y

tests suggest that they had mild infections.


y Heavy exposure leads to flu like illness with CXR showing
y

pneumonitis with hilar or mediastinal lymphadenopathy.


y Classical histoplasmosis – asymptomatic – heal with just a
y

miliary calcification
y DD: Tuberculosis
y

y Progressive Disseminated histoplasmosis (PDH) – occurs


y

only in immunocompromised individuals which affects


Reticuloendothelial system (intracellular infection) and it is
highly fatal

Risk factors for PDH are:


• AIDS with CD4 counts less than 200/uL
• Figure 4: Tuberculate spores (Courtesy: CDC)
• Extremes of age

• Immunosuppressive drug therapy



Treatment
• Inflammatory diseases

Table 4:  Treatment of histoplasmosis


y Lymphadenopathy, hepatosplenomegaly, fever, anaemia
y

Type of histoplasmosis Treatment


occurs
y African histoplasmosis – affects skin, subcutaneous tissues Acute pulmonary illness Lipid amphotericin B ±
with diffuse infiltrates glucocorticoids for 1-2 weeks
y

and bones; lungs infection and dissemination is very rare.


y Chronic cavitary histoplasmosis (resembles TB) is seen in Followed by
Itraconazole for 12 weeks
y

smokers.
y In healed histoplasmosis – the calcified mediastinal nodes or
y

Chronic/Cavitary Itraconazole bd for at least 12


lung parenchyma erodes the airways and causes hemoptysis. pulmonary months
This condition is called as broncholithiasis.
Progressive disseminated Lipid amphotericin B for 1-2 weeks
followed by Itraconazole for 12
Laboratory Diagnosis weeks
y Gold standard for diagnosis is fungal culture
y

y Cultures are positive in 75% cases of PDH and chronic


y
Central nervous system Lipid amphotericin B for 4-6 weeks
pulmonary histoplasmosis involved followed by Itraconazole for 12
months

373
MULTIPLE CHOICE QUESTIONS
Unit 5     Mycology

1. Which of the following is not an endemic mycosis: 6. A patient with HIV develops diarrhea n fecal examination
 (Recent Pattern Dec 2013) shows Isospora belli. He was given treatment with TMP-
a. Histoplasmosis b. Blastomycosis SMX. Diarrhea subsided but fever persisted. Bone
c. Cryptococcosis d. Candidiasis marrow examination showed the following picture with
2. Darling disease is caused by (Recent Pattern Dec 2012) an intracellular fungi. Which of the following is wrong
a. Histoplasma b. Candida statement:
c. Cryptococcus d. Rhizopus
3. “Tuberculate spores” are characteristic feature of
 (Recent Pattern Dec 2014)
a. Candida b. Histoplasma
c. Coccidioidomycosis d. Cryptococcus
4. Fungus that infects reticuloendothelial cells is:
 (Recent Pattern July 2016)
a. Cryptococcus
b. Candida
c. Aspergillus
d. Histoplasma
5. Valley fever or desert rheumatism is caused by
 (Recent Pattern Dec 2013)
a. Sporothrix a. It cannot be grown in SDA
b. Coccidioides b. Spores are infective form
c. Phialophora c. It is intracellular budding yeast
d. Histoplasma d. It can cause systemic disease

ANSWERS AND EXPLANATIONS


1. Ans.  (d) Candidiasis ƒ 37°C – yeast cells
ƒ

ƒ Room temperature – Spores with tubercles (finger like


Ref: Ananthanarayan and Paniker’s Textbook of Microbiol-
ƒ

projections)
ogy – 10th ed – Page 609
• • Blastomyces dermatitidis 4. Ans.  (d) Histoplasma
• Paracoccidioides brasiliensis
Ref: Ananthanarayan and Paniker’s Textbook of Microbiol-

• Coccidioides immitis
ogy – 10th ed – Page 610

• • Histoplasma capsulatum
• • Cryptococcus sp • Classical histoplasmosis – asymptomatic – heal with just

a miliary calcification
2. Ans.  (a) Histoplasma • Disseminated histoplasmosis – occurs in few – affects

Reticuloendothelial system (intracellular infection) –


Ref: Ananthanarayan and Paniker’s Textbook of Microbiol-
highly fatal
ogy – 10th ed – Page 609
• Lymphadenopathy, hepatosplenomegaly, fever, anemia

• Histoplasmosis - Causative agent: Histoplasma capsula-



occurs
tum
ƒ Two types: H.capsulatum.var.capsulatum, H.capsula-
ƒ 5. Ans.  (b) Coccidioides
tum.var.duboisii
Ref: Ananthanarayan and Paniker’s Textbook of Microbiol-
ƒ Disease identified by Darling – hence called as
ogy – 10th ed – Page 611
ƒ

Darling’s disease
• Coccidioidomycosis - Causative agent: Coccidioides im-

3. Ans.  (b) Histoplasma mitis


• Region: USA – called as Rift valley fever or Desert Rheu-
Ref: Ananthanarayan and Paniker’s Textbook of Microbiol-

matism
ogy – 10th ed – Page 610
• Lab diagnosis of Histoplasmosis:

ƒ Tissues – yeast cells occurs within phagocytic cells


374
ƒ

(intracellular)
6. Ans.  (a) It cannot be grown in SDA ƒ Lymphadenopathy, hepatosplenomegaly, fever, ane-

Chapter 50     Endemic/Systemic Mycoses


ƒ

mia occurs.
Ref: Ananthanarayan and Paniker T.B of microbiology –
ƒ African histoplasmosis: affects skin, subcutaneous
10th ed – page 610
ƒ

tissues and bones; lungs infection and dissemination


• Causative agent: Histoplasma capsulatum

is very rare.
• Mode of infection is inhalation of spores

• Lab diagnosis:

• Inhalation of spores: pulmonary entry


ƒ Tissues – yeast cells occurs within phagocytic cells


ƒ

ƒ Classical histoplasmosis: asymptomatic – heal with


ƒ

(intracellular)
just a miliary calcification ƒ 37°C – yeast cells
ƒ

ƒ Disseminated histoplasmosis: occurs in few – affects


ƒ
ƒ Room temperature – Spores with tubercles (finger like
ƒ

Reticuloendothelial system (intracellular infection) projections)


highly fatal ƒ Fungi grows in SDA and blood agar
ƒ

ƒ SDA – colonies are white, cottony, mycelial growth


ƒ

375
51
Opportunistic Mycoses
• Aspergillosis Table1: Microscopic appearance of various species of


Aspergillus

• Penicilliosis

• Zygomycosis Species Microscopic appearance

• Candidiasis Aspergillus Uniseriate, Conidia covers only upper
fumigatus one third of vesicle,

• Cryptococcosis
Aspergillus flavus Uniseriate and Biseriate, covers entire

• Pneumocystis jirovecii vesicle

Aspergillus niger Biseriate, covers entire vesicle – Black
ASPERGILLOSIS colored
y Aspergillosis is a clinical term used to describe all diseases
y
that are caused by species of aspergillus;
y The species that grows at 37°C can cause invasive infections
y
and others can cause only allergic manifestation.
y Most common species are: Aspergillus fumigatus, Aspergillus
y
flavus, Aspergillus niger and Aspergillus terreus.
y Aspergillus is seen ubiquitously in all the environments
y
especially in decomposing plant materials and in bedding.
y Incubation period of invasive aspergillosis after inhalation is
y
2 to 90 days.
y Main important risk factors for aspergillosis are:
y
 Profound neutropenia Figure 1: Acute angled hyphae of Aspergillus (Courtesy: CDC)

 Glucocorticoid usage

Clinical Features
y Clinical diseases caused by Aspergillus are:
y
 ABPA – Allergic Bronchopulmonary Aspergillosis – Type

I (M/c) and Type III hypersensitivity reactions that occurs
after inhalation of spores – spores enters inside lungs and
grows within the lumen of bronchioles – occludes them
 Aspergilloma – Fungus grows within and occurs as a

fungal ball
 Invasive aspergillosis – Pneumonia occurs then dissemi-

nation starts to involve all organs – occurs in immunocom-
promised individuals, and those who are taking chronic
steroids.
 Otomycosis, mycotic keratitis, sinusitis

 Cerebral aspergillosis

 Cutaneous aspergillosis y Serological test: Antibodies can be demonstrated by ELISA,

y
 Endocarditis counter current immunoelectrophoresis and immunodiffu-

sion
Laboratory Diagnosis y Serological tests are most helpful in allergic aspergillosis
y
y Microscopy – Septate hyphae with acute angle branching at y Intradermal injection of Aspergillus antigen – Immediate
y
y
45°C reaction (Type I hypersensitivity) and Arthus type reaction
y Morphology of each species differs and identification is (Type III reaction)
y
mainly by the conidial arrangement y Beta-D-glucan assay helps in invasive aspergillosis
y
y Criteria for diagnosing ABPA includes microbiological pa- ZYGOMYCOSIS

Chapter 51     Opportunistic Mycoses


y

rameters like:
 Serum precipitating antibodies

y Zygo/Mucormycosis is a group of life threatening infections
y

 Elevated serum IgE



that are caused by fungus belonging to order mucorales
 Elevated serum IgE and IgG antiaspergillus antibodies

y Infections have high morbidity and high mortality.
y

 Arthus (late) skin reaction



y Other names: mucormycosis and Phycomycosis
y

y Organisms causing: Rhizopus, Mucor, Rhizomucor, Absidia


y

Definitive confirmation of aspergillosis needs: y These fungi are seen ubiquitously in the moist environment
y

• Positive culture of a sample taken directly from an ordinarily



causing exposure to humans often.
sterile site like brain abscess site y But infections are caused mainly to diabetics and other
y

• Positive results of both histologic testing and culture of a



immunosuppressive patients.
sample taken from an affected organ
Risk factors of Zygomycosis:
Treatment • Diabetes mellitus especially diabetic ketoacidosis

• Patients on glucocorticoid therapy


Table 2:  Treatment of Aspergillosis

• Neutropenic patients

Type of Primary Secondary treatment • Iron overloaded patients (end stage renal failure patients)
aspergillosis treatment

• Haematopoietic stem cell transplantation patients


Invasive Voriconazole AmB, Caspofungin,


aspergillosis Micafungin y Pathogenesis: Spores from the soil or environment enters the
y

body – respiratory cavity – enters the orbit, sinuses and brain;


Prophylaxis Posaconazole or Micafungin, Aerosolized
systemic dissemination (occurs most commonly in diabetics
Itraconazole AmB
and immunosuppressed individuals, long term steroids);
Single Surgery Itraconazole, Fungus causes vascular invasion and leads to thrombosis
aspergilloma Voriconazole and infarction thus it is a fatal condition
Chronic Itraconazole or Posaconazole, IV
pulmonary Voriconazole micafungin Clinical Features
ABPA Itraconazole Voriconazole, y Six categories of mucormycosis are seen:
y

Posaconazole  Rhino-orbital-cerebral


 Pulmonary


PENICILLIOSIS  Cutaneous


 Gastrointestinal


y Ubiquitous species present in the environment


y
 Disseminated


y Clinical features: Fever, Papulo necrotic skin lesions, weight


y
 Miscellaneous


loss, anemia, lymphadenopathy, hepatosplenomegaly y Rhinocerebral mucormycosis:


y

y In HIV infected individuals, papulonecrotic skin lesions seen


y
 Most common form is Rhinocerebral form


which are caused by Penicillium marneffi. It is endemic in  Patient presents with eye or facial pain and numbness and


north east India, Thailand and Myanmar. conjunctival suffusion.


y Penicillium marneffei – Dimorphic fungi
y
 Severe cases present with bilateral proptosis, chemosis,


y Produces dark red colored pigment in culture media


y
vision loss and opthalmoplegia, which indicates cavernous
y Treatment: Amphotericin B, Oral Itraconazole
y
sinus thrombosis.
 When there is a sign of painful necrotic ulcerations in the


hard palate – it indicates well established infection that is


almost fatal

Laboratory Diagnosis
y Microscopy of the clinical specimen shows: Broad, aseptate
y

hyphae
y Culture: Thick cottony fluffy colonies
y

Mucor sp., Sporangiophores arising randomly along the


aerial mycelium; No rhizoids

Rhizopus sp., Sporangiophores arise in groups directly above


the rhizoids
Figure 2: Pigmented Penicillium marneffei
(Courtesy: CDC/ Dr. Libero Ajello) 377
y Balanitis
Unit 5     Mycology

y Erosio interdigitalis blastomycetica – infection between the


y

digits of hands and toes.


y Chronic mucocutaneous candidiasis (CMC) – have an
y

association with endocrine abnormalities named as APECED


syndrome – autoimmune polyendocrinopathy, candidiasis,
ectodermal dystrophy. This syndrome is due to mutations in
the AIRE gene.
y Septicemia
y

y Meningitis
y

y Endocarditis
y

Table 3:  Risk factors for disseminated candidiasis


Figure 3: Mucor sp., (Courtesy: CDC/Dr. Lucille K. Georg) • History of anti bacterial
• • AIDS

y When organisms grow in the culture plate – it is must to know agents


• Indwelling intravenous or • History of surgery (abdominal)
y

about the clinical condition – as these fungi are environmental • •

contaminants of culture plates; Clinical correlation should be urinary catheters • History of immunosuppresive

done to know whether the organism is a true pathogen • Hyperalimentation fluids


• drugs intake
y Most sensitive and specific modality for definitive diagnosis • Neutropenia

• Intake of steroids • Low birth weight neonates


y

is Biopsy with histopathological examination. Biopsy • •

clearly shows wide, thick walled, ribbon like, aseptate hyphal • Severe burns
• • Diabetes mellitus

elements that branch at right angles.


y MRI is most sensitive method in orbital and CNS diseases.
y

Laboratory Diagnosis
y Gram staining of the specimen shows Gram positive budding
Treatment
y

yeast cells with pseudohyphae or hyphae


Primary antifungal AmB deoxycholate
therapy Liposomal AmB
Primary combination Caspofungin + lipid polyene
therapy Micafungin + Anidula fungin + lipid
polyene

CANDIDIASIS
y Candida can cause almost all of the fungal infections de-
y

scribed except mycetoma


y Most common species: Candida albicans – yeast like fungi
y

y Other species are grouped together named as Candida


y

non albicans – C.glabrata, C.tropicalis, C.keyft, C.krusei, Figure 4: Yeast cells in vaginal smear
C.guilliermondii, C.parapsilosis and C.stellatoidea (Courtesy: Image from author’s own thesis)
y They are normal commensals in skin and mucosa in our body
y y It has pseudohyphae
y

y When immunosuppression occurs, they become pathogenic


y y To differentiate whether it is a commensal or pathogen –
y

demonstration of mycelial forms is necessary


y SDA growth shows Creamy white colonies
Clinical Features y

y Differentiation of Candida species is based on glucose


y

y Cutaneous candidiasis: Affects mainly groin, perineum,


y

fermentation and assimilation tests, chlamydospore


axillae and folds production, color production in CHROM agar
y Paronychia: Painful swelling in nail and skin interface
y

y Corn meal agar shows production of chlamydospores which


y

y Onychomycosis: Fungal infection of nail


y

is diagnostic of Candida albicans (called as Dalmau plate


y Vulvo vaginitis: Presents as pruritis, pain and vaginal curdy
y

culture technique)
white discharge. y Germ tube production – Reynolds Braude phenomenon -
y

y Oral thrush: Characterized by white, adherent, painless,


y

diagnostic of Candida albicans


discrete pateches in mouth, tongue and in esophagus. y CHROM agar shows the colors produced by different species
y

y Intestinal candidiasis (occurs in patients who are taking


y

which helps in identification of species.


chronic antimicrobial therapy) y Β –D- glucan assay helps in invasive candidiasis
y

378
CRYPTOCOCCOSIS

Chapter 51     Opportunistic Mycoses


y Cryptococcus is a yeast fungi that is the etiological agent for
y

cryptococcosis.
y Cryptococcosis also called as European blastomycosis, Tor-
y

ulosis
y Main agent:
y

 Cryptococcus neoformans var neoformans




 Cryptococcus neoformans var.gattii




 Cryptococcus neoformans var. grubii




y Capsular serotypes: A, B, C, D (four types)


y

y Capsule is made up of polysaccharide – it inhibits phagocy-


y

tosis
y Source of infection: Pigeon feces (types A and D) and Euca-
y

lyptus tree bark (Type B)

Cryptococcus Source is from Avian excreta (Pigeon


neoformans droppings)
Figure 5: Dalmau plate culture technique done for speciation of
Candida (Source: Image from author’s own thesis work) Cryptococcus gattii Source is from Eucalyptus tree bark

y Other non neoformans species are – C. albidus, C. laurentii


y

y Teleomorph or sexual stage – Filobasidiella neoformans


y

y Pathogenesis: Route of entry is inhalation:


y

 Pulmonary cryptococcosis (M/c form) – mild pneumonitis




 Cryptococcal meningitis – (M/c) in AIDS patients – region




affected are basal ganglia or head of caudate nucleus


 It also affects skin and bone


Clinical Features
y Usually it presents as chronic meningoencephalitis
y

y Most common cause of meningitis in AIDS patients is Cryp-


y

tococcus
y Clinically it shows headache, fever, lethargy, sensory deficits,
Figure 6: Candida species in microscopy
y

memory deficits, cranial nerve palsies, vision defects and


(Courtesy: CDC/Dr. Stuart Brown)
meningismus.
Treatment y Meningeal cryptococcosis can lead to sudden vision loss.
y

y Lesions are most commonly seen in basal ganglia and at the


y Imidazoles, Amphotericin B, 5-Fluorocytosine, Disseminated
y

head of caudate nucleus


y

infections – Echinocandins
y Skin lesion is seen in patients with disseminated cryptococ-
y

cosis. It may present as papules, plaques, purpura, vesicles


Table 4:  Treatment of Candidiasis and rashes.
Type of Prime drug Alternative
Candidiasis therapy Laboratory Diagnosis
Cutaneous Topical azoles Topical nystatin y CSF analysis on performing wet mount with India Ink or
y

Vulvovaginal Oral flucanazole + Nystatin nigrosine stain shows capsulated yeast cells
suppositories suppository y Cell count for CSF shows mononuclear cell pleocytosis with
y

increased protein levels


Oral thrush Clotrimazole paints Nystatin or
y Serological assay that has CRAg detection in CSF and
Flucanazole
y

blood is helpful; it is done by Latex agglutination test – for


Esophageal Fluconazole tablets or Caspofungin, polysaccharide antigen (most useful in meningitis – bed side
candidiasis itraconazole solutions Micafungin or test)
amphotericin B y Special media: Bird seed agar, sunflower seed agar
y

Disseminated Amphotericin B
Candidiasis deoxycholate
Treatment
Azoles y Amphotericin B with Flucytosine as lifelong maintenance
y

Echinocandins therapy
379
Unit 5     Mycology

Figure 7: Capsulated yeast cells in wet mount of CSF


(Courtesy: CDC/ Dr. Leanor Haley)

380
MULTIPLE CHOICE QUESTIONS

Chapter 51     Opportunistic Mycoses


1. Aspergillus causes all except:(Recent Pattern Dec 2013) 10. Orbital mucormycosis is a complication of
a. Bronchopulmonary allergy a. AIDS b. Steroid therapy
b. Otomycosis c. Cushing’s disease d. Diabetic ketoacidosis
c. Dermatophytosis 11. 1,3 beta D-glucan is helpful for identification of:
d. Keratitis  (AIIMS Nov 2017)
2. Allergic BPA which kind of hypersensitivity reaction a. Invasive candidiasis b. Rhizopus
 (Recent Pattern Nov 2015) c. Cryptococcus d. Mucormycosis
a. Type I b. Type II 12. Identify the following organism
c. Type III d. Type IV  (AIIMS Nov 2017)
3. Aflatoxins are produced by  (Recent Pattern Dec 2013)
a. Aspergillus flavus b. Aspergillus niger
c. Aspergillus fumigatus d. Candida
4. Renauld Braude phenomenon is seen in
 (Recent Pattern Dec 2012)
a. Candida albicans b. Candida glabrata
c. Histoplasma d. Cryptococcus
5. Candida albicans causes all the following except
 (Recent Pattern Dec 2015)
a. Endocarditis b. Mycetoma
c. Meningitis d. Oral thrush
6. Latex agglutination study of the antigen in CSF helps in
the diagnosis of (Recent Pattern Jul 2016)
a. Acute angle branching Aspergillus
a. Cryptococcus b. Candidiasis

b. Obtuse angled Mucor


c. Aspergillosis d. Histoplasmosis
c. Acute angled Penicillium
7. Primary site of infection of Cryptococcus is
d. Obtuse angle Rhizopus
(Recent Pattern Dec 2013)
13. A 20-year-old female patient with HIV antibody test


a. Adrenal b. Bone
shows positive result showed a CD4 count of 50. She has
c. Central nervous system d. Lung
been diagnosed with Cryptococcal meningitis. Which of
8. A patient presented with headache and projectile
the following is/are suits this condition? 
vomiting along the alteration in sensorium. The
(PGI pattern 2017)
following parasite demonstrated on Indian Ink staining.


a. Collection of CSF and detection of capsular polysaccharide


What is the likely diagnosis? (AIIMS Nov 2015)
of the organism helps in diagnosis of Cryptococcal
meningitis
b. On bed side, Dark field microscopy helps in diagnosis
c. Mode of infection is mainly due to food contaminated with
pigeon excreta
d. Long term prophylactic treatment is must with Flucanozole
to prevent further episodes
e. Isolation is must as it leads to transmission to other patients
in ward
14. Which of the following fungus gives germ tube test posi-
tive? (PGI pattern 2017)
a. Mucor sp., b. Candida albicans
c. Candida dubliniensis d. All candida species
e. Aspergillus sp.,
a. Cryptococcus b. Blastomyces 15. Which of the following will give positive β-D-Glucan
c. Histoplasma d. Coccidioides test: (PGI May 2018)
9. A diabetic patient presents with pus from eye. Colonies a. Mucor
of isolated organisms are black with microscopic feature b. Candida
of non-septate hyphae and obtuse branching. Diagnosis c. Pseudomonas aeruginosa
is (Recent Pattern Dec 2013) d. Aspergillus
a. Aspergillosis b. Candidiasis e. Cryptococcus
c. Mucormycosis d. Histoplasmosis
381
Unit 5     Mycology

16. Following test is shown positive for identification of: (AIIMS May 2018)

a. Candida albicans b. Candida glabrata


c. Candida parapsilosis d. Candida tropicalis

ANSWERS AND EXPLANATONS


1. Ans.  (c) Dermatophytosis 5. Ans.  (b) Mycetoma
Ref: Ananthanarayan and Paniker’s Textbook of Microbiol- Ref: Ananthanarayan and Paniker’s Textbook of Microbiol-
ogy – 10th ed – Page 613 ogy – 10th ed – Page 616
• ABPA – Allergic Broncho Pulmonary Aspergillosis

• Clinical features of Candidiasis


• Aspergilloma – Fungus grows within and occurs as a

ƒ Cutaneous candidiasis – affects mainly groin, perine-


ƒ

fungal ball um, axillae and folds


• Invasive aspergillosis

ƒ Paronychia
• Otomycosis, mycotic kertatitis, sinusitis
ƒ

ƒ Onychomycosis
ƒ

ƒ Vulvo vaginitis
ƒ

2. Ans.  (a) Type I ƒ Oral thrush


ƒ

ƒ Intestinal candidiasis (occurs in patients who are


Ref: Ananthanarayan and Paniker’s Textbook of Microbiol-
ƒ

taking chronic antimicrobial therapy)


ogy – 10th ed – Page 613 ƒ Septicemia
ƒ

• ABPA – Allergic Broncho pulmonary Aspergillosis –


• ƒ Meningitis
ƒ

Type I (M/c) and Type III hypersensitivity reactions that ƒ Endocarditis


ƒ

occurs after inhalation of spores – spores enters inside


lungs and grows within the lumen of bronchioles – 6. Ans.  (a) Cryptococcus
occludes them Ref: Ananthanarayan and Paniker’s Textbook of Microbiol-
ogy – 10th ed – Page 618
3. Ans.  (a) Aspergillus flavus
• Latex agglutination test – for polysaccharide antigen

Ref: Ananthanarayan and Paniker’s Textbook of Microbiol- (most useful in meningitis – bed side test)
ogy – 10th ed – Page 619
• Fungal toxins contaminates the food – mycotoxicoses

7. Ans.  (d) Lung
• Aflatoxins are mostly produced by Aspergillus flavus and

Ref: Ananthanarayan and Paniker’s Textbook of Microbiol-


Aspergillus parasiticus ogy – 10th ed – Page 618
4. Ans.  (a) Candida albicans • Pathogenesis: Route of entry is inhalation;

• Pulmonary cryptococcosis (M/c form) – mild pneumo-


Ref: Ananthanarayan and Paniker’s Textbook of Microbiol- nitis


ogy – 10th ed – Page 616
• When few candida colonies are incubated in human

8. Ans.  (a) Cryptococcus
serum for two hours - Germ tube production occurs– Ref: Ananthanarayan and Paniker’s Textbook of Microbiol-
Reynolds Braude phenomenon - diagnostic of Candida ogy – 10th ed – Page 617
albicans
• Clinical picture with diagnostic image confirms the

diagnosis of Cryptococcal meningitis


382
• CSF analysis – Wet mount with India Ink or nigrosine • Cryptococcus neoformans – oval budding yeast, with

Chapter 51     Opportunistic Mycoses


• •

stain shows capsulated yeast cells polysaccharide capsule surrounding it


• Grows abundantly in soil containing bird dropping –

9. Ans.  (c) Mucormycosis Pigeon excreta


Ref: Ananthanarayan and Paniker’s Textbook of Microbiol- • Humans occur infection by inhalation of organism from

ogy – 10th ed – Page 614 pigeon droppings


• No human to human transmission
• Spores from the soil or environment enters the body –

• Usually asymptomatic, or mild infection


respiratory cavity – enters the orbit, sinuses and brain; • In immunocompromised individuals – Dissemination

systemic dissemination (occurs most commonly in occurs – meningitis occurs; HIV – IRIS – Cryptococcal
diabetics and immunosuppressed individuals, long meningitis occurs
term steroids) • Lab diagnosis: Spinal fluid aspiration – India ink
• Most common form is Rhinocerebral form

preparation; or latex particle agglutination test for


detection of capsular polysaccharide
10. Ans.  (d) Diabetic ketoacidosis ; (b) Steroid therapy
• Treatment: Liposomal amphotericin B (Treatment);

Ref: Ananthanarayan and Paniker’s Textbook of Microbiol- Flucanozole (long-term prophylaxis)


ogy – 10th ed – Page 614
14. Ans.  (b) Candida albicans; (c) Candida dubliniensis
• Already explained Q.9

Ref: Review of medical microbiology – 13th edition – Levinson


When a question like this is asked in NEET, choose DKA – page 903
over steroid therapy
• Germ tube test helpful in classification and identification

of Candida species
11. Ans.  (a) Invasive candidiasis • Growth inoculated in human serum for two hours and

on visualization with wet mount preparation shows


Ref: T.B of mycology – Jagdish Chander – 3rd edition – Page 353
elongated buds from the yeast cells – germ tube – rey-
• Antigen which is helpful in diagnosis of invasive fungal

nold braude phenomenon
infections is 1,3-beta D glucan test • Positive only in Candida albicans and Candida dublin-

• It is called as G test
• ensis
• Helpful to diagnose invasive candidiasis, invasive asper-
• • To differentiate above both, growth at 45°C will be

gillosis positive in dublinensis.


• But it cannot distinguish whether it is aspergillosis or

candidiasis 15. Ans.  (b) Candida; (d) Aspergillus


Ref: Ananthanarayan and Paniker’s Textbook of
12. Ans.  (a) Acute angle branching Aspergillus
Microbiology – 10th ed – Page 616 Q.11
Ref: Ananthanarayan and Paniker T.B of microbiology –
10th ed – page 613 16. Ans.  (a) Candida albicans
• Most common species are: Aspergillus fumigatus, As-

Ref: Ananthanarayan and Paniker’s Textbook of


pergillus flavus, Aspergillus niger Microbiology – 10th ed – Page 616
• Microscopy – Septate hyphae with acute angle branching
The above image shows Germ tube production

at 45°C
• When few candida colonies are incubated in human
• Morphology of each species differs and identification is

serum for two hours - Germ tube production occurs–


mainly by the conidial arrangement


Reynolds Braude phenomenon - diagnostic of Candida
• Above image is GMS stain from tissue sections
albicans

13. Ans.  (a) Collection of CSF and detection of capsular


polysaccharide of the organism helps in diagnosis of
Cryptococcal meningitis; (d) Long term prophylactic
treatment is must with Flucanozole to prevent further
episodes
Ref: Review of medical microbiology – 13th edition – Levinson
– page 910

383
52
Miscellaneous Fungi
PNEUMOCYSTIS JIROVECII MYCOTIC POISONING
y Pneumocystis is an opportunistic pathogen that is an Mycetism
y
important etiological agent of pneumonia in immuno-
compromised individuals. y This means eating fungus itself is a poison

y
y Thought as a protozoan, now classified as fungi – ascomycetes Claviceps species Ergot poisoning
y
y Former name: Pneumocystis carinii Caprine species Coprine poisoning
y
y Most commonly seen in AIDS patient
Inocybe species Muscarine poisoning
y
y Morphological forms
y
 Trophozoites

 Precyst Mycotoxicosis

 Cyst

y Route of infection is through respiratory; Source: droplets y This happens due to fungal toxins that contaminates the food
y
y
y It has a unique tropism for lung and hence it attacks the
Mycotoxin Fungus
y
alveolar capillaries after entering through inhalation.
y Usually asymptomatic: It causes fatal pneumonia only in Aflatoxin Aspergillus sp.,
y
immunosuppressed patients (HIV) Fumonisin Fusarium
Laboratory Diagnosis Ochratoxin Aspergillus and Penicillium
 Ideal specimen: Bronchoalveolar lavage (BAL) - demons- Ergotoxin Claviceps purpurea

tration of organisms is 100% sensitive and specific. Zearalenone Fusarium graminearum
 Stains: Giemsa, Toluidine blue, Methenamine silver,

Calcofluor white stain – for Trophozoites
 Methanamine silver stain- for Cysts OCULOMYCOSIS

 Not cultivable
Table 1:  Fungi causing keratitis

 Complement fixation test titer ≥1:4 – active disease

 Classic CXR findings are diffuse bilateral interstitial • Fusarium solani • Dematiaceous • Candida albicans




infiltrates that are perihilar and symmetric. • Aspergillus fungi • Candida tropicalis


fumigatus • Candida parapsilosis
Treatment

• Paecilomyces sp • Cryptococcus


y Drug of choice: TMP-SMZ (Hence when CD4 count comes • Penicillium spp neoformans

y
down in HIV patients – TMP-SMZ is started as prophylaxis)
y Alternative drugs given are TMP + Dapsone/Atovaquone/ FUNGI CAUSING ENDOPHTHALMITIS
y
Clindamycin + Primaquine or pentamidine (in sulfa drug
allergic patients) y Candida sp
y
y Aspergillus sp
y
y Fusarium sp
y
y Penicillium sp
y
y Cryptococcus neoformans
y
y Blastomyces dermatitidis
y
y Histoplamsa capsulatum
y
y Coccidioides immitis
y
FUNGI CAUSING CONJUNCTIVITIS
y Candida sp
y
y Malassezia sp
y
y Sporothrix schenckii
y
y Blastomyces dermatitidis
y
y Coccidioides immitis
y
Figure 1: Pneumocystis jirovecii (methenamine silver stain) y Paracoccidioides brasiliensis
y
(Courtesy: CDC/ Dr. Edwin P. Ewing, Jr) y Aspergillus niger
y
MULTIPLE CHOICE QUESTIONS

Chapter 52     Miscellaneous Fungi


1. Type of pneumonia in P. jirovecii  3. Select the false statement about P. jirovecii (AI 2008)
 (Recent Pattern Jul 2016) a. It is seen in immunocompromised individuals
a. Lobar pneumonia b. Frequently associated with CMV
b. Interstitial pneumonia c. May be associated with pneumatocele
c. Bronchopneumonia d. Diagnosed with spectrum microscopy
d. Any of the above 4. A patient of acute leukemia is admitted with febrile
2. P. carinii causes infection of primarily neutropenia. On day four of being treated with broad
 (Recent Pattern Dec 2012) spectrum antibiotics, his fever increases. X-ray chest
a. Rats shows bilateral fluffy infiltrates. Which of the following
b. Mice should be the most appropriate next step in the
c. Humans management?
d. Rabbits a. Add antiviral therapy
b. Add Antifungal therapy
c. Add cotrimoxazole
d. Continue chemotherapy

ANSWERS AND EXPLANATIONS


1. Ans  (b) Interstitial pneumonia 3. Ans  (b) Frequently associated with CMV
Ref: Harrison’s T.B of internal medicine – 19th ed – Page Ref: Ananthanarayan and Paniker’s Textbook of Microbiol-
1358 ogy – 10th ed – Page 618
• Route of infection is through respiratory; Source: drop-
• • It causes fatal infections in immunocompromised

lets individuals
• Usually asymptomic; It causes fatal pneumonia only in
• • As it causes pneumonia – pneumatocele occurs

immuno suppressed patients (HIV) • Toluidine blue staining from biopsy is the most common

• Type of pneumonia seen is Interstitial


• method but microscopy analysis also can be done
• Hence second option which says CMV is frequently

2. Ans  (a) Rats associated cannot be taken as right; as it can occur but
Ref: Ananthanarayan and Paniker’s Textbook of Microbiol- not so frequently
ogy – 10th ed – Page 618
4. Ans  (c) Add cotrimoxazole
• Pneumocystis carinii – rats

• Pneumocystis jirovecii – Humans



Ref: Ananthanarayan and Paniker’s Textbook of Microbiol-
ogy – 10th ed – Page 618
• Drug of choice: TMP-SMZ (Hence when CD4 count

comes down in HIV patients – TMP-SMZ is started as


prophylaxis)
• TMP-SMZ is cotrimaxozole

385
HIGH YIELDING FACTS TO BE REMEMBERED IN MYCOLOGY
Unit 5     Mycology

Examples for dimorphic fungi •• Blastomyces dermatitidis


(BPH – CPS) •• Paracoccidiodes brasiliensis
•• Histoplasma capsulatum
•• Coccidioides immitis
•• Penicilliosis marneffi
•• Sporothrix schenckii
Darling disease • Histoplasmosis

Tuberculate spores are seen in • Histoplasmosis


Arthrospores seen in • Coccidioides


Valley fever • Coccidiodomycosis


Captain wheel formation seen in • Paracoccidiodomycosis


Genus that comes under dermatophytes are • Trichophyton


• Microsporum

• Epidermophyton

Only species in Epidermophyton is • E.floccosum


Tinea versicolor is caused by • It is not caused by dermatophytes; it is caused by Malassezia furfur


Germ tube formation seen in • Candida albicans


• Candida dublinensis

Pseudohyphae is absent in which candida • Candida glabrata


species
Chlamydospores are seen in • Candida albicans

Reynolds Braude phenomenon is seen in • Candida albicans


CHROM agar is useful for • Speciation of Candida


Acute angled dichotomous branching septate • Aspergillus sp


hyphae seen in
Broad aseptate hyphae seen in • Mucor, Rhizopus

Capsulated fungus • Cryptococcus neoformans


Rose Gardener’s disease is caused by • Sporothrix schenckii


Potassium iodide is used in the treatment of • Sporothrix schenckii


which fungi
Sclerotic bodies are seen in • Chromoblastomycosis

Pigmented fungi are called as • Dematacious fungi


Botryomycosis is caused by • Not fungi ; it is caused by Staph aureus and other bacteria

Entomophthromycoses is caused by • Conidiobolus coronatus


• Basidiobolus ranarum

Non cultivable fungi • Rhinosporidium seeberi


Dapsone is used in treatment of which fungi • Rhinosporidium seeberi


Which kingdom does Rhinosporidium seeberi • Aquatic protista – DRIP CLADE


belongs to
Fungi that has trophozoites and cystic stage is • Pneumocystis jirovecii

TMP-SMX drug is given for which fungi • Pneumocystis jirovecii


ABPA is an example of • Type I and III hypersensitivity


Pigmented pencillium species that causes • Penicillium marneffei


infections in AIDS patients


Most common fungal infection in diabetics and • Zygomycosis

steroid intake patients


386
Which bird and Which tree is associated with • Bird – Pigeon

Cryptococcal infection • Tree - Eucalyptus



6

IMMUNOLOGY
Unit Outline
Chapter 53 Immunity
Chapter 54 Structure and Functions of
Immune System
Chapter 55 Antigens
Chapter 56 Antibodies
Chapter 57 Complement System
Chapter 58 Antigen-Antibody Reactions
Chapter 59 Immune Response
Chapter 60 Hypersensitivity
Chapter 61 Immunodeficiency Diseases
Chapter 62 Autoimmunity
Chapter 63 Transplantation and
Tumor Immunology
Chapter 64 Immunohematology
53
Immunity
WHAT IS IMMUNITY?
y When human body is encountering the microorganisms –
y
some protective mechanism are exhibited by various cells –
termed as immunity
y Simply, it is the resistance against infections or other foreign
y
substances.

TYPES OF IMMUNITY
y Mainly divided into two types
y
 Innate

 Acquired or adaptive—active and passive

y When a foreign substance encounters our human body the
y
first-line of defense is intact skin and mucous membranes
y When the skin or mucous membranes are breached then
A
y
the innate system of immunity starts working to attack the
infections (second-line of defense)
y Adaptive immunity or acquired immunity is acquired by an
y
individual after he encounters the antigen and it takes several
days for the immune process
y Based on the cells that are involved in the immune process –
y
the immunity is divided into
 Cell-mediated immunity

 Humoral immunity

Table 1:  Cells involved in types of immunity
B
Types of immunity Cells involved
Innate immunity • Complement Figure 1: Development of immune cells

• Neutrophils

• Macrophages Table 3:  Differences between antibody and cell-mediated

• NK cells immunity

Acquired immunity • B cells and antibodies (Humoral
Antibody-Mediated Cell-Mediated Immunity

immunity)
• Helper T cells and Cytotoxic T cells Immunity

(Cell mediated immunity) Exhibited by B cells Exhibited by T cells

Table 2:  Innate and acquired immunity Host defense – by opsonised Host defense – against
bacteria, neutralize toxin and M.tuberculosis, fungi and
Types of Specificity Immediate Improves Memory viruses viruses (Intracellular infections)
immunity protection after Allergic reactions like Allergic reactions like delayed
exposure anaphylactic shock and hay type reactions, contact
Innate Non Yes No No fever dermatitis
specific Autoimmunity Graft and tumor rejection
Acquired Highly No Yes Yes Regulation of antibody
specific response
Innate Immunity Adaptive Immunity
Unit 6     Immunology

y Mainly influenced by the intactness of skin and mucosal y Acquired or adaptive immunity, i.e. antibody-mediated or
y
y
system cell mediated immunity has four characteristic features:
y Tear secretion: Contains lysozyme which is present in almost  Antigenic specificity
y

all tissue fluids except CSF, sweat and urine  Diversity


y Lysozyme: It splits the polysaccharide components of the cell  Immunologic memory
y

wall of organisms  Recognition of self and nonself


y Flushing action of urine helps to eliminate bacteria y Adaptive immunity is again divided into active and passive
y
y
y Antibacterial substance that are present in the blood and immunity based on the features given below in Table 5.
y
tissues helps in destruction of pathogenic bacteria
 Beta lysin Table 4: Differences between active and passive immunity


 Basic polypeptides: Leukins and plakins

 Acidic substances: Lactic acid Active Immunity Passive Immunity

 Lactoperoxidase in milk
Produced actively by host Immunoglobulin received

 Interferon.
immune system passively

Remember Induced by infection and Acquired by mother to fetus transfer
vaccination and readymade antibody transfer
Acute Phase Reactants
• Serum amyloid A Long lasting Lasts for short time

• CRP Lag period present No lag period

• Complement proteins
Memory present No memory

• Fibrinogen

• Von willebrand factor Booster doses useful Subsequent doses – less effective

• Proteinase inhibitors – alpha 1 antitrypsin
Negative phase may occur No negative phase

• Alpha 1 acid glycoprotein

• Mannose binding protein Not useful in immunodeficient Useful

• Haptoglobin

• Ceruloplasmin
Types of Active Immunity

y Natural active immunity – acquired either because of a
High Yield
y
clinical or subclinical infection; A special type is called
C–Reactive Protein premunition immunity which is seen in syphilis. In this
• CRP is an inflammatory marker that gets raised during many condition, immunity to re-infection lasts only as long as the
original infection is active; if the disease is cured then the

inflammations and infections;
• hCRP is high sensitive CRP that is sensitive than CRP patient is susceptible to re-infection
y Artificial active immunity: Induced by vaccines

• CRP is used as a prognostic marker for sepsis; but nowadays
y
y Natural passive immunity: Transferred from mother to baby

procalcitonin is used as marker for sepsis
y
y Artificial passive immunity: Transferred by administration of
y
Table 4: Conditions in which CRP is increased antibodies in sera (immunoglobulin)

Insignificant increase Heavy exercise, common cold, y Adoptive immunity: Special type of immunity, an extract of
y
of CRP pregnancy immunologically competent lymphocytes is injected named
as transfer factor: Used in the treatment of lepromatous
Moderate increase Bronchitis, cystitis, malignancies, leprosy
of CRP pancreatitis, myocardial infarction y Herd immunity: Immunity for the community by vaccines
y
Marked increase of Acute bacterial infections, major like OPV
CRP trauma, systemic vasculitis

390
MULTIPLE CHOICE QUESTIONS

Chapter 53     Immunity


1. All are true about innate immunity, except: 7. Adoptive immunity is by:  (PGI 2017)
a. Non-specific  (Recent Pattern 2017) a. Infection b. Injection of antibodies
b. First-line of defence c. Injection of lymphocytes

c. Not affected by genetic affected d. Immunization


d. Includes complement 8. Transfer factor used for immunization is:
2. Which is specific for acquired immunity: (Recent Pattern 2018)


a. Immunological memory (Recent Pattern 2017) a. Immunoglobulin b. Vaccine
b. Affected by genetic makeup c. Toxoid d. Lymphocytes
c. No antigen exposure 9. When transfer factor is given as treatment results in: 
d. All of the above a. Natural active immunity  (Recent Pattern 2017)
3. Innate immunity is stimulated by which part of bacteria: b. Artificial active immunity
(Recent Pattern 2017) c. Artificial passive immunity

a. Carbohydrate sequence in cell wall d. Adoptive immunity
b. Flagella 10. Active immunity is:  (Recent Pattern 2017)
c. Bacterial cell membrane a. Immunization by DPT vaccine


d. Nucleus b. Transfer of antibodies from mother
4. Cells involved in humoral immunity: c. Immunoglobulin injection
(Recent Pattern 2017) d. None of the above

a. B-cells b. T-cells 11. First chemical barrier encountered by microorganism
c. Helper cells d. Dendritic cells for common exposed sites: (AIIMS 2017)
5. Active immunity is not acquired by: a. Lysozyme b. Acidic pH
(Recent Pattern 2017) c. Skin d. Lactose

a. Infection 12. Vaccination is based on the principal of:
b. Vaccination (Recent Pattern 2017)

c. Transplacental immunoglobulin transfer a. Agglutination b. Phagocytosis
d. Subclinical infection c. Immunological memory d. Clonal detection
6. True about active immunity: (Recent Pattern 2017) 13. Which of the following is/are the components of innate
a. Less effective immune system: (PGI May 2018)
b. Can be given in immunodeficient states a. NK cell b. Beta defensins
c. Immunological memory present c. T cell d. B–cell
d. No lag period e. Dendritic cell

ANSWERS WITH EXPLANATIONS


1. Ans.  (c)  Not affected by genetic affected • Lipopolysaccharides are the major outer surface

component which is present in all the Gram-negative
Ref: Ananthanarayan and Paniker T.B of microbiology – bacteria; Mannans are seen in the cell wall
10th edition – Page 81 • These are the primary stimulator of innate immunity.

• Innate or native immunity – basically the resistance to

infection based on the individual’s genetic make up 4. Ans.  (a) B-cells
(Racial/Species)
Ref: Ananthanarayan and Paniker T.B of microbiology –
2. Ans.  (a)  Immunological memory 10th edition – Page 147
• Humoral immunity – by B-cells
Ref: Ananthanarayan and Paniker T.B of microbiology –

• Cell mediated immunity – by T cells
10th edition – Page 83

• Adaptive immune system has the characterisitc feature 5. Ans.  (c)  Transplacental immunoglobulin transfer

of immunological memory when it encounters the same
Ref: Ananthanarayan and Paniker T.B of microbiology –
antigen on a second time – the response will be quicker
10th edition – Page 84
and more specific
• Active immunity is mainly acquired by foreign substance

3. Ans.  (a)  Carbohydrate sequence in cell wall like microbes or antigen, i.e. by vaccination, infection
• Passive immunity is acquired by antibodies – by maternal
Ref: Ananthanarayan and Paniker T.B of microbiology – 10th

to fetal transfer or by immunoglobulin injection. 391
edition – Page 83, Kuby Immunology – 7th edition – Page 149
11. Ans.  (c) Skin
Unit 6     Immunology

6. Ans.  (c)  Immunological memory present


Ref: Ananthanarayan and Paniker T.B of microbiology – Ref: Ananthanarayan and Paniker T.B of microbiology – 10th
10th edition – Page 84 edition – Page 83, Kuby Immunology – 7th edition – Page 146
• Adaptive immunity has two types active and passive • Most of the body surfaces especially mucosal surfaces



immunity – protected due to acidic pH – e.g. Skin, gastric juice,
• Ref Q.2 vagina, urinary tract.

7. Ans.  (c)  Injection of lymphocytes 12. Ans.  (c) Immunological memory

Ref: Ananthanarayan and Paniker T.B of microbiology – Ref: Ananthanarayan and Paniker T.B of microbiology –
10th edition – Page 86 10th edition – Page 83
• Adoptive immunity – a special type of immunity by • Vaccination is a type of adaptive immunity—active im-



which immunologically competent lymphocytes are munity
injected (Transfer factor). • The basic characteristic of adaptive immunity is immu-


nological memory
8. Ans.  (d) Lymphocytes • Vaccination gives the specific antigenic stimulus and


helps to combat the infection in a secondary response
Ref: Ananthanarayan and Paniker T.B of microbiology – by quicker and more specific response.
10th edition – Page 86
• Already explained Q.7 13. Ans.  (a) NK cell; (b) Beta defensins

9. Ans.  (d)  Adoptive immunity Ref: Ref: Ananthanarayan and Paniker T.B of microbiology
– 10th edition – Page 83, Kuby Immunology – 7th edition –
Ref: Ananthanarayan and Paniker T.B of microbiology – Page 149
10th edition – Page 86
• Already explained Q.7 Components of Innate immunity are:
• Skin and mucosa


10. Ans.  (a)  Immunization by DPT vaccine • Tears – lysozyme

• Antibacterial substrances in blood like beta lysin (beta

Ref: Ananthanarayan and Paniker T.B of microbiology – defensins)
10th edition – Page 84 • Cells: NK cells, macrophages, complement, neutrophils

• Active immunity is the acquired as a result of an

antigenic stimulus – either by infection or vaccination
• DPT vaccine elicits active immunity.

392
Structure and Functions 54
of Immune System
INTRODUCTION
y Immune system has a complex mechanism involving many
y
cells
y These cells ideally develop from the blood cell precursors in
y
yolk sac and fetal liver during embryonic life; Later from the
bone marrow
y Pleuripotent stem cells gives rise to myeloid, erythroid and
y
lymphoid series
y From the lymphoid series: Two groups of population namely
y
T cells and B cells arise:

Table 1:  Lymphoid Organs


Primary or Central Secondary or peripheral lymphoid
lymphoid organs organs
• Thymus • Spleen


• Bone marrow • Lymph nodes


• Mucosa associated lymphoid tissue

(MALT)
• Lymphoid tissues in gut, liver, lungs

and bone marrow

T CELLS Figure 1: Differentiation of T cells

Origin of T Cells
y Bone marrow (Origin) stem cells gives rise to immature T
Differentiation of T Cells
y
cells; These cells move to the thymus (maturation) y During this process inside the thymus, the T cells that react
y
y After entering into the thymus – T cells acquire antigen with the self antigens are clonally deleted by apoptosis – clonal
y
receptors and various CD receptors deletion – Self restriction – thus preventing autoimmunity.
y Initially the T cells do not express CD4/CD8 (Double negative) y Each double positive T cells acquires T cell receptor on their
y
y
→ then begins to express both CD4/CD8 (Double positive) surface – TCR – has three genes namely variable (V), diversity
→ finally few cells turns CD4 + and few cells CD8 + (Single (D), joining (J) genes – VDJ recombination occurs in different
positive) ways – which helps to recognize millions of antigens with
greater specificity.
y Among the T cells, 40% of population do not develop in
Location of Cells
y
thymus. It goes to gut-associated lymphoid tissue (GALT)
y Double negative (DN) and Double positive (DP) cells – y GALT (intraepithelial T cells) – helps protection against
y
y
located in cortex of thymus intestinal pathogens.
y Single positive cells – located in medulla
y
Types of T Cells PERIPHERAL LYMPHOID ORGANS
Unit 6     Immunology

y Helper T cells:
Remember
y
 T cells that are having CD4 surface marker and bind to

the MHC class II
 Two types of helper T cells are present:
Thymus- Spleen • Periarterial lymphoid


dependent • Malphigian corpuscle

} Effector cells


region (T cells) seen in white pulp
}
} Memory cells
}
 Effector CD4 T cells are again divided into : Th1, Th2, Th17 Lymph node • Para cortical area


cells Thymus- Spleen • Perifollicular region


independent • Mantle layer
Table 2: Characteristics of effector T-helper cells


region (B cells) Lymph node • Cortical area


Types of effector Characteristics • Germinal layer


T helper cells • Medullary cords


Th 1 cells • Activated by IFN gamma y Cytotoxic T cells:

• Produce IL 2, IFN gamma, IL 12

y
 These type of T cells have CD8 cell surface marker; They

• Involved in delayed hypersensitivity,


are MHC Class I restricted

cell-mediated immunity, macrophage  Helpful in killing and lysing the tumor cells, virus infected
activation, killing of intracellular


cells
organisms like M.tuberculosis, M.leprae  Involves in type II hypersensitivity reactions


Th 2 cells • Activated by IL 4 y Suppressor T cells:
y

• Produce IL 4, IL 5, IL 6 and IL 13  These type of T cells have CD8 cell surface marker. They


• Helps in synthesis of all antibodies are MHC Class I restricted

except IgG2  Involves in downregulation of immune system

• Kills helminthic parasites by the

production of IgE
Th 17 cells • Produces IL 17, IL 22

• Promote inflammation

• Involved in autoimmune disease and

cancer

High Yield
• Do T cells have a role in antibody production?

ƒ Not direct role; But indirectly it has a vital role
ƒ
ƒ When antigen is encountered – antigen presenting cells
ƒ
takes this antigen to T cells – secrete some cytokines –
these cytokines go and stimulate B cells – these B cells –
converted to plasma cells – produce antibodies Figure 2: Cytokines produced by T-helper cells

Table 3: Main function of helper T cell and respective B CELLS



cytokine mediator
Origin of B Cells
Main Function of Helper T Cell Cytokine Mediator y B cells originate and mature in the Bone marrow itself
y
Activates the antigen-specific helper T IL - 2 y During the process of synthesis – B cells acquire IgM and IgD
y
cell to produce a clone of these cells as surface markers in the cell
y B cells also undergo clonal deletion but certain B cells that
y
Activates cytotoxic T cells IL - 2 recognize self antigens may get escaped by a process called
receptor editing (This is not seen in T cells)
Activates B cells IL -4 and IL - 5 y Antigen is recognized via these surface markers
y
y When antigen stimulates the B cells – they get transformed
Activates macrophages IFN gamma
y
into plasma cells – secrete immunoglobulins
y Plasma cells are antibody secreting cells
y
394
Table 4:  Sites of origin and maturation of T and B cells ANTIGEN PRESENTING CELLS

Chapter 54     Structure and Functions of Immune System


Cells Origin Maturation y When our body encounters the antigen, whole antigen cannot

y
be directly involved, it has to be processed and presented
T cells Bone marrow Thymus to the T cells. This happens with the help of few cells called
B cells Bone marrow Bone marrow antigen presenting cells (APCs)

NULL CELLS Types of APC


y Also called as large granular lymphocytes y Professional APCs: Macrophages, dendritic cells, B cells

y
y
y Types of null cells: y Nonprofessional APCs – Fibroblasts, thymic epithelial cells,

y
y
 Antibody-dependent cytotoxic cells (ADCC) pancreatic beta cells, vascular endothelial cells, glial cells and

 Natural killer cells Thyroid epithelial cells
y Dendritic cells presents antigen to T cells, Whereas follicular

 Lymphokine activated killer cells (LAK)

y
dendritic cells presents antigen to B cells.

NATURAL KILLER CELLS Table 6:  Sites of dendritic cells
y Cells that do not have either CD4/CD8 receptors
Types of Dendritic Cells Sites
y
y These cells are LGL – large granular lymphocytes that do not
y
pass through thymus Langerhans cells Skin and mucosa
y Their main function is to kill the intracellular cells – virus
Interstitial dendritic cells Organs like lungs, liver, spleen
y
infected and tumor cells
y These function are not antibody or MHC mediated Interdigitating dendritic cells Thymus
y
y NK cells have surface markers for CD16 and CD56 Circulating dendritic cells Blood and Lymph
y
Follicular dendritic cells Lymph nodes
MACROPHAGES
y All of the immune cells come from lymphoid lineage except
MAJOR HISTOCOMPATIBILITY COMPLEX
y
macrophages that come through myeloid lineage
y Macrophages help in chronic inflammation (MHC)
y
y Major functions are:
HLA
y
 Phagocytosis

 Antigen processing and presentation y Human leukocyte antigens encoded by the HLA genes which
y

 Cytokines secretion are present in the body unique to each individual, is the major

y IFN gamma is the major activator for macrophages determinant for organ transplantation, i.e. tissue matching
y
y Blood macrophages is called monocytes (life span 1–3 days) y The gene is located in the maternal chromosome 6
y
y
and tissue macrophages called as histiocytes (life span 3
months to years)

Table 5:  Tissue macrophages


Lung Alveolar macrophages
Brain Microglia
Liver Kupffer cells
Bone Osteoclasts
Skin Langerhans cells
Figure 3: HLA gene
Connective tissue Histiocytes
y Between class I and class II is the Class III loci which has
Kidney Mesangial cells
y
complements

MISCELLANEOUS CELLS MHC Class I


y Eosinophils: Mediate allergic responses and parasitic y Located in the surface of all nucleated cells
y
y
infestation; The granules in the eosinophils have major basic y RBC do not have nucleus – they do not have MHC
y
protein which causes degranulation of mast cells y Consists of heavy chain bound to beta 2 microglobulin
y Neutrophils: First cells to involve in acute inflammation
y
y The heavy chain is highly pleomorphic
y
y Mast cells: Important source of histamine, involved in imme-
y
y MHC polymorphism is responsible for the recognition be-
y
y
diate hypersensitivity tween self and nonself
395
Unit 6     Immunology

Class I MHC Class II MHC


Composed of one peptide Composed of two peptides
encoded in the HLA locus and encoded in the HLA locus
beta 2 microglobulin
Antigen binding site is between Antigen binding site is between
α1 and α2 α1 and β1
CD8 T cells respond to antigen CD4 T cells respond to antigen
presented along with Class I presented along with Class II
MHC MHC

Table 8: Surface markers of various immune cells


Type of cells Surface markers
Helper T cells CD4, TCR, CD28
Figure 4: MHC class I Cytotoxic T cells CD8, TCR
B cells IgM, IgD, B7
MHC CLASS II Macrophages, APC Class II MHC
y Located in certain cells like B cells, Langerhan cells, macro- N K cells CD16, CD56, receptors for class I MHC
y
phages and dendritic cells All cells other than Class I MHC
y Hypervariable regions provide pleomorphism mature red cells
y
Table 9: HLA-Associated disorders

Disease HLA Type
Ankylosing spondylitis B27
Reiter`s syndrome B27
Acute anterior uveitis B27
Juvenile rheumatoid arthritis Dw14, Dw4
Insulin dependent DM DQw8
Grave’s disease DR3
Sjogren’s syndrome DR3
SLE DR2

Figure 5: MHC class II CYTOKINES


y During the immune response or inflammation
Table 7: Differences between class I and class II MHC
y
 T lymphocytes secretes substances called as lymphokines


 Macrophages secrete – monokines
Class I MHC Class II MHC

 Virus infected cells secrete - interleukins

Antigen presentation to CD8 Antigen presentation to CD4 y All these are called in group as cytokines
y
positive cells positive cells
Found on surface of all Found on surface of Table 10: Cytokines and their major sources

nucleated cells professional APC
Major Source Cytokines
Encoded by genes in the HLA Encoded by genes in the HLA
Macrophages IL 1, IL 6, TNF, IL 12
locus locus
Th1 cells IL 2, IFN gamma
Expression of gene is Expression of gene is Th2 cells IL 4, IL 5, IL 10
codominant codominant Th17 cells IL 17
Multiple alleles at each gene Multiple alleles at each gene Macrophage, T cells, B cells TGF beta
locus locus Virus infected cells Interleukins

396
MULTIPLE CHOICE QUESTIONS

Chapter 54     Structure and Functions of Immune System


1. All are peripheral lymphoid organs, except: 13. All are mononuclear-macrophage, except:



a. Lymph nodes  (Recent Pattern 2017) (Recent Pattern 2017)


b. Spleen a. Histiocytes b. Microglia
c. Mucosa associated lymphoid tissue c. Kupffer cells d. B-cells
d. Thymus 14. Which is not true about macrophage?
2. Common between B and T cells: (PGI 2017) a. Activation by IFN-γ (Recent Pattern 2017)
a. Origin from same cell lineage b. Major cells in chronic inflammation
b. Site differentiation c. M2 type involved in inflammation
c. Antigenic marker d. Phagocytic cells
d. Both humoral and cellular immunity 15. All of these are antigen presenting cells [APC’S], except:
e. Further differentiation seen  (PGI 2017)
3. Thymus dependent area in spleen: a. T cells b. B cells

(Recent Pattern 2017) c. Fibroblasts d. Dendritic cells

a. Mantle layer b. Perifollicular region e. Langerhans cells
c. Malpighian corpuscle d. All of the above 16. Most potent stimulator of naïve T cells: 
4. T cell dependent region is:  (Recent Pattern) (Recent Pattern 2017)


a. Cortical follicles of lymph node a. Mature dendritic cells b. Follicular dendritic cells
b. Medullary cords c. Macrophages d. B cells
c. Mantle layer 17. Langerhans cells in skin are:  (Recent Pattern 2017)
d. Para cortical area a. Antigen presenting cells
5. All are true regarding development of T-cells, except: b. Pigment producing cells
(Recent Pattern 2017) c. Keratin synthesizing cells

a. T-cells are formed in bone marrow  d. Sensory neurons
b. Maturation of T-cells take place in thymus 18. Follicular dendritic cells, main function is:
c. T-cells are located in mantle layer of spleen  (Recent Pattern 2017)
d. In lymph nodes, T-cells are found in Para cortical area a. Catches antigen and presents it to T cells
6. Immunoglobulins are produced by:  b. Catches antigen and presents it to B cells
(Recent Pattern 2017) c. Phagocytic activity

a. Macrophages b. B-cells d. Produce immunoglobulin
c. T-cells d. NK-cells 19. Which cells cause E-rosette formation with sheep RBCs:
7. IgE is secreted by: (Recent Pattern 2017) (Recent Pattern 2017)

a. Mast cell b. Basophils a. T cells b. NK cells
c. Eosinophils d. Plasma cells c. Monocytes d. B cells
8. Large granular cells belong to: 20. EAC rosette formation is the property of one of the
(Recent Pattern 2017) following type of immune cells: 

a. Neutrophils b. Macrophages (Recent Pattern 2017)

c. Eosinophils d. Lymphocytes a. T-cells b. B-cells
9. NK cells activity is enhanced by:  c. Macrophage d. All of the above
(Recent Pattern 2017) 21. Cellular immunity is induced by: 

a. IL-1 b. TNF (Recent Pattern 2017)

c. IL-2 d. TGF-β a. NK – cells b. Dendritic – cells
10. All are true regarding NK cells:  (PGI 2017) c. TH1 – cells d. TH2 – cells
a. CD 16 positive 22. Which of the following features is not shared between ‘T
b. CD 56 positive Cells’ and ‘B Cells’:  (AIIMS 2017)
c. Secrete complement like substance a. Positive selection during development
d. Important role in viral infected cell b. Class 1 MHC expression
11. Virus infected cells are killed by:  c. Antigen specific receptors
(Recent Pattern 2017) d. All of the above

a. NK cells b. Plasma cells 23. Longest life span is of:  (Recent Pattern 2017)
c. B-cells d. None a. Macrophage b. Memory T & B cell
12. Antibody dependent cytotoxicity is seen with: c. Neutrophils d. Platelets
 (Recent Pattern 2017) 24. MHC class 2 is coded in which region: 
a. Cytotoxic T cells b. Natural killer cells (Recent Pattern 2017)

c. ADCC d. All of the above a. A b. B
c. C d. D
397
25. HLA-1 is present on  (Recent Pattern 2017) 29. T helper cells recognizes:  (Recent Pattern 2017)
Unit 6     Immunology

a. All nucleated cells a. MHC class 1 b. MHC class 2


b. Only on cells of immune system c. Processed peptides d. Surface Ig
c. Only on B –cells 30. True about MHC: (Recent Pattern 2017))
d. Only on T – cells a. Present on chromosome 4
26. Gene component of HLA class 1 includes: b. Class 2 comprises A,B,C loci
(Recent Pattern 2017) c. Class 3 has complement

a. A,B,C b. DR d. Class 1 is involved in mixed leucocyte reaction
c. DQ d. DP 31. Cytokine that have pyrogenic activity is: 
27. The role played by major histocompatibility complex (Recent Pattern 2017)
proteins [MHC-1 & MHC-2] is to:  (AIIMS 2017) a. IL-6 b. IL -8
a. Transduce the signals to T cells following antigen c. TGF beta d. IL – 3
recognition 32. HLA gene is located in: (Recent Pattern Nov 2017)
b. Mediate immunoglobulin class-switching a. Chromosome 6 b. Chromosome 22
c. Present antigen for recognition by T cell antigen c. Chromosome 11 d. Chromosome 9
receptors 33. Class II MCH molecule are expressed on: (PGI May 2018)
d. Enhance the secretion of cytokines a. Dendritic cell b. Basophils
28. Peptide binding site on class 1 MHC molecule for pre- c. Eosinophils d. Activated T-cell
senting processed antigens to CD8 T cells is formed by: e. Macrophages
(Recent Pattern 2017) 34. Which of the following immunoglobulins are commonly

a. Proximal domain of α subunits present in antigen binding site of B cell: (PGI May 2018)
b. Distal domain of α subunits a. IgA b. IgG
c. Proximal domains of α and β subunits c. IgD d. IgM
d. Distal domains of α and β subunits e. IgE

ANSWERS WITH EXPLANATIONS


1. Ans.  (d) Thymus • T cell dependent area in the lymph node is para cortical

area
Ref: Ananthanarayan and Paniker T.B of microbiology –
10th edition – Page 133 5. Ans.  (c)  T-cell are located in mantle layer of spleen
• Central lymphoid organs – Thymus and Bone marrow
Ref: Ananthanarayan and Paniker T.B of microbiology –

• Peripheral lymphoid organs – Lymph nodes, spleen,
10th edition – Page 133

MALT, bone marrow, GALT
• T cells are seen in the white pulp (malpighian corpuscle)

2. Ans.  (a, e)  Origin from same cell lineage; Further dif- of the spleen
ferentiation seen • Mantle layer – B cells are seen

Ref: Ananthanarayan and Paniker T.B of microbiology – 6. Ans.  (b) B-cells
10th edition – Page 135
Ref: Ananthanarayan and Paniker T.B of microbiology –
• Both the B and T cells are derived from the same
10th edition – Page 138

lymphoid lineage from bone marrow
• Further differentiation occurred during maturation • When antigen is encountered – it is presented by antigen


• T cells mature in thymus presenting cells – to MHC – T cells – secrete cytokines

• B cells mature in bone marrow itself – goes and stimulates B cells – gets transformed into

plasma cells – secrete immunoglobulins
3. Ans.  (c)  Malpighian corpuscle
7. Ans.  (d)  Plasma cells
Ref: Ananthanarayan and Paniker T.B of microbiology –
10th edition – Page 133 Ref: Ananthanarayan and Paniker T.B of microbiology –
10th edition – Page 138
• T cell dependent (thymus) areas in spleen are
• Plasma cells are the antibody secreting cells

periarterial lymphoid collection, malpighian corpuscle

in the white pulp • Immunoglobulins are the antibodies

4. Ans.  (d)  Para cortical area 8. Ans.  (d) Lymphocytes

Ref: Ananthanarayan and Paniker T.B of microbiology – Ref: Ananthanarayan and Paniker T.B of microbiology –
398 10th edition – Page 133 10th edition – Page 139
• Apart from T and B cells, certain cells do not have the ƒ Dendritic cells

Chapter 54     Structure and Functions of Immune System


ƒ

surface markers and neither fit into T and B cells – 5 ƒ B cells

ƒ
to 10% are found – called as null cells/Large granular ƒ Langerhan cells

ƒ
lymphocytes (LGL)
16. Ans.  (a)  Mature dendritic cells
9. Ans.  (c) IL-2
Ref: Robbins – pathology – 8th edition – page 189
Ref: Ananthanarayan and Paniker T.B of microbiology – • Before getting an antigenic exposure the new cells are


10th edition – Page 139 called as Naïve B and T cells
• IL – 2 acts as a growth factor for natural killer cells • When an antigen is encountered – the langerhan cells



go and capture the antigen – presents to the T cells with
10. Ans.  (a) CD 16 positive; (b) CD 56 positive; (c) Secrete corresponding MHC – stimulates the naïve T cells –
complement like substance; (d) Important role in viral produce cytokines
infected cell
17. Ans.  (a)  Antigen presenting cells
Ref: Ananthanarayan and Paniker T.B of microbiology –
10th edition – Page 139 Ref: Ananthanarayan and Paniker T.B of microbiology –
• Natural killer cells have CD16 and CD56 on their surface 10th edition – Page 140

• They secrete perforins – that resembles that of • Langerhans cells are dendritic cells; they belong to



complement C9 antigen presenting cells
• Interferon acts a stimulator for NK cells

• NK cells are mainly involved in immune surveillance 18. Ans.  (b)  Catches antigen and presents it to B cells

and they are the defence against virus infected cells and Ref: Review of medical microbiology and immunology –13th
cancer cells edition - page 1107
11. Ans.  (a)  NK cells • Follicular dendritic cells are located in the B cell

containing germinal centres of the follicles in the spleen
Ref: Ananthanarayan and Paniker T.B of microbiology – and lymph nodes
10th edition – Page 139 • They do not produce MHC class II

Ref: Already explained Q.10 • Hence they do not present antigen to helper T cells

• They directly capture antigen antibody complexes and

12. Ans.  (c) ADCC presents to B cells
Ref: Ananthanarayan and Paniker T.B of microbiology –
19. Ans.  (a) T cells
10th edition – Page 139
• There are three types of large granular lymphocytes Ref: Ananthanarayan and Paniker T.B of microbiology –
10th edition – Page 135

ƒ NK cells – not antibody dependent
ƒ
ƒ Cytotoxic T cells – not antibody dependent • When B cells bind with the sheep erythrocytes – they

ƒ
ƒ ADCC – antibody dependent form EAC rosettes due to the presence of C3 receptor
ƒ
(CR2) on the B cell
13. Ans.  (d) B-cells • When T cells bind to sheep erythrocytes – they form

Ref: Ananthanarayan and Paniker T.B of microbiology – SRBC/E rosette by the CD2 antigen
10th edition – Page 139,140
20. Ans.  (b) B-cells
• B cells are lymphocytes
Ref: Ananthanarayan and Paniker T.B of microbiology –

• Macrophages are histiocytes (tissue macrophages),
10th edition – Page 135

kupffer cells, osteoclasts, mesangial cells, synovial cells
Ref: Already explained Q.19
14. Ans.  (c) M2 type involved in inflammation
21. Ans.  (c)  TH1 – cells
Ref: Review of medical microbiology and immunology –
page 1105 Ref: Ananthanarayan and Paniker T.B of microbiology –
• M2 type of macrophages are having anti inflammatory 10th edition – Page 137

action • Cell mediated immunity is mediated by T helper cells –

type Th1
15. Ans.  (a) T cells; (c) Fibroblasts
22. Ans.  (a)  Positive selection during development
Ref: Ananthanarayan and Paniker T.B of microbiology –
10th edition – Page 140 Ref: Review of medical microbiology and immunology –13th
• Antigen presenting cells are edition - page 1080

ƒ Macrophages • During development – immature cells develop to mature 399
ƒ

cells
• In this process - T cells first acquire double negative
Unit 6     Immunology


30. Ans.  (c)  Class 3 has complement
cells – then double positive – then single positive cells
– which is acquired by process called positive selection Ref: Ananthanarayan and Paniker T.B of microbiology –
followed by negative selection 10th edition – Page 142
• B cells do not have these – it has only negative selection Ref: Already explained Q.29

23. Ans.  (a) Macrophage 31. Ans.  (a) IL-6
Ref: Review of medical microbiology and immunology –13th Ref: Ananthanarayan and Paniker’s Textbook of Microbiol-
edition - page 1105 ogy – 10th ed – Page 159
• Tissue macrophages has the longest life span of months
Function Cytokine mediators

upto years
Pyrogenic TNF alpha, IL-1, IL-6
24. Ans.  (d) D Anti tumour TNF alpha, TNF beta
Ref: Ananthanarayan and Paniker T.B of microbiology – Anti viral IFN alpha and IFN beta
10th edition – Page 142
MHC CLASS I A,B,C loci 32. Ans.  (a) Chromosome 6
MHC CLASS II D region – DR, DQ, DP loci Ref: Ananthanarayan and Paniker T.B of microbiology –
MHC CLASS III Complement region 10th ed – Page 143
25. Ans.  (a)  All nucleated cells • HLA – Human leucocyte antigen


• Counterpart is MHC – Major histocopatibility complex


Ref: Ananthanarayan and Paniker T.B of microbiology – • HLA complex of genes are located on the short arm of

10th edition – Page 142 chromosome 6
• HLA CLASS I - Located in the surface of all nucleated • It has three clusters of genes:


cells
• HLA CLASS II – located in the cells of immune system Class I A, B, C

Class II DR, DQ, DP
26. Ans.  (a)  A, B, C
Class III C4B, C2, BF, TNF
Ref: Ananthanarayan and Paniker T.B of microbiology –
10th edition – Page 142 33. Ans.  (a) Dendritic cell; (e) Macrophages
Ref: Already explained Q.25
Ref: Ananthanarayan and Paniker T.B of microbiology –
10th ed – Page 143
27. Ans.  (c) 
Present antigen for recognition by T cell
antigen receptors • MHC class I is located in the surface of all nucleated cells

• MHC class II is located in B cells, Langerhan cells,

Ref: Ananthanarayan and Paniker T.B of microbiology – macrophages and dendritic cells
10th edition – Page 142
• Main function of MHC molecule is to bind the peptide 34. Ans.  (c) IgD; (d) IgM

fragments of foreign proteins and present them to Ref: Ananthanarayan and Paniker T.B of microbiology –
antigen specific T cells with the helps of APC 10th edition – Page 136
28. Ans.  (b)  Distal domain of α subunits During the process of origin and synthesis – B cells acquire
two immunoglobulins on their surface – IgM and IgD
Ref: Ananthanarayan and Paniker T.B of microbiology –
10th edition – Page 142

MHC CLASS I MHC CLASS II


Antigen binding site is Antigen binding site is
between α1 and α2 between α1 and β1

29. Ans.  (b)  MHC class 2


Ref: Ananthanarayan and Paniker T.B of microbiology –
10th edition – Page 142
• T helper cells has CD4 surface marker

• CD4 binds with MHC Class I

400
55
Antigens
INTRODUCTION TYPES OF ANTIGENS
y Any foreign substance that enters into the human body, y Based on the immune response, antigens are divided into

y
y
elicits immune response by the production of antibodies, is T cell-dependent and T cell-independent antigens
called as antigen
y Two important attributes of antigenicity are: T cell Dependent Antigens (Type 1)
y
 Immunogenicity: Ability to produce an immune response
y Most of the antigens that are protein in nature are T cell
 Immunological reactivity: Specificity of the immunological

y
dependent
reaction with antibodies y When these antigens are encountered – T cells help are

y
y Types of antigens: needed for the B cells to produce antibodies
y
 Complete antigen: Induces antibody and produces a spe- y T cells that are involved in this are T helper (Th) cells
y
cific reaction y When T-cell-dependent antigen is involved in immune
y
 Incomplete antigen: Haptens response – subsequence exposure to the same antigen
y Haptens are the substances that cannot directly induce  Gives more rapid production of antibodies
y
antibody production but they can react with the antibodies  Greater amounts of antibodies
 Types of haptens:  Production of antibodies occur for longer period of time
Š Simple haptens: Univalent, nonprecipitating
Š
Š Complex haptens: Polyvalent, precipitate with specific
T cell Independent Antigens (Type 2)
Š
antibodies
y Epitope y Carbohydrate/Polysaccharide, lipids, certain nucleic
y
acids antigens are T cell independent
y
 There is a small area in the antigen which has the specific
y These antigens directly stimulate B cells without antigen
structure. That determines the antigenicity
y
processing
 The smallest unit of antigenicity is known as antigenic
y No memory response is seen
determinant or epitope
y
 Types of epitopes: Table 1: Differences between type 1 and type 2 antigen

Š Linear or sequential epitope: Presents as single chain
Type 1 Antigen Type 2 Antigen
Š
Š Conformational epitope: Presents as folding
Š
y Paratope: Endotoxin, Proteins Polysaccharides, LPS
y
 The corresponding area in the antibody in which the Activate B cells with the help Directy activates the B cells
epitope combines is called as paratope of cytokines and complements
or with other immune cells
Antigen Epitope Specific immunoglobulin Causes polyclonal B cell
Antibody Paratope production activation

y A substance is made as an antigen by certain properties it Table 2: Differences between cytosolic and endocytic

y
possess: pathways
 Size of the molecule ≈ antigenicity (Larger sized molecules
have greater antigenicity)
Property Cytosolic Endocytic
Pathway Pathway
 Chemical nature: Proteins and polysaccharides are
highly antigenic; Lipids and nucleic acids are less anti- Antigen processed Endogenous Exogenous
genic; Protein that is not antigenic is gelatin Antigen complexed with MHC I molecules MHC II molecules
 Susceptibility to tissue enzymes Antigen presented to Tc Cells Th cells
 Foreignness, i.e. body’s ability to differentiate into self and
non-self SUPERANTIGENS
 Antigenic specificity
y Certain antigens – irrespective of the specificity – they activate
y
 Heterophile specificity large number of T cells, they are called superantigens
Mechanism of Action
Unit 6     Immunology

y Normally antigens bind in the αβ heterodimer groove of the


Remember
Examples for Superantigens
y
MHC molecules connecting α and β of the TCR
• Bacterial superantigens
y Superantigens directly bind outside the antibody-binding


ƒ Staphylococcal toxins like TSST-1, exfoliative toxin and
y
groove

ƒ
enterotoxins;
y It goes and bind lateral aspect of T cell Receptor β chain.
ƒ Streptococcal pyrogenic exotoxin A and C
y
ƒ
ƒ Mycoplasma arthritidis mitogen – I

ƒ
ƒ Yersinia enterocolitica and Yersinia pseudotuberculosis

ƒ
• Viral superantigens


ƒ EBV associated superantigens

ƒ
ƒ Cytomegalovirus associated superantigens

ƒ
ƒ Rabies nucleocapsid

ƒ
ƒ HIV encoded superantigen (nef)

ƒ
• Fungal superantigen


ƒ Malassezia furfur

ƒ
Figure 1: Superantigen

MULTIPLE CHOICE QUESTIONS


1. Haptens are immunogenic when they covalently bind 7. All of the following statements about carbohydrate


to: (Recent Pattern 2018) antigen are true, except: (Recent Pattern 2017)



a. Lipid carrier b. Polysaccharide carrier a. It has lower immunogenicity






c. Protein carrier d. Any of the above carrier b. Memory response is seen






2. Which of the following statements is true about Hapten: c. Cause polyclonal B cell stimulation



a. It induces brisk immune response d. Does not require stimulation by T cells





(Recent Pattern 2018) 8. Which of the following T cell independent antigen acts


b. It needs carrier to induce immune response through: (Recent Pattern 2018)



c. It is a T-Independent antigen a. T-cell b. B-cell






d. It has no association with MHC c. Macrophages d. CD8+ T cells






3. The exact part of the antigen that reacts with the immune 9. Superantigens true is: (AIIMS 2017)



system is called: (Recent Pattern 2017) a. They bind to the cleft of the MHC



a. Clone b. Epitope b. Needs to processed before presentation






c. Idiotope d. Effector c. They are presented by APC’s to T cells






4. Monoclonal antibody binds to: (Recent Pattern 2018) d. Directly attached to lateral aspect of TCR beta chain




a. Epitope 10. Super antigen causes: (Recent Pattern 2018)


a. Polyclonal activation of T-cells


b. Paratope


b. Stimulation of B cells


c. Both epitope and paratope


c. Enhancement of phagocytosis


d. None of the above


d. Activation of complement


5. Which part of bacteria is most antigenic?


11. Which of the following is super antigen:

(Recent Pattern 2017)


(Recent Pattern 2017)

a. Protein coat

a. M. protein b. Streptodornase


b. Lipopolysaccharide




c. Exfoliative toxin d. Hemolysin


c. Nucleic acid




12. Which of the following is a superantigen:


d. Lipids


(Recent Pattern 2017)


6. Activation of naïve B lymphocytes by protein antigens

a. Cholera toxin b. Diphtheria toxin

is: (Recent Pattern 2017)




c. TSST d. Vero- cytoxin

a. T cell independent b. NK cell dependent








c. NK cell independent d. T cell dependent




402
ANSWERS WITH EXPLANATIONS

Chapter 55     Antigens


1. Ans.  (c)  Protein carrier • Polysaccharide (Carbohydrate) antigens are T cell-


independent antigens
Ref: Ananthanarayan and Paniker T.B of microbiology –
• These antigens directly stimulate B cells
10th edition – Page 89


• No memory response is seen
• Haptens are the substances that cannot induce antibody



• They become immunogenic when they combine with 8. Ans.  (b) B-cell

larger molecules
Ref: Ananthanarayan and Paniker T.B of microbiology –
• Proteins are the larger carrier molecules
10th edition – Page 92

2. Ans.  (b)  It needs carrier to induce immune response • T cell-independent antigens directly stimulate B cells


• Antigen presenting cells and T cells have no role
Ref: Ananthanarayan and Paniker T.B of microbiology –


10th edition – Page 89 9. Ans.  (d) Directly attached to lateral aspect of TCR beta
• Already explained Q.1 chain

3. Ans.  (b) Epitope Ref: Ananthanarayan and Paniker T.B. of microbiology –
10th edition – Page 93
Ref: Ananthanarayan and Paniker T.B of microbiology –
10th edition – Page 90 • Superantigens directly bind outside the antibody bind-


ing groove
• Epitope is the smallest part present in the antigen
• It goes and bind lateral aspect of T cell Receptor β

• Antibody binds in this part in the antigen

chain

4. Ans.  (a) Epitope • Ref the image above in theory part

Ref: Ananthanarayan and Paniker T.B of microbiology – 10. Ans.  (a)  Polyclonal activation of T-cells
10th edition – Page 90
Ref: Ananthanarayan and Paniker T.B. of microbiology –
• Already explained Q.3
10th edition – Page 93

5. Ans.  (a)  Protein coat • Superantigens cause a massive proliferation of T. cells –

release massive amount of cytokines
Ref: Ananthanarayan and Paniker T.B of microbiology –
10th edition – Page 90 11. Ans.  (c)  Exfoliative toxin
• Based on the chemical nature of antigen – the immuno-
Ref: Greenwood – Medical microbiology – 16th edition –

genicity differs
page 92
• Order of immunogenicity = Protein > Polysaccharides >
• Staphylococcal exfoliative toxin is a superantigen

lipids and nucleic acids

• Other examples are:

6. Ans.  (d)  T cell dependent ƒ Staphylococcal toxic shock syndrome toxin
ƒ
ƒ Staphylococcal enterotoxins
Ref: Ananthanarayan and Paniker T.B of microbiology –
ƒ
ƒ Streptococcal TSST
10th edition – Page 92
ƒ
ƒ Yersinia pseudotuberculosis
ƒ
• There are two types of antigens based on T cell ƒ Mycoplasma arthritis

ƒ
dependency and chemical nature
• Protein antigens are T cell-dependent antigens 12. Ans.  (c) TSST

• Polysaccharide antigens are T cell-independent antigens
Ref: Ananthanarayan and Paniker T.B of microbiology –

7. Ans.  (b)  Memory response is seen 10th edition – Page 93
• Already explained Q.11

Ref: Ananthanarayan and Paniker T.B of microbiology –
10th edition – Page 92

403
56
Antibodies
INTRODUCTION
y Antibodies are the glycoprotein molecules that are produced during immune response
y
y These antibodies go and specifically bind to the epitope region of antigen
y
y Antibodies are secreted by plasma cells
y
y Most of the antibodies are γ globulins, hence named as immunoglobulins
y
y Among the total serum proteins, Immunoglobulins represent 20–25%
y
STRUCTURE OF AN IMMUNOGLOBULIN

Figure 1: Immunoglobulin molecule

y Immunoglobulins are made up of heavy and light chains y Each heavy chain has a variable region and constant regions
y
y
y Heavy chains have a molecular weight of 50,000 to 70,000 divided into three/four domains – CH1, CH2, CH3, CH4
y
y Light chains have a MW of 20,000
Table 1: Functions of different regions on light and
y
y Shape of the immunoglobulin is Y shaped with two heavy (H)

heavy chain
y
and two light (L) chains
y The four chains are linked by disulfide bonds Light Variable region Antigen binding
y
y The H and L chains in an immunoglobulin molecule are chain Constant region No biological function
y
always identical because of two reasons:
 Allelic exclusion Variable region Antigen binding

 Regulations in the B cells – synthesis of either kappa or Heavy Constant region - CH2 Complement binding site

lambda occus in one immunoglobulin chain Constant region - CH3 IgG receptors attach to
y Each light chain has a constant and variable region
neutrophils and macrophages
y
CLASSIFICATION OF IMMUNOGLOBULINS

Chapter 56     Antibodies


Based on the type of heavy chain, immunoglobulins are divided in to five classes: IgG (gamma), IgA (alpha), IgM (mu), IgD (delta),
IgE (epsilon)

Table 2:  Characteristics of various immunoglobulins


Characteristics IgG IgA IgM IgD IgE
Sedimentation coefficient 7 7 19 7 8
Valency 2 2 or 4 2 or 10 2 2
Molecular weight 150000 160000 900000 180000 190000
Serum concentration (mg/ml) 12 2 1.2 0.03 0.00004
Half life (days) 23 6 5 2-8 1-5
Daily production (mg/kg) 34 24 3.3 0.4 0.0023
Intravascular distribution (%) 45 42 80 75 50
Carbohydrate (%) 3 8 12 13 12
Complement fixation – Classical ++ – +++ – –
Complement fixation – Alternative – + – – –
Placental transport + + – – –
Present in milk + + – – –
Selective secretion by seromucous glands – + – – –
Heat stability (56deg C) + + + + –
Forms – Dimer Pentamer – –

IgM y Produced in secondary immune response


y
y There are four subclasses of IgG
y First Ig class to be synthesized by the fetus
y
y IgG1> IgG2 > IgG3 > IgG4 – based on the concentration pres-
y
y First Ig class to be produced in primary immune response,
y
ent in the serum
y
hence identification of IgM indicates recent infection
y IgG1, IgG3 and IgG2 (in the order) activate classical comple-
y IgM is a pentameric immunoglobulin
y
ment pathway
y
y Blood group antibodies are belonging to IgM class
y IgG4 activates alternate complement pathway
y
y Surface of B cell carries IgM in monomeric form
y
y The only immunoglobulin that crosses the placenta and gives
y
y
passive immunity naturally to fetus
y IgG production by babies starts after 6 months
y
Table 3:  Properties of IgG
Properties of IgG IgG1 IgG2 IgG3 IgG4
% of concentration 70 20 6 4
Half life in serum 23 23 7 23
(days)
Placental passage +++ + +++ +++
Complement + + +++ –
fixation
Binding to Fc +++ + +++ –
receptors
Figure 2: Structure of IgM molecule
IgA
IgG y Immunoglobulin seen in saliva, tears, colostrum, respiratory
y
y Most available immunoglobulin in serum is around 80% and GI secretions
y
y It has the maximum half-life of 23 days
y
405
y Two forms are seen:
Unit 6     Immunology

Immunoglobulin Major Functions


y
Serum IgA • Monomer IgD Uncertain. Found on the surface of many B


• Seen in serum cells as well as in serum.


Secretory IgA • Dimer IgE Mediates immediate hypersensitivity by

• •
United by J chain causing release of mediators from mast cells
• It has a secretory component – that protects and basophils upon exposure to antigen

• IgA from denaturation by bacterial proteases (allergen). Defends against worm infections

by causing release of enzymes from
eosinophils. Does not fix complement. Main
host defense against helminth infections.

Remember
Terminologies

• Antigenic differences in the constant regions


Isotype decide isotype
• IgA, IgG, IgM, IgE and IgD


• Based on the allelic differences in


Allotype immunoglobulins
E.g. Gm type


• Based on the antigenic differences in the
Idiotype


hypervariable regions
Figure 3: Structure of IgA
• All B cells have IgM on their surface and upon


immune response it initially produces IgM
IgE • Because of VDJ genes that gets recombined
Isotypic

y Called as Reagin antibody because of enzyme called Switch recombinase
switching
y
y Heat labile immunoglobulin – isotype switching occurs
• This leads to production of other isotypes, i.e.
y
y Species specific – homocytotropic

other types of immunoglobulins
y
y IgE is responsible for anaphylaxis and atopy (Type 1 hyper-
y
sensitivity)
y IgE levels are elevated in parasitic infections, helminths, atopy
y
y Only human IgE can fix to the surface of human cells – i.e MONOCLONAL ANTIBODIES
y
it is species specific – this is the basis of Prausnitz Kustner
reaction y Following an infection, antibodies are produced – these
y
antibodies are produced against multiple epitopes of antigens
IgD that leads to separate classes of clones of antibodies
y Monoclonal antibody is a single antibody forming cells
y
y Present on the surface of B cells against a single antigenic determinant specifically
y
y No specific function y Kohler and Milstein (1975) found a method for production of
y
y
monoclonal antibodies – they were awarded Nobel prize for
Table 4: Important functions of immunoglobulins medicine in 1984

y Medium used for monoclonal antibody production is HAT
Immunoglobulin Major Functions
y
medium—hypoxanthine, aminopterin and thymidine
IgG Main antibody in the secondary response. medium
Opsonizes bacteria, making them easier y Myeloma cells are used that has deficiency of HPRT enzyme
y
to phagocytize. Fixes complement, which —Hypoxanthine phosphoribosyl transferase
enhances bacterial killing. Neutralizes y Hybridoma – Fusion of antibody forming spleen cells with
y
myeloma cells
bacterial toxins and viruses. Crosses the
placenta.
IgA Secretory IgA prevents attachment Types of Monoclonal Antibodies
of bacteria and viruses to mucous y Mouse mAb: 100% mouse-derived proteins
y
membranes. Does not fix complement. y Chimeric mAb: Recombination of mouse proteins (Variable
y
IgM Produced in the primary response to an region) and human proteins (Constant region)
antigen. Fixes complement. Does not cross y Humanized mAb: Only the antigen-binding site is mouse
y
the placenta. Antigen receptor on the derived ; remaining part human derived
surface of B cells. y Human mAb: 100% human derived
406
y
Enzymatic Digestion of Immunoglobulin Table 5:  Enzymatic digestion of Ig molecule

Chapter 56     Antibodies


Molecule Papain digestion 2 Fab fragments
y Antibody molecule when treated with proteolytic enzyme like 1 Fc fragment
y
papain cleaves the molecule into two Fab fragments and 1 Fc Pepsin digestion 1 F(ab)2 fragment
fragment Fragmented Fc
y Hinge region broken
β mercaptoethanol All disulfide bonds broken
y
y This helps in placental transfer, taking part in complement
treatment Seperated heavy and light chains
y
fixation

MULTIPLE CHOICE QUESTIONS


1. Number of variable regions on each light and heavy c. IgA has minimum concentration
chain of an antibody: (Recent Pattern 2017) d. IgM has minimum concentration

a. 1 b. 2 13. Pentavalent immunoglobulin is:
c. 3 d. 4 (Recent Pattern 2017)


2. Complement attaches to immunoglobulin at: a. IgA b. IgG
(Recent Pattern 2017) c. IgM d. IgE

a. Amino terminal b. Fab region 14. Heaviest immunoglobulin is: (Recent Pattern 2017)
c. Variable region d. Fc fragment a. IgA b. IgG
3. Variable portion of antibody molecule is: c. IgD d. IgM
 (Recent Pattern) 15. Capacity of producing IgG starts at what age:
a. C-terminal b. N-terminal (Recent Pattern 2017)

c. CHO moiety d. None a. 6 months b. 1 year
4. Antigen binding site on antibody is:  (PGI Model Q) c. 2 year d. 3 year
a. Hinge region b. Constant region 16. Which of the following immunoglobulins can cross
c. Variable region d. Hypervariable region placenta:  (Recent Pattern 2017)
e. Idiotype region a. IgA b. IgM
5. Which portion of antibody binds to antigen? c. IgG d. IgD
 (Recent Pattern 2017) 17. The serum concentration of which of the following
a. Hinge region b. Constant region human IgG subclass is maximum: (Recent Pattern 2017)
c. Variable region d. Hypervariable region a. IgG 1 b. IgG 2
6. Antigen idiotype is related to:  (Recent Pattern 2017) c. IgG 3 d. IgG 4
a. Fc fragment b. Hinge region 18. Maximum half-life  (Recent Pattern 2018)
c. C-terminal d. N-terminal a. IgG b. IgA
7. Idiotypic class of antibody is determined by: c. IgM d. IgE
 (Recent Pattern 2017) 19. Molecular mass of IgG (in k Da): 
a. Fc region b. Hinge region (Recent Pattern 2017)

c. Carboxy end d. Amino end a. 150 b. 400
8. Immunoglobulin isotype class switching is determined c. 1000 d. 1500
by:  (Recent Pattern 2018) 20. The earliest immunoglobulin to be synthesized by the
a. Constant region of light chain fetus is:  (Recent Pattern 2017)
b. Constant region of heavy chain a. IgA b. IgG
c. Variable region of light chain c. IgE d. IgM
d. Variable region of heavy chain 21. IgM appears in fetus at what gestational age:
9. Immunoglobulin variation does not depend on:  (Recent Pattern 2018)
(Recent Pattern 2017) a. 10 weeks b. 20 weeks

a. Light chain b. Heavy chain c. 30 weeks d. At birth
c. Amino acid sequence d. Constant region 22. ABO isoantibodies are of which class: 
10. Immunoglobulin changes in variable region: (Recent Pattern 2018)

a. Idiotype b. Isotype a. IgG b. IgM
(Recent Pattern 2017) c. IgD d. IgA

c. Allotype d. Epitope 23. Which of the following immunoglobulin is responsible
11. Heat labile immunoglobulin:  (Recent Pattern 2017) for opsonisation:  (Recent Pattern 2018)
a. IgA b. IgG a. IgA b. IgG
c. IgE d. IgM c. IgM d. IgE
12. True about immunoglobulin:  (Recent Question 2017) 24. Activation of classical complement pathway:
a. IgE has maximum concentration a. IgA b. IgG (Recent Pattern 2018)
b. IgG has maximum concentration c. IgM d. IgD 407
25. The Fc piece of which immunoglobulin fixes Cl: 29. IgE binds to which cell:  (Recent Pattern 2017)
Unit 6     Immunology

(Recent Pattern 2017) a. T cells b. B cells



a. IgA b. IgG c. Mast cells d. NK cells
c. IgM d. IgE 30. IgE receptor present on  (AIIMS 2017)
26. Heavy chain of IgA is: (Recent Pattern 2017) a. Mast cell b. NK cell
a. Gamma c. B cell d. Pro monocyte
b. Mu 31. PK reaction detects (Recent Pattern 2018)
c. Delta a. IgG b. IgA
d. Alpha c. IgE d. IgM
27. Major immunoglobulin secreted by intestine: 32. Antibody elevated in parasitic infection:
(Recent Pattern 2017) a. IgA b. IgE (Recent Pattern 2018)

a. IgG b. IgM c. IgG d. IgM
c. IgA d. IgD 33. Regarding IgE, Which of the following is false:
28. The secretory component of immunoglobulin molecule  (AIIMS 2018)
is: (Recent Pattern 2017) a. Causes anaphylaxis b. Immediate reaction
a. Formed by epithelial cells of lining mucosa c. Fix complement d. Cross placenta
b. Formed by plasma cells 34. Papain acts a gamma globulin to form:
c. Formed by epithelial cell and plasma cell  (Recent Pattern 2017)
d. Secreted by bone marrow a. 2 Fc fragments b. 2Fab fragments
c. 1 Fab fragments d. None

ANSWERS WITH EXPLANATIONS


1. Ans.  (a) 1 5. Ans.  (c) Variable region; (d)  Hypervariable region
Ref: Ananthanarayan and Paniker T.B of microbiology – Ref: Ananthanarayan and Paniker T.B of microbiology –
10th edition – Page 97 10th edition – Page 97
• Heavy chain has one variable region and three/four Ref: Already explained Q.4

constant regions
6. Ans.  (d) N-terminal
• Light chain has one variable region and one constant region

Ref: Ananthanarayan and Paniker T.B of microbiology –
2. Ans.  (d)  Fc fragment 10th edition – Page 97,102
Ref: Ananthanarayan and Paniker T.B of microbiology – • Each individual antigenic determinant of the variable

10th edition – Page 98 region i.e paratope is called idiotope
• Fc fragment – carboxy terminal • It is the specific region of the Fab portion of the Ig


• This terminal has the activity such as complement molecule to which antigen binds

fixation, placental transfer
• It is the catabolic state 7. Ans.  (d)  Amino end

Ref: Ananthanarayan and Paniker T.B of microbiology –
3. Ans.  (b) N-terminal
10th edition – Page 97,102
Ref: Ananthanarayan and Paniker T.B of microbiology – • Idiotype – varible region – hence it is amino terminal end

10th edition – Page 97
• Carboxy terminal (C) – constant unit 8. Ans.  (b)  Constant region of heavy chain

• Amino terminal (N) – variable unit Ref: Ananthanarayan and Paniker T.B of microbiology –

10th edition – Page 97,103
4. Ans.  ( c) Variable region; (d) Hypervariable region;
(e) Idiotype region • Upon antigenic stimulation → the heavy chain VDJ

unit can join any other constant heavy gene → leads
Ref: Ananthanarayan and Paniker T.B of microbiology – to expression of particular antibody → this process is
10th edition – Page 97, Harrison T.B of internal medicine – called as isotypic class switching
19th edition - Page 372 • Because of variations in the amino acid sequence in the
• Amino acid sequences in the variable regions has some

constant region → five classes of antibody are available

invariable sequences and some highly variable sequences – IgG, IgA, IgM, IgE, IgD
• These hypervariable regions form the antigen binding

site of the antibody molecules – complementarity deter- 9. Ans.  (a)  Light chain
mining regions (CDRs)
• Idiotype is defined as the specific region of the fab Ref: Ananthanarayan and Paniker T.B of microbiology –
408 10th edition – Page 97

portion in immunoglobulin molecule where the antigen
binds • Already explained Q.8

10. Ans.  (a) Idiotype 18. Ans.  (a) IgG

Chapter 56     Antibodies


Ref: Ananthanarayan and Paniker T.B of microbiology – Ref: Ananthanarayan and Paniker T.B of microbiology –
10th edition – Page 97,102 10th edition – Page 99
• Already explained Q.6, Q.7 • IgG has the maximum half life of 23 days


11. Ans.  (c) IgE 19. Ans.  (a) 150
Ref: Ananthanarayan and Paniker T.B of microbiology – Ref: Ananthanarayan and Paniker T.B of microbiology –
10th edition – Page 100 10th edition – Page 97
• All of the immunoglobulins are heat stable except IgE – Molecular weight of immunoglobulins in kDa

heat labile
IgG 150
12. Ans.  (b)  IgG has maximum concentration
IgA 160 (monomer)
Ref: Ananthanarayan and Paniker T.B of microbiology – 400 (dimer)
10th edition – Page 99 IgM 1000
• IgG has the maximum serum concentration of 12 mg/ml IgE 190

• IgE has the minimum serum concentration of 0.00004
IgD 175

mg/ml

13. Ans.  (c) IgM 20. Ans.  (d) IgM

Ref: Ananthanarayan and Paniker T.B of microbiology – Ref: Ananthanarayan and Paniker T.B of microbiology –
10th edition – Page 100 10th edition – Page 100
• IgM is a heavy molecule – Molecular weight is 1,00,000 – • Around 20 weeks of age – the first immunoglobulin

secreted by the fetus is IgM

hence called as millionaire molecule
• IgM has a theoretical valency of 10 – but the effective • It is the antibody that is produced in primary immune

response

valency is 5
• It is a pentavalent molecule with a J chain
21. Ans.  (b)  20 weeks

14. Ans.  (d) IgM Ref: Ananthanarayan and Paniker T.B of microbiology –
Ref: Ananthanarayan and Paniker T.B of microbiology – 10th edition – Page 100
10th edition – Page 100 Ref: Already explained Q.20
• Already explained Q.13
22. Ans.  (b) IgM

15. Ans.  (a)  6 months Ref: Kuby – immunology – 7th edition – page 812
Ref: Ananthanarayan and Paniker T.B of microbiology – • Blood group antibodies belong to IgM type

10th edition – Page 99
23. Ans.  (c) IgM
• IgM is the primary antibody secreted immediately after

birth Ref: Ananthanarayan and Paniker T.B of microbiology –
• Maternal IgG crosses placenta and gives passive 10th edition – Page 97, Harrison T.B of internal medicine –

protection 19th edition – page 372
• After 6 months – IgG synthesis attains steady level and • Immunoglobulins that are involved in opsonisation are

body has its own capacity to meet immunity

IgM and IgG
• Comparatively IgM is more effective than IgG
16. Ans.  (c) IgG

• IgM does opsonisation through classical pathway – C3b

Ref: Ananthanarayan and Paniker T.B of microbiology –
24. Ans.  (c) IgM
10th edition – Page 99
• The only maternal immunoglobulin that crosses placen- Ref: Ananthanarayan and Paniker T.B of microbiology –

ta is IgG 10th edition – Page 100
• Activator of classical complement pathway – IgM, IgG
17. Ans.  (a)  IgG 1

• Activator of alternate complement pathway – IgA

Ref: Ananthanarayan and Paniker T.B of microbiology – 25. Ans.  (c) IgM
10th edition – Page 99
Ref: Ananthanarayan and Paniker T.B of microbiology –
• The maximum concentration that is present in the 10th edition – Page 100

serum is IgG
• C1 – involved in classical complement pathway ( will be
• It has four subclasses IgG1-4 409

explained in next chapter)

• Among them IgG1 is the greatest amount ; IgG4 the least

• IgM is the antibody more effective in classical comple- 31. Ans.  (c) IgE
Unit 6     Immunology


ment pathway
Ref: Ananthanarayan and Paniker T.B of microbiology –
26. Ans.  (d) Alpha 10th edition – Page 100
• IgE is homocytotropic – i.e species specific
Ref: Ananthanarayan and Paniker T.B of microbiology –


• Only human IgE can fix the surface of human cells
10th edition – Page 98


• For detecting allergy – atopy – Prausnitz and Kustner
• Heavy chains are the basis for classes of immunoglobulins


made an experiment injecting the serum of other one

• IgG (gamma), IgA (alpha), IgM (mu), IgD (delta), IgE who had allergic preponderance

(epsilon) are the five classes
32. Ans.  (b) IgE
27. Ans.  (c) IgA
Ref: Ananthanarayan and Paniker T.B of microbiology –
Ref: Ananthanarayan and Paniker T.B of microbiology – 10th edition – Page 100
10th edition – Page 99
• IgE is elevated in parasitic infections especially helmin-
• Immunoglobulin seen in saliva, tears, colostrum, respi-


thic infections

ratory and GI secretions
33. Ans.  (d) Cross placenta; (c) Fix complement
28. Ans.  (a)  Formed by epithelial cells of lining mucosa
Ref: Ananthanarayan and Paniker T.B of microbiology –
Ref: Ananthanarayan and Paniker T.B of microbiology – 10th edition – Page 100
10th edition – Page 99
• IgG is the only immunoglobulin that crosses placenta
• IgA has a secretory component – which is secreted by


• Activator of classical complement pathway – IgM, IgG


the mucosal epithelial cells • Activator of alternate complement pathway – IgA
• IgA as such and J chain are produced from plasma cells

• Hence option C and D are wrong (question may be


wrong)
29. Ans.  (c)  Mast cells
Ref: Ananthanarayan and Paniker T.B of microbiology – 34. Ans.  (b)  2 Fab fragments
10th edition – Page 100 Ref: Ananthanarayan and Paniker T.B of microbiology –
• IgE is involved in type 1 hypersensitivity – atopy 10th edition – Page 100

• IgE goes and binds to mast cells and basophils and • Antibody molecule when treated with proteleolytic


secrete lots of histamine enzyme like papain cleaves the molecule into two Fab
fragments and 1 Fc fragment
30. Ans.  (a)  Mast cell • Hinge region broken

Ref: Ananthanarayan and Paniker T.B of microbiology –
10th edition – Page 100
• Already explained Q.29

410
57
Complement System
INTRODUCTION
y There are certain factors that occur in normal sera which are
y
activated by antigen and antibody complexes – these factors
are called complements
y The three main effects of complement are:
y
 Lysis of bacteria, allografts and tumor cells

 Generation of mediators that participate in inflammation

and attract neutrophils
 Opsonization.

SYNTHESIS OF COMPLEMENT

Table 1:  Sites of synthesis of complements


Complements Sites of synthesis
C1 Intestinal epithelium
C2, C4 Macrophages
C5, C8 Spleen

C3, C6, C9 Liver

COMPLEMENT PATHWAYS
y Most of the complements are present as inactive forms, which
y
are then activated to active forms
y The activators of complement are:
y
 Antigen: Antibody complexes

 Nonimmunological molecules

y Activation of complement pathways occurs via three path-
y
ways:
 Classic pathway

 Alternative pathway

 Lectin pathway

Figure 1: Pathways of complement system [(Error in the image –
C5 convertase in alternative pathway is C3bBbC3b – not 8
(not eight – its Bb)]
Table 2:  Characteristics of pathways Table 4:  Biological effects of complement
Unit 6     Immunology

Pathways Characteristics Effects Mediators


Classical pathway • Antibody-dependent pathway;
• Opsonization C3b
• Pathway is triggered by antigen-antibody
Chemotaxis C5a, C5,C6,C7 - complex

complex
• Involved in specific active immunity

Anaphylatoxin C5a > C3a > C4a
Alternative • Antibody-independent pathway

Cytolysis C5b6-9 (MAC ) complex


pathway • Pathway triggered by antigen directly

• Involved in nonspecific innate immunity



Enhancement of antibody C3b
production
Lectin pathway • Antibody-independent pathway

• Involved in nonspecific innate immunity


y All the three pathways lead to the production of central


y
REGULATION OF COMPLEMENT PATHWAYS
molecule named as C3b Inhibitors
y In classical pathway C3 convertase is C4b2a
y Normally serum has an inhibitor of C1 esterase (C1sINH) –
y

y In alternative pathway C3 convertase is C3bBb


y

checks the autocatalytic prolongation


y

y S protein is seen in normal serum – modulates the cytolytic


y

action of the MAC

Inactivators
 Factor I – controls C3 activation and alternative pathway
 Factor H – acts with Factor I – regulates C3 activation
 Anaphylatoxin inactivator degrades the anaphylatoxins
(C3a, C4a, C5a)
Figure 2: C3 and C5 convertase in classical and alternative  C4 binding protein controls the activity of C4b.
pathways
Deficiencies of Complement
Table 3:  Activators of pathways y C2 deficiency is the most common complement defect
y

Pathways Activator
Table 5:  Syndromes resulting from complement deficiency
Classical pathway • Antigen-antibody complex

Group Deficiency Syndrome


Alternative pathway • Endotoxin / LPS

• Teichoic acid

I Cl Inhibitor Hereditary angioneurotic edema


• Fungal cells – Zymosan -

polysaccharide II Early components SLE and other collagen vascular


• Yeast

of classical diseases
• Trypanosomes

pathway C1, C2, C4


• Viral infected cells

• Tumor cells

III C3 and its Severe recurrent pyogenic


• Antibodies in complexed – IgA, IgD

regulatory protein infections


• Cobra venom factor

C3b inactivator
• Dextran sulfate

• Nephritic factor

IV C5 to C8 Bacteremia, mainly with Gram-
• Inulin

negative diplococci, toxoplasmosis
• Agar

V C9 No particular disease
Lectin pathway • Mannose (Carbohydrate) residues on

microbes DAF – Decay PNH – Paroxysmal nocturnal


• Salmonella

accelerating factor hemoglobinuria


• Neisseria

• Listeria

• Cryptococcus

• Candida albicans

412
MULTIPLE CHOICE QUESTIONS

Chapter 57     Complement System


1. Total number of complement factors in complement 12. Anaphylotoxin in complement system: 
system:  (Recent Pattern 2017)  (Recent Pattern 2017)
a. 7 b. 9 a. C3b b. C5-9
b. 13 d. 20 c. C5a d. C1-3
2. Complement components are:  13. Chemoattractant is:  (Recent Pattern 2017)
 (Recent Pattern 2017) a. C5a b. C1
a. Lipids b. Proteins c. C3 d. C2
c. Lipoproteins d. Polysaccharide 14. In cell, lysis by complement:  (Recent Pattern 2018)
3. Complement components are:  a. They activate cyclise
 (Recent Pattern 2017) b. Inhibits elongation factor P
a. Lipoproteins b. Glycoproteins c. Destruction of cell wall
c. Polysaccharides d. Lipid d. Increased permeability of cell membrane
4. Complement formed in liver:  (Recent Pattern 2017) 15. MAC in complement system is: 
a. C2, C4 b. C3, C6, C9  (Recent Pattern 2017)
c. C5, C8 d. C1 a. C3a b. C3b
5. Activator of alternative complement pathway:  c. C5-9 d. C4b
 (Recent Pattern 2017) 16. Function of complement include all, except: 
a. Antigen-antibody complex  (Recent Pattern 2018)
b. Mannose-binding lectin a. Chemotaxis b. Opsonisation
c. Microbial surface polysaccharide c. Cell lysis d. Antigen presentation
d. All of the above 17. Hereditary angioneurotic edema is due to: 
6. Which complement component is involved in both  (Recent Pattern 2018)
classical and alternative pathways:  a. Deficiency of C1 inhibitor
 (Recent Pattern 2017) b. Deficiency of NADPH oxidase
a. C1 b. C2 c. Deficiency of MPO
c. C3 d. C4 d. Deficiency of properdin
7. Center of complement pathway:  (Recent Pattern 2017) 18. Neisseria infection is predisposed by: 
a. C3 b. C1  (Recent Pattern 2017)
c. C5 d. C2 a. Properdin b. Factor D
8. True regarding C4b2a: (Recent Pattern 2017) c. C5-C9 component d. All of the above
a. C3 convertase in classical pathway 19. Recurrent Neisseria infection are predisposed by: 
b. C3 convertase in alternative pathway  (Recent Pattern 2017)
c. C5 convertase in classical pathway a. Early complement component deficiency
d. C5 convertase in alternative pathway b. Late complement component deficiency
9. C3 convertase in alternate complement pathway:  c. C1 esterase deficiency
 (Recent Pattern 2017) d. Properdin deficiency
a. C4b2a b. C3b 20. Complement deficiency has not been implicated in
c. C3bBb d. C3a causing:  (Recent Pattern 2018)
10. C3b is converted to C3 convertase by:  a. SLE
 (Recent Pattern 2017) b. PNH
a. Factor B b. Factor P c. Hereditary angioedema
c. Factor H d. Factor I d. Membranous nephritis
11. Byproducts of complement system anaphylactic-toxins: 21. Which is the opsonin in complements:
 (Recent Pattern 2017) (Recent Pattern 2017)
a. C3b b. C4a  a. C3b b. C3a
c. C5a d. C5678 c. C5a d. C4a

413
ANSWERS AND EXPLANATIONS
Unit 6     Immunology

1. Ans.  (b) 9 8. Ans.  (a) C3 convertase in classical pathway


Ref: Ananthanarayan and Paniker T.B of microbiology – Ref: Ananthanarayan and Paniker T.B of microbiology –
10th edition – Page 123 10th edition – Page 126
• Complement pathway has nine distinct factors from C1
• • In classical pathway C3 convertase is C4b2a

to C9 • In alternative pathway C3 convertase is C3bBb


• C1 has three subunits


9. Ans.  (c) C3bBb


2. Ans.  (b) Proteins
Ref: Ananthanarayan and Paniker T.B of microbiology –
Ref: Ananthanarayan and Paniker T.B of microbiology – 10th edition – Page 126
10th edition – Page 123 • Already explRecent Patternned Q.8

• Complement system consists of effector and regulatory


proteins 10. Ans.  (a) Factor B


• At least a minimum of 30.

Ref: Ananthanarayan and Paniker T.B of microbiology –


10th edition – Page 126
3. Ans.  (b) Glycoproteins
• In alternative pathway C3b is converted to C3Bb by

Ref: Medical immunology – Owen Kuby Factor B


• Most of the complement components are glycoproteins.
• • C3Bb is the C3 convertase

4. Ans.  (b) C3, C6, C9 11. Ans.  (c) C5a


Ref: Ananthanarayan and Paniker T.B of microbiology – Ref: Ananthanarayan and Paniker T.B of microbiology –
10th edition – Page 128 10th edition – Page 125
• Complements are synthesized from various parts of the
• • C5a, C3a are the potent anaphylatoxins

body • C4a has also feature of anaphylatoxin


ƒ Intestine – C1
ƒ • But the answer need to be chosen as one choice – go for

ƒ Macrophages – C2, C4
ƒ the most potent—it is C5a
ƒ Spleen – C5, C8
ƒ • If it is PGI question – choose both C5a and C4a

ƒ Liver – C3, C6, C9


ƒ

12. Ans.  (c) C5a


5. Ans.  (c) Microbial surface polysaccharide
Ref: Ananthanarayan and Paniker T.B of microbiology –
Ref: Ananthanarayan and Paniker T.B of microbiology – 10th edition – Page 125
10th edition – Page 125 • Already expl Answer 11

• Alternative complement pathway is activated by


zymosan – which is a polysaccharide from the yeast cell 13. Ans.  (a) C5a
wall Ref: Ananthanarayan and Paniker T.B of microbiology –
• Other activators are bacterial endotoxins, IgA, IgD,

10th edition – Page 125


cobra venom fator, nephritic factor
• C5a – involved in chemotaxis.

6. Ans.  (c) C3
14. Ans.  (d) Increased permeability of cell membrane
Ref: Ananthanarayan and Paniker T.B of microbiology –
10th edition – Page 123 Ref: Ananthanarayan and Paniker T.B of microbiology –
10th edition – Page 126
• There are C3 convertases in both the pathways which act
• The final by product of complement pathways is MAC

on C3 and splits it into C3a and C3b •

• Hence C3 is the complement seen in both pathways. complex which is C5-C9 complements
• This MAC – membrane attack complex forms channels

7. Ans.  (a) C3 in the lipid cell membrane and leads to cell lysis.

Ref: Ananthanarayan and Paniker T.B of microbiology – 15. Ans.  (c) C5-9
10th edition – Page 126
Ref: Ananthanarayan and Paniker T.B of microbiology –
• All the three complement pathways meet at the center

10th edition – Page 126


molecule – C3 convertase – which act on C3 – the major
component of complement pathways. • Already explRecent Patternned answer 14.

414
16. Ans.  (d) Antigen presentation that is not formed – MAC cannot be formed – indirectly

Chapter 57     Complement System


it also predisposes to Neisseria infection
Ref: Ananthanarayan and Paniker T.B of microbiology –
• But specifically MAC complex deficiency leads to Neis-
10th edition – Page 127

seria infection.
• Biological effects of complement are:

ƒ Phagocytosis – opsonization
ƒ 19. Ans.  (b) Late complement component deficiency
ƒ Chemotaxis
Ref: Ananthanarayan and Paniker T.B of microbiology –
ƒ

ƒ MAC – cell lysis


10th edition – Page 126,128
ƒ

ƒ Inflammatory response
ƒ

ƒ Hypersensitivity reactions
ƒ
• Already explained Q.18

ƒ Autoimmune disease
20. Ans.  (b) PNH
ƒ

ƒ Endotoxic shock
ƒ

ƒ Immune adherence
ƒ

Ref: Ananthanarayan and Paniker T.B of microbiology –


ƒ Conglutination
ƒ

10th edition – Page 128

17. Ans.  (a) Deficiency of C1 inhibitor • Genetic deficiencies of complement components leads

to:
Ref: Ananthanarayan and Paniker T.B of microbiology – • Hereditary angioneurotic edema

10th edition – Page 128 • SLE


• Deficiency of C1 inhibitor leads to hereditary angioneu-



• Severe recurrent pyogenic infections

rotic odema • Gram-negative septicemia


• Characterized by episodic angioedema of the subcuta-



• Toxoplasmosis infection

neous tissues or of the mucosa of alimentary and respi-


ratory tracts 21. Ans.  (a) C3b
Ref: Ananthanarayan and Paniker T.B of microbiology –
18. Ans.  (d) All of the above 10th edition – Page 125
Ref: Ananthanarayan and Paniker T.B of microbiology –
10th edition – Page 128 Effects Mediators
• Terminal components of complements namely C5 to C9

Opsonisation C3b
forms the membrane attack complex (MAC) Chemotaxis C5a, C5, C6, C7 - complex
• Deficiency of any of these componenets – leads to in-
Anaphylatoxin C5a > C3a > C4a

ability to form the complex – that leads to predisposition


of infection with Neisseria sp Cytolysis C5b6-9 (MAC ) complex
• Neisseria meningitidis infections are more common

Enhancement of antibody C3b
• Considering other options – Properdin and Factor D are

production
needed to activate the alternative pathway – hence if

415
58 Antigen-Antibody
Reactions
INTRODUCTION Types Examples
y Antigen and antibody form complexes in the body which is Tube precipitation Kahn test for syphilis
y
the important event of immune system
y These reactions can be done in vitro and this helps in the Precipitation in gel—Immuno diffusion
y
diagnosis of immunological reactions.
• Single diffusion in one • Oudin procedure



TYPES OF ANTIGEN-ANTIBODY REACTIONS dimension
y Precipitation reactions
• Double diffusion in one • Oakley Fulthorpe procedure
y
y Agglutination reactions


dimension
y
y Complement fixation tests
y
y Neutralization tests
• Single diffusion in two • Radial immunodiffusion
y
y Radioimmunoassay


dimension • For screening antibodies in sera
y
y Enzyme linked immunoassay


for influenza virus
y
y Chemiluminescence immunoassay
y
y Immunoelectrophoresis blot
• Double diffusion in two • Ouchterlony procedure
y
y Immunochromatographic tests


dimensions • Elek’s gel test for Diphtheria
y
y Immunoflourescence


bacilli
y
Precipitation Reaction Electroimmunodiffusion

Soluble antigen + antibody (in the presence of NaCl) → antigen- • Counterimmuno- • For cryptococcal and


antibody complex → Precipitate (insoluble) electrophoresis meningococcal antigens in the
CSF
y Precipitation reaction is very sensitive in the detection of
• One dimensional single • Rocket electrophoresis
y
antigens


y Precipitation reaction is based on lattice hypothesis electroimmunodiffusion
y
y Depending upon the amount of antigen and antibody – the • Two-dimensional • Laurell’s electrophoresis
y


precipitate formation differs electrophoresis
y When the amount of precipitate is plotted on a graph vs
y
antigen and antibody concentration, three phases of curve is
obtained Agglutination Reaction
Table 1:  Marrack’s Lattice Hypothesis
Particulate antigen + antibody (in the presence of NaCl) →
Prozone Excess antibody antigen-antibody complex → visible clumps (agglutinated)
phenomenon Visible reaction is not seen
y Agglutination reaction is more sensitive for the detection of
y
Zone of Equal antigen and antibody antibodies
equivalence Lattice formation and visible reaction seen y Incomplete antibodies do not cause agglutination but they
y
Post-zone Excess antigen will combine with antigen and blocks further reaction with a
phenomenon No visible reaction complete antibody—blocking antibodies
y These blocking antibodies can give false negative results by
y
Table 2:  Types of precipitation tests blocking the antigen; thereby true antibodies cannot bind
and cause agglutination
Types Examples y Blocking antibodies are commonly seen in brucellosis
y
Slide flocculation test VDRL test for syphilis y Blocking antibodies can be detected by:
y
 Hypertonic 5% saline
Ring precipitation Ascoli’s thermoprecipitin test

 Albumin saline
Lancefield Grouping of Streptococci

 Antiglobulin (Coomb’s) test
contd…

Table 3:  Types of agglutination reaction y Source of complement is guinea pig serum

Chapter 58     Antigen-Antibody Reactions


y

y Most sensitive test, e.g. Wassermann complement fixation


Types Examples
y

test for syphilis, Treponema pallidum immobilization test


Slide agglutination test Blood grouping
Tube agglutination test • Widal test

Neutralization Test
• Standard agglutination test

y Virus neutralization test, e.g. Plaque inhibition test


for Brucellosis
y

y Toxin neutralization test:


y

Heterophile agglutination test • Weil-Felix test


•  In vitro – Nagler reaction


• Streptococcus MG
•  In vivo – Dick test, Schick test


agglutination test
• Paul Bunnell test

Coomb’s Test
Antiglobulin (Coomb’s) test
Table 4:  Differences between direct and indirect coomb’s test
Direct Coomb’s test Rh incompatibility
Indirect Coomb’s test Rh incompatibility Direct Coomb’s test Indirect Coomb’s test
Passive agglutination test: (Soluble antigens are used) • In vivo test
• • In vitro test

Hemagglutination test Rose Waaler test • Detects the antibodies bound


• • Detects free antibodies

to RBCs
Latex agglutination test •• ASO titer detection
• CRP estimation
Enzyme-Linked Immunosorbent Assay (ELISA)

•• RA factor
•• HCG y Most commonly used serological technique for diagnosis of
y

• Coagglutination test
• • Salmonella

infections
(S.aureus – Protein A is used) • Legionella y Types of ELISA:
y

 Indirect ELISA → Antigen – Antibody – Antibody to anti-


body (Conjugate)
 Sandwich ELISA → Antibody – Antigen – Antibody to an-


tigen (Conjugate)
 Competitive ELISA → Antibody – Known antigen + Un-


known antigen
 Cassette ELISA or Cylinder ELISA → done in disposable


cassettes – quick test – E.g. Dot blot assay


y Enzyme used in ELISA: Horse radish peroxidase (HRP)
y

y Substrate used in ELISA: O-phenylene diamine dihydrochloride


y

Figure 1: Rapid plasma reagin test


(Courtesy: Image from Author’s own thesis work)

Figure 3: ELISA plate with wells


(Courtesy: Image from Author’s own thesis work)
Figure 2: Tube agglutination test – Widal test
(Courtesy: Dr. Vanathi S, KAP Vishwanathan Govt Medical College, Immunochromatographic Test
Trichy)
y Rapid test method
y

Complement Fixation Test y Done in a small cassette which is already impregnated with
y

antibody/antigen with colloidal gold dye conjugate


y Antigen and antibody complexes fix complement and this
y When antigen–antibody reaction occurs, it is visible as
y

principle is used for diagnosis


y

colored product

417
y Used in diagnosis of HIV, HBV, HCV and many other infections
Unit 6     Immunology

Figures 4: Immunochromatographic test


(Courtesy: Dr.S.Vanathi, Asst Prof of Microbiology, Govt KAP Vishwanathan Medical College, Trichy, Tamil Nadu)

FLOW CYTOMETRY
y Based on laser emission – forward and side scattering from the cells are measured
y

y Cell count, size, shape and certain markers can be estimated


y

y Helps for CD4 count


y

418
MULTIPLE CHOICE QUESTIONS

Chapter 58     Antigen-Antibody Reactions


1. Rose waaler test: (Recent Pattern 2018) 11. Prozone phenomenon is responsible for 
a. Complement fixation test  (Recent Pattern 2017)
b. Precipitation in gel a. False negative test
c. Ring precipitation b. False positive test
d. Passive haemagglutination test c. May cause any of the above
2. Precipitation in comparison to agglutination requires: d. Has no relation with accuracy of test
 (Recent Pattern 2017) 12. Prozone phenomenon is a feature is: 
a. Less PH  (Recent Pattern 2017)
b. High temperature a. Tularaemia
c. Specific enzyme b. Legionnaire’s disease
d. Soluble antigen c. Plague
3. VDRL is an example of:  (Recent Pattern 2017) d. Secondary syphilis
a. Slide agglutination b. Tube agglutination 13. Antigen-antibody precipitation is maximally seen in
c. Slide flocculation d. Tube flocculation which of the following:  (AIIMS 2017)
4. Which of the following is agglutination test:  a. Excess of antibody
 (Recent Pattern 2017) b. Excess of antigen
a. Widal test b. VDRL c. Equivalence of antibody and antigen
c. Ascoli’s test d. Kahn test d. Antigen Hapten interaction
5. Paul Bunnell reaction is a type of: (Recent Pattern 2017) 14. Lattice phenomenon is seen in:  (Recent Pattern 2017)
a. Agglutination b. CF a. Neutralization reaction
c. Precipitation d. Flocculation test b. Complement fixation test
6. Weil-Felix reaction is:  (Recent Pattern 2018) c. Precipitation test
a. Ring test d. All of the above
b. Tube precipitation test 15. Maximum lattice formation occurs in: 
c. Slide agglutination test  (Recent Pattern 2017)
d. Tube agglutination test a. Zone of antibody excess
7. Nagler reaction is a type of:  (Recent Pattern 2018) b. Zone of antigen excess
a. Neutralization reaction c. Zone of equivalence
b. CFT d. Can occur in any zone
c. Precipitation 16. Indirect coomb’s test detect:  (Recent Pattern 2017)
d. Agglutination a. Incomplete antibody bound to RBC
8. Coomb’s test is:  (Recent Pattern 2018) b. Free antibody
a. Precipitation test c. Free agglutinable RBCs
b. Agglutination test d. None Of the above
c. CFT 17. Heterophile agglutination test is: (Recent Pattern 2017)
d. Neutralization test a. Widal test
9. Post-zone phenomenon is seen in:  b. Streptococcus MG agglutination
 (Recent Pattern 2017) c. VDRL
a. Antigen excess d. Kahn test of syphilis
b. Antibody excess 18. Heterophile antibody is found in: (Recent Pattern 2018)
c. Equivalence zone a. Weil Felix test b. Widal test
d. None of the above c. VDRL d. All
10. Prozone phenomenon is seen with:  19. Oakley- fulthorpe procedure is:  (Recent Pattern 2017)
 (Recent Pattern 2018) a. Agglutination test
a. Same concentration of antibody and antigen b. Precipitation test
b. In antigen excess to antibody c. Single diffusion in one dimension
c. Antibody excess to antigen d. Double diffusion in one dimension
d. Hyperimmune reaction

419
ANSWERS AND EXPLANATIONS
Unit 6     Immunology

1. Ans.  (d) Passive haemagglutination test 8. Ans.  (b) Agglutination test


Ref: Ananthanarayan and Paniker T.B of microbiology – Ref: Ananthanarayan and Paniker T.B of microbiology –
10th edition – Page 112 10th edition – Page 111
• Rose-Waaler test – passive agglutination test for RA
• • Coomb’s test is an example for agglutination test

• RA factor – is an autoantibody
• • Also called as antiglobulin test

• Helpful in diagnosis of Rh incompatibility.


2. Ans.  (d) Soluble antigen


9. Ans.  (a) Antigen excess
Ref: Ananthanarayan and Paniker T.B of microbiology –
10th edition – Page 107 Ref: Ananthanarayan and Paniker T.B of microbiology –
• Soluble antigen is used for precipitation reactions

10th edition – Page 107
• Particulate antigen is used for agglutination reactions.
• • Lattice hypothesis states three zones

• Pro zone phenomenon – excess antibody


3. Ans.  (c) Slide flocculation • Zone of equivalence – equal amounts of antigen and

Ref: Ananthanarayan and Paniker T.B of microbiology – antibody


10th edition – Page 107 • Post zone phenomenon – excess antigen.

• In precipitation reactions – sedimentation occurs



10. Ans.  (c) Antibody excess to antigen
• Instead of that – if the precipitate remains suspended as

floccules – the reaction is known as flocculation Ref: Ananthanarayan and Paniker T.B of microbiology –
E.g. for slide flocculation test is diagnosis for syphilis – 10th edition – Page 107
VDRL • Already explained Q.9

4. Ans.  (a) Widal test 11. Ans.  (a) False negative test
Ref: Ananthanarayan and Paniker T.B of microbiology – Ref: Ananthanarayan and Paniker T.B of microbiology –
10th edition – Page 111 10th edition – Page 107
• Example for tube agglutination test is Widal test
• • Prozone phenomenon has excess antibody which

• All other tests are precipitation reactions.


• gives false negative reactions as the antibodies forms
complexes and do not react – no agglutination or
5. Ans.  (a) Agglutination precipitation occurs visibly
Ref: Ananthanarayan and Paniker T.B of microbiology – • To avoid this – serial dilutions are done and checked.

10th edition – Page 111


12. Ans.  (d) Secondary syphilis
• Paul Bunnel test is helpful in diagnosis of Infectious

mononucleosis Ref: Ananthanarayan and Paniker T.B of microbiology –


• It is an example of heterophile agglutination test using

10th edition – Page 381
RBCs as antigens • Antibodies starts to raise during secondary stage of

syphilis
6. Ans.  (d) Tube agglutination test • This leads to high titre sera – false negative

Ref: Ananthanarayan and Paniker T.B of microbiology – • Serial dilutions are done to avoid this.

10th edition – Page 111


13. Ans.  (c) Equivalence of antibody and antigen
• Examples of tube agglutination test:

ƒ Widal test
ƒ
Ref: Ananthanarayan and Paniker T.B of microbiology –
ƒ Standard agglutination test for Brucellosis
ƒ
10th edition – Page 107
ƒ Weil-Felix test
ƒ • Already explained Q.9

ƒ Streptococcus MG agglutination test


ƒ

ƒ Paul Bunnell test


ƒ
14. Ans.  (c) Precipitation test
Ref: Ananthanarayan and Paniker T.B of microbiology –
7. Ans.  (a) Neutralization reaction
10th edition – Page 107
Ref: Ananthanarayan and Paniker T.B of microbiology – • Marrack’s lattice hypothesis is the basic principle for

10th edition – Page 114 precipitation and agglutination


• Nagler’s reaction is for identification of alpha toxin of

Clostridium perfringens
• It is an example for in vitro neutralization test
420

Chapter 58     Antigen-Antibody Reactions
15. Ans.  (c) Zone of equivalence 19. Ans.  (d) Double diffusion in one dimension
Ref: Ananthanarayan and Paniker T.B of microbiology – Ref: Ananthanarayan and Paniker T.B of microbiology –
10th edition – Page 107 10th edition – Page 109
• Already explained Answer 9

Immunodiffusion types:
16. Ans.  (b) Free antibody Single diffusion in one Oudin procedure
dimension
Ref: Ananthanarayan and Paniker T.B of microbiology –
10th edition – Page 111
Double diffusion in one Oakley Fulthorpe procedure
• Direct Coomb’s test – detects antibodies bound to RBC

dimension
• Indirect Coomb’s test – detects free antibodies.

Single diffusion in two Radial immunodiffusion


17. Ans.  (b) Streptococcus MG agglutination dimensions For screening antibodies in
Ref: Ananthanarayan and Paniker T.B of microbiology – sera for influenza virus
10th edition – Page 111
Double diffusion in two Ouchterlony procedure
• Examples for heterophile agglutination test:
dimensions Elek’s gel test for Diphtheria

ƒ Weil-Felix test
bacilli
ƒ

ƒ Streptococcus MG agglutination test


ƒ

ƒ Paul Bunnell test


ƒ

• Option b: Precipitation is also correct; Because immu-


nodiffusion is nothing but precipitation in gel


18. Ans.  (a) Weil-Felix test
• But since it is a NEET pattern type – choose the specific

Ref: Ananthanarayan and Paniker T.B of microbiology – answer


10th edition – Page 111
• Already explained Q.17

421
59
Immune Response
IMMUNE RESPONSE y Rejection of grafts: Graft vs host reaction

y
y Immunological surveillance: Immunity against cancer
y When an antigen is encountered by the human body → series

y
y Delayed type hypersensitivity (Type 4)
y
of events occur in the body based on the nature of antigen →

y
called as immune response
Antibody-Mediated Immunity (Humoral
Immunity)
y Antibody-Mediated immunity needs three cells

y
 Antigen presenting cells


 T helper cells


 B cells


y Like in cell mediated immunity—Microbes are processed into
y
peptide antigen (T cell-dependent antigens) along with MHC
Class II it is presented to the receptor in T helper cells
y Activation of these T helper cells—leads to synthesis and
y
secretion of interleukins—IL2, IFN gamma
y These cytokines go and act on B cells that will produce plasma
y
cells in turn produce specific antibodies (Immunoglobulins)
to that antigen

Functions
FIGURE 1: Adaptive immunity y Defence against extracellular bacterial pathogens
y
y Defence against viruses that infect through respiratory or
y
intestinal tracts
TYPES OF IMMUNE RESPONSE y Prevents recurrence of virus infections
y
There are two types of immune response y Participates in the pathogenesis of type 1, 2, 3 hypersensitivity
y
y Cell-mediated immunity (CMI)
Production of Antibodies
y
y Antibody mediated immunity
y
y Antibody production occurs in characteristic phases
y
Cell-Mediated Immunity
y When a bacterium, e.g. Mycobaterium tuberculosis → enters Table 1:  Phases of antibodies production
y
the body → it splits into fragments → peptides → called as
antigens → occurs in macrophages along with MHC class II
Phases Characteristics
y This antigen along with MHC Class II is presented to the Lag phase Antigen stimulates – no antibody in
y
receptor in T helper cells circulation
y Activation of these T helper cells leads to clonal proliferation
y
y In turn leads to synthesis and secretion of interleukins—IL2, Log phase Titre of antibody rises steadily
y
IFN gamma
y The same process happens through MHC Class I when viral Plateau phase Equilibrium between antibody
y
cells or tumor cells are processed synthesis and catabolism

Function Phase of decline Catabolism exceeds production and


titre falls
y Protects against fungi, viruses, facultative intra cellular
y
bacterial pathogens
Table 2:  Features of primary and secondary immune

Chapter 59     Immune Response


response

Features Primary response Secondary


response
Lag period 4–10 days 1–3 days
Type of B cells Naïve B cell Memory B cell
involved
Peak response 7–10 days 3–5 days
Peak antibody Antigen dependent Usually more
response intense than
primary response
Type of antibody IgM IgG
seen
Antibody affinity Lower Higher
Type of antigens Thymus Only thymus
independent as dependent
well as thymus
dependent
Immunization Needed preferably Adjuvants not
protein antigens required
with adjuvants

THEORIES OF IMMUNE RESPONSE


Figure 2: Immune response yySide chain theory: Postulated by Ehrlich
yyDirect template theories
y Indirect template theory
Other Types
y

yyNatural selection theory: Postulated by Jerne


Based on certain other characteristics, immune response can be yyClonal selection theory: Postulated by Burnet
classified as:  Clones of cells which react with self-antigens are


y Primary response
y
destructed during embryonic life – such clones are called
y Secondary response
y
as forbidden clones
 If they persist in adult life they lead to autoimmune disease


y Split gene theory: Postulated by Susumu Tonegawa.


y

423
MULTIPLE CHOICE QUESTIONS
Unit 6     Immunology

1. Antibody mediated immunity helps in: 9. IL-1 is produced by: (Recent Pattern 2017)
a. Graft versus host reaction (Recent Pattern 2017) a. Macrophages b. T cells
b. Immunological surveillance c. B cells d. Lymphocytes
c. DTH 10. Transfer factor mediates: (Recent Pattern 2017)
d. Type 2 hypersensitivity a. Cell-mediated immunity b. Adoptive immunity
2. Feature of secondary immune response: c. Humoral immunity d. Natural immunity
 (Recent Pattern 2018) 11. Growth factor for bone marrow stem cells:
a. Naïve B cell is involved  (Recent Pattern 2018)
b. IgM mediated a. IL -3 b. IFN gamma
c. Only thymus dependent antigens c. IL -1 d. TNF
d. Lower antibody immunity 12. Cytokine responsible for septic shock:
3. Polysaccharide antigens may persist in the body up to  (Recent Pattern 2018)
 (Recent Pattern 2018) a. IL -3 b. IFN gamma
a. 10 days b. 10 weeks c. IL -2 d. TNF
c. 10 months d. 10 years 13. Which theory of immune response was proposed by
4. Polysaccharide antigens are: (Recent Pattern 2017) Paul Ehrlich? (Recent Pattern 2018)
a. T cell dependent antigens a. Side chain theory b. Direct template theory
b. T cell independent antigens c. Natural selection theory d. Clonal selection theory
c. MHC I dependent antigens 14. Distinct amino acid sequences at the antigen combining
d. MHC II dependent antigens site is called: (Recent Pattern 2018)
5. CD8 cells are activated by: (Recent Pattern 2017) a. Idiotype b. Allotype
a. Antigens with MHC Class I c. Epitope d. Paratope
b. Antigens with MHC Class II 15. Who got Nobel Prize for the discovery of split genes?
c. Antigens with MHC Class III  (Recent Pattern 2017)
d. Antigens with complement a. Burnet b. Susumu Tonegawa
6. Example for an adjuvant: (Recent Pattern 2017) c. Niels K Jerne d. Paul Ehrlich
a. Corynebacterium diphtheria 16. Antibody dependent cytotoxic cell killing is mediated
b. Bordetella pertussis by: (AIIMS Pattern Nov. 2018)
c. Brucella abortans a. NK cells only
d. Clostridium tetani b. NK cells and macrophages
7. All of the following are lymphokines affecting lympho- c. NK cells and neutrophils
cytes, except: (Recent Pattern 2017) d. NK cells, macrophages and neutrophils
a. Blastogenic factor b. T cell growth factor 17. Naïve T cell can be stimulated by:
c. B cell growth factor d. Chemotactic factor (AIIMS Pattern Nov. 2018)
8. Vaccination is based on the principle of : a. NK cells and macrophages
a. Agglutination (Recent Pattern 2017) b. NK cells only
b. Phagocytosis c. B lymphocytes
c. Immunological surveillance d. Antigen presenting cells
d. Immunologic memory

ANSWERS AND EXPLANATIONS

1. Ans.  (d) Type 2 hypersensitivity 2. Ans.  (c) Only thymus dependent antigens
Ref: Ananthanarayan and Paniker’s Textbook of Microbiol- Ref: Ananthanarayan and Paniker’s Textbook of Microbiol-
ogy – 10th ed – Page 147 ogy – 10th ed – Page 148
Antibody mediated immunity: Features Primary Secondary
• Primary defence against extracellular bacterial patho-

response response
gens
• Defence against viruses that infect via respiratory and

Lag period 4–10 days 1–3 days
intestinal Type of B cells Naïve B cell Memory B cell
• Prevents recurrent viral infections
• involved
• Type 1, 2, 3 hypersensitivity

Peak response 7–10 days 3–5 days


• Certain autoimmune diseases
424

Contd…
Chapter 59     Immune Response
Features Primary Secondary 8. Ans.  (d) Immunologic memory
response response Ref: Ananthanarayan and Paniker’s Textbook of Microbiol-
Peak antibody Antigen Usually more ogy – 10th ed – Page 86
response dependent intense than • Introduction of inactivated pathogen, i.e. Antigen into
primary response

the body – elicits immune response by a process called


Type of antibody IgM IgG immunologic memory
seen • When the same antigen is encountered later  a much

Antibody affinity Lower Higher greater response will be produced without any active
Type of antigens Thymus Only thymus illness
independent as dependent
9. Ans.  (a) Macrophages
well as thymus
dependent Ref: Ananthanarayan and Paniker’s Textbook of Microbiol-
Immunization Needed Adjuvants not ogy – 10th ed – Page 156
preferably required • • IL -1: Macrophages
protein antigens • • IL -2, 3, 9, 12, 13: T cells
with adjuvants • • IL-4, 5, 6: Th cells
• • IL-7: Spleen, BM stromal cells
3. Ans.  (d) 10 years • • IL-10: T, B cells, macrophages
Ref: Ananthanarayan and Paniker’s Textbook of Microbiol- • • IL-11: BM stromal cells
ogy – 10th ed – Page 149 • • IL-17: Th 17 cells
• Protein antigens – eliminated within days to weeks

10. Ans.  (b) Adoptive immunity


• Polysaccharide antigens – persist from months to years;

E.g. Pneumococcal polysaccharide may persist up to 20 Ref: Ananthanarayan and Paniker’s Textbook of Microbiol-
years in humans following a single injection ogy – 10th ed – Page 158
• Transfer factor – Extracts of leucocytes

4. Ans.  (b) T cell independent antigens • Passive transfer of CMI is achieved by Transfer factor

Ref: Ananthanarayan and Paniker’s Textbook of Microbiol- • Helps in the treatment of T cell deficiency, lepromatous

ogy – 10th ed – Page 149 leprosy


• First step in antigen processing – Capture of Ag by APC –

11. Ans.  (a) IL -3


present to appropriate MHC molecules – to T cells
• This happens for all T cell dependent antigens Eg:
• Ref: Ananthanarayan and Paniker’s Textbook of Microbiol-
Proteins, erythrocytes ogy – 10th ed – Page 157
• For T cell independent antigens E.g. Polysaccharides –

• IL -3 – growth factor for bone marrow stem cells


they do not require T cell participation

• Called as multicolony stimulating factor – as it stimulates


the multilineage hematopoitec cells


5. Ans.  (a) Antigens with MHC Class I
Ref: Ananthanarayan and Paniker’s Textbook of Microbiol- 12. Ans.  (d) TNF
ogy – 10th ed – Page 149
Ref: Ananthanarayan and Paniker’s Textbook of Microbiol-
• CD8 cells – MHC class I with Ag

ogy – 10th ed – Page 157


• CD4 cells – MHC class II with Ag
• Tumor Necrosis Factor (TNF)

6. Ans.  (b) Bordetella pertussis • Responsible for hematological changes in septic chok

• Enhances the initial meningeal inflammation in


Ref: Ananthanarayan and Paniker’s Textbook of Microbiol-

bacterial meningitis
ogy – 10th ed – Page 153
• Adjuvant – increases the immunogenicity of an antigen

13. Ans.  (a) Side chain theory
• Bordetella pertussis has a lymphocytosis promoting

factor – which acts on the B and T cells – acts as a good Ref: Ananthanarayan and Paniker’s Textbook of Microbiol-
adjuvant in DPT vaccine ogy – 10th ed – Page 160
• Side chain theory – proposed by Paul Ehrlich

7. Ans.  (d) Chemotactic factor • First theory of immune response


Ref: Ananthanarayan and Paniker’s Textbook of Microbiol- • Other theories are – Direct template theory, Indirect

ogy – 10th ed – Page 154 template theory, Natural selection theory, Clonal selec-
tion theory
• All of the above are lymphokines affecting the lympho-

cytes except chemotactic factor which affects granulo-


cytes 425
• It is driven by immune cells mostly NK cells but also by
Unit 6     Immunology

14. Ans.  (a) Idiotype •

macrophages and neutrophils


Ref: Ananthanarayan and Paniker’s Textbook of Microbiol-
ogy – 10th ed – Page 161
• Distinct amino acid sequences at the antigen combining

site and near the parts of variable region – IDIOTYPES


• It induces anti idiotypic antibodies

15. Ans.  (b) Susumu Tonegawa


Ref: Ananthanarayan and Paniker’s Textbook of Microbiol-
ogy – 10th ed – Page 161
• One gene – one protein concept was in the past

• It was overcome by the discovery of split genes for


17. Ans.  (d) Antigen presenting cells


immunoglobulins
• Discovered by Susumu Tonegawa – awarded Nobel

Ref: Ananthanarayan and Paniker T.B of microbiology –
prize in 1987 10th ed – page 149
• A naïve T cell is the one that has completed the maturation

16. Ans.  (d) NK cells, macrophages and neutrophils process in thymus and already has undergone positive
• Antibody-dependent cellular cytotoxicity (ADCC),

and negative selection.
also called as antibody-dependent cell-mediated • It is a mature T cell but not activated.

cytotoxicity, is an immune mechanism through which • Once it is activated by antigen presenting cells (Den-

Fc receptor-bearing effector cells can recognize and dritic cells, B cells, and macrophages) with antigens – it
kill antibody-coated target cells expressing tumor or becomes effector T cells.
pathogen on their surfaces. • All naïve T cells have an expression of IL-7 receptors.

426
60
Hypersensitivity
INTRODUCTION Types of Anaphylaxis
y When our body encounters specific antigens, certain under- y Cutaneous anaphylaxis: Small dose injected intradermally
y
y
sirable reactions and consequences occur in the sensitized leads to local wheal and flare reaction, this basis is used for
host called hypersensitive reactions skin tests to diagnose hypersensitivity
y Based on the time required for the sensitized host to develop y Passive cutaneous anaphylaxis: Used to detect human IgG
y
y
reactions, hypersensitivity is divided into: antibody, which is heterocytotropic but not IgE which is
 Immediate hypersensitivity homocytotropic

 Delayed hypersensitivity y Anaphylaxis in vitro: Called as Schultz Dale phenomenon.

y
y Immediate hypersensitivity: It is B cell or antibody-mediated
y
 Anaphylaxis Mediators of Anaphylaxis

 Atopy
y Histamine

 Antibody-mediated cell damage
y
y Serotonin

 Arthus phenomenon
y
y Chemotactic factors

 Serum sickness
y
y Enzyme mediators like proteases and hydrolases

y Delayed hypersensitivity: It is T cell mediated
y
y Prostaglandins
y
 Tuberculin test
y
y Leukotriens

 Contact dermatitis
y
y Platelet activating factor

y
TYPE I HYPERSENSITIVITY Remember
y IgE dependent Anaphylactoid Reaction
y
y Two forms of type 1 hypersensitivity: • Some drugs and chemicals can simulate anaphylaxis
y

 Acute, fatal: Anaphylaxis • But this is not IgE mediated


 Chronic, non fatal: Atopy • Directly activates the complement and releases anaphylatox-


ins
Anaphylaxis Schwartzmann Reaction
y First dose that sensitizes the host is called sensitizing dose • Perturbation in coagulation factors

• Not actually an immune-mediated reaction
y
y Next dose which causes the reaction, is called as shocking

• E.g. Waterhouse Friderichsen syndrome by N. meningitidis,
y
dose

y The interval between both doses should be at least 2–3 weeks DIC caused by S. aureus in septicemia (TSS), Gram-negative
septicemia
y
to produce the reaction

Table 1: Immunologic aspects of hypersensitivity reactions



Type Antibody or Cell Mediated Immunologic Reaction
I (Immediate, Antibody (IgE) • Antigen (allergen) induces IgE antibody that binds to mast cells and basophils.

anaphylactic) • When exposed to the allergen again, the allergen cross-links the bound IgE on

those cells.
• This causes degranulation and release of mediators - histamine

II (Cytotoxic) Antibody (IgG) • Antigens on a cell surface combine with IgG antibody.

• This leads to complement-mediated lysis of the cells (e.g., Transfusion or Rh

reactions) or autoimmune hemolytic anemia.

III (Immune Antibody (IgG) • Antigen-antibody immune complexes are deposited in tissues, complement is

Complex) activated and polymorphonuclear cells are attracted to the site.
• They release Iysosomal enzymes, causing tissue damage.

Contd…
Unit 6     Immunology

Type Antibody or Cell Mediated Immunologic Reaction


IV (Delayed) Cell • T lymphocytes activated/sensitized by an antigen release lymphokines upon

second contact with the same antigen.


• The lymphokines induce inflammation and activate macrophages, which, in turn,

release various inflammatory mediators.

Table 2:  Clinical manifestations of hypersensitivity reactions


Type Typical Time of Onset Clinical Manifestation or Disease
I (Immediate, Minutes •• Systemic anaphylaxis
anaphylactic) •• Urticaria
•• Asthma
•• Hay fever
•• Allergic rhinitis
•• Allergic conjunctivitis,
•• Angioedema
II (Cytotoxic) Hours to days •• Hemolytic anemia,
•• Neutropenia,
•• Thrombocytopenia,
•• ABO transfusion reactions,
•• Rh incompatibility (erythroblastosis fetalis, hemolytic disease of the newborn)
•• Rheumatic fever
•• Goodpasture’s syndrome
•• Grave’s disease
III (Immune Complex) 2 to 3 weeks •• Systemic lupus erythematosus
•• Rheumatoid arthritis
•• Poststreptococcal glomerulonephritis
•• IgA nephropathy
•• Serum sickness
•• Hypersensitivity pneumonitis (e.g., farmer’s lung)
IV (Delayed) 2 to 3 days •• Contact dermatitis,
•• Tuberculin skin test reaction,
•• Stevens Johnson Syndrome,
•• Toxic epidermal necrolysis,
•• Erythema multiforme

High Yield
• Type V hypersensitivity (Stimulatory hypersensitivity):

ƒ It is a modified form of type II reaction;


ƒ

E.g. LATS antibody, i.e long acting thyroid stimulator antibody against thyroid cells and certain drug reactions like SJS, sulphonamide

allergies.
ƒ Here the antibody activates the receptors and causes increased activity of the cells
ƒ

• Skin Tests that are involved with hypersensitivity:


ƒ Type I →
ƒ Casoni’s test
ƒ Type III → Schick’s test
ƒ

ƒ Type IV → Tuberculin test and lepromin test


ƒ

JERISH HERHEXIMER REACTION


When Cell wall antibiotics e.g. Penicillin are used in the treatment - it causes rapid cell wall lysis leading to release of pro-inflammatory
cytokines or toxic bacterial products - a condition called Jerish Herheximer reaction; It occurs in treatment of primary and secondary
syphilis.

JONES MOTE REACTION


y It is cutaneous basophil hypersensitivity. It is delayed form of response which involves basophil leukocytes.
428
y

y It can be tested by skin tests.


y
MULTIPLE CHOICE QUESTIONS

Chapter 60     Hypersensitivity


1. B cell mediated hypersensitivity reaction is: 10. Which of the following cells plays an important role in
a. Arthus phenomenon (Recent Pattern 2017) contact dermatitis? (Recent Pattern 2017)
b. Contact dermatitis a. T cells b. B cells
c. Delayed type hypersensitivity reaction c. Langerhan cells d. Macrophages
d. Tuberculin reaction 11. Reaction due to lysis of bacterial cell wall and necrotic
2. Type I hypersensitivity is mediated through: cell product is: (Recent Pattern 2017)
a. IgM (Recent Pattern 2017) a. Arthus reaction
b. IgG b. Serum sickness
c. IgE c. Jarisch–Herxheimer reaction
d. IgA d. Infectious mononucleosis ampicillin reaction
3. Hemolytic anemia belongs to: (Recent Pattern 2018) 12. Delayed hypersensitivity involves:
a. Type I hypersensitivity b. Type II hypersensitivity  (Recent Pattern 2017)
c. Type III hypersensitivity d. Type IV hypersensitivity a. Neutrophils b. RBCs
4. Passive cutaneous anaphylaxis detects: c. Eosinophils d. Lymphocytes
a. Heterocytotropic antibody (Recent Pattern 2018) 13. Type III reaction is: (Recent Pattern 2017)
b. Wheal and flare response a. Antibody mediated
c. Atopy b. Immunocomplex mediated
d. Delayed type hypersensitivity c. Cell mediated
5. Schultz Dale phenomenon is: (Recent Pattern 2018) d. None
a. Anaphylaxis in vivo 14. Frie test belongs to which type of hypersensitivity:
b. Anaphylaxis in vitro (Recent Pattern)
c. Atopy a. Type I b. Type II
d. Cutaneous hypersensitivity c. Type III d. Type IV
6. Prausnitz –Kustner reaction is mediated by: 15. Mechanism similar to Shwartzman reaction occurs in:
a. IgE b. IgG (Recent Pattern 2017) a. Fitz Hugh Curtis syndrome (Recent Pattern 2017)
c. IgM d. IgA b. Waterhouse Friderichsen syndrome
7. Stimulatory hypersensitivity is: (Recent Pattern 2017) c. Eichwald silmser effect
a. Modified type I hypersensitivity d. Anaphylactoid reaction
b. Modified type II hypersensitivity 16. Prausnitz kustner [PK] reaction is which type of hyper-
c. Modified type III hypersensitivity sensitivity:  (Recent Pattern 2017)
d. Modified type IV hypersensitivity a. Type 1 b. Type 2
8. Erythroblastosis fetalis is an example of which type of c. Type 3 d. Type 4
hypersensitivity: (Recent Pattern 2017) 17. Myasthenia gravis is which type of hypersensitivity:
a. Type I b. Type II  (Recent Pattern 2017)
c. Type III d. Type IV a. Type 1 b. Type 2
9. Which one the following statement is false: c. Type 3 d. Type 4
a. Theobald smith phenomenon is a type I sensitivity 18. Which of the following is/are examples of type 3
reaction (Recent Pattern 2017) hypersensitivity reaction(s): (PGI May 2018)
b. Serum sickness is a type II reaction a. Serum sickness b. Arthus reaction
c. Allograft rejection is a type IV reaction c. Erythroblastosis fetalis d. Farmer’s lung
d. Transfusion reaction is a type II reaction e. Goodpasteur’s syndrome

429
Unit 6     Immunology

ANSWERS AND EXPLANATIONS


1. Ans.  (a) Arthus phenomenon a small quantity of cooked fish antigen is injected in the
same site
Ref: Ananthanarayan and Paniker’s Textbook of Microbiol-
• But this is not in use as it carries the risk of transmission
ogy – 10th ed – Page 163

of infections
B cell mediated hypersensitivity (Immediate )
• Anaphylaxis
• 7. Ans.  (b) Modified type II hypersensitivity
• Atopy
Ref: Ananthanarayan and Paniker’s Textbook of Microbiol-

• Antibody mediated cell injury


ogy – 10th ed – Page 164

• Arthus phenomenon

• Serum sickness

• Stimulatory hypersensitivity – Type V

• Modified form of Type II reaction


2. Ans.  (c) IgE • Antibodies recognize and bind to the cell surface recep-

tors instead of cell surface components


Ref: Ananthanarayan and Paniker’s Textbook of Microbiol-
ogy – 10th ed – Page 165 8. Ans.  (b) Type II
• Type I hypersensitivity – IgE mediated
Ref: Ananthanarayan and Paniker’s Textbook of Microbiol-

• Two forms : Anaphylaxis and atopy


ogy – 10th ed – Page 168

3. Ans.  (b) Type II hypersensitivity Examples of Type II hypersensitivy:


• Lysis of RBC: because of anti-erythrocytic antibodies
Ref: Ananthanarayan and Paniker’s Textbook of Microbiol-

(Autoimmunity)
ogy – 10th ed – Page 164
• Hemolytic diseases of newborn.

• Type II hypersensitivity – Cytolytic and Cytotoxic


• It causes antibody mediated damage


• 9. Ans.  (b) Serum sickness is a type II reaction
• E.g. Thrombocytopenia, Agranulocytosis, Hemolytic
Ref: Ananthanarayan and Paniker’s Textbook of Microbiol-

anemia
ogy – 10th ed – Page 168
4. Ans.  (a) Heterocytotropic antibody Examples of Type III hypersensitivity:
• Arthus reaction
Ref: Ananthanarayan and Paniker’s Textbook of Microbiol-

• Serum sickness
ogy – 10th ed – Page 166

• SLE

••Passive cutaneous anaphylaxis – in vivo method • Farmer’s lung


••For detection of antibodies


••Antigen injection intradermally 10. Ans.  (a) T cells
• Then same antigen with a dye injected i.v. → after 4 to 24
Ref: Ananthanarayan and Paniker’s Textbook of Microbiol-

hours  Wheal and Flare reaction occurs in intradermal


ogy – 10th ed – Page 169
site
• This helps to detect IgG antibody which is heterocyto-

• Type IV hypersensitivity – DTH – cell-mediated immune

tropic response
• Not for IgM which is homocytotropic (Species specific)

• Eg.: Tuberculin infection – reaction, Cutaneous basophil

hypersensitivity (Jones –Mote reaction), Contact


5. Ans.  (b) Anaphylaxis in vitro dermatitis
Ref: Ananthanarayan and Paniker’s Textbook of Microbiol- 11. Ans.  (c) Jerish herheximer reaction
ogy – 10th ed – Page 166
Ref: Ananthanarayan and Paniker’s Textbook of Microbiol-
• Uterine muscle strips from a sensitized guinea pigs →
ogy – 10th ed – Page 384

kept in a Ringer’s solution with the specific antigen →


vigorously contracts → Schultz Dale phenomenon • Cell wall antibiotics e.g. Penicillin causes rapid cell wall

• Sensitization elicited in vitro (Anaphylaxis)



lysis
• Release of proinflammatory cytokines or toxic bacterial

6. Ans.  (a) IgE products


• Occurs in treatment of primary and secondary syphilis.
Ref: Ananthanarayan and Paniker’s Textbook of Microbiol-

ogy – 10th ed – Page 167 12. Ans.  (d) Lymphocytes


• IgE – homocytotropic (Species specific)
Ref: Ananthanarayan and Paniker’s Textbook of Microbiol-

• Serum collected from Kustner who has a atopy to cooked


ogy – 10th ed – Page 169

fish → when injected intracutaneously into Prausnitz →


430 24 hours later → Wheal and flare reaction occurs when • Type IV hypersensitivity – DTH – cell-mediated immune

response - T lymphocytes
• E.g. Tuberculin infection – reaction, Cutaneous basophil 16. Ans.  (a) Type 1

Chapter 60     Hypersensitivity


hypersensitivity (Jones–Mote reaction), Contact derma-


Ref: Ananthanarayan and Paniker’s Textbook of Microbiol-
titis
ogy – 10th ed – Page 167
13. Ans.  (b) Immuno complex mediated • PK reaction is based on the principle that IgE is

homocytotropic – i.e species specific – human IgE can


Ref: Ananthanarayan and Paniker’s Textbook of Microbiol-
fix only human cells
ogy – 10th ed – Page 164
• Using cooked fish – atopy was tested in vivo

• Type III is immune complex mediated


• Mediators are IgG, IgM, leucocytes


• 17. Ans.  (b) Type 2

14. Ans.  (d) Type IV Ref: Ananthanarayan and Paniker’s Textbook of Microbiol-
ogy – 10th ed – Page 168
Ref: Ananthanarayan and Paniker’s Textbook of Microbiol-
• Myasthenia gravis belongs to antibody mediated Type II
ogy – 10th ed – Page 169

hypersensitivity
• Intracellular microbes / haptens – like chemical dyes –

applied on the skin → causes mixed cellular reaction – 18. Ans.  ( a) Serum sickness; (b) Arthus reaction ;
involves lymphocytes and macrophages (d) Farmer’s lung

15. Ans.  (b) Waterhouse Friderichsen syndrome Ref: Ananthanarayan and Paniker’s Textbook of
Microbiology – 10th ed – Page 168
Ref: Ananthanarayan and Paniker’s Textbook of Microbiol-
Examples for Type III hypensensitivity are:
ogy – 10th ed – Page 170
• Arthus reaction

• Factors affecting intravascular coagulation is affected



• Serum sickness

• Not actually an immune mediated reaction



• SLE

• Similar reaction occurs in Waterhouse-Friderichsen



• PSGN

syndrome – reason for purpuric rashes and acute • Rheumatoid arthritis


hemorrhagic adrenal necrosis • IgA nephropathy


• Serum sickness

• Hypersensitivity pneumonitis – Farmer’s lung


431
61 Immunodeficiency
Diseases
INTRODUCTION y More chance of pyogenic infections and increased reports of

y
associated autoimmune disease
y Certain conditions of the body, which has defects in immune
y B cells may be normal, but defective, hence immunoglobulin
y
pathways leading to more prone for infections are called

y
levels are low
immunodeficiency diseases
y Malabsoprtion and Giardia infection is most common
y It can be either primary or secondary

y
y
y Primary immunodeficiency diseases are inherited; Secondary
DiGeorge Syndrome
y
occurs in adult life due to an disease, drugs or nutritional
defects. y Leads to aplasia/hypoplasia of thymus due to defect in 3rd and

y
4th pharyngeal pouches
PRIMARY IMMUNODEFICIENCIES y Primary defect is thymus-dependent lymphocytes are
y
depleted
y Circulating T cells are reduced
Table 1:  Primary immunodeficiency disorders
y
y Neonatal tetany, Fallot’s tetralogy it is associated with.
y
Disorder Immunodeficiency
B cell defect • X linked agammaglobulinemia Chronic Mucocutaneous Candidiasis

• Transient hypogammaglobulinemia of y There is a defect in T cell specifically for Candida infections

y
infancy y Associated with endocrinopathies.
y
• Common variable immunodeficiency

• Selective immunoglobulin deficiencies
Ataxia Telangiectasia

• Immunodeficiencies with hyper IgM

• Transcobalamin II deficiency y Ataxia telangiectasia: Combined immunodeficiency
y

y Autosomal recessive AR
T cell defect • Thymic hypoplasis – DiGeorge syndrome
y
y Ataxia, telangiectasia, ovarian dysgenesis, chromosomal

• Chronic mucocutaneous candidiasis
y
abnormalities

• Purine nucleoside phosphorylase (PNP)
y Death is due to sinopulmonary infection

deficiency
y
y Serum and secretory IgA – absent
y
B and T cell • Nezelof syndrome y IgE deficiency is also seen

y
defects • Ataxia telangiectasia y CMI is also defective.

y
• Wiskott – Aldrich syndrome

• Immunodeficiency with thymoma
Wiskott-Aldrich Syndrome

• Immunodeficiency with short limbed

dwarfism y Wiskott-Aldrich syndrome: Combined immunodeficiency
y
• Severe combined immunodeficiency (SCID) y X linked
y

y Triad: Eczema, thrombocytopenic purpura and recurrent
y
Bruton’s Agammaglobulinemia infections
y Death due to infections, hemorrhage or lymphoreticular
y Bruton’s agammaglobulinemia: X linked (seen only in male
y
malignancy
y
infants) IgM levels low; IgG , IgA normal
y First immunodeficiency to have recognized
y
y It is a type of humoral immunodeficiency
Severe Combined Immunodeficiency (SCID)
y
y Recurrent pyogenic infections by bacteria occurs
y
y For viral infections – immune response is normal because T y Autosomal recessive (mostly) and X linked
y
y
cells are involved in viral infections (Here only B cell is defect) y Three forms of (SCID):
y
y All Ig classes levels are low.  Swiss type agammaglobulinemia
y

Common Variable Immunodeficiency  Reticular dysgenesis of de Vaal

 Adenosine deaminase (ADA) deficiency (First immu-
y Known as late onset hypogammaglobulinemia

nodeficiency to be associated with an enzyme)
y
y It manifests by the age of 15–33 years
y
y X linked have defect is IL-2 receptor Chronic Granulomatous Disease

Chapter 61     Immunodeficiency Diseases


y

y Recticular dysgenesis of de Vaal defects is at multipotent


y Due to defect in the intracellular microbicidal activity of
y

hemopoietic stem cell


y

neutrophils because of lack of NADPH oxidase


y Some patients with autosomal form have either mutation in
y X linked (most common), Autosomal
y

ZAP-70, Janus Kinase 3, RAG-1, RAG-2


y

y Defect in killing the microbes by forming peroxides during


y Immunity is profoundly depressed; hence no live vaccines in
y

phagocytosis
y

this condition should be strictly followed


y Widespread association of granulomas with this condition
y

gives this unique name


HYPER IGM SYNDROME y Lab diagnosis is by Nitroblue tetrazolium dye reduction test.
y

y An inherited deficiency in CD40 ligand (CD40L or CD154)


y

leads to impaired communication between T cells and Chédiak–Higashi Syndrome


antigen-presenting cells (APCs)
y Due to failure of lysosomes of neutrophils to fuse with pha-
y The B-cell response to T-independent antigens, however, is
y

gosomes
y

unaffected, accounting for the presence of IgM antibodies in


y Recurrent Staphylococci and Streptococcal infections
these patients, which ranges from normal to high levels and
y

give the disorder its common name, hyper IgM syndrome


(HIM) Job’s Syndrome
y Hyper IgE syndrome
y

DISORDERS OF PHAGOCYTOSIS y Recurrent cold abscess by Staphylococci, eczema, skeletal


y

defects with increased levels of IgE.


Remember
Following are the disorders of phagocytosis Lazy Leucocyte Syndrome
• Chronic granulomatous disease

y Lazy leucocyte syndrome is due to defect in chemotaxis and


• Myeloperoxidase (MPO) deficiency
y

neutrophil mobility
• Chediak-Higashi syndrome

y In bone marrow there is normal neutrophil count


• Leukocyte G6PD deficiency
y

y But peripheral neutropenia is seen


• Job’s syndrome
y

y Increased susceptibility to bacterial infection


• Tuftsin deficiency
y

y Recurrent stomatitis, gingivitis and otitis.


• Lazy leukocyte syndrome
y

• Hyper IgE syndrome


• Actin binding protein deficiency


• Shwachman’s disease

433
MULTIPLE CHOICE QUESTIONS
Unit 6     Immunology

1. Bruton’s agammaglobulinemia is:(Recent Pattern 2017) 8. The NBT (nitro blue tetrazolium) reduction assay is
a. Seen only in male infants used to:  (AIIMS 2017)
b. Autosomal dominant a. Evaluate granulocyte function
c. Recurrent viral infections b. Evaluate T-cell function
d. No T cell mediated immunity c. Determine whether polymorph nuclear leucocytes can
2. Following are the T cell defects except: produce superoxide
 (Recent Pattern 2017) d. Stain B –lymphocytes
a. DiGeorge syndrome 9. SCID which is true: (Recent Pattern 2017)
b. Chronic mucocutaneous candidiasis a. Adenosine deaminase deficiency
c. Transcobalamin deficiency b. Decreased circulating lymphocytes
d. PNP deficiency c. NADPH oxidase deficiency
3. Lazy leukocyte syndrome is: (Recent Pattern 2017) d. C1 esterase deficiency
a. T cell defect b. B cell defect 10. CD59 deficiency leads to: (Recent Pattern 2017)
c. Combined defect d. Disorder of phagocytosis a. Hereditary angioneurotic odema
4. Ataxia telangiectasia patients have a deficiency of: b. Paroxysmal nocturnal hemoglobinuria
 (Recent Pattern 2017) c. Chediak Higashi syndrome
a. IgA b. IgM d. Job syndrome
c. IgD d. IgG 11. Deficiency of both T and B Lymphocyte involved in all
5. All of the following are clinical features of Wiskott Al- except: (PGI May 2017)
drich syndrome except: (Recent Pattern 2017) a. Chronic mucocutaneous candidiasis
a. Eczema b. Wiskott-Aldrich syndrome
b. Recurrent infections c. DiGeorge Syndrome
c. Atopy d. Ataxia Telangiectasia
d. Thrombocytopenic purpura e. Common variable immunodeficiency
6. The defect in reticular dysgenesis of de Vaal is: 12. True about Severe Combined Immunodeficiency:
a. Multipotent hemopoietic stem cells (PGI may 2018)
b. Lymphoid stem cells (Recent Pattern 2017) a. Autosomal dominant transmission
c. Myeloid stem cells b. Both B & T cell defect
d. Erythroid stem cells c. Decreased lymphocyte count
7. Multiple cold staphylococcal abscesses are features of: d. More common in boys
a. Tuftsin deficiency (Recent Pattern 2017) e. Absent tonsils
b. Job syndrome
c. Chediak Higashi syndrome
d. Myeloperoxidase deficiency

ANSWERS AND EXPLANATIONS


1. Ans.  (a) Seen only in male infants T cell defects – Cellular immunodeficiencies:
• Thymic hypoplasis – DiGeorge syndrome
Ref: Ananthanarayan and Paniker’s Textbook of Microbiol-

• Chronic mucocutaneous candidiasis


ogy – 10th ed – Page 173

• Purine nucleoside phosphorylase (PNP) deficiency


• Bruton’s agammaglobulinemia – X linked (seen only in


male infants) 3. Ans.  (d) Disorder of phagocytosis


• First immunodeficiency to have recognized
Ref: Ananthanarayan and Paniker’s Textbook of Microbiol-

• Humoral immunodeficiency
ogy – 10th ed – Page 178

• Recurrent pyogenic infections by bacteria


• Viral infections – immune response is normal because T



• Lazy leucocyte syndrome – defect in chemotaxis and

cells are involved in viral infections neutrophil mobility


• All Ig classes levels are low

• BM – normal neutrophil count

• Peripheral neutropenia is seen


2. Ans.  (c) Transcobalamin deficiency • Increased susceptibility to bacterial infection


• Recurrent stomatitis, gingivitis and otitis


Ref: Ananthanarayan and Paniker’s Textbook of Microbiol-

434 ogy – 10th ed – Page 174


• The principle is: normally phagocytic cells produce

Chapter 61     Immunodeficiency Diseases


4. Ans.  (a) IgA •

superoxide free radicals with the help of NADPH oxidase


Ref: Ananthanarayan and Paniker’s Textbook of Microbiol- and kill the organisms
ogy – 10th ed – Page 176 • These superoxides reduce nitroblue tetrazolium →

• Ataxia telangiectasia: Combined immunodeficiency



formazan (insoluble) → means the test is positive
• Autosomal recessive AR

(normal individual)
• Ataxia, telangiectasia, ovarian dysgenesis, chromosomal

• If there is no superoxide formation → no reduction →

abnormalities negative test (means chronic granulamatous disease)


• Death due to sinopulmonary infection

• Serum and secretory IgA – absent



9. Ans.  (a) Adenosine deaminase deficiency
• IgE deficiency also seen

Ref: Ananthanarayan and Paniker’s Textbook of Microbiol-


• CMI is also defective

ogy – 10th ed – Page 176


5. Ans.  (c) Atopy • Enzyme deficiency that is first identified with an

immunodeficiency disorder is Adenosine deaminase – SCID


Ref: Ananthanarayan and Paniker’s Textbook of Microbiol-
ogy – 10th ed – Page 176 10. Ans.  (b) Paroxysmal nocturnal hemoglobinuria
• Wiskott-Aldrich syndrome – Combined immunodefi-

Ref: Wintrobe`s clinical hematology – Volume I – Page 1001


ciency
• CD59 is expressed in most of the hematopoeitic cells
• X linked

and endothelial cells


• Triad: Eczema, Thrombocytopenic purpura and


• It helps in protection from the intra vascular complement

Recurrent infections

mediated lysis
• Death due to infections, hemorrhage or lymphoreticular
• CD59 is called as membrane inhibitor of reactive lysis

malignancy

• Deficiency of CD59 causes PNH


• IgM levels low; IgG , IgA normal

11. Ans:  ( a) Chronic mucocutaneous candidiasis;


6. Ans.  (a) Multipotent hemopoietic stem cells
(c) DiGeorge Syndrome; (e) Common variable
Ref: Ananthanarayan and Paniker’s Textbook of Microbiol- immunodeficiency
ogy – 10th ed – Page 176
Ref: Ananthanarayan and Paniker’s Textbook of Microbiol-
• SCID – Three forms are there

ogy – 10th ed – Page 174


ƒ Swiss type agammaglobulinemia
Combined B and T cell defects are:
ƒ

ƒ Reticular dysgenesis of de Vaal


ƒ

ƒ Adenosine deaminase (ADA) deficiency


ƒ
Nezel of Syndrome
• Recticular dysgenesis of de Vaal – defect is at multipotent

• • Ataxia telangiectasia
hematopoietic stem cell • • Wiskott – Aldrich syndrome
• • Immunodeficiency with thymoma
7. Ans.  (b) Job syndrome • • Immunodeficiency with short limbed dwarfism
Ref: Ananthanarayan and Paniker’s Textbook of Microbiol- • • Severe combined immunodeficiency (SCID)
ogy – 10th ed – Page 178
12. Ans.  ( b) Both B & T cell defect; (c) Decreased
• Job syndrome – Disorder of phagocytosis

lymphocyte count; (d) More common in boys


• Multiple large cold staphylococcal abscesses

• Ig levels normal – EXCEPT IgE elevated



Ref: Ananthanarayan and Paniker’s Textbook of
Microbiology – 10th ed – Page 176
8. Ans.  (c) Determine whether polymorph nuclear leuco- • SCID is a X linked – AR disorder

cytes can produce superoxide • It has both B and T cell defects


Ref: Ananthanarayan and Paniker’s Textbook of Microbiol- • Hence lymphocytes will be less

ogy – 10th ed – Page 177


• Nitroblue tetrazolium test is used for diagnosis of

chronic granulomatous disease – which is a disorder of


phagocytosis

435
62
Autoimmunity
INTRODUCTION  Trypsin


 Nystatin
y When the immunological reaction occurs against the body’s


 Mycoplasma
y
own component that leads to structural or functional damage,


 EB virus
it is called as autoimmunity


 Malarial parasite
y Postulated as horror autotoxicus by Ehrlich


 Infectious mononucleosis
y

6. Forbidden clones: Break down of immunological homeosta-
MECHANISM OF AUTOIMMUNITY


sis – leads to cessation of tolerance
1. Antigenic alterations: Sometimes cells may undergo some 7. Altered B or T cell function


8. Defects in idiotype, antiidiotype network

alteration because of physical, chemical damage leads to

autoimmunity 9. Defective Ir or immunoglobulin genes.

2. Sequestered antigens: Antigens in closed system during
AUTOANTIBODIES

embryonic life when exposed later leads to autoimmunity;
Examples are:
 Lens protein Cold Autoantibodies

 Sperm antigen
Complete agglutinating antibodies— IgM

3. Cross-reacting foreign antigens: Similarities between foreign
y Agglutinate erythrocytes at 4°C / not at 37°

and self-antigens are the basis of many of the auto immune
y
y Seen in autoimmune hemolytic anemia, primary atypical
diseases
y
pneumonia, trypanosomiasis, black water fever

Table 1: Cross-reacting antigens


Warm Autoantibodies

Human brain Sheep brain antigen in Anti-rabies
Incomplete, non agglutinating antibodies—IgG
antigens immunization
y Seen in patients taking drugs like sulfonamide and methyldopa
y
Heart muscles Streptococcal M proteins
Renal glomeruli Nephritogenic strains of Streptococci Table 3: Autoimmune diseases

Hemocytolytic Localised Systemic
Remember autoimmune autoimmune autoimmune
4. Molecular mimicry: This is due to presence of epitopes diseases diseases diseases

with identical peptide sequences in both the infecting • Autoimmune • Hashimoto’s • SLE
micro organisms and self-antigens—leads to autoimmune



hemolytic anemia thyroiditis • Rheumatoid
reaction (note the difference between this and cross

• Autoimmune • Grave’s disease arthritis
reacting antigens)


thrombocytopenia • Addison’s disease • Polyarteritis


• Autoimmune • Autoimmune nodosa


leucopenia orchitis • Sjögren’s
Table 2: Molecular mimicry

• Myasthenia gravis syndrome


HLA B 27 Shigella flexneri • Sympathetic

ophthalmia
Joint membranes Mycobacterium tuberculosis • Pernicious anemia

Myocardium Coxsackie B • Guillain Barre

5. Polyclonal B cell activation: Some stimuli may cause non- syndrome
• Pemphigus

specific activation of B cells – leads to IgM production of

autoantibodies - Stimuli is caused by vulgaris
 2- Mercaptoethanol • Bullous

pemphigoid

 Purified protein derivative
• Dermatitis

 Lipopolysaccharide

herpetiformis

MULTIPLE CHOICE QUESTIONS

Chapter 62     Autoimmunity


1. All are example for molecular mimicry except: 5. Streptococcal M protein is an example of:
 (Recent Pattern 2017)  (Recent Pattern 2017)
a. Shigella flexneri and HLA B27 a. Cross-reacting foreign antigen
b. Mycobacterium tuberculosis and joint membranes b. Sequestered antigen
c. Coxsackie B virus and myocardium c. Molecular mimicry
d. Salmonella typhi and HLA DRQ d. Forbidden clone
2. Virus that may trigger autoimmunity is: 6. Lens protein is an example of:
a. Ebstein Barr virus (Recent Pattern 2017)  (Recent Pattern 2017)
b. Herpes simples virus a. Cross reacting foreign antigen
c. Human papilloma virus b. Sequestered antigen
d. Human immunodeficiency virus c. Molecular mimicry
3. Cold autoantibodies are: (Recent Pattern 2018) d. Forbidden clone
a. IgM 7. Rheumatoid factor is: (Recent Pattern 2017)
b. IgG a. IgM against Fc portion of IgG
c. IgA b. IgG against Fc portion of IgM
d. IgD c. IgE against Fc portion of IgM
4. Warm autoantibodies are: (Recent Pattern 2018) d. IgD against Fc portion of IgG
a. IgM 8. Long acting thyroid stimulator (LATS) is:
b. IgG  (Recent Pattern 2017)
c. IgA a. IgM b. IgG
d. IgD c. IgA d. IgD

ANSWERS AND EXPLANATIONS


1. Ans.  (d) Salmonella typhi and HLA DRQ • Seen in patients taking drugs like sulphonamides,

methyldopa
Ref: Ananthanarayan and Paniker’s Textbook of Microbiol-
ogy – 10th ed – Page 181 5. Ans.  (a) Cross-reacting foreign antigen
• Molecular mimicry – presence of similar peptide
Ref: Ananthanarayan and Paniker’s Textbook of Microbiol-

sequences in microorganisms and self-antigens


ogy – 10th ed – Page 181
• It is seen in all of the above options except S. Typhi

which has no role in molecular mimicry • Similarity between foreign (microbe) antigens and self-

antigens
2. Ans.  (a) Ebstein Barr virus • Streptococcal M protein and cardiac muscle

• Nephritogenic strain of Streptococci and renal glomeruli


Ref: Ananthanarayan and Paniker’s Textbook of Microbiol-

ogy – 10th ed – Page 181 6. Ans.  (b) Sequestered antigen


• Certain stimuli can activate multiple B cells - polyclonal
Ref: Ananthanarayan and Paniker’s Textbook of Microbiol-

B cell activation – reason for autoimmunity, E.g. 2-mer-


ogy – 10th ed – Page 180
captoethanol, LPS, Trypsin, Nystatin, Mycoplasma,
EBV, Malaria • Self-antigens in closed systems – when exposed in adult

life – immunological damage


3. Ans.  (a) IgM • E.g. Lens protein and sperms

Ref: Ananthanarayan and Paniker’s Textbook of Microbiol- 7. Ans.  (a) IgM against Fc portion of IgG
ogy – 10th ed – Page 181
Ref: Ananthanarayan and Paniker’s Textbook of Microbiol-
• Cold autoantibodies: Complete agglutinating antibod-
ogy – 10th ed – Page 183

ies – IgM
• Agglutinate erythrocytes at 4°C / not at 37°C

• Rheumatoid factor – autoantibody

• Seen in autoimmune hemolytic anemia, primary



• IgM or IgG or IgA RF

atypical pneumonia, trypanosomiasis, black water fever • Against Fc fragment of IgG


4. Ans.  (b) IgG 8. Ans.  (b) IgG

Ref: Ananthanarayan and Paniker’s Textbook of Microbiol- Ref: Ananthanarayan and Paniker’s Textbook of Microbiol-
ogy – 10th ed – Page 181 ogy – 10th ed – Page 182
• Warm autoantibodies: Incomplete, nonagglutinating

• Thyrotoxicosis – presence of autoantibody – LATS

antibodies—IgG • IgG antibody to thyroid membrane antigen


437
63 Transplantation and
Tumor Immunology
INTRODUCTION Table 2:  Graft rejection
y When an organ is damaged permanently or defective since
Days First Set Response
y
birth, it needs grafting / transplantation
(Mediated by T lymphocytes)
y The person who donates the organ is called donor and who
1–3 days Vascularization occurs
y
receives the organ is called recipient
4 day
th
Inflammation occurs (Graft is surrounded by
Table 1:  Types of grafts lymphocytes and macrophages)

Autograft Transplant from an individual to himself 5–9 days Blood vessels occluded by thrombi lead to necrosis
Isograft Between same genetic constitution 10 day th
Graft becomes a slough
E.g. Identical twins
y After rejection, when another graft from the same donor
Allograft Between two genetically nonidentical members of
y
is applied – these above said reactions occur more rapidly
same species
called second set response (mediated by antibodies) – also
Xenograft Between members of different species called white graft response (as the graft is rejected fastly and
makes it pale)
HISTOCOMPATIBILITY TESTING
y Before any organ transplantation, tissue matching is must HISTOCOMPATIBILITY ANTIGENS
y
y Tests need to be done are:
y When the recipient do not have that specific antigen that is
y
 Blood grouping
y
present in the grafted tissue, it leads to immune reactions and

 HLA compatibility
graft rejection, those antigens are called as histocompatibility

y HLA compatibility is tested by: antigens
y
 Microcytotoxicity test y When transplantation occurs between different sex chromo-

y
 Mixed leucocyte reaction some:

 Mixed lymphocyte culture  Grafts from female (XX) to male (XY) is accepted


 RFLP  Grafts from male (XY) to female (XX) is rejected


 PCR because of Y chromosome—this unilateral sex linked

y Sera for micro cytotoxicity test is obtained from multigravidae, histoincompatibility is called as Eichwald-silmser effect
y
placental fluid and from multiple blood transfusion recipients.
Privileged Sites
GRAFT REJECTION y Certain areas in the body accepts the graft without rejection,
y
y Any organ that is transplanted is considered as a foreign they are called as privileged sites
y Areas which lack circulation or lymphatic drainage are mostly
y
body by the immune system and it tries to reject it; Hence
y
immunosuppression is must before transplantation privileged sites
y The most effective drug is cyclosporine A, azathioprine and y Examples:
y
 Uterus – Fetus (it does not reject fetus as foreign body
y
steroids

y Rate of allograft rejection varies according to the tissue because of protection by placenta, which acts as an
immunological barrier)
y
involved; skin grafts are generally rejected faster than other
 Corneal transplantation (due to lack of vascularity)
tissues, such as kidney or heart.

 Cartilage
y After organ transplantation, certain series of reactions occur

 Brain
y
in some cases called Allograft reaction:

 Testes

GRAFT VERSUS HOST REACTION IMMUNOTHERAPY OF CANCER

Chapter 63     Transplantation and Tumor Immunology


y This reaction is opposite to that of allograft reaction
y y Passive immunotherapy:
y

 A special type of anti sera was given previously – that had




Host attacks graft → allograft reaction de-blocking activity – that did regression of tumors
Graft attacks host →GVH reaction y Active immunotherapy:
y

 Specific active immunotherapy: Purified tumor cell anti-




y GVH reaction occurs due to following conditions:


y

gens was made as vaccine and trial was done, But not in
 Graft contains immunocompetent T cells


use
 Recipient possess transplantation antigens that are absent


 Nonspecific active immunotherapy: BCG, Corynebacteri-




in graft um parvum, Dinitrochlorobenzene, Glucan, Levamisole,


 Recipient should not reject the graft (Allograft reaction)


Interferons
y Clinical features of GVH reaction are growth retardation,
y

y Specific adoptive immunotherapy:


y

diarrhea, HSM, anemia and lead to fatal condition called as  Lymphocytes




Runt disease  Transfer factor




 Immune RNA


MULTIPLE CHOICE QUESTIONS


1. Graft between identical twins is called as: 6. Graft versus Host reactions occurs only if:
 (Recent Question 2017)  (Recent Question 2017)
a. Autograft b. Isograft a. Graft contains immunocompetent B cells
c. Allograft d. Xenograft b. Graft contains macrophages
2. First set response of allograft rejection is by: c. Recipient must not reject the graft
 (Recent Question 2017) d. Recipient is not on immunosuppressive drugs
a. B cells 7. All of the following are Tumor associated antigens ex-
b. T lymphocytes cept: (Recent Question 2016)
c. Macrophages a. CEA
d. Phagocytes b. CA 125
3. Immunological enhancement in graft rejection is medi- c. PSA
ated by: (Recent Question 2017) d. HRP
a. Antibodies 8. Example for an immunomodulator:
b. T lymphocytes  (Recent Question 2016)
c. Macrophages a. Corynebacterium diphtheria
d. Antigens b. Corynebacterium parvum
4. Eichwald – Silmser effect is: (Recent Question 2018) c. Corynebacterium xerosis
a. Autosomal dominant histoincompatibility d. Corynebacterium minutissimum
b. Unilateral sex linked histoincompatibility 9. Nude mice is able to accept xeno graft because they
c. Autosomal recessive histoincompatibility lack? (Recent Question 2017)
d. None of the above a. T cells
5. Method to test for histocompatibility before transplan- b. B cells
tation: (Recent Question 2018) c. NK cells
a. Flow cytometry d. LAK cells
b. Microcytotoxicity test
c. ELISA
d. Latex agglutination test

439
ANSWERS AND EXPLANATIONS
Unit 9     Immunology

1. Ans.  (b) Isograft Histocompatibility testing is done by:


• Micro - cytotoxicity test
Ref: Ananthanarayan and Paniker’s Textbook of Microbiol-

• RFLP
ogy – 10th ed – Page 185

• Southern blotting

Autograft Transplant from an individual to himself • PCR


• Mixed lymphocyte reaction/culture


Isograft Between same genetic constitution

E.g. Identical twins 6. Ans.  (c) Recipient must not reject the graft
Allograft Between two genetically nonidentical Ref: Ananthanarayan and Paniker’s Textbook of Microbiol-
members of same species ogy – 10th ed – Page 188
Xenograft Between members of different species GVH reaction occurs only if:
• Graft – has immunocompetent T cells

2. Ans.  (b) T lymphocytes • Recipient has transplantation antigens; which is absent


in graft
Ref: Ananthanarayan and Paniker’s Textbook of Microbiol-
• Recipient must not reject the graft
ogy – 10th ed – Page 186

• Recipient should be on immunosuppression


• Allograft rejection – After transplantation series of


reactions occurs 7. Ans.  (d) HRP


• First set of response – reactions occurring from Day 1
Ref: Ananthanarayan and Paniker’s Textbook of Microbiol-

up to Day 10 – exclusively done by T lymphocytes


• Second set of response – When graft is rejected in

ogy – 10th ed – Page 189
first set of response, similar graft kept again will lead Tumor-associated antigens:
to drastic rejection is a fastest manner – mainly due to • Alpha feto protein (AFP)

antibodies along with CMI • Carcino embryonic antigen (CEA)


• Prostate specific antigen (PSA)


3. Ans.  (a) Antibodies • CA 125


   (HRP – Horse Radish Peroxidase enzyme used in ELISA)


Ref: Ananthanarayan and Paniker’s Textbook of Microbiol-
ogy – 10th ed – Page 186 8. Ans.  (b) Corynebacterium parvum
• CMI – responsible for graft rejection
Ref: Ananthanarayan and Paniker’s Textbook of Microbiol-

• Humoral antibodies may act in opposition with CMI


ogy – 10th ed – Page 190

– thus prevent the graft rejection – Immunological


enhancement • BCG and C. parvum are used as immunotherapeutic

agents for the treatment of cancer


4. Ans.  (b) Unilateral sex linked histoincompatibility • They act as immunomodulators.

Ref: Ananthanarayan and Paniker’s Textbook of Microbiol- 9. Ans.  (a) T cells


ogy – 10th ed – Page 187
• Grafts from male to female – rejected

Ref: The Mouse in Biomedical Research: Diseases – Page 400
• Grafts from female to male – accepted

• A nude mouse is a laboratory mouse from a strain with a

• Because of presence of antigens determined in Y



genetic mutation in FOXN1 gene
chromosome • This causes a deteriorated or absent thymus, resulting

• This unilateral sex linked histoincompatibility is called



in an inhibited immune system due to a greatly reduced
Eichwald – Silmser effect number of T cells.
• This mouse has no body hair that gives the name – nude

5. Ans.  (b) Microcytotoxicity test mouse


• It is valuable in research because it can receive many
Ref: Ananthanarayan and Paniker’s Textbook of Microbiol-

different types of tissue and tumor grafts, as it mounts


ogy – 10th ed – Page 187
no rejection response

440
64
Immunohematology
ABO BLOOD GROUP SYSTEM has this variant; to overcome this Direct Coomb’s test should
be performed
y ABO blood group system was first described by Landsteiner
y Rh compatibility testing is must only when the recipient is
y
y Based on the antigen present in the red cells and antibodies

y
Rh negative
y
present in the serum – blood groups are identified
y When Rh-negative person receives Rh-positive blood → not

y
all of them mounts immune reaction → some accepts → the
Table 1:  Blood groups reason is unknown but they are termed as non responders
y Fetal Rh incompatibility: Most common in second pregnancy
Blood group RBC Serum

y
because of maternal sensitization in the primigravidae forms
A A Anti B enormous amount of antibodies and that easily attacks in
B B Anti A subsequent pregnancies.
AB A and B None
O None Anti A and Anti B MATERNOFETAL ABO INCOMPATIBILITY
y Anti A, Anti B – IgM type – it wont cross the placenta
y
y These antibodies (isoantibodies) are called as natural y Anti O – IgG type – it will cross the placenta
y
y
antibodies because they seem to arise from genetic control y If mother is O blood group; Baby is whether A/B – chance of
y
without any apparent antigenic stimulation ABO incompatibility occurs
y Anti A and anti B antibodies – blood group antibodies – IgM y But it is a milder disease and not fatal
y
y
type (natural); anti O isoantibodies are IgG type; Following y Diagnosis:
y
any blood transfusion reactions or Rh incompatibility –  Direct Coomb’s test: Negative

antibodies occur—those are of IgG type  Indirect Coomb’s test: Positive

y Red cells of all ABO groups has a common antigen – called as
y
H antigen
y Some individuals lack A, B and H antigen – named OH TRANSFUSION TRANSMITTED INFECTIONS
y
blood or Bombay Blood Group. y Human immunodeficiency virus
y
y Hepatitis B virus
y
y Hepatitis C virus
RH BLOOD GROUP SYSTEM
y
y Cytomegalovirus
y
y When Rh negative mother gives birth to Rh-positive child y Syphilis
y
y
(father is Rh-positive), Child may develop hemolytic disease. y Malaria
y
Through these findings Rh incompatibility was identified y CJD prions
y
y This unique antigen reacts with rabbit antiserum to Rhesus y West nile virus
y
y
monkey erythrocytes (hence called as Rh factor) y Toxoplasmosis
y
y To identify whether a person is Rh-positive or negative →  When the red blood cells in blood bag are contaminated

with bacteria like Pseudomonas → it leads to agglutination
y
antigen D (Rho) presence/absence is identified
y Among Indians : 93% are Rh-positive and 7% Rh negative of blood cells with all type of antisera (A,B,O) → this is
y
y A variant of D antigen called Du is present – problem with due to unmasking of a hidden antigen that is normally
present in RBCs → T antigen → this is called Thomsen
y
this subtype is not all Du antigens reacts with anti D serum –
hence diagnosis of Rh antigen will be difficult when person Friedenreich phenomenon
Unit 6     Immunology

MULTIPLE CHOICE QUESTIONS

1. Serum of blood group A patients has: 5. The most sensitive method for the detection of Rh
 (Recent Pattern 2017) antibodies is: (Recent Pattern 2017)
a. Anti A b. Anti B a. Direct Coomb’s test
c. None d. Anti A and Anti B b. Indirect Coomb’s test
2. Bombay blood group has absence of: c. Precipitation test
 (Recent Pattern 2017) d. Hemagglutination test
a. A antigen 6. All of the following infections are transmitted through
b. A and B antigen blood transfusion except: (Recent Pattern 2017)
c. A, B and H antigen a. Cytomegalovirus
d. H antigen alone b. Variant CJD Prion
3. Rh antibodies responsible for hemolytic disease of c. Hepatitis A virus
newborn are: (Recent Pattern 2018) d. Malaria
a. IgD b. IgG 7. Thomsen Freidenreich phenomenon is due to:
c. IgA d. IgE a. A antigen (Recent Pattern 2017)
4. Rh compatibility testing should be considered only if: b. B antigen
a. Donor is Rh negative (Recent Pattern 2018) c. T antigen
b. Recipient is Rh negative d. H antigen
c. Both are Rh negative
d. None of the above

ANSWERS AND EXPLANATIONS


1. Ans.  (b) Anti B • But Rh negative people should receive only negative

blood group.
Ref: Ananthanarayan and Paniker’s Textbook of Microbiol-
ogy – 10th ed – Page 193 5. Ans.  (b) Indirect Coomb’s test
Blood group RBC Serum Ref: Ananthanarayan and Paniker’s Textbook of Microbiol-
A A Anti B ogy – 10th ed – Page 196
B B Anti A IgG anti-D (Rh) antibodies can be detected by:
• Colloid medium with 20% BSA (bovine serum albumin)
AB A and B None

• RBC treated with trypsin, pepsin or other enzymes


O None Anti A and Anti B • Indirect Coomb’s test (most sensitive method).

2. Ans.  (c) A, B and H antigen 6. Ans.  (c) Hepatitis A virus


Ref: Ananthanarayan and Paniker’s Textbook of Microbiol- Ref: Ananthanarayan and Paniker’s Textbook of Microbiol-
ogy – 10th ed – Page 194 ogy – 10th ed – Page 197
• Absence of A, B, H antigens in the RBCs – Bombay blood

Transfusion transmitted infections:
group • Bacterial contamination, E.g. Pseudomonas

• They have anti A, anti B, anti H in the serum.



• HIV

• HBV B, C,D

3. Ans.  (b) IgG • CMV


Ref: Ananthanarayan and Paniker’s Textbook of Microbiol- • Prions – Variant CJD


ogy – 10th ed – Page 196 • Syphilis


• Malaria
• Most of the Rh antibodies are – Complete IgG type

• Toxoplasmosis

• Minority can be incomplete – IgM type.


7. Ans.  (c) T antigen


4. Ans.  (b) Recipient is Rh negative
Ref: Ananthanarayan and Paniker’s Textbook of Microbiol-
Ref: Ananthanarayan and Paniker’s Textbook of Microbiol-
ogy – 10th ed – Page 197
ogy – 10th ed – Page 195
• RBC contaminated with Pseudomonas (Blood bag) –
• Rh-positive person can receive blood irrespective of the

agglutinates with all the blood group sera – due to the


recipient’s Rh status
442 unmasking of hidden antigen seen in normal human
RBC - T antigen - Thomsen Freidenreich phenomenon
HIGH YIELDING FACTS TO BE REMEMBERED IN IMMUNOLOGY

Chapter 64     Immunohematology


Components of Acquired immunity T cell, B cell, Classical complement pathway, Antigen presenting cells, Cytokines

What are the acute phase reactants? Serum Amyloid A, CRP, Complement factors, Fibrinogen, von Willebrand factor, alpha 1
antitrypsin, Mannose-binding protein, haptoglobin, ceruloplasmin

Herd immunity is seen in which of the Diphtheria, Pertussis, MMR, OPV and Small pox vaccine
vaccinations

What is adoptive immunity? Transfer of CMI from one individual to another (Transfer factor) – used in treatment of
lepromatous leprosy

Define epitope, paratope and haptens? Epitope – Smallest unit of antigenicity; Paratope – Specific site in antibody that reacts with
the antigen; Haptens – low molecular weight molecule that lack immunogenicity but retain
antigenicity

Name some examples for adjuvants? Alum, Mineral oil, Freund’s incomplete adjuvant, Freund’s complete adjuvant, LPS of
Bordetella pertussis, Mycobacterium bovis, Toxoid of DT and TT

Heterophile antigens – uses Weil-Felix reaction, Paul Bunnell test, Cold agglutination test, Streptococcus MG test,
Forssman antigen

Examples of superantigens Staphylococcal toxin – TSST, Exfoliative toxin, Enterotoxins; Streptococcal toxin –
Streptococcal pyrogenic exotoxin A and C ; Mycoplasma arthritidis mitogen – 1, Yersinia
enterocolitica, Y. pseudotuberculosis; EBV, CMV, Rabies nucleocapsid, HIV , Malassezia furfur

Chromosomes linked with different Heavy chain – Chromosome 14; Light chain kappa – Chr 2; Light chain lambda – Chr 22;
immunoglobulin classes

Immunoglobulins and its enzymatic Papain digestion – generates two Fac and one Fc fragment; Pepsin digestion causes one
digestion – List out the enzymes and F(ab’)2 and many small fragments; Mercaptoethanol caused four fragments – two H and two
fragments L chains

Name some abnormal immunoglobulins Bence Jones proteins – Multiple myeloma; Excess IgM – Waldenstrom’s macroglobulinemia;
and diseases alpha chain – Seligmann’s disease ; Gamma – Franklin’s disease; Cryoglobulinemia – multiple
myeloma and Hep C infection

What is the difference between T-independent antigen: No memory, no antigen processing, slowly metabolised, activate
T-independent and T-dependent B cells polyclonally, Activate both mature and immature B cells; T-dependent antigen—
antigen? Memory present, antigen processing step is needed, rapidly metabolized, Activate B cells
monoclonally and activate mature B cells only

What are all the types of Monoclonal Mouse mAb – 100% mouse derived proteins; Chimeric mAb – Recombination of mouse
antibodies? proteins (Variable region) and human proteins (Constant region); Humanized mAb – only the
antigen binding site is mouse derived ; remaining part human derived; Human mAb – 100%
human derived

What is lattice hypothesis? Zone of equivalence – Both antigen and antibody equal; Prozone phenomenon – Antibody
excess; Post-zone phenomenon – Antigen excess;

Examples of Precipitation reaction Lancefield grouping, Ascoli’s thermoprecipitation test , VDRL, RPR, Kahn test

Examples of agglutination tests Widal test, Standard agglutination test for Brucellosis, Coomb’s test, Blood grouping,
Heterophile agglutination test

Complement fixation test is used in Treponema pallidum immobilization test, Sabin Feldman dye test for Toxoplasma antibodies,
Vibriocidal antibody test

MHC Class I and MHC class II location Class I located in all nucleated cells and platelets ; Class II located in antigen presenting cells 443
Contd...
Unit 6     Immunology

What are professional and Professional APCs – Macrophages, Dendritic cells and B cells; Nonprofessional APCs –
nonprofessional antigen presenting Fibroblasts, Thymic epithelial cells, Pancreatic beta cells, Vascular endothelial cells, Glial cells
cells and thyroid epithelial cells

What are the CD markers for B cell, T B cell – CD19,21,24; T cell – CD3,4,8; NK cell – CD16, CD56
cell and natural killer cell?

What is IPEX syndrome? Deficiency of Foxp3 receptors leads to autoimmune disease ; Immune dysregulation,
Polyendocrinopathy, Enteropathy and X linked syndrome

Diseases due to humoral X linked agammaglobulinemia – Bruton disease ; Common variable immunodeficiency;
immunodeficiency (B cell defects) Isolated IgA deficiency; Hyper IgM syndrome; Transient hypogammaglobulinemia of infancy

Diseases due to cellular DiGeorge syndrome (Thymic hypoplasia), Chronic mucocutaneous candidiasis; Purine
immunodeficiency (T cell defects) nucleoside phosphorylase deficiency;

Name the combined SCID, Wiskott-Aldrich syndrome, Ataxia telangiectasia, Nezelof syndrome
immunodeficiencies

Disorders of phagocytosis Chronic granulomatous diseases, Myeloperoxidase deficiency, Chediak-Higashi syndrome ,


Leukocyte adhesion deficiency, Lazy leukocyte syndrome, Job’s syndrome, Tuftsin deficiency,
Shwachman’s disease

Different type of Grafts in transplant Autograft – one part of body site to another; Isograft – between genetically identical E.g.
immunology monozygotic twins; Allograft – genetically nonidentical (M/c); Xenograft – different species;

Lymph node and immune cells B cells located in follicle


T cells located in para cortex

Site of differentiation and maturation of T cells from Thymus


immune cells B cells from Bone marrow

Splenic dysfunction or postsplenectomy •• Streptococcus pneumoniae


leads to infections of: •• H. influenzae type b
•• Neisseria meningitidis
•• E. coli
•• Salmonella sp
•• Klebsiella pneumoniae
•• GBS

Antibody structure – Fab fragment Antigen binding fragment

Antibody structure – Fc fragment Constant


Carboxy terminal
Complement binding
Carbohydrate side chains
It determines the isotype

Complements and their functions C3b – opsonization


C345a – anaphylaxis
C5a – neutrophil chemotaxis
C5-9 – cytolysis by MAC

444
7

APPLIED MICROBIOLOGY
Unit Outline

1 Chapter 65 Applied Microbiology


65
Applied Microbiology
BACTERIOLOGY OF AIR, WATER AND MILK Table 2:  Organisms needed to be tested
Coliforms Marker of fecal contamination,
Water
E.g. E. coli
y Water is the most potential source of infection in many
Fecal streptococci If fecal streptococci are present–then it
y
diseases.
y For example means recent contamination of water
y
 Salmonella Clostridium perfringens If only Clostridium perfringens is seen

 Shigella without E. coli or fecal streptococci–

 Vibrio then it means the fecal contamination

 Poliovirus is not recent

 Hepatitis A

y Hence, constant examination of drinking water and proper
Air
y
decontamination is must to ensure safe drinking.
y Most of the infectious diseases are through inhalation
y
Table 1:  Bacteriological methods for water analysis y Two types of infection:
y
 Airborne infection: Infection that occurs by respiratory
Method to test Features

water droplets that are ≤5 μm in size
 Droplet infection: Infection that occurs by respiratory
Plate count • Water is cultured by pour plate culture

droplets that are >5 μm in size

method
• It is incubated under two different Table 3:  Organisms transmitted through air

temperatures:
• 37°C Droplet infection Airborne infection

• 22°C • Streptococcus pyogenes • Mycobacterium. tuberculosis



• Organisms that grow in 37°C are mostly • Neisseria meningitidis • VZV


• Corynebacterium • Measles virus

from human or animal origin


• Organisms that grow in 22°C are diphtheriae • Influenza virus

• Haemophilus influenzae B

saprophytic in nature

• Bordetella pertussis

Presumptive • Multiple tubes are taken at different • Yersinia pestis


coliform count dilutions of broth medium • Mycoplasma pneumoniae

• Water is inoculated in it • Influenza virus

• Rubella virus

• When coliform count is more than 10/

• Mumps virus

mL–then the water is unsatisfactory

• Adenovirus
• Probable numbers of coliforms/100

• Parvovirus

mL–is read from the probability table of

McCrady y Quality of air is must in operation theaters, ICU to avoid
y
Differential • It is called as Eijkman test nosocomial infections

coliform count • Test is done following presumptive y The bacteria carrying particles per cubic meter should
y

coliform count test not exceed 35 in empty theater and 180 during operative
• Test to check that whether the organism procedures

is E. coli or not y For ultra clean OT’s–the particle should be less that 1 for
y
empty theater and less than 10 during operation
Membrane • A known volume of water is filtered

filtration method through membrane filter–pore size is Table 4:  Amount of particles in OT
22 μm Operation theater Empty theater During operative procedure
• Bacteria if present–it gets filtered
Regular theater <35 <180

through the membrane
• This membrane is then cultured in a Ultra clean OT <1 <10

media
• Colonies are identified

y Measurement of air contamination is done by:
Unit 7     Applied Microbiology

Types of vaccines Features and examples


 Settle plate method
• Pathogens are inactivated by heat or


 Slit sampler or air sampler



Killed inactivated •

vaccine chemicals 

• Require primary and booster doses 

Milk

• Stable and safer 



y The most important milk borne diseases are:


y • Do not require cold facilities for storage 


 Tuberculosis
 E.g. Rabies, pertussis, influenza,
 Brucellosis
 hepatitis A vaccines and Cholera vaccine
 Streptococcal infection


Toxoids • Certain organisms like diphtheria and


 Staphylococcal infection


tetanus–produce exotoxins
 Salmonellosis


• Purified toxins inactivated by treatment


 Q fever


with formaldehyde–Toxoid
 Cowpox


• Toxoids are mixed with other vaccines to


y Examination of milk:
y

enhance their antigenicity


 Viable count method


E.g. DPT (Pertussis is adjuvant)


 Test for coliform bacilli
Cellular fractions • Bacterial polysaccharides


 Methylene blue reduction test


• Safety and efficacy is high 





 Resazurin test

• Limited usage


 Phosphatase test

E.g. Meningococcal vaccine–from the




 Turbidity test
polysaccharide antigen of the cell wall 



IMMUNOPROPHYLAXIS • Pneumococcal vaccine–polysaccharides


contained in the capsule of the organism


y Vaccine is an important contribution in preventing infections.
y

Subunit vaccines • Purified macromolecules–from


y Vaccines are prepared from organisms that have lost their

immunogenic components of
ability to cause disease, i.e. pathogenicity but which retain the pathogenic microorganisms by
immunogenicity to induce the immune response. recombinant DNA technology.
y Vaccine preventable diseases are:
y

E.g. Influenza, HbsAg being incorporated


 Diphtheria


in micelles, liposomes, isocost and


 Pertussis


virosomes for better delivery.


 Tetanus
• Genes encoding antigens–cloned in


 Measles

Recombinant •

 Poliomyelitis

vaccines bacteria, yeast and mammalian systems
 Mumps

using recombinant technology.
 Rubella

E.g. Hepatitis B and Pertussis vaccines
y Types of immunization:
y

y Passive immunization:
y

 Active immunization
 Done with immunoglobulins:


 Passive immunization
 E.g. Pooled immunoglobulins–short term–E.g. 
Hep A or


y Active immunization:
y

measles
 It involves both humoral and cell-mediated immunity
 Hyperimmune Igs–HBIG, HTIG, HRIG 



 It has specificity and immunological memory




y Newer vaccines for children are:


y

 Types of vaccines are:


 HiB


Š Live attenuated vaccines


Š

 Rotavirus–Rotarix


Š Killed inactivated vaccines


Š

 Pneumococcal vaccine


Š Toxoids 

Š

 7 PCV or 13 PCV


Š Cellular fractions 

Š

Š Subunit vaccines
Vaccines for Travelers (International
Š

Š Recombinant vaccines
Š

Table 5:  Types of vaccines and their examples


Regulations)
Traveling to endemic area and based on countries regulations–
Types of vaccines Features and examples following vaccines are must for travelers:
Live attenuated • Attenuation is process of killing the

y Cholera vaccine
y

vaccine pathogenicity but retaining the virulence y Hepatitis A vaccine


y

• More potent than killed vaccines



y Hepatitis B vaccine
y

• The live organisms multiply in the


y Japanese encephalitis vaccine


y

host–antigenic dose is higher than killed y Meningococcal vaccine


y

vaccine y Rabies vaccine


E.g. Oral Polio, Measles, Mumps and
y

448 y Typhoid vaccine


BCG vaccines
y

y Yellow fever vaccine


y

Contd...
Immunomodulation

Chapter 65     Applied Microbiology


Seven steps of handwashing (Duration 15–20 sec)
y Under certain conditions, the immune system needs to
y

Step 5 Rotational rubbing of left thumb clasped in right palm


be modified, i.e. modulated or suppressed. E.g. In organ and vice versa
transplantation–the immune cells should be suppressed
Step 6 Rotational rubbing backwards and forwards with
by immunosuppressant agents–in order to prevent graft
clasped fingers of right hand in left palm and vice versa
rejection
y Other conditions where this is used are:
y
Step 7 Rotational rubbing of right wrist and vice versa
 Crohn’s disease


 Irritable bowel syndrome



y PPE–personal protective equipment are must like face mask,
y

 Ulcerative colitis

gloves and goggles during any outbreak or highly contagious
y Drugs used are:
y
disease like influenza, Ebola.
 Cycloserine


 Azathioprine


BIOMEDICAL WASTE MANAGEMENT:


 6-Mercaptopurine
(NEW RULES 2016)


 Corticosteroids


y According to the new rules of biomedical waste management—


y

HEALTH CARE ASSOCIATED INFECTIONS there are only four waste categories
y Health care associated infections were previously called as
y Table 7:  Biomedical waste categories
nosocomial or hospital acquired infections
Color of Type of waste Waste treatment
y 5–10% of infections is HAI in developed countries
y

the bag
y 25% of infections is HAI in developing countries
y

y Definition of HAI:
y
Human anatomical Incineration or plasma
 Clinical infection that develops after 48–72 hours of

Yellow waste pyrolysis or deep burial
admission to a hospital resulting from exposure to Animal anatomical waste Incineration or plasma
organisms that are endemic within the hospital pyrolysis or deep burial
y Common infections that comes under HAI are:
Soiled waste Incineration or plasma
y

 Catheter associated UTI


pyrolysis or deep burial


 Blood stream infections




 Ventilator associated pneumonia



Expired or discarded Return back to
 Surgical site infections

medicines the manufacturer
 Transfusion associated infections

or supplier for
 Acute gastroenteritis

incineration at
y Sources of HAI:
y
temperature >1200°C
 Endogenous source is the most common source of

Chemical waste Incineration or plasma
infection pyrolysis or deep burial
 Cross infection

or encapsulation
 Infections from environmental source
Chemical liquid waste Pretreatment and then


y The most important precautionary measure to avoid HAI is


disposal
y

 Standard precautions


 WHO moments of hand hygiene



Discarded linen, Nonchlorinated
y WHO’s five moments of hand hygiene are:
y
mattresses, beddings, chemical disinfection
 Before touching a patient

contaminated with blood followed by
 Before a procedure

or body fluids Incineration or plasma
 After a procedure or body fluid exposure risk

pyrolysis
 After touching a patient

Microbiology, Pretreat to sterilize
 After touching patient’s surroundings

biotechnology and other with nonchlorinated
clinical laboratory waste chemicals on site
Table 6:  Steps of handwashing
Red Contaminated waste Autoclaving or
Seven steps of handwashing (Duration 15–20 sec) like plastic bag, bottles, microwaving/
Step 1 Rub hands palm to palm pipes or containers hydroclaving followed
Step 2 Right palm over dorsum with interlaced fingers by shredding or
mutilation
Step 3 Palm to palm with fingers interlaced
Contd...
Step 4 Backs of fingers to opposing palms with fingers
interlocked
Contd...
449
Unit 7     Applied Microbiology

Color of Type of waste Waste treatment Categories Features Organisms


the bag
White Waste sharps included Autoclaving or dry B Easy to disseminate but low • • Brucellosis
translucent needles, syringes with heat sterilization mortality • • Glanders
fixed needles, needles followed by shredding • • Q fever
from needle tip cutter or mutilation or • • Ricin toxin
or burner, scalpels and encapsulation in metal C Mass dissemination is possible Nipah virus
blades container or cement
concrete
ZOONOTIC DISEASES
Blue Glassware: Broken Disinfection with Table 9:  Zoonoses and causative pathogen
cardboard or discarded and sodium hypochlorite Zoonoses Pathogen
box with contaminated glass treatment or
Viral zoonoses
blue label including medicine vials autoclaving or
Rabies • Rhabdoviridae
and ampoules except microwaving or

Tick-borne encephalitis (TBE) • Flaviviridae


those contaminated hydroclaving then sent

with cytotoxic wastes; for recycling Bacterial zoonoses


metallic body implants • Brucellosis
• •• Brucella spp.
• Lyme disease
• •• Borrelia burgdorferi
• Plague
• •• Yersinia pestis
• Q fever
• •• Coxiella burnetii
• Enteric salmonellosis • Salmonella enterica
BIOTERRORISM • •

Protozoan zoonoses
Table 8:  Categories of bioterrorism agents that can be • Toxoplasmosis
• • Toxoplasma gondii

used as an warfare • Cryptosporidiosis


• • Cryptosporidium hominis;

C. parvum
Categories Features Organisms
Helminthic zoonoses
A High risk and gives a threat to •• Anthrax • Echinococcosis
• •• Echinococcus granulosus
national security •• Botulism • Taeniasis
• •• Echinococcus multilocularis
•• Plague •• Taenia saginata
•• Small pox •• Taenia solium
•• Tularemia •• Taenia asiatica
•• Ebola Zoonoses caused by arthropods
•• Marburg • Pseudo scabies
• • Sarcoptes spp.;

• Mite species from


domestic animals
Contd...

450
8

INFECTIOUS DISEASES
Unit Outline
Chapter 66. Introduction
1 Chapter 67.
Chapter 68.
Fever of Unknown Origin
Infections of Ear, Nose and Throat
Chapter 69. Infections of Eye
Chapter 70. Infection in Lower Respiratory Tract–
Pneumonia
Chapter 71. Gastrointestinal Infections
Chapter 72. Cardiovascular Infection–
Endocarditis
Chapter 73. CNS Infections
Chapter 74. Skin and Soft Tissue Infections
Chapter 75. Infections of Bones and joints
Chapter 76. Sexually Transmitted Infections
Chapter 77. Urinary Tract Infections
Chapter 78. Infections Related to Obstetrics and
Gynecology
Chapter 79. Surgical Site and Related Infections
Chapter 80. Infections in Special Hosts
Chapter 81. Antimicrobial Chemotherapy–
A Short Review
Chapter 82. Multiple Choice Questions
66
Introduction
A GLIMPSE ON INFECTIOUS DISEASES CLINICAL APPROACH TO A PATIENT
SUSPICIOUS OF INFECTIOUS DISEASE
• Concepts from

ƒ Harrison principles of internal medicine – 19th edition
ƒ
ƒ Important points from Mandell’s infectious diseases – 8th
ƒ
edition
ƒ Consolidated points from Manson’s tropical disease – 22nd
ƒ
edition
ƒ Important points from various standard extbooks of
ƒ
individual subjects
• Clinical syndromes—an outline

• Infections in special conditions

• Infections in each specialty contributed by subject experts

• Epidemic diseases and outbreaks

• Emerging and re-emerging infections

• Antimicrobial chemotherapy

INTRODUCTION
y Microbes form an integral part of biology where every living
y
being is interdependent on each other
y A healthy human is colonized by 100 trillion bacteria and
y
most of them reside in alimentary tract.
y Although most of the bacterial colonization is beneficial.
y
Infectious diseases are the second most leading cause of
death worldwide.
y Microbes are the pathogens responsible for most of the
y
communicable diseases, especially in developing countries.
EXPOSURE HISTORY Table 3:  Causes for eosinophilia
Unit 8     Infectious Diseases

Table 1:  Exposure history and the types of diseases caused Infectious causes Noninfectious causes
• • Helminthic infections • Atopy

Travel history Endemic diseases like • HIV • DRESS syndrome


• Tuberculosis
• •

• Protozoa like Dientamoeba fragilis • Churg Strauss


• Malaria
• •

• Chlamydia trachomatis syndrome


• Yellow fever

• Bartonella henselae • Hyper IgE syndrome


• Dengue
• •

• Cryptococcus
Some diseases are not in certain countries

• Coccidioides
E.g. Trypanosoma is not in India (we can

rule out such condition from our differential


diagnosis) y Inflammatory markers:
y

 C-reactive protein (CRP)


History of • • Dog bites – Rabies 

 Erythrocyte sedimentation rate (ESR)


exposure to • Dog – Hydatid cyst 

y Marker of sepsis:

animals or • • Cats – Bartonella henselae, Toxoplasmosis y

 Procalcitonin
insects • • Bats – Rabies 

 CRP
• • Mite islands – Scrub typhus 

• • Cattles – Brucellosis
• • Reptiles – Salmonella Table 4:  Causes for very high ESR counts
• Rodents – Leptospirosis and plague
Infectious Inflammatory Malignancies Others

• Rabbits - Tularemia
causes causes

Dietary habits Mainly helpful in parasitic infections and


• SABE • SLE • Multiple • Drug
gastroenteritis:
• • • •

• Abscess • Rheumatoid myeloma induced


• Fried rice – Bacillus cereus
• •

• Osteomyelitis arthritis • Lymphoma • Tissue


• Milk and milk products – Staphylococcus


• • •

• TB • Giant cell • Leukemia injury


aureus
• • •

• UTI arthritis • Ca • Renal


• Honey in babies – Infant botulism
• • •

diseases

• Raw sea foods – Vibrio


• Contaminated water, food – Typhoid


INVESTIGATIONS DONE FOR PATHOGEN

• Unpasteurised eggs – Salmonella


• Consumption of raw liver – Clonorchis


IDENTIFICATION
• Consumption of raw pork – Taenia solium

• Raw beef or undercooked beef – Taenia



Samples usually tested are:
saginata y Blood
y

y CSF
y

y Sputum
IMPORTANCE OF BASIC INVESTIGATIONS IN
y

y Urine
INFECTIOUS DISEASES
y

y Relative bradycardia seen in infections is called as Faget’s


y
Blood Culture
sign – where the patients have lower heart rate despite of y Blood culture is mainly done in bacteremia, septicemia,
y

higher body temperature infective endocarditis and fever of unknown origin


y The volume of blood cultured is the most important factor in
y

recovery of pathogenic microorganisms from blood


Table 2:  Infectious causes of relative bradycardia
y 8 to 10 mL of blood should be inoculated into each of two
y

Bacteria Virus Parasite bottles per blood culture


y A single blood culture lacks sufficient sensitivity to detect
• Salmonella typhi • Yellow fever • Babesia
y

bacteremia, and in some isolates, the result cannot be


• • •

• Brucella • Dengue
interpreted (e.g., coagulase-negative staphylococci).
• •

• Coxiella • Viral hemorrhagic


y Hence serial testing at regular intervals are needed
• •

• Francisella fever
y

y Routine laboratory testing in culture media will take a


• Legionella
y

minimum of 48 hours to a maximum of 7 days for identification


• Corynebacterium
of growth

• Rickettsiae
y The time required for an automated blood culture system to

y Eosinophil count:
y

flag a bottle as positive (i.e., to detect a growth signal), or so-


 Most of the parasitic infestations has eosinophilia


called “time to positivity” – helps in identification of catheter-


 Especially all of the helminthic infections has eosinophilia


related blood stream infections


 Other causes for Eosinophilia are:
454


Š HIV Š

Š Cryptococcus neoformans
Š
Type of Blood Cultures

Chapter 66     Introduction


y Manual method using traditional culture media plates
y

y Automated methods using BACTEC, BacT Alert MALDI-TOF


y

y Blood culture with antimicrobial susceptibility pattern helps in appropriate treatment of the condition.
y

Table 5:  Protocol for blood culture reporting


48 hours Preliminary report
72 hours If culture is positive give report with antimicrobial susceptibility
Day 5 Second report – if positive give report with AST
Day 7 Final report

CSF Examination
y Three parts of CSF are collected separately under strict aseptic precautions
y

ƒ 1 part is used for cell count


ƒ

ƒ 2 part is used for serological testing


ƒ

ƒ 3 part is used for CSF culture


ƒ

Table 6:  Lab diagnosis of meningitis


Parameters Bacterial Viral meningitis Fungal Parasitic TB meningitis Encephalitis
meningitis meningitis meningitis
WBC count (/μL) >100 25–500 40–600 150–2000 25–100 50–500
Differential count More PMN More lymphocytes PMN/ More More More
cells lymphocytes eosinophils lymphocytes lymphocytes
Glucose Less normal Less / normal Normal Less/near normal Normal
Protein Very high Normal to high Very very high Very high high Normal to
high

Sputum Examination GENOMICS AND INFECTIOUS DISEASE


y Sputum needs to be concentrated and for isolation of
y y Traditional culture methods are nowadays being replaced by
y

M. tuberculosis – it needs decontamination procedure nucleic acid based detection methods


y Sputum has normal commensals, which needs to be removed
y y Many methods that are based on nucleic acids are:
y

to isolate specific pathogen  Polymerase chain reaction (PCR)




y Sodium hydroxide (NaOH), the most commonly used


y  16S ribosomal gene PCR


decontaminant, also serves as a mucolytic agent helps in  Nucleic acid amplification test (NAAT)


isolation of MTb  Gene sequencing




y Gram staining of sputum helps in presumptive diagnosis by


y  DNA microarrays.


identifying the pus cells, organisms seen or yeast cells


y Acid fast staining is done to rule out tuberculosis
y
Table 7:  Polymerase chain reaction (Amplification test)
y Sputum culture for aerobic bacteria is done to identify the
y

Types of PCR Characteristics


organism in pneumonia.
Conventional PCR done with the help of thermocycler
Urine Examination PCR and PCR product is visualized by agarose gel
electrophoresis
y Urine collection by proper method is most important in
y

isolating the organism because urine is more prone for Reverse RNA is converted to dsDNA with the help
contamination with vaginal or fecal samples transcriptase of reverse transcriptase and then PCR is
y Mid-stream clean catched urine is most important
y
RT-PCR performed; Helpful for RNA viruses
y Normally urine is sterile, which has no pus cells and no
y

qPCR Quantitative or Real time PCR – Measure the


organisms quantity of PCR product while the process is
y Hence seeing single bacilli/cocci in hpf in Grams’ stain is
y
running (hence named as real time)
suggestive of UTI Detection is based on: Nonspecific fluorescent
y Rapid dipstick tests are available to see the nitrites production
y dyes and specific set of DNA probes
and leucocyte esterase positivity which suggests infections Nested PCR Two set of primers are used to increase the
y Urine culture is done with quantitative method to know that
y

specificity; more specific


>105 bacilli should be counted to coin the term UTI 455
y Antimicrobial testing done with the culture isolates to show Multiplex PCR More than one pathogen can be identified
using multiple primers
y

the susceptible drug.


Nonamplification Tests
Unit 8     Infectious Diseases

Clinical Most Common Empirical Therapy Drugs


y Loop mediated isothermal assay (LAMP): In PCR, different
y
Illness Organisms
temperatures are used; Here at constant temperature the Community- S. pneumoniae Azithromycin
reaction is done; Hence there is no need of thermocycler; acquired Myco.
Primers used are called as loop primers. pneumonia pneumoniae
y DNA microarray: DNA spots are coated on a microarray plate
y

H. influenzae
and this hybridise with the cDNA in the test sample – most Chlamy.
specific and can easily detects large number of nucleic acid pneumoniae
sequences in the target simultaneously
Sequencing based assays: Hospital- S. pneumoniae Antipseudomanal coverage
y Genetic mapping
y

acquired H. influenzae like


y Next generation sequencing
y

pneumonia S. aureus Ceftazidime + IMP/MRP/PT


Molecular methods in CNS infections: GNB + FQ’s or AG’s
y Targeted nucleic acid detection methods are often more
y

sensitive than conventional culture-based or antigen


Intra- Anaerobes Cefoxitin or
detection methods and may detect organisms that are
abdominal Streptococcus Metronidazole +
nonviable or uncultivable.
infection Clostridium Ceftazidime
y Molecular methods are particularly well suited for the
or
y

diagnosis of CNS infections because CNS samples are less


IMP/PT
prone to common causes of false-positive (e.g., contamination
or presence of nonpathogenic colonization) or false-negative Skin and S. aureus Dicloxacillin or
(e.g., inhibition) results compared with other body sites. soft tissue S. pyogenes Cephalexin or
y However, except for herpes simplex virus (HSV) and JC virus,
y

infections Clindamycin or
the true clinical sensitivity of most molecular tests for CNS Nafcillin
infections is not known.
Septic shock S. aureus Vancomycin +
Table 6:  Empirical treatment given for infectious S. pneumoniae PT/IMP/MRP
diseases GNB
Clinical Most Common Empirical Therapy Drugs
Illness Organisms
Abbreviations
Meningitis S. pneumoniae Vancomycin + Ceftriaxone
• IMP, Imipenam; MRP, Meropenam; PT, Piperacillin/Tazobactam
N. meningitidis

• AG, Aminoglycosides; FQ, Fluoroquinolones;


Endocarditis S. aureus Vancomycin + Ceftriaxone


Streptococcus sp.,
CONS
Contd...

456
67
Fever of Unknown Origin
ACUTE FEBRILE ILLNESS Table 2:  Specific presentation of fever along with other
y Any patient who is having increased temperature (>100°F) manifestations
y
with other signs pertaining to systemic illness is called acute
Presentation Examples and Features
febrile illness
y Fever of unknown origin (FUO) is a term coined when it
Sepsis without • Septic shock
y
meets the following criteria:


 Fever higher than 38.3ºC on several occasions foci of primary • Babesiosis


infection • Infection in asplenic patients

 Duration of fever for at least three weeks (21 days)


(S. pneumoniae (M/c), H. influenzae,

 Uncertain diagnosis after 1 week of study in the hospital
N. meningitidis)

y Most common reasons for FUO/PUO are
• Tularemia
y
 Infections


• Yersinia

 Malignancies



 Connective tissue diseases (e.g., vasculitis, rheumatoid

arthritis) Sepsis with skin • Meningococcemia


y Fever should be diagnosed along with other presenting manifestations • Rocky mountain spotted fever
y

complaints or by other systemic manifestations; commonly • Rheumatic fever (Erythema marginatum,


encountered presentations are: subcutaneous nodules)
 Viral fevers • Infective endocarditis (Janeway lesions,


 Pneumonia Oslers’ nodes, Roth spots)

 Abscess • Viral hemorrhagic fevers with sepsis


 Bacterial endocarditis • Purpura fulminans (it is a cutaneous


 Meningitis manifestation of DIC)

 Hemorrhagic fevers • Ecthyma gangrenosum (caused by


Pseudomonas aeruginosa)
Table 1:  FUO definitions • Erythroderma (seen in toxic shock

Terminology Definition syndrome)
used
Classic FUO • Fever of unknown origin is a term coined Sepsis with soft • Necrotizing fasciitis (caused by

tissue infections Streptococcus pyogenes)

when it met the following criteria:
ƒ Fever higher than 38.3°C on several • Clostridial myonecrosis

ƒ
occasions
ƒ Duration of fever for at least three weeks Neurological • Bacterial meningitis

ƒ
(21 days) infections with/ • Suppurative intracranial infections

ƒ Uncertain diagnosis after 1 week of study without sepsis • Brain abscess

ƒ
in the hospital • Cerebral malaria (caused by P. falciparum)

Nosocomial Fever that is occurring in a patient who has • Lemierre’s disease (caused by

FUO been hospitalized for at least 24 hours without Fusobacterium necrophorum)
a defined source prior to admission or after 3
days of evaluation Miscellaneous • Rhinocerebral mucormycosis (caused by

infections with zygomycetes – fungi)
Neutropenia Persistent fever in a patient with neutropenia
fulminant course • Acute bacterial endocarditis
FUO (absolute neutrophil count < 500 cells/mm3

• Inhalational anthrax
even after 3 days of complete diagnostic

• Influenza
testing

• Hanta virus pulmonary syndrome

HIV-related Persistent or recurrent fever for more than
FUO 4 weeks in a HIV patient even after 3 days of
complete diagnostic testing
DIAGNOSIS OF PYREXIA OF y CT scan of chest
Unit 8     Infectious Diseases

y Molecular methods to identify bacterial infection include


UNKNOWN ORIGIN
y

polymerase chain reaction (PCR) and detection of bacterial


A thorough history should include the following information: 16S ribosomal ribonucleic acid (RNA) genes or host RNA
Travel, animal exposure (e.g., pets, occupational, living on a signatures.
farm), immunosuppression (with the degree noted), drug and
toxin history, including antimicrobials and localizing symptoms. Table 3:  Empirical therapy in common infectious
Since all the other investigations would have been completed
diseases with fever
till 21 days – some unique tests are needed to diagnose the cause
for PUO. Infections Empirical Treatment
Bacterial meningitis Ceftriaxone + Vancomycin
DIAGNOSTIC TESTS INCLUDE
Brain abscess Vancomycin + Metronidazole
y Basic blood counts including ESR/CRP
+ Ceftriaxone
y

y y Serum lactate dehydrogenase


y y Tuberculin skin test or IGRA Cerebral malaria Artesunate (or) Quinine
y HIV testing (antibody testing and viral load assay)
Acute bacterial Ceftriaxone + Vancomycin
y

y Three routine blood cultures drawn from different sites over


endocarditis
y

a period of at least several hours without administering


antibiotics (if not done already) Septic shock Vancomycin + Gentamycin
y y Rheumatoid factor + PT (or) cefepime
y y Creatine phosphokinase Meningococcemia Penicillin (or) Ceftriaxone
y y Heterophile antibody test in children and young adults
y y Antinuclear antibodies Necrotizing fasciitis Vancomycin + Clindamycin
y y Serum protein electrophoresis + Gentamycin
y y Computed tomography (CT) scan of abdomen Clostridial myonecrosis Penicillin + Clindamycin

458
Infections of Ear, 68
Nose and Throat
NORMAL BACTERIAL FLORA OF Example of group A.- S. pyogenes. most important



Suppurative (tonsillitis TSS septicemia) and nonsuppura-
NASOPHARYNX



tive (AGM, RF) diseases
y Colonization occurs in 48 hours of birth y B not in nasopharynx- not in ENT disease; cause neonatal
y
y
y Normal Flora sepsis. e.g. S. agalactiae.
y
 Gram positive and negative aerobes like, oral streptococci,

Neisseria, bacteriophages, diphtheroids and anaerobes. Oral Streptococci (Viridians)
y Asymptomatically carried pathogens:
y Alpha hemolysis (incomplete)
y
 Streptococcus pneumoniae,  H. influenza, M. catarrhalis,

y
y Normal oral and nasopharynx flora

N meningitidis.
y
 Patients on antibiotic therapy
y Dental abscess and periodontal disease
y
y Native wall endocarditis

y Enterobacteriaceae (aerobic Gram-ve bacilli) y
y Use as bacteriotherapy beneficial in recurrent otitis media
y
y
Bacterial Interference S. pneumoniae
Prevention of adhesion of pathogens by the normal oral strep- y Alpha hemolysis
y
tococci. y Draughtsman like colonies that are bile soluble and optochin
y
sensitive
y Normal nasopharynx flora in 0-2 years age
BACTERIAL CAUSES OF ENT INFECTIONS
y
y Most common cause of otitis media sinusitis and pneumonia
y
Gram-positive Bacteria in all ages
y Meningitis from hematogenous spread from respiratory
Staphylococci
y
infection
y Gram positive, catalase + y Vaccine available.
y
y
y Divide in 2 planes → clusters
y
y Grow in 18-40° and in NaCl → presence on skin and mucosa Enterococci = fecal streptococci 
y
y Divided on the basis of coagulase test y Group D lancefield
y
 Positive: S. aureus presents in anterior nares. Has many
y
y Resistant to bile and antibiotics

virulence factors- Coagulase, CPS, FBP (fibrinectin
y
y Normal fecal flora → UTI abdominal sepsis
binding protein), Protein A, Staphylokinase Hemolytic
y
toxin, Other toxins: TSST enterotoxin
 Negative: not significant in ENT.
Corynebacteria

y G+ pleomorphic bacilli
y
Streptococci y Chinese letter like on gram stain.
y
y Daisy head appearance on tellurite containing media
y G+ catalase -ve
y
y Mostly bacterial Flora. Only few pathogens 
y
y Divide in single plane.
y
y C. diptheriae
y
y Divided on the basis of - hemolysis on blood agar, Lancefield
y
 Disease due to exotoxin (produced only by bacteria
y
antigen, Growth

infected with lysogenic phage)
 Exotoxin → inhibits protein synthesis → pharynx damage
Pyogenic Streptococci

→ into blood stream → myocardial and neurologic toxicity
y Beta hemolysis (complete)  Three biotypes 

y
y Divided into lancefield Groups A B C G Š Gravis: severe disease
Š
y
y A, C, G -in nasopharynx. Š Intermedius: severe disease 
Š
y
Š Mitis
Š
Gram–Negative Bacteria Pseudomonas
Unit 8     Infectious Diseases

Neisseria yy Thin G –ve bacilli


yy Resistant to many antimicrobial and disinfectants.
y G -ve diplococci
y

yy Opportunist infections
y 10 sps - 2 are the main pathogens. Adapted to survive in man..
y

yy P aeruginosa 
needs CO2 37°C enriched media.  Produce pyocyanin pigment.


 N. gonorrhoeae: genital and urinary infections sexually


 Malignant otitis externa, surgical wounds and leg ulcers


acquired pharyngitis
 N. meningitidis serotypes. A, B, C, W135, Y 
Anaerobes 


Š Normal flora in 10%. Disease only in small percentage 


Š

Š Meningitis, septicemia and septic arthritis


Š
y Only in strict absence of oxygen. Even while transporting the
y

pus (large volume of pus and long incubation needed).


Moraxella Catarrhalis y Resistance to metronidazole used in culture.
y

y y Morphologically like Neisseria y Includes many G + and -ve microbes


y

y y Third most common cause of otitis media y Part of normal and large bowel flora.
y

y Acute exacerbation of COPD


Bacteroides Fragilis
y

y y Resistant to penicillins (produces beta lactamase)


y G -ve bacilli
y

Haemophilus  y Small % of oral and bowel flora


y

y G -ve pleomorphic coccobacilli


y
y Most common anaerobes in infections—usually
y

y Major part of normal resp flora


y
polymicrobial
y Has specific growth requirements factors V and X, hemin,
Fusobacterium Necrophorum
y

NAD
y H influenzae
y
y Fusiform G -ve
y

 Poor growth in blood agar coz needs factors V and X



y Normal oral flora
y

 Forms 10% of nasopharynx flora



y Necrobacillosis - Rapidly progressive disease
y

 Mainly noncapsulated. One of common cause of otitis




media and sinusitis


 Capsular types a-f.

Microaerophilic Bacteria
 Type b severely invasive,  Meningitis, epiglottitis, septic

y Very low concentrations of O2, slow growing, difficult to isolate
y

arthritis y Part of normal oral and large bowel flora


y

 Hib vaccine available.



y E.g. Actinomyces (A. israelii,→ clumps of G +ve bacilli, Sulfur
y

containing granules
Bordetella Pertussis
y G -ve coccobacilli
y
Mycobacteria
y Fastidious growth needs. 5 days incubation in charcoal
y
y Acid and alcohol fast, grow slowly in aerobic conditions. Not
y

containing media  isolated on routine agar


y Virulence factors.
y
y 25 species divided into M. tuberculosis and others (atypical
y

 FHA -cell surface-associated adherence protein 



mycobacteria)
 Pertussis toxin causes cough and lymphocytosis.

y Mycobacterium tuberculosis
y

y Disease in nonvaccinated infants (refusal / “too young ‘)


y
 Grows slowly on agar containing fatty acids (egg


y Reservoir-adult carriers, subclinical cases


y
containing L-J medium → buff colored bread crumbs like
appearance)
Enterobacteriaceae  4-6 weeks for colonies to appear


 Decrease isolation time → use of liquid media with




y y 100 + species like E. coli, Klebsiella, pseudomonas automated detection of growth and DNA probes
y y Aerobic G -ve bacilli  Samples to be handled in category 3 lab using safety


y y Grow in both aerobic and anaerobic condition and bile salts   cabinets.
y y Divided based on lactose fermentation 
y Normal Flora in bowel
y
y

y Not in nasopharynx normally; only in hospitalized patients.


Atypical Mycobacteria
y y UTI, abdominal sepsis and rarely respiratory infection. y M. avium-intracellulare, M. kansasii, M. scrofulaceum and
y

M. malmoense
Klebsiella Rhinoscleromatis y Cervical lymphadenitis in <12 years age
y

y Disseminated M avium intracellulare → late opportunistic


y

y Chronic granuloma in upper respiratory tract


y

infection in HIV patients.


y Endemic in South America, East Europe and Asia
y

460
Mycoplasmas y Candida in temperate countries (C. albicans, C. parapsilosis, C.

Chapter 68     Infections of Ear, Nose and Throat


y

tropicalis)
y No cell wall → not isolated on routine culture → serology
y

used in diagnosis
y No cell wall → antibiotics targeting cell wall ineffective
y
Clinical Features
y M. orale and M. salivarium – normal nasopharynx flora – no
y y Most common complaint- fullness and itching
y

disease y Partial hearing loss, tinnitus


y

y M. pneumoniae – common cause of pneumonia in children


y y Blotting paper like mat of fungus with black sporing heads
y

and young adults with epidemics every 4 years.


y Rare cause of otitis media
y
Treatment
y Aural toileting
Spirochaete
y

y Apply antifungal cream— clotrimazole / econazole.


y

Borrelia and Treponema


y B. vincenti: Acute necrotizing gingivitis and Vincent’s angina,
y External Ear
accompanied by fusobacteria y Dermatophytosis – commonest infection
y

y B. burgdorferi: Lyme disease


y

y Trichophyton rubrum most common fungus


y

FUNGAL CAUSE OF ENT INFECTIONS NOSE


y Otomycosis and oropharyngeal candidiasis – more common
Entomophthoromycosis
y

y Otherwise fungal infections uncommon


y

y Chronic localized subcutaneous fungal infection originating


y

in nasal mucosa
EAR y Hard nontender mass that spreads painlessly to adjacent
y

subcutaneous tissue- severe facial disfigurement.


Otomycosis y Skin intact
y

y Superficial diffuse fungal infection of ear canal


y
y Conidiobolus coronatus
y

y More common in tropical climate and in adults


y
y In men and women working in tropical rain forests, uncom-
y

y Not contagious
y
mon in children
y Other factors: Seborrheic dermatitis, psoriasis, prolonged
y
y Broad, nonseptate, thin walled, irregularly branching hyphae,
y

antibiotic steroid drops may be surrounded by splendore – hoeppli material


y Aspergillus most common cause especially in tropical and
y
y Oral fluconazole – 200- 400 mg /day. For at least a month after
y

subtropical (A. niger most common, others – A. flavus, resolution


A. fumigatus) y Surgery useless
y

y Relapse common.
y

Table 1:  Rhinosporidiosis

Acute invasive Chronic invasive Chronic granulomatous invasive


Rapidly progressing Slow progressing Slow progressing
Immunocompromised/ diabetics with DKA Both immunocompetent and immune Immunocompetent with chronic
compromised sinusitis
• Mucosa → orbit and brain
• • Fungal ball → invasive on
• • Profuse fungal growth with

• Complications – vascular invasion – thrombosis,


• suppression of immunity localized tissue invasion and non-
infarction, ischemic necrosis • Can spread to orbit and brain
• caseating granulomas with giant
cells
• Pressure necrosis and proptosis

can occur
• Can spread to orbit and brain

MCC – Moulds – mucorales Rhizopus (R. arrhizus), MCC – Alternaria, Aspergillus, Bipolaris, A. flavus
Absidia, Rhizomucor – broad non-septate hyphae Curvularia, Exserohilum
with right angled branching. Ubiquitous in environment
Less common – all with septate nonpigmented
hypae.
Aspergillus - flavus, Aspergillus fumigatus
Fusarium and S. apiospermum
461
Contd...
Unit 8     Infectious Diseases

Acute invasive Chronic invasive Chronic granulomatous invasive


• Fever, unilateral facial swelling and headache,
• • History of nasal obstruction and
• Long standing nasal obstruction,
nasal obstruction/ pain, serosanguinous discharge sinunsitis unilateral facial discomfort, enlarging
• Necrotic black lesions on hard palate / nasal
• • Thick nasal polyposis and thick
• mass, silent proptosis
turbinate purulent mucus
• Periorbital/ perinasal swelling → ptosis, proptosis,
• • Orbital apex syndrome if spread

ophthalmoplegia, loss of vision to orbit from ethmoids – impaired


vision, ophthalmoplegia, proptosis,
cavernous sinus thrombosis
Imaging
Several sinuses involved unilaterally, no air fluid Hyperdense mass due to dense Opacification of ethmoid maxillary /
levels, thickening of sinus lining, destruction of bone accumulation of hyphae with all sinuses, erosion of bone
Cavernous sinus and cerebral involvement (better associated bone erosion
seen in MRI)
• Prompt diagnosis ( HPE – vascular invasion) and
• • HPE- to exclude vascular invasion
• • Surgical removal – complete

prompt correction of host factors • Extensive surgical debridement


• removal difficult
• Infected and necrotic material removed
• • Itraconazole 100 mg bd - more
• • Recurrence common

• Effective antifungal
• effective than voriconazole • Long-term itraconazole reduces

• Liposomal Amp- B(10-15 mg/kg/day)


• and posaconazole. (Long term recurrence
• Amp B (1-1.5 mg/kg/day)
• itraconazole reduces recurrence)
• Itraconazole only for susceptible fungi like

S. apiospermum that are resistant to Amp-B


• Posaconazole – vs Aspergillus and mucorales

• Other treatment options


• Hyperbaric )2, iron chelators, cytokines


• Diabetic patients – favorable prognosis on


• Prolonged duration and intracranial No studies
correction of DM extension – lower survival rates
• Other diseases- less favourable prognosis

y Uncommon chronic granulomatous infection affecting nasal


y

Paranasal Sinuses
mucosa, conjunctiva and other mucosa
y Three categories
y Large thick-walled sporangium (<350 micrometer diameter)
y

 Invasive fungal sinusitis


y

like structures with large number of endospores




 Noninvasive fungal sinusitis


y Member of protoctista mesomycetozoea based on 18s small


 Allergic fungal sinusitis


y

sub-unit ribosomal DNA




y South India and Sri Lanka


Invasive
y

y Stagnant pools of fresh water in rural areas


y

y 15–40 year males


y
y Three syndromes
y

 Acute fulminant (acute invasive)




Clinical Features  Chronic invasive




 Chronic granulomatous invasive / paranasal granuloma


y Nose in 70 % cases


y Needs radiographic and HP evidence in mucosa, submucosa


y

y Insidious onset → obstruction


y

or bone
y

y Papular or nodular smooth surfaced lesion that becomes


y

pedunclated and acquires proliferative or papillomatous


appearance Noninvasive Fungal Sinusitis/ Fungal Sinus Ball/
y Colour- pink, red or purple
y Sinus Mycetoma
y Large sessile / pedunculated lesion in one / both nostril
y

y Dense fungal hyphal mass in immunocompetent patients


y

y Keep enlarging unless treated.


y

y Can become invasive, if patient’s immunity decreases


y

y Causes
Treatment
y

 Most common cause aspergillus fumigatus,




y Surgical
y
 Less common- other aspergillus species, S. apiospermum,


y Recurrence common
y
Alternaria

462
Clinical Features y Unilateral nasal polyposis with thick yellow-green mucus

Chapter 68     Infections of Ear, Nose and Throat


y

y Pressure necrosis from mucus


y Long standing unilateral obstruction, purulent discharge,
y

y Proptosis if lamina papyracea breached.


y

facial pain, cacosmia


y

y Maxillary sinus >sphenoid> frontal


Investigations
y

CT Scan yy Endoscopy: Allergic mucin


y Mucus microscopy and HPE
y Partial/ total opacification, with flocculent calcification, bone
y

y CT scan
y

thickening and sclerosis.


y

y Lab: Eosinophilia, total IgE, specific IgE against fungi, fungal


y No air fluid levels (seen more commonly with bacterial
y

culture.
y

sinusitis)

HPE Management
y Surgery: Remove polyp and mucin-containing fungi
y Dense matted conglomeration of hyphae separate from but
y

y Postoperative oral steroids


y

adjacent to mucosa
y

y Adjunctive
y No e/o allergic mucin or granulomatous reaction in mucosa
y

y Allergen immunotherapy, nasal steroids, antihistamines, an-


y

y No invasion of mucosa/ blood vessels/ bone.


y

tileukotrienes and sinonasal saline lavage


y

y Antifungals not needed


Treatment
y

Recurrence common: Needing regular follow-up and repeat


y Surgical removal
y
surgery
y No local or systemic antifungals needed.
y

Mycotic Diseases of Throat


Allergic Fungal Sinusitis
Oropharyngeal Candidiasis
Noninvasive disorder in immunocompetent persons
y Criteria
y
Causes
 CT- chronic rhinosinusitis – serpiginous sinus opacification

y MCC – C. albicans
y

of >1 sinus, mucosal thickening and erosion of bone y C. glabrata, C. krusei, C. tropicalis, and C. parapsilosis
y

 Presence of allergic mucin with clusters of eosinophils




 Presence of noninvasive fungal elements in the mucin




Table 2:  Predisposing factors


 Type 1 hypersensitivity to cultured fungi


 Nasal polyposis
 General Local
 Fungal cultures: variable yield hence not used as criteria


for diagnosis Broad spectrum antibiotics, Unhygienic / ill-fitting


y Cause – ABCD
y
steroids, DM, nutritional deficiency, dentures
 A– Alternaria,

immunosuppressive disease HIV Tobacco smoking
 B– Bipolaris

(one of the earliest manifestation in
 C – Cladosporium, Curvularia

asymptomatic patients, candidiasis was
 D- Drechslera

most common opportunistic infection
before antiretrovirals)
Clinical Features Oropharyngeal (not life threatening but discomfort) → esopha-
y Young immunocompetent adults, in warm humid areas
y
geal (life threatening)
y h/o CRS, with H/o multiple surgeries
y

Table 3:  Clinical forms of oral candidiasis


Pseudomembranous Erythematous/atrophic Hyperplastic / hypertrophic
• Most common in HIV, extremes of age,
• • Associated with broad spectrum
• • Premalignant lesion

and immunosuppressed – malignancies antibiotics, chronic steroids, HIV • Small palpable translucent areas to large

• White raised lesions on tongue, palate,


• • Consequence of persistent
• opaque, white, dense hard and rough
buccal mucosa, tonsils pseudomembranous mass
• Can be wiped off to reveal eroded
• • Red flat lesion on dorsum of tongue or
• • Inner surface of cheek

erythematous bleeding base palate • Cant be wiped off unlike


• Painless in oral
• • Depapillated area on tongue
• pseudomembranous
• Painful if spreads to throat
• • premalignant
• • Usually asymptomatic

463
Other Forms of Candidiasis Viruses Causing Respiratory Illness
Unit 8     Infectious Diseases

y Chronic atropic denture candidiasis


y
y Pharyngitis: PIV, CMV, In-A and B, HSV, Coxsackie, enterovi-
y

y Laryngeal candidiasis: Can obstruct airway if untreated


y
rus
y Investigations
y
y Common cold – rhinovirus, corona virus and PIn-4
y

 Scraping and microscopy in oral and oropharyngeal



y Croup – In, Pin -1, 2, 3, RSV
y

 Fiberoptic laryngoscopy and biopsy for laryngeal



y Flu – In – A and B
y

Treatment Diagnosis
y Infants: nystatin oral suspension (100,000 units/mL) or Amp
y
Nasopharyngeal aspirates used instead of nose and throat swabs
B oral suspension (100 mg/mL) 6 hourly × 2 weeks due to better yield.
y Older children and adults- clotrimazole troche (like candy) –
y
y Traditional: Inoculation of secretions of tissue culture - time
y

10 mg troche 5 times a day, nystatin or Amp B oral suspension consuming


/ miconazole gel y Recent:
y

 Immunoassays and immunofluorescent labeled antibody


y HIV patients: Oral fluconazole (100–200 mg/ day – 1–2 weeks)


assay – rapid
y

or itraconazole oral solution ( 200–400 mg day for 1–2 weeks)


 PCR- sensitive and not needed for virus to be present.
y For recurrent and disabling cases (HIV, Malignancy) –


flucomazole long term


y Refractory: iv Amp B 0.3-0.5 mg / kg/day or Caspofungin – 50
y
Corona Virus
mg/day × 1 week y Largest human RNA virus
y

y Laryngeal candidiasis: iv Amp B 0.7-1.0 mg / kg/day, oral


y y Aerosol spread
y

fluconazole after recovery, ET intubation. y 2 human strains – 229E and OC4 – epithelium of trachea,
y

nasal mucosa, alveoli of lungs


Other Mycotic Infections of Pharynx and Larynx y 10 % of common cold esp in winter
y

y Nasal discharge, malaise and exacerbation of chronic


y Histoplasmosis and paracoccidiodomycosis
y

obstructive pulmonary disease (COPD)


y

y y Sore throat, dysphagia, dyspnea and cough y Lymphadenitis cough and fever less common
y

y y Ulcer/ granuloma on vocal cords/ laryngeal epithelium


y Biopsy and HPE with culture -to rule out malignancy
Adenovirus
y

Treatment y Double standard DNA virus originally isolated from human


y

y Itraconazole (better tolerated): 200–400 mg/day × 6–18


y
adenoid tissue.
months or ketoconazole 200–400 mg/day y 45 serotypes
y

y Children asymptomatically infected with types 1 and 2 in


 Delay in diagnosis → fibrosis → laryngeal stenosis –
y

early childhood and type 3 and 5 in later life




permanent speech and breathing difficulties.


y Spread, Aerosol, feco-oral
y

y Outbreaks of URTI and LRTI in small communities and insti-


Colonization of TE Prosthesis
y

tutions.
y Biofilm formation and invasion of the silastic by Candida sps.
y
y Swimming pool outbreaks of pharyngoconjunctival fever
y

C. glabrata, C. krusei, C. tropicalis. y Exudative tonsillitis with high fever pharyngitis and pneumo-
y

y Occur in 2-4 months of implantation – requires replacement nia.


y Each type specific features
y

y Radiation shortens this time


y

 Type 2 – milder
y

y Prevention


 Type 3 and 7 – severe disease – pneumonia in children.


y

 Buccal bioadhesive slow release miconazole nitrate tablet




 Metal coating of prosthesis


Rhinovirus


VIRAL CAUSES OF ENT INFECTIONS yySmall nonenveloped RNA virus, >100 serotypes
yyReplicates only in URT. Optimal temperature – 33°
y Aerosol and contact with fomites
Respiratory Tract Infections y

yySymptoms partly due to bradykinin release – nasal congestion,


y Most common disease of humans
y

sneezing, sore throat, headache and cough.


y 30% of all childhood deaths worldwide
y

y Infections throughout year, several serotypes.


y

y Seasonal activity: Parainfluenza type 1 and 2- autumn, 3 in


y

spring and RSV in winter


y Pathogenesis: Virus binds with host cell receptors, e.g.
y
Influenza
Rhinoviruses: ICAM-1 (intercellular cell adhesion molecule) y RNA orthomyxovirus
y

y Interferon response → systemic symptoms y Subtypes – A,B,C


y

464 y Droplets → incubate in 1–3 days


y

y Bradykinin → local symptoms


y
y
y Pandemics: Unpredictable y Diagnosis by indirect immunoflourscence of antigen and

Chapter 68     Infections of Ear, Nose and Throat


y y

y Symptoms: Fever, chills headache, myalgia, anorexia, malaise


y PCR of nasopharyngeal aspirates.
y Complications: Pneumonia (staphylococcal superinfection of
y

the damaged columnar epithelium with impaired mucociliary


clearance), myocarditis, pericarditis. Exacerbations of COPD, Herpes Simplex Virus
Asthma, CCF y Double standard DNA
y

y Complications in children: OM, croup, bronchiolitis, febrile fits


y y Types -1 and 2 – both cause primary infection and reactivation
y

y Complications more serious in: Elderly diabetics, immune


y y HSV 1 – oral secretions → 2 days – 2 weeks → sore throat,
y

compromised, liver, cardiac and renal disease pharyngitis, painful vesicles on oropharynx → extensive
y Avian H5N1: Virulent, infects humans, significant mortality,
y
gingivostomatis, extending to lips and cheeks
originates from wild and domestic birds in South-East Asia y Complications: Encephalitis, meningitis, erythema multi-
y

forme, eczema herpeticum, Bell’s palsy.


Mumps
y RNA virus
y
Recurrent Herpes
y Incubation period: 2–3 weeks
y

yy Itching, burning, tingling and pain lasting for 6 hours – 2 days.


y Transmitted via droplet and close contact
y

y Lesion on lips and perioral skin


Fever → parotid swelling → subsides
y

y y

y Multiple lesions on same site


y VII nerve swelling/orchitis (1 in 5 men) and atrophy of testis
y

y Some secondarily infected with S. aureus.


y

Sterility uncommon
y

y Complete healing <2 weeks


y Encephalitis and aseptic meningitis with residual deafness
y

y Systemic treatment: Famciclovir and valacyclovir preferred


y

y Live attenuated trivalent MMR vaccine very effective.


y

over acyclovir.
y

Measles Epstein-Barr Virus


y Incubation period of 2 weeks
y

yy DNA virus
y Children 3–6 years age
y

y Infects B cells and epithelial cells of oral and genital tracts


y Around 1 million child deaths in developing countries
y

y Infectious mononucleosis in young adults


y

y Virus replicates in respiratory and conjunctival epithelium →


y

y Sore throat, fever and disproportionate fatigue


y

reticuloendothelial cells
y

y Transient periorbital edema. Maculopapular rash in patients


y Symptoms- rhinorrhea, cough, fever, Kopliks spot on mucus
y

taking ampicillin
y

membrane of cheeks → rash after few days (face → trunk →


limbs), bronchitis and diarrhea yy 50% have dysphagia with tonsillar and pharyngeal edema
y Complications
y
yy Parenteral steroids might be needed for tonsillar edema
y Croup, otitis media, conjunctivitis, enteritis, febrile fits, rarely
y
yy Complications: Airway obstruction, splenic rupture, sec-
Subacute sclerosing pan encephalitis (SSPE). ondary bacterial pharyngitis, hematological problems, gul-
lian-barre syndrome
Respiratory Syncytial Virus (RSV) yy EBV implicated in nasopharyngeal carcinoma, oral hairy
leukoplakia (premalignant – painless white raised lesions on
y y RNA virus lateral side of tongue)
y y Two serotypes: A, B
y Winter outbreaks in infants and epidemics lasting for months
Human Papilloma Virus
y

y y Million deaths in developing countries


y y Highly contagious: Contacts with respiratory secretions (not yyDouble standard DNA papovavirus
aerosol) yyPredilection for human epithelial cells.
y y Incubation: 2–8 days yy60 types
y y Age mostly 6 weeks to 6 months yyType 6, 11 → maternal genital tract → Juvenile onset recurrent
y y Coryzal symptoms → ARDS in 24 hours → death respiratory papillomatosis
y y Necrosis of bronchiolar epithelium and lung parenchyma → y Respiratory Papillomas – MC laryngeal tumor
bronchiolitis → recurrent wheezing
y

y Usually single, pedunculated occurring in young adults and


y

y y Severe systemic symptoms due to greater interferon response under 5 age.


y y Vaccine not available → preventive measures crucial (limiting
aerosol, cohorting and hand wash) Table 4:  Papillomas and HPV types
y y Ribavarin in high-risk infants.
Tumor HPV type Less common
Parainfluenza Virus Oral Various
y y One-third of all RTIs Heck’s tumor 13, 32
y y 1 and 2 – autumn croup outbreaks every 1–2 years Multiple papillomatosis 6,11, 67
y 3- endemic – infantile bronchilolitis, pneumonia
Laryngeal pailllomatosis 16,11 6
y

y 4- less common both URI and LRI


Laryngeal CA 16,18 6,11, 30
y

y Exacerbation of COPD, asthma, cystic fibrosis


465
y

y y Transmitted: Person to person Inverted papilloma 6,11 18


Conditions with Viral Etiology y Sinusitis
Unit 8     Infectious Diseases

y Due to same bacteria as in normal population, and fungus


Bells Palsy
y

(aspergillus- most common) in low immunity


y y Acute, idiopathic, LMN type unilateral facial palsy y Broad spectrum antibiotics – augmentin and cephalosporins
y

y y 20 /100,000 per year. and decongestants.


y y Viral, autoimmune, ischemic causes
y y Treated with prednisolone 1 mg/kg body weight/ day Oral Cavity
Sensorineural Hearing Loss yyRecurrent aphthous ulcers: Larger than in normal pop
y Oral candidiasis
y Rubella, Mumps, Measles, Herpes zoster, CMV, Influenza
y

y Oral hairy leukoplakia


y

y Mumps and silent mumps – idiopathic sudden sensorineural


y

y Oral herpes simplex


y

hearing loss
y

yyGingivitis and periodontal disease


Postviral Olfactory Disorder yyPapillomas
y Xerostomia
y Caused by parinflueza -3
y

y Kaposis sarcoma- Most common HIV related oral malignancy;


y

treatment only if functional or cosmetic problem


Viral Acute Otitis Media y Non-Hodgkin lymphoma.
y

y Most common - Rhinovirus and RSV


y

HIV and Head and Neck Manifestations Pharynx and Larynx


y Extension from oral cavity – like candidiasis, HSV, kaposis
Otologic y

sarcoma.
External Ear
y Sebrrheic dermatitis, Kaposi sarcoma
y

Esophagus
y S. aureus and Pseudomonas aeruginosa infection
y Inflammation and ulceration
y

y Aspergillus infection
y

y Most common: Candidiasis followed by CMV, HSV and aph-


y

thous ulcer
Middle Ear
y Most common ear manifestations: Serous and recurrent
Salivary Glands
y

Otitis media
y S. aureus and Pseudomonas aeruginosa infection more com-
y y Xerostomia most common complaint
y

mon aprat from S. pneumonia, H. influenzae, M. catarrhalis y Lymphoepithelial cyst: Benign parotid swelling
y

y Occur due to nasopharyngeal neoplasm, adenoid hypertro-


y y Sialadenitis
y

phy, allergies.
y Myringotomy and tympanotstomy tube placed.
y

NECK
Inner Ear Neck mass: Tumor, infection, HIV lymphadenopathy, Parotid
disease
y SNHL, tinnitus, vertigo
y

y Due to HIV / drugs/ opportunistic infections – CMV, HSV, VZ,


y

Otosyphilis Tumor Kaposi, NHL, HL, SCC


Infections • Toxoplasmosis- protozoal infection
Facial Nerve Palsy

• Pneumocystis carinii causing lymphadenitis and


y Direct effect of HIV, neoplasm, opportunistic infections, AIDS


y
thyroiditis
encephalopathy, toxoplasmosis • M. tuberculosis, MAC.

• Fungal infection – cryptococcal neoformans


most common fungal infection


NOSE AND PNS
Nasal Obstruction
y Due to lymphoid tissue hypertrophy
y

466
69
Infections of Eye
CONJUNCTIVITIS
Definition: Inflammation of the conjunctiva is called as conjunctivitis:
The main etiological agents are discussed below.

Bacterial Conjunctivitis
Table 1: Different causes of bacterial conjunctivitis—pathogenesis, clinical features and treatment

Causes Pathogenesis Clinical features Treatment

Compromised epithelial Unilateral eyelid Broad spectrum


Acute Hyperacute Chronic
surface edema, topical
(mucopurulent) (Purulent)
Followed by enzymatic Conjunctival antibiotics.
• Staphylococcus • Neisseria • Staphylococcus degradation injection, Moxiflxacin -FDA



aureus gonorrheae aureus Mucopurulent approved
• Streptococcus • Neisseria discharge,


pneumoniae meningitidis With involvement

• Haemophilus (less severe of 2nd eye in 1–2

influenzae form) days

y Organisms penetrating intact conjunctival epithelium: y Maceration and ulceration of inner and outer canthus seen
y
y
 Neisseria gonorrheae in chronic blepharoconjuctivitis caused by Moraxella species

 Corynebacterium diphtheriae (angular conjunctivitis) → treatment → oxytetracycline eye

 Haemophilus aegypti (Koch-weeks bacillus) ointment

 Listeria monocytogenes y Lacrimal apparatus and intestinal flora (most common –

y
y H. inflenzae: (Gram-negative organism) often associated with Proteus and Klebsiella species) serves as reservoir of chronic
y
upper respiratory tract disease. conjunctivitis
y Conjunctivitis + associated membranous reaction – Strepto-  Chronic staphylococcal blepharoconjunctivitis lead
y

coccus viridans and Streptococcus pyogenes to marginal corneal ulceration by immune-mediated
y Hyperacute conjunctivitis more common in neonates hypersensitivity reaction which requires mild topical
y
sexually active adolescents and young adults. Characterized corticosteroids.
by copious thick yellowish green purulent discharge and
tender preauricular lymphadenopathy. Remember
Parinaud's oculoglandular syndrome- triad:
Treatment • Fever, follicular conjunctivitis and preauricular

lymphadenopathy.
y Mainstay of treatment: Topical antibiotics + systemic ceftri-
• Caused by Bartonella Henslae.
y
axone

+
Doxycycline/azithromycin (to treat concurrent Chlamydia Neonatal Conjunctivitis

infections) y Also known as ophthalmia neonatorum
y
y Occurs within 4 weeks after birth
y
Table 2:  Differential diagnosis for neonatal conjunctivitis
Unit 8     Infectious Diseases

Features Topical silver nitrate Neonatal gonococcal Herpes simplex Neonatal Chlamydial
ocular toxicity conjunctivitis virus HSV2 conjunctivitis
Onset Begins few hours after Hyperacute, within 24–48 hours 5–7days Begins 7–14days after
birth and resolves in after birth. birth
24–36 hours.
Treatment Wash eyes Single IM/IV ceftriaxone 125 mg Acyclovir 3%ointment Erythromycin or
Erythromycin ointment Hourly saline irrigation. tetracycline ointment
Resolves in 24 hours within 1 hour of birth

y Topical silver nitrate which was used as a prophylaxis against neonatal gonococcal infection is obsolete now
y

y Chlamydial conjunctivitis in neonates does not produce follicular reactions due to immature immune system unlike in adults,.
y

which is also the reason for increased propensity to form membrane on palpebral conjunctiva in neonates.
y HSV 2> HSV1 in causing neonatal herpetic conjunctivitis.
y

Viral Conjunctivitis
Adenoviral Conjunctivitis
Table 3:  Features of Adenoviral Conjunctivitis
Features Pharyngoconjunctival fever Epidemic Keratoconjunctivitis
Serotypes 3, 4, 7 8, 19
Incubation period 5–12 days (Avg -8 days) 2–14 days
Remains infectious for 10–14 days after onset of symptoms
Clinical features • Triad – pharyngitis+fever+ conjunctivitis
• • More severe form with severe corneal sequelae

• Conjunctiva – follicular rxn, mild watery discharge,


• • Conjunctiva- papillary and follicular response

hyperemic, chemosis • Subconjunctival hemorrhage and conjunctival


• Cornea-fine punctate erosions


• pseudomembrane
• Preauricular adenopathy
• • Corneal involvement severe but resolves without

neovascularisation or scarring
Treatment Cold compresses , artificial tears, resolves in 2 weeks Cold compresses, artificial tears, removal of membrane
with topical corticosteroids.

Other Viral Conjunctivitis


Table 4:  Features of various viral conjunctivitis
Acute haemorrhagic Herpes simplex conjunctivitis Varicella and herpes zoster conjunctivitis
conjunctivitis
Coxsackie A24 Herpes simplex virus Varicella zoster virus
Enterovirus 70
Severe painful papillary Acute follicular conjunctivitis+ preauricular Papules seen on eyelids, conjunctiva,limbus.
conjunctivitis, chemosis, adenopathy. Papules → pustules → ulcerates—resolves
subconjunctival haemorrhage Pseudomembrane. spontaneously.
Herpetic blisters differentiates it from Acute follicular conjunctivitis is rare
adenovirus

Molecular serotyping Rosebengal or fluorescein staining reveals


dendritic ulcer

y Acute hemorrhagic conjunctivitis also known as Apollo Disease.


y

y Treatment for all viral conjunctivitis is supportive care and prophylactic antibiotic ointment.
y

468
Chlamydial Conjunctivitis

Chapter 69     Infections of Eye


Trichiasis At least 1 eyelash rubs the ocular surface
y Chlamydia trachomatis serotypes A, B, C (C-commonest in
y
Opacity Easily visible corneal opacity in pupillary area
India) causing significant visual impairment
y Adult inclusion conjunctivitis caused by serotypes D-K
y

Conjunctival signs:
Mccallan’s Classification
• Follicles: 5 mm in diameter, rare on bulbar conjunctiva but

Stage 1 → Incipient trachoma with immature follicles
pathognomonic, if present. Stage 2 → Established trachoma with mature follicles and
• ARLT'S LINE: upper tarsus

progressive pannus
• HERBERT'S PIT: Upper tarsus

Stage 3 → 
Cicatrising/scarring especially of palpebral con-
junctiva
Corneal signs: Stage 4 → Healed trachoma or its sequelae
• Superficial punctate keratitis
Treatment

• Pannus

• Corneal ulceration

y Topical azithromycin and Systemic doxycline/ erythromycin.


y

SAFE stratergy
S—surgery for trichiasis/entropion Vernal Keratoconjunctivitis
A—antibiotics y Maxwell Lyon sign (ropy discharge)
F—facial cleanliness
y

y Cobble stone appearance of upper tarsus


E—environmental hygiene.
y

y y Horner trantas spots of limbal papillae


y y Gerontoxon
WHO Classification of Trachoma-FISTO y y Shield ulcer
y y Surgical treatment: Excision of papillae followed by amniotic
Follicular Active disease, predominant follicles at least ≥5 in membrane transplantation.
upper tarsal conjunctiva
Intense Inflammatory thickening of upper palpebral Phlyctenular Keratoconjunctivitis
conjunctiva obscuring half of normal deep tarsal
vessels y Type 4 hypersensitivity reaction
y

y Most common cause staphylococcus aureus and mycobacte-


y

Scarring Scaring in upper tarsal conjunctiva as white bands rium tuberculosis.


Contd...

KERATITIS
Definition: Inflammation of cornea produced by infectious or noninfectious agent or stimuli

Table 2:  Infections agents causing Keratitis

Bacterial Viral Fungal


Common organisms Staphylococcus aureus Herpes simplex Aspergillus fumigatus
Streptococcus pneumoniae Herpes zoster virus
Pathogenesis: Loss of intactness of corneal epithelium and bowman’s membrane by trauma or contact lens wear
Clinical features ••Photophobia HSV Keratitis: HZV keratitis: • Signs>symptoms

••Blepharospasm • Decreased corneal • Punctate • Mild watering


• Ciliary congestion • Fungal ulcer dry like


• •

sensation epithelial keratitis •

••Corneal ulcer (wet • Blepharoconjuctivitis • Micro/pseudo parchment paper


looking) • Wessely ring
• •

• Dendritic ulcer dendritic ulcers •

• Corneal rings (endotoxins • Satellite lesions


• Geographical ulcer • Nummular •

infiltrating stroma in • Hypopyon +


• •

• Stromal necrotic keratitis •

Gram negative infections) • is infected and fixed


Keratitis (superficial •

• Hypopyon+/-(sterile)

• Disciform keratitis
• stroma)
• Neurotrophic or

metaherpetic keratitis

Contd... 469
Bacterial Viral Fungal
Complications Long standing ulceration • Cranial nerve palsies (most common –3 nerve
rd
Rare


→ corneal perforation → palsy)
adherence of iris (leucoma • Optic neuritis


adherence) → pseudo • Post herpetic neuralgia


cornea formation
Treatment • Topical antibiotics • Topical acyclovir/valacyclovir/ famciclovir • Topical antifungal



• Cefazolin 5% and • Topical steroids(only in stromal Keratitis not in natamycin/fluconazole


tobramycin 1.3% epithelial) • Steroids contraindicated


• Subconjunctival

injections
• Topical cycloplegics

IMPORTANT POINTS
y Hypopyon in pneumococcal keratitis is → ulcus serpens High Yield
y
y Hutchison’s rule → involvement of nasociliary nerve (tip of
• Contact lens wearers Keratitis:
y
the nose) is generally associated with corneal involvement


(Herpes zoster ophthalmicus) ƒ Pseudomonas aeruginosa

ƒ
ƒ Acanthamoeba Keratitis
y Cogans syndrome → vertigo + tinnitus + hearing loss +

ƒ
• Pseudomonas Keratitis → greenish exudates, rapidly
y
recurrent interstitial Keratitis

progressive, can perforate in 48 hours
y Salmon patch cornea → congenital syphilis
• Acanthamoeba Keratitis → ring ulcer, radial keratoneuritis
y
y Fluorescein sodium stains absent areas of epithelium

(symptoms>>signs)
y
y Rose Bengal stains virus laden cells • Investigation: Culture on non-nutrient agar with overlay of
y
y Neurotrophic keratitis → 5th cranial nerve palsy

E. coli
y
y Exposure Keratitis. → 7th cranial nerve palsy • Cysts demonstrated on confocal corneal microscopy
y

y Superior limbic keratoconjunctivitis → most common in • Biguanides are the drug of choice. (Polyhexamethylene
y


hyperthyroidism biguanide).
Infection in Lower 70
Respiratory Tract—Pneumonia
PNEUMONIA y Atypical organisms are:

y
y Infection of lung parenchyma is called pneumonia  Mycoplasma pneumoniae


y
y It is classified in the past as:  Chlamydia pneumoniae


y
 Community-acquired pneumonia (CAP)  Legionella



 Hospital or ventilator acquired pneumonia (VAP)  Influenza viruses



y New term that is coined for multi-drug resistant pneumonia  Adenoviruses


y
named as health care-associated pneumonia (HCAP)  Human metapneumovirus


Conditions that are commonly encountered in HCAP with  Respiratory synctial virus


causative organisms:*
1. Hospitalization for ≥48 hours Remember

2. Hospitalization for ≥2 days in prior 3 months Special points to be known in CAP:

3. Nursing home or extended care facility residence • S. aureus pneumonia is the m/c cause for post-influenza


4. Antibiotic therapy in preceding 3 months infection (as a complication)

5. Chronic dialysis • When a history of aspiration is present, then anaerobes


6. Home infusion therapy have a chance for causal role in CAP

7. Home wound care • Anaerobic pneumonia usually complicates as abscess


8. Family member with MDR infection. formation and empyemas, parapneumonic effusion

• Newly identified pathogens are: Metapneumovirus, corona
*Conditions (serial numbers) Organisms commonly

virus, SARS, MERS, CA-MRSA
as mentioned above: encountered
1–4 Pseudomonas aeruginosa
1–4, 8 MDR Enterobacteriaceae
Risk Factors
y Alcoholism
1–8 MRSA
y
y Asthma
y
1–3 Acinetobacter spp., y Immunosuppression
y
y Institutionalization
y
y Age ≥70 years
Pathological events in Pneumonia
y
y Structural lung disease
y Initial phase in pneumonia is presence of edema
y
y H/o contact with birds, animals
y
y Then it goes for rapid transition to red hepatization phase
y
y Travel history to epidemic outbreaks
y
(name is because of presence of erythrocytes in the cellular
y
intra-alveolar exudate). Table 1: Etiological agent possible based on history

y Next phase is Gray hepatization phase where no new
y
erythrocytes can extravasate and those that are already Specific history Etiological agent possible
present in the intra-alveoli starts degrading. Exposure to bats or birds Histoplasma capsulatum
y Final phase is called phase of resolution—where the
Exposure to rabbits Francisella tularensis
y
macrophage reappears as the dominant cell type in the
alveolar space. Exposure to parrot or birds Chlamydia psittaci
Exposure to sheep and goats Coxiella burnetti
COMMUNITY ACQUIRED PNEUMONIA (CAP) Stay in AC hotels or Cruise Legionella sp
y Most common cause for community acquired pneumonia Cystic fibrosis patient Pseudomonas aeruginosa
y
(CAP) is S. pneumoniae
y Typical organisms are:
Diagnosis of CAP
y
 S. pneumoniae

 H. influenzae y The presence of an infiltrate on plain chest radiograph is

y
 S. aureus considered the gold standard for diagnosing pneumonia

 GNB when clinical and microbiologic features are supportive.

y Microbiological investigations include: Table 3:  Microbiological causes for ventilator associated
Unit 8     Infectious Diseases

 Gram stain

pneumonia
 Sputum culture and sensitivity


 Blood culture

Multidrug resistant Non-Multidrug resistant
y Sputum analysis has its own limitations: Expectorated
y
organisms organisms
sputum has its own commensals which may lead to different (LAMBP) (HAEMS)
isolation
y Hence expectorated sputum is allowed under only in ••Legionella pneumophila • • Haemophilus influenzae
• Aspergillus sp • Antimicrobial sensitive
y

following conditions according to 2007 IDSA/ATS consensus • •

guidelines : ••Acinetobacter sp • • Enterobacteriaceae


 Intensive care unit admission
••Antimicrobial resistant • • MRSA
Enterobacteriaceae • Streptococcus pneumoniae


 Failure of antibiotic therapy


• MRSA


 Cavitary lesions

• Burkholderia cepacia


 Active alcohol abuse


• Pseudomonas


 Severe obstructive or structural lung disease


aeruginosa


 Immunocompromised host


 Pleural effusion


 Epidemic pneumonia


y To differentiate whether it’s a commensal or pathogen in


y
Table 4:  Clinical criteria for diagnosis of VAP
sputum culture – following correlation should be made:
 Quantitation of growth (heavy, moderate or light)

CLINICAL PULMONARY INFECTION SCORE (CPIS)
 Clinical correlation

Criteria Score
 Correlation with the Gram stain
Fever ≥ 38.5 but ≤38.9 1


y To differentiate the viral and bacterial cause of pneumonia


y

– the ideal marker is Procalcitonin: Fever >39 but <36 2


 Procalcitonin is a peptide precursor of calcitonin that is
Leukocytosis <4000 or >11,000/μl 1


released by parenchymal cells in response to bacterial


toxins, leading to elevated serum levels in patients with Bands >50% 1 (additional)
bacterial infections
 In contrast, procalcitonin is downregulated in patients
Oxygenation (mm Hg) 2
Pao2 / Fl o2 < 250 and no ARDS


with viral infections


 Procalcitonin is detectable in the serum within 4 hours,
 Chest radiograph:
and peak levels occur between 12 and 48 hours. • Localized infiltrate
• 2
• Patchy or diffuse infiltrate
• 1
Table 2:  Empirical management of CAP • Progression of infiltrate
• 2
Tracheal aspiratre
Category Treatment drugs
• Moderate or heavy growth
• 1
Outpatient Macrolides • Same morphology on Gram's stain
• 1 (additional)
Fluoroquinolones
Maximal score 12
Beta lactams
Inpatient – Non-ICU Fluoroquinolones
Table 5:  Empirical management of VAP
Beta lactams
Inpatient - ICU Beta lactams Category Treatment drug
Patients without risk fators for Ceftriaxone
For Pseudomonas - Add anti-pseudomonal beta lactam MDR pathogens Moxifloxacin
pneumonia Ampicillin/Sulbactam
For CA MRSA Add linezolid or vancomycin Ertapenam

Patients with risk factors for Beta lactam


MDR pathogens + gentamicin/FQs (Gram
HEALTH CARE-ASSOCIATED PNEUMONIA negative)
y Health care-associated pneumonia (HCAP) becomes a bridge
y

+ linezolid/vancomycin (Gram
between classic CAP and typical HAP positive)
y MDR pathogens form the main stay for HCAP
y

y The most common entity in HAP is VAP – Ventilator-associ-


y

ated pneumonia

472
71
Gastrointestinal Infections
ACUTE DIARRHEAL DISEASES y These organisms bring about noninflammatory diarrhea

y
which is mostly by enterotoxin production
y As defined by WHO, Diarrhea is the passage of unusually
y Site of action: small bowel (proximal part)
y
loose or watery stools, usually at least three times in a period

y
y Manifestation: Acute watery diarrhea
of 24 hours

y
y Stool examination: No evidence of leucocytes and fecal
y It can be classified as acute, chronic or persistent depending

y
leucoferrin not elevated
y
on whether it was present for few hours or days
y Diarrhea is caused by a variety of microorganisms with the
Inflammatory Agents
y
most common being the viruses (50–60 %) followed by
bacteria (20 – 30 %), then parasites (10–20 %).
y The severity of diarrhea is more in children when compared Table 2:  Inflammatory diarrhea
y
to adults mainly in developing countries like India
y Diarrheal disease is the second most common cause of Diarrheal Bacteria Viruses
y
mortality and morbidity in children less than five years of age mechanism
in developing countries next to respiratory illness.
Inflammatory EHEC, EIEC Nil Entamoeba
y The term Dysentery is defined by the passage of blood or
diarrhea histolytica
y
mucous in the stools.
(predominantly
y The term Gastroenteritis is defined as inflammation of Shigella species Balantidium
dysentery)
y
mucosa of stomach and intestine along with diarrhea or coli
dysentery
Campylobacter
CLASSIFICATION OF AGENTS CAUSING jejuni

DIARRHEA Vibrio
parahaemolyticus
y Based on the mechanism by which a particular organism is
y
causing diarrhea they have been classified as those causing Inflammatory Salmonella species
non-inflammatory diarrhea, inflammatory diarrhea diarrhea
Yersinia
and those causing diarrhea by invasive or penetrating (predominantly
enterocolitica
mechanism inflammatory
y Agents responsible for causing acute diarrhea. diarrhea) Clostridium difficile
y
Non-inflammatory Agents Listeria Nil Nil
monocytogenes
Table 1:  Non–inflammatory diarrhea
Plesiomonas
Diarrheal Bacteria Viruses Parasites shigelloides
mechanism
Aeromonas
Non- • Vibrio cholerae Rotavirus Cryptospori-
hydrophila

Inflammatory dium sp
diarrhea • ETEC, EAEC, Norovirus Cyclospora sp Klebsiella oxytoca

EPEC
• Clostridium Adenovirus Microsporidia y Mechanism of action: Cytotoxin production or invasion
y

perfringens (enteric) y Site of action: Distal small bowel or colon
y
• Staphylococcus Giardia y Manifestation: Inflammatory diarrhea or dysentery
y
Stool examination: Fecal leucocytes- elevated, fecal lactofer-

aureus lamblia y
y
• Bacillus cereus rin levels- elevated

• Plesiomonas

shigelloides
• Aeromonas

hydrophila
Table 3:  Diarrhea by penetrating mechanism
Unit 8     Infectious Diseases

Incubation period Organisms


Diarrheal Bacteria Viruses Parasites Vibrio parahaemolyticus
mechanism Campylobacter jejuni
Penetration Salmonella typhi Nil nil Salmonella species
(enteric fever)
Shigella species
Yersinia
enterocolitica
(typhoid like
Table 5:  Classification based on toxin agents
features) Toxin Agents
y y Mechanism of action: Penetration ENTEROTOXINS Vibrio parahaemolyticus, Vibrio cholerae
y y Site of action: Distal small bowel ETEC (LT and ST of ETEC)
y y Manifestation: Fever , vomiting and diarrhea (enteric fever ) EAEC (EAST of EAEC)
y Stool examination: Fecal leucocytes elevated
EHEC (VT of EHEC)
y

Table 4:  Classification based on incubation period Aeromonas species


Clostridium difficile (toxin A)
Incubation period Organisms
Rotavirus (NSP4)
6 hours Staphylococcus aureus
Campylobacter jejuni
Bacillus cereus
CYTOTOXINS Shigella dysenteriae (type 1)
Clostridium botulinum
EHEC
8 -16 hours Clostridium perfringens
Clostridium difficile (toxin B)
Bacilus cereus
NEUROTOXINS Staphylococcus aureus (enterotoxin)
More than 16 hours ETEC, EHEC
Bacillus cereus toxin
Vibrio cholera
Clostridium botulinum toxin
Contd...

y Diarrhea can also result due to food poisoning or can be associated with travel.
y

 Food poisoning result due to contaminated food or drink intake. The bacterial causes of food poisoning are listed below. Some


nonbacterial causes are capsaicin , a substance present in hot peppers and toxins found in sea foods like fish and shell fish.
 Travellers diarrhea occurs mainly among people traveling from temperate to tropical countries. The agents causing them are


listed below
y Based on toxin produced the Diarrheal agents are classified as given below
y

Table 6:  Agents causing food poisoning and travellers diarrhea

Agents causing food poisoning Agents causing travellers diarrhea


Staphylococcus aureus –Ham, poultry, pastries Bacteria: ETEC
Bacillus cereus- Fried rice EAEC
Clostridium botulinum- canned food items Campylobacter species
Clostridium perfringens- Beef, legumes, gravies Shigella species
Bacillus cereus- Meat, vegetables and dried beans, cereal Salmonella species
ETEC – Salads, cheese, meat and water Viruses Norovirus
EHEC – Salami, ground beef, raw milk and apple juice Rotavirus
Vibrio cholera: Shellfish and water Parasites Giardia lamblia
Vibrio parahaemolyticus: Crustaceans, molluscs Cryptosporidium
Campylobacteria jejuni: Poultry, raw milk Entamoeba histolytica
Salmonella species: Beef, poultry, eggs and dairy Cyclospora
474
Shigella species: Potato, egg salad,lettuce
Table 7:  Incidence and pathogenesis of Diarrheal agents:

Chapter 71     Gastrointestinal infections


Agents Incidence Pathogenesis Comments
Viruses
Rotavirus Responsible for one million out Site: proximal small intestine Infects all ages
of the total 3 to 5 billion cases of Gastric and colonic mucosa not affected Most common cause of diarrhea
diarrhea among children NSP 4 → functions as enterotoxin → alters among children
the epithelial cell function and permeability
→ damaged cells slough out and shed in
intestinal lumen
Secretory diarrhea

Norovirus Responsible for 95 % of viral Causes changes in the epithelial cells of Most common cause for
outbreaks of diarrhea intestine without causing shedding of the epidemic viral gastroenteritis in
Genotype II of norovirus is cells adults
responsible for most cases Blunting of the microvilli noted
Secretory diarrhea
Adenovirus Adenovirus serotypes 40 and 41 Atrophy of villi and compensatory Second most common viral
cause gastroenteritis hyperplasia of crypts → malabsorption of agent causing endemic diarrhea
Responsible for 2–12 % of fluids and nutrients → secretory diarrhea in infants and young children
diarrhea in children
Bacteria
Vibrio cholera Annual report of cholera cases – Cholera toxin (enterotoxin) → binds its Current most outbreaks and
3 million per year fragment B ro GM1 ganglioside receptors epidemics of diarrhea are due to
→ fragment A causes ADP ribosylation of Eltor biotype
G protein → upregulates adenyl cyclase 0139 causes minority of
→ increases cAMP → inhibits absorptive infections
mechanism → secretory diarrhea
EAEC Emerging as important cause Adheres to intestine → release of Persistent diarrhea in children
of sporadic diarrhea in healthy enterotoxins and cytotoxins → Travelers diarrhea
children and adults in the US inflammatory diarrhea
ETEC Leading cause of travelers' By the action of the labile and Stable toxin LT ETEC is more difficult to
diarrhea and ST respectively recognize and therefore is often
Major cause of diarrheal disease LT--) MOA similar to cholera toxin discussed not appreciated as being a
in lower-income countries, earlier major cause
especially among children. ST → binds to guanyl cyclase → increased
cyclic GMP → fluid accumulation in gut
EHEC O157:H7 (Most common Verocytotoxin or shiga like toxin → binds Self-limiting, but it may lead to
serotype) responsible for tp Gb3 receptor on intestinal epithelium → a life-threatening disease like
outbreaks and sporadic infections inhibits protein synthesis hemolytic uremic syndrome
attributed to EHEC worldwide (HUS), especially in young
children and the elderly
EIEC Only pathotype of E coli which Similar to Shigella Infection similar to shigellosis
affects the colon with fever and dysentery
EPEC Prevalence 8–10 % of Diarrheal Nontoxigenic and non invasive Infantile diarrhea outbreaks and
infections Adhesion to intestine and forms cup like sporadic diarrhea in adults
projections (pedestals) → produces A/E
lesions → disruption of brush border
epithelium → secretory diarrhea
Campylobacter Hyperendemic in developing Heat labile enterotoxin similar to cholera Contaminated poultry in the
jejuni countries toxin most important cause
Exists as asymptomatic infection
mostly except in children less
than 2 years of age
475
Contd...
Unit 8     Infectious Diseases

Agents Incidence Pathogenesis Comments


Clostridium Mainly causes food poisoning Type A enterotoxin → forms pores in Mainly due to consumption of
perferingenes intestinal cells → damage to intestinal cells improperly cooked meat
→ malabsorption → diarrhea
Clostridium Clostridium difficile infection Toxin mediated- toxin A and B Associated with prolonged
difficile (CDI)– 20 to 25% of all infections Toxin A: enterotoxin, toxin B- cytotoxin antibiotic usage
Diarrhea is the most common Toxins → glycosylate the GTP binding Causes AAD (antibiotic
manifestation of CDI proteins → disruption of cellular associated diarrhea) or CDAD
cytoskeleton → loss of shape and epithelial (clostridium difficile associated
cell barrier → diarrhea and pseudo diarrhea)
membranous colitis
Bacillus cereus Mainly causes food poisoning Produced 2 toxins- Diarrheal toxin and Vegetables, cereals, milk,
emetic toxin meat and poultry are common
Emetic toxin resembles enterotoxin of sources
S. aureus
Diarrheal toxin penetrates the intestine →
causes diarrhea
Staphylococcus Mainly causes food poisoning Enterotoxin → stimulates vagus nerve and Milk products, bakery foods,
aureus vomiting centre in brain and peristaltic potatosalads- common sources
action of intestine → diarrhea Type A is the most common
enterotoxin involved
Aeromonas The rates of fecal carriage in the Enterotoxins and various extracellular Commonly spread through
species absence of disease - 0% to 4% enzymes secreted by the bacteria → drinking water
while the isolation rate from damages the intestinal cells → leads to
persons with diarrheal illness is diarrhea
from 0.8 to 7.4% .
Plesiomonas Important new enteric pathogen Mechanism not fully known. Secretory or Waterborne and oyster related
shigelloides invasive diarrhea outbreaks are found to occur
Yersinia Zoonosis Produces an enterotoxin which is similar to Self-limiting gasteroenteritis
enterocolitica Found worldwide esp Europe and the stable toxin of Escherichia coli
America
Listeria Prevalence varies fron 2 to 25% Enters intestinal cells with help of internalin Acute, self limiting febrile
monocytogenes Asymptomatic carriage rate 1 to proteins → inhibits phagolysosome and gasteroenteritis
5% survives inside host cell → spread from
one cell to another with the help of actin
filament formation
Salmonella Zoonosis Entry of bacilli through the M cells in Gastroenteritis is common with
National health problem mucosa → membrane ruffle formation → nontyphoidal salmonella (NTS)
0.1% become carriers and spread Bacterial mediated endocytosis → reach the
the infection submucosa and release toxins → infected
cells release proinflammatory cytokines
→ ulcer and damage to intestine →
inflammatory diarrhea
Shigella S. flexneri: Most prevalent in Entry of bacitti through m-cells in mucosa → Low infectious dose – 10 to 100
developing countries engulfed by macrophages in submucosa and bacilli
S. sonnei: Most prevalent in released → cytokines secreted by affected
developed countries intestinal cells → attracts inflammatory cells
S.dysenterie: Mostly related to → acute colitis and inflammatory diarrhea
high mortlity
Parasite
Entamoeba Third most common parasitic Trophozoites (invasive form) → invade colon Major health problem world
histolytica cause of death in world → ulcerative lesions → failure of absorption wide
Approximately 50 million cases → diarrhea
476 annually
Contd...
Chapter 71     Gastrointestinal infections
Agents Incidence Pathogenesis Comments
Balantidium coli Common worldwide in tropical Site : large intestine Rare in India
and subtropical areas Trophozoites → Invade the mucosa and
Highest prevalence: Indonesia cause ulcers → inhibits absorption →
secretory diarrhea
Giardia lamblia Prevalence : 2.67 to 32% Trophozoits adhere to mucosa → disruption Most common parasite causing
Genotype B is the most common of brush border → malabsorption → endemic and epidemic diarrhea
diarrhea

Rarely enterotoxin CRP 136 plays role .

Cryptosporidium Zoonotic disease CP 47 protein of sporozoite → attaches to Self-limiting illness in


Prevalence: 2.4 to 15% among small intestine → penetration → forms immunocompetent
immune competent and a vacuole inside the cells → activates Opportunistic pathogen
12 – 46% among kinase pathway → releases cytokines → and severe illness in
immunocompromised liberate soluble factors from phagocytes → immunocompromised
secretory diarrhea
Cyclospora spp Prevalent in central America and Similar to Cryptosporidium Cyclospora cayetenensis is
South Asia the most recently reognized
Most cases from Haiti and Nepal coccidian parasite as human
intestinal pathogen

LABORATORY DIAGNOSIS  Hanging drop preparation from the liquid part of stool
sample– done to identify Vibrio cholerae by its darting
Specimen Collection motility from cases of acute gastroenteritis.
y Freshly collected ideally before the start of antibiotics and
y
 Confirmation of Vibrio cholerae can be performed by


sent to laboratory adding a drop of anti-H antiserum, which inhibits the


y Sterile, leak proof, screw capped, wide mouthed container
y
motility shown by Vibrio cholerae
should be used for collection yy Gram staining of the stool sample: not routinely done for stool
y Feces with mucus flakes should be collected
y
samples due the presence of normal flora in feces. But it is
y In case of pediatric patients and nonambulatory individuals-
y
useful in few situations where the morphology of the bacteria
rectal swabs can be obtained gives some clue to the preliminary identification
y In case of food poisoning, food samples, stool and vomitus
y
 E.g.: comma-shaped bacteria- Vibrio spp,


can be collected.  Gram-positive rods in sheets- Clostridium difficile




 Budding oval cells- Candida species




y Acid fast staining: Oocyst of Cyclospora, Cryptosporidium


Transport of the Sample
y

and Isospora are acid fast in nature


y Transport medias can be used if any delay is expected in
y
yy Trichrome staining for the parasites can be done to identify
bringing the sample to the laboratory the internal structures.
y The most commonly used ones are
y
yy Electron microscopy: for the detection of viruses based on
 Cary blair medium – for all enteric pathogens

morphology
 Buffered glycerol saline medium for shigella

 E.g: Rotavirus- spokes around the wheel appearance


 VR medium and Autoclaved sea water for Vibrio choleare



 Adenovirus: Space vehicle shaped


 Astrovirus- star-like appearance




Stool Examination
y Macroscopic examination:
y
Bacterial Culture
 Color, consistency and odor of the stool sample should be

y The stool samples should be plated on two solid media one
y

noted. For examples being highly selective and other moderately selective and
Š Rice watery consistency: vibrio cholera
Š
also in a liquid medium for enrichment
Š Blood and mucous in stool: dysentery
Š
y Moderately selective media: MacConkey medium
y

Š Sweetish offensive odor: cholera


Š
y Highly selective medium: XLD medium (Xylose lysine deoxy
y

y Direct microscopic examination:


y
cholate) or DCA (Deoxycholate agar) or TCBS medium
 Saline or iodine wet mount of the stool sample – to look for

(thiosulphate citrate bile salt sucrose) can be used
pus cells, RBCs, budding yeast cells, ova or cyst or larvae y Enrichment medium: Selenite F broth or alkaline peptone
477
y

of parasites water can be used


y The colonies suspected to be enteric pathogens can be y ELISA: for the detection of VP6 capsid antigen of Rotavirus are
Unit 8     Infectious Diseases

y y

confirmed using biochemical tests available


y Serogrouping or Serotyping is later performed with
y

corresponding antisera
y Antimicrobial susceptibility testing is further performed with
y
Toxin Detection Methods
the colonies inorder to choose the drug for the treatment. y Various ELISAs are available for detection of toxins
y

y The gene responsible for toxin production can also be


y

identified using molecular methods such as PCR


Tissue Culture
y Tissue cuture is mainly performed for the identification of
y

enteric viruses. Sometimes Diarrhoegenic Escherichia coli


Molecular Methods
strains can be identified by the effects produced in cell lines y For identification of organisms like diarrhoegenic Escherichia
y

 EAEC: stacked brick pattern of adherence in HeLa or Hep2



coli whose pathotype differentiation cannot be performe with
cell line biochemical tests,molecular tests are used immensely
 EPEC: produces attaching and effacing lesions (A/E) in

y The various virulence genes ike genes that code for invasion
y

HeLa or Hep 2 and toxin production are targeted and organisms are
 EHEC: produced verocytotoxin, which is toxic or cytolyic

identified
to Vero cells  E.g.: lt and st gene coding for LT and ST of ETEC


 Eae gene- coding for EAEC




 Bfp gene – coding for EPEC


Serological Methods: for Antigen Detection


y Latex agglutination tests: Available for diagnosis of Crypto-


y

sporidium, Giardia and Entamoeba histolytica

478
Cardiovascular Infection– 72
Endocarditis
INFECTIVE ENDOCARDITIS Table 1: Most common agent of various types of


y Vegetative type of lesions that cause inflammation lead to endocarditis
y
endocarditis. Classification of Most common agent associated
y These vegetations are a combination of platelets, fibrin, endocarditis
y
microcolonies of organisms and inflammatory cells.
y The process usually affects cardiac valves.
Native valve Staphylococcus aureus
y
endocarditis
Classification of Endocarditis
y Acute endocarditis Prosthetic valve • Early prosthetic valve endocarditis
y

y Subacute endocarditis endocarditis (within 12 months of valve
y
y Native valve endocarditis replacement): S.epidermidis
y
y Prosthetic valve endocarditis • Late prosthetic valve endocarditis
y

(After 12 months of valve
Risk Factors for IE replacement): Viridans Streptococci
y Age > 60 years
y
y Male sex Endocarditis in IV drug Staphylococcus aureus
y
y Poor dental hygiene abusers
y
y IV drug users
y
y Comorbid cardiac valvular diseases Subacute endocarditis Viridans Streptococci
y
y Previous history of IE
y
y Chronic hemodialysis
y
Clinical Manifestations
Etiological Agents Causing IE y Fever is the most common symptom of IE; other manifestations
y
y Staphylococcus aureus  are chills, anorexia, and weight loss.
y
y Viridans group streptococci y Other common symptoms of IE include malaise, headache,
y
y Enterococci
y
myalgias, arthralgias, night sweats, abdominal pain, dyspnea,
y
y Coagulase-negative staphylococci cough, and pleuritic pain.
y
y Streptococcus bovis 
y Cardiac murmurs
y
y Other streptococci (including nutritionally variant strepto-
y
y Splenomegaly
y
cocci)
y
y Cutaneous manifestations such as petechiae or splinter
y Non-HACEK gram-negative bacteria
y
hemorrhages.
y
y Fungi
y Janeway lesions: Nontender erythematous macules on the
y
y HACEK
y
palms and soles
y
y Osler nodes: Tender subcutaneous violaceous nodules mostly
Remember
y
on the pads of the fingers and toes, which may also occur on
Infective endocarditis the thenar and hypothenar eminences
• Most common cause for native valve endocarditis is
S. aureus y Roth spots – Exudative, edematous hemorrhagic lesions of

y
• Most common cause in IV drug user in S. aureus – the retina with pale centers

Tricuspid valve endocarditis y Complications can arise like septic emboli, metastasis and
• Left sided endocarditis in drug users – Pseudomonas and
y
neurological complications (stroke)

Candida
• Culture negative endocarditis is caused by:

� HACEK � Nutritionally variant


Streptococci
� Coxiella burnetiti � Abiotrophia sp
� Tropheryma whipplei


� Bartonella sp


Diagnosis Bartonella spp
Unit 8     Infectious Diseases



 Chlamydia spp
Accepted criteria for diagnosis of IE is the modified Duke criteria


 Legionella spp
Table 2:  Modified Duke criteria for IE  Mycoplasma spp
 Brucella spp
Major Criteria Minor Criteria


• Positive blood culture: • Predisposing cardiac


Microorganism isolated should


conditions or IV drug
Pathological Criteria
correlate with etiological users y Presence of microorganisms (demonstrated by culture or
y

agent for IE (or) persistently • Fever ≥ 38°C



histology of a vegetation or intracardiac abscess)
positive blood culture i.e. blood • Vascular manifestations

y Histology (vegetation or intracardiac abscess demonstrating
y

cultures drawn >12 hours like emboli, aneurysms, active endocarditis)


apart have same organism (or) infarct, hemorrhages
single positive blood culture of • Immunological

Echocardiography
Coxiella burnetii (or) positive manifestations like
y Echocardiography is mandatory in all patients with suspected
IgG antibody titer of 1:800 glomerulonephritis,
y

IE as soon as possible after the diagnosis of IE is suspected


• Evidence of endocardial Osler's nodes, Roth's
y Positive findings for IE include the setting of vegetation,

involvement: Positive spots, Rheumatoid factor


y

abscess, or new dehiscence of a prosthetic valve


Echocardiogram with oscillating • Microbiological evidence
y In general, TTE (Transthoracic echocardiography) is the first

intracardiac mass or abscess like positive blood culture


y

diagnostic test for patients with suspected IE. TEE (Trans


of new partial dehiscence of or serological evidences
Esophageal Echocardiography) has a sensitivity of >90% for
prosthetic valve or new valvular
detection of valvular vegetation and is superior to TTE for
regurgitation
detection of cardiac complications such as abscess, leaflet
Interpretation of Criteria: perforation, and pseudoaneurysm
• 2 major
Definite endocarditis

• 1 major + 3 minor

Table 3 :  Antibiotic treatment based on organisms


• 5 minor

Organisms Drug of choice


Blood Cultures Penicillin susceptible Penicillin/Ceftriaxone/
y Positive blood culture is the mainstay of microbiological Streptococci Vancomycin/
+ Gentamycin
y

diagnosis of IE
y Three sets of blood cultures detect 96 to 98% of bacteremia;
y

Penicillin resistant Streptococci Vancomycin


Minimum of 20 mL is needed for culture from adults for best
chance of isolation.
y Blood culture results should be interpreted based on the
y
Enterococci Penicillin + Gentamycin (or)
modified Duke criteria. Ampicillin + Gentamycin (or)
y Most clinically significant bacteremias are detected within 48
y
Vancomycin + Gentamycin (or)
hours Ampicillin + Ceftriaxone (or)
y Most of the pathogens are detected within five days of
y

MSSA infecting native valves Nafcillin/Flucloxacillin/oxacillin


incubation with modern automated blood culture detection (or)
systems. Cefazolin
y In patients who have received prior antibiotics, additional
y

blood cultures may be useful (two to three sets over several MSSA infecting prosthetic Nafcillin/Flucloxacillin/oxacillin
days). valves + Gentamycin + Rifampin
MRSA infecting native valves Vancomycin
CULTURE-NEGATIVE ENDOCARDITIS MRSA infecting prosthetic Vancomycin + Gentamycin +
y Culture-negative endocarditis is defined as endocarditis with
y valves Rifampin
no definitive microbiologic etiology following inoculation
HACEK organisms Ceftriaxone + Ampicillin/
of at least three independently obtained blood samples in a
Sulbactam
standard blood-culture system, with negative cultures after
five days of incubation and subculturing. Coxiella burnetii Doxycycline +
y Culture-negative IE should also be suspected in patients with
y
Hydroxychloroquine
vegetation on echocardiogram and no clear microbiologic Bartonella sp Ceftriaxone or Doxycycline +
diagnosis. Gentamycin
y Organisms that are difficult to culture are responsible for IE in
y

such conditions.
480  Coxiella burnetii

Emergency Interventions

Chapter 72     Cardiovascular Infection—Endocarditis


High Yield
y Emergency cardiac interventions are done in patients with
y

endocarditis who has strong evidence of following findings: Patient with following risk factors needs prophylaxis before
 Acute aortic regurgitation + preclosure of mitral valve

dental procedures:
 Sinus of Valsalva abscess ruptured into right heart

• Prosthetic cardiac valves

 Rupture into pericardial sac



• Previous history of endocarditis

Indications for Surgical Treatment are: • Cyanotic congenital heart disease (CHD)

y Congestive cardiac failure


y
• Completely repaired congenital heart defects (within 6

y Perivalvular infection
y
months)
y Uncontrolled infection
y
• CHD not repaired properly with residual defects

y S. aureus endocarditis
y
• Valvulopathy after cardiac transplantation

y Systemic emboli risk


y
Antimicrobial prophylaxis for above said condition:
y Cardiovascular implantable electronic devices (CIED)
y
• Amoxicillin 2 g PO 1 hour before the procedure

endocarditis • Penicillin allergic patients can take clarithromycin/


azithromycin, cephalexin, clindamycin

481
73
CNS Infections
ACUTE BACTERIAL MENINGITIS Table 1:  Causative organisms for meningitis
y Acute purulent infection within subarachnoid space associat- Predisposing Factors Organisms
y
ed with an inflammatory reaction for Meningitis
y Organisms that are responsible for community acquired
Endocarditis • Streptococcus viridans
y
bacterial meningitis are:


• S. aureus
 Streptococcus pneumoniae (most common)


• Streptococcus bovis

 Neisseria meningitidis


• HACEK

 Group B Streptococcus


• Enterococcus sp

 Listeria monocytogenes



 Haemophilus influenzae B Otitis media • Streptococcus species



Mastoiditis • Gram negative anaerobies


Sinusitis • S. aureus
Predisposing Factors


• H. influenzae


y Meningitis caused by Streptococcus pneumoniae: • Enterobacteriaceae
y

 Pneumococcal pneumonia – most commonly
Invasive neurological • S. aureus

 Acute/chronic sinusitis or otitis media


procedures like shunts or • CONS

 Immunodeficiency disorders like hypogammaglobulin-


craniotomy

emia and complement deficiencies
 Alcoholism
Clinical Manifestations

 Diabetes mellitus

 Basilar skull fracture or CSF rhinorrhea y Acute or subacute presentation are seen

y Neisseria meningitidis
y
y Classical triad is:
y
 Serogroup vaccines A, C,W135, Y has reduced the inci-
y
 Fever

dence

 Headache

 Serogroup B is responsible for 1/3rd of cases  Nuchal rigidity


 Asymptomatic nasopharyngeal carriers y Decreasing level of consciousness >75%

y
 Host factor/bacterial virulence y Nausea, vomiting, photophobia

y
 Complement deficiencies like properdin deficiency y Seizures in 20-40%

y Gram-negative bacilli:
y
y Increased intracranial tension is the major cause for obtun-
y
y
 Chronic debilitating diseases dation and coma

 Diabetes mellitus y Because of increased ICT – the most disastrous complication

y
 Cirrhosis is cerebral herniation

 Chronic URI y In meningococcemia – erythematous maculopapular rash is

y
 Alcoholism seen – petechiae found in trunk and lower extremities.

 Head trauma y Mechanisms of neurological complications:

y Group B streptococcus:
y
 Altered blood brain barrier permeability leads to vasogenic
y

 Predominantly seen in infants and frequency more seen in edema

individuals of age >50 years  Inflammatory cytokines leads to release of toxic materials
y Listeria monocytogenes:

 Exudates in subarachnoid space surrounds and infiltrate
y
 Important cause of meningitis in neonates

vasculature leading to cerebral ischemia causing

 Pregnant females obstruction and communicative hydrocephalus

 > 50 years  Loss of autoregulation leading to reduced blood flow


 Immunocompromised  Reactive oxygen species formation leads to cell injury and


 Exposure to contaminated food like milk, cheeses death

y Haemophilus influenzae type B:  All these finally lead to increased intracranial tension and
y

 Unvaccinated children and adults coma

 Non B H. influenzae is an emerging pathogen

Treatment

Chapter 73     CNS Infections


y Empirical antimicrobial treatment should be started with
y

third generation cephalosporin and vancomycin as ABM is a


medical emergency

Table 2:  Drug of choice according to causative organism

Organism Drug of choice


Penicillin sensitive Penicillin G
N.meningitidis
Penicillin resistant Ceftriaxone
N.meningitidis
Streptococcus pneumoniae Penicillin G
Streptococcus agalactiae Penicillin G
Gram-negative bacilli Ceftriaxone
Pseudomonas spp Ceftazidime or meropenem
MSSA Nafcillin
MRSA Vancomycin
Haemophilus influenzae Ceftriaxone
Listeria monocytogenes Ampicillin + Gentamycin

VIRAL ENCEPHALITIS
Definition
The infectious process and associated inflammation responses
involve both meninges and the brain parenchyma

Etiology
Fig. 1: Pathogenesis of Meningitis In Immunocompetent individuals:
y HSV
y

y AZV
Laboratory Diagnosis
y

y EBV
y

y Three parts of CSF are collected seperately under strict


y
y Arbovirus alphavirus flavivirus (Japanese encephalitis)
y

aseptic precautions
y 1 part is used for cell count
Clinical Manifestation
y

y 2 part is used for serological testing


y

y 3 part is used for CSF culture


y
yy Acute febrile illness
yy Altered level of consciousness
y Confusion abnormal behavior
CSF Findings in Acute Bacterial Meningitis
y

yy Hallucination agitation
y Polymorphonuclear (PMN) leukocytosis (>100 cells/μL in
y
yy Focal or generalized seizures
90%) yy Most common focal findings
y Decreased glucose concentration (<2.2 mmol/L [<40 mg/dL]
y
 Aphasia


y CSF/serum glucose ratio of <0.4


y
 Ataxia


y Increased protein concentration (>0.45 g/L [>45 mg/dL]


y
 Weakness


y Increased opening pressure (>180 mmH2O in 90%)


y
 Myoclonic jerk tremor


y CSF bacterial cultures are positive in >80% of patients


y
 Cranial nerve palsies


y CSF Gram stain demonstrates organisms in >60%


y
 SIADH


 Diabetes insipidus


Neuroimaging
y MRI is preferred than CT for demonstrating cerebral edema
y

and ischemia
483
Laboratory Diagnosis y VZV- CSF antibody positive and PCR
Unit 8     Infectious Diseases

y WNV – CSF IgM antibodies are confirmatory as they do not


CSF Analysis
y

cross BBB
y 20 mL should be collected
y

y 5- 10 mL frozen for later studies


MRI/CT/ EEG
y

CSF Finding y To rule out other diagnosis


y

y Focal findings always raise the possibility of HSV encephalitis


y Lymphocyte pleocytosis
y

80% will have abnormality in temporal lobe


y

y Mildly elevated protein


y CT is less sensitive
y

y Normal glucose concentration


y

y EEG abnormalities occur > 75 % of PCR documented cases of


y

y Hemorrhagic encephalitis by HSV – RBC >500 per micro liter


y

HSV involving temporal lobe


y

y A decrease in glucose is unusual


y WNV: Abnormalities often involve brain stem thalamus and
y

basal ganglia
CSF PCR y VZV- Multifocal areas of hemorrhagic and ischemic infarc-
y

y Primary diagnostic test for CMV, EBV, HHV-6, and entero-


y
tions (CNS vasculopathy)
viruses y CMV: Enlarges ventricles
y

y HSV (SF PCR 96% sensitivity 99% specificity


y

y 98% positive during first week


y

y VZV CSF PCR and IgM antibody


y
Treatment
yy Basic management and supportive therapy
y Careful monitoring of ICP
CSF Culture
y

yy Fluid restriction
y Limited Utility
y
yy Suppression of fever
Serological Studies yy Anti-convulsants
y Acyclovir – HSV encephalitis
y HSV – both antigen and antibody are detected only after the
y

y Ganciclovir and foscarnet- CMV related CNS infections


y

first week
y

yy Cidofovir – CMV retinitis, CMV encephalitis


yy No specific antiviral for WNV encephalitis

484
Skin and Soft Tissue 74
Infections
y Skin and soft tissue infections are most commonly caused by Table 2:  List of lesions with clinical syndromes and
y
bacteria followed by fungi and viruses. etiological agents
y Skin and soft tissue infections are classified as follows
y
y Each skin infection has a characteristic lesions which gives Vesicles seen in:
y
clinical clue to diagnosis: • Smallpox


• Chicken pox


Table 1:  Classification of skin and soft tissue infections • Shingles or zoster


• Herpetic whitlow
Terminology Condition


• HFMD


Primary pyoderma • Impetigo • Orf


• Folliculitis • Molluscum contagiosum


• Furuncle • Rickettsialpox


• Carbuncle Bullae seen in:

• Paronychia • SSSS


• Ecthyma • Necrotizing fasciitis


• Erysipelas • Gas gangrene


• Chancriform lesions • Halophilic vibrio infections


• Membranous ulcers Crusts seen in: (Mostly fungal)

• Cellulitis • Bullous impetigo


Infectious gangrene and • Necrotizing fasciitis • Ring worm

• Sporotrichosis

gangrenous cellulitis • Gas gangrene

• Histoplasmosis

Secondary bacterial infections • Burns

• Coccidioidomycosis

complicating pre-existing skin • Eczematous dermatitis

• Blastomycosis

lesions • Dermatophytosis

• Cutaneous leishmaniasis

• Hidradenitis suppurativa

• Cutaneous TB

• Pyoderma gangrenosum

• Nocardiosis

• Intertrigo

Papule and nodules seen in:

• Pilonidal abscess • Swimming pool granuloma


Cutaneous Involvement in • Bacteremia • Cutaneous larva migrans


Systemic Bacterial and Mycotic • Fungemia • Dracunculiasis


Infections • Infective endocarditis • Cercarial dermatitis


• Listeriosis • Verruca vulgaris


• Leptospirosis • Condyloma acuminata


• Rat bite fever • Onchocerciasis nodule


• Melioidosis • Cutaneous myiasis


• Glanders • Verruca peruana


• Carrion’s disease • Cat scratch disease


Scarlet Fever syndromes • Scarlet fever • Lepromatous leprosy

• Syphilis

• Scalded skin syndrome


• Toxic shock syndrome

Postinfectious Nonsuppurative • Purpura fulminans

complications • Erythema nodosum

Others • Erythrasma

Table 3:  Treatment of common skin infections
Unit 8     Infectious Diseases

Common conditions Choice of drug for treatment

MRSA skin infections (abscess, furuncle, carbuncle) • Vancomycin


• Linezolid

Group A streptococci–necrotizing fasciitis Clindamycin


+Penicillin/3rd generation cephalosporin
Gas gangrene Clindamycin
+ Penicillin/Cefoxitin
Mixed aerobic and anaerobic necrotising fasciitis Ampicillin + Clindamycin + Ciprofloxacin
(or)
Vancomycin + Metronidazole + Ciprofloxacin
Herpes simplex Acyclovir (or) Famciclovir
Herpes zoster Acyclovir (or) Famciclovir
History of animal bites Amoxicillin/Clavulanate or Doxycycline

486
Infections of Bones 75
and Joints
INFECTIOUS ARTHRITIS  Depresses delivery of inflammatory cells to infectious foci


and serum complement components
y Most common cause for infectious arthritis is Staphylococcus
y Basal energy requirement of a traumatized or infected patient
y
aureus

y
increases from 30 to 55%
y Infection affecting the joints leads to articular cartilage
y Fever of just 1°F above normal increases the body’s metabolic
y
damage rapidly, hence proper evaluation is must to avoid

y
rate by 13%
damage
y Nutritional support is recommended before surgery for
y Infectious arthritis can be classified into:

y
patients with
y
 Acute monoarticular arthritis, usually bacterial infection
 Recent weight loss of >10 lb

 Subacute or chronic monoarthritis/oligoarthritis, usually


 Serum albumin levels <3.4g/dL

mycobacterial or fungal infection


 Lymphocyte counts <1500 cells/mm3
 Polyarticular arthritis, usually as a result of immunologic


y Screening formula who needs nutritional support

reaction.
y
 [(1.2 x serum albumin) + (0.013 x serum transfenin)] – 6.43

if the sum is 0 or negative patient is nutritionally depleted
Risk of Infection and needs support.

Patient Dependent Factors Immunological Status


Nutritional Status y To fight infection, patient must mount immune (Antibody)
y
y Malnourished or immunocompromised patients cannot be and inflammatory (WBC) responses that initially stop the
spread of infection and then ideally destroy the infecting
y
able to mount a response to an infection
y Malnutrition adversely affects organism.
y Body’s defence mechanism
y
 Humoral and cell-mediated immunity
y
 Neutrophil response

 Impairs neutrophil chemotaxis

 Humoral immunity

 Diminishes bacterial clearance

 Cell-medicated immunity

 Depresses neutrophil bacterial function

 Reticuloendothelial cells


Table 1:  Immunological status of patient with infectious arthritis
• Neutrophil • Causes of neutrophil abnormalities • Encapsulated bacteria in infants and



ƒ Diabetes elderly
ƒ
ƒ Alcoholism • Pseudomonas infection in Heroin
ƒ

ƒ Hematological malignancy addicts
ƒ
ƒ Cytotoxic therapy • Salmonella and Pneumococcus
ƒ

infection in Sickel Cell anemia
• Neutrophil count <55/mm3

infection caused by staphylococcus
aureus, GN bacilli, Aspergillus
organisms, Candida organism
• Humoral immunity • Causes of Humoral immunity abnormalities Increased risk of infection caused by


ƒ Immunoglobulins and Complement factors ƒ Hypogammaglobulinemia and encapsulated bacteria such as
ƒ
ƒ
are the two plasma protein that play crucial splenectomy • Streptococcus pneumonia

roles in humoral immunity • Haemophilus influenza

• Neisseria organisms

• In culture negative septic arthritis →

Mycoplasma pneumonia, Uroplasma
urealyticum septic arthritis.
• Defect in component of complement • S.aureus and Gram-negative bacilli


cascade infection are common.
(Contd...)
Unit 8     Infectious Diseases

• Cell-mediated immunity
• •• Primary cell-mediated deficiency → rare • • Fungal infection
ƒ Depends on the interaction between
ƒ •• Secondary cell-mediated deficiencies • • Mycobacterial infection
T-lymphocytes and macrophages •• Corticosteroid therapy • • Herpes virus
•• Malnutrition • • Pneumocystis jiroveci
•• Lymphoma
•• SLE
•• Immunodeficiency in elderly patient
•• Autoimmune deficiency syndrome

Surgeon Dependent Factors– Special Antibiotic Administration


consideration of Ortho OT y Given 30 minutes before surgery
y

y Skin preparation:
y
y Maximal dose of antibiotic should be given
y

 Skin can never be disinfected completely but the number



y Repeated every 4 hours intraoperatively or whenever blood
y

of bacteria present can be reduced markedly before surgery loss exceeds 1,000 to 1,500ml
 Skin and hair can be sterilized with alcohol, iodine,

y Should not be extended past 24 hours even if drains and
y

hexachlorophene or chlorhexidine, but it is almost catheters are placed


impossible to sterilize the hair follicles and sebaceous
glands where bacteria reside normally OSTEOMYELITIS
 Hand washing or hand rubbing with an aqueous alcohol


y Defined as an inflammation of the bone caused by an


y

solution that is preceded by a 1 minute nonantiseptic infecting organism.


hand washing for the first case of the day was found to y Infection may be limited to a single portion of the bone or
y

be as effective in prevention of surgical site infections as may involve numerous regions, such as the marrow, cortex,
traditional hand scrubbing with antiseptic soap. periosteum and surrounding soft tissue.
 Hair removal at the operative site is not recommended


unless done in the operating room. Shaving the operation


site the night before the surgery can cause local trauma
High Yield
that produce a favorable environment for bacterial General Principles
reproduction. • When does osteomyelitis occur?

 Surgical gloves should be changed every 2 hours


 ƒ Mere presence of bacteria in bone from bacteremia or from
ƒ

y Operating Room environment:


y direct inoculation is insufficient to produce osteomyelitis.
 Airborne bacteria usually Gram-positive bacteria originate
 ƒ Osteomyelitis occurs when an adequate number of a
ƒ

exclusively from humans in the operating room (or), sufficiently virulent organism overcomes the host’s natural
source of wound contamination. defenses (inflammatory and immune response) and
 5,000 to 55,000 particles are shed per minute by each
 establishes a focus of infection.
individual in OR. • Why acute hematogenous osteomyelitis is common in

 Conventional OR air may contain 10 to 15 bacteria/cubic


 metaphysis?
foot and 250,000 particles/cubic foot. ƒ The relative absence of phagocytic cells in the metaphysis
ƒ

 Airborne bacterial concentration reduced by at least 80%


 of bone in children makes metaphysis a common site of
laminar air flow system and even more with personnel osteomyelitis.
isolator systems • What is peculiar about an abscess in bone and why there is

 UV light or decrease the incidence of wound infection by


 failure of medical treatment of osteomyelitis?
decreasing the number of airborne bacteria. ƒ Peculiarity is that it is confined within a firm structure with
ƒ

little chance of tissue expansion


ƒ As infection progresses purulent material works its way
Prophylactic Antibiotic Therapy ƒ

through the haversian system and Volkmann canals and lifts


y During the first 24 hours infection depends on the number of
y

periosteum off the surface of the bone


bacteria present. ƒ Pus in the medullary cavity and in subperiosteal space
ƒ

y First 2 hours host-defence mechanism works to decrease the


y

together causes necrosis of cortical bone known as


overall number of bacteria. sequestrum.
y Next 4 hours the number of bacteria remains fairly constant—
y

ƒ Antibiotics and inflammatory cells cannot adequately


ƒ

bacteria multiplied and killed by host defence are equal. access this avascular area, resulting in failure of medical
y This first 6 hours is called Golden Period.
y

treatment of osteomyelitis.
y After 6 hours bacteria multiply exponentially
y

y Antibiotic decreases the bacterial growth geometrically and


y

delays the reproduction of the bacteria. Expands the golden


Classification
period.
y Based on duration of symptoms
488  Patient’s skin remains the major source of infection most
y

 Acute
common S. aureus. 
Subacute Pathogenesis

Chapter 75     Infections of Bone and Joints




Chronic

y Bacterial seeding leads to → inflammatory reaction →


y Based on mechanism of infection


y

cause → local ischemic necrosis of bone → flowed by


y

 Exogenous
abscess formation → abscess enlarges → intramedullary


Š Open fracture
pressure increases → causes cortical ischemia → purulent
Š

Š Surgery iatrogenic
material escapes through the cortex into superiosteal space
Š

Š Contiguous spread from infected local tissue


→ subperiosteal abscess develops → results in extensive
Š

 Hematogenous
formation of sequestra → chronic osteomyelitis.


Š Results from bactermia


Š

y Based on host response


Other Facts
y

 Pyogenic


 Nonpyogenic
Most common Cause of


y Cirny Mader classification


y

• Postsurgical osteomyelitis

Staphylococcus aureus
Acute Hematogenous Osteomyelitis • Post-traumatic osteomyelitis

y y Most common type of bone infection • Osteomyelitis of spine


y y Usually seen in children • Septic arthritis


y More common in male


• Osteomyelitis in Drug abuser Pseudomonas aeurginosa
y

y Bimodal <2 years and 8–12 years


y y Caused by bacteremia, common occurrence in childhood • Bone and joint infection is


• Hematogenous
y y Bacteriological seeding of bone is generally associated with • Joint affected in septic
• Knee
other factors such as arthritis
 Localized trauma
The organism associated with Salmonella


 Chronic illness
SS or Sc haemoglobinopathies It is mostly diaphyseal rather


 Malnutrition
than metaphyseal


 Inadequate immune system.




Chronically ill patients receiving Fungal osteomyelitis


parenteral nutrition or
intravenous therapy

489
76 Sexually Transmitted
Infections
SEXUALLY TRANSMITTED INFECTIONS
y Sexually transmitted infections (STI) include diseases that are transmitted by sexual intercourse.
y
y STD includes infections that result in clinical diseases involving genitalia and other parts of the body in sexual interactions.
y
Table 1:  Causes of STI
Bacteria Virus Protozoa Fungus Ectoparasite
• Neisseria gonorrhoea Herpes Trichomonas vaginalis Candida Pthyris pubis

• Chlamydia trachomatis HBV albicans Sarcoptes scabiei

• Chlamydia granulomatis HPV

• Treponema pallidum CMV

• Haemophilus ducreyi Molluscum

• Calymatobacterium granulomatis HIV

• Mycoplasma hominis HTLV

• Gardnerella vaginalis

Table 2:  Clinical clues for each STI y Primary Syphilis–presents as chancre (Hunterian chancre)
y
y Heals within 3 to 8 weeks leaving behind thin atrophic scar.
Disease Ulcer Lymphnodes
y
y Serological test positive 5 to 6 weeks after infection or 2 to 3
y
Primary syphilis Chancre painless Painless-bilateral weeks after primary chancre
Chancroid Painful Painful–usually
unilateral Table 4:  Differences between chancre and chancroid
Donovanosis Painless (beefy red) Pseudobubo (lesion Chancre Chancroid
appearing in groin Single Multiple
may present like Painless Painful
lymphadenopathy Clean base Necrotic base
prior to rupture) Bilateral inguinal nodes Unilateral inguinal nodes
LGV Transient Painful usually No exudates Yellow exudates
Herpes Painful Painful
Table 5:  Differences between condyloma lata and
Important Clinical Differences in STI condyloma accuminata
y Bubos: Condyloma lata Condyloma accuminata
y
 Enlarged and tender lymph nodes (adenitis and periad- T.pallidum HPV

enitis) Pale in color Pink in color
 True bubo: LGV Flat topped Thrown to folds

 Pseudobubo: Donovanosis Bilateral lymphadenopathy -

Table 3:  Differences between chancroid bubo and LGV Table 6:  HPV types and their corresponding clinical
bubo diseases
Chancroid bubo LGV Bubo Clinical diseases HPV types
90% unilateral 2/3 unilateral Condyloma accuminata 6,11
CIN (Low grade) 6,11
Matted Not matted
CIN (High grade) 16,18
Unilocular Multilocular Bowenoid papulosis 16
Rupture with single opening Rupture with multiple opening Cervical cancer 16,18
Accompanied by genital ulcer Not accompanied by genital Giant condyloma Buschke and 6,11
ulcer Lowenstein
Groove sign negative Groove sign positive Oral and laryngeal papilloma 6,11
HIV 7,72,73
Other Features of Congenital Syphilis

Chapter 76     Sexually Transmitted Infections


Remember
Hutchinson’s Triad of Congenital Syphilis Bones
• Eighth Nerve deafness

y y Saddle nose
• Hutchinson’s teeth i.e. notched incisors

y y Frontal and parietal bossing
• Interstitial keratitis

y y Short maxillae
y y Bull dog jaw
* Sabre tibia is one of the stigmata of congenital syphilis, but not
 y y High arched palate
included in Hutchinson’s triad. y y Higoumenakis’ sign
y Sabre tibia
High Yield y

• Hutchinson’s sign 1: Hyperpigmentation of proximal nail fold


• Teeth
along with melanonychia is suggestive of melanoma in the y Hutchinson’s teeth
y

subungual region y Mulberry or Moon’s molars


• Hutchinson’s sign 2: Herpes zoster presenting with vesicles
y

on the tip of the nose implies involvement of the nasociliary


branch of the Ophthalmic Nerve, and heralds eye involvement
Eyes
• Hutchinson’s teeth: Notched incisors seen in Congenital

y Corneal opacities
y

Syphilis y Salt and pepper fundus


y

y Optic atrophy
y

Table 7: Major treatment for STI is based on kit colors


Clinical features Kit number Color Drug


Urethral, cervical, anorectal 1 Grey Azithromycin 1 g stat
Cefixime 400 mg stat
Vaginal 2 Green T.Secnidazole 2g stat
T.Flucanazole 150 mg stat.
Genital ulcer (non herpetic) 3 White Inj Benzathine penicillin 2.4 MU +T.azithromycin 1 g stat
Genital ulcer (non herpetic allergic to penicillin) 4 Blue T.Doxy 100 mg BD for 14 days + T.azithromycin 1 g stat
Genital ulcer (Herpetic) 5 Red T.Acyclovir 400 mg tds for 7 days
Lower abdominal pain 6 Yellow T.cefixime 400 mg stat
T.metronidazole 400 bd for 14 days
T.Doxy 100 mg bd for 14 days
Bubo 7 Black T.Doxy 100 mg bd for 21 days
T.Azithromycin 1 g stat

491
77
Urinary Tract Infections
INTRODUCTION y Urine and serum antibodies play an important part in the

y
pathogenesis of UTI.
y Urinary tract infection (UTI) is mainly a disease of the females,
y The adherence of bacteria to the uroepithelial cells is
y
due to the anatomical structure of the female urethra

y
prevented by the urinary antibodies and thereby they resist
y There are 2 types of urinary tract infection (UTI):
UTI
y
Asymptomatic and symptomatic urinary tract infection.
y Definition of asymptomatic bacteriuria is the occurrence of
Remember
y
actively multiplying bacteria, more than 105 bacteria per mL
of urine inside the urinary tract, exclusive of the distal urethra • Most common organism is UTI: E.coli


at a time when the patient has nil symptoms of UTI. • Most common organism in CAUTI: E.coli


y Urinary tract infections (UTI’s) can be defined as bacteriuria • Most common organism of UTI in sexually active females:
y

(>105 CFU/mL in adults; >104 CFU/mL in children) of an S.saprophyticus
urinary pathogen with associated clinical symptoms and
signs like urgency, painful micturition and hematuria.
y According to the Centers for Disease Control and Prevention CATHETER-ASSOCIATED URINARY TRACT
y
(CDC), a symptomatic UTI must meet at least one of the
following criteria:
INFECTIONS
 Patient was not on an indwelling urinary catheter in place y Catheter-associated urinary tract infection (CAUTI) is a major
y

at the time of specimen collection or during the onset of clinical problem which leads to considerable morbidity and
signs and symptoms. mortality.
 Patient has atleast any one of the following signs or y The overall prevalence of CAUTI accounts for up to 40% of all
y

symptoms with no other recognized causes for fever, the nosocomial infections and 80% of all nosocomial UTI.
urgency, frequency, dysuria, suprapubic tenderness and y Among the nosocomial UTI, Escherichia coli is the most
y
or costovertebral angle tenderness. common organism, followed by Enterococci sp., and then
 Patients` urine sample shows a positive growth for urinary Pseudomonas aeruginosa.

pathogens of ≥105 with no more than 2 species in growth.
Definition of CAUTI
ETIOLOGICAL AGENTS y UTI, where an indwelling urinary catheter was in place for
y
y The most common pathogen is Escherichia coli contributing >2 calendar days on the date of event, with day of device
y
to 70–80%. placement being Day 1 and an indwelling urinary catheter
y Other organisms are: Klebsiella pneumoniae, Proteus mira- was in place on the date of event or the day before. If an
y
bilis, coagulase-negative staphylococci, group B streptococci indwelling urinary catheter was in place for > 2 calendar
and Gardnerella vaginalis. days and then removed, the date of event for the UTI must
y Other organisms causing asymptomatic bacteriuria are be the day of discontinuation or the next day for the UTI to be
y
Enterobacter species and Pseudomonas aeruginosa catheter-associated.

PATHOGENESIS Clinical Syndromes


y Bacteria that originate in the large bowel most likely colonize y Asymptomatic bacteruiria
y
y
the urinary tract transperineally. y Cystitis
y
y Escherichia coli cause 75–90% of bacteriuria in antenatal y Pyelonephritis
y
y
women.
y Adherence of the organisms to epithelial cells is important for Lab Diagnosis
y
the colonization and infection of mucous membranes.
y The colonization of the urogenital epithelium with the Sample–Urine
y
bacteria of susceptible persons is related to the successful y Clean voided, mid stream urine sample need to be collected
y
invasion of the urinary tract. y For culture–transportation should be within ½ hour to a
y
maximum of 2 hours
y If delayed–refrigeration should be done–refrigerated sample Confirmatory Method for Diagnosis of CAUTI

Chapter 77     Urinary Tract Infections


y

is accepted for a maximum of 4–6 hours


y The ideal method is to do quantitative loop method that
y Other way to collect urine is from catheters and in infants and
y

determines bacterial number by inoculating plates using


y

babies by suprapubic aspiration calibrated platinum loops and counting colonies directly.
y Microscopic examination of urine–when Gram stained smear
y

y Recently, many automated methods of ascertaining bacterial


shows one bacilli/ HPF–then it is indicative of UTI (But
y

counts are currently available, including staining bacterial cells


culture is confirmatory) with safranin (Bac-T-Screen) or acridine orange (Autotrak).
y Culture of urine: Media used are:
y

y Once these screening tests are completed, the presence of


y

 MacConkey agar or CLED agar




a CAUTI is confirmed by the determination of bacterial cell


 Blood agar


counts.
y Two urine cultures with repeated isolation of the same
y

High Yield uropathogen with 100 CFU per mL of urine or the isolation
of a single uropathogen with 105 colonies per mL for a patient
• CLED agar is preferred over MAC because:

being treated with antimicrobial agents is indicative of


ƒ It supports the growth of both Gram-positive cocci like
ƒ

urinary tract infection.


Staphylococci and Gram-negative bacilli y In addition with bacterial cell counts, a urine culture
y

ƒ It supports growth of Candida species


ƒ

containing an abnormal number of red blood cells (hematuria,


ƒ We can differentiate LF and NLF
ƒ

5 red blood cells per high-power field) and leukocytes (pyuria,


10 WBCs per mL3) signifies the presence of CAUTIs.
y Quantitative count estimation of colonies (by Kass criteria)
y

is must to diagnose UTI:


TREATMENT
 105colonies/mL = diagnosed as UTI


 Exception: 102 colonies/mL is enough to diagnose UTI in




Clinical condition Drugs given


certain conditions like
Acute uncomplicated cystitis • Nitrofurantoin
Š When the organism is Staphylococci

• Fosfomycin
Š

Š Catheterized samples

• TMP-SMX
Š

Š Hematogeous infections

• Fluoroquinolones
Š

Š Patients already on antimicrobials


y Other supportive tests done in urine sample to diagnose


y
UTI in pregnancy • Nitrofurantoin

UTI: • Ampicillin

 Catalase test

• Cephalosporins

 Griess nitrite test


 UTI in men • Cephalosporins

 Triphenyl tetrazolium chloride test


 • TMP-SMX

 Dip slide culture




Complicated UTI • Ciprofloxacin (7 days course)


 Leukocyte esterase test




• Double strength TMP-SMX


493
78 Infections Related to
Obstretics and Gynecology
PUERPERAL PYREXIA  Anaerobes:


Š Peptoccocci and peptostreptococci

Š
y Defined as temperature of >100.4°F (38°C) on any two of the Š Bacterioides
y
Š
first 10 days after delivery excluding the first 24 hours (low Š C. Sordellii and C. perfringens

Š
grade fever in first 24 hours is common) Š Fusobacterium and mobilincus

Š
Etiology Pathogenesis
y Uterine sepsis y It is an endometritis that can subsequently involve the
y
y
y Pelvic infection parametrium (pelvic cellulitis) and eventually the adnexa in
y
y Breast infection (mastitis) some cases too.
y
y Urinary tract infection y Even in cesarean section, it is an ascending infection into the
y
y
y Postsurgical wound infection (LSCS & Episiotomy wounds) genital tract
y
y Respiratory tract infection (associated with anesthesia) y In puerperium, the collection of blood and devitalized
y
y
tissue provides a good medium for growth of normal vaginal
Predisposing/Risk Factors bacterial flora
y Placental site almost often involved along with extraplacental
y Cesarean section-single most important risk factor. Risk is
y
tissue
y
13 times more and mortality is 25 times more than vaginal
y The spread of infection is through
delivery
y
 Contiguous spread from uterus (most common)
y Bacterial vaginosis (vaginal colonization with M. hominis,

 Hematogenous
y
Ureaplasma urealyticum, G. vaginalis, C. trachomatis and

 Lymphatics.
group B streptococcus)

y Prolonged labor (>12 hours)
Clinical Features
y
y Frequent vaginal examinations (>5)
y
y Operative vaginal delivery
Signs
y
y Maternal anemia
y
y GDM y Most common: Fever
y
y
y Meconium-stained amniotic fluid  Within 24-48 hours: Group A streptococci

y
y Manual removal of placenta  Later onset: Mixed infection

y
y Low socioeconomic status, young age and nullipara  After 7 days of deliver: Chlamydia should be suspected

y
y Uterine tenderness
y
y Coronary tachycardia
Microbiology
y
y Purulent lochia
y
y The most common pathogens involved are Escherichia coli, y Subinvolution of uterus
y
y
Klebsiella pneumoniae and Proteus species.
y Most common pathogens associated with puerperal sepsis Symptoms
y
are grouped as below
Fever, chills, low, abdominal pain, headache, malaise,
 Aerobic Gram-positive:
malodorous lochia, excessive bleeding per vaginum

Š Group A, B and D streptococcus
Š
Š Staphylococcus aureus
Diagnosis
Š
Š Enterococcus
Š
 Aerobic Gram-negative: y Thorough examination of abdomen, perineum, breasts,

y
Š E. Coli respiratory tract.
Š
Š K. pneumoniae y High vaginal swab
Š
y
Š N. gonorrhoea y Urine culture and sensitivity
Š
y
Š Proteus y Blood culture in selective patients.
Š
y
Treatment They lodge in urethra and cervix and get carried along

Chapter 78     Infections Related to Obstretics and Gynaecology




with sperm to the tubes


y Broad spectrum antibiotics covering aerobic Gram positive,
They create an environment for secondary infection
y


Gram negative and anaerobic organisms are used


by normal vaginal flora as a result almost all PID are


y Mild endometritis—oral amoxicillin-clavulanate and oral
polymicrobial in nature.
y

metronidazole
y More severe infections—first-line of treatment is a
Organisms Causing PID
y

combination of gentamycin and clindamycin (95% cure rate )


y Other antibiotics used: cephalosporins, piperacillin and
Primary Infections (Sexually Transmitted)
y

ampicillin sulbactum
y Duration of therapy: IV antibiotics are continued up to 24
y
y 30% Gonococcus (2–10% of cultures are by penicillinase
y

hours after patient becomes afebrile producing (penicillin resistant) gonococci)


y If blood culture is positive—continue for 2 weeks
y
y 30% Chlamydia
y

y If no response, consider—drug resistance, other causes for


y
y 10% Mycoplasma (M. genitalium is the newer organism
y

fever, pelvic cellulitis, wound dehiscence, retained products, known to cause PID)
and unusual organisms. y Trichomonas
y

Prevention Secondary Infections


y Antibiotic prophylaxis in cesarean section-single dose of 2 g y Pyogenic
y

 Aerobes
y

ampicillin IV or other 1st generation cephalosporin (not more 

than 30 mins before skin incision) Š Staphylococcus


Š

y Spontaneous delivery of placenta in cesarean section Š Streptococcus


Š

Š E. coli
y

y Avoiding unnecessary PV examination Š

y Anaerobes
y

y Cleansing the vagina with povidone iodine solution prior to y

 Bacteroides fragilis
y

cesarean section. 

 Peptococcus


 Clostridium
PELVIC INFLAMMATORY DISEASE (PID)


 Actinomyces (IUCD)


y It is a spectrum of infection and inflammation of upper genital


y
y Tuberculous Salpingitis
y

tract almost always involving the fallopian tubes and ovaries


y It includes endometritis, salpingitis, tuboovarian abscess,
y
Risk Factors
pelvic peritonitis and parametrial infection too. It excludes
cervicitis Developed Countries
y Most often bilateral (one tube more affected than the other)
y
y STDs (75%)
y

y Most common route of spread is ascending infection.


y
y Multiple sex partners and promiscuities
y

(exception-Tuberculous PID which spreads hematogenously)


y Ovaries are almost always involved along with the tubes.
y

Developing Countries
(Exception-mumps where ovaries are selectively affected)
y Septic abortion
y

y Puerperal sepsis
Etiology
y

Normal barriers to infection of genital tract Incidence


y Acidity of vaginal secretion
y Among sexually active women 1–2%
y

y Hymen (in virgins, it is usually tubercular)


y

y 85% of PID is spontaneous in sexually active women


y

y Small cervical lumen


y

y 15% is iatrogenic (following D&C, IUCD insertion, HSG,


y

y Mucus plug in cervix


y

Endometrial biopsy, etc)


y

y Downward ciliary movement of endometrial lining of uterus


y

and cervix
 These barriers are impaired during menstruation,
 Shift in Epidiemology
abortion, delivery and D & C favoring spread of pathogens y From inpatient PID to OP PID
y

 PID is most commonly seen in sexually and reproductively




y To less severe and more common disease


y

active young women. y To more of chlamydial etiology than gonococcal or others.


y

 Most common cause of PID is sexually transmitted




diseases of which gonococcal and chlamydial infections


are most common. Pathology
 These two organisms ride in a PIGGY-BACK fashion on
 Infection of urethra and endocervix, endometritis, salpingitis
sperms to reach the upper genital tract. salpingoophoritis tubo-ovarian abscess, peritonitis.

495
Symptoms Definitive Criteria
Unit 8     Infectious Diseases

y y Most common bilateral lower abdominal pain y Histopathological evidence of endometritis on endometrial
y

y y Abnormal vaginal discharge biopsy


y y Menometrorrhagia y Evidence of tubo-ovarian abcess on ultrasound, CT or MRI
y

y y Fever y Laparoscopic findings suggestive of PID.


y

y y Nausea
y Postcoital bleeding.
y

Management
Signs Principles
y y High fever yy Treat acute infection and symptoms
y y Tachycardia yy Treat partner
y y Dehydration yy Follow-up to prevent sequale
y y Abdominal tenderness sometimes rigidity yy Counseling.
y Cervical movement tenderness
CDC Guidelines of OP Management of PID
y

y y Foul smelling purulent discharge


y y Forniceal tenderness y Ceftriaxone 250 mg IM single dose (or) cefoxitin 2g IM plus
y

y y Tender adnexal mass. probenecid 1 g orally (or) other parenteral 3rd generation
cephalosporins
Prevention y Add C. Doxycycline 100 mg BID orally for 14 days with or
y

without T. metronidazole 400 mg BID for 14 days.


Syndromic Approach
It involves treatment of PID based on signs and symptoms rather Definite Indications for Hospitalization
than lab investigations yy Not responding to oral medication
y CDC recommends empirical treatment of sexually active
y
yy Pregnant women
young women with C/O lower abdominal pain (after yy Adolescents
excluding other causes) and in the presence of one or more yy When surgical emergencies cannot be ruled out
minimum criteria yy Intolerance to OP oral regimen
yy Generalized peritonitis
Diagnostic Criteria (CDC Guidelines) yy Tuboovarian abscess
y Severe illness
Minimum Criteria
y

y Lower abdominal tenderness


y

y Adnexal tenderness
y
Fitz Hughs Curtis Syndrome
y Cervical motion tenderness.
y
• Mostly associated with Chlamydia trachomatis

• It is the association of Right upper quadrant pain and


Additional Criteria tenderness with PID due to peritoneal spread resulting in


y Fever of 101°F or more perihepatitis with inflammation of hepatic capsule.
• Also associated with violin string like adhesions from
y

y y Abnormal cervical or vaginal discharge •

y Elevated CRP undersurface of liver and other pelvic tissue


• Investigation of choice: Laparoscopy
y

y Elevated ESR •

• Surest sign of salpingitis is pus extruding from fimbrial end.


y

y y Lab documentation of infection with N. gonorrheae or •

C. trachomatis.

496
Surgical Site and 79
Related Infections
SURGICAL INFECTIONS y Increased adherence

y
y Increased adhesion molecule generation
Definition

y
y Spontaneous activation of neutrophils

y
Surgical infections  includes any infection that affects surgical
patients. Examples of infections that may complicate Effects of Stress Response on Carbohydrate
perioperative care include surgical site infections (SSIs), central
Metabolism
line–associated bloodstream infections (CLABSIs), urinary
tract infections (UTIs), and hospital-or ventilator-associated y Enhanced peripheral glucose uptake

y
pneumonia (HAP, VAP). y Hyperlactatemia

y
y Increased gluconeogenesis

y
y Depressed glycogenolysis
General Considerations yy
y
Peripheral insulin resistance
y Invasiveness creates portals of entry for pathogens to invade
y
the host through natural epithelial barriers.
y Surgical illness is immunosuppressive (e.g., trauma, burns, Remember
y
malignant tumors), as is therapeutic immunosuppression Medical Conditions Predisposing to Postoperative Infection
after solid organ transplantation. • Extremes of age (neonates, very old adults)

y General anesthesia → poses a risk of pulmonary aspiration of • Malnutrition

y
gastric contents; both increase the risk of pneumonia. • Obesity

y The development of a postoperative infection has a negative • Diabetes mellitus

y
impact on surgical outcomes, recognizing and minimizing • Prior site irradiation

risk and an aggressive approach to the diagnosis and • Hypothermia

treatment of these infections are crucial. • Hypoxemia

• Coexisting infection remote to surgical site

• Corticosteroid therapy
Risk Factors to Infection

• Recent operation, especially of chest or abdomen

Host • Chronic inflammation

• Hypocholesterolemia
y Potential pathogens are ubiquitous in the environment, but

y
although colonization of epithelia occurs even in healthy
hosts, invasion generally requires through portal of entry, High Yield
which for surgical patients might include injured tissue,
incision, puncture site for vascular access, or indwelling Recent Updates on Surgical Infections Causes
catheter. • Supplemental oxygen (FiO2–0.8) reduces risk of infections

y Stress of surgery + Infection → accelerates cardiac following elective surgery
• Persistent SIRS increases incidence of infection
y
function  through the autonomic nervous system, promotes

glycogenolysis, catabolizes peripheral lean tissue and • Moderate hyperglycemia (>200 mg/dL) at any time on the

fat for gluconeogenesis, enhances coagulation to stanch first postoperative day increases the risk of SSI fourfold after
hemorrhage, and stimulates a proinflammatory cytokine cardiac and noncardiac surgery.
response to begin the tissue repair process. • Nutritional support is crucial, considering that restoration of

y Innate immunity and adaptive immunity are depressed in anabolism requires calories and nitrogen in excess of basal
requirements of 25 to 30 kcal and 1 g nitrogen/kg/day
y
large part by the actions of cortisol
• Each day of endotracheal intubation and mechanical

ventilation increases the risk of pneumonia by 1% to 3%
Effects of Hyperglycemia on Immune Cell
Function
Surgical Site Infections
y Decreased respiratory burst of alveolar macrophages
y Major SSI: Wound that either discharges significant amount
y
y Decreased insulin-stimulated chemokinesis
y
of pus or needs surgical procedure to drain
y
y Glucose-induced protein kinase C activation
y Systemic signs of toxicity are present
y
y
y Other infections are called Minor SSI
Unit 8     Infectious Diseases

Grade Appearance
y SSI remains a clinical diagnosis. Presenting signs and
IIc Along wound
y

symptoms depend on the depth of infection, typically as early


as postoperative day 4 or 5 IId Around wound
Table 1:  Infection rate and rate before prophylaxis in III Clear or hemoserous discharge
various types of surgeries IIIa At one point only (≤2 cm)

Type of Surgery Infection Rate Rate Before IIIb Along wound (>2 cm)
Prophylaxis IIIc Large volume
Clean (No Viscus 1–3% Same IIId Prolonged (>3 days)
opened)
IV Pus
Clean contaminated <10% (5– 8%) Gastric surgery
IVa At one point only (≤2 cm)
(Viscus opened, up to 30%
minimal spillage) Biliary surgery IVb Along wound (>2 cm)
up to 20% V Deep or severe infections; hemotoma requiring
Contaminated (Open 15–20% (20–25%) Variable but aspiration
viscus with spillage or up to 60%
inflammatory disease Precautions
Dirty (Pus, Perforation <40% (30–40%) Up to 60% and y Hand hygiene is the most effective means to reduce the
y

or Feces) more spread of infection. Alcohol gel hand cleansers are effective
except against the spores of Clostridium difficile,  which
The surgical wounds can be scored using following scoring requires cleansing with soap and water.
systems: y Any invasive catheterization to be removed and kept not
y

beyond 24 hours unless indicated


Table 2:  Asepsis wound score y Surgical Skin preparation: Chlorhexidine 0.5 to 4% is used.
y

Criterion Points y Skin preparation for vascular catheter insertion; it is superior


y

to povidone-iodine solution  and is also recommended for


Additional treatment:
surgical skin preparation
~ Antibiotics for wound infection 10 y For topical bathing of critically ill patients and As an antiseptic
y

~ Drainage of pus under local anethesia 5 coating for indwelling central vascular catheters.
y A single dose of a first-generation cephalosporin (e.g.,
~ Debridement of wound under general anethesia 10
y

cefazolin) may prevent some infections after emergency tube


Serous discharge Daily 0-5 thoracostomy or ventriculostomy.
Erythema Daily 0-5 y Central venous catheter is inserted by infraclavicular
y

approach into subclavian vein decreases infections


Purulent discharge Daily 0-10
y Double lumen ET tube with continuous aspiration and
y

Separation of deep tissues Daily 0-10 Silver impregnated ET tube reduces Ventilator associated
Isolation of bacteria from wound 10 Pneumonia
y Drains placed in incisions probably cause more infections
Stay as in patient for more than 14 days because of 5
y

than they prevent. Epithelialization of the wound is prevented


wound infection
and the drain becomes a conduit
y Wound irrigation is a controversial means to reduce the risk
Table 3:  Southampton wound grading system
y

of SSIs. Routine low-pressure saline irrigation is ineffective


Grade Appearance but high-pressure (i.e., pulsed) irrigation may be beneficial.
y Antibiotic Prophylaxis
0 Normal healing
y

 Prophylaxis of patients who have their antibiotic dose




I Normal healing with mild bruising and erythema initiated within 1 hour before surgical incision (2 hours for
Ia Some bruising vancomycin or a fluoroquinolone)
 Prophylaxis of patients whose prophylactic antibiotics
Ib Considerable bruising


were discontinued within 24 hours of the surgery end time


Ic Mild erythema (48 hours for cardiac surgery).
II Erythema plus other signs of inflammation y Glucose Control
y

 Blood glucose concentration must be maintained <200




IIa At one point mg/dL for the first 2 days after surgery.
IIb Around sutures
(Contd...)
498
y Hair Removal y Early onset VAP -Trauma patients is often a result of aspiration

Chapter 79     Surgical Site and Related Infections


y y

 No hair removal should be performed; if hair is removed,


 of gastric contents and is usually caused by antibiotic-sensitive
clippers or a depilatory agent should be used immediately bacteria such as methicillin-sensitive S. aureus, Streptococcus
before surgery. Razors are not to be used. pneumoniae,  and Haemophilus influenza.
y Normothermia (Colorectal Surgery Patients)
y y Late onset: Patients with late-onset VAP are at increased
y

 Core body temperature should be between 96.8°F and


 risk for infection with MDR pathogens (e.g., Pseudomonas
100.4°F within the first hour after leaving the operating aeruginosa, Acinetobacter  spp.).
room.
Table 4:  Antibiotic prophylaxis UTI
y Avoid prolonged catheterization.
Type of surgery Most common Prophylaxis
y

y Classical symptoms of UTI are absent with ICU patients


organisms
y

y Organisms associated with UTI in ICU patients and


y

Vascular CONS Augmentin +/– multiresistant, nosocomial, gram-negative bacilli other than
MRSA Gentamicin E. coli, Enterococcus  spp., and yeasts.
GNB Vancomycin y Community acquired UTI is most commonly caused by E. coli
y

Orthopedic S.epidermidis Augmentin y Counts higher than 103 CFU/mL represent true bacteriuria or
y

S.aureus candiduria in catheterized patients.


Esophagogastric Enterobacteriaceae Second gen.
Enterococci cephalosporin with Intra-abdominal Infection
metronidazole y Intra-abdominal infections (IAIs) represent a diverse group of
y

Biliary Enterobacteriaceae Second gen. diseases


Enterococci cephalosporin y Divided into uncomplicated (uIAI) and complicated (cIAI)
y

and, more recently, as to whether they arose in the community


Small bowel Enterobacteriaceae Second gen.
associated (CA-IAI) or hospital-associated (HA-IAI) setting
Anaerobes cephalosporin with/
(e.g., associated with a colon anastomotic dehiscence
out metronidazole
y In uIAIs, infection is contained within a single organ, and
y

Appendix/ Enterobacteriaceae Second gen. there may be no perforation of the gastrointestinal tract.
colorectal Anaerobes cephalosporin with Almost never cause serious complications.
metronidazole y cIAIs extend beyond the source organ and into the peritoneal
y

cavity through a perforated viscus, thereby stimulating a


Table 5:  Antibiotic prophylaxis given during elective
greater SIRS response.
surgery
 Contained infection results in the formation of an


Cardiac surgeries Cefazolin (or) abscess, which is facilitated by foreign bodies to lower
Cefuroxime (or) the inoculum size and microbial synergy, and creates a
Vancomycin low pH environment that impairs phagocyte function and
impedes permeation of immune cells and antibiotics.
Hip or knee arthroplasty Cefazolin (or)
 Uncontained spread of infection leads to diffuse peritonitis
Cefuroxime (or)


y In cases of severe sepsis or septic shock secondary to IAI,


Vancomycin
y

termed abdominal sepsis, mortality is approximately 25% to


Colonic surgeries Oral–Neomycin sulfate + 35% but may exceed 70%.
erythromycin + metronidazole  Treatment of abdominal sepsis is predicated on adequate


Parenteral–Cefoxitin (or) Cefotetan physical drainage or resection of the infected focus (termed
(or) Ertapenam (or) Cefazolin + source control), which may range from percutaneous
Metronidazole drainage to serial laparotomies and open abdominal
Hysterectomy Cefazolin (or) Cefoxitin or Cefotetan wound management in severe cases.
(or) Cefuroxime  15% need ICU care → penetrating intestinal injury that is


not recognized or treated promptly (>12-hour delay).


 Dehiscence of a bowel anastomosis with leakage or
IMPORTANT SURGICAL INFECTIONS


development of an intra-abdominal abscess.


Ventilator Associated Pneumonia y Secondary peritonitis → polymicrobial with anaerobic gram-
y

negative bacilli (e.g., Bacteroides fragilis) predominating and


y Defined as pneumonia occurring 48 to 72 hours after
E.coli  and Klebsiella spp.
y

endotracheal intubation
y When HA-IAI is a complication of disease or therapy, the flora
y The most common ICU infection among surgical and trauma
y

is more likely to reflect MDR pathogens  


y

patients. Two type  early-onset VAP (occurring <5 days after


y Failure of two source control procedures with persistent intra-
intubation) and late-onset VAP (occurring ≥5 days after
y

abdominal collections is referred to as tertiary peritonitis.


intubation)
y Tertiary peritonitis is also characterized by complete failure of
y

499
intra-abdominal host defences.
y Bacteria isolated in tertiary peritonitis are avirulent Peritonitis Associated with Chronic Ambulatory
Unit 8     Infectious Diseases

opportunists, such as MRSE, enterococci, Pseudomonas, and


Candida albicans, supporting the incompetent host defence
Peritoneal Dialysis
hypothesis. y Peritonitis is one of the most common complications of
y

y Peritoneal toilet can be provided manually—at the bedside


y
chronic ambulatory peritoneal dialysis
in some cases—under sedation or anesthesia, until local y Patients present with abdominal pain, fever, and cloudy
y

host defences  recover. There may be no alternative to open peritoneal dialysate containing more than 100 leukocytes/
abdominal management if the infection extends to involve mm3, with more than 50% of the cells being neutrophils.
the abdominal wall, and extensive débridement is required. y Gram staining detects organisms only in approximately 10%
y

to 40% of cases.
Peritonitis y Approximately 75% of infections are caused by gram-positive
y

organisms, with Staphylococcus epidermidis accounting for


y Peritonitis is inflammation of the peritoneum and peritoneal
30% to 50% of cases.
y

cavity, usually caused by a localized or generalized infection


y S.aureus, gram-negative bacilli, and fungi are also important
y Primary – bacterial, chlamydial, fungal or mycobacterium
y

causes of dialysis-associated peritonitis.


y

y Secondary- Secondary to GI perforation commonly.


y Peritoneal dialysis–associated peritonitis is treated by the
y

intraperitoneal administration of antibiotics, usually a first-


Spontaneous Bacterial Peritonitis generation cephalosporin.
Spontaneous bacterial peritonitis (SBP) is defined as a bacterial y Recurrent or persistent peritonitis requires removal of the
y

infection of ascitic fluid in the absence of an intra-abdominal dialysis catheter and resumption of hemodialysis
source of infection, such as visceral perforation, abscess, acute
pancreatitis, or cholecystitis.
y The most common pathogens in adults with SBP are
y
Appendicitis
the aerobic enteric  flora Escherichia coli and Klebsiella The following are the common organisms encountered at
pneumoniae. perforated appendix:
y In children with nephrogenic or hepatogenic ascites, group
y

A streptococcus, Staphylococcus aureus, and Streptococcus Table 6:  Bacteria commonly isolated in perforated
pneumoniae  are common isolates appendicitis
y SBP is seldom produced by anaerobic microorganisms
y

because of their incapacity to translocate to the intestinal Aerobic bacteria Anaerobic bacteria
mucosa and because of the high volume of oxygen in the • • Escherichia coli • Bacteroides fragilis

intestinal wall and in the tissues that surround it. • • Streptococcus viridans • Bacteroides

y Translocation of bacteria from gut is the mechanism of SBP


y • • Group D streptococcus thetaiotaomicron
y The diagnosis of SBP is made initially by demonstrating more
y • • Pseudomonas aeruginosa • Bilophila wadsworthia

than 250 neutrophils/mm3 of ascitic  fluid in a clinical setting • Peptostreptococcus spp


consistent with this diagnosis, that is, abdominal pain, fever,


or leukocytosis in a patient with low-protein ascites.

Treatment
y Broad Spectrum Antibiotics – 3rd generation cephalosporins
y

y Response is seen in up to 95% cases


y

y Prognosis is good.
y

500
80
Infections in Special Hosts
TRANSFUSION AND TRANSPLANT Table 2: List of organisms that get transferred through


TRANSMITTED INFECTIONS transplantation
y Transfusion and transplant transmitted infections (TTI) Bacteria Viruses Parasites Fungi
y
occur when a pathogen is spread from donor to recipients M. • CMV • Balamuthia • Histoplasma




through blood or any other organs. tuberculosis • HIV mandrillaris • Coccidioides



y Nowadays screening is mandatory in almost all the places, • HBV • Leishmania • Cryptococcus
y



hence TTI are rarely seen. • HCV • Trypanosoma • Microsporidium




y Risk of transmission is more in organs than in blood. • EBV cruzi • Apophysomyces
y


• HTLV • Toxoplasma
Table 1: List of organisms that get transferred through



• HHV 8 • S. stercoralis

blood transfusion



• HSV


Bacteria Viruses Parasites Prions • West Nile


virus
• Brucella • CMV • Plasmodium Variant CJD • Rabies



• Treponema • HIV • Babesia

virus



pallidum • HBV • Leishmania


• Coxiella • HCV • Trypanosoma
Laboratory Screening Tests



burnetii • EBV • Toxoplasma


• Ehrlichia • HTLV y Laboratory screening tests are available for many of TTI


y
• Rickettsia • HHV 8 agents


rickettsii • Parvovirus y In India–most commonly done screening tests are for:

y
• Pseudomonas B19  HIV


• Dengue  HBV


virus  HCV

• West Nile  Plasmodium


virus  Syphilis

y Tests that are recommended according to US FDA are:
y
 Hepatitis B surface antigen

 Hepatitis B core antibody

Remember  Hepatitis C virus antibody

 Anti HIV-1 and HIV-1 (antibody)
Contamination of blood and blood products with following

 HIV p24 antigen
agent leads to fatal bacteremia

 Anti HTLV-1 and HTLV-2 (antibody)
• Group B Streptococcus

 Serological test for syphilis

• E.coli

 NAAT for HIV-1, HBV, HCV

• P. rettgeri

 NAAT for West Nile virus

• Serratia marcescens

 Serological test for T.cruzi

• S. epidermidis


• Serratia liquefaciens

INFECTIONS IN SOLID ORGAN TRANSPLANT y Although most common is bacterial infections, comparatively
Unit 8     Infectious Diseases

fungal infections are higher than in any other organ


RECIPIENTS transplant.
y Infections in organ transplant patients mostly present as
y
y Candida is the predominant agent mostly by nonalbicans
y

opportunistic infections or reactivation of latent infections, or species.


health care associated infections and surgical site infections. y Aspergillus infection occurs after 90 days of transplantation.
y

y Type of infections:
y

 Bacterial infections are the most common seen presenting


Heart Transplant Recipients


as bacteremia.
 Next commonly seen is fungal infections followed by

y Most common infections that occur after heart transplantation
y

virus. are bacterial pneumonias, urinary infections, herpes virus


infections, invasive fungal infections.
Table 3: Time of occurrence of infections after
Remember

transplantation
Postoperative complications that are unique to heart are:
First 30 days (<1 month) • Infections are primarily surgical

• Mediastinitis
or technical complications that

• Sternal wound infections


are related to transplantation

and/or health care associated y Mediastinitis and sternal wound infections in heart are caused
y

• Most common is bacterial


• by M. hominis, Legionella, Aspergillus, Mucor, Nocardia.
infections
• Clostridium difficile colitis (M/c)

Table 4: Causes of early and late pneumonia


• Surgical site infections

• Candidiasis

Causes of Early pneumonia Causes of late pneumonia
• Aspergillosis
• (<1 month) (>1 month)
• Viral infections rare–only HSV

Common Less Common causes Less common


reactivation can occur causes common causes
30–180 days (1–6 months) • Opportunistic infections due to
• causes
immunosuppression occurs • GNB • Aspergillus • Pneumococcus • Pneumocystis
• Pneumocystis jirovecii
• • • •

• S. aureus • HSV • H.influenzae • Nocardia


• Fungi
• • • •

• Aspiration • Legionella • Influenza virus • Legionella


• Toxoplasma gondii
• • • •

• T. gondii • Idiopathic • Aspergillus


• Nocardia
• • •

• GNB

• CMV •

• S. aureus

After 6 months • Mostly community acquired


•• Aspiration
infection • Systemic fungi
• Herpes zoster

• TB
• Warts

• Viruses
• EBV

• Cryptococcus
INFECTIONS IN HEMATOPOIETIC STEM CELL

Kidney Transplant Recipients TRANSPLANT RECIPIENTS


y Most of the infections come from urinary tract–uncomplicated
y
y Stem cell transplant donors are divided into three main
y

cystitis categories:
y Asymptomatic bacteriuria is common in kidney transplant
y
 Syngeneic: Donor is an identical twin


patients  Autologous: Recipient can donates to self




y Mycoplasma hominis can cause breakdown of uretero


y
 Allogeneic: Recipient is different individual


vesical anastomosis leading to subsequent graft loss y Following types of cells are used in stem cell transplantation:
y

y Pneumonia is the most common cause of death (25–30%)


y
 Peripheral blood stem cells


y Polyomavirus infection acts as an important cause of


y
 Umbilical cord blood


nephropathy in renal transplant patients–it is called as BK  Bone marrow




nephropathy  Donor lymphocyte infusion




y BK nephropathy has a characteristic ground grass inclusions


y
y Complications that may arise because of HSCT are:
y

with interstitial nephritis; it is an important cause of allograft  Mucosal inflammation




dysfunction.  Hemorrhagic cystitis




 Engraftment syndrome


Liver Transplant Recipients  Veno-occlusive disease




 Diffuse alveolar hemorrhage


y Liver transplantation has the higher rates of infection that 

502
y

 Graft versus host disease


kidney and cardiac transplantation. 

 Posterior reversible encephalopathy syndrome



y Antimicrobial prophylaxis are given for transplant recipients INFECTIONS IN ASPLENIC PATIENTS

Chapter 80     Infections in Special Hosts


y

to avoid post-transplant infections:


y Asplenia i.e. absence of spleen can be congenital or acquired.
 Acyclovir is given in loose for HSV prophylaxis and in high
y

y Infections can occur in patients with congenital asplenia and




doses for CMV.


y

acquired but it is more common in acquired condition.


 Levofloxacin is used for prevention of bacterial infections.
y Acquired asplenia is due to:


 Fluconazole is used for prevention of candidiasis.


y

 Splenectomy


 TMP-SMX–drug of choice for Pneumocystis, Toxoplasma.




 Sickle cell anemia




 Penicillin is given against Streptococcus pneumoniae




 Acquired hyposplenia due to HIV or autoimmune disor-




 Voriconazole or Echinocandins is given as anti fungal




ders


prophylaxis
y Risk factors for infections are:
y

 Extremes of age (younger and older)


Table 5: Infections common after HSCT according to


 Duration less than 1 year of splenectomy


time period of risk




 Indication of splenectomy is immune related




 Lack of vaccination
Time period of risk Infections that are common after HSCT 

 Lack of prophylactic antibiotics




1 to 4 weeks •• HSV
• Gram positive bacteremia

• Gram negative bacteremia


Remember
• Clostridium difficile diarrhea Infections that are encountered in Asplenic patients are:
• Streptococcus pneumoniae

• Candia •

• Haemophilus influenzae type b


• Aspergillus and other molds •

• Neisseria meningitidis

• Respiratory virus •

• Capnocytophaga

• Idiopathic pneumonia syndrome •

• Ehrlichia

4 to 26 weeks • Aspergillus and other molds


• • Klebsiella pneumoniae

• Toxoplasma gondii
• • Bartonella spp

4 to 52 weeks •• CMV
•• VZV Treatment
• BK virus
y Self-administration of an antimicrobial agent at the first sign

• Adenovirus
y

of a suspicious illness in the asplenic or hyposplenic person is


Any time can occur •• HSV commonly advised


but rare •• CMV y Empirical antimicrobial treatment:
y

•• VZV  Gram stain showing Streptococcus pneumoniae or no




•• Streptococcus pneumoniae evidence seen – Vancomycin + Ceftraixone (or) moxifloxa-


•• H.influenzae cine is given
•• Neisseria meningitidis  Gram stain showing diplococci or GNB – Cefepime + Gen-


•• EBV tamycin +/– Vancomycin


•• Nocardia y Pneumococcal and Meningococcal vaccines are must given
y

•• Legionella in asplenctomy patients.


•• Mycobacterium sp
•• Listeria monocytogenes

Vaccinations for Recipients after HSCT


y y Influenza vaccine
y y Hepatitis A
y y Meningococcal conjugate vaccine
y y HPV vaccine

503
81 Antimicrobial
Chemotherapy–A Short Review
INTRODUCTION y The antimicrobial drug moiety (ligand) binds selectively

y
to a microbial protein (target receptive entity for ligand)
y Chemotherapy–refers to “treatment of systemic/topical
→ disrupts a pivotal step required for the physiological/
y
infection with agents that are selectively toxic to the invading
biochemical sustenance of micro-organism → selective
pathogen, without considerable harm to the host cells.”
toxicity.
y Ehrlich’s concept of ‘Pharmacophore’ explains the basis for
y
selective toxicity of antimicrobial drugs.

Figure 1: Analogy between antimicrobial chemotherapeutics and other receptor operant pharmacological moieties. Penicillin is taken as
an example to draw this analogy

y It is interesting to note that this Pharmacophore concept runs synthesis → cell wall deficient bacterial progeny most liable
y
in parallel analogy to Clark’s theory of drug action of Receptor to lysis is formed (Effect) – Bactericidal action of b-lactam
operant chemical entities, Drug (D) + Receptor expressed antibiotics explained.
in target site (R) → [Drug-Receptor Complex] → Effect y Action of every class of AMA could be most easily understood
y
Antimicrobial agent + Receptive site expressed in microbe → based on this Ehrlich’s conceptualization.
Antimicrobial action y Target for Antimicrobial agents (AMA) include 4 classes of
y
y To exemplify the Pharmacophore concept of AMA action, the medically significant pathogens–Bacteria, Viruses, Fungi and
y
above figure illustrates the mechanism of action of Penicillin Parasites.
group of antibiotics. Penicillin (‘Ligand’/‘Drug’) specifically y In a broad purview, Antibiotics can be classified based on:
y
binds to the Penicillin Binding Protein (‘Receptor target for  The Class and Spectrum of antimicrobial activity

Ligand’) expressed in cell wall of susceptible Gram Positive  The Biochemical/Physiological pathway they interfere

Bacteria. Binding of Penicillin to Penicillin Binding Protein with
disrupts transpeptidase based final step of bacterial cell  Their Chemical/Structural/Pharmacophore characteris-

tics
Highlight on working classification of the various AMAs– Table 3:  List of actions of AMAs

Chapter 81     Antimicrobial Chemotherapy–A Short Review


without drowning in the depth (for further information on
Type Characteristics Examples
classification of AMAs–please refer to any standard Pharmacology
textbook). Time • AMA effect ∝ time • Betalactam
• •

y Antibacterials: (For further information on classification of


y dependent of drug exposure antibiotics: (Penicillin,
AMAs–please refer to any standard Pharmacology Textbook). killing above the MIC Cephalosporins,
y Antifungals: (For further information on classification of • Post-antibiotic
• Carbapenems and
Effect–short Monobactams)
y

AMAs–please refer to any standard Pharmacology Textbook).


y Antivirals: (For further information on classification of • Requires multiple/ • Clindamycin
• •

continuous dosing • Erythromycin


y

AMAs–please refer to any standard Pharmacology Textbook).


• Linezolid
y Antiretrovirals: (For further information on classification of

• Vancomycin

AMAs–please refer to any standard Pharmacology Textbook). • 5-Fluorocytosine


y Antihelminthics:
Clinical Pearls: Sub-therapeutic dose of Ritonavir (100–200
y

Table 1:  List of antihelminthics mg once or twice daily) is often combined with other HIV
Protease Inhibitors. This boosting spares the concurrently used
Drugs against Drugs against Drugs against nematodes Protease Inhibitors from metabolism by the CYP3A4 and CYP2D6
cestodes trematodes microsomal enzymes-thereby maintaining their time above EC95.
• • Niclosamide •• Praziquantel •• Albendazole Concentration • AMA effect
• • Aminoglycosides

• • Praziquantel •• Bithionol •• Mebendazole dependent ∝ plasma • Rifampicin


• • Albendazole •• Metrifonate •• Ivermectin killing concentration


• Mebendazole • Oxamniquine • Diethylcarbamazine achieved above
the MIC
• • •

• Pyrantel pamoate
• Post-antibiotic

y Antiprotozoals:
y
Effect–long
• Amenable to

Table 2:  List of antiprotozoals single/intermittent


dosing
Antiamoebic drugs Antimalarial drugs
Clinical Pearls: Single dosing schedule of Aminoglycosides and
• • Metronidazole •• Chloroquine Rifampicin precludes development of toxicity.
• Diloxanide • Artemisinins
Concentration- • AMA effect • Fluoroquinolones
• •

• Emetine • Mefloquine • •

time ∝ Bioavailability • Daptomycin


• •

• Iodoquinol • Primaquine •

dependent or the Area Under • Azithromycin


• •

• Quinine •

killing Curve (AUC) • Metronidazole


• Antifolates •

achieved by the

drug over the


period of exposure
KEY CONCEPTS TO UNDERSTAND • Post-antibiotic

ANTIMICROBIAL DRUG ACTION Effect–prolonged


• Dosing schedule
Before we dive deeper into the topic, it is imperative that we

has no influence
understand some fundamental concepts governing AMA action. on AMA effect
y Minimum Inhibitory Concentration (MIC): Measure of
y

AMA’s Potency. The minimal concentration of AMA-that Clinical Pearls: Steady‐state pharmacokinetics of fixed dose
inhibits visible growth of an isolate after overnight incubation. combination of Emtricitabine and Tenofovir disoproxil fumarate
Lower the MIC more is the potency of the AMA. in HIV management is based on CTDK.
y IC-50 (Half Maximal Inhibitory Concentration): Measure
y

of Intrinsic Activity of AMA against a particular micro- In standard testing conditions, the microbe in exposed to


organism. It is the concentration at which 50 out of the 100 fixed drug concentrations in the incubation media. But
isolates are inhibited. invivo, the antimicrobial concentration is quite dynamic.
y IC-90: Concentration inhibiting 90 percentile of susceptible
y
Hence it is crucial to understand the Concentration-Time
isolates of a microbial species. It is a calculated value based on Curve.
large, inter-center studies. It reflects the various resistance y Concentration time curve: Pharmacokinetic/ Pharmacody-
y

mechanisms operant in the micro-organism affecting its namic correlation of AMA action–a tracing plotted with time
AMA susceptibility. of drug exposure along X-axis and plasma concentration of
 Optimal Dose of an AMA is designed based on the invitro

AMA along the Y-axis.
MIC (Potency) and IC-90 (susceptibility) values.

505
Unit 8     Infectious Diseases

Figure 2: Concentration time curve


Post-antibiotic Effect: Length of time post brief exposure to an AMA that suppresses microbial growth/function.
y Persistence of microbial suppression after a brief contact/exposure to an AMA–is of primal importance with certain Antimicrobials
y

and is utilized clinically to maximize efficacy and suppress toxicity.


y Resistance mechanism to antimicrobial drug action
y

 For a given clinical infection, Microbiological Susceptibility Testing plays a crucial role in narrowing down to a tailored fit of


rationale antimicrobials.
 Each isolate of the same microbial parent species is different from one another–as a consequence of its evolutionary process of


propagation and survival.


 These quasi species change the susceptibility patterns of the microbe–resulting in resistance to the established first line


antimicrobials for the parent species.

Clinically Significant Resistance Mechanisms against some Important AMAs

506

Figure 3: Antibiotic targets and mechanisms of resistance


METHODS OF ANTIBIOTIC RESISTANCE Example: Pre-exposure Chemoprophylaxis against Malaria in

Chapter 81     Antimicrobial Chemotherapy–A Short Review


travelers to endemic area, Postexposure anti-HIV prophylaxis


following needle stick injury.
yy Pre-emptive therapy: Presymptomatic abortive strategy
to suppress the development of an impending infection,
especially of value in high risk patients with positive
laboratory finding suggestive of infection.
Example: Use of Anti-CMV Ganciclovir after liver
transplantation.
yy Empirical therapy: After established suggestive symptoma-
tology of a microbial infection, a broad spectrum nonspecif-
ic/nondefinitive AMA is used to suppress an infection.
Example: Use of broad spectrum antibacterials and
antifungal agent in a neutropenic patient developing fever.
yy Definitive therapy: Narrow targeted AMA treatment after
systematic confirmation of the etiological microorganism
causing it and tailoring the treatment regimen based on
susceptibility patterns.
Example: Using Penicillin in Penicillin sensitive Gram-
postive bacterial infection.
yy Suppressive therapy: Use of low dose AMA after completion
of definitive therapy to prevent recrudescence or relapse from
the reservoir sites that may have eluded from the definitive
therapy.
Example: Acyclovir suppressive oral therapy in recurrent
Herpes infection.

RATIONALE COMBINATION OF AMAs


y Following microbiological confirmation of the etiological
y

pathogen in a clinical condition, an appropriate sensitive


single AMA is used in management of the condition.
y But in some circumstances, a combination antimicrobial
y

chemotherapy is considered rationale over the use of


monotherapy.

Rational Combinations
Figure 4: Methods of antibiotic resistance y Preventing the development of resistance to first line
y

agent: HAART, Anti-TB regimens.


y To augment the speed of microbial clearance/extend the
ANTIMICROBIAL CHEMOTHERAPY–TYPES
y

spectrum/improve efficacy as a virtue of combination


AND GOALS of 2 AMAs: Sequential block–seen in Cotrimoxazole due
Based on the utility of an AMA along the disease progression to synergistic combination of sulfamethoxazole and
timeline, an antimicrobial can fall into the following categories: trimethoprim.
y To overcome biochemical mechanisms of antimicrobial
y

resistance:
 Facilitate drug entry into fungi: Rationale for combining


Amphotericin B with Flucytosine


 Tackle AMA inactivating enzymes: Rationale for


combining Beta-lactamase inhibitors to Penicillin group


of drugs, Cilastatin to Imipenem.
Figure 5: Types of antimicrobial chemotherapy y To reduce the toxicity: Combination therapy limits toxicity
y

y Prophylactic therapy: Specific protection form of primary


y
and pill load of a narrow therapeutic index drug by optimally
level of prevention, wherein a single, non-toxic and highly minimizing their effective dose required.
efficacious AMA is employed with an intention to prevent Ritonavir boosting in Anti-HIV treatment substantially
infection or to eradicate an early infection by a specific reduces the dose of the concomitantly used Protease
organism. inhibitors (like Indinavir, Saquinavir, etc.)
507
82
Multiple Choice Questions
1. Hutchinson’s triad involves all except: 9. A 19-year-old female, who is not sexually active, presents
a. Eighth nerve deafness b. Hutchinson’s teeth with a foul-smelling grayish white vaginal homogenous
c. Interstitial keratitis d. Sabre tibia discharge since 3 months. Which of the following is false
2. Tzanck smear showing multinucleate giant cells in: regarding her condition?
a. Herpes simplex a. pH < 4.5
b. Pemphigus vulgaris b. Fishy odor due to presence of Amines
c. Bullous pemphigoid c. Decreased lactobacilli
d. Dermatitis herpetiformis d. Caused by Gardnerella vaginalis
3. Congenital syphilis, all are true except: 10. Most common causative organism of acute osteomyeli-
a. Scaphoid scapula tis:
b. Conductive deafness a. Staphylococcus aureus b. E. coli
c. Mulberry molars c. Streptococcus d. Pneumococcus
d. Higoumenakis sign 11. The earliest bone change in X-ray to appear in acute
4. Chlamydia trachomatis is implicated in all of the follow- osteomyelitis:
ing, except: a. Periosteal new bone formation (periosteal reaction)
a. Lymphogranuloma venereum b. Sequestrum
b. Reiter’s disease c. Involucrum
c. Granuloma inguinale d. Cloacae
d. Pelvic inflammatory disease 12. Which of the following is true of Salmonella osteomyelitis:
5. Groove sign of Greenblatt is seen in: a. Associated with typhoid fever and sickle cell anemia
a. Tertiary syphilis b. Multiple sites sometimes bilaterally symmetrical
b. Lymphogranuloma venereum c. Radiological feature: diaphyseal sclerosis
c. Granuloma inguinale d. All of the above
d. Chancroid 13. Watery Diarrhea in children is caused by:
6. Lesions of secondary syphilis are all except? a. EHEC b. EPEC
a. Condyloma acuminata c. EIEC d. EAEC
b. Snail track ulcers 14. Regarding ETEC true is:
c. Mucus patches a. Invades submucosa
d. Syphilides b. Most common in developing countries
7. A 24-year-old male, unmarried, with a history of a trip c. Fomite borne and person to person
abroad a few months ago, presents to the skin OPD with d. Not a common cause of traveler’s diarrhea
a history of a painless genital ulcer since the last 4 days, 15. Shigellosis is best diagnosed by:
which bleeds easily, and painless inguinal lymph node a. Stool examination b. Stool culture
swelling. On examination, a beefy red ulcer is seen c. Sigmoidoscopy d. Enzyme
on the preputial skin. Which of the following is true 16. Infective dose of Shigella is:
regarding his diagnosis? a. 105-8 b. 10 – 100
a. On HPE, safety pin appearance within neutrophils c. 1011 d. 10000
b. Both inguinal and femoral lymph nodes form buboes 17. Shigella commonly causes:
c. Incubation period of 2 years a. Constipation b. Food poisoning
d. Subcutaneous granulomas present c. Vomiting d. Bacillary dysentery
8. School of fish appearance on HPE seen in 18. Salmonella gastroenteritis is mainly due to:
a. Chancroid a. Eggs b. Maize
b. LGV c. Fried rice d. Honey
c. Granuloma inguinale 19. Cholera toxin effects are mediated by stimulation of
d. Gonorrhea which of the following second messengers:
a. cAMP
b. cGMP
c. Ca ++ - calmodulin 28. Commonest cause for sporadic encephalitis is:

Chapter 82     Multiple Choice Questions


d. IP3 / DAG a. Japanese B virus b. Herpes simplex virus
20. Incubation period for emetic type of Bacillus cereus: c. HIV d. Rubeola virus
a. 8–16 hours b. 1-5 hours 29. Most common presentation of neurocysticercosis is:
c. 6-12 hours d. 24 hours a. Seizures b. Focal neurological deficits
21. Mesenteric adenitis is caused by: c. Dementia d. Radiculopathy
a. Y.pestis b. Y.pseudotuberculosis 30. Most common organism causing late onset VAP:
c. Y.enterocolitica d. P.multocida (AIIMS pattern May 2018)
22. The most common causative agent for lobar pneumonia
is: a. Klebsiella b. Pseudomonas
a. S. aureus b. S. pyogenes c. E. coli d. Staph aureus
c. Pneumococci d. H. influenzae 31. Common organism involved in community acquired-
23. Which is not true about Legionnaire pneumonia: native valve infective endocarditis: (PGI May 2018)
a. Seen mainly in immunocompromised patients a. Staphylococcus aureus
b. Erythromycin is the drug of choice b. Pseudomonas aerogenosa
c. Common in patients with hairy cell leukemia c. Streptococcus viridans
d. Smoking and alcohol is the risk factor d. Enterococcus
24. Predisposing factors for pneumococcal pneumonia are e. Candida
all except: 32. Which of the following is/are causes fever with rash:
a. CRF b. Lymphoma (PGI may 2018)
c. Old age d. Thalassemia a. Streptococcus pyogens b. Staphylococcus aureus
e. Sickle cell disease c. Vibrio cholerae d. Neisseria meningitidis
25. CSF findings of TB meningitis are all except: e. Salmonella typhi
a. Raised protein b. Low sugar 33. Category A bioterrorism agents is/are: (PGI may 2018)
c. Low chloride d. High RBC count a. Anthrax
26. A 10-year-old girl with fever, convulsions and neck b. Cl. Botulinum toxin
rigidity. Her CSF findings show protein levels 150 c. Vibrio cholerae
mg, Sugar levels 40 mg and chlorides 50 ug/dl with d. Burkholderia pseudomallei
lymphadenopathy; Probable diagnosis: e. Yersinia pestis
a. Viral meningitis 34. Food poisoning caused due to preformed toxin of:
b. Pyogenic meningitis (PGI may 2018)
c. Cryptococcal meningitis a. Bacillus cereus b. Cl. botulinum
d. Tubercular meningitis c. Shigella d. Vibrio cholera
27. Drug of choice for cryptococcal meningitis: e. Staphylococcus aureus
a. Pentostatin b. Amphotericin B
c. Clotrimazole d. Zidovudine

ANSWERS AND EXPLANATIONS

1. Ans.  (d) Sabre tibia 9. Ans.  (a) pH < 4.5

2. Ans.  (a) Herpes simplex 10. Ans.  (a) Staphylococcus aureus

3. Ans.  (b) Conductive deafness 11. Ans.  (a) Periosteal new bone formation (periosteal
reaction)
4. Ans.  (c) Granuloma inguinale
12. Ans.  (a) Associated with typhoid fever and sickle cell
5. Ans.  (b) Lymphogranuloma venereum anemia

6. Ans.  (a) Condyloma acuminata 13. Ans.  (b) EPEC

7. Ans.  (d) Subcutaneous granulomas present 14. Ans.  (b) Most common in developing countries

8. Ans.  (a) Chancroid 15. Ans.  (b) Stool culture


509
• Late onset VAP is mainly caused by multi drug resistant
Unit 8     Infectious Diseases

16. Ans.  (b) 10 – 100 •

organisms
17. Ans.  (d) Bacillary dysentery • Late onset VAP – causes are:

Pseudomonas aeruginosa (M/c)


Acinetobacter
18. Ans.  (a) Eggs
Klebsiella pneumoniae
19. Ans.  (a) cAMP 31. Ans.  (a) Staph aureus

20. Ans.  (b) 1–5 hours Ref: Harrisons T.B of internal medicine – 19th ed – page 816
• Most common cause for native valve endocarditis is

21. Ans.  (c) Y. enterocolitica S.aureus (both community and hospital acquired)

22. Ans.  (c) Pneumococci 32. Ans.  ( a) Streptococcus pyogens; (b) Staphylococcus
aureus; (e) Yersinia pestis
23. Ans.  (b) Erythromycin is the drug of choice Ref: Harrisons T.B of internal medicine – 19th ed – page 780
Fever with rashes - DD are:
24. Ans.  (d) Thalassemia • Bacterial
• infections–Staphylococci, Streptococci,
Pneumococci, Meningococci
25. Ans.  (d) High RBC count • Viral infections–Enteroviruses, Measles, EBV, CMV,

Parvo virus, Influenza virus


26. Ans.  (d) Tubercular meningitis
33. Ans.  ( a) Anthrax; (b) Cl. Botulinum toxin; (e) Yersinia
27. Ans.  (b) Amphotericin B pestis
Ref: Harrisons T.B of internal medicine – 19th ed – page
28. Ans.  (b) Herpes simplex virus 261e-3
Category A of bioterrorism agents:
29. Ans.  (a) Seizures
• Anthrax

• Botulism
30. Ans.  (b) Pseudomonas

• Plague

Ref: Mandells’ infectious diseases – 7th edition – page 3717 • Small pox

• Tularemia
• Early-onset VAP (Occurring during the first 5 days of

• Ebola

the hospital stay) is usually caused by antimicrobial


• Marburg
sensitive organisms.

• Early onset VAP – causes are:


34. Ans.  (a) Bacillus cereus; (e) Staphylococcus aureus


Streptococcus pneumoniae
Moraxella catarrhalis Ref: Harrisons T.B of internal medicine – 19th ed – page 856
Haemophilus influenzae Preformed toxins (Neurotoxins)
Anaerobic bacteria • Staphylococcus aureus

Legionella pneumophila • Bacillus cereus


510
SELF-ASSESSMENT
Self-Assessment Test
(By Author)
1. Phage directed virulence is seen in: 10. ADCC cells mediate:
a. Tetanus a. Innate immunity b. Adaptive immunity
b. Syphilis c. Both d. None of the above
c. Diphtheria 11. Which of the following is a viral superantigen?
d. Pertussis a. Toxic shock syndrome toxin – 1
2. All are true regarding disinfectants except: b. Enterotoxins
a. Glutaraldehyde is effective for bronchoscopes c. Herpes simplex virus associated superantigen
b. Iodophores are more active than iodine solutions d. Rabies nucleocapsid
c. Hydrogen peroxide fogging is a better method of fumi- 12. Number of hotspots seen in L chain are:
gation a. 2 b. 3
d. Phenolic compounds are low level disinfectants c. 4 d. 5
3. Best culture medium for more demanding organisms: 13. Coagglutination reaction by binding protein-A of
a. Peptone water S. aureus is mediated by:
b. Neopeptone a. IgG1 b. IgM
c. Brain heart infusion broth c. IgG3 d. IgA
d. Thioglycollate medium 14. Waldenstrom`s macroglobulinemia is:
4. Following are true about transferable drug resistance a. T cell lymphoma
except: b. B cell lymphoma
a. Drug combinations can prevent resistance c. Alpha chain disease
b. Multiple drugs are resistant at a time d. Gamma chain disease
c. High degree resistance 15. Papain cleaves the immunoglobin molecule into:
d. Virulence is not decreased a. I Fc and 2 Fab fragments
5. Attenuation is: b. 2 FC and 1 FAB fragments
a. Enhancement of virulence c. 1 Variable chain and 1 constant chain
b. Attained by serial passage in susceptible hosts d. I FAB and one hypervariable region
c. Attained by serial passage in unfavourable hosts 16. Paroxysmal nocturnal hemoglobinurea is due to defi-
d. Increases pathogenicity to produce disease ciency of:
6. The following are antibacterial substances in blood and a. Properdin b. Factor I
tissues except: c. DAF d. Factor H
a. Lactoperoxidase b. Alpha lysin 17. HLA B27 is associated with all of the following, except:
c. Interferon d. Plakins a. Multiple sclerosis
7. Substances which can precipitate and are incapable of b. Reactive arthritis
inducing antibodies: c. Reiters syndrome
a. Simple haptens b. Complex haptens d. Ankylosing spondylitis
c. Immunogens d. Antigens 18. Perforins are produced by
8. Heterophile antigens are responsible for following tests a. Plasma cells
except: b. Suppressor T cells
a. Rapid plasma reagin test c. Cytotoxic T cells
b. Weil-felix reaction d. Memory helper T cells
c. Paul bunnell test 19. Autoimmunity is caused due to the following reasons,
d. Cold agglutinin test except:
9. Passive immunity featured by all, except: a. Pressure of forbidden clones
a. Negative phase occurs b. Release of sequestered antigens
b. No lag period c. Expression of cryptic antigens
c. Useful in immunodeficient individuals d. Negative selection of T cells in the thymus
d. Lasts for short time
20. Severe combined immunodeficiency is due to genetic 31. A 25-year-old female in good health developed a corneal
   Self-Assessment Test

defect in: ulcer in her right eye associated with daily disposable
a. Class I MHC deficiency soft contact lens wear. There was no history of trauma
b. RAG mutation to the eye, and she denied overnight wear. Corneal
c. Deficiency of tyrosine kinase scrapings were taken and direct examination revealed
d. Deficiency of PNP the presence of hyaline septate fungal hyphae and
21. Most common agent in subacute endocarditis: cultures grew the fungus shown below:
a. Staphylococcus aureus a. Fusarium solani b. Rhizopus sp.,
b. Staphylococcus epidermidis c. Scedosporium sp., d. Mucor sp.,
c. Streptococci viridans 32. How many moments of hand hygiene have been laid
d. Enterococcus sp., down by WHO?
22. Cytotoxin mediated diarrhea is caused by all of a. 5 b. 6
following, except: c. 7 d. 8
a. Clostridium difficile (toxin A) 33. The following virus causes eye infections that are highly
b. Shigella dysenteriae type 1 contagious. Which of the following is least likely to be
c. Enterohemorrhagic E. coli means of transmission during an outbreak of epidemic
d. Clostridium difficile (toxin B) keratoconjunctivitis?
23. A patient coming from Himachal Pradesh, presents with a. Swimming pools
multiple skin lesions. Microscopy reveals cigar shaped b. Hand towels
yeast cells and asteroid bodies. Microscopy of culture c. Mosquito bites
shows flower like pattern. Identify the agent: d. Contaminated ophthalmic equipment
a. Candida sp., 34. Which of the following microscope helps in cell culture
b. Sporothrix schenckii technique?
c. Epidermophyton floccosum a. Phase contrast microscope
d. Rhizopus b. Electron microscope
24. Reynolds braude phenomenon may be seen in: c. Inverted microscope
a. Candida tropicalis b. Candida dublinensis d. Confocal microscope
c. Candida krusei d. Candida parapsilosis 35. A 5-year-old child presented with fever, rashes all over
25. Tuberculate spores are characteristically seen in: her body, head ache and altered sensorium. Which is
a. Candida b. Coccidiomycosis the best method for diagnosis of this encephalitis?
c. Histoplasma d. Cryptococcus a. PCR b. Viral culture
26. The most common type of systemic mycosis in India is: c. ELISA d. Tzanck smear
a. Blastomycosis b. Histoplasmosis 36. Which of the following is referred to as Dane particle?
c. Coccidiomycosis d. Cryptococcosis a. 22 nm spherical particle
27. Primary site of infection is cryptococcosis is b. 42 nm larger spherical particle
a. Adrenal b. Bone c. 27 nm inner nucleocapsid particle
c. CNS d. Lung d. 24 nm elongated particle
28. Yellow black granules are seen in which fungal infection: 37. Which of the following gene product corresponds to
a. Mucormycosis p32?
b. Mycetoma a. Reverse transcriptase b. Integrase
c. Aspergillosis c. Matrix protein d. Envelope glycoprotein
d. Rhinosporidiosis 38. A 19-year-old female college student has fever, sore
29. Most common fungus causing orbital cellulitis in throat and lymphadenopathy accompanied by
diabetic ketoacidosis: lymphocytosis with atypical cells and an increase in
a. Mucor sheep cell agglutinins. Diagnosis is most likely?
b. Aspergillus a. Infectious hepatitis
c. Candida b. Infectious mononucleosis
d. Cryptococcus c. Chicken pox
30. A 33 year old HIV positive man from Thailand presented d. Herpes simplex infection
with a four month history of a generalized papular rash, 39. An eight-year-old child recently had erythema
associated with diarrhea and weight loss. Biopsy showed infectiosum. Her 33 year old mother subsequently
numerous small yeast cells and culture showing red developed arthralgia followed by painful arthritis with
pigmented colonies was isolated. Identify the etiological swelling in the small joints of both hands. In addition to
agent: the apparent tropism for joints, the virus causing above
a. Cryptococcus neoformans illness is highly tropic for which cell type?
b. Penicillium marneffei a. CD4 T lymphocytes
c. Histoplasma capsulatum b. Renal tubule cells
d. Candida albicans c. Erythroid cells
514 d. Glial cells
40. Which of the following serologic patterns is suggestive 49. Endemic relapsing fever is caused by:

   Self-Assessment Test
of a patient with chronic hepatitis B with a pre core a. Borrelia recurrentis transmitted by louse
mutation? b. Borrelia duttoni transmitted by louse
a. HbsAg positive, HbsAb negative, anti HBc positive, c. Borrelia recurrentis transmitted by tick
HbeAg positive, HBV DNA positive d. Borrelia duttoni transmitted by tick
b. HbsAg negative, HbsAb negative, anti HBc positive, 50. Gram negative, non-motile agent causing rat bite fever
HbeAg positive, HBV DNA positive is:
c. HbsAg positive, HbsAg negative, anti HBc positive, a. Spirillum minus
HbeAg negative, HBV DNA positive b. Streptobacillus moniliformis
d. HbsAg negative, HbsAg positive, anti HBc positive, c. Francisella tularensis
HbeAg positive, HBV DNA positive d. Pasteurella multocida
41. The most commonly used method for isolation of 51. Pertussis toxin acts by all of the following mechanisms
Chlamydia: except?
a. Culture on artificial media a. ADP ribosylation of proteins associated with receptors
b. Culture on Vero cell line b. Increase cyclic AMP
c. Inoculation into Guinea pig c. Increased calcium release from sarcoplasmic reticulum
d. Culture on McCoy cell line d. Acts through G alpha subunit
42. Carrion`s disease is caused by: 52. A child is admitted with meningitis and diagnosed as
a. Bartonella henselae causative agent Haemophilus influenzae. Cefotaxime
b. Bartonella quintana was started instead of ampicillin. Which of these is likely
c. Bartonella bacilliformis reason for this?
d. Coxiella burnetti a. H. influenzae strains are known to produce
43. Drug of choice for MRSA strains: betalactamase
a. Dicloxacillin b. Vancomycin b. H. influenzae strains are known to have altered
c. Nafcillin d. Cefazolin penicillin binding proteins
44. Which of the following infectious agent causes neonatal c. Cefotaxime is easier to administer than ampicillin
meningitis as acquired through infected birth canal? d. Drug of choice is sulphamethoxazole and trimethoprim
a. Streptococcus pyogenes but cannot be given
b. Neisseria gonorrhea 53. Endotoxin of one of the following Gram-negative
c. Escherichia coli bacteria does not play any part in the pathogenesis of
d. Streptococcus agalactiae natural disease.
45. All of the following meningococcal serogroups cause a. E. coli
invasive disease, except: b. Klebsiella
a. W135 c. Vibrio cholerae
b. A d. Pseudomonas
c. D 54. 35-year-old cook in a hotel suffered from enteric fever
d. Y two years back. To diagnose the chronic carrier state in
46. Vincent`s angina is caused by: this patient, which of the following method is used?
a. Porphyromonas gingivalis a. Vi agglutination test
b. Porphyromonas endoodontalis b. Blood culture in BHI broth
c. Leptotrichia buccalis c. Widal test
d. Treponema denticola d. Bone marrow culture
47. Confirmation of Anthrax can be done during bioterror- 55. Xpert/MTB/RIF test is used for
ism attacks by following methods except (according to a. For assessing resistance to isoniazid
CDC, 2001): b. For assessing multi drug resistant TB
a. Gelatin stab agar culture c. For assessing rifampicin resistance
b. Lysis by gamma phage d. monitoring drug response in MDR TB
c. Direct immunoflouresence test 56. Most common cause of community acquired UTI
d. PCR a. E. coli
48. A patient with localized ulcerations in throat, on throat b. Proteus
swab culture showed black coloured colonies with c. Pseudomonas
brown halo in tinsdale medium with positive urease test. d. Klebsiella
Gram-staining shows Gram positive bacilli in palisade 57. Early onset neonatal disease caused by Listeria
arrangement. Identify the agent? monocytogenes is characterized by all, except:
a. Corynebacterium diphtheriae mitis a. Mortality rate is less than 10%
b. Corynebacterium diphtheriae gravis b. Acquired from maternal genital flora
c. Corynebacterium pseudotuberculosis c. Presents as neonatal sepsis
d. Corynebacterium parvum d. Does not cause nososcomial outbreaks
515
58. A 10-year-old boy following a RTA presents to casualty 66. A 45-years-old male from bihar presented with fever,
   Self-Assessment Test

with contaminated wound over left leg. He has received hepatosplenomegaly and routine blood examination
his complete primary immunization before preschool reveals anaemia, granulocytopenia. Bone marrow
age and received a booster of DT at school entry age. All aspiration reveals the following finding. Identify the
of the following can be done, except: finding.
a. Injection of TT
b. Injection of human antiserum
c. Broad spectrum antibiotics
d. Wound debridement and cleaning
59. Which type of diphtheria has highest mortality?
a. Pharyngeal
b. Nasal
c. Laryngeal
d. Conjunctival
60. Pontiac fever is caused by:
a. Listeria monocytogenes
b. Legionella pneumophila
c. Calymmatobacterium granulomatis
d. Pasteurella multocida
61. A 48-year-old woman develops fever and focal neuro-
logical signs. MRI shows a left temporal lobe lesion.
Most appropriate test that can be used to confirm the a. Amastigote of L.donovani
diagnosis of HSV encephalitis is: b. Promastigote of L.donovani
a. Brain biopsy c. T.cruzi
b. Tzanck smear d. T.bruzi
c. PCR assay for viral DNA in CSF 67. Identify the parasite from this modified ziehl neelson
d. Serum IgM antibody detection stained smear and choose the true statement from the
62. Bivalent HPV vaccine (Cervarix) includes which of the following.
following serotypes:
a. 16 and 18
b. 6 and 11
c. 6 and 16
d. 11 and 18
63. Vector for Kyasanur forest disease is:
a. Aedes mosquitoes
b. Culex mosquitoes
c. Ticks
d. Sandfly
64. Which of the following statements about congenital
rubella syndrome is correct?
a. Following vaccination, seroconversion occurs in 40% of
recipients
b. Congenital abnormalitites occur when a nonimmune
pregnant women is infected at any time during
pregnancy
c. Deafness is a common defect associated with congeni- a. Developmental stages of the parasite are extracellular.
tal rubella syndrome b. Both sexual and asexual multiplication of the parasite
d. Only rare strains of rubella virus are teratogenic occurs in a single host.
65. Phage typing is employed for typing of following c. Each oocyst contains eight naked sporozoites.
bacteria, except: d. It causes meningoencephalitis in AIDS patients.
a. Staphylococcus aureus 68. A 48-year-old immunocompromised patients attended
b. Vibrio cholerae the emergency block with complaints of abdominal
c. Streptococcus pain, vomiting and dyspnea. On examination
d. Brucella and baseline investigation, he was diagnosed as

516
gastrointestinal perforation with paralytic ileus. The 75. Invasive amoebiasis can be best diagnosed by:

   Self-Assessment Test
most common cause of this disseminated infection in a. ELISA
immunocompromised patient is due to the following. b. Countercurrent immunoelectrophoresis
Identify it. c. Indirect hemagglutination test
d. Complement fixation test
76. Which of the following is a risk factor for bladder carci-
noma:
a. Clonorchis sinensis
b. Opisthorchis sp
c. Schistosoma haematobium
d. Fasciola hepatica
77. Malignant hydatid cyst is caused by:
a. E. granulosis
b. E. multilocularis
c. E. vogeli
d. None of the above
78. Which of the following is used for antibiotic sensitivity
testing?
a. Rhabditiform larvae of S. stercoralis a. Thayer martin medium
b. Filariform larvae of S. stercoralis b. Mueller martin medium
c. Adult female worm of S. stercoralis c. Chocolate agar
d. Adult male worm of S. stercoralis d. None of the above
69. Skin transplant was done from sister to brother. After 79. Espundia is caused by:
few years, brother to sister skin transplant was done, but a. Leishmania mexicana
rejection occurred. What is this phenomena called? b. L.braziliensis
a. Schwartzman reaction c. L. donovani
b. Theobald smith phenomena d. L. major
c. Eichwald silmser effect 80. Which one of the following does not pass through the
d. Schultz dale phenomena lungs
70. Role played by MHC 1 and 2 is to: a. Hookworm
a. Stimulate interleukin production b. Ascaris
b. Immunoglobulin class switch over c. Strongyloides
c. Transduce the signal to T cell following antigen binding d. Enterobius vermicularis
d. Presenting the antigen for recognition by T cell antigen 81. Identify the following immunodeficiency disorder,
binding receptors which has an inherited deficiency in CD40 Ligand:
71. All of the following statements about carbohydrate a. X linked agammaglobulinemia
antigens are true, except: b. Hyper IgE syndrome
a. Lower immunogenicity c. Hyper IgM syndrome
b. Memory response is seen d. Job syndrome
c. Cause polyclonal B cell stimulation 82. Regarding Zika virus, which of the following is accurate?
d. Does not require stimulation by T cells a. Route of transmission seen are breast milk and sexual
72. Filarial stage of adult worms responsible for disease in b. Concentration of virus is more in semen than blood
all of the following, except: c. Zika virus first identified in brazil
a. Onchocerca volvulus d. Virus not spread through saliva
b. Brugia malayi 83. Each of the following regarding bacterial spores is
c. Wucheria bancrofti correct, except?
d. Mansonella ozzardi a. Spores are formed under adverse environmental
73. All of the following parasites enter the human body by conditions such as the absence of a carbon source
penetrating the skin except: b. Spores are resistant to boiling
a. Ancylostoma duodenale c. Spores are metabolically inactive
b. Necator americanus d. Spores are formed by Bacillus, Neisseria and Clostridi-
c. Strongyloides stercoralis um
d. Trichinella spiralis 84. Lymphangitis is caused by:
74. Which one of the following is detected by antigen detec- a. Staphylococcus
tion test used for diagnosis of P. falciparum malaria? b. Streptococci
a. Circumsporozoite protein c. Pneumococci
b. Merozoite surface antigen d. Neisseria
c. Histidine rich protein (HRP -1)
d. Histidine rich protein (HRP-2) 517
85. Which of the following is enrichment medium of V 93. Identify the below microscopic picture:
   Self-Assessment Test

cholera: a. Nocardia sp.,


a. Monsur taurocholate peptone water b. Mycobacterium leprae
b. TCBS medium c. Mycobacterium tuberculosis
c. LJ medium d. Actinomyces sp.,
d. Regan Lowe medium
86. Oakley fulthrope procedure is:
a. Agglutination test
b. Precipitation test
c. Flocculation test
d. None of the above
87. Identify the organism from the egg provided below:

94. Noninvasive diarrhea is caused by:


a. Shigella
b. Salmonella
c. Yersinia enterocolitica
d. B.cereus
95. Which of the following staphylococcal infection is not
toxin mediated?
a. Giardia lamblia a. Toxic shock syndrome
b. Ascaris lumbricoides b. Staphylococcal scalded skin syndrome
c. Tinea solium c. Food poisoning
d. Enterobius vermicularis d. Septicemia
88. Which of the following is correct about prions? 96. Fried egg colonies are produced by:
a. Long incubation period a. Bacillus cereus
b. Destroyed by autoclaving at 121°C b. Haemophilus influenzae
c. Nucleic acid present c. Neisseria subflava
d. Immunogenic d. Mycoplasma pneumoniae
89. Chromosomal mutation can be identified by all, except: 97. Quality of drinking water supply according to
a. Single strand polymorphism bacteriological tests is described as Satisfactory when
b. Agarose gel electrophoresis the coliform count/100 ml is:
c. Denaturating gradient gel electrophoresis a. 0 b. 1-3
d. Dideoxynucleotide trail sequencing c. 4-9 d. ≥10
90. Factor affecting electron microscope is? 98. All are true regarding superantigens, except:
a. Eye piece a. Activate large number of B cells
b. Wavelength of the light b. Bind outside the antibody groove
c. Type of glass lens used c. Medium sized proteins
d. Thickness of the sample d. Cause release of cytokines
91. Staph aureus causes vomiting in 6-8 hours. Mechanism 99. A 40 years old immunocompromised man presented
of action is: with a mononucleosis-like syndrome and blurred
a. Stimulation of cAMP vision. The virus is excreted in the urine. Identify the
b. Vagal stimulation virus.
c. Stimulation of cGMP a. Cytomegalovirus b. Epstein barr virus
d. Acts through ganglioside GM receptor c. Human herpes virus 6 d. Varicella zoster virus
92. Infective dose of S. Typhi, causative agent of typhoid 100. True about secondary immune response is:
fever is a. Long latent period
a. 10 b. 103 to 106 b. Usually of low titre
c. 10 to 10
8 10
d. 102 c. Antibodies appear after a prolonged lag period
d. Persists for long time

518
   Self-Assessment Test
ANSWERS AND EXPLANATIONS
1. Ans.   (c) Diphtheria 5. Ans.   (b) attained by serial passage in susceptible hosts
• Classical example of phage directed virulence –

• Virulence of a strain undergoes spontaneous or
Diphtheria induced variation
• Gene for toxin production – beta or tox + corynephages • Induced variation → Exaltation and Attenuation
• Enhancement of virulence – Exaltation ; demonstrated

2. Ans.   (d) Phenolic compounds are low level disinfectants experimentally by serial passage in susceptible hosts
• Reduction of virulence – Attenuation; demonstrated
• Disinfectants are classified into high, intermediate and

by passage through unfavourable hosts, repeated


low level cultures in artificial media, growth in high temperature,
High level Kills all pathogens Eg: Glutaraldehyde in presence of weak growth in high temperature, in
presence of weak antiseptics, desiccation or prolonged
disinfectant except large no. and Hydrogen
storage in culture.
of spores peroxide
Intermediate Kills all pathogens Eg: alcohol, 6. Ans.   (b) alpha lysin
level disinfectant including phenolic The following are the substances which possess antibacte-
mycobacteria and compounds, rial properties
non enveloped iodophores • Beta lysin → thermostable substance active against

viruses except anthrax and related bacilli


spores • Basic polypeptides → leukins extracted from leucocytes

Low level Kills vegetative Eg: Quarternary and plakins from platelets
disinfectant bacteria, ammonium • Acidic substances → lactic acid

fungi and lipid compounds • Lactoperoxidase in milk


enveloped viruses • Interferon


7. Ans.   (b) Complex haptens


3. Ans.   (c) Brain heart infusion broth
Haptens – substances that are incapable of inducing anti-
• Universal ingredient of common media is peptone
body formation by themselves but can react with antibod-

• Neopeptone and Proteose peptone are special brands of


ies specifically

peptone
• Simple haptens: non precipitating (also called as univa-
• Brain heart infusion broth – buffered fluid culture

lent haptens)

medium – suitable for cultivation of more demanding


• Complex haptens: can precipitate with specific antibod-
(fastidious) organisms

ies (also called as polyvalent haptens


• Thioglycollate medium – semisolid, nutrient medium –

supports growth of organisms based on 8. Ans.   (a) Rapid plasma reagin test
Oxygen requirements
• Closely related antigens occur in different species →

4. Ans.   (a) Drug combinations can prevent resistance Heterogenetic or heterophile antigens
• Eg: Forssman antigen
• Drug resistance is of two types: Mutational and

• Tests use this heterophile antigen – heterophile reaction


Transferable

namely:
Mutational drug Transferable drug • Weil felix reaction – typhus fever

resistance resistance • Paul bunnel test – Infectious mononucleosis


One drug resistance at a Multiple drug resistance • Cold agglutinin test – Primary atypical pneumonia

time
9. Ans.   (a) Negative phase occurs
Low degree resistance High degree resistance
Can be overcome by high High doses ineffective Active immunity Passive immunity
drug doses Produced actively by host Immunoglobulins received
immune system passively
Drug combinations can Combinations cannot
prevent prevent Induced by infection and Acquired by mother to fetus
vaccination transfer and readymade
Resistance does not spread Spreads to same or
antibody transfer
different species
Long lasting Lasts for short time
Mutants may be defective Not defective
Lag period present No lag period
Virulence may be low Virulence not decreased
Memory present No memory
519
(Contd...)
ƒ Deficiency of Properdin and Factor D → Neisseria
   Self-Assessment Test

Active immunity Passive immunity ƒ

and pyogenic infections


Booster doses useful Subsequent doses – less ƒ Deficiency of membrane attack complex (C5 to C9) →
ƒ

effective Disseminated Neisseria infection


Negative phase may occur No negative phase ƒ C1 esterase inhibitor deficiency → Hereditary angio-
ƒ

neurotic oedema
Not useful in useful
ƒ DAF (Decay accelerating factor) deficiency → PNH
immunodeficienty
ƒ

ƒ Factor H and I deficiency → immune complex disease


ƒ

10. Ans.   (c) both and recurrent pyogenic infections


• ADCC – Antibody dependent cell mediated cytotoxicity

17. Ans.   (a) Multiple sclerosis


• Type of cell mediate immune response

• Involves both innate and adaptive components


• HLA Allele Associated disease
HLA B27 Ankylosing spondylitis, reactive arthritis,
11. Ans.   (d) Rabies nucleocapsid
reiters syndrome
• Bacterial superantigens
DR 2 Multiple sclerosis, Goodpasture’s

ƒ Staphylococcal toxins like TSST-1, exfoliative toxin


syndrome
ƒ

and enterotoxins;
ƒ Streptococcal pyrogenic exotoxin A and C
ƒ
DR 3 Myasthenia gravis, SLE
ƒ Mycoplasma arthritidis mitogen – I
ƒ

DR3/DR4 Insulin dependent diabetes mellitus


ƒ Yersinia enterocolititca and Yersinia pseudotubercu-
DR 4 Rheumatoid arthritis
ƒ

losis
• Viral superantigens

A3/B14 Hereditary hemochromatosis
ƒ EBV associated superantigens
ƒ

ƒ Cytomegalovirus associated superantigens


ƒ
18. Ans.   (c) Cytotoxic T cells
ƒ Rabies nucleocapsid
ƒ • CD8 cells – Cytotoxic T lymphocytes

ƒ HIV encoded superantigen (nef )


ƒ • Principal effector cells of CMI

• Fungal superantigen
• • Functions of CTL (Target cell lysis) → Produces perforins

ƒ Malassezia furfur
ƒ and Granzymes

12. Ans.   (b) 3 19. Ans.   (d) Negative selection of T cells in the thymus
• Antigen binding site of the antibody is called as variable
• Mechanisms of autoimmunity are as follows:
regions • Breakdown of T cell anergy

• Within the variable region → some zones show rela-


• • Failure of activation induced cell death

tively higher variability in the amino acid sequence → • Loss of Treg cells

hotspots or hypervariable regions • Providing T cell help to stimulate self reacting B cells

• H region – four hotspots


• • Release of sequestered antigens

• L region – three hotspots


• • Molecular mimicry

• Polyclonal lymphocyte activation


13. Ans.   (a) IgG1 • Exposure of cryptic self epitopes


• Coagglutination reaction by binding protein A of S. au-



• Epitope spreading

reus – IgG subclasses except IgG3 • Bystander activation


14. Ans.   (b) B cell lymphoma 20. Ans.   (b) RAG mutation
• It is a B cell lymphoma producing excess igM

• Types of genetic defect in SCID:

• Seen in multiple lymphoma



ƒ Mutation in cytokine receptor
ƒ

• Somatic mutations in MYD88 is seen in 90% of patients



ƒ Adenosine deaminase deficiency
ƒ

ƒ RAG (Recombinase activating genes) mutation


ƒ

15. Ans.   (a) I Fc and 2 Fab fragments ƒ Jak3 mutation


ƒ

• Antibody molecule when treated with proteleolytic



ƒ Class II MHC deficiency (Bare lymphocyte syndrome)
ƒ

enzyme like papain cleaves the molecule into two Fab


fragments and 1 Fc fragment 21. Ans.   (c) Streptococci viridans
• Hinge region broken

Native valve endocarditis Staph. aureus


• This helps in placental transfer, taking part in comple-
Early prosthetic valve endocarditis Staph epidermidis

ment fixation
Late prosthetic valve endocarditis Enterococcus
16. Ans.   (c) DAF
Right sided endocarditis in iv drug Staph aureus
• Complement deficiencies: abusers
520

ƒ Deficiency of C1-C4 → SLE, glomerulonephritis and


(Contd...)
ƒ

pyogenic infections
26. Ans.   (d) Cryptococcosis

   Self-Assessment Test
Left sided endocarditis in iv drug Enterococcus
abusers Systemic infections are caused by this:
• Blastomyces dermatitidis -- North America
Overall in iv drug abusers Staph aureus

• Paracoccidiodes brasiliensis -- South America


Subacute endocarditis Viridans • Coccidioides immitis – South western USA


Streptococci • Histoplasma capsulatum -- USA


• Cryptococcus neoformans – Worldwide


22. Ans.   (a) Clostridium difficile (toxin A)


27. Ans.   (d) Lung
Enterotoxins Cytotoxins Neurotoxins
• Cryptococcosis (torulosis) – yeast infection
Cholera toxin Shigella dysenteriae Staph aureus

• Commonest systemic mycosis in India


type 1

• • Route of infection – inhalation


Vibrio Enterohemorrhagic Bacillus • • Pulmonary cryptococcosis occurs first, eventhough
parahemolyticus E. coli cereus most are asymtomatic
E. coli → LT and Clostridium difficle Cl. botulinum • • Dissemination occurs – systemic – cryptococcal menin-
ST of ETEC, EAEC (Toxin B) toxin gitis is the most serious type of infection
and EHEC • • Diagnosis: India Ink preparation- capsules seen
• Culture – cream coloured colonies at 37°C
Clostridium difficle (Toxin A)

• Treatment: Amphotericin B
Aeromonas

Rota virus (NSP4) 28. Ans.   (b) Mycetoma


Campylobacter jejuni
• Swelling in feet and pus discharging granules is

23. Ans.   (b) Sporothrix schenckii characteristic of Mycetoma


• Madurella sp., Actinomycotic mycetoma causes granules
• Sporotrichosis – also called as Rose Gardner’s disease

• Colour vary from yellow to black depending upon the


• Caused by Sporothrix schenckii

species causing it
• Due to minor trauma caused by thorn prick or splinter

• No effective drug; surgical excision is the only treatment


injury

• HP staining of the tissue sections shows cigar shaped


29. Ans.   (a) Mucor


asteroid bodies
• Zygomycosis – life threatening infections caused by
• It’s a dimorphic fungi; 37°C → creamy white colonies;

aseptate fungi

25°C Flower like pattern of conidial arrangement in


• Rhizopus, Mucor, Absidia and Rhizomucor belongs to
slender delicate hyphae

this category
24. Ans.   (b) Candida dublinensis • Predisposing factors: DKA, End stage renal disease,

patients on iron therapy and phagocytic dysfunctions


• Germ tube test is a test for species identification of

• Rhinocerebral mucormycosis manifesting as orbital


Candida sp. ,

cellulitis and proptosis with facial pain and vision loss


• Specific test for C. albicans and also positive for

occurs most commonly in DKA


C. dubliniensis
• Colonies mixed with human or sheep serum and

30. Ans.   (b) Penicillium marneffi


incubated for 2 hours → wet mount shows germ tubes
• Penicillium marneffei (New nomenclature: Talaromyces
• To differentiate → growth at 45°C → C. albicans (grows)

Marneffei) is unique dimorphic fungus which is an


and C. dubliniensis (does not grow)


emerging opportunistic fungal disease endemic in
25. Ans.   (c) Histoplasma southeast Asia.
• The clinical feature includes umbilicated papular,
• Histoplasmosis – intracellular infection of reticuloendo-

molluscum contagiosum-like skin lesions, in face, upper


thelial system caused by Histoplasma capsulatum. trunk, pinnae and arms and non specific febrile illness
• Called as Darling’s disease

in patient at risk.
• Route of infection: inhalation

• The fungus produces brick red coloured pigment in


• Most infections are asymptomatic; some develop

SDA.
disease resembling TB; some go for disseminated
histoplasmosis 31. Ans.   (a) Fusarium solani
• Lab diagnosis: Tissues – Yeast phase occurs within
• Oculomycosis includes keratomycosis, fungal endoph-

phagocytic cells

thalmitis and ocular adenexal infections


• SDA – growth – mycelial phase – Spherical spores with
• Fungal agents causing keratomycosis include Aspergillus

tubercles (finger like projections)


sp. , Fusarium sp. , Penicillium sp. , Candida sp. ,


• Identification: Mycelial growth with pigmentation and

bean shaped macroconidia with septate hyphae. 521


32. Ans.   (a) 5 Enlarged lymph nodes and spleen are characteristic.
   Self-Assessment Test

• WHO’s five moments of hand hygiene are Some develop signs of hepatitis
• Self limited, lasts for 2-4 weeks.

• Before touching a patient •

• Increase in WBC count with predominance of


• • Before a procedure •

• • After a procedure or body fluid exposure risk lymphocytes. Many of these are LARGE, ATYPICAL T
• • After touching a patient LYMPHOCYTES
• After touching patient’s surroundings
39. Ans.   (c) Erythroid cells

33. Ans.   (c) Mosquito bites • Parvo viruses are small viruses with single stranded

• Adenoviruses are non enveloped, icosahedral viruses DNA genomes


• Human B19 viruses targets erythroid progenitor cells

with DNA genome •

• Types 1-7 causes respiratory infections in children and



• B19 is associated with erythema infectiosum, transient

types 3,4 and 7 in military recruits aplastic crisis, pure red cell aplasia and hydrops fetalis
• Types 8,19 and 37 cause epidemic keratoconjuctivitis

(occurs in early pregnancy)
• Enteric adenoviruses 40,41 causes gastroenteritis in
40. Ans.  (c) HbsAg positive, HbsAg negative, anti HBc

young children
positive, HbeAg negative, HBV DNA positive
34. Ans.   (c) Inverted microscope
Table:  Hepatitis B Panel interpretation
• Cell culture techniques uses cell culture vials to see the

cytopathic effect HBsAg anti IgM Anti Anti Interpretation


• Inverted microscopy helps in visualizing the CPE

HBc HBc HBc
– – – – Susceptible
35. Ans.   (a) PCR
– – – + Immune due to
• Typical rash with history suggests Varicella Zoster
vaccination

infection
• For VZV encephalitis – preferred method for the

– – + + Immune due to
diagnosis is VZV PCR natural infection
+ + + – Acute infection
36. Ans.   (b) 42nm larger spherical particle
+ – + – Chronic Infection
• Three types of particles in Hepatitis B virus

• Most numerous are spherical particles – 22nm – made



– – + – Interpretation
up of HbsAg unclear (given below)
• Larger, 42nm spherical virions – lesser in amount – Dane

Four 1. Resolving 2. False positive anti HBc
particles possibilities: infection
• 22nm inner nucleocapsid core that contains HbcAg

3. Low level 4. Resolving acute infection


chronic infection
37. Ans.   (b) Integrase

Gene product Description 41. Ans.   (d) Culture on McCoy cell line
gp160 Precursor of envelope glycoproteins • Cell line culture is the traditional method of diagnosis

• Choice of cell lines are McCoy, HeLa 229, Green monkey


gp120 Outer envelope glycoproteins of virion

and baby hamster kidney (BHK-21) cell lines


p66 Reverse transcriptase and Rnase H from • NAAT – Nucleic acid amplification tests have revolu-

polymerase gene product tionized the diagnosis and gold standard technique
p55 Precursor of core proteins, polyprotein nowdays
from gag gene
42. Ans.   (c) Bartonella bacilliformis
p51 Reverse transcriptase, RT
• B. henselae – cat scratch disease, bacillary angiomatosis,

gp41 Transmembrane envelope bacillary peliosis, Bacteremia and endocarditis


glycoproteins, TM • B. quintana – Trench fever, Chronic bacteremia, endo-

p32 Integrase , IN carditis, bacillary angiomatosis


• B. bacilliformis – Bartonellosis (Carriion’s disease),
p24 Nucleocapsid core protein of virion

Oroya fever, Verruga peruana


p17 Matrix core protein of virion
43. Ans.   (b) Vancomycin
38. Ans.   (b) Infectious mononucleosis
Sensitivity Drug of choice
• EBV causes infectious mononucleosis

• After an incubation period of 30-50 days, symptoms of Sensitive to penicillin Penicillin G


522

head ache, fever, malaise, fatigue and sore throat occur. Sensitive to methicillin Nafcillin or oxacillin
49. Ans.   (d) Borrelia duttoni transmitted by tick

   Self-Assessment Test
Sensitivity Drug of choice
Resistant to methicillin Vancomycin Epidemic relapsing fever Endemic relapsing fever
(MRSA) Alternate drugs: B. recurrentis B. duttoni, B. hermsii, B.
Teicoplanin, Daptomycin, turicatae
Linezolid, Quinupristin/
Transmitted by louse Tick
Dalfopristin, Ceftobiprole
Hemorrhage and CNS Less common
MRSA in skin and soft Clindamycin, Cotrimoxazole,
features more common
tissue infections doxycycline and linezolid
Doxycycline – single dose Doxycycline – 1 week
44. Ans.   (d) Streptococcus agalactiae
• Group B Streptococci – S. agalactiae

50. Ans.   (b) Streptobacillus moniliformis
• Major cause of neonatal sepsis and meningitis – 30%

Streptobacillary rat bite Spirillary rat bite


of women are vaginal or rectal carriers of group B fever fever
streptococcus (GBS)
• CAMP test positive

Streptobacillus moniliformis Spirillum minus
• Penicillin is the drug of choice

Also called as Haverhill fever Called as sodoku

45. Ans.   (c) D Gram negative, highly Gram negative, spirally


pleomorphic, non motile coiled, motile
• Capsular polysaccharide is the principal virulence factor
Cultivable by artificial media Non cultivable

of meningococci
• Based on the antigenic nature of the capsule →
51. Ans.   (c) Increased calcium release from sarcoplasmic

Meningococci is type into 13 sero groups


reticulum
• Most common for invasive diseases are A,B,C,Y and

W135 • ADP ribosylating activity with an A/B structure and


mechanism of action similar to that of cholera toxin


46. Ans.   (c) Leptotrichia buccalis • ADP ribosylation of GTP binding protein → activation of

• Leptotrichia buccalis formerly called as Fusobacterium adenyl cyclase → activation of cAMP


• Pertussis toxin – promotes lymphocytosis, sensitisation

fusiforme •

• Anaerobe , normal oral flora



to histamine, enhances insulin secretion
• Causes acute necrotising gingivostomatitis – Vincents’
52. Ans.   (a) H. influenzae strains are known to produce

angina
betalactamase
• Porphyromonas gingivalis and endodontalis causes

dental infections • DOC for H. influenzae type b invasive infections is


cephalosporins like ceftriaxone, cefotaxime


47. Ans.   (a) Gelatin stab agar culture • Non typable strains of H. influenzae → resistant to beta

Guidelines for diagnosis of Anthrax during bioterrorism lactams mostly → mechanism for this is production of
attacks (CDC,2001) beta lactamase
• Presumptive identification → Any large Gram positive
53. Ans.   (c) Vibrio cholerae

bacillus with morphology and cultural properties


similar to anthrax bacillus • Cholera vibrios possess lipopolysaccharide O antigen

• Initial confirmation – Lysis by gamma phage and direct



(LPS- endotoxin), similar to all other Gram negative
immunoflourescent test intestinal bacilli. Here LPS is has no role in pathogenesis
• Further confirmation – PCR

but it is responsible for the immunity induced by killed
vaccine.
48. Ans.   (c) Corynebacterium pseudotuberculosis
54. Ans.   (a) Vi agglutination test
• Diphtheroids are nondiphtherial corynebacteria

which are Gram positive bacilli arranged in palisade • Chronic carrier state in typhoid is diagnosed by

appearance – cause infections in immunosuppresed measurement of Vi agglutinins


states as they are normal commensals. • Vi agglutinins > 1:10 – significant

• Clinically resembling Diphtheria is caused by C.


55. Ans.   (c) For assesing rifampicin resistance

ulcerans and C. pseudotuberculosis


• Differentiation is by urease test which is positive in

• The Genexpert test is a new molecular test for TB which

above organisms and negative in C. diphtheriae diagnoses TB by detecting the presence of TB bacteria,
as well as testing for resistance to the drug Rifampicin

523
56. Ans.   (a) E. coli 62. Ans.   (a) 16 and 18
   Self-Assessment Test

• E. coli – single most common pathogen – accounts for


• • HPV vaccine – Subunit vaccine

70-75% of UTI • Quadrivalent – included serotype 6,11,16 and 18


• Uropathogenic E. coli – serotypes O1, O2, O4, O6, O7 and


• (Gardasil)
O75 are responsible • Bivalent – serotype 16 and 18 (Cervarix)

• Opportunisitic pathogen; causing struvite stone in


bladder → Proteus 63. Ans.   (c) Ticks


• Aedes → Chikungunya, Dengue, Yellow fever

57. Ans.   (a) Mortality rate is less than 10% • Culex → Japanese B encephalitis, West nile fever

Early onset neonatal Late onset neonatal • Ticks → KFD, Omsk hemorrhagic fever, crimean congo

listeriosis listeriosis hemorrhagic fever, colorodo tick fever

Less than five days of birth More than five days of birth 64. Ans.   (c) Deafness is a common defect associated with
Acquired from maternal Acquired from environment congenital rubella syndrome
genital flora • Most serious consequence of rubella virus infection →

Most common is neonatal Most common is neonatal Congenital Rubella syndrome (CRS)
sepsis meningitis • Risk is maximum if acquired during first trimester of

pregnancy
Mortality rate > 30% Mortality rate <10% • Risk after 5th month is almost negligible

Does not cause nosocomial Nosocomial outbreaks seen • Classical triad: Ear defect (Sensory neural deafness –

outbreaks most common defect) , Ocular defect (Salt and pepper


Miliary microabscesses Not seen retinopathy – M/c), Cardiac defect (PDA)
– Granumolatosis
65. Ans.   (c) Streptococcus
infantiseptica
Phage typing:
58. Ans.   (b) Injection of human antiserum • Staph. aureus

• Depending on the wound category → belongs to • Vi antigen typing of S. Typhi


• Vibrio cholerae

category B •

• Taken complete course of TT/booster within past • Brucella


• C. diphtheriae

>5years to <10years •

• Antibiotic has no role after six hours because the toxin


66. Ans.   (a) Amastigote of L. donovani

has already formed


• Immunoglobulin is for category C and D

• L. donovani is an obligate intracellular parasite of RES

• Category B → treatment is toxoid one dose



predominantly affecting liver, spleen, bone marrow and
lymph nodes.
59. Ans.   (c) Laryngeal • Parasite exists in two morphological forms: Amastigote

• Respiratory type is most common which affect tonsils and & Promastigote
• Amastigote – resides in the cells of RES

pharynx → Faucial diphtheria; forms pseudomembrane •

coat; bull neck appearance; highest mortality • Promastigote – found in the vector (Sandfly) and in

• Cutaneous diphtheria → not toxin mediate; due to



culture media.
organism per se; hence can be caused by non toxigenic
67. Ans.  (d) It causes meningoencephalitis in AIDS
strains also
patients
• Systemic complications also occur

• Spherical or oval around 5μm in diameter with four


60. Ans.   (b) Legionella pneumophila sporozoites.


• Pontiac fever – acute flu like illness – self limiting – never

• They are intracellular and extracytoplasmic, seen in

develops pneumonia vacuoles


• Legionners disease – pneumonia

• Both schizogony and gametogony takes place in same

• Caused by Legionella pneumophila



host like man, cat, dog or cattles

61. Ans.   (c) PCR assay for viral DNA in CSF 68. Ans.   (a) Rhabditiform larvae of S. stercoralis
• Cytopathology (Tzanck preparation) by Wright’s or

• Spherical or oval around 5μm in diameter with four

Giemsa stain detects inclusion bodies of multinucleated sporozoites.


giant cells → in HSV skin lesions • They are intracellular and extracytoplasmic, seen in

• Viral DNA detection by PCR is diagnostic in encephalitis vacuoles


• Both schizogony and gametogony takes place in same

and meningitis and it also differentiates HSV 1 and 2. •

• Antibody detection cannot differentiate recent and past host like man, cat, dog or cattle
524

infections
69. Ans.   (c) Eichwald silmser effect es Egg granuloma → causes DTH reaction → fibrotic

   Self-Assessment Test
• While transplants between members of a highly in bred changes → traumatic lesions, hemorrhage, mechanical
irritation continuously → Malignant tumours

strain of animals are successful, an exception is seen


when the donor is a male and the recipient is female Species Clinical manifestations
• Such grafts are rejected as the grafted male tissue will
Schistosoma haematobium Haematuria and dysuria

have antigens determined by the Y chromosome which


will be absent in the female recipient Schistosoma mansoni Dysentery
• Grafts from female to male will succeed

Schistosoma japonicum Katayama syndrome
• This unilateral sex linked histoincompatibiility →
Schistosoma intercalatum Intestinal disorders

Eichwald silmser effect


Schistosoma mekongi Hepatomegaly
70. Ans.   (d) Presenting the antigen for recognition by T
cell antigen binding receptors 77. Ans.   (b) E. multilocularis
• MHC I molecule → Presents the peptide antigen to Tc

• Echinococcus multilocularis – parasite of foxes, cats and

Cells dogs
• MHC II → presents the peptide antigen to T helper cells

• Accidental host – humans

• Mode of infection – ingestion of food contaminated with


71. Ans.   (b) Memory response is seen faeces of foxes


• Lipopolysaccharide – Eg of Carbohydrate antigen

• Multilocular hydatid cysts in liver – no hydatid mem-

• T independent antigen → Directly stimulates B cell for



brane and fluid – absence of brood capsules – more
antibody production without participation of T cell spread like a malignant tumour – metastasise to brain
• Dose dependent immunogenic with limited antibody and necrosis of liver occurs
• Condition is fatal

response – IgM and IgG3 •

• Lack memory response • E. granulosus – Hydatid disease


• E. multilocularis – Alveolar echinococcosis


• LPS can cause polyclonal B cell activation •

• E. vogeli – Polycystic echinococcosis


72. Ans.   (d) Mansonella ozzardi


78. Ans.   (b) Mueller martin medium
• For lymphatic filariasis (Wucheria and Brugiya) → Adult
• Antimicrobial susceptibility testing is done by many

female worm is pathogenic; wheras the microfilaria is •

responsible for allergic manifestations methods


• In Onchocerca volvulus → Skin nodules or

• Diffusion tests:

onchocercoma (due to adult worms) → due to hyper ƒ Kirby-Bauer disk diffusion method
ƒ

reactive immune response to microfilarial antigens ƒ Stokes disk diffusion method


ƒ

• Dilution methods test:


73. Ans.   (d) Trichinella spiralis ƒ Agar dilution method


ƒ

ƒ Broth dilution method


• Trichinella spiralis → transmitted by consumption of ƒ

• Most used method: Kirby-bauer disk diffusion method


contaminated pig’s meat containing L1 stage larvae. •

• Agar used: Mueller hinton agar or nutrient agar


74. Ans.   (d) Histidine rich protein (HRP-2) • Interpretation: based on zone of inhibition size

• Guidelines used : Clinical Laboratory Standards Institute


• Antigen detection tests – rapid diagnostic – immuno-

(CLSI)

chromatographic tests
• Thayer martin medium: Selective medium for Neisse-
• pLDH and aldolase → common to all plasmodium

ria gonorrhea

species
• Chocolate agar: Enriched media for Neisseria gonor-
• HRP 2 Ag detection : Specific for P. falciparum

rhea, H. influenzae

• False positive in RA factor positive cases


79. Ans.   (b) L. braziliensis


75. Ans.   (a) ELISA
• Oriental sore → L. tropica
• Amoebic antigen in stool is positive in intestinal

• Acute necrotising cutaneous leishmaniasis → L. major


amoebiasis while negative in invasive type


• Visceral leishmaniaisis → L. donovani


• Amoebic antigen and antibodies in serum is positive in

• Espundia (most dangerous form of cutaneous leish-


invasive type (ELISA)


maniasis ) → L. braziliensis
• PCR is the confirmatory test for identification of

Entamoeba histolytica in faeces. 80. Ans.   (d) Enterobius vermicularis


76. Ans.   (c) Schistosoma haematobium • Life cycle of Hookworm,Ascaris and Strongyloides

involves lungs
• Schistosoma sp. , belong to trematodes – blood flukes
• Enterobius vermicularis passes only via GIT

• Eggs of Schistosoma haematobium → causes strong in-


• Pulmonary ascariasis: due to migrating larvae in lung 525


flammatory reaction in the ureter and bladder → caus-


tissue – Loefflers’ syndrome – due to hypersensitivity


reaction of the host against allergens produced by the 84. Ans.   (b) Streptococci
 Self Assessment Test

migrating larvae • S. pyogenes – causes suppurative infections of skin


• Eosinophilia and urticaria are common


• Wound or burns infections → lymphangitis or cellulitis

• In hookworm infection → migrating larvae produces


• Spread of infections can even lead to fatal septicemia


mild symptoms like cough, pharyngitis, dyspnoea • Two typical streptococcal skin infections are Erysipelas


and Impetigo
81. Ans.   (c) Hyper IgM syndrome
• Necrotising fasciitis → due to M types 1 and 3 forming
• An inherited deficiency in CD40 ligand (CD40L or


pyrogenic exotoxin A → Flesh eating bacteria →

CD154) leads to impaired communication between T Vancomycin is the drug of choice

cells and antigen-presenting cells (APCs) • Non suppurative post streptococcal infectious sequelae
• The B-cell response to T-independent antigens,


→ Acute rheumatic fever and Acute glomerulonephritis

however, is unaffected, accounting for the presence
of IgM antibodies in these patients, which range from 85. Ans.   (a) Monsur taurocholate peptone water
normal to high levels and give the disorder its common • Enrichment media for Vibrio cholerae – Alkaline


name, hyper IgM syndrome (HIM) peptone water (pH=8. 6) and Monsur’s taurocholate
tellurite peptone water (pH=9. 2)
82. Ans.   (b) Concentration of virus is more in semen than
• Plating media – Alkaline bile salt agar (BSA), Gelatin
blood


taurocholate trypticase tellurite agar (GTTA),
• Zika virus was first identified in Uganda in 1947 Thiosulphate citrate bile salt sucrose (TCBS)

• Currently, Zika virus disease has been reported so far • Holding or transport media – Venkatraman-


in the following countries; Brazil, Barbados, Bolivia, Ramakrishnan (VR) medium, Cary Blair medium,
Columbia, Dominican Republic, Equador, El Salvador, Autoclaved sea water
French Guyana. Guadeloupe, Guatemala, Guyana, Haiti, • LJ – Lowenstein Jensen’s medium – selective medium

Honduras, Martinique, Mexico, Panama, Paraguay, for M. tuberculosis
Puerto Rico, St Martin, Suriname, Virgin Island and • Regan Lowe medium–Charcoal blood agar–for Bordetella

Venezuela.
• Zika virus disease is an emerging viral disease 86. Ans.   (b) Precipitation test

transmitted through the bite of an infected Aedes • Oakley fulthorpe procedure – Double diffusion in one

mosquito (Aedes agypti). dimension
• There is no vaccine or drug available • Precipitation is a type of antigen-antibody reaction:

• A majority of those infected with Zika virus disease

types of precipitation test are:

either remain asymptomatic (up to 80%) or show mild • Ring test Eg: Ascoli’s thermoprecipitin test, Lancefields

symptoms of fever, rash, conjunctivitis, body ache, grouping
joint pains. Zika virus infection should be suspected in • Slide test Eg: VDRL test

patients reporting with acute onset of fever, maculo- • Tube test Eg: Kahn test for syphilis

papular rash and arthralgia. • Immunodiffusion (Precipitation In gel))
• As of now, the disease has not been reported in India.  

ƒ Single diffusion in one dimension – Oudin procedure

• There can be transmission through sexual and non-
ƒ
ƒ Double diffusion in one dimension – Oakley –

ƒ
sexual ways. The virus can spread through oral sex. It Fulthorpe procedure
can spread through saliva if oral mucus membrane is ƒ Single diffusion in two dimensions – Radial
ƒ
disrupted. immunodiffusion
• The viral load in semen is 100,000 times more than both ƒ Double diffusion in two dimensions – Ouchterlony

ƒ
blood and urine. procedure
• Zika virus has been found in breast milk but no case

of infants getting Zika virus through breastfeeding 87. Ans.   (d) Enterobius vermicularis
reported. • Enterobius vermicularis (Pin worm)

• Infection occurs only in humans; no animal reservoir
83. Ans.   (d) Spores are formed by Bacillus, Neisseria and

• Perianal pruritus – most prominent symptom
Clostridium

• Children are the most commonly affected (usually less
• Spores are highly heat resistant form of bacteria formed

than 12 years)

under nutritional deficient conditions • Larvae and eggs seen in stools – scotch tape technique
• Boiling will not kill spores; Disinfectants also do not kill

• Treatment: Mebendazole


spores; the only effective method is autoclaving
• They are mostly formed by Gram positive rods such as 88. Ans.   (a) Long incubation period

Bacillus and Clostridium • Prions are infectious proteins that cause degeneration of the
• Spores have a thick keratin like layer surrounding it,

CNS. Prion diseases are disorders of protein conformation

which makes them survive for years in soil. • Reproduce by binding to the normal, cellular isofrom
• Spores are metabolically inactive; but they contain DNA,

526 of the prion protein (PrPc) and converts to disease form

ribosomes and other essential components. PrPsc
• PrPc – rich in alpha helix and little beta structure 95. Ans.   (d) Septicemia

 Self Assessment Test



• PrPsc – less alpha helix and high amount of beta
Organism Type of Typical Disease Predisposing Mode of

structure
Pathogenesis Factor Prevention
• They are small infectious protein particles without any
S aureus • Toxigenic Toxic shock Vaginal Reduce time of

detectable nucleic acid


(super- syndrome or nasal tampon use
antigen) Food poisoning tampons Refrigerate
89. Ans.   (b) Agarose gel electrophoresis Improper food
• to visualise amplified DNA after PCR– agarose gel food storage

electrophoresis • Pyogenic • Skin infection • Poor skin • Cleanliness;





(abscess) (e. g. , hygiene; hand-


90. Ans.   (d) Thickness of the sample ƒ Local impetigo, failure washing;

ƒ
ƒ Dissem- surgical- to follow reduce
• Sample thickness is prime important in EM

ƒ
inated wound aseptic nasal

• Very thin specimen cut by freeze etching technique of infections) prcedures carriage
• Sepsis, • IV drug • Reduce IV

size 20 to 100nm thickness is needed




endocarditis use drug use
• Helpful in viral antigen detection (Eg rota virus)

• Types: TEM and SEM 96. Ans.   (d) Mycoplasma pneumoniae

• To visualise: organism is made dead.
• Mycoplasma pneumoniae – causative agent for primary


91. Ans.   (b) Vagal stimulation atypical pneumonia
• Solid medium – fried egg appearance, colonies
• Enterotoxin secreted by S. aureus in food before


examined by method called Dienes method

consumption (preformed food) – acts rapidly – • Specific tests – CFT, ELISA
incubation period is 1-6 hours


• Non specific tests – cold agglutination tests and Strepto-
• Site of action : toxin stimulates the vagus nerve and

coccal MG test

vomiting center of the brain; also stimulates the
intestinal peristaltic activity 97. Ans.   (b) 1-3
• Source: food handler, carrier of S. aureus
Quality of Coliform E. coli

• Food items: Milk products, bakery foods, potato salad or
drinking water count/100ml count/100ml

processed meats.
Excellent 0 0
92. Ans.   (b) 103 to 106 Satisfactory 1-3 0
• Infective dose 103 to 106 which is higher than that of Intermediate 4-9 0

Shigella (10-100) Unsatisfactory ≥10 ≥1
93. Ans.   (a) Nocardia sp. , 98. Ans.   (a) Activate large number of B cells
• The above acid fast stain (blue background with • Superantigens activate large number of T cells

pink bacilli) shows Nocardia asteroides – Gram

• Medium sized proteins highly resistant to proteases
positive branching filaments – causes nocardiasis – in

• Release of cytokines causing massive proliferation of T
immunocompromised individuals

cells
• Weakly acid fast – with 1% H2SO4 – when done with acid

fast staining method 99. Ans.   (a) Cytomegalovirus
• DD: Actinomyces israelli
• Humans are the only known host for CMV.


94. Ans.   (d) B. cereus • Transmission requires close person-to-person contact.

Virus may be shed in urine, saliva, semen, breast milk,
• Important cause of food poisoning and cervical secretions and is carried in circulating

• Foods – milk, cereals, spices, meat and poultry white blood cells.

• Two patterns: Diarrhoea type and Vomiting type • Oral and respiratory spread are probably the dominant

• Diarrhea type - Characterised by non invasive diarrhea

routes of CMV transmission. Other routes are blood

and abdominal pain, 8 to 16 hours after ingestion of transfusion and organ transplantation.
contaminated foods
• Vomiting type – consumption of cooked rice, usually 100. Ans.   (d) Persists for long time

fried rice, incubation period is 6 hours, due to preformed
toxin Primary Response Secondary Response
• The diarrhoeal disease is caused by serotypes – 2, 6, 8, Slow and sluggish Fast and powerful

9, 10, 12 Long lag phase Short or negligible lag phase
• Emetic type caused by 1, 3 and 5 Short lived Prolonged – Long lived

• Diagnosis is by culture of food , feces. Predominant IgM Predominant IgG

• Selective Medium – MYPA medium
More specific Less specific

Qualitative response Quantitative response
527

You might also like